Вы находитесь на странице: 1из 446

G S y

E
B R v
NO ICE

M v - h .A h p
, h h py q .
T h h p h h h v h h
v h p v h p
y h h p h p .
H v , v h p y h h
, h h h h p h y h p y h h
v v h p p p h h
h h v y p p ,
h y p y y h -
h h .R
h h h h .
F p p , v h h p
h h p h h yp -
h h h
h h h v h h
.T p
p h q y .
G S y
E
B R v

Editors
Robert B. Lim, MD, FACS, FASMBS
A P S y
U S v U v y H hS
Ch , M A v Lp p S y
p A yM C
H ,H

Daniel B. Jones, MD, MS, FACS


P S y
H v M Sh
V Ch S y
O h y I v
Ch , M yI v v S S v
D h B P
B hI D M C
B ,M h

N Y Ch S F L L M M C y
N D h S J S S p Sy y
Copyright 2017 by McGraw-Hill Education. All rights reserved. Except as permitted under the United States Copyright
Act of 1976, no part of this publication may be reproduced or distributed in any form or by any means, or stored in a database
or retrieval system, without the prior written permission of the publisher.

ISBN: 978-0-07-183994-5
MHID: 0-07-183994-1.

The material in this eBook also appears in the print version of this title: ISBN: 978-0-07-183993-8,
MHID: 0-07-183993-3.

eBook conversion by codeMantra


Version 1.0

All trademarks are trademarks of their respective owners. Rather than put a trademark symbol after every occurrence of a
trademarked name, we use names in an editorial fashion only, and to the bene t of the trademark owner, with no intention
of infringement of the trademark. Where such designations appear in this book, they have been printed with initial caps.

McGraw-Hill Education eBooks are available at special quantity discounts to use as premiums and sales promotions or for use in
corporate training programs. To contact a representative, please visit the Contact Us page at www.mhprofessional.com.

The views expressed herein are those of the authors and do not necessarily re ect the of cial policy or position of the De-
partment of the Army, the Department of the Air Force, the Department of the Navy, Department of Defense, or the U.S.
Government.

TERMS OF USE

This is a copyrighted work and McGraw-Hill Education and its licensors reserve all rights in and to the work. Use of this
work is subject to these terms. Except as permitted under the Copyright Act of 1976 and the right to store and retrieve one
copy of the work, you may not decompile, disassemble, reverse engineer, reproduce, modify, create derivative works based
upon, transmit, distribute, disseminate, sell, publish or sublicense the work or any part of it without McGraw-Hill Educa-
tions prior consent. You may use the work for your own noncommercial and personal use; any other use of the work is
strictly prohibited. Your right to use the work may be terminated if you fail to comply with these terms.

THE WORK IS PROVIDED AS IS. McGRAW-HILL EDUCATION AND ITS LICENSORS MAKE NO GUARAN-
TEES OR WARRANTIES AS TO THE ACCURACY, ADEQUACY OR COMPLETENESS OF OR RESULTS TO BE
OBTAINED FROM USING THE WORK, INCLUDING ANY INFORMATION THAT CAN BE ACCESSED THROUGH
THE WORK VIA HYPERLINK OR OTHERWISE, AND EXPRESSLY DISCLAIM ANY WARRANTY, EXPRESS OR
IMPLIED, INCLUDING BUT NOT LIMITED TO IMPLIED WARRANTIES OF MERCHANTABILITY OR FITNESS
FOR A PARTICULAR PURPOSE. McGraw-Hill Education and its licensors do not warrant or guarantee that the functions
contained in the work will meet your requirements or that its operation will be uninterrupted or error free. Neither McGraw-
Hill Education nor its licensors shall be liable to you or anyone else for any inaccuracy, error or omission, regardless of
cause, in the work or for any damages resulting therefrom. McGraw-Hill Education has no responsibility for the content of
any information accessed through the work. Under no circumstances shall McGraw-Hill Education and/or its licensors be
liable for any indirect, incidental, special, punitive, consequential or similar damages that result from the use of or inability
to use the work, even if any of them has been advised of the possibility of such damages. This limitation of liability shall
apply to any claim or cause whatsoever whether such claim or cause arises in contract, tort or otherwise.
T is book is dedicated to my amazing wi e, Lisa. She has given
me the strength and grace to pursue a project like this.
RBL

And to my better hal , Stephanie.


DBJ
This page intentionally left blank
Contents

Co n t r ibu t o r s 8. A v Lp pyS I
In t r o d u c t io n h R S y
Jigesh A. Shah and Omar Yuse Kudsi
Pr e a c e
Page 28
Ac k n o wl ed g emen t s v
Ethical and Legal Issues 33
Section 1: General Surgery Section Editor: Robert B. Lim
Anesthesia and OR Concerns 3 9. E h /L I D M
Section Editor: Robert B. Lim Steve Alcazar and Shawn suda
1. S p Ap Page 35
Ashley D. Willoughby 10. E L C
Page 5 Jenny Lam and Shawn suda
2. M Hyp Page 37
Harry . Aubin Skin and So issue 39
Page 8 Section Editor: Richard Smith
3. F U S E y 11. S C
F D p A yB p y Jigesh A. Shah and Omar Yuse Kudsi
Paul Wetstein Page 41
Page 11
12. D U
4. A G E Pamela C. Masella and
Roger Eduardo Mark K. Markarian
Page 13 Page 44
Advances in Laparoscopy 17 13. M
Section Editor: Robert B. Lim Amber E. Ritenour and Lauren Greer
5. FLSA P Page 47
Megan Bowen
14. S Ly ph
Page 19
Joshua S. Ritenour
6. A v Lp py NO ES Page 50
Mark A. Gromski and Kai Matthes
15. h S yL
Page 21
C C h
7. R -A S y Ashra A. Sabe
Brenda Schmidt and Gordon Wisbach Page 55
Page 25

vii
vi i i Co n t e n t s

Abdominal Wall, Abdomen, and 28. M U


Gastrointestinal ract 59 Richard M. Peterson
Section Editors: Robert B. Lim, Ronald A. Page 100
Gagliano, Jr., and Richard Smith
29. B S yByp A
16. A W R P
erri L. Carlson Richard M. Peterson
Page 61 Page 103
17. H U /V hC h 30. S v G yL
Richard M. Peterson Alec C. Beekley
Page 64 Page 106
18. A y p I H 31. B p Dv W hD
Lorenzo Anes-Bustillos S h M O y
Page 67 Matthew J. Martin
19. F H Page 109
Harry . Aubin 32. B S y
Page 71 N C p
20. I N Marcos Molina
Harry . Aubin and Eric Balent Page 115
Page 73 33. R v B S y
21. H P ph H Hussna Wakily
Steven M. Henriques Page 117
Page 77 34. A j G B C p
22. A h Ashley D. Willoughby
Mary . ODonnell and E. Matthew Ritter Page 120
Page 79
35. G M
23. G C Douglas Farmer
Brian J. Pottor and Farah A. Husain Page 123
Page 82
36. B yC
24. G S Alexander Malloy
William Cole Page 127
Page 86
37. B y Ch
25. G Ly ph Harry . Aubin
Cletus A. Arciero Page 130
Page 89
38. Ch y P y
26. G ph R D Alan P. Gehrich
Mary . ODonnell and E. Matthew Ritter Page 133
Page 94
39. L v M
27. P P p U D Kevin M. Lin-Hurtubise and Robert L. She er
Henry Lin Page 137
Page 98
Co n t e n t s ix

40. L v C h 52. C ,R , A R C
Richard Smith Ronald A. Gagliano, Jr.
Page 140 Page 183
41. P P Cy 53. P A
Kiran Lagisetty Alexander Malloy
Page 144 Page 186
42. H P M Breast Disease 189
Richard Smith Section Editor: Richard Smith
Page 146 54. B M
43. Sp /I p h h y p Anita Mamtani
P p Page 191
Ali Linsk and John Paul Sanders
55. D C S
Page 150 Angela Penn
44. S B O Page 195
Patrick Golden
56. I yB C
Page 153 Ranjna Sharma
45. I D -I A P Page 199
Bonnie B. Hunt
57. B R
Page 156 Pamela C. Masella and
46. C Mark K. Markarian
Christopher Yheulon Page 202
Page 160 Endocrine Surgery/Head and Neck
47. M I h umors 207
Danielle E. Ca asso Section Editors: Robert B. Lim and Richard Smith
Page 163 58. A G
48. App Yong Choi
Erik Criman Page 209
Page 167 59. Hyp p hy
49. C C Richard K. Inae
Robert B. Lim Page 212
Page 171 60. hy D
50. C h D Richard K. Inae
Andrew . Schlussel Page 215
Page 176 61. P E
51. D v Jigesh A. Shah and
Erik Roedel Omar Yuse Kudsi
Page 179 Page 217
x Co n t e n t s

62. S v y G 72. O h - P v F
Maxwell Sirkin and William J. Jordan
William V. Rice Page 262
Page 221
73. H I j
Pediatric Surgery 225 Ahmed B. Bayoumi, Fares Nigim, and
Section Editor: Mary J. Edwards Ekkehard M. Kasper
63. H N P Page 265
Margaret E. Clark 74. N yHyp /
Page 227 Hyp
Albert Jesse Schuette
64. Py S
Margaret E. Clark Page 279
Page 232 75. Oph h y
Morohunranti O. Oguntoye and
65. H h p D
Robert A. Mazzoli
Mary J. Edwards
Page 282
Page 236
76. h
66. O ph /G h
Booker . King
Paul Wetstein
Page 288
Page 239
77. S p M -O F
rauma Surgery and Critical Care 243
Robert Shawhan, Matthew Eckert, and
Section Editor: Matthew J. Martin
Matthew J. Martin
67. C C R Page 291
B
Matthew Eckert and Matthew J. Martin 78. M j B S I h
ovy Haber Kamine, Stephen R. Odom, and
Page 245
Booker . King
68. K W Page 297
Laura Mazer
79. S N
Page 249
Julia B. Greer
69. P M p G h Page 301
W
Erik Criman and Matthew J. Martin 80. B P
Stephen R. Odom
Page 252
Page 304
70. C v Sp C
Matthew R. Fusco, Ajith J. T omas, and 81. N z F
Christopher S. Ogilvy Allyson L. Berglund and John M. Giurini
Page 256 Page 308

71. M E v I 82. G y
P Peter L. Steinberg
Matthew R. Fusco, Ajith J. T omas, and Page 312
Christopher S. Ogilvy
Page 259
Co n t e n t s xi

83. F y 93. Ov M
Katherine Carlisle Charles S. Dietrich III and
Page 314 Brad ord P. Whitcomb
Page 353
Section 2: Surgical Subspecialties
94. E p P y
ransplantation Surgery 319 Charles S. Dietrich III and
Section Editor: Ronald A. Gagliano, Jr. Brad ord P. Whitcomb
84. p S yK y Page 358
Joy Sarkar 95. P v I yD
Page 321 Alan P. Gehrich
85. p S yL v Page 362
Joy Sarkar 96. E
Page 324 Alan P. Gehrich
Vascular Surgery 327 Page 366
Section Editor: Dwight C. Kellicut Neurosurgery 369
86. C O Section Editor: Matthew J. Martin
ony Katras 97. B P /S
Page 329 Albert Schuette
87. A A A y Page 371
Dwight C. Kellicut Orthopedic Surgery 375
Page 331 Section Editor: Matthew J. Martin
88. V A 98. C F
Booker . King Justin . Fowler and Justin Robbins
Page 334 Page 377
89. C p H y 99. Sh
Alexander Malloy Kelly G. Kilcoyne
Page 337 Page 380
90. Ly ph D 100. P M
Robert C. McMurray Kelly G. Kilcoyne
Page 340 Page 383
91. V S D 101. K I j
John illou Jeremy McCallum and Douglas Rowles
Page 343 Page 386
92. h O Sy Urology 393
Courtney E. Barrows Section Editor: Joseph R. Sterbis
Page 347
102. N ph h
Gynecology and Obstetrics 351 Peter L. Steinberg
Section Editor: Alan P. Gehrich Page 395
xii Co n t e n t s

103. B C 106. U I j y
Joseph R. Sterbis Raf aella DeRosa and Joseph R. Sterbis
Page 397 Page 405
104. R 107. P C
Joseph R. Sterbis Joseph R. Sterbis
Page 399 Page 408
105. L p
Joseph R. Sterbis Index 411
Page 402
Contributors

Steve Alcazar Alec C. Beekley, MD, FACS


U v y N v Sh M A P S y
R ,N v Dv A C S y
Dv B S y
Lorenzo Anes-Bustillos, MD S yK M C T J
P S yF U v y
B Ch H p Ph ph , P yv
B ,M h
Allyson L. Berglund, DPM
Cletus A. Arciero, MD, FACS P S
A P S y D p P S y
P D ,B S O yF hp H v V M A ,A
Dv S O y H v M Sh
E yU v yS h M B ,M h
A ,G
Megan Bowen, DO
Harry . Aubin, MD A P
G S yR S y, U H hS v U v y
p A yM C Dv G S y
H ,H S A M yM C
S A ,
Eric Balent, MD
R ,G S y Danielle E. Ca asso, DO, MPH
D p G S y F
p A yM C Dv V E v S y
H ,H N Y -P y H p
W C M
Courtney E. Barrows, MD C U v yM C
G S yR N Y ,N Y
B hI D M C
H v M Sh Katherine Carlisle, MD
B ,M h R Phy ,U y
D p U y
Ahmed B. Bayoumi, MD, MSc p A yM C
Dv N y H ,H
D p S y
B hI D M C
H v M Sh
B ,M h

xiii
xiv Co n t r i b u t o r s

erri L. Carlson, DO Roger Eduardo, MD


R PGY5 R
p A yM C D p S y
D p S y B hI D M C
H ,H H v U v yS h M
B ,M h
Yong Choi, MD, FACS, FASMBS
A P S y Mary J. Edwards, MD
U S v U v y h H hS Ch , P S y
Ch , M yI v v B S y p A yM C
A ,G H ,H

Margaret E. Clark, MD Douglas Farmer, MD, MS


G S yR R
p A yM C G S y, p A y M C
H ,H H ,H

William Cole, MD Justin . Fowler, MD


S R O h p S
G S yD p M A yM C
p A yM C O h p S yS v
H ,H ,W h

Erik Criman, MD Matthew R. Fusco, MD


R Phy ,G S y A P ,N S y
D p S y V U v yM C
p A yM C N hv ,
H ,H
Ronald A. Gagliano, Jr., MD, FASCRS, FACS
Ra aella DeRosa, MD A P S y
U yR C h U v yS h M
p A yM C Ch ,D p S y
H ,H D yH h M G pA z
Ph ,A z
Charles S. Dietrich III, MD
P D , OBGYN R y Alan P. Gehrich, MD, COL, MC
Gy O yS v Ch , F P v M R v
p A yM C S y
H ,H A Ch , D p O /Gy
p A yM C
Matthew Eckert, MD H ,H
A S A P
M A yM C U S v U v y h H hS
,W h W h , DC
Co n t r i b u t o r s xv

John M. Giurini, DPM Richard K. Inae, MD, FACS, MC, FSs


A P S y G S y
H v M Sh Ch W A yC yH p
Dv P S y F S ,G
D p S yB h I D M
C William J. Jordan, MD
B ,M h S S
S R O h p Sp M
Patrick Golden, DO T W ,
S S
B h A yC yH p ovy Haber Kamine, MD
F C p ,K y Ch R
D p S y
Julia B. Greer, MD, MPH B hI D M C
A P ,M B ,M h
D p M
Dv G y, H p y Ekkehard M. Kasper, MD
N Dv N y, D p S y
U v y P hS h M P h B hI D M C
P h, P yv H v M Sh
B ,M h
Lauren Greer, MD
S S ony Katras, MD, RV , FACS, COL, MC, USAR
p A yM C S V S yC
H ,H p AMC C P S y USUHS
D
Mark A. Gromski, MD USUHS M S S yC hpS
C F V S
G y A v E py S C H p
Dv G y H p y H ,H
I U v yS h M
I p ,I Dwight C. Kellicut, MD, FACS
A P
U S v U v y h H hS
Steven M. Henriques, MD
D p S y
G M yI v v S y
p A yM C
M D S G p
H ,H
M ,F

Kelly G. Kilcoyne, MD
Bonnie B. Hunt, DO A P S y
S yR U S v U v y h H hS
p A yM C D p O h p S y
H ,H W B A yM C
E P ,
Farah A. Husain, MD, FACS, FASMBS
A P ,S y Booker . King, MD, COL, MC
Dv B S v D US A y B C
O H h S U v y US A y I S R h
P ,O F S H ,
xvi Co n t r i b u t o r s

Omar Yuse Kudsi, MD, MBA, FACS Anita Mamtani, MD


A P S y S R S y
f U v yS h M D p S y
B ,M h B hI D M C
H v M Sh
Jenny Lam, MD B ,M h
G S yR
U v y C Mark K. Markarian, MD, MSPH
S D ,C Dv P S y
B ,M h
Kiran Lagisetty, MD
R Matthew J. Martin, MD, FACS, FASMBS
B hI D M C A P S y
B ,M h U S v U v y h H hS
D p S y
Henry Lin, MD, FACS M A yM C
A P J B L -M Ch
U S v U v y h H hS W h
C p ,M C p ,U S N vy
D p H G S y U y Pamela C. Masella, DO
N v H p C pL J p A yM C
C pL J ,N h C H ,H

Kevin M. Lin-Hurtubise, MD, FACS, MAJ, MC, Kai Matthes, MD, PhD
USAR A P A h
Ch , I Rv B , PRMC H v M Sh
A C P S y, J h A. B A j S ,B Ch H p
Sh M (JABSOM) B ,M h
S S O P v P A h
D p S y AMGI M , H
p A yM C
H ,H Laura Mazer, MD, MS
S E F
Ali Linsk, MD D p S y
G S yR S U v y
D p S y S ,C
B hI D M C
B ,M h Robert A. Mazzoli, MD, FACS, COL(Ret), MC
D ,E , ,S ,
Alexander Malloy, DO R
S R D D-VA V C E
G S y Oph h P ,R v, O
p A yM C S y
H ,H A P Oph h y
U S v U v y h H hS
(USUHS)
M A yM C
,W h
Co n t r i b u t o r s xvii

Jeremy McCallum, MD, CP , MC Angela Penn, MD, FACS


D p S y, O h p D p S y
p A yM C p A yM C
H ,H H ,H

Robert C. McMurray, MD, CP Richard M. Peterson, MD, MPH, FACS, FASMBS


D p G S y A P ,S y
p A yM C Dv M yI v v G S y
H ,H U v y H hS C S A
S A ,
Marcos Molina, MD
R Brian J. Pottor , MD
B hI D M C G S
B ,M h C H h Phy G p
C ,C
Fares Nigim, MD
Dv N y, D p S y William V. Rice, MD
B hI D M C A P D ,G S yR y
H v M Sh W B A yM C
B ,M h A P S y
P LF Sh M
Mary . ODonnell, MD E P ,
Ch R ,G S A P S y
ACS AEI E F U S v U v y h H hS
USUHS h F B h ,M y
W R N M yM C
B h ,M y Amber E. Ritenour, MD, FACS
G S y
Stephen R. Odom, MD p A yM C
A P S y H ,H
D p S y
B hI D M C Joshua S. Ritenour, MD, FACS
B ,M h G S y
p A yM C
Christopher S. Ogilvy, MD H ,H
P ,N y
B hI D M C E. Matthew Ritter, MD, FACS
H v M Sh A P S y
B ,M h V Ch ,E
P D
Morohunranti O. Oguntoye, MD N Cp C G S yR y
C p h v Oph h P D
D p Oph h y ACS A S E /S
M A yM C F hp
,W h T D p S y U S v
U v y h H hS h W R
N M yM C
B h ,M y
xvi i i Co n t r i b u t o r s

Justin Robbins, MD Jigesh A. Shah, DO


O h p S C A ,S y
M A yM C D p S y
O h p S yS v S .E z h M C
,W h f U v yS h M
B ,M h
Erik Roedel, MD
A S Ranjna Sharma, MD, FACS
p A yM C I S y
H ,H H v M Sh
Dv S O y
Douglas Rowles, MD, CDR B hI D M C
D p S y, O h p B ,M h
p A yM C
H ,H Robert Shawhan, MD
S yR
Ashra A. Sabe, MD M A yM C
B hI D M C ,W h
B ,M h
Robert L. Shef er, MD, COL, MC
John Paul Sanders, MD D p y IRB Ch
C RD A yM C C C F y, I M R y
F H , S M O ,D p M
A P S y p A yM C
U S v U v y h H hS H ,H
B h ,M y
Maxwell Sirkin, MD
Joy Sarkar, MD G S yR , PGY4
R S D p G S y
D p G S y W B A yM C
p A yM C E P ,
H ,H
Richard Smith, MD, FACS
Andrew . Schlussel, DO A P ,S y
D p G S y D p G S y
M A yM C U v y H
,W h H ,H

Brenda Schmidt, MD Peter L. Steinberg, MD


D p G S y U ,B h I D M C
N v M C S D A P (S y)
S D ,C H v M Sh
B ,M h
Albert Jesse Schuette, MD, FAANS
N y Ch I v Joseph R. Sterbis, MD
N y P D ,U yR y
W R N M yM C p A yM C
B h ,M y H ,H
Co n t r i b u t o r s xix

Ajith J. T omas, MD Brad ord P. Whitcomb, MD


A P ,N y Ch
B hI D M C Gy O yS v
H v M Sh p A yM C
B ,M h H ,H

John illou, MD Ashley D. Willoughby, DO


G S yR D p G S y
D p S y p A yM C
B hI D M C /H v H ,H
M Sh
B ,M h Gordon Wisbach, MD, MBA, FACS
A P S y
Mike ran, MD U S v U v y h H hS
S yR F. E H Sh M
B hI D M C B h ,M y
H v M Sh D p G S y
B ,M h N v M C S D
S D ,C
Shawn suda, MD, FACS
A P S y Christopher Yheulon, MD
Ch , D v M yI v v B A P ,S y
S y U S v U v y H hS
U v y N v Sh M B h ,M y
L V ,N v D p S y
p A yM C
Hussna Wakily, MD H ,H
S S
L H M S H p
N Y ,N Y

Paul Wetstein, MD
S yR
p A yM C
H ,H
This page intentionally left blank
Introduction

A y H h H T .I y .
y y ABSI E - I y h p .I
?T v p p h h q / .T
. y p
T v p hp .
h p p y j . R h , h
R v y p , .I ,y y
h q .T v h p .I j y
h h h h h v h h h y h .
p h p. T h P j y h v .
h p h h
y y p h Daniel B. Jones, MD
.

xxi
This page intentionally left blank
Pre ace

I v yh h .T B h q G
I v v p y S h q
p h h h y. F h
.I h I v h q ,h h h v -
h p h - y h .T v -
p I h h v h p p h
p y h - h B C h p
.A I p ,I z h p .W h h , h y y
y h h h y
p .T p v .
p v
. Robert B. Lim, MD

xxiii
This page intentionally left blank
Acknowledgements

T E P h h h v h p v
v h p p :

Chris Barrett, MD
William Cole, MD
James Greenwood
ovy Haber Kamine, MD
Brian Nguyen, MD
Erik Roedel, MD

W z Mike ran, MD h .

xxv
This page intentionally left blank
SECTION 1

General Surgery
This page intentionally left blank
Anest h esia a nd OR Co nc er ns
Robert B. Lim
This page intentionally left blank
1
Sleep Apnea

A l y D. W ll ug by

A 54-year-old male presents to his perioperative D. T e critical closing pressure o this patients
appointment to undergo elective inguinal hernia repair. airway is higher than non-OSA patients.
T e patient has a BMI o 38 with a height o 72 in E. T e patient should be placed in the rendelen-
(1.83 m) and a weight o 127 kg (280 lb). He has a his- burg position.
tory o hypertension currently being managed with a
calcium channel blocker. His wi e reports increasing 3. W p lb yw g b
snoring at night with noticeable gasps or air when lying JD R b f mul ?
supine. Patient denies increasing daytime drowsiness. A. 75 kg
He has no history o prior surgeries and no amily his- B. 98 kg
tory o complications with anesthesia. C. 64 kg
On physical exam, he has a neck circum erence o D. 106 kg
44 cm and a Mallampati score o 3. No cardiovascular or E. 86 kg
respiratory abnormalities are observed. His abdomen is
obese with no evidence o caput medusa. Right inguinal 4. W f f ll w g w ul b b p
ring weakness palpated on exam. f p ly g p ?
A. Vecuronium
1. W f f ll w g NOT f B. Cistracurium
b y yp v l y m (OHS)? C. Rocuronium
A. Obesity (BMI > 30 Kg/m 2) D. Pancuronium
B. Neck circum erence o > 48 cm E. Atracurium
C. Daytime hypoventilation
D. Hypercapnia with PaCO2 > 45 mm Hg 5. W f f ll w g u g g p -
E. Hypoxia with PaO2 < 70 mm Hg p v m gm p w up OSA?
A. T e patient should be placed in the supine position
2. W f f ll w g u g g OSA while recovering to protect the surgical site.
p p v p v v lu B. CPAP should be immediately available or use
m g m f p ? in postoperative patients with known or sus-
A. T is patient has 2 out o 3 risk actors or OSA and pected OSA.
there ore does not require polysomnography. C. Opioid dosing should be based on BW rather
B. Face mask pre-oxygenation will create a higher than IBW.
tidal volume than nasal prongs. D. T e use o thoracic epidural post-operatively is
C. T e Mallampati score o 3 is not a risk or a contraindicated in OSA patients.
di cult intubation. E. Use o CPAP post-operatively could increase the
risk o complications.
6 G EN ERAL S U RG ERY EXAM I N ATIO N AN D BO ARD REVI EW

ANSWERS gradient between the nasopharyngeal and oropharyn-


geal cavities pushing the so palate and tongue orward
1. B. Patients with a neck circum erence o > 48 cm and there ore opening the airway; whereas the positive
have a high probability o developing obstructive sleep pressure through the ace mask will induce an obstruc-
apnea (OSA); however it is not a criterion or OHS. tion. Mallampati score o 3 includes visualization o the
OHS results in hypoventilation and hypoxemia due to so palate and base o uvula. Mallampati scores o
the obesity, while OSA is the blockage o airway that 3 and 4 demonstrate di culty intubation; however
occurs during sleep. Many obese patients have both. they cannot predict di culty o BVM ventilation.
Patients with OHS are at a higher risk or periopera- T e upstream pressure o the pharynx at which air
tive morbidity and mortality. T ese patients are at a entry/ ow ceases is considered the critical closing pres-
higher risk o airway collapse, blunted central respira- sure. T is pressure can be increased by increase in lateral
tory stimulation, and pulmonary hypertension there- pillar at pads compressing the airway, sleep resulting in
ore placing these patients at a higher surgical risk. muscle relaxation or induced by anesthesia. T e ideal
Criteria or OHS: positioning or a patient with OSA is the ramped posi-
a. Obesity (BMI > 30 kg/m 2) tion o intubation or the lateral recumbent, i possible
b. Serum bicarbonate > 27 mEq/L and in reverse rendelenburg position to ease ventila-
c. SpO2< 93% tion, increase total lung capacity, and decrease the lon-
d. ABG demonstrating hypercapnia PaCO 2 gitudinal tension on the upper airway.
> 45 mm Hg and hypoxemia PaO2 < 70 mm Hg
e. An alternative cause o hypoventilation cannot 3. A. JD Robinson ormula states:
be identi ed Man: 52 Kg (115 lbs) + 1.9 Kg (4.2 lbs) per inch
over 60 in;
2. D. I the surgery is elective, is likely to require large Woman: 49 Kg (108 lbs) + 1.7 Kg (3.7 lbs) per
doses o anesthetic agent or opioids intraoperatively, inch over 60 in.
and i there is a high suspicion o undiagnosed OSA
T is calculation is use ul in this patient to deter-
in the perioperative period, it should be postponed
mine optimal paralytic, anesthetic, and opioid dosing
with imminent evaluation and treatment as needed
as well as mechanical ventilation control intraopera-
preoperatively. Evidence has shown that pre-oxygen-
tively and post-operatively as needed.
ation via nasal CPAP mask is superior to ace mask
oxygenation, despite potential air leaks i the mouth is 4. C. All non-depolarizing neuromuscular blockers
allowed to be open. Nasal CPAP increases the pressure act by antagonizing the acetylcholine receptor in a

CLASS 1 CLAS S 2 CLASS 3 CLASS 4

MALLAMPATI CLASS IFICATION


CLASS 1: Soft palate , fa uce s , uvula, pilla rs
CLASS 2: Soft palate , fa uce s , portion of uvula
CLASS 3: Soft palate , bas e of uvula
CLASS 4: Hard pala te only
C H AP TER 1 S LEEP AP N EA 7

reversible/competitive manor. A rapid onset, short than BW due to potential or prolonged duration o
acting non-depolarizing agent would be the best action with BW in obese patients resulting in sup-
option in this patient. Out o the options listed Rocu- pression o respiratory drive and decrease pharyn-
ronium has an onset o 45 to 60 sec and duration o geal muscle stimulation. T e use o a post-operative
30 to 60 min and would be the most ideal. Also obe- epidural can be bene cial in patients undergoing a
sity has not been ound to alter the pharmacokinetics large abdominal operation at risk or requiring large
o Rocuronium and there ore can be dosed on IBW doses o opioids or pain control. T e use o CPAP
or actual body weight. Pancuronium is the longest in the post-operative period has not been shown to
acting and is used in patients that require paralysis increase complications, speci cally the positive pres-
> 1 hr and in patients with normal hepatic and renal sure ventilation has not been proven to increase leak
unction. Cistatracurium and atracurium undergo rates in bariatric surgery patients.
Hof man elimination with an onset o 1 to 2 min and
are intermediate acting. T ese agents would be rec-
ommended in patients with renal or hepatic insu - BIBLIOGRAPHY
ciency. Vecuronium is also an intermediate acting Chau EHL, Mokhlesi B, Chung F. Obesity hypoventilation
NMBA and would be recommended in patients with syndrome and anesthesia. NIH Sleep Medicine Clinics.
2013; 8(1):135147.
cardiovascular disease as it has the least adverse side Hillman et al. Obstructive sleep apnea and anaesthesia. Sleep
ef ect pro le. Prolonged duration o paralysis can Medicine Reviews. Elsevier. 2004;8:45971.
occur when using actual body weight in dosing atra- Oto et al. Continuous positive airway pressure and ventilation
curium and vecuronium. Avoidance o prolonged are more ef ective with a nasal mask than a ull ace mask
paralysis or large doses o longer acting neuromus- in unconscious subjects: a randomized controlled trial.
Critical Care. 2013;17:111.
cular blockers is key.
Puhringer FK et al. Pharmacokinetics o rocuronium bromide
in obese emale patients. European Journal of Anaesthesiology.
5. B. CPAP should be available or patients in the 1999;16(8):507.
immediate post-operative period i OSA is known Robinson JD et al. Determination o ideal body weight or
or suspected. I OSA is suspected, introducing CPAP drug dosage calculations. American Journal of Hospital
in the immediate post-operative period can induce Pharmacology. 1983; Jun; 40(6):10169.
anxiety due to the discom ort o the mask and con- Schumann R. Anaesthesia or bariatric surgery. Best Practice
and Research Clinical Anaesthesiology. 2011;25:8393.
usion rom the remaining sedatives on board. T e
ietze K. Use o neuromuscular blocking medications in critically
proper positioning or optimal airway patency is in ill patients. http://www.uptodate.com/contents/use-o -
the upright and lateral decubitus position i possi- neuromuscular-blocking-medications-in-critically-ill-
ble. Opioid dosing should be based on IBW rather patients. J Pharm D. 2013.
2
Malignant Hypertension

Harry T. Aubin

A 59-year-old emale with poorly controlled hyperten- C. emperature > 38.5


sion on three antihypertensive medications including a D. Pulmonary edema
beta-blocker, angiotensin converting enzyme inhibitor, E. achycardia > 100
and a calcium channel blocker, presents or a routine
surgery. At her preoperative visit, she is instructed to 3. Regarding the medication nitroprusside, which o
hold her home ACE inhibitor the day o surgery. She the ollowing is a eared side e ect with excessive
then undergoes an otherwise uncomplicated cholecys- use?
tectomy with intraoperative cholangiogram. However, A. Cyanide toxicity
in the postoperative care unit, she develops hyperten- B. remor
sion with readings o 200/140. Her other vital signs are C. Stroke
within the normal range. T e anesthesia provider per- D. Myocardial in arction
orms an eye exam and notes papilledema. Nitroprus- E. Angina pectoris
side is administered IV or treatment.
4. Regarding medications to treat malignant hyper-
1. Regarding the immediate management o this tension acutely, which o the ollowing is most
patients malignant hypertension, which o the likely to cause ref exive tachycardia?
ollowing is correct? A. Labetolol
A. Goal BP is reduction to normal rage o SBP B. Hydralazine
< 120, DBP < 80 as soon as possible to prevent C. Metoprolol
stroke. D. Clevidipine
B. Reduction o DBP to 100-105 over 3 hours with E. Fenoldapam
maximum all in BP by 25% over 24 hours is
needed to prevent stroke. 5. Regarding underlying causes o poorly controlled
C. Heart rate control is acutely needed to prevent hypertension, which o the ollowing is the most
worsening cerebral edema. likely cause in this patient?
D. Blood pressure should be maintained at current A. Pheochromocytoma
elevated levels. B. Cushings syndrome
C. Renal artery stenosis
2. Regarding malignant hypertension, which o the D. T yroid storm
ollowing is a clinical sign o this diagnosis? E. Glomerulonephritis
A. Blood pressure > 150/110
B. Papilledema
C H AP TER 2 MAl i g n An T H y P ERTEn s i o n 9

ANSWERS or renal ailure. T e treatment o cyanide toxicity is


multimodal and includes sodium nitrite, hyperbaric
1. B. T e goal in malignant hypertension is reduction oxygen, and sodium thiosul ate.
in DBP to 100-105 with maximum all in by 25% o remor is not a known side e ect o nitroprusside,
highest BP value over 24 hours. T is slow decrease but hypere exia is commonly seen in toxic levels o
prevents re exive vasoconstriction via normal body this drug leading to cyanide toxicity.
auto-regulatory mechanisms leading to stroke. Reduc- Stroke, myocardial in arction, and angina pectoris
tion o blood pressure to the normal range increases are not published side e ects o nitroprusside. T ese
the risk o stroke and is not recommended. are more characteristic o malignant hypertension
Heart rate control is not a main goal o care in malig- itsel due to end organ damage rom capillary and
nant hypertension. T e mechanism o cerebral edema arteriole damage to the heart.
is elt to be overcoming the bodys auto-regulatory
mechanism o vasoconstriction with increase in MAP 4. B. Hydralazine is a direct arteriolar vasodilator. It
allowing or a relatively constant end organ per usion has rapid onset and short hal -li e administered
pressure. When BP increases above 180 systolic, auto- IV. It can commonly cause re exive tachycardia by
regulatory vasoconstrictions ails, and vasodilation is two mechanisms. Re exive catecholamine release
seen leading to an increase in blood ow to the brain. in response to vasodilation and decreased vascular
BP control, not heart rate, prevents cerebral edema resistance directly stimulates the cardiac myocytes
by reducing cerebral per usion pressure (intracranial by beta-1 adrenergic receptors leading to tachycar-
pressure mean diastolic pressure) and permitting dia. Additionally, due to decrease in renal blood ow,
auto-regulation thus reducing end organ damage. the juxtaglomerular apparatus secretes renin leading
to increased aldosterone secretion. Aldosterone is a
2. B. Malignant hypertension is de ned by blood pres- potent vasoconstrictor that decreases venous return.
sure > 180/120 with signs o cerebral edema and/or As a compensatory mechanism, heart rate increases
end organ damage. Cerebral edema is characterized to compensate and keep cardiac output constant (CO
by clinical signs o brain swelling. Papilledema is the = HR SV). Re exive tachycardia is commonly
most worrisome sign. However, retinal hemorrhages seen in patients who are not concomitantly on beta
and retinal exudates are indicative o hypertension blockers and angiotensin-converting enzyme (ACE)
causing damage to arterioles and capillary beds. Other inhibitors.
end organ ndings include acute kidney injury, myo- Labetolol is both and alpha-1 and beta-1 antago-
cardial in arction, aortic dissection or bowel ischemia. nist. It has rapid onset and is ideal or patients with
achycardia is not a criterion or diagnosing tachycardia and some hypertension. It does not
malignant hypertension. achycardia is commonly cause re ex tachycardia due to inhibition o beta-1
seen in postoperative patients and could be attrib- receptors.
uted to catecholamine release rom the stressors o Metoprolol is a beta-blocker and will decrease
surgery, pain, intravascular depletion, medications, heart rate. It is ideal in atrial brillation, with little
arrhythmias, and/or atelectasis. Its presence necessi- e cacy in malignant hypertension.
tates close observation and thorough work-up. Clevidipine is a dihydropyridine calcium channel
blocker. It has rapid onset, very short hal -li e, and is
3. A. Nitroprusside is an artrio-venous dilator that has
administered intravenously. Because it works periph-
rapid onset and short hal -li e. It, as well as ast act-
erally, it does not cause re exive tachycardia.
ing medications like clevidipine, nicardipine, labeto-
Fenoldapam is a dopamine-1 receptor agonist. It
lol, and enoldapam, are used or acute treatment o
commonly causes ushing and hypotension. It does
hypertension. With over administration o nitroprus-
not cause tachycardia because it works peripherally.
side, cyanide toxicity can develop as this medication
contains cyanide groups (carbon triple bonded with 5. C. Renal artery stenosis is a common underlying
nitrogen). Cyanide toxicity is detrimental to aerobic cause o malignant hypertension and is requently
metabolism at the cellular level by inhibiting the last seen in Caucasians who have poor blood pressure
enzyme in oxidative phosphorylation, cytochrome control despite multimodal therapy. Renal artery ste-
oxidase (a3). Cyanide toxicity can present with nosis can present as worsening azotemia in relatively
headache, nausea, emesis, and ushing, hepatic and/ young individuals, poorly controlled hypertension,
10 g En ERAl s U Rg ERy EXAM i n ATi o n An D Bo ARD REVi EW

and/or malignant hypertension. It is diagnosed non- alone can cause renal ailure. However, renal artery
invasively via renal artery duplex and can be treated stenosis and not GN is associated with malignant
with renal artery stenting. However, this treatment is hypertension.
becoming more controversial given the recent publi-
cation o the CORAL trial (Hermann SM et al. 2013) BIBLIOGRAPHY
arguing or medical management alone. Armario P, Dernandez del Rey R, Pardell H. Adverse E ects
Pheochromocytoma is a catecholamine releas- o Direct-acting Vasodilators. Drug Saf. 1994; Aug; 11(2):
ing tumor that can cause hypertension. It is usually 805.
Davis B, Crook J, Vestal R, Oates J. Prevalence o renovascular
episodic and can present with ushing, palpitations,
hypertension in patients with grade III or IV hypertensive
diaphoresis, and other signs o catecholamine release. retinopathy. N Engl J Med. 1979;301(23):1273.
It is diagnosed clinically by history and urine VMAs. Hermann S, Saad A, extor S. Management o atherosclerotic
Cushings syndrome can cause hypertension due renovascular disease af er Cardiovascular Outcomes in
to cortisol excess, but is not as common as renal Renal Atherosclerotic Lesions (CORAL). Nephrology Dial-
artery stenosis in malignant hypertension. Conns ysis Transplant. 2014; Epub ahead o print.
Kaplan NM. Management o Hypertensive Emergencies. Lancet.
Syndrome or hyperaldosteronism is the over pro- 1994;344(8933):1335.
duction o aldosterone by the adrenal gland can also Marik PE, Varon J. Hypertensive crises: challenges and man-
cause hypertension. agement. Chest. 2007;131(6):1949.
T yroid storm typically presents with tachycardia. Pasch , Schulz V, Hoppelshauser G. Nitroprusside-induced
It is treated typically with nonselective beta-blockade ormation o cyanide and its detoxi cation with thiosulphate
during deliberate hypotension. J Cardiovasc Paharmacol.
and propylthiouracil (P U).
1983;(5):7785.
Glomerulonephritis seen in nephritic syndrome Strandgaard S, Paulson O. Cerebral blood ow and its patho-
can cause hypertension and renal ailure. Additionally, physiology in hypertension. Am J Hypertension. 1989;
both renal artery stenosis and malignant hypertension 2(6 Pt 1):486.
3
Fundamentals or Use o
Surgical EnergyFire During
emporal Artery Biopsy
Paul Wetstein

A 74-year-old emale with COPD and a baseline oxygen 2. Which o the ollowing is correct regarding this
requirement presented to the emergency department with scenario?
subjective evers, headache, and jaw claudication. Labora- A. Surgery on the head and neck should be identi ed
tory evaluation was notable or an erythrocyte sedimen- preoperatively by the surgeon and anesthesiolo-
tation rate (ESR) o 56 mm/hr. A non-contrast C o the gist as low risk.
head was obtained which revealed no acute pathology. T e B. Intraoperative communication between the sur-
patient was admitted to the internal medicine service and geon and anesthesiologist is not needed in a case
started on high dose prednisone with a working diagnosis o expected short duration.
o giant cell (temporal) arteritis. During a temporal artery C. Sedation with open gas delivery device would be
biopsy, an operating room re occurs. pre erred to general endotracheal anesthesia in
this patient to prevent an OR re.
1. Which o the ollowing is correct regarding
D. Surgical drapes should be con gured in a manner
operating room f res?
as to minimize the accumulation o oxidizers.
A. wo o the three components o the classically E. Moistening surgical sponges has no impact in
described re triad must be present or an OR preventing an OR re.
re to occur.
B. T e most common OR re uel is the monopolar 3. In the event o f re involvement o the airway or
electrosurgical cautery Bovie. breathing circuit, the best f rst step is to:
C. An oxidizer enriched atmosphere o en exists in A. Stop the f ow o all gases to the airway.
the entire operating room. B. Remove all uels rom the airway.
D. Alcohol containing prep solutions need to be C. Activate re alarm.
completely dry be ore starting a procedure. D. Per orm beroptic bronchoscopy with the endo-
E. Fiberoptic light sources or endoscopic surgery tracheal tube in place.
do not serve as an ignition source. E. Pour saline into the airway.
12 G EN ERAL S U RG ERY EXAM IN ATI O N AN D BO ARD REVIEW

ANSWERS regardless o the length o the procedure. Surgical


drapes should be arranged to prevent an accumula-
1. D. In order or a re to occur, all three components tion o oxidized air. Moistened surgical sponges can
o the re triad must be present. T ese include help prevent OR res.
uel, and oxidizer, and an ignition source. Fuel or
re is plenti ul in the operating room. Some exam- 3. A. Immediate actions to be per ormed in the event
ples include drapes, patients hair, surgical gowns, o an airway re include rst removing the endotra-
blankets, endotracheal tubes, and lyngeal mask air- cheal tube or LMA, then stopping the f ow o ALL
ways and volatile surgical compounds (e.g., alcohol gases, removal o uel sources away rom the airway,
containing prep solutions, acetone, etc.). It has been and pouring saline into the airway. Once the re has
shown that alcohol containing prep solutions with been extinguished, actions should include ventilation
as little as 20% alcohol can ignite with diathermy o the patient while avoiding oxidizer-enriched envi-
or hot wire cautery and so they must be allowed to ronments, inspection o the tracheal tube or LMA to
dry be ore surgical electricity is used. Oxidizers in ensure no ragments remain in the patients airway,
the operating room are generally either oxygen or and consideration o bronchoscopy. Bronchoscopy is
nitrous oxide. T ese oxidizers can accumulate and a relatively sa e procedure in experienced hands in
orm an oxidizer enriched atmosphere in closed or diagnosing inhalational injury but is not part o the
semi-closed breathing systems and rom tenting o immediate management o an airway re.
surgical drapes. Ignition sources in the operating
room are equally as plenti ul. Some common exam- BIBLIOGRAPHY
ples include electrosurgical devices, heated probes, Ap elbaum JL, Caplan RA, Barker SJ, et al. Practice advisory
lasers, beroptic light cables, argon beam coagula- or the prevention and management o operating room
tors, drills and de brillator pads. res: an updated report by the American Society o Anes-
thesiologists ask Force on Operating Room Fires. Anes-
2. D. According to the American Society o Anesthe- thesiology. 2013; Feb; 118(2):27190.
Bai C, Huang H, Yao X, et al. Application o f exible bronchos-
siologists 2013 ask Force on operating room res,
copy in inhalation lung injury. Diagnostic Pathology. 2013;
an endotracheal tube or LMA should be considered Oct 21:8:174.
in patients undergoing moderate to deep sedation Briscoe CE, Hill DW, Payne JP. Inf ammable antiseptics and
or that have a baseline oxygen requirement. Head theatre res. Br J Surg. 1976; Dec; 63(12):9813.
and neck surgery should be considered high risk DeMaria S, Schwartz AD, Narine V, et al. Management o
or an OR re and as such, communication between Intraoperative Airway Fire. Simulation in Healthcare: T e
Journal of the Society for Simulation in Healthcare. 2011;
the surgeon and the anesthesiologist is mandatory Dec(6):3603.
4
Argon Gas Embolism

Roger Eduardo

T e patient is a 37-year-old emale without any signi cant 3. What is the most important factor associated with
past medical history undergoing a laparoscopic partial an increased risk of venous gas embolism when
right hepatic lobectomy or a large symptomatic hepatic using argon beam coagulation?
adenoma in segment VI o the liver. Endotracheal intuba- A. Use under pneumoperitoneum
tion is per ormed without complication and the abdomen B. High ow rate o argon gas
is entered via the Hassan technique. T e lesion is identi- C. Holding the tip o the electrode at a right angle to
ed on the in eromedial aspect o segment VI. o dissect the tissue
the lesion away rom the liver parenchyma, an argon beam D. Placing tip o argon beam electrode in direct
coagulator is used. wo hours a er the start o the proce- contact with tissue sur ace
dure, there is an abrupt decrease in the patients E CO2
rom 30 to 10 mm Hg and spO2 rom 100% to 40%. T is is 4. What steps can be taken to reduce these risk
rapidly ollowed by a decrease in arterial blood pressure to factors?
60/25 mm Hg and heart rate rom 80 to less than 20. A. Never place the electrode tip less than several
millimeters rom the surgical site.
1. Why is the argon beam coagulator used over other
B. Limit argon ow settings to lowest level that pro-
types of electrocautery?
vides the desired clinical e ect.
A. T e ow o gas clears the site o uids and blood, C. Move the hand piece away rom the tissue a er
enhancing visibility. each activation.
B. Rapid non-contact uni orm tissue coagulation D. Flush abdominal cavity with CO2 between
over a large area extended activation periods o use.
C. Less adjacent tissue damage rom reduced depth E. All o the above
o penetration
D. Less generation o surgical smoke 5. What would be your next step in management of
E. All o the above this patient?
A. Continue the surgery.
2. What was the most likely cause of this patients
B. Administer atropine and initiate vasopressors.
decrease in ETCO2 and arterial blood pressure?
C. Discontinue pneumoperitoneum and place patient
A. Acute myocardial in arct in Durants position.
B. Decreased venous return secondary to pneumo- D. Per orm emergent EE to diagnose a gas embolism.
peritoneum E. Begin immediate volume resuscitation.
C. Aspiration
D. Gas embolism
E. Severe cerebral vascular accident
14 G EN ERAL S U RG ERY EXAM I N ATIO N AN D BO ARD REVI EW

ANSWERS can embolize not only through major veins but also
through small peripheral veins.
1. E. Argon beam coagulation has gained popularity
among surgeons as a use ul tool to achieve hemosta- 3. D. T e rst ew cases o venous embolism with use
sis in bleeding sur aces o highly vascularized organs o the argon beam were reported during laparoscopic
such as the liver and spleen. It utilizes a monopolar procedures and there ore, the theory o over-insuf a-
electrode to partially ionize a stream o argon gas tion and over-pressurization o the abdominal cav-
that is directed towards the tissue or coagulation. ity caused by the accumulation o argon gas under
T e ionized argon beam acts as an e cient pathway, pneumoperitoneum was thought to lead to these
conducting a high- requency electric current rom embolic events. However, given that venous emboli
the electrode to the target tissue resulting in a ne have occurred in several cases o patients undergo-
spray o electrical sparks. ing procedures without pneumoperitoneum, this
As the electrical beams directed rom the electrode theory cannot ully explain the incidence o these
to the target tissue causes desiccation, the electrical events. Ikegami et al. ( J Hepatobiliary Pancreat Surg.
conductivity o the targets tissue is lost. I continu- 2009;16(3):3948) compared seven reported cases o
ally applied, the beams automatically move to nearby venous embolism using argon beam coagulation and
non-desiccated and still electrically conductive tissue identi ed the ollowing risk actors:
allowing or rapid uni orm coagulation over a large
1. Using the argon gas under pneumoperitoneum;
area without any tissue contact. Furthermore, as a
2. Puncturing the liver parenchyma (hepatic needle
result o the loss o electric conductivity at a treated
biopsy);
site, the depth o penetration o the electrical energy
3. Possible injury to the hepatic venous system; and
is reduced. T is, coupled with the act that the use o
4. Placing tip o argon beam electrode in direct
argon gas, due to its inert nature, neither carbonizes
contact with tissue sur ace.
nor vaporizes biologic tissue so that the thermal e ects
are limited, results in less adjacent tissue damage. On review o the literature, it appears that more
important than the issue o use under pneumoperi-
2. D. With use o the argon beam coagulation system toneum is that o placing the tip o the argon beam
in laparoscopic procedures, the argon system acts electrode in close or direct contact with the tissue that
as a secondary source o pressurized gas and argon is being treated. Multiple cases have been described,
can accumulate in the closed peritoneal cavity. With without the use o penumoperitoneum, where this
damage to any signi cant blood vessels, the gas under is clearly the issue and in the series described above,
pressure can enter the vasculature posing a risk o though only three cases described this, it is possible
embolism that could be a mixture o both argon and that more might have had this condition and simply
carbon dioxide. Moreover, the argon gas stream that not reported.
ows between the electrode and the tissue can cross When used at ow rates o 0.2 to 2 L/min and a
any disrupted mucosal membrane sur ace and be power o 20 to 80 W as described in the eld o inter-
ushed directly into the microvasculature. ventional pulmonology or ablation o small lesions,
Although argon is physiologically inert, it is 17 the argon beam system can penetrate the tissue up to
times less soluble than carbon dioxide (0.029 ver- 5 mm in depth. However, when used or the purpose
sus 0.495 ml gas ml1 blood) and as such, argon-rich o hemostasis in highly vascularized tissues such
emboli are not as readily absorbed rom the blood as the spleen or hepatic parenchyma, a ow rate o
stream as CO2 and may pass into the systemic cir- 4 L/min and power o 150 W is typical. T is allows
culation. At the standard ow setting o 4 L/min or even urther penetration and when coupled with
used typically or hemostasis in highly vascularized direct sur ace contact could allow or vessel damage
organs, the argon beam electrode can produce 67 ml and or the argon gas to be ushed directly into the
o gas in only one second which, i embolized, can venous system.
lead to signi cant cardiopulmonary dys unction
and be potentially lethal in an average size adult. 4. E. T e cautions o the manu acturer include:
Furthermore, at such a high ow rate argon gas
1. Never place the electrode tip less than several
clearly exceeds pressure in the venous system and
millimeters rom the surgical site.
C H AP TER 4 ARG O N G AS EM BO LI S M 15

2. Limit argon ow settings to lowest level that pro- BIBLIOGRAPHY


vides the desired clinical e ect. Cornejo A, Liao L, Kenneth W. Argon gas embolism with
3. Hold the tip o the electrode at an oblique angle. the use o argon beam coagulation during open hepatic
4. Move the hand piece away rom the tissue a er resection. 2009;22(2). Available rom http://ispub.com/
each activation. IJS/22/2/7972. T e Internet Journal o Surgery Web site.
Croce E, Azzola M, Russo R, Golia M, Angelini S, Olmi S.
5. Flush abdominal cavity with CO 2 between Laparoscopic liver tumour resection with the argon beam.
extended activation periods o ABC. Endosc Surg Allied echnol. 1994;2(34):1868.
6. Always leave one instrument cannula open to Farin G and Grund KE. echnology o argon plasma coagu-
the atmosphere. lation with particular regard to endoscopic applications.
Endosc Surg Allied echnol. 1994;2:717.
Another recommended tip is the use o a venting Feldman L, Fuchshuber P, and Jones DB. T e SAGES Manual
port at all times when operating laparoscopically and on the Fundamental Use of Surgical Energy. Berlin, Ger-
using surgical energy. T is will allow gas to escape many: Springer-Verlag; 2012.
but maintain a pneumoperitoneum. Kono M, Yahagi N, Kitahara M, Fujiwara Y, Sha M, Ohmura A.
Cardiac arrest associated with use o an argon beam coag-
ulator during laparoscopic cholecystectomy. Br J Anaesth.
5. C. In conjunction with appropriate cardiac resus-
2001;87(4):6446.
citation according to ACLS guidelines i indicated, Park EY, Kwon JY, Kim KJ. Carbon dioxide embolism during
the treatment o a patient suspected o having a CO2 laparoscopic surgery. Yonsei Med J. 2012;53:45966.
or any gas embolism should include immediate dis- Ikegami , Shimada M, Imura S, et al. Argon gas embolism
continuation o CO2 insuf ation. T e patient should in the application o laparoscopic microwave coagulation
be placed in Durants position (le lateral decubitus therapy. J Hepatobiliary Pancreat Surg. 2009;16(3):3948.
Economic Cycle Research Institute (ECRI). Fatal gas embo-
with steep head down). T is allows the gas to rise into lism caused by overpressurization during laparoscopic use
the apex o the right heart, preventing entry into the o argon enhanced coagulation. Health Devices. 1994 Jun;
pulmonary artery and keeping it there until it slowly 23(6):2579.
absorbs. Use o nitrous oxide inhalant should be dis- Mann C, Boccara G, Grevy V, Navarro F, Fabre JM, Colson P.
continued to allow or hyperventilation with 100% Argon pneumoperitoneum is more dangerous than CO2
pneumoperitoneum during venous gas embolism. Anesth
oxygen to increase clearance o CO2 or any other
Analg. 1997;85:136771.
gas and to relieve hypoxemia. Volume expansion Min SK, Kim JH, Lee SY. Carbon dioxide and argon gas embo-
with bolus crystalloid may reduce urther gas entry lism during laparoscopic hepatic resection. Acta Anaesthe-
by elevating CVP. And lastly, placement o a central siol Scand. 2007;51(7):94953.
venous catheter or attempted aspiration o the gas Reddy C, Majid A, Michaud G, Feller-Kopman D, Eberhardt
rom the right heart may also be per ormed. While a R, Herth F, et al. Gas embolism ollowing bronchoscopic
argon plasma coagulation: a case series. Chest. 2008; 134(5):
transesophageal echo ( EE) may be diagnostic in the 10669.
event o a venous gas embolism, it is not a priority in Veyckemans F, Michel I. Venous gas embolism rom an argon
the unstable patient with suspected gas embolism. coagulator. Anesthesiology. 1996;85(2):4434.
This page intentionally left blank
Adva nc es in La pa r o sc o py
Robert B. Lim
This page intentionally left blank
5
FLSAortic rocar Per oration

Megan Bowen

A 32-year-old woman, with a body mass index o 3. Injury to a major vessel a er trocar placement is
30 kg/m 2, was admitted to the hospital with acute usually signif ed by?
appendicitis. She was consented or a laparoscopic A. Visible bleeding
appendectomy. She had a previous surgical history o B. Retroperitoneal hematoma
cesarean section. C. Bradycardia
Following the administration o general anesthesia, D. Hypoxia
a curvilinear in ra-umbilical skin incision was made.
Blunt dissection was carried down to the ascia and the 4. T e distance between the abdominal wall and the
ascia was elevated and incised. Entry into the perito- aortic bi urcation in normal weight women (BMI
neal cavity was con rmed visually and a Hasson trocar < 25 kg/m2) is?
was inserted into the peritoneal cavity. A. 1.5 cm
B. 2.4 cm
1. What is the sa est technique to gain access to the
C. 3.5 cm
peritoneum or laparoscopic surgery?
D. 0.4 cm
A. Open technique (Hasson)
B. Veress needle
C. Direct trocar technique
D. None o the above

2. Following insu ation, a 30 laparoscope was


inserted. Upon general laparoscopic examination
o the peritoneal cavity, it was apparent that a
retroperitoneal hematoma was orming. What is
the most commonly injured vessel during trocar
placement?
A. Iliac vein
B. In erior vena cava
C. Aorta
D. Lumbar veins
E. Superior mesenteric vessels
20 G EN ERAL S U RG ERY EXAM IN ATIO N AN D BO ARD REVI EW

ANSWERS Un ortunately, 15% to 50% o the vascular inju-


ries are not diagnosed at the time o injury. T is delay
1. D. T ere have been many studies done comparing has contributed to mortality rates o 3% to 30% or
the sa ety o the open technique to the closed and vascular injuries.
direct entry techniques. T ere has been no obvi-
ous advantage o one technique over another. One 4. D. A study by Hurd et al. (Obstet Gynecol. 1992;80(1):
large meta-analysis showed an incidence o vascular 4851) that ound that the distance between the
injury to be 0.44% in the closed cases compared to umbilicus and the aortic bi urcation was 0.4 cm in
0% in the open cases. Another large study compared normal weight women (BMI < 25 kg/m), 2.4 cm in
the Veress, open and direct trocar techniques, and overweight patients (BMI 2530 kg/m) and 2.9 cm
ound a rate o vascular injury o 0.04%, 0.01%, and in obese patients (BMI > 30 kg/m). Lif ing the abdom-
0% respectively. inal wall may improve sa ety by increasing the distance
between the abdominal wall and the viscera. Lif ing
2. A. Vascular injuries may involve retroperitoneal, the abdominal wall by placing towel clips within 2 cm
intraperitoneal or abdominal wall vessels. Rates o o the umbilicus has been shown to provide signi cant
major vascular injury during initial trocar entry are elevation o the peritoneum (6.8 cm above the viscera)
between 0.05% and 0.5%. T e sites o injury rom that was maintained during insertion.
most common to least common are iliac vein, greater
omental vessels, in erior vena cava, aorta, pelvic
and superior mesenteric veins, and lumbar veins. BIBLIOGRAPHY
Injury to a major vessel is usually signi ed by visible Hurd WW, Bude RO, DeLancey JO, Pearl ML. T e relation-
bleeding and hemodynamic instability. I an injury ship o the umbilicus to the aortic bi urcation: implications
or laparoscopic technique. Obstet Gynecol. 1992;80(1):
is con rmed or highly suspected, especially in the 4851.
retroperitoneum, convert to an open procedure and Pemberton RJ, olley DA, van Velthoven RF. Prevention and
explore the area in question. Management o Complications in Urological Laparoscopic
Port Site Placement. European Urology. 2006;50:95868.
3. A. Vascular injury is usually diagnosed by direct view Seidman DS, Nasserbakht F, Nezhat F, et al. Delayed recogni-
o bleeding in the abdominal cavity. T e absence o tion o iliac artery injury during laparoscopic surgery. Surg
ree intraperitoneal blood caused by retroperitoneal Endosc. 1996;10:1099101.
Vilos GA, ernamian A, Dempter J, Laberge PY. Laparoscopic
bleeding may delay the diagnosis, because blood is not Entry: A Review o echniques, echnologies, and Compli-
observed through the laparoscope. Clinical signs o cations. J Obstet Gynaecol Can. 2007;29(5):433447.
hemodynamic instability (tachycardia, hypotension), Vilos GA, Vilos AG, Abu-Ra ea B, Hollett-Caines J, Nikkhah-
shortly af er needle or trocar insertion, suggest a vas- Abyaneh Z, Edris F. T ree simple steps during closed lapa-
cular injury. High intraabdominal pressure secondary roscopic entry may minimize major injuries. Surg Endosc.
2009;23:758764.
to pneumoperitoneum is associated with a decrease
Wind J, Cremers J, van Berge Henegouwen M, Gouma D,
in venous return, which in turn, can reduce arterial Jansen F, Bemelman W. Medical liability insurance claims
bleeding. Furthermore, a retroperitoneal hematoma on entry-related complications in laparoscopy. Surg Endosc.
can decrease a vessel leak, restraining the bleeding. 2007;21:20949.
6
Advanced Laparoscopy
and NO ES
Mark A. Gromski and Kai Matthes

Ms. B is a 33 year-old emale presenting to a general sur- 2. T e SIL approach is more technically demanding
gery clinic to be evaluated or a potential cholecystec- than standard multi-port laparoscopic surgeries,
tomy. She has a history o periodic symptomatic biliary and requires additional specialized training.
colic over the past nine months. Approximately 6 weeks Which is the most relevant reason or the increased
ago, she was evaluated in the local emergency depart- technical di culty in SIL?
ment or severe right upper quadrant pain. A right upper A. Loss o triangulation
quadrant ultrasound showed no evidence o cholecysti- B. Cramped external working space secondary to
tis but did show multiple stones in the gallbladder. Her close proximity o instruments causing ergo-
pain subsided with pain medications and IV uids, and nomic di culty
she was discharged rom the emergency department C. Decreased eld o view
with outpatient general surgery ollow-up. Prior to this, D. Reduced range o motion o instruments
she had had two additional episodes o intermittent right
upper quadrant pain. 3. A patient was re erred to a surgeon within the same
She is an otherwise healthy individual, only tak- institution who is well-experienced in single incision
ing oral birth control medications and a multivitamin laparoscopic cholecystectomy. When discussing the
daily. Her body mass index is 26 kg/m 2. She has had no single incision approach with the patient, attention
previous abdominal operations. should be brought to which potential complication
Her main concern is scarring that may result rom a that may be seen more requently in single-
potential gallbladder operation, and she comes to you incision cholecystectomy compared to standard
to discuss this. multi-port cholecystectomy?
A. Signi cant bleeding complications
1. What is the most appropriate intervention to B. Post-operative herniation
recommend at this time? C. Pain
A. Standard laparoscopic cholecystectomy by general D. Surgical site in ections
surgeon
B. Single-incision laparoscopic (SIL) cholecystectomy 4. A patient with achalasia is expressing signif cant
by general surgeon with no experience in SIL concerns over cosmesis. Which developing min-
C. Re erral to specialty center or transvaginal imally invasive technique is considered an acceptable
Natural Ori ce ranslumenal Endoscopic Surgery alternative to a multi-port laparoscopic Heller
(NO ES) cholecystectomy myotomy?
D. Ursodiol 8 to 10 mg/kg/day (in 2 divided doses A. Per-oral endoscopic myotomy (POEM)
daily) B. Single-port laparoscopic Heller myotomy
C. Hand-assisted laparoscopic Heller myotomy
D. ransvaginal NO ES myotomy
22 G EN ERAL S U RG ERY EXAM I N ATIO N AN D BO ARD REVI EW

5. Which o the ollowing is true regarding NO ES? that procedure be ore, but a multi-port laparoscopic
A. Sa e access to the peritoneal cavity is generally cholecystectomy by a general surgeon.
not di cult to accomplish.
B. E ective management o iatrogenic intraperito- 2. B. Given the instrument ports in single-port lapa-
neal complications are easily recognized. roscopy (SIL) surgery are in such close proximity
C. E ective training requirements are well estab- to each other, the surgeon must utilize laparoscopic
lished. devices which are, in essence, operated in parallel,
D. Most new equipment have multitasking plat orms causing di culty in triangulation and awkward oper-
to accomplish procedures. ating postures. Curved instruments are commer-
cially available providing some limited triangulation.
T e close proximity o the laparoscopic instruments,
ANSWERS of en requiring a crossing o the devices, is consid-
1. A. Multi-port laparoscopic is considered the gold ered the most challenging aspect o SIL in compari-
standard approach to cholecystectomy, compared son to a multi-port approach. Furthermore, given
to open approach, given equivalent serious com- the camera can only be placed in one anatomical
plications, improved cosmesis, improved pain and position (umbilicus), it can create a decreased eld
decreased length o stay. Particularly considering o view compared to multi-port access. In the same
the patients concerns regarding cosmesis, an open regard, given the instruments are relegated to a sin-
approach would not be appropriate in this case. gle anatomical position, the range o motion o the
Ursodiol does not exert an in uence on symp- instruments can at times be compromised com-
tomatic gallstones that is nearly as e cacious or pared to a multi-port approach, but these latter two
durable compared to cholecystectomy. Particularly reasons are considered less signi cant and encoun-
considering this patient is a very low risk surgical tered less requently than the ergonomic di culties
candidate, cholecystectomy is the correct approach. o SIL.
Although there have been a number o large It has been proposed that robotic single-incision
series published regarding intra-abdominal natural laparoscopic techniques may overcome the ergo-
ori ce transluminal endoscopic surgery procedures nomic di culties o current SIL procedures.
(NO ES; the majority o which are transvaginal
cholecystectomy), NO ES procedures require highly 3. B. A recent meta-analysis per ormed demonstrated
specialized training, specialized technology and that there were no clinically signi cant di erences
mastery o the exible endoscope. A rigorous rand- in bleeding or pain between single incision and
omized controlled trial comparing NO ES cholecys- standard multi-port cholecystectomy. Another meta-
tectomy to standard laparoscopic cholecystectomy is analysis also demonstrated no di erence in surgical
underway and nearing completion (NC 01171027). site in ections. T ere are no high quality data demon-
Furthermore, the vast majority o clinical NO ES strating that there is a comparative mortality di erence
transvaginal cholecystectomy cases reported were with either procedure.
hybrid cases, with at least one laparoscopic port. In T ere is, however, a signi cant di erence in post-
the absence o rigorous comparative data regard- operative incisional hernia, with more occurring in
ing NO ES cholecystectomy to the standard o care, the single incision group. T is should be explained
it would be premature to recommend this as the best to the patient in the clinic, particularly i she is plan-
treatment option available. ning to become pregnant in the uture. T is increase
Finally, there is a de nite learning curve to single in post-operative hernia is likely secondary to the
incision cholecystectomy. One particular study dem- length o the ascial incision required by whichever
onstrated a learning curve o about 25 cases, with the single-incision plat orm is chosen. T e ascial incision
majority o major complications, including conversion can be up to 7 cm with some devices.
to standard or open cholecystectomy, to occur in the Furthermore, there are data that the rate o pro-
rst 10 cases. Given this, despite the patients concerns cedural ailure is higher in the single incision group
regarding cosmesis, the sa est and most appropriate (mainly relating to conversion to multi-port laparo-
option presented would not be a single incision chol- scopic) and the time o procedure is longer in the
ecystectomy by a surgeon who had never per ormed single incision group.
C H AP TER 6 AD VAN C ED LAPARO S CO P Y AN D N O TES 23

Finally, relative to this patient, there are data that T ere remain signi cant concerns regarding
single incision laparoscopic cholecystectomy ares adequate training or NO ES procedures. T ere are
better on post-operative cosmesis scores than multi- questions regarding the appropriate composition o
port cholecystectomies. teams (gastroenterologist versus surgeon versus both),
the amount o advanced exible endoscopy experi-
4. A. Multi-port laparoscopic Heller myotomy is the ence required, and the appropriate amount o ex-vivo
current gold standard operative approach or the simulation and live animal training prior to human
treatment o achalasia. A single incision laparoscopic attempts.
approach has been described in the literature, but In addition, there is a 5% to 10% rate o compli-
only as single case reports. T e anatomical location cations cited in current NO ES registries. T ere is
o the repair makes a single incision approach di - concern that current endoscopic techniques may not
cult, and this technique has not been widely adopted. be able to adequately address intraperitoneal surgical
Peroral Endoscopic Myotomy (POEM) is an endo- complications, and that the delay in transitioning to
scopic surgical approach that was rst described less a laparoscopic approach to de nitively manage com-
than a decade ago, and has gained increasing atten- plications may result in poorer outcomes. Addition-
tion and adoption by surgeons and gastrointestinal ally, i gastroenterologists are a part o the NO ES
endoscopists alike. T e procedure consists o endo- team (as they may be more acile with advanced
scopically creating a submucosal tunnel in the distal exible endoscopy), they are likely to be less amiliar
esophagus, ollowed by creating a complete myotomy, with the intraperitoneal anatomy and may not as
then retraction o the endoscope and closure o the easily recognize intraoperative complications.
submucosal tunnel with endoscopic clips. T ere are Finally, a pure NO ES approach using a exible
no external scars involved in this procedure and it endoscopic approach requires a single operator to
does not require laparoscopic equipment or ports. A manage both the visualization and spatial position-
recent comparative study o POEM and laparoscopic ing o the endoscopic tools concurrently, which can
Heller myotomy demonstrated equal resolution o be cumbersome and is di cult in a NO ES appli-
symptoms in both groups, equal esophageal acid cation. A multitasking NO ES plat orm would be o
exposure in both groups and shorter hospitalization pro ound bene t to the operator.
or the POEM group. T e POEM procedure should
be per ormed in an experienced center by an expert BIBLIOGRAPHY
endoscopist.
Bhayani NH, Kurian AA, Dunst CM, Sharata AM, Rieder E,
T ere are no cases o transvaginal NO ES myot- Swanstrom LL. A comparative study on comprehensive,
omy in the literature. T is would be a risky NO ES objective outcomes o laparoscopic Heller myotomy with
procedure, given the extreme length o instruments per-oral endoscopic myotomy (POEM) or achalasia. Ann
needed to per orm the myotomy with this approach. Surg. 2014;259(6):1098103.
T ere are also no reports o a hand-assisted laparo- Chukwumah C, Zorron R, Marks JM, Ponsky JL. Current
status o natural ori ce translumenal endoscopic surgery
scopic approach.
(NO ES). Curr Probl Surg. 2010;47:63068.
Douard R, Gaudric M, Chaussade S, Couturier D, Houssin
5. A. Although all o the answer choices were initially D, Dousset B. Functional results af er laparoscopic Heller
cited in the original White Paper on NO ES pub- myotomy or achalasia: A comparative study to open sur-
lished by Rattner et al. (Surg Endosc. 2006;20:185), gery. Surgery. 2004;136:1624.
translumenal access (predominantly transvaginal Feinberg EJ, Agaba E, Feinberg ML, Camacho D, Vemulapalli
P. Single-incision laparoscopic cholecystectomy learning
and transgastric in published clinical cases) has curve experience seen in a single institution. Surg Laparosc
developed su ciently over the past decade, includ- Endosc Percutan Tech. 2012;22:1147.
ing primary transvaginal opening techniques and the Gill IS, Advincula AP, Aron M, et al. Consensus statement o
submucosal tunnel technique or transgastric access. the consortium or laparoendoscopic single-site surgery.
ranslumenal closure applications have increased Surg Endosc. 2010;24:7628.
Inoue H, Minami H, Kobayashi Y, et al. Peroral endoscopic
since the publication o the initial NO ES White
myotomy (POEM) or esophageal achalasia. Endoscopy.
Paper, including ull thickness endoscopic clips (e.g., 2010;42:26571.
O SC clip by Ovesco) and endoscopic suturing Johansson M, T une A, Nelvin L, Stiernstam M, Westman
devices (e.g., Overstitch by Apollo Endosurgery). B, Lundell L. Randomized clinical trial o open versus
24 G EN ERAL S U RG ERY EXAM I N ATIO N AN D BO ARD REVI EW

laparoscopic cholecystectomy in the treatment o acute Myotomy (POEM) or Achalasia. Ann Surg. 2014;259:1098
cholecystitis. Br J Surg. 2005;92:449. 103.
Kobayashi M, Mizuno M, Sasaki A, Arisue A, Akiyama S, Pietrabissa A, Sbrana F, Morelli L, et al. Overcoming the chal-
Wakabayashi G. Single-port laparoscopic Heller myotomy lenges o single-incision cholecystectomy with robotic
and Dor undoplication: initial experience with a new single-site technology. Arch Surg. 2012;147:70914.
approach or the treatment o pediatric achalasia. J Pediatr Song , Liao B, Liu J, Yin Y, Luo Q, Cheng N. Single-incision
Surg. 2011;46:22003. versus conventional laparoscopic cholecystectomy: a sys-
Lee SH, Kim SJ, Lee H, et al. Human applications o submu- tematic review o available data. Surg Laparosc Endosc
cosal endoscopy under conscious sedation or pure natural Percutan Tech. 2012;22:e1906.
ori ce transluminal endoscopic surgery. Surg Endosc. 2013; Rattner D. Introduction to NO ES White Paper. Surg Endosc.
27:301620. 2006;20:185.
Lehmann KS, Ritz JP, Wibmer A, et al. T e German registry Soper NJ, Stockmann P , Dunnegan DL, Ashley SW. Lapa-
or natural ori ce translumenal endoscopic surgery: report roscopic cholecystectomy. T e new gold standard? Arch
o the rst 551 patients. Ann Surg. 2010;252:26370. Surg. 1992;127:91721; discussion 213.
Liu L, Chiu PW, Reddy N, et al. Natural ori ce transluminal rastulli S, Cirocchi R, Desiderio J, et al. Systematic review
endoscopic surgery (NO ES) or clinical management o and meta-analysis o randomized clinical trials comparing
intra-abdominal diseases. Dig Endosc. 2013;25:56577. single-incision versus conventional laparoscopic cholecys-
Marks JM, Phillips MS, acchino R, et al. Single-incision lapa- tectomy. Br J Surg. 2013;100:191208.
roscopic cholecystectomy is associated with improved cos- Venneman NG, Besselink MG, Keulemans YC, et al. Ursode-
mesis scoring at the cost o signi cantly higher hernia rates: oxycholic acid exerts no bene cial e ect in patients with
1-year results o a prospective randomized, multicenter, sin- symptomatic gallstones awaiting cholecystectomy. Hepa-
gle-blinded trial o traditional multiport laparoscopic chole- tology. 2006;43:127683.
cystectomy vs single-incision laparoscopic cholecystectomy. Wood SG, Panait L, Du y AJ, Bell RL, Roberts KE. Compli-
J Am Coll Surg. 2013;216:103747; discussion 478. cations o transvaginal natural ori ce transluminal endo-
Milas M, Devedija S, rkulja V. Single incision versus stan- scopic surgery: a series o 102 patients. Ann Surg. 2014;259:
dard multiport laparoscopic cholecystectomy: Up-dated 7449.
systematic review and meta-analysis o randomized trials. Yamada H, Yano . Single incision laparoscopic approach or
Surgeon. 2014;12(5):27189. esophageal achalasia: A case report. Int J Surg Case Rep.
Mo d H, Emmermann A, Alm M, von Walden els HA, Felix- 2013;4:14.
muller C, Zornig C. Is the transvaginal route appropriate Yano F, Omura N, suboi K, et al. Single-incision laparoscopic
or intra-abdominal NO ES procedures? Experience and Heller myotomy and Dor undoplication or achalasia:
ollow-up o 222 cases. Surg Endosc. 2013;27:280712. report o a case. Surg Today. 2012;42:299302.
Montero PN, Acker CE, Heni ord B , Ste anidis D. Single inci- Zorron R, Palanivelu C, Galvao Neto MP, et al. International
sion laparoscopic surgery (SILS) is associated with poorer multicenter trial on clinical natural ori ce surgery
per ormance and increased surgeon workload compared NO ES IM N study: preliminary results o 362 patients.
with standard laparoscopy. Am Surg. 2011;77:737. Surg Innov. 2010;17:14258.
7
Robotic-Assisted Surgery

Brenda Schmidt and Gordon Wisbach

A 65-year-old man with a medical history o hyperten- 2. O the ollowing surgeries, which has been shown
sion presents to the emergency room complaining o to benef t rom the robotic approach?
rectal bleeding or 3 months. On physical exam, a rectal A. Cholecystectomy
mass and scant gross blood is discovered during a digi- B. Nissen undoplication
tal rectal exam. A proctoscopic evaluation localizes the C. Prostatectomy
mass in the anterior rectum only 4 cm rom the dentate D. Adrenalectomy
line. A colonoscopy does not reveal any concomitant E. Pancreatectomy
lesions and a biopsy o the lesion con rms the mass
is adenocarcinoma. On endoscopic ultrasound, the 3. Which o the ollowing patients would most likely
mass extends through the muscularis propria with one benef t rom a robotic-assisted versus laparoscopic
abnormal lymph node. T ere is no involvement with surgery or colorectal cancer?
the sphincter muscles. His workup includes a C o the A. 75 year old admitted with a lower gastrointesti-
abdomen and pelvis, with no signs o metastasis. He nal bleed, ound to have a bleeding cecal tumor
completes neoadjuvant chemotherapy and returns to on colonoscopy
clinic to discuss surgical treatment. Based on the loca- B. 60-year-old man with a low rectal cancer that is
tion o the tumor you recommend a robotic-assisted abutting the prostate on endoscopic ultrasound
laparoscopic anterior resection with a diverting loop C. 64-year-old man with no signi cant medical
ileostomy. history with a descending colon mass
D. 55-year-old man admitted with obstipation
1. Which o the ollowing is true regarding the
ound to have an obstructing sigmoid tumor
di erences between robotic and laparoscopic
resection o rectal cancer? 4. O the ollowing, which is considered to be one o
A. Improved oncologic resection lymph node sam- the advantages o robotic surgery?
pling and margin resections A. Decreased overall cost
B. Decreased incidence o bladder and erectile dys- B. Motion scaling, which increases the light in
unction in men undergoing robotic-assisted di icult to reach places
low anterior resection C. Improved ergonomics or the operating surgeon,
C. T e robotic approach has decreased conversion avoiding atigue and potential disability
rates compared to laparoscopic. D. T e ability to per orm multi-quadrant procedures
D. T e increased cost o the robotic approach is o with ease
set by the signi cantly decreased hospital length E. Endowrist technology, which allows better tac-
o stay and complication rate. tile sensation
26 G EN ERAL S U RG ERY EXAM I N ATIO N AN D BO ARD REVI EW

5. Which o the ollowing is true regarding the pelvis. Despite the ongoing debate, some have shown
training required to per orm robotic-assisted a cost-e ectiveness o robotic-assisted prostatectomy
laparoscopic surgery? in high volume centers i over 150 surgeries are per-
A. As part o the learning curve or robotic-assisted ormed per year.
laparoscopic colorectal surgery, surgical e ciency
3. B. Robotic-assisted laparoscopic surgery is rarely
is best determined by operative time.
applied to emergent operations. As well, a laparo-
B. T e learning curve or robotic-assisted laparo-
scopic approach is rarely pre erred in an urgent
scopic colorectal surgery is well de ned.
colorectal operation including obstipation due to
C. Simulation or robotic surgery is undeveloped,
an obstructing colon cancer. A randomized trial o
and, there ore, not use ul in honing these skills.
robotic-assisted versus laparoscopic colectomy or
D. T ere are no regulated, standardized training
right colon cancer showed hospital stay, surgical
programs or surgeons interested in per orming
complications, postoperative pain score, resection
robotic-assisted laparoscopic surgery.
margin clearance and number o lymph nodes har-
vested were similar in both groups, but the costs asso-
ANSWERS ciated with the robotic-assisted right colectomy were
higher. A nationwide analysis o robotic colorectal
1. C. T e literature that analyzes robotic-assisted low
surgery revealed anterior resections or rectal cancers
anterior and abdominoperineal resections, demon-
were the most common robotic assisted colorectal
strates a lower conversion rate to open laparotomy as
surgery per ormed in the United States, accounting
the most signi cant advantage. A systematic review
or 40% o all robotic cases. A larger percentage o all
and meta-analysis in 2011 suggested that the conver-
rectal cancers are being per ormed robotically com-
sion rate to open surgery in the robotic group was
pared to colon resections, with a tendency toward
signi cantly lower than that with laparoscopic sur-
higher use o robotic-assisted techniques in males
gery. Another meta-analysis recently con rmed these
undergoing anterior resection or rectal cancer. T is
ndings by reporting a trend toward lower conver-
study suggests that robotic assistance may be o no
sion rates in the robotic surgery study arm. However,
added bene t in routine colon resections, but selec-
there was no signi cant advantage o the robotic
tive application in complex rectal cancer procedures
approach in regards to the operative time, blood
may prove to have long-term bene ts. O the choices
loss, length o stay, and complications. T e potential
listed, the patient with the low rectal cancer abutting
advantages o robotic surgery include a more accu-
the prostate is most likely to bene t rom robotic-
rate pelvic dissection and, there ore, a decrease in
assisted surgery.
nerve injury in men resulting in decreased bladder
and erectile dys unction. However, these advantages
have not been well elucidated in the literature. Some 4. C. T ere are many advantages o robotic-assisted
small studies have shown that there may be an earlier laparoscopic surgery including: three-dimensional
return o unction in the robotic group but the inci- high de nition vision; visual magni cation up to
dence o genitourinary dys unction appears to be the 15 times normal; motion scaling that will eliminate
same in either approach. a natural hand tremor as motions are deampli ed
up to a scale o 5 to 1; improved ergonomics or
2. C. Although the types o surgeries per ormed with the operating surgeon; and EndoWrist technology,
robotic-assisted laparoscopy have greatly increased which allows the instrument to imitate normal wrist
over the last several years, there is still debate regard- and elbow motions. However, there are many dis-
ing the bene t o robotic surgery. Although the advan- advantages including: increased operative time and
tages o robotic-assisted laparoscopic prostatectomy cost, the loss o tactile sensation, an evolving learn-
(RALP) are still debated, there is an increasing amount ing curve and di culty when operating in multiple
o literature about this procedure. RALP has become quadrants o the abdomen. T is latter challenge is
one o the most common robotic-assisted surgeries, a particular problem is highlighted during mobili-
which accounted or 67% o all prostatectomies per- zation o the splenic exure in the lef upper quad-
ormed in 2009. T is demonstrates the pre erence o rant during a rectal resection. T is can be solved by
surgeons to use robotics in a small space such as the per orming a hybrid procedure, in which the splenic
C H AP TER 7 RO BO TI C - AS S I S TED S U RG ERY 27

exure is initially mobilized laparoscopically and the health technology assessment rom the perspective o
rest is completed robotically. the UK National Health Service. European Urology. 2013;
64(3):3619.
5. D. Operative time is an inadequate surrogate to DAnnibale A, Pernazza G, Monsellato I, et al. otal mesorec-
tal excision: a comparison o oncological and unctional
determine learning curve or robotic surgery or
outcomes between robotic and laparoscopic surgery or
many reasons; most importantly shorter operative rectal cancer. Surgical Endoscopy. 2013;27(6):188795.
times are not re ected in patient outcomes. A sys- Halabi WJ, Kang CY, Ja ari MD, et al. Robotic-assisted
tematic review o trials examining learning curve in colorectal surgery in the United States: a nationwide anal-
laparoscopic and robotic-assisted colorectal surgery ysis o trends and outcomes. World Journal of Surgery;
suggests that using operative time is too simplistic 2013;37(12):278290.
Kim J, Kim N, Lee K, et al. A comparative study o voiding
and the authors conclude that a shif toward a mul- and sexual unction af er total mesorectal excision with
tidimensional assessment, such as a cumulative sum autonomic nerve preservation or rectal cancer: laparo-
(CUSUM) model, which assesses trends in surgical scopic versus robotic surgery. Annals of Surgical Oncology.
outcomes, should be encouraged. Because learning 2012;19(8):248593.
curve has been studied with multiple ways with di - Kwon Y, Park S. Current choices in robotic surgery: Whether
to increase use. JAMA Surgery. 2014;149(7):627628.
ering numbers o parameters, the learning curve is
Lin S, Jiang H, Chen Z, et al. Meta-analysis o robotic and
not well de ned. Simulation or robotic surgery is laparoscopic surgery or treatment o rectal cancer. World
available and shown to be use ul in attaining com- Journal of Gastroenterology. 2011;17(47):521420.
petency prior to per orming surgeries on patients. Lowrance W, Elkin EB, Jacks LM, et al. Contemporary open
For instance, an inexpensive and synthetic pelvic and robotic radical prostatectomy practice patterns among
training model has been developed to teach the urologists in the United States. T e Journal of Urology.
2012;187(6):208793.
complex skills needed or success ul completion o Marecik SJ, Prasad LM, Park JJ, et al. A li elike patient simula-
robotic rectal dissection. Surgeons are not necessar- tor or teaching robotic colorectal surgery: how to acquire
ily required to complete a competency based training skills or robotic rectal dissection. Surgical Endoscopy. 2008;
program prior to integrating robotic-assisted surgery 22(8):187681.
into their practice. A systematic review has demon- Park JS, Choi GS, Park SY, et al. Randomized clinical trial o
robot-assisted versus standard laparoscopic right colec-
strated that there are many di erent types o training
tomy. British Journal of Surgery. 2012;99(9):121926.
programs described, and provides guidelines or the Parra-Davila E, Ramamoorthy S. Lap colectomy and robotics
development o a structured training program. or colon cancer. Surgical Oncology Clinics of North America.
2013;22(1):14351.
Ramamoorthy S, Obias V. Unique complications o robotic
BIBLIOGRAPHY colorectal surgery. Surgical Clinics of North America. 2013;
Barrie J, Jayne DG, Wright J, et al. Attaining surgical compe- 93(1):27386.
tency and its implications in surgical clinical trial design: Schreuder HW, Wolswijk R, Zweemer RP, et al. raining
A systematic review o the learning curve in laparoscopic and learning robotic surgery, time or a more structured
and robot-assisted laparoscopic colorectal cancer surgery. approach: a systematic review. BJOG: An International Jour-
Annals of Surgical Oncology. 2013;21(3):82940. nal of Obstetrics and Gynaecology. 2012;119(2):13749.
Close A, Robertson C, Rushton S, et al. Comparative cost- rastulli S, Farinella E, Cirocchi R, et al. Robotic resection
e ectiveness o robot-assisted and standard laparoscopic compared with laparoscopic rectal resection or cancer:
prostatectomy as alternatives to open radical prostatectomy systematic review and meta-analysis o short-term out-
or treatment o men with localized prostate cancer: a come. Colorectal Disease. 2012;14.4:e134e156.
8
Advanced LaparoscopySingle
Incision and roubleshooting
Robotic Surgery
Jigesh A. Shah and Omar Yuse Kudsi

You are asked to start a Minimally Invasive Surgical 2. Regarding sa e use o energy devices in laparoscopic
(MIS) program at a new community hospital. T e chie surgery, such as monopolar instruments, which o
o sta has asked you to evaluate all the new minimally the ollowing is true?
invasive techniques and equipment, speci cally with A. T ermal injury cannot occur rom another
regards to patient sa ety, so that the hospital can pur- instrument i the electrically active instrument is
chase the proper equipment or your program. not touching it.
B. Disruptions in the insulation o an active instru-
1. With regards to CO2 insuf ation in laparoscopic
ment may discharge energy to surrounding
surgery, which o the ollowing statements is true?
structures or tissues leading to inadvertent
A. Reports estimate that an average laparoscopic thermal injury.
colectomy requires approximately 200 to 280 C. Using monopolar devices on the so-called coag-
liters o carbon dioxide. ulation mode creates less lateral energy spread
B. Insu ation lters composed o mesh with 0.1- than the so-called cut mode.
to 0.3-micron pores have been developed to D. Capacitive coupling cannot occur when a non-
exclude the possibility o peritoneal contamina- conductor separates two conductors.
tion or disease transmission through the ow o E. I the active electrode o a robotic instrument is
intra-peritoneal uid or particulate matter rom inserted through a non-metal trocar, the non-
the patient to the insu ator and then to the next metal trocar acts can unction as a capacitor,
patient. causing potential injury.
C. Several randomized controlled trials have dem-
onstrated a clinical bene t as well as a decrease 3. With regards to robotic-assisted laparoscopic
in lens ogging with the use o warmed and surgery, which o the ollowing is correct?
humidi ed air. A. T e current robotic surgery system has our
D. ouching the laparoscope tip on viscera is a degrees o reedom with its instruments.
maneuver that is commonly used in laparoscopic B. T e current robotic surgery system has a orded
surgery. However, this should be per ormed cau- the surgeon the ability to per orm laparoscopic
tiously as the laparoscope tip temperature can surgery with ull control o up to two surgical
commonly exceed 200F. instruments and the camera.
E. I the surgeon experiences di culty with pneu- C. Potential advantages o the robotic surgery system
moperitoneum and inadequate relaxation o the include tremor reduction, 3-D visualization, and
abdominal wall, a discussion with the anesthe- increased wrist action at the surgical site com-
sia team regarding consideration or additional pared with traditional laparoscopic instruments.
sedation or paralysis is not appropriate.
C H AP TER 8 Ad v An C Ed LAPARo s Co P y s i n g LE i n C i s i o n An d TRo u b LEs H o o Ti n g Ro b o Ti C s u Rg ERy 29

D. Industry representatives can be present to ensure colectomy requires approximately 110 to 180 L o car-
that the equipment is unctional, and are trained bon dioxide. T e surgeon should be aware o that act,
to in uence surgical decision. making sure to check CO2 pressure in the tank prior
E. I needed, intra-operative cholangiogram is not to start the surgery and clari y that a replacement tank
possible during a robotic-assisted laparoscopic is available, i needed.
cholecystectomy. T e possibility o peritoneal contamination or
disease transmission through the ow o intra-
4. While per orming robotic-assisted or traditional peritoneal uid or particulate matter rom the patient
laparoscopic surgery, which o the ollowing is to the insu ator (and then to the next patient) has
true regarding management o an intra-operative been a concern since the early days o laparoscopy.
complication? o address this concern, insu ation lters composed
A. While per orming a robotic-assisted Nissen un- o mesh with 0.1 to 0.3-micron pores have been
doplication, the surgeon encounters an isolated developed, which limit, but do not exclude the pos-
iatrogenic distal esophageal per oration. T e sibility o disease transmission.
next step would include primary single-layer Warmed, humidi ed gas is sent directly rom the
repair without the planned undoplication. insu ator (or gas is sent rom the insu ator through
B. While per orming a laparoscopic single-site a warming and humidi cation system prior to
Nissen undoplication, the surgeon accidently patient delivery) in order to enter the peritoneal cav-
entered the pleural cavity. T e appropriate next ity at body temperature. In addition to the theoreti-
step would be immediate placement o a chest cal bene t o improved visualization attributable to
tube and discontinuation o the procedure. condensation prevention, there is some evidence that
C. During a robotic-assisted laparoscopic inci- warm, humidi ed gas can also reduce hypothermia
sional hernia repair, the surgeon encounters an and pain, however, several randomized controlled
enterotomy while per orming enterolysis on the trials have ailed to show either a clinical bene t or
anterior abdominal wall. T e next step in man- a decrease in lens ogging with the use o warmed,
agement should include primary repair and humidi ed air.
mesh placement i the spillage is controlled and Wiping the laparoscope tip on the viscera, of en
considered minimal. the liver or small bowel, is another maneuver that is
D. Crepitus observed in the chest or the neck dur- commonly used and is of en e ective. However, this
ing robotic-assisted or laparoscopic Nissen un- should be per ormed with extreme caution as the
doplication is a potential side e ect and will laparoscope tip temperature can of en exceed 200F.
most likely improve without any intervention.
E. Af er completing a di cult single-site laparo- 2. B. Current concepts in electrosurgery are based on
scopic bilateral inguinal hernia repair, the sur- the idea o monopolar and bipolar instrumentation.
geon notices that the insu ation pressure was Monopolar instruments are an active electrode,
set at 40 mm Hg during the case. T e next step to deliver concentrated energy, and require a sec-
includes explaining the technical error to the ond, dispersive electrode, placed on the patient to
patient in the recovery room and scheduling a complete the circuit. Of en, the size o the dispersive
ollow-up in the o ce one week later. electrode is much larger than the active electrode to
decrease the delivered energy and prevent thermal
injury. Bipolar instruments in comparison contain
ANSWERS both the active and dispersive electrodes in a sin-
1. D. o maintain pneumoperitoneum throughout a case, gle hand instrument, thereby delivering concentrated
a signi cant amount o CO2 is requently required. and ocused energy to a particular location. Addition-
T e duties o the surgical team include: monitoring ally, two modes o energy delivery are used in mod-
the intra-abdominal pressure during the case, identi y- ern day electrosurgical units. T e cut mode delivers
ing the amount o CO2 remaining prior to the start o a low-voltage, continuous output, while the coagula-
the surgical procedure and having a replacement tank tion mode delivers a high-voltage, interrupted out-
in reach i needed, especially or long or di cult cases. put. T ese modes re ect the energy wave orms and
One report estimated that an average laparoscopic not their designated labels. For urther in ormation,
30 g En ERAL s u Rg ERy EXAM i n ATi o n An d b o ARd REv i EW

please re er to the SAGES FUSE (Fundamental however they are not trained or credentialed to in u-
Use o Surgical Energy) Handbook. ence any surgical decision making.
Metallic trocars are of en utilized in both laparo-
scopic and robotic surgery or cost reduction with 4. D. Given that sa ely per orming minimally invasive
current robotic surgery instrumentation requiring surgery requires an advanced set o skills, the Ameri-
the use o metal trocars. It is important to under- can Board o Surgery adapted the Fundamentals o
stand the associated possible injuries i these trocars Laparoscopic Surgery (FLS) course prior to being eli-
are placed beyond the suggested ascial point and/ gible sit or board examination. Surgeons per orm-
or come in touch with intra-abdominal organs. With ing minimally invasive surgery should be prepared
direct coupling, the capacitor generates an elec- to deal with potential complications that could arise,
trostatic eld between the active electrode and the as well as be com ortable with the open technique, i
metal trocar, and as electricity passes through the needed.
active electrode, the electrostatic eld can trans er Possible complications o both robotic-assisted
energy to the metal trocar, which will then discharge and laparoscopic Nissen undoplication include
the energy in the orm o heat, leading to burns o entering into the pleural cavity, which is of en man-
the skin and abdominal wall, which is common in aged by decreasing the insu ation pressure and
robotic-assisted surgery per ormed using metal tro- good communication with the anesthesiologist. As
cars. T e use o non-conductive plastic trocars can long as the patient remains stable and there is no
avoid this phenomenon. di culty in ventilation the procedure can continue
T e use o monopolar instruments possess an addi- as planned and post-operative chest X-rays can be
tional risk, known as capacitive coupling, where by obtained to con rm the resolution o CO2. Chest
electrical energy is stored in a capacitor (instrument) tube placement is not immediately required, unless
and then discharged when the circuit is completed. the patient becomes symptomatic.
Of en, injuries o this type occur with accidental Iatrogenic injury to the esophagus is a potential
contact o a recently used instrument, which retains complication during Nissen undoplication and can
stored energy, with nearby tissue causing accidental be avoided by handling the esophagus with umbilical
thermal injuries. T us, whenever monopolar electro- tape (the no-touch technique). It is recommended to
cautery is being utilized, it is important to ensure that repair an iatrogenic injury with a primary, two layers
there is a sa e distance between the active instrument closure with the planned portion o the Nissen wrap
and all other metallic objects. covering the primary repair site. An enterotomy is a
contraindication or synthetic mesh placement irre-
3. C. Robotic surgery has up to 7 degrees o reedom spective to the amount o spillage or contamination
with its instruments and provides the ability to per- and should not be per ormed.
orm laparoscopic surgery with ull control o up to Crepitus is of en seen af er mediastinal dissec-
three di erent surgical instruments as well as the tion and usually resolves spontaneously without any
camera by a single operator. T e potential advantages additional treatment. Monitoring insu ation pres-
o robotic-assisted laparoscopic surgery include sures during surgery is important throughout the
tremor reduction, 3-D visualization, and increased duration o a case since high insu ation pressures
wrist action at the surgical site. can lead to serious complications such as di culty
Intra-operative cholangiogram is possible during with extubation due to CO2 retention and end-organ
robotic cholecystectomy and i needed any o the ischemia ( or example, acute renal ailure). For these
robotic arms can be undocked and moved away or reasons, patients that undergo procedures with high
cholangiography and/or placement o assistant ports. insu ation pressures should be admitted to the hos-
Application o surgical sa ety standards to robotic pital or observation and closely ollowed clinically.
surgery include credentialing, mentorship, knowledge
o laparoscopic physiology, case selection appropri- BIBLIOGRAPHY
ateness to skills level, conversion to an open proce- Brunt LM. Fundamentals o electrosurgery part II: T ermal
dure when needed, and industry representatives can injury mechanisms and prevention. In: Feldman LS,
be present to ensure that the equipment is unctional, Fuchshuber PR, Jones DB, eds. T e SAGES Manual on the
C H AP TER 8 Ad v An C Ed LAPARo s Co P y s i n g LE i n C i s i o n An d TRo u b LEs H o o Ti n g Ro b o Ti C s u Rg ERy 31

Fundamental Use of Surgery Energy (FUSE); New York, NY: or nonmale icence. J Am Coll Surg. 2014; Feb; 218(2):
Springer-Verlag; 2012;6179. 2903.
Diamantis , Kontos M, Arvelakis A, et al. Comparison o Randles VC. T e art and science o monopolar electrosurgery.
monopolar electrocoagulation, bipolar electrocoagulation, In: Feldman LS, Fuchshuber PR, Jones DB, eds. T e SAGES
Ultracision, and Ligasure. Surg oday. 2006;36:90813. Manual on the Fundamental Use of Surgery Energy (FUSE);
Farley DR, Greenlee SM, Larson DR, Harrington JR. Double- New York, NY: Springer-Verlag; 2012;8191.
blind, prospective, randomized study o warmed, humidi- Sammour , Kahokehr A, Hayes J, et al. Warming and humidi-
ed carbon dioxide insu ation vs standard carbon dioxide cation o insu ation carbon dioxide in laparoscopic
or patients undergoing laparoscopic cholecystectomy. colonic surgery: a double-blinded randomized controlled
Arch Surg. 2004;139:73943; discussion 434. trial. Ann Surg. 2010;251:102433.
Funk LM, Greenberg JA. Minimally invasive surgery: Equip- rastulli S, Cirocchi R, Desiderio J, et al. Robotic versus lapa-
ment and troubleshooting. In: Souba WW, Fink MD, roscopic approach in colonic resections or cancer and
Jurkovich GJ, Pearce WH, Pemberton JH, Soper NJ, eds. benign disease: Systematic review and meta-analysis. PLoS
ACS Surgery: Principles and Practice. New York, NY: Marcel One. 2015;10(7):126.
Dekker; 2011;(21):110. yler JA, Fox JP, Desai MM, et al. Outcomes and costs associ-
Larson JA, Johnson MH, Bhayani SB. Application o surgical ated with robotic colectomy in the minimally invasive era.
sa ety standards to robotic surgery: Five principles o ethics Dis Colon Rectum. 2013; Apr; 56(4):45866.
This page intentionally left blank
Et h ic a l a nd Leg a l Issues
Robert B. Lim
This page intentionally left blank
9
Ethics/Legal Issues
Decision Making
S v A z r S w

A 37-year-old man and his 16-year-old daughter are B. Her ather still maintains legal right over her
brought into the emergency department via ambulance because she is not o legal age. Make sure he is clear
a er being involved in an automobile collision. T e man is o any added risks associated with her surgery i
ully conscious and alert. He in orms the hospital sta that he re uses any blood products or his daughter.
both he and his daughter are Jehovahs Witnesses. Radio- C. She is old enough to be considered a mature
logical studies indicate that the man su ered a broken pel- minor. T ere ore her decision to re use blood
vis and his daughter is experiencing internal bleeding. It is products must be respected.
concluded that both individuals will need surgery. D. Call risk management.

1. L b work m ow mo ob 3. T m m o p b oo
m o r o r by p m pro .W po b o q or
r r or b oo r o .H m o wor o o ?
o r v y b oo pro b o A. He would be ostracized by his mother.
r o .W x p yo m o? B. Be required to pay a $25,000 donation to expunge
A. Respect his wishes and proceed with bloodless his transgression.
surgery protocols. C. Lose the privilege o attending church services until
B. Begin searching or a nearby bloodless surgery church elders deemed him su ciently repentant.
center that is better equipped to handle his D. All o the above
situation.
C. Discuss his belie . ANSWERS
D. Call the Hospital Liaison to determine what are
your legal responsibilities. 1. C. alk to the patient privately in order to get a better
understanding o his personal belie s. T e Jehovahs
2. T m r o o o Witnesses (JW) religion is known or its re usal to
o o . A o po o v rb y allow blood trans usions. T e current JW position
o f rm r r o b oo pro . How v r includes unconditional re usal o whole blood, packed
r f b b oo r r red blood cells, white blood cells, platelets, and plasma.
r po k . W ppropr r po ? T is includes the autologous trans usion o their own
A. Use whatever means are necessary during sur- pre-deposited blood. However, many Witnesses accept
gery. T e patient is a legal minor and thus you intra-operative blood salvage so long as their blood
have the right under doctrine o parens patriae remains in circulation within the body. T ey may
to do so. also accept albumin, immune globulin, hemophilia
36 G EN ERAL S U RG ERY EXAM I N ATI O N AN D BO ARD REVI EW

preparations, sera, and even organ transplantation on blood products. T ese proceedings would ultimately
an individual decision. It is there ore vital that each yield an involuntary con ession rom the patient
case be discussed and treated individually. and result in his dis ellowshipping. Witnesses, which
Witnesses cannot be treated appropriately without included amily and riends, would then be instructed
exploring personal conviction and pre erence. It is impor- to ostracize and shun the expelled individuals. T is
tant that the physician discuss these matters in private. practice is used by the JW to serve as a deterrent or
T e patients autonomy to make medical decisions anyone acting against its teachings. Dissidents within
can be greatly in uenced by amily members, riends, the JW have voiced that the practice coerces and com-
and congregational members. Case reports have even promises autonomous decisions in medical care.
shown that patients have changed their earlier decision In June 2000, the church released a statement indi-
to accept blood treatment a er being visited by hospital cating that there would be a procedural change in
liaison committee members. T ese members are o en which the congregation would no longer initiate the
called upon by doctors in order to help determine indi- action to revoke membership or a baptized member
vidual treatment. Due to their potential in uence, this who wil ully and without regret accepts blood trans-
practice may have to be reconsidered. usions. It would be le up to the individual to come
orth and divulge his own actions to the church. T e
2. D. Contact risk management i unsure. It is impor- person would then be labelled as disassociated
tant or medical pro essionals to be amiliar with because he had will ully broken a core tenet o their
their particular states laws regarding minors con- aith. T is too would subject the individual to being
sent authority. Most states require parental consent ostracized. T e important di erence between these
and do not permit mature minors to consent to gen- two processes is that a JW member could now keep
eral medical care. However, parental rights are not quiet about receiving blood and avoid excommuni-
absolute. T ey have a responsibility to make deci- cation altogether. Maintaining patient conf dentiality
sions that ensure the health and sa ety o their child. is there ore paramount.
Courts can exercise their power under the doctrine
o parens patriae to orce medical treatment when BIBLIOGRAPHY
parents put their childs health in jeopardy. T e case Coleman DL, Roso PM. T e legal authority o mature
o Prince v Massachusetts is o en cited because it minors to consent to general medical treatment. Pediatrics.
clearly mentions that parents have every right to die 2013;131(4):78693. doi:10.1542/peds.20122470.
or their religious belie but cannot make that same Mann MC, Votto J, Kambe J, McNamee MJ. Management o
decision or their children. the severely anemic patient who re uses trans usion: les-
sons learned during the care o a Jehovahs Witness. Annals
3. A. He would be ostracized by his mother. Muramoto o internal medicine. 1992;117:10428.
Muramoto O. Bioethics o the re usal o blood by Jehovahs
has sited Church literature that appears to teach that Witnesses: Part 1. Should bioethical deliberation consider
JWs who come across conf dential medical in orma- dissidents views? J Med Ethics. 1998; 24(4): 22330. Avail-
tion are obligated to bring that in ormation to the able rom: http://www.pubmedcentral.nih.gov/articleren-
attention o the church elders. Not only could there der. cgi?artid= 1377670&tool= pmcentrez&rendertype=
be legal ramif cations or any healthcare worker abstract. Accessed May 26, 2014.
Muramoto O. Bioethical aspects o the recent changes in the
caught doing so, but there could be a resulting coer-
policy o re usal o blood by Jehovahs witnesses. BMJ. 2001;
cive e ect in a JW patients decision making process. 322(7277):379. Available rom: http://www.pubmed-
A breach in conf dentiality could a ect a church central.nih.gov/articlerender. cgi?artid= 1119307&tool=
member in other serious ways. pmcentrez&rendertype = abstract. Accessed May 26, 2014.
In 1961 the JW began to en orce the policy o blood Muramoto O. Bioethics o the re usal o blood by Jehovahs
re usal by dis ellowshipping, or expelling, un-repentant Witnesses: Part 3. A proposal for a dont-ask-dont-tell policy.
J Med Ethics. 1999;25:4638. doi:10.1136/jme.25.6.463.
members who will ully accept blood components that Woolley S. Children o Jehovahs Witnesses and adolescent
are prohibited. T e Church conducted judicial proceed- Jehovahs Witnesses: What are their rights? Arch Dis Child.
ings whenever members were suspected o receiving 2005;90:7159. doi:10.1136/adc.2004.067843.
10
End o Li e Care

Jenny Lam and Shawn Tsuda

An 85-year-old emale presents to the hospital with 2. In regards to advance directives, which statement
nausea, vomiting, abdominal pain, and distension or is correct?
the past two days. T e pain rapidly worsens over the A. Advanced directives are not trans erable between
next hour and the patient is taken to the OR with the states.
diagnosis o mesenteric ischemia. T e patient has a past B. Oral advance directives are legally valid.
medical history o coronary artery disease, COPD, and C. A living will takes precedence over appointed
osteoporosis. She currently lives in a nursing home and surrogate decision making.
is able to per orm activities o daily living with assis- D. Advance directives are always ollowed verbatim.
tance. Be ore taking the patient to the OR, you con rm
that the patient has no living will or durable power o 3. In regards to life-sustaining medical treatment,
attorney, but does state that she wouldnt want to be a which statement is correct?
vegetable on a machine. During the surgery the patient A. T ere is no ethical di erence between withdraw-
is ound to have extensive necrotic bowel rom the ing and withholding treatments.
ligament o reitz to the transverse colon. T e patient B. Risk management personnel must be consulted
is closed without resection. A er surgery the patient be ore li e-sustaining medical treatment may be
rapidly deteriorates and develops severe septic shock. terminated.
A discussion occurs with the patients amily about the C. A patient has to be terminally ill or li e support
poor prognosis o this illness. T e patients daughter, to be stopped.
whom is the next o kin, states that she wants everything D. It is permissive to withhold extraordinary care,
to be done or her mother and threatens legal action i but not ordinary care.
her wishes are not granted. E. It is illegal to provide palliative care to a termi-
nally ill patient i there is a possibility that this
1. What is the next appropriate step in managing this
may hasten the patients death.
patient?
A. ake the patient back to the OR to per orm a
subtotal colectomy as per the daughters wishes. ANSWERS
B. In orm the daughter that she has no decision 1. E. When making decisions about end o li e care it
making capabilities and proceed with palliative is important or all involved parties to have a gen-
care. eral consensus about care. Emotions and psychologi-
C. Remove the patient rom nutritional support cal strain can heavily in uence decision making, but
and hydration. it is important to remind involved parties that the
D. Consult the hospitals ethics committee. most important decision making actor is what the
E. Continue to discuss options with the amily. patient would have wanted. I the patient is no longer
38 G EN ERAL S U RG ERY EXAM I N ATIO N AN D BO ARD REVI EW

competent to make decisions, it is reasonable to Risk management personnel are consulted to


ascertain what they would have wanted rom previ- limit legal risk in regards to patient care. Some hos-
ous comments about similar situations. Advanced pital policies may require a consult be ore terminat-
directives, living wills, or appointed surrogates ing li e-sustaining treatment, but there is no law that
are commonly used to make decisions but in their indicates this as a prerequisite.
absence, in ormal advanced directives or next o kin T ere is a very ne line that marks the distinction
are implicated. In this case the patient has stated she between extraordinary care and ordinary care.
would not want any heroic li e-saving measures to Extraordinary care or heroic measures are ones that
be taken. Oral advance directives are legally valid, take over normal physiologic unction o the body in
and should be documented in the patients medical end-organ ailure. Examples would include the use o
records. T e next o kin has rights to make decisions ventilators or pulmonary ailure or hemodialysis or
about patient care as long as these decisions are con- kidney ailure. Ordinary care helps the body sustain
sistent with the patients autonomous wishes. Further itsel , with treatment such as IV uids, tube eeds, or
discussion with the amily may make this clear and antibiotics. In a Supreme Court ruling in 1983, it was
avoid unwanted court intervention. ound that any treatment could be declined under
the Fourteenth Amendment o the U.S. Constitution,
2. B. Advance directives provide in ormation about a be it extraordinary or ordinary, as long as this is in
patients wishes regarding their healthcare when they the best interests in the patient and the burden o
are no longer competent to make decisions. Many medical care outweighs the bene ts.
advance directives contain provisions allowing the T e principle o double e ect states that it is ethi-
directive to be en orceable in whichever state they cal to initiate a treatment that is intended to bene t
are currently residing in. Even without such a provi- a patient even i the there is an unintended negative
sion oral in ormal advance directives made about consequence. T e bene t must be suf ciently substan-
treatment pre erences are legally valid and are upheld tial to outweigh the risk. T e act must also directly
in court as a sworn testimony about statements the cause the bene t and not be intrinsically harm ul. An
patient made prior to their illness. Living wills o en example would be giving a patient high dose opiates
lack the speci c details to direct care in un oreseen to relieve pain, but then unintentionally hastening the
circumstances. A surrogate can be appointed to make patients demise by way o respiratory depression.
real-time decisions on behal o the patient remov-
ing the responsibility o a provider to attempt inter- BIBLIOGRAPHY
pretation a patients wishes. When a durable power Annas GJ: Nancy Cruzan and the right to die. N Engl J Med.
o attorney is appointed they become an extension 1990;323(10):6703.
Emanuel EJ. Palliative and End-o -Li e Care. In: Longo DL,
o the patient and may make decisions that con ict Fauci AS, Kasper DL, et al. Harrisons Principles of Inter-
with a written advance directive, so long as they nal Medicine. Eighteenth ed. New York, NY: McGraw-
promote the best interest o the patient. Regardless Hill; 2012. Available rom http://www.accessmedicine
o the preparation a patient takes in preparing an [accessmedicine].com
advanced directive, their wishes cannot always be Hall DE, Angelos P, Dunn GP, et al. Ethics, Palliative Care, and
Care at the End o Li e. In: Brunicardi F, Andersen DK,
carried out. A recent study showed that a large por-
Billiar R, et al. Schwartzs Principles of Surgery. Ninth ed.
tion o patients that requested aggressive care did not New York: McGraw-Hill; 2010. Available rom http://www.
receive it as either such care was not an option or the accessmedicine[accessmedicine].com
patient had appointed a surrogate that overrode the Lo B. Surrogate Decision Making. In Lo B. Resolving Ethical
previous pre erence. Dilemmas: A Guide For Clinicians. Fourth ed. Philadelphia,
Pennsylvania: Lippincott Williams & Wilkins: 2009;1016.
Meisel A, Synder L, Quill . Seven legal barriers to end-o -
3. A. T ere is no di erence between withdrawing and li e care: Myths, realities, and grains o truth. JAMA. 2000;
withholding care, such that one action is not more 284(19):2495501.
ethically sound then the other. It is not more uneth- Pawlik M: Withholding and withdrawing li e-sustaining
ical to stop li e sustaining therapy versus never hav- treatment: A surgeons perspective. J Am Coll Surg. 2006;
202(6):9904.
ing started in the rst place. However, physicians Silveira M, Kim SYH, Langa KM: Advance directives and out-
are less likely to withdraw care once it has been ini- comes o surrogate decision making be ore death. N Engl J
tiated. Med. 2010;362(13):121118.
Skin a nd So f t Tissue
Richard Smith
This page intentionally left blank
11
Skin Cancer

Jigesh A. Shah and Omar Yuse Kudsi

1. A 70-year-old male presents to your o ce with a D. Excisional biopsy and a total parotidectomy
concerning 2 cm painless lump on his le hip that E. Complete surgical excision with a 0.5 cm nega-
he noticed over the last year, which is f rm and tive margin
dome-shaped. An excisional biopsy is per ormed,
demonstrating a Merkel cell carcinoma (MCC) 3. An 86-year-old emale presents to your o ce
with negative margins. Regarding MCC, which o with a new 1.8-cm raised pearly nodule with
the ollowing is true? sur ace telangiectasias on her cheek. Which o
the ollowing would be the best option in her
A. T e Merkel cell polyomavirus likely contributes
management?
to the development o most Merkel cell carci-
noma. A. Surgical excision with 1 cm margins
B. A sentinel lymph node biopsy is required only B. Cryotherapy
or tumors > 2 cm in size. C. Mohs microsurgery
C. Risk actors or Merkel cell carcinoma include D. Radiation therapy
having dark skin, a weakened immune system, E. opical imiquimod
and overexposure to UV radiation.
4. A 61-year-old male presents to your o ce with
D. Merkel cell dense-core granules stain positively
a lesion on his lower lip. Biopsy conf rms a
or the neuroendocrine marker neuron speci c
squamous cell carcinoma, 2 cm in diameter. In
enolase.
regards to squamous cell carcinoma (SCC), which
E. Current recommendations or tumor excision
o the ollowing is true?
are with a 0.5-cm margin or tumors < 2 cm in
size and 2-cm margins or those > 2 cm in size. A. Squamous cell carcinoma o the lip most o en
presents on the upper lip.
2. A 50-year-old air skinned, blue-eyed male presents B. Squamous cell carcinoma is the second most
to your o ce with a skin lesion on the pinna o common cutaneous cancer in patients who have
his right ear concerning or a melanoma. Biopsy had a kidney transplant.
determines a superf cial spreading melanoma with C. Actinic keratosis is not a risk actor or the devel-
a depth o 1.1 mm. What is the next step in the opment o squamous cell carcinoma.
management o this patient? D. Lip de ects involving at least one-third o the lip
A. Wide local excision and sentinel lymph node require regional aps such as an Abbe ap.
biopsy E. Margins or low risk squamous cell carcinoma
B. High dose inter eron alpha 2b is recommended range rom 1.0 to 2.0 cm.
C. Radiation therapy
42 G EN ERAL S U RG ERY EXAM I N ATI O N AN D BO ARD REVI EW

5. A 79-year-old male presents with 2-cm scaly pink basin is recommended. In cases where SLN positivity
lesion on his scalp that his primary care physician is ound on immunostaining but not H&E staining
has been ollowing or the last two years. Biopsy o the lymph node, radiotherapy without complete
conf rms the diagnosis o a basal cell carcinoma. lymph node dissection has been suggested as sole
Which o the ollowing is correct in regards to regional therapy. Dark skin is not a risk actor, white
basal cell carcinoma? skin is. T e other two are risk actors.
A. Basal cell carcinoma is the second most common
2. A. umor thickness is critical or establishing the
orm o skin cancer, a er squamous cell carci-
prognosis in melanoma and regional metastases indi-
noma.
cates poor prognosis. Frozen sections have no role in
B. Super cial basal cell carcinomas are scaly, pink
the diagnosis or treatment o melanoma. Once the
to red lesions requently con used with psoriasis
dermis in invaded the probability o regional or dis-
or other eczematous dermatoses.
tant metastases increases substantially. Lymphoscin-
C. Basal cell carcinomas commonly develop in
tigraphy and sentinel lymph node biopsy became the
burn scars or chronic in ammatory wounds.
primary method o identi ying nodal drainage pat-
D. Surgical margins or low risk basal cell carcinoma
terns replacing the prior suggested nodal drainage
range rom 0.5 to 1.0 cm.
based on location. ragus and anterior pinna lesions
E. Mohs microsurgery cannot be considered an
were thought to metastasize to the parotid gland and
option in the treatment o basal cell carcinomas.
anterior cervical lymph nodes, whereas posterior
pinna lesions were thought to spread to the mastoid
ANSWERS bone and occipital and posterior cervical nodes. Com-
1. A. T e mainstay o therapy or patients newly diag- plete surgical excision with 1 to 2 cm margin is the
nosed with primary MCC remains surgical. Current treatment o choice. Elective neck dissection is generally
recommendations are based on the clinical size o not recommended or lesions less than 1 mm in thick-
the primary tumor and call or tumor excision with ness, whereas lymphadenectomy may of er survival
1 cm margins or tumors that are 2 cm in size and advantage and better local control or lesions > 1 mm
2 cm margins or those that are greater than 2 cm in depth and positive sentinel lymph node biopsy (see
in size. Radiotherapy has been used as monother- able 11-1). Inter eron alpha-2b is recommended or
apy or primary tumors with reported success, but patients with lymph node positive disease. Dif erent
until more data become available, surgery remains types o melanoma are described in able 11-2.
the mainstay o therapy or primary MCC tumors. Table 11-1 EXCISION MARGIN OF MELANOMAS
Furthermore, Feng et al. characterized a novel poly-
omavirus, the MCPyV, and suggested an association Depth o Suggestion
between it and the pathogenesis o MCC. T is work T Stage Invasion (mm) Margin (cm)
has sparked great interest in MCC and has opened a 1 <1 1
new pathway in the study o viral tumorigenesis. 2 12 1
T e role o chemotherapy in the treatment o
3 2.014 2
MCC remains unclear. Since nearly one-third o
clinically node-negative patients harbor microscopic 4 >4 2
nodal disease, sentinel lymph node biopsy (SLN) is
currently recommended or Merkel cell carcinoma at 3. C. Basal cell carcinoma could be treated either surgi-
the time o wide local excision. SLN biopsy has been cal or medically. reatment options include wide local
shown to be important in the staging and progno- excision, Mohs microsurgery with 4 mm margins,
sis o MCC, and SLN status is included in the most curettage, and cryosurgery. T e standard treatment
recent American Joint Committee on Cancer (AJCC) or larger basal cell carcinoma is surgical excision with
staging guidelines. SLN biopsies should be examined cure rates greater than 99% or primary lesions; how-
by both hematoxyin and eosin (H&E) and immu- ever or small lesions or lesions in areas such as the
noperoxidase staining, including CK20. I sentinel ace, Mohs microsurgery is the pre erred treatment
nodes are positive, completion lymph node dissec- option. Non-surgical treatments, typically reserved
tion o the nodal basin ollowed by radiotherapy o the or non-surgical candidates include radiotherapy,
C H AP TER 1 1 S k I N C AN C ER 43

Table 11-2 TYPES OF MELANOMA Basal cell carcinoma o en presents as pearly nod-
Type Characteristics
ules with telangiectases and may bleed occasionally.
A skin biopsy to establish a diagnosis is important
Super cial Most common, associated with be ore treatment o any skin cancer. Margins or
spreading congenital nevi/dysplastic nevi. low risk squamous cell carcinoma range rom 0.5 to
Prominent radial growth
1.0 cm. Margins or low risk basal cell carcinoma
Nodular Prominent vertical growth, dark range rom 0.3 to 0.5 cm. Additionally, Mohs micro-
pigmentation
surgery is considered an option in non-melanoma
Lentigo maligna Arise in sun exposed areas, occur at skin cancers. Bottom line, squamous cell carcino-
dermalepidermal junction mas most o en arise in chronically damaged skin or
Acral lentiginous Least common, arise on palmar, within actinic keratosis, in burn scars, and chronic
plantar, mucosal, and subungual in ammatory wounds.
regions. Common in dark-skinned
and Asian patients
BIBLIOGRAPHY
Allen PJ, Bowne WB, Jaques DP, et al: Merkel cell carcinoma:
topical and photodydnamic therapy. opical therapy
Prognosis and treatment o patients rom a single institu-
includes 5- uorouracil and imiquimod. Imiquimod, tion. J Clin Oncol. 2005;23:23009.
a nonspeci c immune response modi er, has been Campbell JP. Surgicla management o lip carcinoma. J Oral
approved or the treatment o super cial basal cell Maxillofac Surg. 1998;56:955.
carcinoma smaller than 2 cm or ve times per week Cole MD, Jakowatz J, Evans GR. Evaluation o nodal patterns
or the duration o 6 weeks with clearance rate more or melanoma o the ear. Plast Reconstr Surg 2003;112:506.
Diepgen L, Mahler V. T e epidemiology o skin cancer. Br J
than 80%. Dermatol. 2002;146:16.
Feng H, Shuda M, Chang Y, Moore PS. Clonal integration o
4. D. In immunosuppressed transplants patients, SCC a polyomavirus in human Merkel cell carcinoma. Science.
is the most common skin cancer and it tends to have 2008. 319:1096100.
a more aggressive behavior. Lip carcinoma is the Funk GF, HyndsKArnell L, Robinson RA, et al. Presentation,
most common oral cavity cancer, with a majority o treatment, and outcome o oral cavity cancer: a National
these lesions occuring on the lower lip. A majority o Cancer Data Base Report. Head Neck. 2002;24:165.
Jemal A1, Siegel R, Ward E, Hao Y, Xu J, Murray , et al. Cancer
patients with oral cavity carcinomas have a history
statistics, 2008. CA Cancer J Clin. 2008;58:7196.
o either excessive alcohol intake or tobacco use. Lip Odgson NC: Merkel cell carcinoma: Changing incidence
carcinoma most likely presents as an exophytic mass trends. J Surg Oncol. 2005;89:14.
and diagnosis is obtained by biopsy. Risk actors or Pockaj BA, Jaroszewski DR, DiCaudo DJ, et al. Changing sur-
SCC include actinic keratosis, burn wound scars and gical therapy or melanoma o the external ear. Ann Surg
chronic in ammatory wounds. Large de ects that Oncol. 2003;10:68996.
Rubin AI, Chen EH, Ratner D: Basal cell carcinoma. N Eng J
involve up to two thirds o the lip require local aps Med. 2005;353:22629.
such as Abbe or Estlander. Usam KJ, Jungbluth AA, Rekthman N, et al. Merkel cell poly-
omavirus expression in Merkel cell carcinomas and its
5. B. Basal cell carcinoma is the most common orm absence in combined tumors and pulmonary neuroendo-
o skin cancer ollowed by squamous cell carcinoma. crine carcinomas. Am J Surg Pathol. 2009;3:137885.
12
Decubitus Ulcers

Pamela C. Masella and Mark K. Markarian

You are consulted on a 35 year-old male, who other- A. Chronic wounds


wise healthy, was initially admitted 26 days ago in B. Diabetic wounds
septic shock secondary to a necrotizing so tissue C. Wound location
in ection o the right lower extremity. He required D. Meshed skin gra s
high-dose vasopressors or several days ollowing E. Wounds that require hemostasis
emergent operative debridement and institution o
broad-spectrum antibiotics. He was recovering well 3. Which o the ollowing patients would be the best
in the ICU and was recently extubated. However he candidate or ap reconstruction?
developed tachycardia, ever, and leukocytosis. T e A. A 48-year-old male with Stage I sacral ulcer and
ICU team calls you to evaluate because they are unable has no medical comorbidities
to identi y a source o in ection. You examine his back B. A 34 year-old male recovering rom brain injury
and nd a pressure sore overlying the right ischial with a Stage IV ischial ulcer that recently under-
tuberosity approximately 4 5 cm in size with a black went treatment or osteomyelitis
eschar overlying, expressible malodorous uid, and C. A 74-year-old homeless male with Stage III
surrounding cellulitis. sacral ulcer
D. A 48-year-old poorly controlled diabetic male
1. What is the stage o this pressure ulcer? with Stage III pressure sore overlying the right
A. Stage I greater trochanter
B. Stage II E. A 56-year-old male with Stage III chronic sacral
C. Stage III ulcer who re uses to stop smoking despite hospi-
D. Stage IV talization
E. Unstageable
4. Multiple options or treatment o pressure sores
2. Surgery is per ormed and when the black eschar exist. Which o the ollowing options should be
is removed, purulent uid is expressed. A er avoided?
extensive operative debridement to healthy tissue, A. Primary closure
you f nd that the wound extends to bone. Which B. Local wound care
o the ollowing would preclude use o Negative C. Skin gra s
Pressure Wound T erapy (NPW )? D. Musculocutaneous reconstruction
E. Fasciocutaneous reconstruction
C H AP TER 1 2 D EC u b i Tu s u l C ERs 45

ANSWERS tissue ormation by acilitating cell migration


and proli eration
1. E. Accurate staging o pressure wounds is important Maintenance o wound homeostasis: the semi-
because it guides management. Staging is as ollows: occlusive dressing and oam with insulation qualities
Stage I: Non-blanchable erythema o intact skin; minimizes evaporation, desiccation, and heat loss
impending skin ulceration. T e ollowing are common indications or nega-
Stage II: Partial-thickness skin loss involving epider- tive pressure wound therapy:
mis and/or dermis; ulcer is super cial and presents
clinically as an abrasion, blister, or shallow crater. Chronic, diabetic wounds or pressure ulcers
Stage III: Full-thickness skin loss involving dam- Meshed gra s (be ore and a er)
age or necrosis o subcutaneous tissue that may Flaps
extend down to, but not through, underlying as- Chronic and acute wounds
cia; ulcer presents clinically as a deep crater with Subacute wounds (dehisced incisions)
or without undermining o adjacent tissue. T e ollowing are contraindications to negative
Stage IV: Full-thickness skin loss with extensive pressure wound therapy:
destruction, tissue necrosis, or damage to muscle,
bone, or supporting structures. Fistulae to organs/body cavities
Necrotic tissue that has not been debrided or
Unstageable/Unclassi ed: T ere are also unstage- eschar
able pressure sores that are commonly incorrectly Untreated osteomyelitis
staged. T ese are pressure sores with slough/eschar Wounds that require hemostasis
than need to be debrided be ore one can see how Placing dressing on exposed blood vessels (includ-
deep they truly are, and can then be accurately staged. ing anastomotic sites) or organs
Suspected Deep issue Injury: Purple or maroon Wound malignancy
localized area o discolored intact skin or blood-
lled blister due to damage o underlying so tissue Caring or the pressure sore patients involves
rom pressure and/or shear. more than addressing the wound. Wound healing in
chronic wounds requires a systemic strategy, includ-
2. E. Negative pressure wound therapy (NPW ) is ing nutritional assessment and maintenance, control
becoming a valuable resource used by surgeons to o both systemic and local in ection, avoidance o
manage di cult wounds. NPW devices consist o excessive moisture/incontinence, pressure and mus-
an adhesive semi-occlusive dressing, tubing con- cle spasm relie , surgical debridement and wound
nected to a collection canister and a vacuum source, closure.
and an inter ace material to distribute the vacuum
(open-pore polyurethane hydrophobic oam). 3. B. Most commonly, Stage III/IV pressure sores are
Negative pressure on the sealed, airtight wound re erred or so tissue reconstruction. Un ortunately
results in: some o these patients are not suitable candidates or
medical or social reasons. Since these surgeries are
Increased blood ow to the wound requently raught with complications, there are multi-
Removal o excess uid that may retard cell ple patient characteristics to optimize be ore coverage
growth and proli eration can be considered:
Micro- and macro-de ormation o the wound:
Macrode ormation is the visible stretch that Nutritional status
occurs when the sponge contracts. It serves to Control o medical comorbidities
draw the wound edges together, provide direct Presence o muscle spasticity (Can this be con-
and complete wound bed contact, distribute trolled with anti-spasmodics?)
negative pressure, remove exudate and in ec- obacco dependence
tious materials. Social situation (assess or presence o a responsi-
Microde ormation occurs at the cellular level ble caretaker at homeand subsequent appropri-
and leads to cell stretch. It reduces edema, pro- ate residence at own home vs acility), appropriate
motes per usion, and promotes granulation specialty mattress at residence, strict regimen o
46 G EN ERAl s u RG ERY EXAM i N ATi O N AN D b O ARD REVi EW

requent turning to prevent ap necrosis/ ailure, and primary closure leads to tension, scarring over
appropriately padded wheelchair the original bony prominence, and dehiscence. Skin
I osteomyelitis is present, then the patient likely gra ing has a limited success rate, as gra ing tends to
needs bony debridement and tailored IV antibiotic provide unstable coverage. Musculocutaneous aps
therapy (typically or 6 weeks) be ore so tissue provide adequate blood supply, bulky padding, and
reconstruction can be attempted. are ef ective in treating in ected wounds. Fasciocuta-
Consideration o adverse drug actors like use o neous aps of er an adequate blood supply, durable
steroids or immunosuppressants coverage, and low rates o unctional de ormity.
Control o dif erent causes o maceration like
ecal or urine incontinence BIBLIOGRAPHY
Medical non-compliance Bauer JD, Mancoll JS, Phillips LG. Pressure Sores. In: T ome,
CH. Grabb and Smiths Plastic Surgery. Fi h ed. Philadel-
Once these medical/social issues are addressed, phia: Lippincott Williams & Wilkins, 2007;72229.
the wound can then be optimized with thorough European Pressure Ulcer Advisory Panel and National Pres-
debridement and dressing care, in preparation or sure Ulcer Advisory Panel. Prevention and treatment o
ap reconstruction. pressure ulcers: quick re erence guide. Washington DC:
National Pressure Ulcer Advisory Panel; 2009.
4. A. When planning therapeutic treatment o pres- KCI. T e V.A.C. therapy clinical guidelines: a re erence source
or clinicians [report on the Internet]. 2005. Kinetic Con-
sure sores, the choice o closure strategy depends cepts, Inc. 7-1-2006.
not only on the location, size, and depth o the ulcer, Ohjimi H1, Ogata K, Setsu Y, et al. Modi cation o the glu-
but also on the previous management strategies teus maximus V-Y advancement ap or sacral ulcers: the
employed. Primary closure should be avoided. T ese gluteal asciocutaneous ap method. Plast Reconstr Surg.
wounds tend to have an absence o adequate tissue 1996;98(7):124752.
13
Melanoma

Amber E. Ritenour and Lauren Greer

A 50-year-old Caucasian emale was re erred to general 4. Regarding sentinel lymph node biopsy in patients
surgery by her primary care provider or evaluation o a with melanoma, which of the following is FALSE?
mole to her le medial cal . On examination o her le A. Completion lymph node dissection is recom-
medial cal , she has a 9 mm diameter pigmented lesion mended to achieve regional control in melanoma
with irregular borders and color variation. Excisional patients with positive sentinel lymph nodes.
biopsy is per ormed. B. Sentinel lymph node biopsy is indicated or inter-
mediate thickness melanomas (Breslow thickness,
1. Pathology will most likely show which histologic
1 to 4 mm) regardless o ulceration or mitotic rate.
subtype of melanoma?
C. Sentinel lymph node biopsy is indicated or stag-
A. Acral lentiginous ing in patients with thick melanomas ( 4; Breslow
B. Desmoplastic thickness > 4 mm).
C. Lentigo maligna D. In patients with clinically localized melanoma,
D. Nodular sentinel node status is the most important prog-
E. Super cial spreading nostic indicator.
E. Sentinel lymph node biopsy is mandatory in all
2. If her melanoma is 2.2 mm thick, the surgical skin
patients with thin melanomas (< 1 mm) i there
margin for a wide local excision of the primary
are high-risk eatures.
lesion should be at least
A. 0.5 cm 5. Regarding metastatic melanoma, which of the
B. 1 cm following is FALSE?
C. 2 cm A. Surgery or patients with Stage IV melanoma
D. 3 cm con ers a survival advantage over systemic medi-
E. 4 cm cal therapy alone.
B. T e primary objective o metastatectomy or Stage
3. Which of the following is a poor prognostic
IV melanoma is resection o all known disease.
indicator in melanoma patients?
C. Recurrent Stage IV disease is not a contraindica-
A. Extremity location tions to surgery or metastatic melanoma.
B. Low Breslow thickness D. In-transit metastases occur less than 2 cm rom
C. Female gender the primary melanoma and are located between
D. Elevated LDH the primary melanoma and the regional lymph
E. Younger age node basin.
E. More than 50% o patients with Stage IV mela-
noma will develop brain metastases.
48 G EN ERAL S U RG ERY EXAM I N ATI O N AN D BO ARD REVI EW

ANSWERS thin melanomas (6%, Breslow thickness 1 mm).


At all thickness levels, patients with ulcerated mel-
1. E. Seventy- ve percent o all malignant melanomas anomas were ound to have survival curves that
are super cial spreading melanomas. Most arise de were similar to patients with melanomas o the next
novo but they may be associated with a pre-existing higher Breslow thickness. T e investigators ound
nevus. T ey grow radially be ore growing vertically. a signi cant step-wise decrease in survival based
ypical locations are in sun-exposed areas, namely on increasing age. Patients with higher number o
the back in men and legs in women. Nodular mela- nodal metastases and patients with macrometastases
nomas comprise 15% to 30% o melanomas and are (clinically palpable nodes) also had poorer survival.
o en dome-shaped and dark. T ey quickly develop Seetharmu et al. (Oncology. 2011;81(56):4039)
a vertical growth phase. Lentigo maligna melanoma ound that elevated pre-treatment LDH was corre-
typically develops as a brown macule in sun dam- lated with poorer 2 year survival in Stage IV mela-
aged skin o older individuals and may grow radi- noma patients. umor mitotic rate has also been
ally or years be ore vertical growth develops. Acral determined to be a signi cant prognostic indicator.
lentiginous melanoma is the rarest melanoma in
caucasians but is the most common type in Asians 4. E. Metastasis to regional lymph nodes is the most
and dark-skinned people. T ey are aggressive and important prognostic actor in patients with early
commonly arise on palmar, plantar, subungual, and stage melanoma. T e American Society o Clini-
mucosal sur aces. Desmoplastic melanoma is a rare cal Oncology (ASCO), Society o Surgical Oncology
variant that may be mistaken or a scar, broma other Joint Clinical Practice Guideline, and T e National
benign lesion and should be re erred to an experi- Comprehensive Cancer Network (NCCN) Guidelines
enced dematopathologist or evaluation. Version 4 (2014) recommend routine sentinel lymph
node biopsy in patients with patients with intermedi-
2. C. Wide local excision is standard treatment or ate thickness melanomas (Breslow thickness 14 mm).
melanoma. Excision should be carried through skin Although there is less data to support use o senti-
and subcutaneous tissue down to muscle ascia. Cur- nel lymph node biopsy in thick melanomas (Breslow
rent recommendation is or 0.5 to 1 cm margins thickness > 4 mm), sentinel node biopsy is recom-
or melanoma in situ, 1cm margins or melanomas mended to acilitate accurate staging. ASCO guide-
1 mm thick, 1 to 2 cm or melanomas 1.01 to lines state that there is insu cient evidence to support
2 mm thick, and 2 cm margins or melanomas > 2 mm. routine sentinel node biopsy in thin melanomas (Bres-
A randomized multicenter trial demonstrated no low thickness < 1 mm), but it may be considered in
bene t to 4 cm versus 2 cm margins in patients with patients with high risk eatures. NCCN guidelines state
melanomas > 2 mm thick. Other trials have corrobo- that sentinel lymph node biopsy is not generally indi-
rated these results. cated in patients with Breslow thickness 0.75 mm.
However, consideration should be given to sentinel
3. D. Evidence based prognostic indicators were care-
lymph node biopsy in patients with Breslow thickness
ully considered and integrated into the current
0.761 mm, especially i ulceration or mitotic rate
AJCC staging system to provide staging that re ects
1 per mm is present. Completion lymph node dis-
disease biology. In 2001, Balch et al. published a
section remains the standard recommendation or
prospective study o 17,600 melanoma patients to
patients with tumor positive sentinel nodes. Current
determine actors that were predictive o melanoma-
evidence suggests that completion lymph node dissec-
speci c survival. T e investigators determined that
tion is ef ective at achieving local control. T e ongo-
patients with melanomas o the head, neck, or trunk
ing Multicenter Selective Lymphadenectomy rial II
had a signi cantly worse survival rate than patients
(MSL II) is investigating whether completion lymph
with melanomas o the extremities. Males had a
node dissection also improves survival. Current AJCC
poorer prognosis than emales. Increasing Breslow
staging re ned N categories to account or the prog-
thickness was predictive o lower survival.
nostic dif erence between patients with clinically
Presence o ulceration was more requent among
palpable (macrometastasis) and clinically negative but
patients with thick melanomas (63% ulceration,
pathologically positive sentinel nodes (micrometastasis).
Breslow thickness < 4 mm) than among patients with
C H AP TER 1 3 M ELAN O MA 49

5. D. In appropriately selected patients with Stage IV Balch CM, Urist MM, Karakousis CP, Smith J, emple WJ,
melanoma, surgery should be considered as ini- Drzewiecki K, et al. E cacy o 2-cm surgical margins or
tial therapy. Analysis o MSL -1 data demonstrated intermediate-thickness melanomas (1 to 4 mm). Results o
a multi-institutional randomized surgical trial. Ann Surg.
improved survival in patients with Stage IV mela- September 1993;218(3):2627; discussion 2679.
noma who underwent surgery with or without sys- Balch CM, Soong SJ, Gershenwald JE, T ompson JF, Reintgen
temic medical therapy compared with systemic DS, Cascinelli N, et al. Desmoplastic malignant melanoma
medical therapy alone. Multiple sites o disease and (a rare variant o spindle cell melanoma). Cancer. 1971;
even recurrent Stage IV disease are not contraindica- 28(4):914.
Desmond R, Zhang Y, Liu PY, Lyman GH, Morabito A. Prog-
tions to resection. T e most important preoperative
nostic actors analysis o 17,600 melanoma patients:
considerations include resectability o all known dis- Validation o the American Joint Committee on Cancer
ease, tumor biology such as tumor volume doubling melanoma staging system. Journal of Clinical Oncology.
time, and patient comorbidities. T e most common August 2001;19(16):362234.
metastatic sites or melanoma are skin, lung, lymph Dzwierzynski WW. Managing malignant melanoma. Plastic and
nodes, brain, liver, and gastrointestinal tract. Reconstructive Surgery. September 2013;132(3):446e60e.
Gillgren P1, Drzewiecki K , Niin M, Gullestad HP, Hellborg
In-transit metastasis is de ned as intra-lymphatic H, Mnsson-Brahme E, et al. wo cm versus our cm sur-
tumor in skin or subcutaneous tissue more than 2 cm gical excision margins or primary cutaneous melanoma
rom the primary tumor but not beyond the near- thicker than two mm: A randomised, multicentre trial.
est regional lymph node basin. T e lung is a typical Lancet. November 2011;378(9803):1626.
site o metastasis and good evidence or pulmonary Howard JH, T ompson JF, Mozzillo N, Nieweg OE, Hoekstra
HJ, Roses DF, et al. Metastasectomy or distant metastatic
metastatectomy exists. Melanoma is the most com-
melanoma: Analysis o data rom the rst Multicenter Selec-
mon metastatic tumor in the small bowel and may tive Lymphadenectomy rial (MSL -I). Annals of Surgical
present as pain, obstruction, bleeding, palpable mass Oncology. August 2012;19(8):254755.
or weight loss. More than 50% o patients with Stage Leung AM, Hari DM, Morton DL. Surgery or distant mela-
IV melanoma will develop brain metastasis. Because noma metastasis. Cancer J. 2012;18:17684.
o this, NCCN Version 4 (2014) Melanoma recom- McGovern VJ, Mihm MC Jr, Bailly C, Booth JC, Clark WH
Jr, Cochran AJ, et al. T e classi cation o malignant mela-
mends consideration o annual brain MRI in addition noma and its histologic reporting. Cancer. 1973;32(6):
to more requent whole body PE /C as a part o 1446.
routine surveillance in patients with Stage IV disease. National Comprehensive Cancer Network (NCCN) Clinical
Practice Guidelines in Oncology: Melanoma,Version 4 Web
BIBLIOGRAPHY site. Accessed rom http://www.nccn.org/pro essionals/
Azzola MF, Shaw HM, T ompson JF, Soong SJ, Scolyer RA, physician_gls/ _guidelines.asp. Published 2014.
Watson GF, et al. umor mitotic rate is a more power ul Seetharamu N, u J, Christos P, Ott PA, Berman RS, Shap-
prognostic indicator than ulceration in patients with pri- iro RL, et al. Enrollment in clinical trials correlates with
mary cutaneous melanoma: an analysis o 3661 patients improved survival in metastatic melanoma. Oncology. 2011;
rom a single center. Cancer. March 2003;97(6):148898. 81(56):4039.
Balch CM, Soong SJ, Smith , Ross MI, Urist MM, Karakousis Wong SL, Balch CM, Hurley P, Agarwala SS, Akhurst J,
CP, et al. Long-term results o a prospective surgical trial Cochran A, et al. Sentinel lymph node biopsy or mela-
comparing 2 cm versus 4 cm excision margins or 740 noma: American Society o Clinical Oncology and Society
patients with 1 to 4 mm melanomas. Ann Surg Oncol. o Surgical Oncology joint clinical practice guideline. J Clin
March 2001;8(2):1018. Oncol. Aug 2012;30(23):29128.
14
Sarcoma and Lymphoma

Joshua S. Ritenour

SCENARIO 1 3. Regarding neoadjuvant and adjuvant therapies or


A 47-year-old emale presents with a several months sof tissue sarcoma, which o the ollowing is true?
history o a slowly growing painless mass in her right A. Chemotherapy is the most important therapeutic
thigh. On physical exam, she is noted to have a 5 cm intervention or outcomes rom sarcoma.
rm, non-tender mass in the right lateral mid-thigh. B. Metastatectomy has no role in the management
o sarcomas.
1. Which o the ollowing is true? C. Radiation therapy has been shown to be o
A. Routine imaging o the mass should include a bene t in the treatment o sarcomas.
PE scan. D. Local recurrence is rare a er appropriate therapy.
B. Incisional biopsy should be routinely per ormed E. Gastrointestinal stromal tumors (GIS ) are
in order to con rm the diagnosis and identi y treated similarly to other sarcomas.
the histological subtype.
C. T e most common histologic subtypes or SCENARIO 2
extremity sarcomas include liposarcoma and A 26-year-old male presents with a painless mass in his le
malignant brous histiocytoma. axilla that has been present or several months. Upon ques-
D. Staging includes an abdominal and pelvic C tioning, he endorses unexplained low-grade evers, night
scan to evaluate or metastatic disease in this sweats, and an unplanned weight loss o 10 lbs over the
lesion. previous 3 months. On physical exam, he is noted to have
E. Multidisciplinary evaluation is not needed or a 2 cm rm, rubbery, non-tender mass in the le axilla.
simple, early-stage sarcomas.
4. Which o the ollowing considerations is true or this
2. Regarding surgical management o this patient, patient?
which o the ollowing is true? A. Observation is warranted.
A. A unctional outcome is the highest priority. B. History o ever, chills, night sweats, and weight
B. Surgical margin o 1 to 2 cm should be sought loss are considered A level symptoms.
whenever possible. C. Physical examination should include thorough
C. Regional lymph node dissection is also required. exam o all accessible lymph node basins as well
D. Amputation should be strongly considered as the as potential sites o a primary malignancy.
primary therapy or most extremity sarcomas. D. Sexually transmitted diseases can be ruled out by
E. Histologic subtype has little impact on surgical physical exam in younger patients.
planning.
C H AP TER 1 4 S ARCo m A An d Ly m P H o m A 51

E. Fine-needle aspiration (FNA) biopsy o suspicious T ere are over 50 histologic subtypes o so tis-
lymph nodes in this patient is usually adequate or sue sarcoma recognized. Overall, the most common
diagnosis. subtypes are liposarcoma, malignant brous histio-
cytoma (MFH), and leiomyosarcoma, however the
5. Regarding lymphoma, which o the ollowing is prevalence is site speci c. T e most common types
true? or extremity sarcomas are liposarcoma and MFH,
A. Surgeons play a major role in the treatment o while retroperitoneal sarcomas are more commonly
lymphoma. liposarcoma and leiomyosarcoma. GIS and leiomyo-
B. T e hallmark o treatment is surgery and chemo- sarcoma are the histological types ound in visceral
therapy. tumors.
C. Staging or Hodgkins lymphoma is based upon Overall, lung is the most common site o metasta-
a di erent staging system than is Non-Hodgkins sis or extremity sarcomas. For visceral tumors, liver
lymphoma. is more common. Chest C is generally advocated
D. HIV in ection has been shown to increase the or extremity sarcomas to rule out metastatic disease.
incidence o lymphoma but other immunosup- Due to the complexity, rarity, and histology-
pressive states have not. based therapy or so tissue sarcomas, prior to ini-
E. Mucosa-associated lymphoid tissue (MAL ) tiating therapy, all patients should be evaluated by
lymphoma may be de nitively treated with H. a multidisciplinary team with expertise in sarcoma
pylori eradication in its early stages. therapy.

2. B. Until the early 1980s, amputation remained a pri-


ANSWERS mary therapy or most extremity sarcomas. In 1982,
1. C. Regarding pre-operative imaging o the tumor, Rosenberg and colleagues at the National Cancer
MRI is generally pre erred or extremity sarcomas, Institute published a randomized trial o amputation
while C tends to be pre erable or abdominal and vs. limb-sparing surgery plus radiation and demon-
retroperitoneal sarcomas. However, there has not strated equivalent overall survival and acceptable
been demonstrated a statistically signi cant di er- local recurrence rate o 15% with limb-salvage versus
ence between the two modalities. High quality cross 0% with amputation. Currently, limb salvage surgery
sectional imaging is nevertheless critical or pre- can be sa ely per ormed in over 90% o patients with
operative evaluation and planning. PE can give extremity sarcoma with excellent local recurrence
in ormation regarding the grade, prognostication, rates. Proximity o critical structures (bone, nerves,
and response to chemotherapy in select high grade, blood vessels, etc.) should be considered care ully
large, deep sarcomas, and can be considered or use, during pre-operative planning and the expected
but does not have a role in routine evaluation o all unctional outcome assessed to assist in making the
sarcomas. decision or limb salvage versus amputation.
Incisional biopsy may be required, but gener- Care ul pre-operative planning is imperative
ally speaking, or both extremity and retroperito- or success ul resection o these tumors. Failure to
neal sarcomas, needle core biopsy (with or without obtain a negative surgical margin is the most impor-
image guidance) provides adequate tissue sampling tant risk actor or local recurrence. Care ul scrutiny
with good diagnostic correlation to nal pathology. o pre-operative imaging can result in improved out-
However, when unavailable or inadequate, incisional comes by allowing the surgeon to have a thorough
biopsy or extremity sarcomas remains a reasonable understanding o the extent o the tumor and rela-
diagnostic option. Care should be taken i an incisional tionship to local structures. Functional outcomes are
biopsy is necessary. Incisions should be oriented lon- also improved by care ul consideration o the extent
gitudinally and thought ully, as re-excision o biopsy o resection required. Similarly, this allows or appro-
site will be necessary with de nitive operation. In priate consultation with subspecialists including
some selected institutions with clinical and patho- plastic or vascular surgeons pre-operatively when
logic expertise, FNA may be adequate, but should anticipated being necessary.
probably not be considered adequate at low-volume Wide margins should be the goal o therapy. A
centers. margin o 1 to 2 cm o normal tissue can help to
52 G En ERAL S U RG ERy EXAm In ATI o n An d Bo ARd REVIEW

minimize the risk o local recurrence. However, brachytherapy, and intraoperative radiotherapy have
strong tissues such as ascia can severely limit the all been used with some success. Patients at low risk
spread o most types o sarcoma into adjacent struc- or recurrence (i.e., small (< 5 cm) super cial tumors,
tures. T ere ore, a narrow ascial margin may be low grade tumors, wide surgical margins) may not
acceptable where such a narrow margin o muscle or derive a signi cant bene t rom radiation and may be
at would likely result in an increased risk o recur- treated with surgery alone.
rent disease. In some cases, such as retroperitoneal Surgery is the dominant therapeutic modality or
sarcomas, wide margins may not be easible. extremity, retroperitoneal, and visceral sarcomas.
Resection o nerves and blood vessels is occasion- Complete surgical resection is the primary actor
ally unavoidable, but can o en be avoided by care ul in outcomes. Only a ew clinical trials have dem-
skeletonization o blood vessels and resection o the onstrated a statistically signi cant improvement
perineurium along with the tumor. T e concept o in outcomes with adjuvant chemotherapy. Speci c
planned positive margin is easible in many cases histology-directed regimens have shown the most
with the use o adjuvant radiation therapy. As long promise and have achieved excellent and sustained
as the structure o concern is not xed to the tumor, results in speci c subtypes to include gastrointestinal
this care ully planned positive margin provides simi- stromal tumors (GIS ), rhabdomyosarcoma, Ewing
lar outcomes to controls with negative margins. Neo- sarcoma, and osteosarcoma.
adjuvant radiation has been advocated in this setting Patients with an isolated metastasis can be consid-
or marginally resectable tumors in order to allow a ered or metastatectomy i resection with or without
negative margin unctional resection. chemotherapy or radiation may result in a cure. T is
Histologic subtype does play an important actor may include removal o limited disease in a single
in planning surgical intervention. Well-di erentiated organ or regional node dissection i nodal metasta-
liposarcoma ( ormerly called atypical lipomas) have sis is isolated. Speci cally in the setting o isolated
similar recurrence rate to other types o sarcoma, but lung metastasis, median survival is lengthened rom
metastatic disease is rare, so surgical resection can 11 to 33 months compared to observation. Patients
be less aggressive when necessary. Dermato brosar- with widely metastatic disease may be considered
coma protuberans (DFSP) and myxo brosarcoma or palliation using a variety o modalities including
are particularly di cult due to the common nding surgery, chemotherapy, radiation, embolization, and
o microscopic tentacles that extend laterally rom ablation procedures.
these lesions. DFSP has a tendency to respect ascial Local recurrence rate varies with the site o the
borders, whereas myxo brosarcoma o en penetrates initial tumor and the adequacy o surgical resection,
ascia and can have multi ocal skip lesions, so these but is signi cant. Local recurrence a er treatment
actors must be considered. Lymph node metastasis or extremity so tissue sarcoma approaches one out
is rare in S S in general, but can be seen more com- o three patients. T e median disease- ree interval
monly with epithelioid or clear cell variants. For this is 18 months, but can be quite remote. Considera-
reason, sentinel lymph node biopsy can give prognos- tions or treatment o recurrent disease are similar to
tic in ormation, so though no clear therapeutic bene t primary disease and can include reexcision, chemo-
has been shown, it may be considered in this setting. therapy, and radiation, o en with similar success to the
primary tumors or extremity sarcomas. Recurrence o
3. C. Radiation therapy has been repeatedly demon- visceral and retroperitoneal sarcomas o en is unable
strated to be o bene t in disease ree survival, though to be completely re-excised.
not signi cantly di erent overall survival. T ere GIS is the most common mesenchymal tumor
remains some controversy regarding the optimal tim- o the GI tract, and has gained signi cant inter-
ing o the radiation therapy. When comparing neo- est in recent years. T e discovery o the KI proto-
adjuvant versus adjuvant therapy, local control rate oncogene mutation present in the majority o GIS
is similar. However, preoperative radiation has been tumors led to the ability to speci cally target GIS
ound to double wound complications in the months tumors at a molecular level with the tyrosine kinase
ollowing surgery compared to increased rates o long inhibitor imatinib mesylate (Gleevec; Novartis
term complications such as brosis, edema, and joint Pharmaceutical, Basel, Switzerland). T is advance-
sti ness with postoperative radiation. External beam, ment marks a new era in treating solid tumors with
C H AP TER 1 4 S ARCo m A An d Ly m P H o m A 53

speci c molecular targeting. Surgery remains o non-speci c laboratory tests ref ecting inf ammation
vital importance, but several studies have demon- to include ESR, CRP, and brinogen may be used, but
strated improved disease- ree and overall survival do not typically help narrow the di erential diagno-
or patients at signi cant risk or recurrence who are sis. Lactate dehydrogenase (LDH) is o en similarly
treated adjuvantly with tyrosine kinase inhibitors, so non-speci c though very high levels may suggest
this has become an important, disease-speci c therapy a lymphoid neoplasm. Ultimately, persistent lym-
or these rare tumors. phadenopathy without an obvious etiology warrants
lymph node biopsy as discussed above.
4. C. In patients without a tissue diagnosis o malig- Fever, chills, night sweats, and weight loss > 10%
nancy, but concern or lymphoma, FNA is generally over 6 months are the classically described B symp-
not considered to be adequate or diagnosis. FNA toms associated with lymphoma and the presence o
may be able to identi y some cases o metastatic these ndings should increase the suspicion o this
carcinoma (i.e., breast, lung, etc.) but is limited in diagnosis and prompt a more aggressive approach to
hematologic malignancies (especially lymphoma) as diagnosis.
lymph node architecture is a critical pathologic com- Care ul physical exam should be an important part
ponent o making these diagnoses. FNA may be use- o the evaluation o every patient with lymphadenop-
ul in searching or recurrent disease. As knowledge athy. T e di erential diagnosis is highly dependent
and use o molecular markers is expanded in the upon the presence o generalized versus isolated lym-
uture, FNA may become a viable alternative to open phadenopathy and the lymph node basin involved
biopsy o suspicious nodes. When lymphadenopathy when localized. Examining sites o possible meta-
is generalized the largest, most suspicious, and acces- static spread, or example breast, skin, oropharynx,
sible note is selected or biopsy. T e diagnostic yield and so on is important in narrowing the di erential.
does vary by site with inguinal nodes having the low-
est yield and supraclavicular nodes the highest. Care- 5. E. Lymphoma re ers to a spectrum o diseases
ul and appropriate handling o tissue specimens is including Hodgkins and Non-Hodgkins (NHL) lym-
critical to allow pathologic diagnosis. Adequate tissue phomas with multiple subtypes within each group.
sampling is critical to allow assessment o nodal archi- T e role o the surgeon in patients with lymphoma
tecture. Specimens should be submitted to pathology is limited. By ar, the most important role o the sur-
dry or in saline, not ormalin or other preserva- geon is in assisting with the diagnosis by means o
tive, in order to allow f ow cytometric and immu- lymph node biopsy. Once the diagnosis has been
nohistochemical studies to supplement traditional made, the treatment o lymphoma is overwhelmingly
pathology. the role o chemotherapy and radiation. Historically,
T e di erential diagnosis or lymphadenopathy surgery was more involved through the use o stag-
is extremely broad and includes malignancy (lym- ing laparotomy, but this is now rarely indicated with
phoma, metastatic disease), in ections (cat-scratch the availability and use o PE /C or staging pur-
disease, HIV, mononucleosis, tuberculosis, etc.), auto- poses. Surgical intervention to include splenectomy
immune disorders, iatrogenic causes such as medica- is limited to signi cantly symptomatic situations
tions and unusual causes including sarcoidosis and such as anemia, thrombocytopenia, neutropenia and
Kawasakis disease. A thorough history and physical massive splenomegaly, and should be done only in
can be used to help narrow this broad di erential. In close consultation with oncology.
patients without concerning signs or symptoms sug- Staging or lymphoma is based upon modi ca-
gestive o malignancy or other severe disease (e.g., tions o the original Ann Arbor classi cation, ini-
B symptoms, etc.) a period o observation o our tially described or Hodgkins lymphoma, but now
to six weeks is appropriate. I the lymphadenopathy also used or NHL. Staging is based upon number
persists beyond this period o observation, urther and location o lymph node groups, extra-nodal
evaluation to include CBC and chest X-ray should be involvement, and has modi ers to signi y presence
pursued. Other testing to include serology or CMV, or absence o B symptoms.
heterophile testing (monospot), PPD, HIV testing, Several risk actors or lymphoma have been
RPR, and so on may be utilized based upon the his- identi ed. T ere has been a signi cant increase in
tory and physical and speci c concerns. Several incidence since the 1970s, which has been partially
54 G En ERAL S U RG ERy EXAm In ATIo n An d Bo ARd REVI EW

explained by the HIV epidemic. Other contributors Kingham P, DeMatteo RP. Multidisciplinary treatment o
to the increase include other in ections (i.e. Helico- gastrointestinal stromal tumors. Surg Clin North Am. 2009;
bacter pylori-induced mucosal associated lymphoid 89(1):21733.
Larrier NA, Kirsch DG, Riedel RF, et al. Practical radiation
tissue (MAL ) lymphoma), autoimmune diseases, oncology or extremity sarcomas. Surg Oncol Clin N Am.
immunosuppression (as with organ transplant), envi- 2013;22(3):43343.
ronmental actors including pesticides, and aging o Motyckova G, Steensma DP. Why does my patient have
the population (NHL increases in incidence with age lymphadenopathy or splenomegaly? Hematol Oncol Clin
and peaks during the h through seventh decades). North Am. 2012;26(2):395408.
National Comprehensive Cancer Network (NCCN) Clinical
Hodgkins lymphoma is more common in higher
Practice Guidelines in Oncology: So tissue sarcoma. https://
socioeconomic groups and less common in Asian www.nccn.org/pro essionals/physician_gls/pd /sarcoma.
populations. pd . Accessed May 15, 2014.
MAL lymphoma has been shown to be depend- National Comprehensive Cancer Network (NCCN) Clinical
ent on Helicobacter pylori in ection. Gastric inf am- Practice Guidelines in Oncology: Hodgkin Lymphoma.
matory response to this in ection stimulates the https://www.nccn.org/pro essionals/physician_gls/pd /
hodgkins.pd . Accessed May 15, 2014.
acquisition o genetic abnormalities and malignant National Comprehensive Cancer Network (NCCN) Clinical
trans ormation o B cells. In the early stages o this Practice Guidelines in Oncology: Non-Hodgkins Lympho-
disease, H. pylori eradication can reverse the disease mas.https://www.nccn.org/pro essionals/physician_gls/pd /
process in as many o 77% o patients. However, in nhl.pd . Accessed May 15, 2014.
later stages o the disease, urther genetic injury to Nayak LM, Deschler DG. Lymphomas. Otolaryngol Clin N Am.
2003;36(4):62546.
these malignant cells make the tumor resistant to
OSullivan B, Davis AM, urcotte R, et al. Preoperative versus
bacterial eradication, so more aggressive therapy postoperative radiotherapy in so -tissue sarcoma o the
including chemotherapy and radiation is indicated. limbs: a randomized trial. Lancet. 2002;359(9325):223541.
Like other orms o lymphoma, surgery is generally Panicek DM, Gatsonis C, Rosenthal DI, et al. C and MR
reserved or rare situations with complication such imaging in the local staging o primary malignant muscu-
as li e-threatening hemorrhage. loskeletal neoplasms: Report o the Radiology Diagnostic
Oncology Group. Radiology. 1997;202(1):23746.
Ravi V, Patel S. Adjuvant Chemotherapy or So issue Sarco-
BIBLIOGRAPHY mas. Surg Oncol Clin N Am. 2012;21(2):24353.
Bazemore AW, Smucker DR. Lymphadenopathy and malig- Rosenberg SA, epper J, Glaststein E, et al. T e treatment o
nancy. Am Fam Physician. 2002;66(11):210310. so -tissue sarcomas o the extremities: prospective ran-
Billingsley KG, Burt ME, Jara E, et al. Pulmonary metasta- domized evaluations o (1) limb-sparing surgery plus radi-
ses rom so tissue sarcoma: analysis o patterns o dis- ation therapy compared with amputation and (2) the role
eases and post-metastasis survival. Ann Surg. 1999;229(5): o adjuvant chemotherapy. Ann Surg. 1982;196(3):30515.
60210. Schuetze SM, Rubin BP, Vernon C, et al. Use o positron
Du MQ, Isaccson PG. Gastric MAL lymphoma: rom aetiol- emission tomography in localized extremity so tissue
ogy to treatment. Lancet Oncol. 2002;3(2):97104. sarcoma treated with neoadjuvant chemotherapy. Cancer.
Ferrone ML, Raut CP. Modern surgical therapy: Limb salvage 2005;103(2):33948.
and the role o amputation or extremity so -tissue sarco- Singer S. So issue Sarcomas. In: ownsend DM, Beau-
mas. Surg Oncol Clin N Am. 2012;21(2):20113. champ RD, Evers BM, et al. eds. Sabiston extbook o Sur-
Fong Y, Coit DG, Woodru JM, Brennan MF. Lymph node gery. Eighteenth ed. Pliladelphia, PA: Saunders Elsevier:
metastasis rom so tissue sarcoma in adults. Analysis o 2008:78699.
data rom a prospective database o 1772 sarcoma patients. aghizadeh M, Muscarella P. Splenectomy or hematologic
Ann Surg. 1993;217(1):727. disorders. In: Cameron JL, Cameron AM. Current Surgi-
Heslin MJ, Lewis JJ, Woodru JM, Brennan MF. Core needle cal T erapy. enth ed. Philadelphia, PA: Elsevier Saunders:
biopsy or diagnosis o extremity so tissue sarcoma. Ann 2011:4739.
Surg Oncol. 1997;4(5):42531. wist CJ, Link MP. Assessment o lymphadenopathy in children.
Hueman M , T ornton K, Herman J, Ahuja N. Management Pediatr Clin N Am. 2002;49(5):100925.
o extremity so tissue sarcomas. Surg Clin N Am. 2008; Wong SL. Sarcomas o so tissue and bone. In: Mulholland
88(3):53957. MW, Lillemoe KD, Doherty GM, et al. Green elds Surgery:
Kaushal A, Citrin D. T e role o radiation therapy in the manage- Scientif c Principles and Practice. Fi h ed. Philadelphia, PA:
ment o sarcomas. Surg Clin North Am. 2008;88(3):62946. Lippincott Williams & Wilkins: 2010.
15
T oracic Surgery
Lung CancerCough
Ashra A. Sabe

A 77-year-old male who is a ormer smoker (quit 10 A. Superior pulmonary vein/Upper lobe bronchus/
years ago; smoked 1 pack per day or 50 years) with runcus anterior
COPD, type 2 diabetes mellitus, and CAD status post B. runcus anterior/Superior pulmonary vein/
CABG 3 10 years ago, presents with a 4 month history Upper lobe bronchus
o a non-productive cough and 10 pound weight loss C. Superior pulmonary vein/ runcus anterior/
in the past 3 months. A CXR and C chest demonstrate Upper lobe bronchus
a 2 3.1 cm mass in his right upper lung that was not D. Upper lobe bronchus/ runcus anterior/Superior
present on a CXR taken 5 years earlier. pulmonary vein
E. runcus anterior/Upper lobe bronchus/Superior
1. What is the best appropriate next step in the pulmonary vein
management o this patient?
A. Positron emission tomography (PE ) scan 4. A er resection o his tumor by a right upper
B. Cervical mediastinoscopy lobectomy, pathology demonstrated squamous cell
C. Right upper lobectomy carcinoma with negative margins. Lymph nodes
D. Right upper lobectomy with mediastinal lymph sampled were also negative or malignancy. Which
node sampling o the ollowing is true?
E. Bone scan and abdominal C scan A. T e patient will not need any urther surveillance.
B. T e only surveillance needed will be a repeat C
2. Assuming this patient was diagnosed with a primary chest in 2 years.
non-small cell lung cancer (NSCLC) in the right C. T e most common site o relapse or NSCLC is
upper lobe o his lung, which o the ollowing is an bone.
absolute contraindication to surgical resection? D. T e patient will require radiation therapy or
A. A post-operative predicted FEV1 o 45% 6 months post-operatively.
B. A malignant pleural e usion in the right chest E. Most recurrences occur within the rst two years
C. umor directly invading the chest wall a er surgery.
D. umor directly invading the right main bronchus
0.5 cm away rom the carina 5. I another patient presented with signif cant
E. umor directly invading the mediastinal pleura hyponatremia and seizures and was incidentally
ound to have similar sized lung mass in the right
3. When per orming a right upper lobectomy, a er upper lobe, the patient would most likely benef t
the pleura overlying the anterior hilum are divided rom which o the ollowing treatment modalities?
rom anterior to posterior, what is the order o A. Surgical resection o the tumor alone
structures that would need to be divided? B. Single agent chemotherapy
56 G EN ERAL S U RG ERY EXAM I N ATI O N AN D BO ARD REVI EW

C. Radiation therapy alone the preoperative FEV1. Conversely, i the lung to be


D. Combination chemotherapy alone resected happens to be unctional, but the patient has
E. Combination chemotherapy and radiation therapy other areas o lung that are non- unctional, than the
FEV1 will actually decrease more dramatically when
ANSWERS this unctional lung is resected.

1. A. PE scanning uses uorodeoxyglucose (FDG) 3. C. Structures that would be need to be divided or a


with radiolabeled uorine to help detect malignant right upper lobectomy rom anterior to posterior are
cells. Malignant cells metabolize glucose at a higher the superior pulmonary vein, truncus anterior, and
rate than normal cells, resulting in a radiolabeled upper lobe bronchus. T is is a simpli ed description
metabolite o FDG, which results in a visual marker o the most commonly encountered anatomy or the
to help identi y malignant cells, nodal stage, and main structures seen. During dissection the surgeon
metastases. Surgical resection without any attempt must be mind ul o aberrant anatomy (o en noted
at preoperative staging is not appropriate. Although on preoperative imaging) and more commonly
cervical mediastinoscopy may be an important step branching vessels, which may appear in di erent
in assessing the stage o this patients lung cancer, planes than listed above.
PE scanning may identi y suspicious nodes and
distant metastasis and helps guide the surgeon on 4. E. NSCLC is most likely to relapse within the rst two
where to obtain the appropriate tissue or diagnosis. years a er resection (over 60%). T us, oncologic sur-
veillance should be done every 3 to 4 months during
2. B. Surgery is contraindicated or Stage IIIB and IV this time. T ough there is no single recommendation
lung cancer. 4 lesions are classi ed as stage IIIB i or surveillance beyond 2 years, coordinated oncologic
they are associated with N2 disease. umors o any size surveillance should be per ormed long-term. T ough
are 4 i they invade the heart, great vessels, trachea, relapse o NSCLC is common in the bone, the most
esophagus, vertebral body, or carina. umors associ- common site o relapse or all stages o NSCLC is the
ated with malignant pleural or pericardial e usions brain. Other sites o relapse include the lung (ipsilat-
are considered to have M1A disease, which precludes eral or contralateral), liver, and adrenal glands. T ere is
surgical intervention. 3 tumors include tumors o no indication or radiation therapy post-operatively.
any size that directly invade any o the ollowing: dia-
phragm, chest wall, mediastinal pleura, parietal peri- 5. E. Paraneoplastic syndromes develop in as many
cardium, or a tumor invading the main bronchus less as 40% o patients with newly diagnosed lung can-
than 2 cm rom the carina (without involvement o the cers. Signi cant hyponatremia in the setting o a
carina). Depending on nodal and metastatic involve- newly diagnosed lung mass is highly suspicious o
ment, 3 tumors may not result in a staging level o syndrome o inappropriate antidiuretic hormone
IIIB or IV. T us, surgery may be appropriate in select (SIADH). It is important to recognize that SIADH is
patients presenting with 3 tumors. most commonly associated with small cell lung can-
Preoperative testing o pulmonary unctional cer (SCLC). In addition to hyponatremia, the diag-
reserve is essential prior to determining whether a nosis o SIADH is con rmed by demonstrating a
patient is a candidate or surgical resection. In gen- serum osmolarity < 275 mOsm/kg, urine osmolarity
eral, a predicted post-operative FEV1 less than 40% is > serum osmolarity, and urine sodium > 25 meq/L.
thought to be a contraindication to surgical resection. Associated symptoms may include lethargy, nausea,
Although a post-operative predicted FEV1 o 45% is vomiting, altered mental status, and seizures.
not an absolute contraindication to surgical resec- SCLC account or 20% o all lung cancers. T ese
tion, it certainly merits thorough preoperative patient tumors are known or rapid growth, and thus o en
assessment (including evaluation o comorbidities, present with necrosis. T ey commonly invade vas-
and predicted lung di usion capacity). It is also impor- cular and lymphatic tissue, and metastasize early.
tant to remember that i the lung to be resected is not Up to 70% o patients present with metastasis at the
actually unctional (secondary to collapse or bronchial time o diagnosis. Given that SCLC is a systemic
obstruction) than the post-operative predicted FEV1 disease, chemotherapy is the primary medical treat-
should not be expected to change signi cantly rom ment. Combination chemotherapy with the addition
C H AP TER 1 5 TH O RAC I C S U RG ERY LU N G C AN C ER CO U G H 57

o radiotherapy or locoregional control has been Martini N, Bains MS, Burt ME, Zakowski MF, McCormack P,
shown to improve survival. Additional prophylactic Rusch VW, et al. Incidence o local recurrence and second
cranial irradiation has been shown to decrease the primary tumors in resected stage I lung cancer. T e Journal
o thoracic and cardiovascular surgery. 1995;109:1209.
incidence brain metastasis, but has not been shown Morabito A, Carillio G, Daniele G, et al. reatment o small
to improve overall survival. cell lung cancer. Crit Rev Oncol Hematol. 2014;91(3):
Other paraneoplastic syndromes associated with 25770.
hormone alterations should also be recognized. Morabito A, Carrillio G, Daniele G, Piccirillo MC, Mon-
O en Patients with SCLC may exhibit elevations in tanino A, Costanzo R, et al. reatment o small cell lung
cancer. Critical reviews in oncology/hematology Web site.
adrenocorticotropic hormone (AC H). T is may
Accessed rom http://www.croh-online.com. 2014.
mani est with hypokalemia, hyperglycemia, and Pate P, enholder MF, Grif n JP, Eastridge CE, Weiman
metabolic alkalosis. Diagnosis can be con rmed by DS. Preoperative assessment o the high-risk patient or
measuring elevated AC H in the blood or elevated lung resection. he Annals o thoracic surgery. 1996;61:
17-hydroxycorticosteroid levels in the urine. 1494500.
Silvestri GA, Gould MK, Margolis ML, anoue L , McCrory
D, oloza E, et al. Noninvasive staging o non-small cell
BIBLIOGRPAHY lung cancer: ACCP evidenced-based clinical practice guide-
Cook RM, Miller YE, Bunn PA Jr. Small cell lung cancer: lines (2nd edition). Chest. 2007;132:178S-201S.
etiology, biology, clinical eatures, staging, and treatment. T omas P, Rubinstein L. Cancer recurrence a er resection: 1
Curr Probl Cancer. 1993;17(2):69141. N0 non-small cell lung cancer. Lung Cancer Study Group.
Cromartie RS, Parker EF, May JE, Metcal JS, Bartles DM. Car- T e Annals o thoracic surgery. 1990;49:2426; discussion
cinoma o the lung: a clinical review. T e Annals o thoracic 67.
surgery. 1980;30:305. Yan D, Cao C, DAmico A, Demmy L, He J, Hansen H,
Darnell RB, Posner JB. Paraneoplastic syndromes a ecting et al. Video-assisted thoracoscopic surgery lobectomy at 20
the nervous system. Semin Oncol. 2006;33(3):27098. years: A consensus statement. European journal o cardio-
Mountain CF. Revisions in the International System or Stag- thoracic surgery. Of cial journal o the European Associa-
ing Lung Cancer. Chest. 1997;111:17107. tion or Cardio-thoracic Surgery. 2014;45:6339.
This page intentionally left blank
Abdo mina l Wa l l , Abdo men, a nd
Gast r o int est ina l Tr ac t
Robert B. Lim, Ronald A. Gagliano, Jr., and Richard Smith
This page intentionally left blank
16
Abdominal Wall Reconstruction

erri L. Carlson

A 53-year-old emale presents to your clinic with B. T e type o suture used or closure o a midline
complaint o abdominal bulge and occasional dis- incision has been clearly de ned as a determi-
com ort. She tells you that the bulge is worse when nant o postoperative hernia.
she stands and causes pain throughout the day and C. Secondary ventral hernias are thought to be
when she is active, and it seems to be getting bigger related to an abnormal ration o type I and III
over the past year. She has no obstructive symptoms collagen.
at this time. She has a history o a midline laparotomy D. Ventral hernia repair is the most common proce-
or trauma when she was younger. She had a ventral dure per ormed by general surgeons.
hernia repair 6 years ago and thinks they put a syn-
thetic mesh in at that time. She had an episode o cel- 3. O the ollowing positions o mesh placement
lulitis 1 year a er the hernia repair that was treated or hernia repairs, which has the highest rate o
with antibiotics. She has diabetes, which is controlled recurrence?
by oral medications but is otherwise healthy. On exam A. Underlay repair
she has a large palpable de ect and on C scan she B. Onlay repair
has a 6 cm de ect in the midline containing omentum C. Retro-rectus inlay repair
and small bowel as well as two additional 1cm de ects D. Interpositional repair
superior to the larger hernia without evidence o
obstruction. 4. Regarding complications o hernia repair, which
o the ollowing is true?
1. What grade would this hernia be considered based A. Surgical site in ection rates range rom 0% to
on T e Ventral Hernia Work Group (VHWG) 12% or clean cases and up to 34% or clean-
classif cation? contaminated and contaminated cases.
A. Grade 1 B. T e Centers or Disease Control (CDC) de nes
B. Grade 2 mesh in ections as occurring up to 6 months
C. Grade 3 a er implantation o prosthetic mesh.
D. Grade 4 C. Seroma ormation is common postoperatively
despite drain placement and there ore drains
2. Regarding ventral hernia which o the ollowing is should be removed within 1 to 2 weeks to prevent
correct? retrograde in ection.
A. Primary suture repair has the same recurrence D. T e most common organisms identi ed in mesh
rate as does mesh repair. in ections are gram-negative organisms such as
Klebsiella and Proteus spp.
62 G EN ERAL S U RG ERY EXAM IN ATI O N AN D BO ARD REVIEW

5. Which o the ollowing is true with respect to recurrence rate with the exception o small 1 to 2 cm
biologic and synthetic mesh materials? primary hernias. T ere has been speculation that at the
A. Use o biologic mesh material in a contaminated time o primary operation the type o suture used or
eld has around a 30% surgical site in ection rate. closure correlates with rate o incisional hernia orma-
B. Recurrence rates a er biologic mesh can be as tion; however, there is no conclusive evidence to sup-
high as 30% in contaminated elds. port this notion. Primary ventral hernias are thought
C. Advanced age is associated with an increased to have a genetic predisposition, while secondary or
risk o complications a er abdominal wall incisional hernias are likely related to abnormal ratios
reconstruction. o ype I and III collagen as well as the amount o
D. Synthetic mesh in ections have been success ully metallopreotease expression. An average o 150,000 to
treated using a vacuum-assisted closure (VAC) 250,000 ventral hernias are per ormed each year mak-
technique without the need or mesh explantation. ing it the h most common procedure.

3. D. Interpositional repair where the mesh is sutured


ANSWERS directly to the ascial edge in a bridging ashion has
1. C. A grading system was created to help identi y the largely been abandoned due to the extremely high
risk o morbidity rom a ventral hernia repair and recurrence rates. Onlay mesh repair places the mesh
to help decide which type o mesh, biologic or syn- above the rectus sheath. T ere are several types o inlay
thetic, would be best utilized. T is patient would be repair. In the retrorectus repair, the mesh is placed
considered a Grade 3 because o her history o previ- between the rectus abdominis muscle and the posterior
ous in ection despite not requiring mesh removal. I rectus sheath. Alternatively the mesh can be placed pre-
there was no previous in ection then she would be peritoneal between the posterior rectus sheath and the
considered a Grade 2. preperitoneal at. An intraperitoneal or underlay type
Grade 1 (low risk) would mean no comorbidities repair places the mesh underneath the peritoneum. T is
(i.e., a young healthy individual). type o repair is used or laparoscopic repairs. Because
A Grade 2 (comorbid) consideration means the mesh is placed under the peritoneum, intraperito-
nding comorbidities (e.g., smoking, diabetes, or neal repair is thought to have higher rates o adhesions
malnutrition) that increase the risk o surgical site and erosion o the mesh into the bowel. In general, the
in ections (SSIs). T ere was no evidence o wound type o repair is surgeon pre erence.
contamination or active in ection. Advances in mesh products such as composite mesh
Grade 3 (potentially contaminated) shows evi- with anti-adhesive barriers have been developed to
dence o wound contamination (e.g., seroma, viola- decrease complications. Onlay type mesh repairs have
tion o the GI tract, or history o wound in ection). more potential or seroma ormation because a large
T is includes patients with active or suspected subcutaneous dissection is required and is also more
wound contamination. susceptible to contamination rom in ection because
Grade 4 (in ected) would be considered i active o its super cial location. Inlay mesh repairs are gener-
in ections such as in ected synthetic mesh or a septic ally pre erred because o their ability to distribute intra-
dehiscence was ound. abdominal wall pressure and decrease overall tension.
Also underlay and retrorectus repairs have been shown
2. C. It has long been established that direct suture to have the lowest recurrence rates o all types o mesh
repair or ventral hernia has an unacceptably high repair with the highest being interposition and onlay.

Re trore ctus pla ce me nt of me s h


C H AP TER 1 6 ABD O M IN AL WALL RECO N S TRU C TI O N 63

4. A. Surgical site in ections are common a er ventral closure (VAC) techniques can be success ul in treat-
hernia repair and are de ned as super cial, deep, and ing surgical site in ections, including synthetic mesh
organ space in ections. Intraoperative level o wound in ections. A meta-analysis by Bellows et al. includ-
contamination based on CDC criteria include clean, ing 60 studies showed a 17% surgical site in ection
clean-contaminated, contaminated and dirty corre- and 15% recurrence rate in Grade 3 contaminated
lates with rate o surgical site in ections. Clean cases elds a er biologic mesh repair.
have an in ection rate o 0% to 12% while clean-
contaminated and contaminated have in ection rates BIBLIOGRAPHY
at high as 34%. T e CDC de nes mesh in ections as
Albino FP, Patel KM, Nahabedian MY, Sosin M, Attinger CE,
occurring up to 1 year a er implantation o pros- Bhanot P. Does mesh location matter in abdominal wall
thetic mesh. T e most common organism cultured reconstruction? A systematic review o the literature and a
rom in ected mesh is staphylococcus aureus and is summary o recommendations. Plast Reconstr Surg. 2013;
seen in up to 81% o cases. T is suggests a possible 132(5):12951304.
skin f ora contamination at the time o mesh implan- Bellows CF, Smith A, Malsbury J, Helton WS. Repair o
incisional hernias with biological prosthesis: a system-
tation. While gram-negative organisms are presents atic review o current evidence. Am J Surg. 2013;205(1):
in mesh in ections, they only occur about 17% o 85101.
the time. Seromas are common a er ventral hernia Butler CE, Baumann DP, Janis JE, Rosen MJ. Abdominal
and drains are recommended to decrease the dead wall reconstruction. Curr Probl Surg. 2013;50(12):557
space. While they can cause retrograde bacterial con- 86.
Fischer JP, Wink JD, Nelson JA, Kovach SJ 3rd. Among 1,706
tamination, there is no de nitive recommendation o
cases o abdominal wall reconstruction, what actors inf u-
time or removal and are o en necessary or up to 4 ence the occurrence o major operative complications?
to 6 weeks. Surgery. 2014;155(2):3119.
Lati R and Higa G. Surgery of complex abdominal wall defects.
5. D. Prolonged operative time and American Society New York, NY: Springer Science & Business Media. 2013;
o Anesthesiologists (ASA) > 2 have been associated 5354.
with major operative complications, but advanced Montgomery A. T e battle between biological and syn-
thetic meshes in ventral hernia repair. Hernia. 2013;17:
age has not been shown as a predictor o morbid-
311.
ity. Particular comorbidities identi ed were obe- urza KC and Butler CE. Adhesions and meshes: synthetic ver-
sity, smoking, hypertension, diabetes and anemia. sus bioprosthetic. Plast Reconstr Surg. 2012;130(5 Suppl 2):
Berrevoet et al. showed that use o vacuum-assisted 206S-13S.
17
HerniaUmbilical/Ventral
with Cirrhosis
Richard M. Peterson

A 44-year-old male presents to the emergency depart- 2. Which o the ollowing actors is associated with
ment. He has had complaints o abdominal pain over increased incidence o abdominal wall hernias in
the last several days. oday he was bending over to pick patients with cirrhosis?
up something o o the oor and he elt a sudden sharp A. Increased clotting time
pain at his belly button. He states the area is now very B. Single episode o ascites ormation
tender and achy. He has elt some nausea or the last C. Rising creatinine
hour or two. No change in bowel habits prior to this D. Recanalized umbilical vein
episode. His past medical history is signi cant or an E. Decreased mobility
umbilical hernia, hypertension and cirrhosis. Past sur-
gical history is or an appendectomy when he was 27. 3. Which o the ollowing relates to worse outcomes
He smokes ppd and has 23 drinks per day. He denies in the setting o emergent repair o an umbilical
any drug use. When pressed he states that his doctors hernia in the cirrhotic patient?
told him his liver problems are because he drinks too A. Mesh insertion
much. B. Smoking
Vital signs are: HR 105, BP 140/91, RR 18, Pulse Ox C. Omentectomy
98% on RA. On exam there is no evidence o jaundice. He D. Small bowel resection
is noted to have tenderness directly at the umbilicus with E. Anemia
a protuberant abdomen. It is dull to percussion. T ere are
skin changes noted at the umbilicus, speci cally an area 4. Best management o an umbilical/ventral hernia in
o ulceration and a non-reducible hernia is noted. a patient with cirrhosis and ascites would include
Labs: which o the ollowing?
WBC: 13.1; Hgb/Hct: 13.5/40; Platelets: 202 A. Elective repair rather than emergent management
Na: 138; K: 3.7; BUN/Cr: 23/1.4; Bili: 1.7; INR: 1.6 B. Preoperative ascites control only
C. Postoperative ascites control only
1. Which o the ollowing negatively a ects the D. Primary repair o the hernia
patients perioperative mortality risk specif cally E. Intra-operative drain placement only
due to his cirrhosis?
A. White blood cell count o 13.1
B. Hypertension
C. Alcohol consumption
D. Smoking
E. INR
C H AP TER 1 7 H ERn i A U m b i l i C Al / VEn TRAl w i TH C i RRH o s i s 65

ANSWERS creatinine rom kidney dys unction, and the


increased platelet dys unction due to uremia do not
1. E. In a cirrhotic patient, the extent o the cirrhosis in uence the development o a hernia.
a ects the perioperative mortality; but the etiology o
cirrhosis does not have to be taken into consideration.
3. E. In the setting o emergent repair o abdominal
o determine the patients 90-day mortality, the Model
wall/umbilical hernia in the cirrhotic patient, pre-
or End Stage Liver Disease (MELD) score can be
operative anemia was the only preoperative predic-
used. T e MELD score is calculated by the equation:
tor o complicated outcome. T e other actors that
MELD = 3.78 ln[serum bilirubin (mg/dL)] + 11.2 were ound to be statistically signi cant and related
ln[INR] + 9.57 ln[serum creatinine to major complications were age greater than 65 and
(mg/dL)] + 6.43 higher Model or End Stage Liver Disease (MELD)
score. Additionally the authors in this study noted that
As such, the patients bilirubin, INR, and creati-
small bowel obstruction in this setting had a nine old
nine a ect his survival chances. T is system was
increase in risk or postoperative complications. How-
developed to determine outcomes or patients having
ever, smoking, omentectomy, mesh insertion and small
undergone transjugular intrahepatic portosystemic
bowel resection were not ound to be signi cant actors
shunt ( IPS) procedures. It has now become the
in the development o postoperative complications.
standard or determination o severity o liver dis-
ease (having replaced the Childs Classi cation) and
4. A. In a randomized controlled trial o 80 patients
based on the new criteria as outlined by the United
having repair with or without mesh by Ammar, the
Network or Organ Sharing (UNOS), requency o
author ound that primary repair o an umbilical/
dialysis was applied to the ormula. In addition the
ventral hernia in the setting o cirrhosis and ascites
etiology o the cirrhosis (or end-stage liver disease)
has a signi cantly higher recurrence rate than when a
was not ound to be a signi cant prognostic indicator
mesh repair is per ormed (14.2% vs 2.7%, p < 0.05).
and was removed rom the original MELD score. In
Most authors now advocate or more aggressive elec-
our particular patient the MELD score is 17, which
tive repair o hernias in patients with cirrhosis and
correlates with a 6% 3-month mortality.
ascites even in the setting o Class B and C cirrhosis.
MELD Scores In the study by Carbonell et al they ound that elective
surgical morbidity in cirrhotics was no di erent rom
40 or more 71.3% mortality non-cirrhotics (15.6% vs. 13.5%; p = 0.18). Emergent
3039 52.6% mortality surgery morbidity was statistically signi cant between
2029 19.6% mortality the 2 groups (17.3% vs. 14.5%; p = 0.04). While di -
erences in elective surgical mortality in cirrhotics
1019 6.0% mortality
approached signi cance (0.6% vs. 0.1%; p = 0.06),
<9 1.9% mortality mortality was 7- old higher in emergency surgery
(3.8% vs. 0.5%; p < 0.0001). In addition the oppor-
2. D. Patients with liver cirrhosis and ascites have a risk tunity to utilize laparoscopy in the elective setting in
o developing an umbilical hernia that approaches this patient population has the ability to mitigate post-
20%. A single episode o ascites is not highly asso- operative complications and decrease length o hospi-
ciated with abdominal wall hernia ormation. How- talization. Medical diuresis, postoperative paracentesis
ever as the number o episodes o ascites increases, and drain placement at time o surgery are all actors
so does the incidence o hernia ormation, such that that improve the postoperative course af er hernia
by the third episode about 70% o patients will have repair in these patients. Odom et al noted a signi cant
developed an umbilical hernia. T e other actors, increase in major complications when no invasive
which include increased intra-abdominal pressure, measure was used to control the ascites.
muscle wasting and weakening o the abdominal
ascia releated to malnutrition and recanalization BIBLIOGRAPHY
o the umbilical vein, are all contributing actors Ammar SA. Management o complicated umbilical hernias
to the development o umbilical hernias in the cir- in cirrhotic patients using permanent mesh: randomized
rhotic patient. T e decreased unctional status, rising clinical trial. Hernia. 2010;14(1):358.
66 G En ERAl s U RG ERY EXAm i n ATi o n An D b o ARD REVi Ew

Belli G, DAgostino A, Fantini C, et al. Laparoscopic incisional with liver cirrhosis and ascites. Surgery. 2011;150:
and umbilical hernia repair in cirrhotic patients. Surg Lap- 5426.
arosc Endosc Percutan Tech. 2006;16:33033. Kamath PS, Kim WR. T e model or end-stage liver disease
Carbonell AM, Wol e LG, DeMaria EJ. Poor outcomes in cir- (MELD). Hepatology 2007;45(3):797805.
rhosis-associated hernia repair: a nationwide cohort study Odom SR, Gupta A, almor D, et al. Emergency hernia repair
o 32,033 patients. Hernia. 2005;9:3537. in cirrhotic patients with ascites. J Trauma Acute Care Surg.
Choi SB, Hong KD, Lee JS, et al. Management o umbilical 2013;75:4049.
hernia complicated with liver cirrhosis: An advocate o Online MELD calculator: http://www.mayoclinic.org/medical-
early and elective herniorrhaphy. Dig Liver Dis. 2011;43: pro essionals/model-end-stage-liver-disease/meld-model-
9915. unos-modi cation
Eker HH, van Ramshorst GH, de Goede B, et al. Prospec-
tive study on elective umbilical hernia repair in patients
18
Asymptomatic Inguinal Hernia

Lorenzo Anes-Bustillos

An otherwise healthy 31-year-old male presents with a 2. Which o the ollowing is considered the most
painless bulge in his right groin. Although it does not common early complication af er open inguinal
bother him or limit his daily activities, he notices uc- hernia repair?
tuation in its size, rom being absent when he lies at or A. Surgical site in ection
a ew minutes, to a gol -ball sized mass upon coughing B. Hematoma/seroma
or strenuous exercises. Physical examination con rms C. Urinary tract in ection
the presence o a right inguinal hernia. No abnormalities D. Small bowel obstruction
were noted on the contralateral side. E. Recurrence
1. Regarding the management and indications or 3. Which o the ollowing is true regarding laparo-
surgical intervention o asymptomatic inguinal scopic inguinal hernia repair?
hernias, which o the ollowing is correct?
A. It is generally less expensive than the open repair.
A. Observation is not recommended, as the inci- B. Potential complications are similar, or less severe,
dence o inguinal hernia strangulation is greater to those seen with open repair.
than 10% a er 5 years. C. Procedure is limited to the repair o only one
B. Elective surgical repair is advised or otherwise de ect in the inguinal region given poor visual-
healthy patients, as procedure is airly low-risk ization o the anatomy.
and post-operative complications are always D. Although patients have a aster return to daily activ-
minor, i any. ities, they are associated with greater persistent pain
C. Both watch ul waiting and surgical repair are and numbness compared to the open approach.
sa e as treatment options, however most patients E. It is usually the avored approach in special situ-
will develop symptoms over time and require an ations such as in recurrent or bilateral inguinal
operation. hernias.
D. T e type o inguinal hernia, whether direct
or indirect, heavily dictates the management 4. Af er an open inguinal hernia repair, which
approach, as incarceration is at least ten times symptom is likely to appear as a result o the most
more o en in the case o direct hernias. commonly injured nerve in this type o approach?
E. As emergency and elective inguinal hernia A. Numbness on ipsilateral upper lateral thigh
repairs share similar morbidity and mortality B. Numbness on ipsilateral medial thigh
rates, watch ul waiting is always considered a C. Hyper response o the cremasteric re ex
sa e option. D. Hypersthesia o the ipsilateral hemiscrotum
E. Numbness o the suprapubic region
68 G EN ERAL S U RG ERY EXAM I N ATI O N AN D BO ARD REVI EW

5. Which o the ollowing is true regarding emoral with low short-term morbidity, mortality, and recur-
hernias? rence rates. T is is di erent rom emergent operations,
A. Watch ul waiting or surgical repair are both rea- which carry higher morbidity and mortality rates
sonable options to consider in otherwise healthy given the additional risks o gangrene, per oration,
patients diagnosed with asymptomatic emoral and in ection o the peritoneal cavity. Nonetheless,
hernia. watch ul waiting spares the patient rom the complica-
B. T e mesh plug repair is the pre erred approach tions o elective surgery such as surgical site in ection,
in emergent cases. hematoma, urinary retention, and more in the long-
C. Rate o incarceration and strangulation is reported term, chronic groin pain, neuralgia and recurrence.
as greater than 40%. Studies by Fitzgibbons et al. and ODwyer et al.
D. Men are more likely than women to experience have been key in addressing the di erences between
this type o hernia. elective surgery and watch ul waiting. Although
E. Its sac protrudes through the emoral canal, they both show no di erence in terms o discom ort
bounded medially by the emoral vein. and pain across groups, there is a rate o conversion
driven mainly by pain o 54% and 72% a er 5 and 7.5
years, respectively, in those managed non-operatively.
ANSWERS
A more recent systematic review o the evidence by
1. C. As one o the naturally weak points in the abdom- Mizrahi et al. (Arch Surg. 2012;147:27781) shows
inal wall, the groin area is prone to the protrusion that the rate o strangulation in those who do not
o peritoneal sac. When this occurs in the presence pursue surgery is quite low at less than 1% a er a two-
o minor symptoms or in the absence o symptoms and a our-year ollow-up period. On the other hand,
altogether, the condition is known as asymptomatic the range o operative complications in those under-
inguinal hernia. One-third o patients with inguinal going elective surgery goes rom 0% to 22.3%, with a
hernias all within this category, presenting with a recurrence rate o 2.1%. Both treatment options are
non-tender bulge in the area. thus considered sa e, although most patients will pro-
T e in erior epigastric vessels, as well as the inter- gress to develop symptoms and eventually require an
nal and external inguinal rings, provide anatomic operation.
landmarks that help in distinguishing direct and
indirect inguinal hernias. T e sac o a direct hernia 2. B. T e overall risk o complications ollowing open
protrudes outward and orward, medial to the inter- inguinal hernia repair is low, and ortunately, these
nal inguinal ring and the in erior epigastric vessels. are o entimes transient and easy to manage. Several
On the other hand, indirect hernias pass rom the actors play a role in the occurrence o complications,
internal ring obliquely toward the external ring, lat- including the surgical technique, surgeon experience,
eral to the vessels. A pantaloon-type hernia occurs and anesthetic choices. Wound and scrotal hemato-
when both o these eatures are present. Regardless, mas are amongst the most common early postopera-
the anatomic distinction between these is o little tive complications, with reported rates o 6.1% and
importance given the similarities in the approach 4.5%, respectively. Neumayer L et al. compared open
or operative repair. More traditional descriptions o mesh versus laparoscopic inguinal hernia repair and
inguinal hernias are made on the basis o its contents reported a combined incidence o hematoma or ser-
(e.g., sliding, Richters, Littres), or the status o the oma o 13.6% in open cases, and 16.4% in those per-
contents (e.g., sliding, incarcerated, strangulated). ormed laparoscopically. Other less common adverse
With more than 20 million operations per ormed events seen in the immediate postoperative period
annually around the world, inguinal hernia repair is include urinary retention, orchitis, pain, and urinary
the most common elective procedure per ormed in the tract and wound in ections. As or long-term com-
United States and Europe. T ere is a well-documented plications, chronic pain tops the list, with a reported
debate as to what the best management is or asympto- rate o 14% to 19% ollowing open repairs with some
matic inguinal hernias, that being between the opera- series reporting requencies as low as 6% or as high as
tive and the watch ul waiting approach. T e rationale 75.5%. T e risk is lower ollowing laparoscopic cases,
o repairing all inguinal hernias, whether symptomatic with rates ranging between 9.8% to 13.8%. Orchitis,
or not, lies on their risk o incarceration and strangu- in ection, and hernia recurrence are amongst other
lation. Elective repairs are considered relatively sa e common late complications.
C H AP TER 1 8 AS YM P TO MATIC I N G U IN AL H ERN I A 69

3. E. T e approach and operative technique or the Special situations may avor the use o the lapa-
repair o inguinal hernias has evolved throughout roscopic approach. In the setting o bilateral ingui-
the years. Repairs can be either done open or laparo- nal hernias, the ability to use the same access sites
scopically. T e open approach can be either a tissue evidently translates into aster recovery and less
repair or a prosthetic (tension- ree) repair. Although postoperative pain. However, whether done open or
the ormer might be o historical interest, there are laparoscopically, the simultaneous repair o bilateral
some situations in which these may be indicated such hernias does not increase the risk o re-operation or
as in a contaminated eld, in pediatric patients, or recurrence. Another typical challenge or the surgeon
in those places where access to prostheses is limited. is that o recurrent hernias. T ese may be caused by
By identi ying increased tension as the main cause either technical problems (e.g., knot slipped, crushed
o recurrence, Lichtenstein popularized the use o a suture, inadequate bites o ascia), or patient ac-
synthetic mesh to bridge the hernia de ect and pro- tors such as malnutrition, steroid use, and smoking.
vide a tension- ree repair. T is approach is the gold A second groin exploration entails cutting through
standard o open hernia repairs, as it has proven to scar tissue that in itsel adds a certain degree o di -
decrease the rates o postoperative discom ort, dura- culty and additional trauma with increased risk o
tion o hospital stay, and recurrence. T e laparoscopic damage to the testicular blood supply and sensory
approach also o ers a tension- ree repair. T ose who nerves. Access through virgin territories through a
support this technique emphasize the quicker time to posterior approach explains the advantage o laparo-
recovery, less pain, better visualization o the anatomy, scopic repairs in these situations.
and the ability to repair all the de ects in the inguinal
region. On the other hand, critics insist on the longer 4. B. Depending on the approach used, speci c nerves
operative times, technical challenges, risk o recur- in the area will be more prone to injury. For example,
rence, and increased costs. A meta-analysis study by in open hernia repairs, the ilioinguinal nerve can be
Voyles et al. (Am J Surg. 2002;184:610) compared most commonly injured at the external ring where it
the two approaches and showed that both provide runs on top o the cord. T is results in loss o the cre-
equivalent outcomes, with open repairs being lower masteric re ex and numbness to the ipsilateral penis,
in cost, and entailing a lower risk o severe postopera- scrotum, and thigh. On the other hand, the posterior
tive complications. T ose unique to the laparoscopic approach used in laparoscopic surgery can result in
approach include small bowel obstruction, internal injuries to the lateral emoral cutaneous, emoral
hernia, bladder per oration, in arcted omentum, and branch o the genito emoral (sensory, upper lateral
port-site hernia. thigh), and rarely, the emoral nerves. Hypersthesia

Exte rna l oblique


a pone uros is

Iliohypoga s tric n.

Re ctus a bdominus m.

Ilioinguina l n.
Inte rna l oblique m.
a nd a pone uros is Exte rna l s pe rma tic v.

Ge nita l bra nch of


ge nitofe mora l n.

P ubic tube rcle

Ing uinal anato my view,


o pe n appro ac h
70 G EN ERAL S U RG ERY EXAM I N ATI O N AN D BO ARD REVI EW

o the hemiscrotum and numbness o the suprapubic Hachisuka . Femoral hernia repair. Surg Clin North Am. 2003;
region are not complications o nerve injury rom 83:1189205.
inguinal hernia repairs. Humes DJ, Radcli e RS, Camm C, and West J. Population-
based study o presentation and adverse outcomes a er
emoral hernia surgery. Br J Surg. 2013;100:182732.
5. C. Rate o incarceration and/or strangulation is
Jones DB. Master echniques in Surgery: Hernia. Philadelphia,
reported to be greater than 40%. Another potential PA: Lippincott Williams & Wilkins; 2012.
de ect where hernias can occur is that o the emoral Kald A, Fridsten S, Nordin P, Nilsson E. Outcome o repair o
canal, bounded superiorly by the iliopubic tract, in e- bilateral groin hernias: a prospective evaluation o 1,487
riorly by the lacunar ligament, laterally by the emoral patients. Eur J Surg. 2002:168;1503.
vein, and medially by the junction o the iliopubic tract Kingsnorth A, LeBlanc K. Hernias: Inguinal and incisional.
Lancet (London, England) 2003;362;156171.
and lacunar ligament. Its dif cult diagnosis and treat- Kurzer M, Belsham PA, Kark AE. Prospective study o open
ment pose a challenge to even the more experienced preperitoneal mesh repair or recurrent inguinal hernia. Br
surgeon. Added to this, a large proportion o patients J Surg. 2002;89:903.
with this type o hernia present late to medical care, Lichtenstein IL, Shulman AG, Amid PK, Montllor MM. T e
o entimes requiring emergent interventions that tension- ree hernioplasty. Am J Surg. 1989;157(2):18893.
Malangoni MA, Rosen MJ. Hiatal Hernia and Gastroesopha-
translate in higher risk o adverse postoperative events.
geal Re ux Disease. In: ownsend CM, Beauchamp RD,
Women are more likely to experience this type o hernia Evers BM, Mattox K, eds. Sabiston extbook of Surgery:
than men, comprising about 30% o their groin hernias T e Biological Basis of Modern Surgical Practice. 19th ed.
(versus only 2% in men). As they are usually associated Philadelphia, PA: Elsevier Saunders; 2012:11141140.
with incarceration and strangulation (reported rates Matthews RD, Neumayer L. Inguinal hernia in the 21st century:
between 44% and 86%), the most adequate treatment An evidence-based review. Curr Probl Surg. 2008;45:261312.
McCormack K, Scott NW, Go PM, Ross S, Grant AM. Lapa-
once diagnosed is surgery. All emoral hernias should roscopic techniques versus open techniques or inguinal
be repaired, and in the presence o incarcerated con- hernia repair. Cochrane database Syst. Rev. CD001785
tents, the sac should be assessed or viability. Delayed (2003). doi:10.1002/14651858.CD001785
diagnosis will lead to higher morbidity and mortality. Mizrahi H, Parker MC. Management o asymptomatic inguinal
Di erent repair techniques are available and mainly hernia: A systematic review o the evidence. Arch Surg. 2012;
147:27781.
depend on the clinical presentation. Many consider the
Naude GP, Ocon S, Bongard F. Femoral hernia: T e dire con-
mesh plug repair as the technique o choice in elective sequences o a missed diagnosis. Am J Emerg Med. 1997;15:
and non-in ected cases. In contrast, the tissue repair 6802.
(i.e., McVay operation) should be pre erred in strangu- Neumayer L, Giobbie-Hurder A, Jonasson O, et al. Open mesh
lated cases in which severe in ection is present. versus laparoscopic mesh repair o inguinal hernia. N Engl
J Med. 2004;350:181927.
ODwyer PJ, Norrie J, Alani A, Walker A, Du y F, Horgan P.
BIBLIOGRAPHY Observation or operation or patients with an asymptom-
Aasvang EK, Bay-Nielsen M, Kehlet H. Pain and unctional atic inguinal hernia: a randomized clinical trial. Ann Surg.
impairment 6 years a er inguinal herniorrhaphy. Hernia. 2006;244:16773.
2006;10:31621. Page B, Paterson C, Young D, ODwyer PJ. Pain rom primary
Alimoglu O, Alimoglu O, Kaya B, Okan I, et al. Femoral hernia: inguinal hernia and the e ect o repair on pain. Br J Surg.
a review o 83 cases. Hernia. 2006;10:703. 2002;89:13158.
Amid PK, Shulman AG, Lichtenstein IL. Open tension- ree Primatesta P, Goldacre MJ. Inguinal hernia repair: incidence
repair o inguinal hernias: the Lichtenstein technique. Eur o elective and emergency surgery, readmission and mor-
J Surg. 1996;162:44753. tality. Int J Epidemiol. 1996;25:8359.
Bittner R, Schwarz J. Inguinal hernia repair: current surgical Sherman V, Macho JR, Brunicardi FC. Inguinal Hernias. In:
techniques. Langenbecks Arch Surg. 2012;397:27182. Brunicardi FC, Andersen D, Billiar , Dunn D, Hunter
Chung L, Norrie J, ODwyer PJ. Long-term ollow-up o J, Matthews J, et al, eds. Schwartzs Principles Surgery. 9e.
patients with a painless inguinal hernia rom a random- (pp 13051342) New York, NY: McGraw-Hill Pro essional;
ized clinical trial. Br J Surg. 2011;98:5969. 2010: http://mhmedical.com/o content.aspx?aid=5031117
Fitzgibbons RJ et al. Watch ul waiting vs repair o inguinal van den Heuvel B, Dwars BJ, Klassen DR, Bonjer HJ. Is surgi-
hernia in minimally symptomatic men: a randomized clin- cal repair o an asymptomatic groin hernia appropriate? A
ical trial. JAMA. 2006;295:28592. review. Hernia 2011;15:2519.
Fitzgibbons RJ Jr, Ramanan B, Arya S, et al. Long-term results Voyles CR, Hamilton BJ, Johnson WD, Kano N. Meta-analysis o
o a randomized controlled trial o a nonoperative strategy laparoscopic inguinal hernia trials avors open hernia repair
(watch ul waiting) or men with minimally symptomatic with preperitoneal mesh prosthesis. Am J Surg. 2002;184:
inguinal hernias. Ann Surg. 2013;258:50815. 610.
19
Femoral Hernia

Harry T. Aubin

A 65-year-old emale is seen in your clinic or a lump emoral sheath medial to emoral vein, and
in her groin. She states the lump has been present or inguninal ligament (iliopubic tract), approxi-
a ew years and intermittently becomes larger in size. mate the conjoint tendon to the inguinal liga-
On exam, there is no evidence o adenopathy or venous ment laterally to the internal ring.
thrombosis. A reducible hernia is palpated. You per- C. Expose Coopers ligament, make relaxing inci-
orm an uncomplicated McVay repair and she is seen in sion on anterior rectus sheath, suture transversus
ollow up one year later with a recurrence. abdominus aponeurosis to Coopers ligament
beginning at the pubic tubercle towards the em-
1. Regarding emoral hernias, which o the ollowing oral sheath laterally to the internal ring.
is true? D. Expose Coopers ligament, suture transversus
A. It is an acquired de ect. abdominus aponeurosis to Coopers ligament
B. It has a male preponderance. beginning at the pubic tubercle towards the em-
C. It is more common than inguinal hernias. oral sheath laterally to the internal ring.
D. T e incidence o incarceration is lower than
inguinal hernias. 3. Regarding recurrence o this patients hernia,
which o the ollowing is the most likely etiology?
2. Regarding the McVay technique or hernia repair, A. Congenital collagen disorder
which answer choice displays the key steps in order? B. Wound in ection
A. Expose Coopers ligament, suture transversus C. ension on suture line
abdominus aponeurosis to Coopers ligament D. Postmenopausal
beginning at the pubic tubercle towards the
emoral sheath, place transition stitch containing 4. Regarding repair o groin hernias, which o the
transversus abdominus, Coopers ligament, em- ollowing techniques will f x a emoral hernia?
oral sheath medial to emoral vein, and inguni- A. Bassini repair
nal ligament (iliopubic tract), approximate the B. Marcy repair
conjoint tendon to the inguinal ligament later- C. Shouldice repair
ally to the internal ring. D. Preperitoneal repair
B. Expose Coopers ligament, make relaxing inci- E. Lichtenstein tension ree with mesh repair
sion on anterior rectus sheath, suture transver-
sus abdominus aponeurosis to Coopers ligament
ANSWERS
beginning at the pubic tubercle towards the
emoral sheath, place transition stitch contain- 1. A. Femoral hernias are acquired hernias and NO o
ing transversus abdominus, Coopers ligament, congenital origin. T ey are more common in emales
72 G EN ERAL S U RG ERY EXAM IN ATI O N AN D BO ARD REVIEW

and more common in older women who are multip- developing a recurrent hernia with mesh repairs were
arous, as laxity o the abdominal wall and stretching reduced by about hal , though recurrence rates were
o the emoral ring with aging or pregnancy is elt relatively small with each repair. Other actors can
to be an etiology. Given the mostly xed, con ned contribute to recurrence such as wound in ection, but
spaces o the emoral canal, these hernias are noto- are not the primary/most common cause. Congenital
rious or incarcerating leading to emergent/urgent collagen disorder is a rare disorder and is not likely to
hernia repair. be seen in this patient.
A study rom the Swedish Hernia Register showed
an incidence o 2 to 4 percent o all groin hernias over 4. D. Lichtenstein tension ree hernia repair with mesh
a 14 year period. O these hernias, 35.9% o emoral is the most common hernia repair done at most
hernias were ound to present as an emergency (incar- institutions. It requires little suturing, does not need a
ceration) needing surgery, and o these, 22.7% required relaxing incision, and does not need general anesthe-
bowel resection. T is is compared to only 4.9% need- sia. Un ortunately, the drawback o this repair is that
ing emergency surgery in the inguinal group. It is it does not close, nor cover, the emoral ring. T ere-
seen more commonly in women, roughly 2:1 emale: ore, it is not used or repair o emoral hernias.
male. T e Marcy repair only repairs the deep inguinal
ring and is mostly used in pediatric patients. T e
2. B. T e McVay repair, aka the Coopers Ligament Bassini and Shouldice repairs only repair the ingui-
repair, is a tissue repair that is e ective in repair o all nal f oor and will not treat a emoral hernia.
three groin hernias (indirect, direct, and emoral). It T e preperitoneal repair, a variation known as the
is per ormed with non-absorbable sutures in an inter- Kugel repair, involves placing mesh in the preperito-
rupted ashion. Sutures are placed to sew the trans- neal space and suturing this rom the pubic tubercle
versus abdominus aponeurosis to Coopers ligament to Coopers ligament. Bi-layer mesh repair, in theory
beginning at the pubic tubercle towards the emo- is a combined preperitoneal and Lichtenstein repair
ral sheath. Once this is reached, place a transition with mesh and is used or emoral hernia repair. Plug
stitch containing transversus abdominus, Coopers and patch repair can be utilized to obliterate the
ligament, emoral sheath medial to emoral vein, and emoral canal in emoral hernia repair. Laparoscopic
inguinal ligament (iliopubic tract), then approximate repair, as with the above repairs, is a described tech-
the transversus abdominal aponeurosis to the ingui- nique or repair o emoral hernias and utilizes the
nal ligament laterally to the internal ring. Exposure o preperitoneal space.
Coopers ligament is done prior to suturing. Addition-
ally, to avoid tension, a curvilinear relaxing incision
is made through the anterior rectus sheath starting BIBLIOGRAPHY
1cm cephalad o the pubic tubercle to near its lateral Dahlstrand U, Wollert S, Nordin P, Sandblom G, Gunnarsson U.
border. Emergency emoral hernia repair: A study based on a
national register. Ann Surg 2009;249(4):6726.
3. C. ension on the suture line is elt to be the most EU Hernia rialists Collaboration. Repair o groin hernia
common cause o hernia recurrence in general and with synthetic mesh: Meta-analysis o randomized con-
trolled trials. Ann Surg 2002;235(3):32232.
especially in tissue repairs. T is is why an adequate Kark AE, Kurzer M. Groin hernias in women. Hernia. June
relaxing incision is necessary in the McVay repair. 2008;12(3):26770.
Most likely, there was some tension on the repair McVAY CB, CHAPP JD. Inguinal and emoral hernioplasty:
despite the relaxing incision. T e evaluation o a basic concept. Ann Surg. 1958;148(4):
A shi has been made to tension ree repairs with 499510; discussion 510-2.
Malagoni MA, Rosen MJ. Hernias. In: ownsend CM, Beau-
the usage o mesh. issue vs mesh repair was com-
champ RD, Evers BM, Mattox K, eds. Sabiston extbook of
pared in a large meta-anaylsis containing over 11,000 Surgery: T e Biological Basis of Modern Surgical Practice.
patients speci cally looking at recurrence rates a er 19th ed. Philadelphia, PA: Elsevier Saunders; 2012:1114
hernia surgery. Findings o this show the odds o 40.
20
Inguinal Neuralgia

Harry T. Aubin and Eric Balent

A 35-year-old male presents to clinic three months a er C. Re erral to a pain management specialist is
an uncomplicated open right inguinal hernia repair with recommended.
mesh or a chronic, minimally symptomatic, indirect D. riple neurectomy is recommended only a er
inguinal hernia. During the operation, the ilioinguinal ailing less invasive treatments by a pain man-
nerve was intentionally divided. He reports continued agement specialist.
right sided, sharp, episodic groin pain radiating to his E. All are correct.
testicle that is worse than the symptoms he had prior
to repair. He took ibupro en and acetaminophen or 3. Regarding nerve injuries in hernia repair, which o
6 weeks a er the surgery with minimal relie . T e pain the ollowing is correct?
is beginning to limit his activities at work. He now com- A. Injury to the ilioinguinal nerve causes loss o
plains o worsening pain. cremasteric re ex, and numbness to the ipsilat-
eral scrotum, penis, and medial thigh.
1. Regarding the pathophysiology o chronic groin B. Injury to the emoral branch o the lateral emo-
pain af er hernia surgery, which o the ollowing is ral cutaneous nerve causes loss o sensation to
correct? the medial thigh.
A. Most commonly, it is elt to be neuropathic and C. ack placement in erior to the iliopubic tract and
due to primary nerve injury during the operation. medial to the spermatic cord is avoided to mini-
B. It is elt to be due to in ammatory mechanisms mize nerve damage in laparoscopic hernia repair.
rom the operation and healing. D. Injury to the genital branch o the genito emo-
C. It is nociceptive. ral nerve results in loss o sensation o the entire
D. acking mesh to nerves in a laparoscopic repair scrotum and lack o cremestric re ex on the con-
is an uncommon cause. tralateral side.
E. Secondary nerve injury rom either in amma-
tion or nerve degeneration rom mesh contact is 4. Regarding surgical management o post-
the most common cause. herniorrhaphy neuralgia, which o the ollowing is
correct?
2. Regarding the management o chronic groin pain A. ailored neurectomy is more ef ective than triple
af er hernia surgery, which o the ollowing is neurectomy at decreasing symptoms postopera-
correct? tively.
A. Imaging studies (ultrasound, C scan, and/or B. riple neurectomy is ef ective at eliminating pain
MRI) are suggested to rule out underlying causes. in upwards o 80% o patients, making this the
B. Peripheral nerve block should be attempted to most ef ective surgical therapy.
con rm diagnosis.

http://surgerybook.net/
74 G EN ERAL S U RG ERY EXAM I N ATI O N AN D BO ARD REVI EW

C. Mesh explantation alone is an e ective strategy 2. E. Chronic groin pain, or chronic posthernior-
and has been shown to be superior to neurec- rhaphy inguinal pain, a er hernia surgery is diag-
tomy with or without mesh explantation. nosed by chronic pain at site or region o prior
D. Patients with pre-existing pain hyper-sensitiza- hernia repair that persists postoperatively or over
tion are ideal candidates or neurectomy. three months that cannot be attributed to another
cause. T e prevalence o postoperative chronic groin
5. Regarding intentional division of the ilioinguinal pain varies rom study to study. A large Swedish sur-
nerve at time of initial herniorrhaphy, which of the vey including 2500 patients, noted 14% o patients
following is correct? had li estyle limiting groin pain, and 30% chronic
A. T ere is no di erence in sensory loss between pain that didn't hinder their li estyle. A smaller
division and preservation o the nerve. series noted 1.5% o patients had moderate to severe
B. A signi cant decrease in postoperative chronic pain at ve years. Interestingly, it is more common
pain is seen when the ilioinguinal nerve is inten- in younger patients, with persistent pain in 58% o
tionally divided during initial surgery. patients under the age o 40 and only 14% o those
C. T ere is no signi cant di erence in decreasing older than age 40.
chronic postoperative groin pain. Upon presentation o a patient you suspect has
D. T ere is less debilitating pain with routine divi- chronic postoperative groin pain, anti-in ammatory
sion o the ilioinguinal nerve. treatment should be attempted but usually has little
E. T ere is less pain i the nerve is clipped versus e ect on neuropathic pain. In this instance, an ilioin-
being divided by electrocautery. guinal nerve block can help con rm diagnosis and is
indicated. Additionally, nerve ablation with phenol or
radio requency ablation is used. Patients who ail less
ANSWERS
invasive means o treatment should undergo triple
1. A. Chronic postoperative groin pain is elt to neurectomy.
be secondary to neuropathic pain rom aberrant Non-neuropathic pain must be excluded. T is
nerve conduction as a result o either primary or is done with history, physical, and imaging studies.
secondary nerve injury. ypically, it is ongoing T e ideal imaging study has not been determined by
pain, which is di icult to manage. Initial postoper- randomized trials. In general, ultrasound is the least
ative pain is due to in lammatory cytokine release expensive test with minimal risk to the patient. How-
and nociceptive mechanisms. Nocioceptive pain is ever, C and MRI can provide a better representation
pain elt via neural pathways in which tissue dam- o the location o mesh, location o neuronal struc-
age surrounding the nerves is the stimulus. his tures, and presence o recurrent hernias.
type o pain typically resolves over 6 weeks and
is amendable to anti-in lammatory medications 3. A. An injury to the emoral branch o the lateral em-
such as NSAIDS. oral cutaneous nerve causes lack o sensation to the
Primary nerve injury is de ned as direct nerve lateral thigh and not the medial thigh as mentioned
injury and can occur during hernia repair in mul- in the answer. T is nerve is seen in laparoscopic
tiple ways. During dissection, complete or partial hernia repairs and is not encountered during open
nerve transection can occur later orming a neu- inguinal hernia repair. Other nerves encountered
roma. Additionally, handling o the nerve can result laparoscopically include the lateral emoral cutane-
in crushing, stretching or burns rom cautery. Most ous nerve, the ilioinguinal nerve lateral to the inter-
commonly, the nerve is incidentally entrapped with nal ring, the iliohypogastric (which cannot be seen
mesh, suture and/or staple. but could be injured with mesh xation), the genital
Secondary nerve injury is de ned as nerve degen- branch o the genito emoral nerve, and the emoral
eration/demyelination rom an in ammatory pro- nerve. T e so called triangle o pain is de ned as the
cess. It is not as common as primary nerve injury iliopubic tract superiolaterally, the spermatic vessels
in the pathogenesis o postherniorrhaphy neuralgia. posteriomedially, and the re ected peritoneal edge
Secondary nerve injury is elt to be rom meshoma, laterally. T is contains the genito emoral nerve and
excessive scar tissue or contact with mesh not involving the lateral emoral cutaneous nerve. Minimization o
entrapment. nerve injuries laparoscopically is achieved by avoiding

http://surgerybook.net/
C H AP TER 2 0 IN G U I N AL N EU RALG IA 75

tack placement in erior to the iliopubic tract later- 4. B. Given the signi cant morbidity and dif culty o
ally beyond the external iliac artery. the operation, neurectomy is reserved or patients
For open inguinal hernia repairs, the ilioinguinal who ail pain management strategies. Patients with
nerve is the most common cause o pain. Injury to pre-existing pain syndromes or hyper-sensitization
this nerve can also cause ipsilateral scrotal, thigh, are not ideal surgical candidates. O the operative
and penis numbness as well as loss o the cremasteric strategies, triple neurectomy o the ilioinguinal, ili-
re ex. Injury to the genital branch o the genito emo- ohypogastric and genital branch o the genito emo-
ral nerve can less commonly cause pain. More com- ral nerve has been shown to be the best at a surgical
monly, scrotal sensation and lack o cremasteric cure. T ere have been a ew small trials studying suc-
re ex on the ipsilateral side are seen. cess rates or triple neurectomy, the largest looking

He s s e lba chs tria ngle

Epiga s tric ve s s e ls

Indire ct inguina l he rnia


Re ctus m.
Inguina l liga me nt
Dire ct inguina l
he rnia

P ubic tube rcle

Fe mora l he rnia

Va s de fe re ns
Tria ngle of pa in
Tria ngle of doom

Ilia c ve s s e ls Te s ticula r
ve s s e ls

Ing uinal anato my vie w,


pre -pe rito ne al appro ac h

at 225 patients. O these, 80% reported resolution or nerve contact with the mesh not associated with
o pain, 15% had transient pain, and only 2 patients entrapment, or meshoma by imaging is suspected. No
reported no improvement. randomized controlled trials have compared triple
ailored neurectomy could be bene cial as it is neurectomy with and without mesh explantation; but
less morbid, leaving the patient with less sensory loss. i mesh removal alone does not relieve the pain then
Although tailored neurectomy has not been com- proceeding with a third operation in the inguinal
pared to triple neurectomy, and only been studied in region may result it a very dif cult dissection, inabil-
small studies, it was ound to provide complete pain ity to identi y all 3 nerves, and worsening pain.
relie in only 54% o patients, giving partial relie
in 24% o patients and leaving 24% with no bene t. 5. C. Intentional ilioinguinal nerve division dur-
Mesh explantation alone is not likely to be an ing herniorrhaphy has been postulated to decrease
e ective strategy or the treatment o chronic post- chronic groin pain. Its bene ts have been studied in
herniorraphy neuralia unless secondary nerve injury multiple randomized control trials. It is clear that
either by excessive scar ormation over the mesh operative division will decrease sensation along the

http://surgerybook.net/
76 G EN ERAL S U RG ERY EXAM IN ATIO N AN D BO ARD REVI EW

distribution o the nerve (the groin and hemiscro- randomized controlled trials. World J Surg. October 2012;
tum). However, no statistically signi cant advan- 36(10):23119.
tage at decreasing post-herniorrhaphy neuralgia at Hakeem A, Shanmugam V. Current trends in diagnosis and
management o post-herniorraphy chronic groin pain.
1month, 6 month, and 1 year ollow ups has been World J Gast Surg. 2011;3:73.
shown in large studies and meta-analysis o over Kehlet H, Jensen S, Wool CJ. Persistent postsurgical pain:
1200 patients. Risk actors and prevention. Lancet 2006; 367:1618.
Malangoni MA, Gagliani RJ. Hernias. In: ownsend CM,
Beauchamp RD, Evers BM, Mattox K, eds. Sabiston ext-
BIBLIOGRAPHY book of Surgery: T e Biological Basis of Modern Surgical
Amid PK. Causes, prevention, and surgical treatment o Practice. 19th ed. Philadelphia, PA: Elsevier Saunders;
postherniorrhaphy neuropathic inguinodynia triple neu- 2012:111440.
rectomy with proximal end implantation. Hernia. 2004; Picchio M, Palimento D, Attanasio U, Matarazzo PF, Bambini
Dec; 8(4):3439. C, Caliendo A. Randomized controlled trial o preservation
Bjurstrom MF, Nicol AL, Amid PK, Chen DC. Pain control or elective division o ilioinguinal nerve on open inguinal
ollowing inguinal herniorrhaphy: Current perspectives. hernia repair with polypropylene mesh. Arch Surg. July
Jour of Pain Research. 2014;7:27790. 2004;139(7):7558.
Franneby U, Sandblom G, Nordin P, Nyren O, Gunnarsson U. Poobalan AS, Bruce J, King PM, Chambers WA, Krukowski
Risk actors or long-term pain a er hernia surgery. Ann ZH, Smith WC. Chronic pain and quality o li e ollowing
Surg. 2006;244:212. open inguinal hernia repair. Br J Surg. 2001;88:1122.
Hsu W, Chen CS, Lee HC, Liang HH, Kuo LJ, Wei PL, am Reinpold WM, Nehis J, Eggert A. Nerve management and
KW. Preservation versus division o ilioinguinal nerve on chronic pain a er open inguinal hernia repair: A prospec-
open mesh repair o inguinal hernia: A meta-analysis o tive two phase study. Ann Surg. 2011;254:163.

http://surgerybook.net/
21
Hiatal and Paraesophageal Hernia

Steven M. Henriques

A 63-year-old emale presents with a history o gastroe- 5. T e patient undergoes an uneventful diaphragm
sophageal re ux disease (GERD). She takes a PPI twice repair and a Nissen fundoplication. wo years post-
daily and complains o having more regurgitation over operatively the patient presents with a recurrence
the past 6 months. She has a history or diabetes mel- of her symptoms. On evaluation, it is noted that
litus and hypertension. She has no history o alcohol the Nissen fundoplication is slipped. What is an
or tobacco use. She is otherwise in good health with no alternative for repair?
other problems. Her BMI is 38 kg/m 2. A. Colles gastroplasty
B. Roux-En-Y gastric bypass
1. What is the most likely cause of her symptoms?
C. Ivor-Lewis esophagectomy
A. Gastroparesis D. Esophagomyotomy
B. Esophageal cancer
C. Gastric ulcers
D. Hiatal hernia ANSWERS
E. Obesity 1. D. Hiatal hernia is quite common in the population
with up to 60% o the population having such hernias.
2. What should be the next appropriate test?
Approximately 9% are symptomatic. Regurgitation
A. CXR can be the only symptom but at times some will have
B. Manometry cardiac and pulmonary symptoms. Esophageal cancer
C. pH monitoring must be on the di eretial diagnosis with anyone pre-
D. Endoscopy senting with regurgitation and a history o GERD. It
is unlikely to be cancer as the patient has not had any
3. What is the most common type of hiatal hernia?
dysphagia or weight loss. Gastric ulcers are associated
A. ype I with GERD but will usually present with abdominal
B. ype II pain. Diabetic neuropathy can a ect the intestinal
C. ype III tract and can cause gastroparesis. Patients with this
D. ype IV condition can have regurgitation, abdominal ullness,
and pain. T is patient has no other symptoms con-
4. Regarding the etiology of hiatal hernias most are?
sistent with diabetic periperal neuropathy. Obesity is
A. Congenitally acquired a risk actor to re ux disease but the most common
B. Have no amilial hereditary pattern cause in this patient is likely a hiatal hernia.
C. Similar to a Bochdalek hernia
D. Result rom a weakening o the phrenoesopha- 2. D. T e workup has to exclude other pathologies.
geal ligament Endoscopy is essential to the evaluation o patients

http://surgerybook.net/
78 G EN ERAL S U RG ERY EXAM IN ATIO N AN D BO ARD REVI EW

presenting with GERD, to deteremine the extent o o the gastroesophageal junction. Most cases o hiatal
esophagitis, and to determine the extent o a hiatal hernia are acquired rather than congenital. A small
hernia, to rule out maligancy. Manometry can be used number o cases o amilial hiatal hernias have been
to evaluate the LES and look or motility disorders. shown. Bochdalek hernias are congenital hernias
pH monitoring is the gold standard or diagnosing involving the right posterior diaphragm usually
and quanti ying acid re ux. Impedance pH can also ound in children, it is extremely rare to f nd in the
be per ormed to discern the di erence between non- adult population with less than 100 published cases.
acid and acidic re ux. Additionally, the use o either
C or a swallow study can be used or evaluation o 5. B. A good alternative or ailed undoplications is a
the motility o the stomach and can help decipher i Roux-En-Y Gastric Bypass. Kim et al. showed that
there is a need or pyloroplasty or pyloromyotomy. at 11 months, 93.3% o patients were symptom- ree.
Colles Gastroplasty is used as a lengthening proce-
3. A. ype I (sliding) hernia: Upward herniation dure and has no role in the management o revision
o the cardia in the posterior mediastinum, the GE on its own. Esophagectomy has no role in the revi-
junction migrates above the diaphragm. ype I hiatal sion o slipped undoplication. Esophagomyotmy is
hernias are the most common at 90%. used to treat achalasia to release the pressure on the
ype II (paraesophageal) hernia: T e GE junc- LES. A slipped Nissen re ers to the stomach that slips
tion remains in the normal anatomical position, the under the wrap creating an hourglass shape de orm-
undus herniates through the hiatus. ity that can be seen on UGI. T is is di erent rom a
ype III (mixed) hernia: Characterized by an recurrence where the stomach re-herniates into the
upward herniation o both the cardia and the gastric chest.
undus.
ype IV hiatal hernia: An additional organ, usu- BIBLIOGRAPHY
ally the colon but could involve the spleen or liver, Alam U, Asghar O, Malik RA. Diabetic gastroparesis: thera-
herniates as well. peutic options. Diabetes T er. August 2010;1(1):3243.
Baglaj SM, Noblett HR. Paraoesophageal hernia in children:
amilial occurrence and review o the literature. Pediatr
Surg Int. 1999;15:8587.
Curci JA, Melman LM, T ompson RW, Soper NJ, Matthews BD.
Elastic f ber depletion in the supporting ligaments o the
gastroesophageal junction: a structural basis or the devel-
opment o hiatal hernia. J Am Coll Surg. 2008;207:1916.
Kim M, Navarro F, Eruchalu CN, Augenstein VA, Heni ord B ,
Ste anidis D. Minimally invasive Roux-en-Y gastric bypass
or undoplication ailure o ers excellent gastroesophageal
re ux control. Am Surg. July 2014;80(7):696703.
Kohn GP, Price RR, DeMeester SR, Zehetner J, Muensterer
OJ, Awad Z, et al. Guidelines or the management o hiatal
hernia. SAGES Guidelines Committee. Surg Endosc.
December 2013;27(12):440928.
Maish MS. Esophagus. In: ownsend CM, Beauchamp RD,
Evers BM, Mattox K, eds. Sabiston extbook of Surgery:
T e Biological Basis of Modern Surgical Practice. 19th ed.
Philadelphia, PA: Elsevier Saunders; 2012:10121066.
Patti MG, Fisichella PM, Perretta S, et al. Impact o minimally
invasive surgery on the treatment o esophageal achalasia:
a decade o change. J Am Coll Surg. 2003;196:698705.
Petersen RP, Pellegrani CA, Oelschlager BK. Hiatal hernia
and gastroesophageal re ux disease. In: ownsend CM,
Beauchamp RD, Evers BM, Mattox K, eds. Sabiston extbook
of Surgery: T e Biological Basis of Modern Surgical Practice.
4. D. Cephalad migration o the gastroesophageal junc-
19th ed. Philadelphia, PA: Elsevier Saunders; 2012:106786.
tion may result rom weakening o the phrenoesoph- Salstio R, Nabais C, Paredes B, Sousa FV, Porto E, Fradique C.
ageal ligament. Depletion o elastin f bers leads to Association o intestinal malrotation and Bochdalek hernia
stretching o the ligament and proximal displacement in an adult: a case report. BMC Res Notes. May 2014;13;7:296.

http://surgerybook.net/
22
Achalasia

Mary . ODonnell and E. Matthew Ritter

A 42-year-old male or whom you recently per ormed a


success ul inguinal hernia repair, presents to your clinic
with a chie complaint o increasing di culty swallow-
ing some solids and occasionally liquids. He reports
this has been worsening or the last 5 months. He has
been treated by his primary care physician with daily
omeprazole, but this does not seem to be improving his
symptoms. Since you did such a good job in repairing
his hernia, he is now coming to you or advice on this
problem. He appears healthy, his vital signs are normal,
and he is maintaining his normal weight.

1. T e most appropriate initial study to help you sort


through the di erential diagnosis o this patients
dysphagia is?
A. Contrast esophagram
B. Esophageal manometry
C. Esophagogastroduodenoscopy
D. H. pylori testing
E. Chest X-ray 3. Upper endoscopy reveals no evidence o
esophagitis, but some ood is present in the distal
2. A contrast esophagram image is shown to the right. esophagus. Manometry shows aperistalsis and a
What additional study or studies should be ordered non-relaxing LES. Which initial treatment o ers
next in the diagnostic workup o this patient? the patient the best chance o long term-relie ?
A. Esophageal manometry A. Pneumatic dilation o the LES
B. Esophagogastroduodenoscopy (EGD) B. Calcium channel blocker therapy
C. 24-hour pH monitoring C. Esophageal myotomy with undoplication
D. 24-hour pH monitoring and EGD D. Botulinum toxin injection at LES
E. Esophageal manometry and EGD E. Esophagectomy

http://surgerybook.net/
80 G EN ERAL S U RG ERY EXAM IN ATIO N AN D BO ARD REVI EW

4. Suppose in this same patient, that the results o


Di erential Diagnosis Conf rmatory est
the manometry do not show achalasia. Instead,
the standard esophageal manometry shows 10/10 Neoplasm EGD
normal propagated swallows with a mean distal Eosinophilic Esophagitis
esophageal amplitude pressure o 293 mm Hg. T e
GERD
lower esophageal sphincter pressure is normal and
relaxes completely. T e results o ambulatory pH Schatzskis Ring/Stricture
monitoring and EGD were both normal. What is Esophageal Ulcer
the most likely diagnosis? ypical Re ux 24 hr pH monitoring
A. Achalasia Atypical Re ux
B. Pseudoachalsia
Achalasia Manometry
C. Di use esophageal spasm
D. Nutcracker esophagus DES
Nutcracker Esophagus
ANSWERS Connective issue Disorder

1. A. Contrast esophagram o ers the most use ul


in ormation in initially sorting through this di er- 3. C. T is patient has achalasia. In the absence o an
ential. T e esophagram shown essentially eliminates obstructing entity (neoplasm, hiatal hernia, diverticuli)
diverticula and a hiatal hernia. retained ood in the esophagus is suspicious or acha-
lasia. T e diagnosis is con rmed by manometry. In
2. B. T is esophagram would typically be ollowed patients with achalasia, the manometry demonstrates
up with an EGD. While the esophagram shown an esophagus in which there is complete absence o
makes the diagnosis o a diveritcula or an hiatal peristalsis and an LES pressure that is normal to mod-
hernia less likely, there are still several diagno- erately elevated, but ails to completely relax. Although
ses that are not ruled out or con rmed. An EGD a hypertensive LES and an LES that ails to completely
would be needed to exclude anatomical causes or relax are of en associated with achalasia, only the
dysphagia, such as neoplasm or stricture. T e EGD complete absence o peristalsis is required or the
would also be used to document any evidence o diagnosis. T e pathophysiology o achalasia is loss o
re ux or eosinophilic esophagitis. I there is no ganglion cells in the myenteric plexus and interrup-
re ux seen on EGD, a 24-hour pH study could tion o inhibitory vagal nerve innervation.
con rm the presence o unctional re ux disease. T e treatment or achalasia requires relaxing the
I the 24-hour pH study is normal, then a manom- hypertensive smooth muscles o the LES. In general,
etry study could be done to rule out motility dis- that can be done surgically by dividing the muscles
orders like achalasia, di use esophageal spasm, o the LES, injecting botulinum toxin endoscopi-
connective tissue disorders, and nutcracker esoph- cally, or dialating the LES using a balloon. T is is also
agus. Even i the 24-hour pH study is con rma- done endoscopically. Surgical treatment with myot-
tory, though, an esophageal manometry should be omy provides long-term treatment o achalasia with
done to rule out a concominant underlying motil- a high success rate. Calcium channel blockers have
ity disorder. By skipping the EGD and proceeding shown inconsistent success and do not have a role in
directly to manometry, a diagnosis, such as pseu- achalasia treatment. Botulinum toxin usually requires
doachalasia secondary to a distal anatomic partial repeated interventions every 6 to 12 months, as does
obstruction, may be missed. dilation. Both eventually become less e ective. T ese
may be good options in persons who are poor opera-
tive risks or in those who do not want surgery.
Di erential Diagnosis Conf rmatory est Esophagectomy would be required in the setting
Neoplasm Esophagram o cancer and is indicated as a last resort i multi-
ple myotomies ail. T e POEM or per oral endo-
Diverticulae
scopic myotomy technique involves dividing the LES
Hiatal Hernia muscle endoscopically. T is has the advantage o

http://surgerybook.net/
C H AP TER 2 2 AC H ALAS I A 81

avoiding most surgical risks. T e long-term success DES would be characterized by high pressures through-
rate, though, is unknown and is not yet considered out the esophagus (2550 mm Hg) and poor peristalsis.
the equivalent o a surgical myotomy. Psuedoachalasia would show a slightly hypertensive but
relaxing LES and abnormal but not absent peristalsis.
4. D. Manometry is used to rule out esophagility motil- Connective tissue disorders, like scleroderma, would be
ity disorders. Not only does it measure how well ood hallmarked by poor peristalsis and a normal LES.
travels down the esophagus into the stomach, it also
measures the pressure inside the esophagus and the BIBLIOGRPAHY
LES. T e normal pressure o an LES is 10 to 15 mm Castell, Donald O. Linda Diederich, June A. Castell. Esophageal
Hg. Patients with GERD will of en have a hypoten- Motility and pH esting, echnique and Interpretation, 3rd
sive LES with a pressure o around 5 to 8 mm Hg and edition. Highlands Ranch, CO: Sandhill Scienti c, Inc.; 2000.
Guidelines or the Surgical reatment o Esophageal Achalasia.
normal to moderately abnormal peristalsis. T e low Society o American Gastrointestinal and Endoscopic Sur-
LES pressure is thought to allow gastric contents to geons (SAGES) Web site. Available rom http://www.sages.
re ux back into the esophagus. org/publications/guidelines/guidelines- or-the-surgical-
wenty- our-hour ambulatory manometry can treatment-o -esophageal-achalasia/ Published May 2011.
be used to diagnose spastic disorders such as Di use Accessed June 2014.
Ste anidis, D. et al. Guidelines for the Surgical reatment of
Esophageal Spasm (DES), nutcracker esophagus, or
Esophageal Achalasia, Society o American Gastrointesti-
hypercontractile esophageal motility disorder is a rare nal and Endoscopic Surgeons (SAGES), May 2011.
cause o dysphagia. T e nutcracker esophagus is char- utuian R, Castell DO. Review article: esophageal spasm
acterized by very high LES pressures (> 50 mm Hg) diagnosis and management. Aliment Pharmacol T er. 2006;
during swallowing with otherwise normal peristalsis. 23(10):13931402.

http://surgerybook.net/
23
Gastric Cancer

Brian J. Pottor and Farah A. Husain

A 68-year-old male with a history o hypertension and D. Diagnostic laparoscopy


hypercholesterolemia presents to his primary care phy- E. riple-phase helical computed tomography (C )
sicians o ce with a chie complaint o worsening epi- scan
gastric pain and weakness. T e pain is improved with
oral intake, especially milk-based products. T e patient 3. T e EUS suggests a 3N0 lesion. T e most appro-
has been treating his pain with naproxen. In the o ce, priate next step would be:
the patient is non-toxic with normal vital signs. His A. Neoadjuvant therapy
physical examination reveals mild epigastric tenderness B. Proximal gastrectomy with negative margins
with deep palpation. Serum hemoglobin was 8.3 g/dL. (R0) only
Fecal occult blood testing was positive. T e patient C. otal gastrectomy
underwent colonoscopy, which was normal. Esophago- D. otal gastrectomy with splenectomy and distal
gastroduodenoscopy (EGD) revealed a 2.5 cm ulcerated pancreatectomy
lesion with elevated, irregular borders 5 cm distal to the E. Esophagogastrectomy with colonic interposition
gastroesophageal junction. gra

1. Appropriate management o the ulcer includes? 4. T e f nal pathology revealed a 4N1 lesion with
A. Observation negative margins. T e patient should next receive:
B. Cessation o naproxen and begin sucral ate and A. No additional therapy
a proton-pump inhibitor with repeat EGD in B. Imatinib
3 months C. External-beam radiation only
C. Biopsy the ulcer D. Fluorouracil-based chemotherapy only
D. Proximal gastrectomy E. External-beam radiation and uorouracil-based
E. otal gastrectomy chemotherapy

2. Final pathology reveals a poorly di erentiated 5. Which o the ollowing describes the association
adenocarcinoma. T e most sensitive preoperative between Irishs node and gastric cancer?
examination to determine and N stage is: A. An anterior mass palpable on digital rectal
A. Positron emission tomography (PE ) scan examination
B. Endoscopic ultrasound (EUS) B. A metastatic le supraclavicular lymph node
C. Magnetic resonance imaging (MRI) with gado- C. An ovarian mass rom metastatic tumor
linium D. Metastatic le axillary lymph node
E. Umbilical mass suggestive o metastatic gastric
cancer

http://surgerybook.net/
C H AP TER 2 3 G As TRi C C An C ER 83

ANSWERS
1. C. Historically, biopsy o gastric ulcers was uni orm
practice throughout medical and surgical disciplines
since there was a 5% to 11% attendant risk o malig-
nancy. However, data now suggest that the incidence
o gastric cancer is decreasing, thereby rendering
mandatory biopsy o all gastric ulcers unnecessary.
When gastric ulcers have eatures suggestive o
malignancy such as elevated irregular olds, associa-
tion with a polypoid or ungated mass, and abnor-
mal adjacent mucosal olds, then biopsy is warranted
(Fig. 23-1). Several biopsies, typically 6 or more,
are necessary to minimize the alse negative risk. I
benign ulcers are diagnosed, then EGD is repeated in Figure 23-2 Ex vivo surgical specimen containing giant gas-
6 weeks to ensure resolution. All ulcers should be ol- tric ulcer (white arrow). Note the ulcerated, necrotic center
lowed and biopsied until complete resolution occurs. with irregular circum erential mucosal olds. Final pathology
I malignancy is detected, then urther work-up with revealed a poorly dif erentiated adenocarcinoma.
potential operative intervention is pursued.
Ulcers with a diameter o 3 cm or greater are termed malignant lymph nodes. Increasing stage directly
giant gastric ulcers (Fig. 23-2). T ese large ulcers correlates with increased risk o nodal and distant
harbor an underlying malignancy in 30% o lesions. metastasis (> 80% likelihood o nodal metastasis in
Given the higher incidence o malignancy, per oration, 3 disease versus < 5% in stage 1 m).
and bleeding surgical treatment is warranted. C remains an important preoperative tool to
2. B. EUS is important in preoperative locoregional evaluate or metastatic disease. I metastatic disease
staging or gastric cancer. It is currently the best is present, an unnecessary operation can be avoided.
imaging modality or assessing both tumor depth and staging accuracy with C approaches 80% (66%
nodal invasion. Spatial resolution o 0.1 mm can be to 77%). N stage determination is variable with a
achieved with EUS. staging accuracy ranges rom wide range o 25% to 86%. Small gastric tumors and
60% to 90%, whereas N staging accuracy ranges rom metastases less than 5 mm can be missed on C .
50% to 80%. EUS is better at identi ying 1 (80%) and C , MRI, and PE scanning show promise or pre-
3 (90%) lesions as opposed to 2 (38.5%). EUS is not operative staging, but have yet to become standard
reliable at delineating between individual benign and o care.
Routine diagnostic laparoscopy to minimize
unnecessary operations has become a less popular
pre-resection strategy. However, diagnostic lapa-
roscopy still has a role in advanced gastric cancer.
Power et al., in 2009, evaluated patients with known
gastric cancer without obvious metastitc disease and
strati ed them into low-risk ( 12, N0) and high-
risk ( 34, N+ , or both) groups based on EUS. Both
groups underwent diagnostic laparoscopy, which
identi ed M1 disease in 20.5% o the high-risk
patients and 4% o the low-risk patients. T e study
concluded that laparoscopy can be avoided in patients
with EUS early stage cancer, whereas more advanced
gastric cancers would bene t rom diagnostic lapa-
roscopy to rule out occult metastatic disease. When
Figure 23-1 Endoscopic view o an ulcerated ungating gastric diagnostic laparoscopy is per ormed, peritoneal
mass (white arrow) with irregular mucosal olds (black arrow). lavage cytology should be obtained as positive results

http://surgerybook.net/
84 G En ERAL s U RG ERY EXAM i n ATi O n An D BO ARD REVi EW

can alter urther therapy. Diagnostic laparoscopy, thereby increasing the risk o microscopic residual
however, does not address the or N stage. positive margin (R1 resection). In order to minimize
the risk o leaving microscopic disease or recurrence,
3. A. Although the patient will ultimately need an a 5 to 6 cm margin is considered acceptable or an R0
operation, the MAGIC trail demonstrates that the resection. Newer studies emerging rom Japan sug-
patient will bene t rom neoadjuvant therapy instead gest that smaller proximal resection margins o 2 to
o proceeding straight to the operating room, unless 3 cm are adequate or 1 lesions. I the patient went
the patient is hemorrhaging rom the mass resulting straight to surgery, total gastrectomy is pre erred as
in hemodynamic instability. Neoadjuvant therapy the tumor is within 5 cm o the gastroesophageal
consisting o epirubicin, cisplatin, and uorouracil is junction. Esophagogastrectomy is unnecessary when
recommended or patients with 2 lesions or higher. the gastroesophageal junction has no direct tumor
T e bene ts rom the preoperative therapy are to involvement and surgical margins exceed 5 cm.
reduce tumor size and stage, eliminate micrometas- Assessing nodal disease at the time o operation
tases, improve tumor-related symptoms, and deter- can be di cult. A minimum o 15 lymph nodes is
mine whether tumors are sensitive to chemotherapy. recommended or staging. Most surgeons tend to
Gastric adenocarcinoma exists as two distinct enti- remove the perigastric lymph nodes (D1 resection).
ties: dif use and intestinal type. able 23-1 summarizes In countries like Japan where gastric cancer has a
these distinct subtypes. Controversies surrounding the higher prevalence, a more aggressive D2 lymphad-
surgical management o gastric adenocarcinoma enectomy is requently employed harvesting lymph
include: adequacy o surgical margins, need or resec- nodes along the celiac trunk and its named branches,
tion o adjacent structures (i.e. spleen and distal pan- the middle colic artery, the superior mesenteric
creas), and extent o lymphadenectomy. Dif use type artery, and the periaortic area. Several studies have
gastric adenocarcinoma spreads in the submucosa, demonstrated prolonged survival with the more
aggressive (D2) lymphadenectomy. T is is thought
able 23-1 SUMMARY COMPARING IN ES INAL to be related to better locoregional disease control.
AND DIFFUSE GAS RIC ADENOCARCINOMA A recent randomized trial comparing D1 versus D2
lymphadenectomy did not reveal a signi cant di -
Characteristic Intestinal Di use erence in long-term survival. Accordingly, more
Age Older Younger studies regarding the extent o lymphadenectomy
are required be ore a long-term endorsement o this
Gender Male > Female Male = Female
more aggressive strategy can be made.
Metastatic Hematologic Lymphatic, sub- Removal o adjacent structures (i.e., distal pan-
route mucosal spread creas and spleen) con er no survival bene t and
which can result
in a thickened,
actually increase morbidity and mortality. Resection
non-distensible o these adjacent structures should be reserved or
stomach known as primary tumor invasion.
linitis plasticans
Site o Liver Peritoneum 4. E. T e nal pathology revealed Stage III gastric can-
metastasis cer. Given the high rate o locoregional ailure (40% to
Risk actors Atrophic gas- CHD-1 mutation, 70%) and a 5-year survival rate o 20% to 28%, adju-
tritis, intestinal obesity vant therapy is recommended. T is recommendation
metaplasia, originates rom the Intergroup rial 0116, which dem-
Helicobacter onstrated a bene t or those patients with advanced
pylori in ection, gastric cancer undergoing curative resection combined
and diet high in
salt, smoked, and with postoperative uorouracil-based chemotherapy
preserved oods and radiation. T e CLASSIC trial demonstrated sur-
vival advantages using an adjuvant chemotherapy
Cellular Glandular Lamina propria
etiology gastric mucosa therapy regimen o capecitabine and oxaliplatin.
Resection without adjuvant therapy resulted in
Prognosis Better Poor
decreased survival when compared with those who

http://surgerybook.net/
C H AP TER 2 3 G As TRi C C An C ER 85

received postoperative chemoradiation. Palliation Committee ASoP, Banerjee S, Cash BD, et al. T e role o
can be achieved with either external-beam radia- endoscopy in the management o patients with peptic
tion or chemotherapy, but local control and long- ulcer disease. Gastrointestinal endoscopy. 2010;71:6638.
De Angelis C, Pellicano R, Man re SF, Rizzetto M. Endoscopic
term survival are poor. Imatinib is a tyrosine-kinase ultrasound in the 2013 preoperative evaluation o gastric can-
inhibitor currently used or gastrointestinal stromal cer. Minerva gastroenterologica e dietologica. 2013;59: 112.
tumors and other malignancies. Degiuli M, Sasako M, Ponti A, et al. Randomized clinical trial
comparing survival a er D1 or D2 gastrectomy or gastric
5. D. In general, physical ndings portend advanced cancer. BJS. 2014;101:2331.
disease. Patients are typically cachectic and jaundiced Joensuu H, Eriksson M, Sundby, Hall K, et al. One versus three
when nodal metastatic disease obstructs the com- years o adjuvant imatinib or operable gastrointestinal stro-
mal tumor: a randomized trial. JAMA. 2012;307:126572.
mon bile duct. Irishs node is an enlarged lymph node Macdonald JS, Smalley SR, Benedetti J, et al. Chemoradiother-
within the le axilla. A prerectal mass palpable on apy a er surgery compared with surgery alone or adeno-
digital rectal examination is a Blumers shel sugges- carcinoma o the stomach or gastroesophageal junction.
tive o a drop metastasis. Virchows node, also known NEJM. 2001;345:72530.
as roisiers sign, re ers to carcinomatous involvement McLoughlin JM. Adenocarcinoma o the stomach: a review.
Proceedings. 2004;17:3919.
o the le supraclavicular lymph nodes at the junc-
Naeem AN, Mu eed S, Makary MA. T e Stomach. In: Cam-
tion o the thoracic duct with the subclavian vein. eron JL, Cameron AM, eds. Current Surgical T erapy. 10th
Krukenberg tumors are ovarian masses rom meta- ed. Philadelphia, PA: Elsevier; 2011:6392.
static gastric cancer. T e Sister Mary Joseph node is a Pavlidis E, Pavlidis E , Sakantamis AK. T e role o laparo-
periumbilical nodule suggestive o carcinomatosis. It scopic surgery in gastric cancer. JMAS. 2012;8:358.
re ects tumor extension rom the alci orm ligament. Power DG, Schattner MA, Gerdes H, et al. Endoscopic ultra-
sound can improve the selection or laparoscopy in patients
with localized gastric cancer. Journal o the American College
BIBLIOGRAPHY o Surgeons. 2009;208:1738.
Al-Re aie WB, Abdalla EK, Ahmad SA, Mans eld PF. Gastric Shin D, Park SS. Clinical importance and surgical decision-
Cancer. In: Fieg BW, Berger DH, Fuhrman GM, eds. T e making regarding proximal resection margin or gastric
M D Anderson Surgical Oncology Handbook. 4th ed. Phila- cancer. WJGO. 2013;5:411.
delphia, PA: Lippincot Williams & Wilkins; 2006:20540. Stolte M, Seitter V, Muller H. Improvement in the quality o
Arrington AK, Nelson R, Patel SS, et al. iming o chemother- the endoscopic/bioptic diagnosis o gastric ulcers between
apy and survival in patients with resectable gastric adeno- 1990 and 1997An analysis o 1,658 patients. Zeitschrif
carcinoma. WJGNE . 2013;5:3218. ur Gastroenterologie. 2001;39:34955.
Bang YJ, Kim YW, Yang HK, et al. Adjuvant capecitabine Wu CW, Hsiung CA, Lo SS, et al. Nodal dissection or patients
and oxaliplatin or gastric cancer a er D2 gastrectomy with gastric cancer: a randomised controlled trial. T e Lancet
(CLASSIC): a phase 3 open-label, randomised controlled Oncology. 2006;7:30915.
trial. Lancet. 2012;379:31521.

http://surgerybook.net/
24
Gastrointestinal Stromal umors

William Cole

A 62-year-old man with hypertension was re erred B. Endoscopic ultrasound with FNA
to the general surgery clinic or urther evaluation o C. Percutaneous image-guided biopsy
chronic abdominal pain, bloating, and early satiety, D. Diagnostic laparoscopy with biopsy and perito-
which had been worsening over several months. He was neal washings
previously healthy except or hypertension controlled
with metoprolol and a history o inguinal hernia repair. 2. Biopsied tissue is positive or KI (CD117) upon
His last screening colonoscopy per ormed 2 years ago immunochemical staining. Which o the ollowing
was negative. On review o systems, he endorses sig- is true o this type o tumor?
ni cant atigue. Laboratory results are consistent with A. Gastrointestinal stromal tumor (GIS ) most
mild anemia. An abdominal C scan was obtained or commonly arises rom the stomach.
urther evaluation and revealed a large tumor o gastric B. T e most common subtype is epithelioid.
origin (pictured below). C. A positive stain or KI (CD117) is required to
make the diagnosis o GIS .
D. T e most common site o metastatic spread is
the peritoneum.
E. All tumors >1 cm should be considered poten-
tially malignant.

3. Further review o the C scan raises concerns


that this 6-cm tumor may involve the neck o the
pancreas. T ere is no evidence o distant metastatic
disease. Further therapy in this case should include:
A. Surgical resection with en-bloc removal o the
involved pancreas to achieve 1 cm negative
margins.
Reproduced with permission rom Lua S et al. Imaging o B. Neoadjuvant imatinib prior to surgical therapy.
gastrointestinal stromal tumor (GIS ). Clin Radiol. 2004; C. Avoidance o pancreatectomy by enucleation o
59:48798. the tumor.
D. An open rather than laparoscopic approach
1. T e best next step to de nitively diagnose this should be used.
lesion is: E. An extended lymphadenectomy should be per-
A. Abdominal MRI ormed.

http://surgerybook.net/
C H AP TER 2 4 G As TRo i n TEs Ti n Al s TRo m Al Tu m o Rs 87

4. Af er appropriate therapy, nal pathology returns lar role has not been established in GIS . Laparoscopic
with a GIS o gastric origin, 6 cm in greatest excision o the lesion may be per ormed without a
dimension with 15 mitoses per high-power eld. tissue diagnosis or a small tumor, but the goal in this
Which o the ollowing is true regarding this patient? case is resection to clear margins, rather than simply
A. Adjuvant therapy with imatinib will increase his to obtain tissue or diagnostic purposes and there ore,
chance o recurrence- ree and overall survival at endoscopic ultrasound is the best choice.
5 years.
B. I this lesion were in the small bowel, the progno- 2. A. T e presence o the c-kit receptor tyrosine kinase
sis would be better. on tumor cells, as in this case, is diagnostic o GIS .
C. T is patient is at low risk o tumor recurrence. However, it is noteworthy that about 5% o gastroin-
D. Five year overall survival or all GIS patients is testinal stromal cells tumors are KI -negative and only
about 50%. about 80% have a KI mutation. Other use ul histo-
logic markers include CD34 and smooth-muscle actin,
5. Which o the ollowing is true o gastrointestinal i KI negative GIS is suspected. T e most common
stromal tumors? histologic subtype is the spindle-cell variety (70%), ol-
A. Because they arise rom the mucosa, GIS s are lowed by epithelioid (20%) and mixed subtypes (10%).
easily identi ed at endoscopy. All tumors greater than 2 cm in size should be consid-
B. Surgical resection is o en appropriate or ered malignant, even in the absence o metastases on
patients with recurrent or metastatic GIS . initial work-up. T e most common site o metastatic
C. T ese tumors arise rom the smooth-muscle spread o GIS is the liver, ollowed by the omentum
cells o the intestinal wall. and peritoneum. I present, these metastases are o en
D. GIS tends to arise as a solitary lesion. identi ed by contrast enhanced C scanning. Metasta-
E. Abdominal pain is the most common clinical sis to the lymph nodes, lung, or other distant sites may
mani estation o GIS . occur, but this is quite rare. T us, extended surgical
lymphadenectomy is not indicated or these tumors.
Over hal o all GIS s arise rom the stomach, making
ANSWERS
it the most common primary site.
1. B. T e C slice shown demonstrates a large, well-
demarcated, heterogeneously enhancing mass that 3. B. When surgical morbidity can be reduced by its use,
appears to grow outward rom the wall o the stomach. preoperative therapy with imatinib, a receptor tyrosine
T ese ndings are characteristic o gastric GIS , kinase inhibitor, should be strongly considered. In this
although the di erential diagnosis includes gastric case, tumor down-staging could potentially eliminate
adenocarcinoma, carcinoid, lymphoma, or leiomyosar- involvement with the pancreas and obviate the need
coma, as well as tumors o pancreatic, renal, or adrenal or pancreatectomy. For localized GIS , surgical resec-
origin. GIS is a relatively uncommon neoplasm, with tion is indicated and is curative or low risk lesions. I
an incidence o about 7 per million population in the necessary to achieve an R0 resection, en-bloc removal
United States and Europe. T e bene ts o EUS include o involved organs outside the primary site is indi-
de ning the layer o stomach wall rom which the cated. However, there is no additional survival bene t
tumor originates, delineating its relationship to sur- to resection beyond microscopically negative margins.
rounding structures, and obtaining a tissue diagnosis An extended lymphadenectomy also o ers no bene t
transluminally, which avoids the risk o seeding a per- to the patient, as nodal metastasizes are uncommon
cutaneous biopsy tract. Percutaneous image-guided with GIS occurring about 1% o the time.
biopsy may result in intraperitoneal tumor spillage or Enucleation o the tumor risks violating its pseu-
hemorrhage as a result o the riable, vascular nature o docapsule which may result in intraoperative tumor
these tumors and is there ore less desirable. MRI o ers spillage, resulting in recurrence rates approaching
no additional bene t over C diagnostically, though it 100%. T ough laparoscopic surgery or GIS has
may provide more in ormation regarding the tumors not been prospectively evaluated, there is good ret-
relationship to surrounding tissues. Diagnostic lapa- rospective evidence to show adequate oncologic
roscopy with peritoneal washings or cytology has a outcomes with this approach. Rates o R0 resection
prognostic role in gastric adenocarcinoma, but a simi- between 97% and 100% and disease ree survival

http://surgerybook.net/
88 G En ERAl s u RG ERY EXAm i n ATi o n An D Bo ARD REVi EW

and overall survival rates o over 90%. In the past, an line therapy or metastatic or recurrent GIS and
open approach or tumors larger than 5 cm has been surgical resection is o en inappropriate due to high
recommended. Current guidelines indicate that lapa- rates o recurrence. However, some patients with
roscopy is appropriate or larger tumors, providing tumors response to imatinib and lesions that are elt
sound oncologic principles are maintained. to be completely resectable may bene t.

4. A. umor size, mitotic rate, and location are impor- BIBLIOGRAPHY


tant prognostic actors in GIS . umors with a size
D,ematteo RP et al. umor mitotic rate, size, and location
< 5 cm have a 5-year overall survival o about 70%. independently predict recurrence a er resection o pri-
T is drops to about 45% when tumor size is > 10 cm. mary gastrointestinal stromal tumor (GIS ). Cancer. 2008;
Similarly, about 75% o patients with < 5 per 112:60815.
high-power eld will survive 5 years, only 20% o Dematteo RP, et al. Results o tyrosine kinase inhibitor therapy
those with more than 5 per high-powered eld are ollowed by surgical resection or metastatic gastrointesti-
nal stromal tumor. Ann Surg. 2007;245:34752.
alive at 5 years. umors o gastric origin are more Dematteo RP, et al. Long-term results o adjuvant imatinib
avorable than those originating in the small bowel, mesylate in localized, high-risk, primary gastrointestinal
with survival rates o approximately 75% and 50%, stromal tumor. Ann Surg. 2013;258:4229.
respectively, a er 5 years. umor rupture be ore or Demetri GD, et al. NCCN task orce report: update on the
during surgery also portends a poor prognosis, as dis- management o patients with gastrointestinal stromal
tumors. J Natl Compr Canc Netw. 2010;8:S1S41.
cussed above. Given the mitotic rate o the tumor in
Emory S, Sobin LH, Lukes L, Lee DH, OLeary J. Prognosis
this case, the patient has a relatively poor prognosis. o gastrointestinal smooth-muscle stromal tumors: depen-
Adjuvant therapy with imatinib or 1 year has been dence on anatomic site. Am J Surg Pathol. 1999;23(1):827.
shown to increase recurrence ree survival by 15% Eisenberg BL, et al. Phase II trial o imatinib mesylate (IM)
and, i continued or 3 years, improve 5-year overall or advanced primary and metastatic/recurrent operable
survival by 10%. However, about hal o all patients gastrointestinal stromal tumor (GIS )early results o
R OG 0132. J Surg Oncol. 2009;99(1):427.
will develop resistance to the drug within 2 years Gervaz P, Huber O, Morel P. Surgical management o gastro-
o its initiation. For these patients, other tyrosine intestinal stromal tumors. Brit J Surg. 2009;96:56778.
kinase inhibitors (i.e. sunitinib) remain e ective sec- Gold JS, DeMatteo RP. Combined surgical and molecular ther-
ond line therapy. Historically, the overall survival or apy: the gastrointestinal stromal tumor model. Ann Surg.
all patients with GIS at 5 years has been about 50%. 2006;244:17684.
Hohenberger P, et al. Pattern o recurrence in patients with rup-
However, in the era o imatinib, the 5-year OS has
tured primary gastrointestinal stromal tumor. Brit J Surg.
improved to 84%, though survival varies markedly 2010;97;18549.
between patients with Stage 1 tumors (nearly 100%) Joensuu H, et al. One versus three years o adjuvant imatinib
versus more Stage 3 and higher tumors (22%). or operable gastrointestinal stromal tumor. JAMA. 2012;
307(12):126572.
5. D. Gastrointestinal stromal tumors are more likely to Joensuu H, Hohenberger P, Corless C. Gastrointestinal stromal
be solitary than multiple. T is stands in contrast to tumour. Lancet. 2013;382:97383.
Lee SD, et al. Prognostic signi cance o peritoneal washing
carcinoid tumors, which o en occur multiply. T ey
cytology in patients with gastric cancer. Br J Surg. 2012;
arise rom the muscular layer o the intestinal wall, but 99(3):397403.
rom the interstitial cells o Cajal, not the smooth mus- Lua S, et al. Imaging o gastrointestinal stromal tumor (GIS ).
cle cells. T eir location in the muscular layer can make Clin Radiol. 2004;59:48798.
small GIS s somewhat dif cult to detect and lead to Novitsky YW, Kercher KW, Sing RF, Heni ord B . Long-term
underestimation o tumor extent by endoscopy. outcomes o laparoscopic resection o gastric gastrointesti-
nal stromal tumors. Ann Surg. 2006;243:73847.
At presentation, GIS is requently ound to be Rutkowski P, et al. Clinical utility o the new American Joint
metastatic, most commonly to the liver or perito- Committee on Cancer staging system or gastrointestinal
neum. Presenting symptoms may include abdominal stromal tumors. Cancer. 2011;117:491624.
pain, dyspepsia, or early satiety, but gastrointestinal Sepe PS, Brugge WR. A guide or the diagnosis and manage-
bleeding is the most common occurrence, leading ment o gastrointestinal stromal cell tumors. Nat Rev Gastro-
enterol Hepatol. 2009;6(6):36371.
to the eventual diagnosis o a GIS . Li e-threatening
abrizian P, Sweeney RE, Uhr JH, Nguyen SQ, Divino CM.
hemorrhage rom intraperitoneal rupture o these Laparoscopic resection o gastric and small bowel gastro-
highly vascular tumors may also occur. Generally intestinal stromal tumors: 10-year experience at a single
speaking, imatinib chemotherapy is considered rst center. J Am Coll Surg. 2014;218:36773.

http://surgerybook.net/
25
Gastric Lymphoid umors

Cletus A. Arciero

A 56-year-old man with a 4-month history o vague epi- D. Partial/total gastrectomy with no lymph node
gastric abdominal pain, decreased appetite and weight dissection
loss presents to his local gastroenterologist or evalua- E. Surgical resection is not warranted in most cases
tion. An esophagogastroduodenoscopy (EGD) reveals
non-speci c gastritis and a polypoid lesion in the region 3. First line therapy or a MAL lymphoma as noted
o the antrum. Laboratory ndings note mild anemia, in the patient above would consist o which o the
elevated LDH, and H. pylori positive samples rom the ollowing?
EGD. A. Surgical resection
Follow-up endoscopic ultrasound (EUS) notes a B. Radiation therapy
thickened antral wall, and multiple biopsies obtained C. Chemotherapy
reveal an extra-nodal marginal zone B cell lymphoma D. H. pylori eradication
o mucosa (gut)-associated lymphoid tissue (MAL ) E. Watch ul waiting
type (MAL lymphoma). Computed tomography o the
chest, abdomen, and pelvis reveals thickening o the dis- 4. What is the management or persistent, localized,
tal hal o the stomach with no evidence o adenopathy. MAL lymphoma ollowing repeatedly ailed
Bone marrow biopsy reveals no evidence o disease dis- ef orts at H. pylori eradication therapy?
semination. A. Radiation therapy
B. Surgical resection
1. A er the patient has undergone a complete staging C. Chemotherapy
work-up as noted above, what stage low-grade D. Rituximab
gastric MAL lymphoma does this patient have? E. Bevacicumab
A. Stage I
B. Stage II 5. How does the treatment strategy change in patients
C. Stage III with high-grade gastric lymphoma (advanced MAL
D. Stage IV lymphoma or dif use large B-cell lymphoma)?
E. Unknown A. Surgical resection
B. Radiation therapy alone
2. T e proper surgical management o this patient C. Chemotherapy alone
with gastric lymphoma is: D. H. pylori eradication
A. otal gastrectomy with D2 lymph node dissection E. Combined chemotherapy with or without radia-
B. otal gastrectomy with D1 lymph node dissection tion therapy
C. Partial gastrectomy with D1 lymph node dissection

http://surgerybook.net/
90 G EN ERAL S U RG ERY EXAM I N ATIO N AN D BO ARD REVI EW

ANSWERS 2. E. Gastric lymphoma is the second most common


gastric malignancy (behind gastric adenocarcinoma)
1. A. T e staging o gastric lymphoma is paramount and the most requent cite o extra-nodal non-
in the proper management o the disease. Although Hodgkins Lymphoma. For decades, surgical resec-
o en indolent, approximately 10% o patients with tion was considered appropriate therapy or all stages
gastric lymphoma will present with advanced disease. o resectable gastric lymphomas. Although surgical
EGD allows or visualization o the lesion and o en resection has provided outstanding results and excel-
tissue sampling as well as H. pylori diagnosis. Endo- lent long-term survival, stomach preserving methods
scopic ultrasound is also an important diagnostic have provided equivalent results without the mor-
procedure that can determine extent o disease, depth bidity associated with gastrectomy (partial or total)
o invasion, and o en allows more complete tissue or both low-grade and high-grade B-cell lympho-
sampling. EUS also allows or accurate estimates o mas. A prospective trial by the German Multicenter
depth o invasion, which is an important prognostic rial group in 2005 examined 185 patients with Stage
marker or disease recurrence. C o the chest, abdo- I and II low-grade gastric lymphoma and noted no
men, and pelvis is important to assess or dissemi- di erence in survival between those patients treated
nated disease while a bone marrow biopsy will detect surgically and those receiving no surgical interven-
evidence o distant disease in up to 15% o patients. tion. A ollow-up study by the same group exam-
Standard laboratory testing includes complete blood ined an additional 393 patients and provided similar
count and LDH, as well as other standard chemistries. results. T e results are similar or high-grade gastric
Based on the National Comprehensive Cancer lymphomas, where survival rates between primary
Network (NCCN) Clinical Practice Guidelines, the surgery and primary chemotherapy and radiation
Lugano Staging System or Gastrointestinal Lym- therapy groups is equivalent. Surgical intervention
phoma is adequate. T is modi cation o the Ann or gastric lymphoma is largely reserved or rare
Arbor Staging System is noted below, directly com- cases o per oration, or hemorrhage that cannot be
pared with the Ann Arbor Staging System. Utiliz- controlled endoscopically. Stomach preserving treat-
ing either staging system is appropriate. Based these ment strategies are now the standard o care in the
staging systems, the above patient has Stage I disease. management o gastric lymphoma.

Lugano Staging System Ann Arbor Staging System

Stage I - T e tumor is con ned to the gastroin- Stage I - Involvement o a single lymph node region (I) or o a
testinal tract. It can be a single primary lesion or single extralymphatic organ or site (IE)*
multiple, noncontiguous lesions.
Stage II - T e tumor extends into the abdomen. T is Stage II - Involvement o two or more lymph node regions or
is urther subdivided based upon the location o lymphatic structures on the same side o the diaphragm alone
nodal involvement: (II) or with involvement o limited, contiguous extralymphatic
Stage II1: Involvement o local nodes (paragastric organ or tissue (IIE)
nodes or gastric lymphoma or para-intestinal
nodes or intestinal lymphoma)
Stage II2: Involvement o distant nodes (para-
aortic, para-caval, pelvic, or inguinal nodes or
most tumors; mesenteric nodes in the case o
intestinal lymphoma)
Stage IIE: T e tumor penetrates the serosa to
involve adjacent organs or tissues
Stage III - T ere is no stage III disease in this Stage III - Involvement o lymph node regions on both sides o the
system. diaphragm (III), which may include the spleen (IIIS) or limited,
contiguous extralymphatic organ or site (IIIE) or both (IIIES)
Stage IV - T ere is disseminated extranodal Stage IV - Di use or disseminated oci o involvement o one
involvement or concomitant supra-diaphragmatic or more extralymphatic organs or tissues, with or without asso-
nodal involvement. ciated lymphatic involvement

http://surgerybook.net/
C H AP TER 2 5 G AS TRIC LYM P H O I D TU M O RS 91

3. D. Gastric MAL lymphomas arise rom B-cells therapy was ound to be superior to even combined
and constitute approximately 50% o gastric lym- modality approaches.
phomas, with the di use large B-cell lymphoma Further support or radiation therapy is provided
making up the other large proportion o gastric by Goda, et al. who noted an overall remission rate
lymphomas. Although surgical therapy was consid- o 92% with excellent long-term (10-year) survival
ered the mainstay o therapy or decades, in the late data. Additional studies show long-term remis-
1980s, a connection between Campylobacter (later sion rates o 88% to 97% (5 to 7.8 years). For some
Helicobacter) pylori, chronic gastritis and mucosal patients with Stage I disease, chemotherapy may be
associated lymphoid tissue (MAL ) was suspected. added to radiotherapy, but this is not considered the
By the early 1990s, the connection between the two standard approach. Chemotherapy or low-grade
was rmly established and the treatment o gastric gastric MAL lymphoma is typically reserved or
MAL lymphoma with H. pylori eradication was persistent Stage II disease or patient presenting with
instituted. A meta-analysis o 34 studies with 1271 more advanced disease (Stage IIE or IV). T e exact
patients noted an overall H. pylori eradication rate chemotherapeutic regimen or advanced stage gas-
o 98.3%, associated with a complete remission o tric MAL lymphoma is not well established, but is
gastric lymphoma in 77.8% o patients. T e relapse o en treated with agents utilized against ollicular
rate or patients was 2.2% per year, and only 0.05% lymphomas.
o patients had trans ormation o low-grade lym-
phoma into an aggressive, high-grade lymphoma. 5. E. As addressed earlier, surgical therapy has been
Frequent endoscopic monitoring (3 month inter- largely abandoned as the primary treatment o gas-
vals initially) is paramount to assess or treatment tric lymphoma, including patients with advanced
response. Patients with pathology revealing clear- gastric MAL lymphoma and gastric di use large
ance o H. pylori and resolution o lymphoma will B-cell lymphoma (DLBCL). A study examining the
require continued surveillance. T ose patients with long-term outcomes in patients treated with surgery
persistent H. pylori should receive additional eradi- alone, surgery ollowed by radiation therapy, surgery
cation therapy. I gastric lymphoma persists despite ollowed by chemotherapy and chemotherapy alone
multiple rounds o anti-H. pylori therapy, the noted that complete response rates were similar in
addition o external beam radiation and/or chemo- the 4 arms. But, survival was signi cantly improved
therapy is warranted. in patients receiving chemotherapy, with no clear
Patients with H. pylori negative gastric MAL bene t to combined modality therapy with surgery
lymphoma o en have a documented translocation and chemotherapy. In act, late toxicity was more re-
t(11;18), and thus eradication therapy is o en not quent and severe in patients who had surgery.
e ective. T ese patients are treated with radiation In terms o H. pylori eradication, studies are ongo-
therapy as rst-line therapy. Rituximab has shown ing. It was once thought that there was little bene t in
some e ectiveness in these patients as well, and is treating H. pylori in these patients, since there was no
currently considered in those patients with per- MAL component in most patients. Although there
sistent localized disease and a contraindication to are cases exhibiting both gastric MAL lymphoma
radiotherapy. and DLBCL, rst-line therapy was typically targeted
at the more aggressive entity. Recently, there has
4. A. T e management o persistent, localized, early been some success noted with H. pylori eradication
stage gastric MAL lymphoma is radiation therapy. in early stage DLBCL patients that are H. pylori posi-
Although there are various approaches taken in the tive. In comparing patients with pure gastric DLBCL
management o these patients, radiation therapy and mixed (MAL and DLBCL), H. pylori eradica-
(external beam, 3040 cGy) has shown excellent tion rates were 100% and 94.1%, respectively. Remis-
results and is the recommended treatment by NCCN sion rates were 68.8% or pure DLBCL and 56.3% or
guidelines. In general, rst-line salvage therapy pro- the mixed gastric DLBCL. Prospective studies are
vides remission rates o 90.1%. Radiation therapy was ongoing. Currently, chemotherapy, with or without
superior to chemotherapy or surgery, with a 97.3% biologic therapy, is considered rst-line therapy or
remission rate versus 92.5% or surgery, and 85.3% gastric DLBCL, which is considered the more aggres-
or chemotherapy. In act, radiation therapy as a sole sive o the two most common gastric lymphomas.

http://surgerybook.net/
92 G EN ERAL S U RG ERY EXAM I N ATI O N AN D BO ARD REVI EW

NCCN clinical practice guidelines note that Stage I Koch P, del Valle F, Berdel WE, Willich NA, Reers B,
and II disease are treated with chemotherapy with Hiddemann W, et al. Primary gastrointestinal non-
localized radiation therapy added in certain cases. Hodgkins lymphoma: II. Combined surgical and conser-
vative or conservative management only in localized gastric
T e chemotherapeutic regimens utilized vary, but lymphomaresults o the prospective German Multicenter
the most common regimen combines cyclophos- Study GI NHL 01/92. J Clin Oncol. 2001;19(18):387483.
phamide, doxorubicin, vincristine, and prednisone Koch P, Probst A, Berdel WE, et al. reatment results in local-
(CHOP) and o en the addition o a biologic agent ized primary gastric lymphoma: data o patients registered
(riuximab) (R-CHOP). T e CHOP regimen shows within the German multicenter study (GI NHL 02/96).
J Clin Oncol. 2005;23(28):70509.
complete remission (CR) rates o 87% to 100% with
Kuo SH, Yeh KH, Wu MS, Lin CW, Hsu PN, Wang HP, et al.
good long-term survival. An examination o the Helicobacter pylori eradication therapy is e ective in the
R-CHOP regimen reveals similar statistics with a CR treatment o early-stage H pylori-positive gastric di use
rate o 87%, with the remaining 13% exhibiting a par- large B-cell lymphomas. Blood. 2012;119(21):483844.
tial remission. T e addition o radiation therapy or Liu H, Ye H, Ruskone-Fourmestraux A, De Jong D, Pileri S,
early stage disease has been examined and there was a T iede C, et al. (11;18) is a marker or all stage gastric
MAL lymphomas that will not respond to H. pylori eradi-
notable decrease in local recurrences in those patients cation. Gastroenterology. 2002;122(5):128694.
treated with radiation therapy. But, radiation therapy Martinelli G, Gigli F, Calabrese L, Ferrucci PF, Zucca E, Crosta
did not add to overall survival when compared with C, et al. Early stage gastric di use large B-cell lymphomas:
chemotherapy alone. Overall, radiation therapy is results o a randomized trial comparing chemotherapy
selectively added to chemotherapy in patients with alone versus chemotherapy and involved eld radiother-
apy. (IELSG 4). Leuk Lymphoma. 2009;50(6):92531.
gastric DLBCL, speci cally or local control.
Martinelli G, Laszlo D, Ferreri AJ, Pruneri G, Ponzoni M,
For patients with more advanced disease (speci - Conconi A, et al. Clinical activity o rituximab in gastric
cally Lugano Stage IV), chemotherapy alone is uti- marginal zone non-Hodgkins lymphoma resistant to or
lized with radiation therapy reserved as needed or not eligible or anti-Helicobacter pylori therapy. J Clin
local control o symptoms. Oncol. 2005;23(9):197983.
National Comprehensive Cancer Network. National Compre-
hensive Cancer Network Clinical Guidelines: Non-Hodkins
BIBLIOGRAPHY Lymphoma, Version 2. 2014. Accessed from www.nccn.org.
Aleman BM, Haas RL, van der Maazen RW. Role o radiother- Raderer M, Chott A, Drach J, Montalban C, Dragosics B, Jger
apy in the treatment o lymphomas o the gastrointestinal U, et al. Chemotherapy or management o localised high-
tract. Best Pract Res Clin Gastroenterol. 2010;24(1):2734. grade gastric B-cell lymphoma: how much is necessary?
Andriani A, Zullo A, Di Raimondo F, Patti C, edeschi U, Ann Oncol. 2002;13(7):10948.
Caruso L, et al. Clinical and endoscopic presentation o Rohatiner A, dAmore F, Coif er B, Crowther D, Gospodaro-
primary gastric lymphoma: a multicenter study. Aliment wicz M, Isaacson P, et al. Report on a workshop convened
Pharmacol T er. 2006;23:72126. to discuss the pathological and staging classi cation o gas-
Avils A, Neri N, Nambo MJ, Huerta-Guzman J, Cleto S. Sur- trointestinal tract lymphoma. Ann Oncol. 1994;5:397400.
gery and chemotherapy versus chemotherapy as treat- Ruskon-Fourmestraux A, Fischbach W, Aleman BM, Boot H,
ment o high-grade MAL gastric lymphoma. Med Oncol. Du MQ, Megraud F, et al. EGILS consensus report. Gastric
2006;23(2):295300. extranodal marginal zone B-cell lymphoma o MAL . Gut.
Avils A, Nambo MJ, Neri N, Huerta-Guzman J, Cuadra I, 2011;60(6):74758.
Alvarado I, et al. T e role o surgery in primary gastric Seiichiro A, Ichiro O, Koji I, Haruhisa S, Shigetaka Y, Satoru N,
lymphoma: results o a controlled clinical trial. Ann Surg. et al. A retrospective study o 5-year outcomes o radio-
2004;240(1):4450. therapy or gastric mucosa-associated lymphoid tissue
Cavalli F, Isaacson PG, Gascoyne RD, et al. MAL lympho- lymphoma re ractory to Helicobacter pylori eradication
mas. Hematology Am Soc Hematol Educ. Program 2001; therapy. Jpn J Clin Oncol. 2013;43(9):91722.
24158. Stolte M, Eidt S. Lymphoid ollicles in antral mucosa:
Eidt S, Stolte M, Fischer R. Helicobacter pylori gastritis and immune response to Campylobacter pylori? J Clin Pathol.
primary gastric non-Hodgkins lymphomas. J Clin Pathol. 1989;42(12):126971.
1994;47(5):4369. Wirth A, Gospodarowicz M, Aleman BM, et al. Long-term
Goda JS1, Gospodarowicz M, Pintilie M, et al. Long-term outcome or gastric marginal zone lymphoma treated with
outcome in localized extranodal mucosa-associated lym- radiotherapy: a retrospective, multi-centre, International
phoid tissue lymphomas treated with radiotherapy. Cancer. Extranodal Lymphoma Study Group study. Ann Oncol.
2010;116(16):381524. 2013;24(5):134451.
Janssen J. T e impact o EUS in primary gastric lymphoma. Whrer S, Pspk A, Drach J, et al. Rituximab, cyclophospha-
Best Pract Res Clin Gastroenterol. 2009;23:6718. mide, doxorubicin, vincristine and prednisone (R-CHOP)

http://surgerybook.net/
C H AP TER 2 5 G AS TRI C LYM P H O I D TU M O RS 93

or treatment o early-stage gastric di use large B-cell lym- Zullo A, Hassan C, Andriani A, Cristo ari F, De Francesco
phoma. Ann Oncol. 2004;15(7):108690. V, Ieradi E, et al. E ects o Helicobacter pylori eradica-
Ye H, Liu H, Raderer M, Chott A, Ruskone-Fourmestraux A, tion on early stage gastric mucosa-associated lymphoid
Wotherspoon A, et al. High incidence o t(11;18)(q21;q21) tissue lymphoma. Clin Gastroenterol Hepatol. 2010;8(2):
in Helicobacter pylori-negative gastric MAL lymphoma. 10510.
Blood. 2003;101(7):254750. Zullo A, Hassan C, Andriani A, et al. reatment o low-grade
Zullo A, Hassan C, Andriani A, Cristo ari F, De Francesco V, gastric MAL -lymphoma unresponsive to Helicobacter
Ieradi E, et al. Eradication therapy or Helicobacter pylori pylori therapy: a pooled-data analysis. Med Oncol. 2010;27(2):
in patients with gastric MAL lymphoma: a pooled data 2915.
analysis. Am J Gastroenterol. 2009;104(8):19327.

http://surgerybook.net/
26
Gastroesophageal Re ux Disease

Mary . ODonnell and E. Matthew Ritter

SCENARIO 1 2. He is started on the medication and experiences


A 58-year-old male presents to your clinic with upper relie . He returns to your clinic 1 year later, but
abdominal pain and heartburn a er meals. He occasion- now the medication is no longer relieving his
ally relieves symptoms with chewable antacids (calcium symptoms. He also reports occasional cough and
carbonate). He denies other problems except or gaining the sensation that he has to clear his throat. What
5 lbs over the last year with decreased exercise. He has no should be your next step?
surgical history, takes no medications, and has no signi - A. Esophagogastroduodenoscopy (EGD)
icant amily history. He used to smoke, but quit 15 years B. Per orm barium swallow
ago and drinks 2 glasses o red wine each night. C. Per orm 24 hr ambulatory pH monitoring
D. Per orm pulmonary unction tests
1. Other than improving his diet and decreasing
his alcohol intake, what medication would you 3. He ollows up in 3 weeks with a barium swallow, pH
prescribe to best limit his esophageal acid exposure? monitoring, and an EGD rom an outside provider.
A. Proton pump inhibitor (PPI) He provides a copy o the 24 hr ambulatory pH
B. H2 receptor antagonist (H2RA) monitoring report to you, which can be viewed
C. Calcium carbonate below. He has a sliding ( ype I) hiatal hernia, and
D. Sucral ate as needed no evidence o intestinal metaplasia or ulcers.

Acid Re ux Composite Score Analysis (Johnson/DeMeester) (pH)

T reshold Patient Value Patient Score Normal


Upright time in re ux 10.4% 5.1 < 6.3
Recumbent time in re ux 10.0% 21.9 < 1.2
otal time in re ux 10.2% 7.3 < 4.2
Episodes over 5 min 5.4 4.9 < 3.1
Longest episode 41.7 min 15.1 < 9.2
otal episodes 48.6 2.9 < 50.1
Composite score 57.1 < 22.0
NO E: Composite score patient values are normalized or 24 Hours.

http://surgerybook.net/
C H AP TER 2 6 G As TRo Es o P H AG EAl REf l u x D i s EAs E 95

What is the intervention would you o er him at 6 months. She presents to the ER 12 hours later with
this time? rapid respirations, tachycardia, ever, and elevated
A. Esophagectomy WBC. Based upon your clinical suspicions, what is
B. Nissen undoplication the best initial diagnostic test?
C. Heller myotomy with Dor undoplication A. Gastrogra n esophagography
D. Botulinum toxin injection B. T in barium esophagography
C. C with IV contrast or PE/DV protocol
4. What is the most common long-term complication D. EGD
o a Nissen undoplication?
A. Stricture 3. A small intrathoracic per oration is conf rmed. In
B. Gastroesophageal leak your determination o a management plan, which
C. Gas bloat syndrome o the ollowing is an absolute contraindication to
D. Dysphagia non-operative management?
E. Slipped Nissen A. ime o per oration > 72 hrs
B. A history o Barretts esophagus with low grade
5. He undergoes a Nissen undoplication without dysplasia
complication and is discharged the ollowing C. Per oration contained in the mediastinum
day. Five days a er surgery, he returns or ollow D. Evidence o SIRS (systemic in ammatory
up appointment reporting le upper quadrant response syndrome)
pain. He is hemodynamically normal and has a
hemoglobin one point lower than his preoperative 4. T e patient has a small thoracic esophageal
values. C scan o the chest, abdomen, and pelvis per oration with a le sided e usion and is
shows a heterogeneous le upper quadrant uid developing low blood pressure. A er beginning
collection without rim enhancement and an resuscitation, you determine she needs to go to
associated small le pleural e usion. surgery or repair o the leak. Based upon the most
What is the most appropriate initial management likely location o these types o injuries, which
or this patient? incision would you use to expose the esophagus or
repair?
A. Admit or uids and observation
B. Admit or antibiotics and total parenteral nutrition A. Right posteriolateral thoracotomy
C. Return to the operating room B. Le posteriolateral thoracotomy
D. Percutaneous drainage C. Median sternotomy
D. Laparotomy
SCENARIO 2
A 63-year-old emale with a long history o gastro- ANSWERS O SCENARIO 1
esophageal re ux disease presents to your clinic a er a 1. A. T e mainstay o medical treatment o GERD is
gastroenterologist per orms an EGD. She is diagnosed acid suppression. Patients with persistent symptoms
with Barretts esophagus without evidence o dysplasia should be given PPIs, such as omeprazole. In doses
based upon the results o several biopsies. as high as 40 mg/d, they can ef ect an 80% to 90%
1. When should her next EGD with biopsy be reduction in gastric acidity. In patients with re ux
per ormed or appropriate surveillance? disease, esophageal acid exposure is reduced by up
to 80% with H 2RAs and up to 95% with PPIs. Despite
A. 3 months the superiority o the latter class o drug over the or-
B. 6 months mer, periods o acid breakthrough still occur.
C. 1 year
D. 3 years 2. A. I a er a year o success ul symptom relie ,
the symptoms are no longer controlled by a single
2. Her ollow-up EGD demonstrates Barretts medication and he has developed extraesophageal
esophagus with low-grade dysplasia on multiple symptoms (cough and sensation o postnasal drip)
biopsies, so she is scheduled or ollow up EGD in o GERD, then consideration should be given to

http://surgerybook.net/
96 G EN ERAl s u RG ERY Ex AM i N ATi o N AN D Bo ARD REVi EW

prescribing another PPI. It is important, though, to requiring urgent reoperation. Percutaneous drainage
rule out ulcers, malignancy, a hiatal hernia, esophagi- would increase the risk o introducing bacteria and
tis or other esophageal, gastric, or duodenal erosive in ecting the uid collection. Conservative manage-
pathology. T ere ore, EGD is the best option or ment including hydration and observation is recom-
direct mucosal visualization. mended or a small, stable hematoma. Antibiotics are
I no other pathology can account or the symp- not indicated without evidence o in ection.
toms, a 24-hour pH monitoring would be the next
step in diagnosis. T is would determine i acid re ux
ANSWERS O SCENARIO 2
is the cause o the patients symptoms. A barium
swallow could add some more in ormation but is 1. C. Barretts esophagus without dysplasia has a risk
not diagnostic. Halitosis would be an indicator o o 0.1% to 1% rate o progression to adenocarcinoma
a diverticulum (Zenkers) or possibly achalasia in and initial surveillance should be with annual EGD
which ood is retained within the esophagus. A better with biopsy. Low grade dysplasia requires surveil-
study to evaluate these pathologies is barium swal- lance every 6 months until no dysplasia is ound.
low esophagram. Pulmonary unction tests are not I no more dysplasia is ound, surveillance can be
indicated or evaluation o the patients cough. extended to once every 3 years. High-grade dyspla-
sia has a 5-year risk o adenocarcinoma o 30% and
3. B. T e acid re ux composite score analysis shown must be intervened upon with excision or ablation or
is consistent with the diagnosis o GERD. T ere ore, undergo endoscopic surveillance every 3 months.
a Nissen undoplication is an appropriate surgical
choice or this patient with re ractory GERD and a 2. A. T e patient has suf ered and iatrogenic esopha-
sliding hiatal hernia. Since the patient does not have geal per oration. T e best initial test or diagnosis o
high-grade dysplasia or evidence o esophageal can- this is esophagram with water-soluble contrast such
cer, esophagectomy would not be the correct choice. as gastrogra n. I this does not demonstrate leak, thin
T e patient has a type I sliding hiatal hernia, not a barium should be used next. T e WBC elevation and
paraesophageal hernia ( ypes II, III, and IV), there- acute nature o the presentation in relationship to the
ore answer C would not be correct (see diagram). EGD procedure makes PE less likely than esophageal
Botulinum toxin and Heller myotomy with Dor per oration. EGD would not be recommended in this
undoplication are used in the treatment o achalasia, scenario as it could enlarge the per oration.
but would not be indicated in a patient with GERD.
3. D. Operative management is imperative in the
4. D. Early complications o Nissen undoplication: patient who is becoming unstable (developing a SIRS
gastroesophageal leak, pneumothorax, abscess, and response) as it may become li e-threatening. It should
hematoma. O these, the most common is dysphagia. be considered i the per oration is not well-contained,
Late complications o Nissan undoplication: stric- in the acute time period a er the procedure, or there
ture, gas bloat syndrome, wrap disruption, wrap her- is associated malignancy. In contrast, non-operative
niation, and dysphagia. management should be considered i the time o per-
oration > 72 hrs, there is no evidence o associated
5. A. Development o uid collection postoperatively malignancy, or the per oration is well-contained.
without instability in vital signs, ever, WBC or other
evidence o in ection is most suspicious or hema- 4. B. T e surgical approach to the distal esophagus is
toma. T e development o the collection o ve days a le posteriolateral thoracotomy. Any esophageal
postoperatively would indicate a slow bleed, not injury should be buttressed with other tissue like

1. GERD Dia gnos is a nd ma na ge me nt a ccording to S AGES GERD guide line s

2. Hia ta l he rnia GERD s ymptoms only


S ymptoma tic
Non-GERD s ymptoms

As ymptoma tic Re fe r to the s e guide line s

http://surgerybook.net/
C H AP TER 2 6 G As TRo Es o P H AG EAl REf l u x D i s EAs E 97

nearby pleura, a pericardial at pad, pedicled inter- Peghini PL, Katz PO, Bracy NA, et al. Nocturnal recovery o
costal muscle, or the diaphram. gastric acid secretion with twice-daily dosing o proton
pump inhibitors. Am J Gastroenterol. 1998;93:763.
Kohn GP, Price RR, Demeester SR, et al., SAGES Guidelines
BIBLIOGRAPHY Committee. Guidelines or the management o hiatal her-
nia. Surg Endosc. 2013;27:440928.
Castell, Donald O. Linda Diederich, June A. Castell. Esopha- Laheij RJF, Sturkenboom MCJM, Hassing R-J. Risk o
geal Motility and pH esting, echnique and Interpretation. community-acquired pneumonia and use o gastric acid-
3rd ed. Highlands Ranch, CO: Sandhill Scienti c, Inc.; suppressive drugs. JAMA. 2004;292:195560.
2000. Morgenthal CB, Shane M, Stival A, et al. T e durability o lap-
Davis CS, Baldea A, Johns JR, Fisichella PM. T e evolution aroscopic Nissen Fundoplication: 11-year outcomes. Jour-
and long-term results o laparoscopic antire ux sur- nal of Gastrointestinal Surgery. 2007;11:693700.
gery or the treatment o gastroesophageal re ux dis- Ozawa S, Kitajima M, Esophageal Per oration. In: Fischer JE,
ease. Journal of the Laparoendoscopic Surgeons. 2010;14: Jones DB, Pomposelli FB, Upchurch GR Jr., Klimberg VS,
33241. Schwaitzberg SD, et al. Fischers Masters of Surgery. 6th ed.
Fernando H, Murthy SC, Ho stetter W, Shrager JB, Bridges B, Philadelphia, PA: Lippincott Williams & Wilkins; 2011.
Mitchell J. T e Society o T oracic Surgeons Practice Ste anidis D, Hope WW, Kohn GP, Reardon PR, Richardson
Guideline Series: Guidelines or the management o WS, Fanelli RD, SAGES Guidelines Committee. Guidelines
Barretts Esophagus with high grade dysplasia. Annals of or surgical treatment o gastroesophageal re ux disease.
T oracic Surgery. 2009;87:19932002. Surg Endosc. 2010;24:264769.
Hughes SGI, Chekan EG, Ali A, Reintgen KL, Eubanks WS. Wang K, Sampliner RE. Updated guidelines 2008 or the
Unusual complications ollowing laparoscopic Nissen diagnosis, surveillance and therapy o Barrett sesophagus.
undoplication. Surg Laparosc Endosc Percutan ech. 1999; American Journal o Gastroenterology. Am J Gastroenterol.
9(2):1437. 2008;103:78897.

http://surgerybook.net/
27
Per orated Peptic Ulcer Disease

Henry Lin

A 78-year-old emale with arthritis and asthma presents E. Laparoscopic Graham patch with parietal cell
to the emergency room with an acute onset o epigastric vagotomy
pain a couple o hours ago. Her pulse is 104, her blood
pressure is unchanged rom her baseline o 110/74 mm 3. What testing should be done or ollow-up?
Hg, and her temperature is 98.2 Fahrenheit. Her medi- A. Secretin stimulation test
cations include occasional naproxen and prednisone or B. H. pylori stool antigen testing
occasional exacerbations o her asthma. On physical C. Emergent urea breath testing
exam, her abdomen demonstrates signi cant epigastric D. Colonoscopy
tenderness with rebound. She has a mild leukocytosis E. Both A and C
o 12.5 cells/mcL. Her acute abdominal series demon-
strates a small amount o ree air. 4. What urther ollow-up is necessary i she has no
urther symptoms?
1. What is the most reasonable current treatment A. Long-term intravenous pantoprazole
option or this patient as the next step? B. Nothing
A. Laparoscopic highly selective vagotomy without C. Chronic suppressive antibiotics
resuscitation D. Serum gastrin level
B. Nasogastric tube insertion, cessation o all oral E. Esophagogastroduodenoscopy with biopsy o
eeds, & intravenous uid initiation or the next ulcer i still present
24 hours
C. Open truncal vagotomy with pyloroplasty
ANSWERS
D. Open Graham patch with parietal cell vagotomy
despite laparoscopic experience and resources 1. B. O all the listed options, nasogastric tube inser-
E. Emergent anterior seromyotomy tion with NPO status and initiation o IV uid is
a very reasonable rst step in the modern era o
2. I she does not demonstrate improvement during H. pylori detection and treatment, especially in
the 12 hours af er onset o symptoms, what is the a stable patient. Graham patch with parietal cell
most reasonable and expedient next step? vagotomy is also a very reasonable option but an
A. runcal vagotomy with antrectomy and a Bilroth open approach is more problematic i laparoscopic
II reconstruction experience and resources are available. T e veri ca-
B. Continued observation tion o resuscitation is required prior to going to the
C. Selective angioembolization operating room or patients. T e veri cation in this
D. Laparoscopic Graham patch only and H. Pylori patient can simply be assessing volume status (e.g.,
testing with possible treatment urine output o 0.5 cc/kg/hour or normal heart rate

http://surgerybook.net/
C H AP TER 2 7 P ERf o RATEd P EP Ti C U l C ER d i s EAs E 99

or a patient not on a beta-blocker. runcal vagot- 4. E. Esophagogastroduodenoscopy (EGD) is neces-


omy with pyloroplasty is not a rst-line treatment in sary to rule out a gastric carcinoma that caused the
the modern era o H. Pylori. Anterior seromyotomy, per oration, especially in older patients. Pantopra-
division o the seromuscular layer o the lesser cur- zole, another proton-pump inhibitor, or H2 blockers
vature in order to achieve a highly selective vagot- are reasonable to start on admission or per orated
omy e ect, is a reasonable approach but not as a rst peptic ulcer disease but long-term treatment is not
option. absolutely necessary, especially af er treatment or
H. Pylori. reatment o H. Pylori with a course o
2. D. Laparoscopic Graham patch with H. Pylori test- antibiotics but chronic suppression should not be
ing and subsequent testing is the best choice o those necessary. Serum gastrin level can be checked or a
presented. runcal vagotomy with antrectomy is patients who is suspected o gastrinoma but is not
no longer a rst- line option in the modern era o necessary i the patients peptic ulcer is healed.
H. Pylori detection and treatment. Observation is
not reasonable i the patient is not improving and a BIBLIOGRPAHY
more aggressive management choice is most likely Baker RJ, T e Per orated Duodenal Ulcer. In: Fischer JE,
necessary. Angioembolization may be considered or Jones DB, Pomposelli FB, Upchurch GR Jr., Klimberg VS,
Schwaitzberg SD, et al. Fischers Masters of Surgery. 5th ed.
bleeding peptic ulcer disease in selective cases but
Philadelphia, PA: Lippincott Williams & Wilkins; 2007:
not or per oration o an ulcer. Parietal cell vagotomy 891901.
is no longer considered one o the early line treat- Bertle MJ, Per orated peptic ulcer disease: a review o his-
ments but to be reserved as a treatment option or tory and treatment. Dig Surg. 2010:1619.
re ractory peptic ulcer disease. Bhogal RH, Comparison between open and laparoscopic repair
o per orated peptic ulcer disease. World J Surg. 2008:23714.
Donovan AJ, Berne V, Donovan JA. Per orated duodenal
3. B. H. Pylori stool antigen testing is a very reasonable ulcer: an alternative therapeutic plan. Arch Surg. 1998;
approach and can even be ordered semi-emergently 133(11):116671.
in the emergency room at some institutions to help Gusta son J,Welling D. No Acid, No Ulcer100 Years Later: A
in decisions with early treatment options. Secretin review o the history o peptic ulcer disease. J Amer Coll
stimulation test is utilized or gastrinoma work-up, Surgeons. 2010:11017.
Jones DB, Mathiel SK, Schneider BE. Laparoscopic anagement
which is not yet necessary during this part o the
o peptic ulcer disease, atlas o minimally invasive surgery,
work-up or this patient since recalcitrant peptic 2006:196225.
ulcer disease is not yet identi ed. Urea breath test- Lin H and Jones DB, Surgery or Peptic Ulcer Disease, In alley
ing or H. pylori is reasonable but is not an emergent et al Practical Gastroenterology and Hepatology: Esopha-
process as obtaining this test requires a clinical lab. gus and Stomach, Blackwell Publishing; 2010:404411.
Colonoscopy is reasonable in a patient over the age Napolitano L. Re ractory peptic ulcer disease. Gastroenterol
Clin N Am. 2009; 38:26788.
o 50 who has not had a screening process done but is Stabile BE.Rede ning the Role o Surgery or Per orated Duode-
not required to address the ollow-up or peptic ulcer nal Ulcer in the Helicobacter pylori Era. Ann Surg. Feb 2000;
disease per oration. 231(2):15960.

http://surgerybook.net/
28
Marginal Ulcers

Richard M. Peterson

A 51-year-old emale presents to the emergency depart- 1. Which of the following is a factor that is potentially
ment. She complains o abdominal pain that has gotten contributing to this patients current problem?
worse over the last several days. T e pain is sharp and A. Hypercholesterolemia
located right under her breast bone. It seems to hap- B. Active smoking
pen right a er she eats or drinks and this morning it C. Obstructive sleep apnea
doubled her over. She relates that she has had this eeling D. Increase in carbohydrate intake
be ore, but it was never this bad. She has some nausea E. Age > 50
but no vomiting. Past medical history is signi cant or
hypertension, obstructive sleep apnea on CPAP, hyper- 2. Which of the following is the most common
cholesteremia, and type 2 diabetes (which she states has complication (early or late) following laparoscopic
been normal since her one month post op visit). Her gastric bypass?
surgical history is signi cant or one C-section about A. Internal hernias
15 years ago and a laparoscopic gastric bypass 9 months B. Small bowel obstruction
ago. She denies any drug use. She drinks a glass o red C. Marginal ulceration (anastomotic/gastro-jejunal
wine most evenings to help her heart. A er much ulcer)
probing she relates that even though she quit smoking D. Gastrojejunal leak
6 months prior to surgery, she resumed smoking about
6 months ago and is back up to one pack per day. She 3. Which of the following statements is true with
relates that at the time o her surgery she weighed 345 regards to marginal ulcer following gastric bypass?
lbs with a BMI o 59 kg/m 2 and now has lost 120 lbs A. Most marginal ulcers are asymptomatic.
with a BMI o 38.6 kg/m 2. She hasnt seen a bariatric B. Over one-third o patients with marginal ulcer
surgeon since her 3 month postoperative visit, mostly ormation smoke.
because she is embarrassed that she started smoking C. Active H. pylori in ection is an independent risk
again. She stopped taking her omeprazole 4 months ago actor or ulcer per oration.
when her prescription ran out. She also started taking D. Use o proton pump inhibitors is not protective
81 mg aspirin a er watching a documentary on heart o ulcer ormation in the setting o nonsteroidal
disease and obesity. anti-inf ammatory drugs (NSAID) use.
Vital signs are: HR 115, BP 97/62, RR 18, Pulse Ox 98% E. Suture material or type o anastomosis per-
on RA. Her exam is noted to have di use abdominal ten- ormed does not relate to ulcer ormation.
derness, signi cant tenderness in the epigastrium, with
voluntary guarding. Bowel sounds are absent. Hemoc-
cult testing is positive.

http://surgerybook.net/
C H AP TER 2 8 MARg i n Al U l C ERs 101

4. Regarding treatment of marginal ulcers (MU) complications (early or late) o gastric bypass sur-
following gastric bypass: gery. Reported rates o MU range rom 1% to 25%,
A. T e majority can be success ully managed medi- with most series indicating 5% incidence. GJ stric-
cally. ture rates are reported between 3% to 27%. T e
B. Nearly hal o patients will require revision o the incidence o internal hernia a er gastric bypass is
gastrojejunostomy or persistent and/or recur- nearly non-existent in the open gastric bypass, but
rent ulcers. a er laparoscopic gastric bypass occurs in approxi-
C. Late MU are sel -limiting and rarely require mately 2.5% o patients. Small bowel obstruction is
treatment. linked to internal hernia ormation and the rates are
D. H. pylori in ection pre-operatively or persistence equivalent. GJ leaks are an early complication a er
postoperatively increases per oration rates. gastric bypass. In the laparoscopic approach, rates are
E. Endoscopy is o limited value in the treatment o reported at about 1% to 1.8%.
marginal ulcer.
3. B. O patients presenting with MU, over 30% are
5. Which of the following is true regarding ound to be smoking at the time o diagnosis. Most
perforated marginal ulcers following gastric MU are symptomatic (72%). T ose symptoms that
bypass: are most common a er surgery that lead to the
A. About 40% o patients with marginal ulceration diagnosis o MU are: pain (34%), dysphagia (17%),
will develop a per oration. weight gain (13%), nausea and vomiting (8%), and
B. May be managed operatively (open or laparo- GI bleed (3%). Active H. pylori in ection has not
scopically) with oversewing o the ulcer and been determined to be an independent risk actor in
omental patch. the development o per orated MU. T ere is data that
C. Most commonly occurs a er 18 months rom suggests that in the setting o patients that must use
surgery. NSAIDs ollowing gastric bypass that proton pump
D. Develops in about 10% o all patients that inhibitors (PPIs) are protective o MU ormation.
undergo gastric bypass. While there is still debate as to whether hand-sewn
gastrojejunostomy (GJ) versus stapled anastomosis is
better with relationship to post operative outcomes,
ANSWERS
the data clearly relates that non-absorbable suture
1. B. T is patient most likely has a marginal ulcer, material at the anastomosis has a high association
which, at a rate o about 5% a er Roux-en-Y gastric with MU ormation. Because o this act, the use o
bypass, is one o the more common complications. non-absorbable suture at the GJ anastomosis has
Risk actors associated with the development o essentially ceased.
marginal (gastrojejunal/anastomotic) ulcers include
environmental (smoking and alcohol), medication 4. A. T e treatment o MU is largely medical. T e
(NSAIDs), anatomical (gastro-gastric stula or an standard o treatment is PPI therapy initiation/
enlarged gastric pouch), and technique (use o non- continuation and cytoprotective agents (i.e., sucral-
absorbable sutures). T ere is no link associated with ate or cara ate). Additionally cessation o smoking
speci c ood types and ulcer ormation. Other risk and/or NSAID use is critical, as recurrence is high
actors that are associated with marginal ulcer (MU) in patients that continue with these high level risk
are increased acid exposure via a gastro-gastric s- actors. Studies report the incidence o surgical
tula (not con rmed in this patient but more com- intervention or MU to be 4% to 10%. T is usually
monly seen with patients that have had an open occurs in this subset o MU or recalcitrant and/or
gastric bypass versus laparoscopic), hypertension, recurrent ulcers. Continued smoking and NSAID
and use o non-absorbable suture in the anastomosis use were ound to be independent risk actors or
and recent surgery. continued non-healing ulcers. Late ulcers (those
that occur a er 30 days) are rarely sel -limiting. In a
2. C. Marginal ulcers are a late complication o gas- study by Csendes they did a prospective evaluation
tric bypass surgery. Along with gastrojejunostomy o patients and per ormed endoscopy at 1 month
(GJ) stricture they are one o the most common and at 1 to 2 years. T ey ound a 12% rate o MU

http://surgerybook.net/
102 g En ERAl s U Rg ERY EXAM i n ATi O n An D BO ARD REVi EW

at 1 month. Many authors believe this to be part o Bhayani NH, Oyetunji A, Chang DA, et al. Predictors o
the natural progression and healing process o the marginal ulcers a er laparoscopic gastric Roux-en-Y gas-
anastomosis within such a short time rame. Most tric bypass. J Surg Res. 2012;177(2):2247.
Coblijn UK, Goucham AB, Lagarde SM, Kuiken SD, van
clinicians advocate or PPI use in the immedi- Wagensveld BA. Development o ulcer disease a er
ate postoperative period because o this. Clinically Roux-en-Y gastric bypass, incidence, risk actors, and
apparent MU is unlikely to heal without interven- patient presentation: a systematic review. Obes Surg. 2014;
tion (as mentioned above). While the data is not 24(2):299309.
completely clear about the role o H. pylori in ec- Coblijn UK, Lagarde SM, de Castro SMM, Kuiken SD, van
Wagensveld BA. Symptomatic marginal ulcer disease a er
tion and ulcer ormation, the risk o per oration o
Roux-en-Y gastric bypass: Incidence, risk actors and man-
MU is not increased by the presence o H. pylori. agement. Obes Surg. 2015;25:80511.
One study ound that in patients that were H. pylori Csendes A, Burgos AM, Altuve J, Bonacic S. Incidence o
positive and eradicated prior to surgery, the rate o marginal ulcer 1 month and 1 to 2 years a er gastric
MU a er surgery with short term PPI use was sig- bypass: A prospective consecutive endoscopic evaluation
ni cantly reduced. Endoscopy should be part o the o 442 patients with morbid obesity. Obes Surg. 2009;19:
1358.
armamentarium o diagnosing and treating MU. DHondt MA, Pottel H, Devriendt D, Van Rooy F, Vansteen-
kiste F. Can a short course o prophylactic low-dose pro-
5. B. Most ulcers become apparent within the rst ton pump inhibitor therapy prevent stomal ulceration
12 months ollowing surgery. While they can develop a er laparoscopic Roux-en-Y gastric bypass? Obes Surg.
beyond 18 months ollowing surgery this is not the 2010;20:5 9599.
most common time rame that they are seen. T e inci- Eckhauser A, ourquati A, Yousse Y, et al. Internal hernia:
dence o per oration in all patients undergoing lapa- postoperative complication o roux-en-Y gastric bypass
surgery. Am Surg. 2006;72(7):58184.
roscopic gastric bypass is approximately 1% and the El-Hayek K, imratana P, Shimizu H, Chand B. Marginal
incidence o MU on average is 5% (range 1% to 16%), ulcer a er Roux-en-Y gastric bypass: what have we really
there ore the rate o per oration o MU is 20%. Felix learned? Surg Endosc. 2012;26:278996.
et al. in 2008 ound that many cases could be man- Felix EL, Kettelle J, Mobley E, Swartz D. Per orated mar-
aged laparoscopically. In their series, over 30% were ginal ulcers a er laparoscopic gastric bypass. Surg Endosc.
2008;22:212832.
managed by oversewing o the ulcer and utilizing an
Ibele AR, Bendewald FP, Mattar SG, McKenna D . Incidence
omental patch. Other series have con rmed similar o gastrojejunostomy stricture in laparoscopic Roux-en-Y
treatment strategies or MU per oration utilizing the gastric bypass using an autologous brin sealant. Obes
omental patch. imes when it may be necessary or Surg. 2014; Mar 6 [Epub ahead o print].
more appropriate to consider revising the G-J may Kalaiselvan R, Exarchos G, Hamza N, Ammori BJ. Incidence
include MU with bleeding, with or without per ora- o per orated gastrojejunal anastomotic ulcers a er lapa-
roscopic gastric bypass or morbid obesity and role o
tion; recurrent ulceration, when a gastro-gastric s- laparoscopy in their management. Surg Obes Relat Dis.
tula is present; or when the pouch is greatly enlarged. 2012;8(4):4238.
Yurcisin BM, DeMaria EJ. Management o leak in the bariatric
BIBLIOGRAPHY gastric bypass patient: Re-operate, drain, and eed distally.
J Gastrointest Surg. 2009;13:156466.
Azagury DE, Abu Dayyeh BK, Greenwalt I , T ompson CC. Wilson JA, Romagnuolo J, Byrne K, et al. Predictors o endo-
Marginal ulceration a er Roux-en-Y gastric bypass sur- scopic ndings a er Roux-en-Y gastric bypass. Am J Gas-
gery: Characteristics, risk actors, treatment, and outcomes. troenterol. 2006;101(10):21949.
Endoscopy. 2011;43(11):9504.

http://surgerybook.net/
29
Bariatric SurgeryBypass
Abdominal Pain
Richard M. Peterson

A 37-year-old emale presents to the emergency depart- B. Small bowel obstruction a er laparoscopic gas-
ment. She complains o abdominal pain that awoke tric bypass can be conservatively managed in
her rom sleep and has been persistent over the last about 80% o cases.
34 hours. She cant get com ortable, is experiencing C. Intusseption at the jejunojejunostomy (JJ) can be
nausea, and some emesis. She relates that she has had a source o abdominal pain.
similar episodes in the past couple o months about 1 to D. A dilated gastric remnant is a normal nding on
2 times per week, but it was usually sel -limited, but this C scan in post gastric bypass patients.
is the worst she has elt. Her surgical history is signi - E. Large de ects like the Petersens hernia as seen in
cant or two C-sections about 4 and 6 years ago, a laparo- a retrocolic gastric bypass do not need de nitive
scopic gastric bypass a little over 2 years ago, and an open closure.
appendectomy when she was 12. She denies any drug use,
alcohol use, or smoking. She relates that at the time o 3. Small bowel obstruction a er a laparoscopic
her surgery she weighed 290 lbs with a BMI o 53 kg/m 2 gastric bypass:
and now has lost 155 lbs with a BMI o 24.7 kg/m 2. She A. Requires emergent exploration.
hasnt seen a bariatric surgeon in over a year since she B. Can be managed conservatively in 80% o cases.
just moved here or her new job 11 months ago. C. Is o en caused by adhesions rom the original
Vital signs are: HR 102, BP 138/67, RR 20, Pulse Ox surgery.
99% on RA. Her exam is noted to have di use abdomi- D. Should never be evaluated laparoscopically.
nal tenderness, with the area tender in the mid abdo-
men. Bowel sounds are present and high-pitched. 4. T e most common cause o abdominal pain a er
an Roux-en-Y gastric bypass (RYGB) is?
1. Which o the ollowing would is the next most A. Internal hernia causing a small bowel obstruc-
appropriate step in this patients management? tion
A. Upper endoscopy (EGD) B. Marginal ulcer (ulcer at the gastrojejunostomy)
B. Nasogastric tube decompression C. Gastrojejunostomy stricture
C. Abdominal X-ray D. Intussusception o the jejunojejunostomy
D. C scan o abdomen and pelvis E. Biliary colic or dyskinesia

2. Which o the ollowing statements is true regarding 5. When evaluating a patient or intermittent
laparoscopic gastric bypass? abdominal pain in your of ce a er laparoscopic
A. Internal hernias a er laparoscopic gastric bypass gastric bypass:
are ar less common than those seen with open A. UGI should be considered to evaluate or mar-
gastric bypass. ginal ulcer.

http://surgerybook.net/
10 4 G EN ERAL S U RG ERY EXAM I N ATI O N AN D BO ARD REVIEW

B. C scan should be per ormed to de nitively rule is the most sensitive indicator seen and is associated
out an internal hernia. with an internal hernia. Up to 20% o cases can be
C. T e patient should be o ered psychological missed on C scan. Intermittent (chronic) symptoms
counseling since most pain a er a laparoscopic o bowel obstruction may not show signs o an inter-
gastric bypass is psychosomatic. nal hernia on a C scan and there ore the clinical
D. Diagnostic laparoscopy should be considered to impression may lead to a diagnostic laparoscopy or
evaluate or potential sources o internal hernia. evaluation o potential hernia de ects (these are com-
monly ound at the JJ mesenteric de ect or the retro-
roux limb (Petersens hernia) de ect).
ANSWERS
1. D. In this scenario, nasogastric tube (NG ) decom- 2. C. Intussusception o the JJ can lead to chronic
pression is insu cient to treat a patient that has intermittent abdominal pain. Possible causes o
had a gastric bypass and bowel obstruction. T ese intussusception can be related to stricture o the JJ
patients require an operative intervention as con- or a dilated JJ pouch, either o which can serve as a
servative management can have catastrophic conse- lead point during peristalsis and subsequent intus-
quences. Bowel obstruction a er laparoscopic gastric susception. Internal hernia a er laparoscopic gastric
bypass is o en associated with an internal hernia. In bypass are more common than open gastric bypass
the setting o an open gastric bypass the obstruction (this relates to more adhesions in the open gastric
is more likely adhesions versus an internal hernia bypass and hence greater likelihood o the closure o
(typically less adhesion ormation a er laparoscopic the potential spaces by adhesion ormation). Small
gastric bypass). Conservative management o bowel bowel obstruction a er laparoscopic gastric bypass
obstruction a er gastric bypass can be dangerous as is rarely success ully managed conservatively, with
the bowel will need to be manually decompressed most cases requiring operative intervention. Delay in
and conservative management is rarely success ul. getting to the operating room can lead to signi cant
T e rate o internal hernia a er laparoscopic gastric loss o small bowel. A dilated gastric remnant is not
bypass is about 2.5% with studies ranging between a normal nding on a C scan a er gastric bypass.
0.2% to 8%. A dilated remnant strongly suggests obstruction o
the biliopancreatic limb. Stricture o the JJ, intussus-
Re tro c o lic and ante c o lic pathways o f the Ro ux limb
ception o the JJ, or internal hernia are all sources o
3 obstruction that can lead to gastric remnant dilation.
Petersens hernia in a retrocolic gastric bypass is a
small de ect and there ore must be closed. T e small
1 1 size o the de ect can lead to incarceration and stran-
gulation o bowel. T ere are also reported cases o
2 2
strangulation o bowel in the Petersens hernia o an
antecolic gastric bypass, but this space is larger and
there ore theoretically less likely to have strangula-
tion o bowel.
Re trocolic Ante colic
3. A. Small bowel obstruction a er laparoscopic gas-
Intravenous hydration is appropriate as patients tric bypass requires emergent exploration. T e most
with bowel obstruction can requently be dehydrated. likely source is an internal hernia either occurring at
An abdominal X-ray may provide some in ormation the JJ mesenteric de ect (50% to 62%) the transverse
and is a quick test. Air f uid levels may be seen or air mesocolon (23% to 46%) or the Petersens hernia
in the gastric remnant can indicate an obstruction, (12% to 15%). Most surgeons close these de ects at
but they are most o en non-speci c. A C scan o en the time o surgery, but as patients lose weight (esti-
can provide the in ormation relating to the source o mated greatest incidence a er > 50% o excess weight
the bowel obstruction. In patients with active (acute) lost) or other sources o increased intra-abdominal
symptoms o bowel obstruction a er gastric bypass pressure such as pregnancy with an enlarged uterus
and undergoing a C scan, swirling o the mesentery can lead to a weakening o the original repair. Bowel

http://surgerybook.net/
C H AP TER 2 9 BARI ATRI C S U RG ERY BYPAS S ABD O M IN AL PAI N 10 5

obstruction secondary to adhesions a er laparo- e ective to utilize ursodeoxycholic acid (Ursodiol) as


scopic gastric bypass is airly uncommon. Small a prophylactic treatment modality or gallstone or-
bowel obstruction even in non-bariatric surgery cases mation prevention.
reportedly is non-operatively managed in about 40%
o cases. Laparoscopic evaluation o internal hernia 5. D. Patients that present with abdominal pain a er
a er laparoscopic gastric bypass is well documented laparoscopic gastric bypass warrant a ull evalua-
and sa e. T e clinicians com ort with laparoscopy tion. It is rarely a psychological source and as such
versus open surgery should be the de ning actor appropriate history should be taken. I the patient
regarding exploration as either can provide good still has complaints o abdominal pain a er a nega-
outcomes. T e key component is de ning the anat- tive ull evaluation, including upper endoscopy to
omy. It is recommended that reduction o the inter- rule out gastrojejunal ulcer, gastro-gastric stula, a
nal hernia begin with identi cation o the ileocecal C scan, colonoscopy to rule out a colonic source,
valve and ollowing the bowel proximally. T is will a right upper quadrant ultrasound to rule out gall-
serve to both reduce the hernia on the appropriate stone diseases and a HIDA scan to rule out biliary
side o the de ect and will acilitate the identi cation dyskinesia, then diagnostic laparoscopy is the next
o the anatomic landmarks o the JJ, biliopancreatic step in management. Since upwards o 20% o C
limb, and Roux limb. scans will not have evidence o an internal hernia,
visual evaluation o the potential internal hernia
4. A. Internal hernia is o en associated with pain, with spaces is imperative. Postoperative internal her-
or without nausea and emesis. T is can be acute onset nia ormation occurs in approximately 2.5% o
or it can be intermittent and chronic warranting ur- patients, with approximately 50% to 60% o hernias
ther workup and ultimately surgical intervention. It at the JJ mesenteric de ect, 20% at the transverse
is the most common cause o abdominal pain a er mesocolon mesentery and 12% to 15% o hernias
RYGB. Symptoms o a gastrojejunostomy stricture at the Petersens hernia. An upper GI f uoroscopic
are rarely related to pain. T e most common pres- study may provide in ormation about pouch size,
entations are dysphagia and ood intolerance. T ese absence or presence o a hiatal hernia, stricture and
symptoms typically occur about 6 to 8 weeks ollow- sometimes ulcer, however, it is not a rst choice or
ing surgery with a gradual regression o ood toler- evaluation. Given the higher speci city o an EGD
ance rom having been advanced to solid oods and evaluation, this study would yield ar less in orma-
then noting di culty, ollowed by inability to toler- tion in lieu o the other choices o ered.
ate solids and only able to tolerate liquids. Marginal
ulcer ollowing gastric bypass is also associated with BIBLIOGRAPHY
pain. T is usually occurs immediately a er eating Ahmed AR, Rickards G, Husain S, et al. rends in internal
or drinking. Risk actors or developing a marginal hernia a er laparoscopic Roux-en-Y gastric bypass. Obes
Surg. 2007;17(12):15636.
ulcer are use o NSAIDs, smoking, gastro-gastric s-
Caruana JA, McCabe MN, Smith AD, et al. Incidence o
tula ormation, and oreign body (i.e., suture/staple). symptomatic gallstones a er gastric bypass: is prophy-
Intussusception also presents with pain and is lactic treatment really necessary? Surg Obes Relat Dis.
similar to that o an internal hernia. It is commonly 2005;1(6):5647.
intermittent and o en unseen on C scan unless Eckhauser A, ourquati A, Yousse Y, et al. Internal hernia:
the event is occurring during the imaging study. Bil- postoperative complication o roux-en-Y gastric bypass
surgery. Am Surg. 2006;72(7):5814.
iary colic or dyskinesia is also source o abdominal Obeid A, McNeal S, Breland M, et al. Internal hernia a er
pain ollowing weight loss surgery. Gallstone orma- laparoscopic Roux-en-Y gastric bypass. J Gastrointest Surg.
tion occurs as a result o rapid weight loss and may 2014;18(2):2505.
become symptomatic. A standard o practice is not Schirmer BD. Strictures and Marginal Ulcers in Bariatric
to remove the gallbladder at the time o laparoscopic Surgery. In Buchwald H, Cowan GH, Pories WJ. Surgical
Management of Obesity. 1st ed. Philadelphia, PA: Saunders
gastric bypass. T e incidence o symptomatic gall-
Elsevier: 2007:297303.
stone ormation a er surgery approaches 8%. In the Williams SB, Greenspon J, Young HA, et al. Small bowel
study by Caruana et al. (Surg Obes Relat Dis. 2005; obstruction: conservative vs. surgical management. Dis Col
1(6):5647), they also determined that it was not cost Rectum. 2005;48(6):114046.

http://surgerybook.net/
30
Sleeve Gastrectomy Leaks

Alec C. Beekley

A 42-year-old emale with a body mass index o D. reatment o the staple line with brin glue has
42 kg/m 2 along with diabetes mellitus, hypertension, been shown to decrease the staple line leak rate.
hyperlipidemia, and obstructive sleep apnea (OSA) E. Choice o staple height does not have an e ect on
has completed her preoperative evaluation or sur- staple line leak rates.
gical management o her obesity and weight related
co-morbidities. She uses Continuous Positive Airway 3. T e patient presents with ever, abdominal pain,
Pressure (CPAP) nightly or her OSA. She is elt to be and sustained tachycardia on post operative day 4.
a good candidate or surgery and she opted or a Sleeve Which o the ollowing is correct?
Gastrectomy (SG). A. A normal upper gastrointestinal contrast series
rules out a staple line leak.
1. Regarding the immediate postoperative manage- B. Operative management o a staple line leak is
ment, which o the ollowing is correct? more likely to be success ul in this patient than
A. Use o CPAP is contraindicated or the rst post- in patients whose leaks present 10 days or more
operative week or ear o it causing a leak. a er operation.
B. Sustained tachycardia is the most reliable indica- C. I a closed suction drain is le at the initial oper-
tor o a leak. ation and the drain amylase level is the same
C. T e most likely cause o death a er bariatric sur- as the serum amylase level, this nding reliably
gery is a myocardial in arction. rules out a staple line leak.
D. T is patient should have had an IVC lter to pre- D. A positive preoperative H. Pylori test increased
vent a pulmonary embolus. this patients chance o having a staple line leak.
E. T e leak rate rom a SG is less than that o a E. Endoscopic therapies should be the rst
Roux-en-Y gastric bypass (RYGB). approach to gain control o a staple line leak.

2. Regarding intra-operative actors that a ect 4. T e patient undergoes operative repair o the
staple line leaks af er SG, which o the ollowing is staple line leak on postoperative day 4. A closed
correct? suction drain is lef . Which o the ollowing is
A. Staple line buttressing has conclusively shown to correct?
decrease the incidence o staple line leak. A. I this patient develops a gastro-cutaneous s-
B. Over-sewing o the staple line has been shown to tula controlled by the closed suction drain,
decrease the incidence o staple line leak. endoscopic treatment with a covered stent, clip-
C. Per ormance o sleeve gastrectomy over a smaller ping, and/or brin glue should be considered an
diameter bougie decreases the likelihood o sta- experimental option and discouraged unless the
ple line leak. patients condition is severe.

http://surgerybook.net/
C H AP TER 3 0 S l EEv E G AS TREC To m y l EAk S 107

B. T e median time or staple line leaks to heal is prophylaxis, ambulation on the night a er surgery,
less than 2 weeks. and chemical prophylaxis with low molecular weight
C. I an endoscopic stent is placed across the leak, heparin or un ractionated heparin. T ere is no abso-
the biggest problem that can occur is stent ero- lute indication or preoperative IVC lter placement
sion. but it can be considered, along with mechanical and
D. I the patient develops a controlled gastro- chemical prophylaxis in high risk patients, especially
cutaneous stula, drainage alone is still an i the risk o a V E is higher than that o a lter
acceptable option or treatment. related complication.
E. Chronic leaks/ stulae can be managed with late
surgical therapy, such as Roux-en-Y anastomosis 2. E. Multiple intra-operative techniques have been
to the stula site. employed and studied to attempt to reduce staple line
leak rates a er sleeve gastrectomy. Although there is
5. Which o the ollowing is correct regarding staple some evidence that the use o buttressing material
line leaks af er sleeve gastrectomy? can reduce bleeding rom the staple line, there are no
A. Leaks > 10 days a er operation are more likely conclusive data that staple line leak rates are reduced.
to occur a er gastric bypass than sleeve gastrec- Similarly, choice o staple height and over sewing o
tomy. the staple line did not show any bene t in reducing
B. Leaks most commonly occur in the distal third staple line leak.
or antral staple lines. T e use o a bougie > 40 French in diameter has
C. Patients whose sleeve leaks are treated by endo- been shown to decrease the leak rate without a dis-
scopic stenting still must be restricted rom tak- cernible impact on weight loss at up to 3 years com-
ing nutrition by mouth until the leak heals. pared to smaller bougies (< 40 French). Distance
D. Non-operative management o leaks is unac- rom pylorus that the staple line was begun also did
ceptable in any sleeve patient. not a ect leak rates. Finally, treatment o the staple
E. Patients with stulae that ail to close may require line with brin glue did not conclusively demon-
conversion to gastric bypass or gastrectomy. strate any bene t in reducing staple line leak in gas-
tric bypass patients and little data exist to suggest it
has any bene t in sleeve gastrectomy patients.
ANSWERS
1. B. Sustained tachycardia greater than 120 is the 3. B. Staple line leaks a er sleeve gastrectomy present
most reliable indicator o a postoperative leak rom challenging problems in terms o both diagnosis and
bariatric surgery. Other complications that can lead management. Surgical treatment o staple line leaks
to tachycardia like myocardial in arction, pulmonary that occur early (< 10 days) a er operation are more
embolism, bleeding, hypovolemia, and respiratory likely to be success ul than surgical treatment o late
insu ciency should also be ruled out but unex- (> 10 days) leaks. Although clinical signs can be sub-
plained and sustained tachycardia should prompt an tle, upper gastrointestinal (UGI) series have been
intervention to nd and treat a leak, which includes noted to be unreliable. Closed suction drain amylase
operative exploration. T e leak rate o a SG (2.4%) is has been suggested as a reasonable adjunct to clinical
higher than that o a RYGB (0.8%). suspicion o a leak, but patients with leaks and normal
CPAP is not contraindicated a er a SG, or a drain amylase levels have been demonstrated. T ere-
RYGB or that matter, and should plan to be used in ore, a normal drain amylase or a negative UGI should
the postoperative period or people who su er rom not prevent a surgeon rom returning a patient to the
OSA. T ere is no data that suggests it increases the OR i clinical suspicion or a leak remains.
risk o a leak. While people who have a respiratory Although pre-operative detection and eradica-
complication a er bariatric surgery are at an increase tion o H. Pylori in ections has been advocated to
risk o mortality, the most common cause o death reduce post operative dyspepsia or ulceration, the
a er bariatric surgery is sepsis, ollowed by myocar- presence o H. Pylori does not appear to impact sta-
dial in arction, then a pulmonary embolism. ple line leak rates. Although endoscopic therapies
T e recommended venous thromboembolic provide a use ul adjunct in the treatment o gastric
(V E) prophylaxis or this patient is mechanical sleeve staple line leaks, they generally should not be

http://surgerybook.net/
10 8 G EN ERAl S U RG ERy EXAm I N ATIo N AN D Bo ARD REv I EW

used as the initial or primary therapy. Resuscitation, a er laparoscopic sleeve gastrectomy. Obes Surg. July 2009;
intravenous hydration and antibiotics, and either 19(7):8216.
early surgical or interventional radiologic drainage Chen BI, Kiriakopoulos A, sakayannis D, Wachtel MS,
Linos D, Frezza EE. Rein orcement does not necessarily
o intra-abdominal f uid collections to control sepsis reduce the rate o staple line leaks a er sleeve gastrectomy.
should be rst line therapies. A review o the literature and clinical experiences. Obes
Surg. February 2009;19(2):16672.
4. E. One o the largest reviews o staple line leaks a er Gaspari A, Gentileschi P, acchino R, Basso N. T e use o brin
sleeve gastrectomy demonstrated that the median sealant to prevent major complications ollowing laparo-
time or leak healing was 40 days (range 2 to 270). scopic gastric bypass: results o a multicenter, randomized
Endoluminal therapies or staple line leaks show a trial. Surg Endosc. November 2008;22(11):24927.
airly high rate o success at resolving leaks and s- Gupta PK, et al. Predictors o pulmonary complications a er
bariatric surgery. SOARD. 2012;8:57481.
tulas, with the main problem being stent migration. Maher JW, Bakhos W, Nahmias N, Wol e LG, Meador JG,
Although endoluminal therapies are use ul adjuncts, Baugh N, et al. Drain amylase levels are an adjunct in
standard stula management with drainage is still an detection o gastrojejunostomy leaks a er Roux-en-Y
acceptable option but it also mandates nutritional gastric bypass. J Am Coll Surg. May 2009;208(5):8814;
support. Chronic stulae that develop can be suc- discussion 8856.
Nelson DW, Blair KS, Martin MJ. Analysis o obesity-related
cess ully managed by late surgical therapy.
outcomes and bariatric ailure rates with the duodenal
5. E. Late leaks (> 10 days a er initial surgery) are more switch v gastric bypass or morbid obesity. Arch Surg. 2012;
147(9):84754.
likely to occur in sleeve gastrectomy patients than in Parikh MI, Issa R, McCrillis A, Saunders JK, Ude-Welcome A,
gastric bypass patients. T e majority (89%) o leaks Gagner M. Surgical strategies that may decrease leak
occur in the proximal third o the stomach. Non- a er laparoscopic sleeve gastrectomy: a systematic review
surgical therapy is clearly an appropriate option in and meta-analysis o 9991 cases. Ann Surg. 2013;257(2):
selected patients with leak, and endoluminal stenting 2317.
Rossetti G, et al. Does helicobacter pylori in ection have inf u-
has the speci c advantage o allowing patients to take
ence on outcome o laparoscopic sleeve gastrectomy or
ood by mouth. otal gastrectomy with esophagoje- morbid obesity? Int J Surg. 2014;12 Suppl 1:S6871.
junostomy, conversion to gastric bypass, and lateral Silecchia GI, Boru CE, Mouiel J, Rossi M, Anselmino M,
Roux-en-Y gastrojejunostomy all have been described Morino M, et al. Utility o routine versus selective upper
as surgical therapy or sleeve stulae that ail to heal. gastrointestinal series to detect anastomotic leaks a er
laparoscopic gastric bypass. Obes Surg. August 2011;21(8):
123842.
BIBLIOGRAPHY T aler K. reatment o leaks and other bariatric complications
Aurora AR, Khaitan L, Saber AA. Sleeve gastrectomy and the with endoluminal stents. J Gastrointest Surg. September
risk o leak: a systematic analysis o 4,888 patients. Surg 2009;13(9):15679.
Endosc. June 2012;26(6):150915. Sakran N, et al. Gastric leaks a er sleeve gastrectomy; a multi-
Brethauer SA, et al. and the American Society o Metabolic center experience with 2834 patients. Surg Endosc. January
and Bariatric Surgery Clinical Issues Committee. ASMBS 2013;27(1):2405.
updated position statement on prophylactic measures to van de Vrande S, Himpens J, EI Mourad H, Debaerdemaeker
reduce the risk o venous thromboembolism in bariatric R, Leman G. Management o chronic proximal stu-
surgery. SOARD. 2013;9(4):4937. las a er sleeve gastrectomy by laparoscopic Roux-limb
Casella G, Soricelli E, Rizzello M, rentino P, Fiocca F, placement. Surg Obes Relat Dis. November-December
Fantini A, et al. Nonsurgical treatment o staple line leaks 2013;9(6):85661.

http://surgerybook.net/
31
Biliopancreatic Diversion
With Duodenal Switch or
Morbid Obesity
Matthew J. Martin

A 34-year-old morbidly obese emale with a current body B. T e duodenal switch involves a sleeve gastrec-
mass index (BMI) o 55 kg/m2 is being evaluated or poten- tomy and preservation o the pylorus.
tial bariatric surgery. She has tried numerous diet and C. T e duodenal switch creates less bypassed
exercise programs, including a medically supervised pro- intestine than the gastric bypass, resulting in less
gram or 12 months and has ailed to maintain any signi - malabsorption.
cant weight loss. She has type 2 diabetes, gastroesophageal D. T e alimentary limb o the small intestine is
re ux disease (GERD), hypertension, and sleep apnea. She anastomosed to the stomach in both operations.
has had two prior c-sections and a laparoscopic cholecys- E. T e amount o malabsorption with the duodenal
tectomy but no other surgeries. She has done independent switch is primarily a unction o the length o the
research on the internet and is interested in getting more biliopancreatic limb.
in ormation about the duodenal switch procedure.
3. T e patient is interested in knowing about the
1. Which o the ollowing statements about bariatric potential benef ts o surgery, and whether she is a
surgical procedures is correct? candidate or a laparoscopic or open surgery. Which
A. All true bariatric surgery achieves weight loss o the ollowing statements are correct regarding
through restrictive e ects and malabsorptive the duodenal switch?
e ects. A. T e patient is not a candidate or a laparoscopic
B. T e duodenal switch is the most malabsorptive approach due to her prior abdominal surgery.
o the currently per ormed bariatric procedures. B. T e surgery has a 50% chance o improving her
C. T e duodenal switch is the most restrictive o the diabetes, but only af er she has achieved signi -
currently per ormed procedures. cant weight loss.
D. Candidates or the duodenal switch must have a C. T e surgery has a near 100% likelihood o curing
body mass index above 55 kg/m 2. her gastroesophageal re ux disease.
E. T is patients medical comorbidities disquali es D. She can expect to lose 40% to 50% o her excess
her or bariatric surgery. body weight in the rst year af er surgery.
E. T e duodenal switch is associated with an 80% to
2. T e patient is con used about the anatomic 90% rate o resolution or her hypertension, sleep
di erence between the duodenal switch and the apnea, and diabetes at one year af er surgery.
gastric bypass. Which o the ollowing statements
is correct about the duodenal switch versus gastric 4. T e patient also wishes to know about the risks o
bypass anatomy? surgery and common complications associated
A. Both the duodenal switch and the gastric bypass with the duodenal switch. Which o the ollowing
involve the creation o a small proximal gastric pouch. statements is correct?

http://surgerybook.net/
110 G EN ERAL S U RG ERY EXAM IN ATIO N AN D BO ARD REVIEW

A. Unlike the gastric bypass, there is no postopera- o intestine available or absorption. T e duodenal
tive risk o an internal hernia af er the duodenal switch (also known as the biliopancreatic diversion
switch procedure. with duodenal switch) involves the longest length o
B. Chronic diarrhea and steatorrhea may be a com- bypassed intestine, and thus is more malabsorptive
plication o this procedure. than other procedures such as the gastric bypass.
C. Protein and calorie malabsorption, that requires T e restrictive component o the duodenal switch
parenteral nutrition or surgical revision, only involves ormation o a gastric sleeve, which achieves
occurs in less than 1% o patients. moderate restriction. Other procedures such as the
D. Fat malabsorption may result in de ciencies o gastric bypass and adjustable gastric band are more
vitamins B6 and C. restrictive than the duodenal switch. Candidacy or
E. Symptoms o dumping syndrome af er a duode- a duodenal switch is the same as or any bariatric
nal switch are likely. procedure (BMI > 40 or > 35 with obesity-related
co-morbidities), although most surgeons reserve
5. T e patient undergoes an uncomplicated this option or higher BMI patients. Medical comor-
laparoscopic duodenal switch procedure and is bidities related to obesity are not contra-indications
discharged home on postoperative day 3. On post- to surgery, and improvement or resolution o these
operative day 5 she presents to the emergency comorbidities is the primary goal o bariatric surgery.
department with worsening abdominal pain,
distension, evers, and emesis. She is ebrile to 2. B. T e gastric bypass involves creation o a small
102.7F, tachycardic to 125, tachypneic, and has proximal gastric pouch that is highly restrictive. In
a systolic blood pressure o 80. Her abdomen contrast, the duodenal switch involves creation o a
is di usely tender. She states that she has had vertical sleeve gastrectomy (see Figure 31-1) and divi-
diarrhea or the past 3 days and has not been able sion o the duodenum just past the pylorus, resulting
to tolerate oral intake. Which o the ollowing in a greater gastric capacity and less restriction than
statements about the evaluation and management the gastric bypass. T e duodenal switch operation
o this patient is correct? preserves the pylorus and at least part o the antrum,
A. Intravenous uid resuscitation should not be which provides the advantage o controlled gastric
given due to the concern or possible volume emptying and avoidance o the dumping syndrome
overload and pulmonary edema. that is requently seen with the gastric bypass. T e
B. A normal upper GI series with oral contrast duodenal switch results in MORE bypassed intestine
(swallow study) will rule out an abdominal than the gastric bypass and is the most malabsorp-
source o pathology. tive bariatric procedure currently in use.
C. An urgent C Pulmonary Angiogram must be T e nal anatomy o the duodenal switch is as
obtained to rule out a pulmonary embolus. shown in Figure 31-1: T e alimentary limb (AL,
D. T e patient will likely require emergent surgical and analogous to the roux limb in gastric bypass)
exploration to identi y the cause o her symptoms. is typically 150 to 200 cm long and the proximal
E. T e most likely cause o her symptoms is an end is anastomosed to the duodenum just past the
internal hernia. pylorus. T e biliopancreatic limb (BPL) is typically
long and unmeasured. T e small intestine distal to
the junction o the AL and BPL is called the com-
ANSWERS mon channel or common limb (CL), and is typically
1. B. All currently per ormed bariatric procedures 50 to 150 cm in length rom the ileocecal valve. T e
achieve weight loss primarily through either restric- amount o malabsorption is primarily a unction o
tion, malabsorption, or both. Restriction is achieved the length o the common channel, as this is where
by reducing the volume capacity o the stomach nutrients are mixed with the biliopancreatic diges-
through reduction in size or external compression. tive enzymes. A shorter common channel will result
T e Sleeve Gastrectomy and Adjustable Gastric Band in greater malabsorption, and vice-versa.
are the only endorsed restrictive procedures. Malab-
sorption is achieved through re-routing o the small 3. E. Among the currently per ormed bariatric pro-
intestine and digestive enzymes to decrease the length cedures, the duodenal switch has been associ-

http://surgerybook.net/
C H AP TER 3 1 BI LI O PAN C REATI C D IVERS IO N WITH D U O D EN AL S WITC H f O R M O RBID O BES I T Y 111

Bilio panc re atic dive rs io n with duo de nal s witc h

Alime nta ry
limb
Biliopa ncre a tic
limb

Common cha nne l


Figure 31-1 Diagram o the duodenal switch anatomy including the ali-
mentary limb (AL), the biliopancreatic limb (BPL), and the common limb or
common channel (CL). Reprinted with permission (Gagner M et al. 2009).

ated with the greatest absolute and relative weight surgery, and prior pelvis or laparoscopic abdominal
loss (Figure 31-2), as well as the highest rates o surgery is not a contraindication to a laparoscopic
improvement or resolution o obesity-related comor- approach. T e duodenal switch has been reported to
bidities. T e duodenal switch can be per ormed as achieve an average 80% to 100% rate o improvement
an open or a laparoscopic procedure, although it is or resolution o type 2 diabetes and normalization o
arguably the most technically challenging bariatric HgBA1C levels (Figure 31-3).
procedure to per orm laparoscopically. T is patient
7
would be an excellent candidate or laparoscopic
Roux-e n-Y ga s tric bypa s s
n
i
b
o
l
6
g
Biliopa ncre a tic dive rs ion
o
m
RYGB Roux-e n-Y ga s tric bypa s s with duode na l switch
e
BP D/DS Biliopa ncre a tic dive rs ion with duode na l s witch
h
5
d
BPD/DS RYGB
e
t
a
0
l
o
yc
4
l
G
20
s
s
%
o
l
rs
-
3
I
a
M
40 a
r
ye
ye
B
ly 3
e 1
s
tiv
s
60 ra
e
e
xc
p
o
E
re
80 P
%
Figure 31-3 Average change in glycolated hemoglobin
100
(HgBA1C) levels at 1 and 3 years postoperatively in a pro-
p <0.001
spective randomized study o duodenal switch versus gastric
Figure 31-2 Average weight loss (percent BMI lost) at 4 years bypass. Despite starting at a higher HgBA1C level at baseline,
or duodenal switch (BPD/DS) versus gastric bypass (RYGB) patients undergoing duodenal switch demonstrated lower
patients in a prospective randomized study. Figure repro- levels at 1 and 3 years. Figure reproduced with permission.
duced with permission (Hedberg J et al. 2012). (Hedberg J et al. 2012).

http://surgerybook.net/
11 2 G EN ERAL S U RG ERY EXAM I N ATI O N AN D BO ARD REVIEW

In addition, there is of en a marked improvement Dumping syndrome is very uncommon af er the


in diabetes immediately af er surgery and this has duodenal switch as the pylorus is preserved and gas-
been attributed to both the marked change in dietary tric emptying should be more controlled compared
intake as well as the bypass o digested ood away to the gastric bypass.
rom the duodenopancreatic complex. One caveat
to the high rates o improvement o obesity-related 5. D. Bariatric surgical patients are at risk or a number
comorbidities is that gastroesophageal re ux disease o potential perioperative complications in the early
may not improve, or may even worsen, af er convert- postoperative period. T ese include surgical site
ing the stomach to a sleeve gastrectomy con gura- in ections, venous thrombosis, pulmonary embo-
tion. T e percent o excess weight loss seen with the lus, anastomotic or staple line leaks, bleeding, iat-
duodenal switch is typically in the 60% to 80% range rogenic bowel injury, and early postoperative bowel
at one year, and has been shown to be superior to obstruction. T is patients presentation is extremely
gastric bypass or sleeve gastrectomy alone in multi- concerning due to her abnormal hemodynamics,
ple series. Resolution rates o 80% to 100% or other abdominal pain, and ever. T e primary concerns in a
weight related comorbidities including hyperten- patient presenting like this af er a duodenal switch
sion, sleep apnea, hyperlipidemia, and hypertriglyc- are or an anastomotic leak, staple line leak ( rom
eridemia have been demonstrated, and the duodenal the sleeve gastrectomy or the duodenal stump), or an
switch has been shown to result in superior reduc- iatrogenic bowel injury. Initial management should
tion in cardiovascular risk pro les. ocus on resuscitation o the patient, identi cation o
the source o the problem (i possible), and prepa-
4. B. An internal hernia can occur ollowing any sur- ration or likely operative intervention. A normal
gery that involves division o the small intestine and swallow or upper GI study does not de nitively
then per ormance o an anastomosis, and is one rule out an anastomotic leak, and does not evaluate
o the most eared late complications o the duode- the duodenal stump or the distal anastomosis.
nal switch or gastric bypass. Any acute small bowel Although a pulmonary embolus is possible, this
obstruction or chronic abdominal pain syndrome in patients constellation o symptoms are much more
these patients should prompt immediate evaluation consistent with an acute abdominal pathology and
or an internal hernia. T e duodenal switch primar- intervention should not be delayed to obtain a C
ily a ects absorption o ats and proteins, and the scan or a less likely diagnosis. T is patient most
at malabsorption may result in signi cant diarrhea likely has a leak rom a staple line or anastomosis
and steatorrhea postoperatively. T e duodenal switch resulting in peritonitis and sepsis. Immediate sur-
carries the highest risk o protein/calorie malnutri- gical exploration, either open or laparoscopically,
tion due to the high degree o malabsorption, with is indicated to identi y and correct the source o
approximately 5% o patients requiring parenteral the problem. I the patient presented with mini-
supplementation or surgical revision. mal symptoms and normal hemodynamics then a
Surgical revision or this problem usually thorough imaging evaluation would be indicated
involves lengthening o the common channel to to identi y or rule out a leak, and to direct possible
reduce the degree o malabsorption. T e high nonoperative management with placement o an
degree o at malabsorption a ects the absorption o intraluminal stent. Although an internal hernia with
the at soluble vitamins (A, D, E, and K), and these incarcerated and compromised bowel is possible,
should either be routinely supplemented or closely these are typically seen much later in the postopera-
monitored and selectively supplemented. Dump- tive period (months to years) af er signi cant weight
ing syndrome is thought to be due to rapid passage loss has occurred.
o high-carbohydrate and high-osmolar nutrients
BIBLIOGRAPHY
directly into the jejunum and is commonly seen
af er gastric bypass surgery. T is can cause immedi- Aasheim E , Bjorkman S, Sovik , et al. Vitamin status af er
bariatric surgery: A randomized study o gastric bypass
ate symptoms (early dumping) o abdominal pain,
and duodenal switch. Am J Clin Nutr. 2009;90:1522.
bloating, diarrhea, ushing, nausea, and emesis and Anderson B, Gill RS, de Gara CJ, et al. Biliopancreatic diver-
then delayed symptoms (late dumping) o sweat- sion: T e e ectiveness o duodenal switch and its limita-
ing, weakness, and dizziness due to hypoglycemia. tions. Gastroenterol Res Pract. 2013;2013:974762.

http://surgerybook.net/
C H AP TER 3 1 BILIO PAN C REATIC D I VERS I O N WI TH D U O D EN AL S WI TC H f O R M O RBI D O BES IT Y 113

Anthone GJ. Duodenal switch operation or morbid obesity. Hedberg J, Sundbom M. Superior weight loss and lower
Adv Surg. 2004;38:293309. HbA1c 3 years af er duodenal switch compared with
Banerjee A, Ding Y, Mikami DJ, et al. T e role o dumping Roux-en-Y gastric bypassA randomized controlled trial.
syndrome in weight loss af er gastric bypass surgery. Surg Surg Obes Relat Dis. 2012;8:33843.
Endosc. 2013;27:15738. Hedberg J, Hedenstrom H, Karlsson FA, et al. Gastric emp-
Biertho L, Lebel S, Marceau S, et al. Laparoscopic sleeve gas- tying and postprandial PYY response af er biliopancre-
trectomy: With or without duodenal switch? A consecutive atic diversion with duodenal switch. Obes Surg. 2011;21:
series o 800 cases. Dig Surg. 2014;31:4854. 60915.
Blachar A, Federle MP. Internal hernia: an increasingly com- Hedberg J, Sundstrom J, Sundbom M. Duodenal switch versus
mon cause o small bowel obstruction. Semin Ultrasound Roux-en-Y gastric bypass or morbid obesity: Systematic
CT MR. 2002;23:17483. review and meta-analysis o weight results, diabetes resolu-
Buchwald H, Kellogg A, Leslie DB, et al. Duodenal switch tion and early complications in single-centre comparisons.
operative mortality and morbidity are not impacted by Obes Rev. 2014;15:55563.
body mass index. Ann Surg. 2008;248:5418. Hess DS, Con erence ABSC. Biliopancreatic diversion with
Causey MW, Fitzpatrick E, Carter P. Pressure tolerance o newly duodenal switch. Surg Obes Relat Dis. 2005;1:32933.
constructed staple lines in sleeve gastrectomy and duodenal Hess DS, Hess DW. Biliopancreatic diversion with a duodenal
switch. Am J Surg. 2013;205:5714; discussion 5745. switch. Obes Surg. 1998;8:26782.
Chandler RC, Srinivas G, Chintapalli KN, et al. Imaging in Iqbal A, Miedema B, Ramaswamy A, et al. Long-term outcome
bariatric surgery: a guide to postsurgical anatomy and af er endoscopic stent therapy or complications af er bar-
common complications. AJR Am J Roentgenol. 2008;190: iatric surgery. Surg Endosc. 2011;25:51520.
12235. Jacobsen HJ, Nergard BJ, Lei sson BG, et al. Management o
Chiu CC. Randomized clinical trial o laparoscopic gastric suspected anastomotic leak af er bariatric laparoscopic
bypass versus laparoscopic duodenal switch or superobe- Roux-en-y gastric bypass. Br J Surg. 2014;101:41723.
sity. Br J Surg. 2010;97:1606. Kawkabani Marchini A, Denys A, Paroz A, et al. T e our di -
Comeau E, Gagner M, Inabnet WB, et al. Symptomatic erent types o internal hernia occurring af er laparascopic
internal hernias af er laparoscopic bariatric surgery. Surg Roux-en-Y gastric bypass per ormed or morbid obe-
Endosc. 2005;19:349. sity: are there any multidetector computed tomography
Dolan K, Hatzi otis M, Newbury L, et al. A clinical and nutri- (MDC ) eatures permitting their distinction? Obes Surg.
tional comparison o biliopancreatic diversion with and 2011;21:50616.
without duodenal switch. Ann Surg. 2004;240:516. Laurenius A, Olbers , Naslund I, et al. Dumping syndrome
Dorman RB, Rasmus NF, al-Haddad BJ, et al. Bene ts and ollowing gastric bypass: validation o the dumping symp-
complications o the duodenal switch/biliopancreatic tom rating scale. Obes Surg. 2013;23:74055.
diversion compared to the Roux-en-Y gastric bypass. MacPherson BH. Iron absorption and the duodenal switch
Surgery. 2012;152:75865; discussion 76567. operation. Obes Surg. 1999;9:2212.
DuPree CE, Blair K, Steele SR, et al. Laparoscopic sleeve gastrec- McConnell DB, ORourke RW, Deveney CW. Common chan-
tomy in patients with preexisting gastroesophageal re ux nel length predicts outcomes o biliopancreatic diversion
disease: a national analysis. JAMA Surg. 2014;149:32834. alone and with the duodenal switch surgery. Am J Surg.
Elder KA, Wol e BM. Bariatric surgery: a review o procedures 2005;189:53640; discussion 540.
and outcomes. 2007;132:225371. Mason RJ. Duodenal switch or morbid obesity: is it sa e? Adv
Ellison SR, Ellison SD. Bariatric surgery: a review o the Surg. 2013;47:15376.
available procedures and complications or the emergency Nelson D, Porta R, Blair K, et al. T e duodenal switch or
physician. J Emerg Med. 2008;34:2132. morbid obesity: Modi cation o cardiovascular risk mark-
Ferchak CV, Meneghini LF. Obesity, bariatric surgery and type ers compared with standard bariatric surgeries. Am J Surg.
2 diabetesa systematic review. Diabetes Metab Res Rev. 2012;203:603608.
2004;20:43845. Prachand VN, Davee R , Alverdy JC. Duodenal switch
Frenken M, Cho EY, Karcz WK, et al. Improvement o type 2 provides superior weight loss in the super-obese
diabetes mellitus in obese and non-obese patients af er the (BMI > or =50 kg/m 2) compared with gastric bypass. Ann
duodenal switch operation. J Obes. 2011;2011:860169. Surg. 2006;244:61119.
Gagner M, Deitel M, Kalberer L, et al. T e Second Interna- Prachand VN, Ward M, Alverdy JC. Duodenal switch pro-
tional Consensus Summit or Sleeve Gastrectomy, March vides superior resolution o metabolic comorbidities
1921, 2009. Surg Obes Relat Dis. 2009;5:47685. independent o weight loss in the super-obese (BMI > or
Gagner M, Boza C. Laparoscopic duodenal switch or morbid = 50 kg/m 2) compared with gastric bypass. J Gastrointest
obesity. Expert Rev Med Devices. 2006;3:10512. Surg. 2010;14:21120.
Gagner M, Matteotti R. Laparoscopic biliopancreatic diversion Ren CJ, Patterson E, Gagner M. Early results o laparoscopic
with duodenal switch. Surg Clin North Am. 2005;85:1419; biliopancreatic diversion with duodenal switch: a case
xxi. series o 40 consecutive patients. Obes Surg. 2000;10:514
Hamoui N, Chock B, Anthone GJ, et al. Revision o the duode- 23; discussion 524.
nal switch: indications, technique, and outcomes. J Am Coll Sudan R, Jacobs DO. Biliopancreatic diversion with duodenal
Surg. 2007;204:6038. switch. Surg Clin North Am. 2011;91:128193, ix.

http://surgerybook.net/
114 G EN ERAL S U RG ERY EXAM IN ATI O N AN D BO ARD REVIEW

Sovik , Aasheim E , aha O, et al. Weight loss, cardiovas- Roux-en-Y gastric bypass, and biliopancreatic diver-
cular risk actors, and quality o li e af er gastric bypass sion with duodenal switch. Surg Obes Relat Dis. 2012;8:
and duodenal switch: a randomized trial. Ann Intern Med. 2504.
2011;155:28191. Ukleja A, Stone RL. Medical and gastroenterologic manage-
Strain GW, Gagner M, Inabnet WB, et al. Comparison o ment o the post-bariatric surgery patient. J Clin Gastroen-
e ects o gastric bypass and biliopancreatic diversion with terol. 2004;38:31221.
duodenal switch on weight loss and body composition 1-2 Yimcharoen P, Heneghan HM, ariq N, et al. Endoscopic stent
years af er surgery. Surg Obes Relat Dis. 2007;3:316. management o leaks and anastomotic strictures af er ore-
opart P, Becouarn G, Ritz P. Comparative early outcomes o gut surgery. Surg Obes Relat Dis. 2011;7:62836.
three laparoscopic bariatric procedures: sleeve gastrectomy,

http://surgerybook.net/
32
Bariatric Surgery
Nutrition Complications
Marcos Molina

A 35-year-old emale with past medical history o A. Iron de ciency anemia


hypertension, ype II diabetes mellitus, and morbid B. Megaloblastic anemia
obesity recently underwent a gastric bypass surgery C. Spherocytosis
8 months ago without complication. On her last visit D. Anemia o chronic disease
o ce visit 2 months ago, no complications were noted
and weight was ound to be appropriate. She is currently ANSWERS
mourning the loss o her grandmother who passed
away 2 weeks ago. oday, her daughter noticed that her 1. D. Vitamin de ciency is a very common complica-
gait was very unsteady and she seemed to be con used. tion a er bariatric surgery due to the bypass itsel
She also mentioned that she had very poor appetite and and the partial gastrectomy, which reduces the
was very unclear i she has been taking her medications. absorptive sur ace area o the stomach. T ere ore, all
T ere is no history o alcohol intake. She was taken to patients post-surgery are discharged with multivita-
the emergency department or urther evaluation. mins supplements or li e that includes thiamine. T is
case describes the constellation o symptoms con-
1. What is the vitamin de ciency most likely sistent with acute Wernicke encephalopathy (WE),
associated with the above presentation? which classically presents with the symptoms o
A. Vitamin D dementia, ataxic gait, and ocular problems. T ere are
B. Folate many other micronutrient and vitamin de ciencies
C. Vitamin A such as calcium, vitamin B12, at-soluble vitamins
D. T iamine (A,D,E,K), olate, and other minerals. Supplementa-
E. Cobalamine tion o these vitamins is essential post-op.

2. What is the next step in management? 2. C. WE management requires prompt identi ca-
A. C scan o the brain tion, hydration, and administration o IV thiamine
B. Hydration and immediate administration o only based on clinical grounds. T e recommended
100 mg o IV vitamin B1 regimen is 500 o IV thiamine over 30 minutes three
C. Hydration, T iamine, ollowed by glucose times a day or 2 consecutive days or intramuscular
D. Hydration glucose ollow immediately by or 5 days in combination with other vitamins that
thiamine include B12 and olate. It important to administer
E. Oral T iamine glucose a er thiamine de ciency and not be ore
since it is known to worsen the WE symptoms.
3. What is the most common hematologic nding
af er Roux-en-Y gastric bypass (RYGB) gastric 3. A. Iron de ciency anemia is commonly seen in
bypass? patients who have undergone RYGB secondary

http://surgerybook.net/
116 G EN ERAL S U RG ERY EXAM I N ATIO N AN D BO ARD REVI EW

to the decrease in pH in the gastric pouch, which Love AL, Billett HH. Obesity, bariatric surgery, and iron de -
af ects absorption in the duodenum and proximal ciency: true, true, true and related. Am J Hematol. 2008;
jejunum. T e anemia is more evident in the men- 83:403.
Obinwanne KM, Fredrickson KA, Mathiason MA, et al. Inci-
struating emale, there ore, prophylactic iron sup- dence, treatment, and outcomes o iron de ciency a er
plementation is shown to decrease the incidence o laparoscopic Roux-en-Y gastric bypass: A10-year analysis.
this presentation. J Am Coll Surg. 2014;218:246.
Salgado W Jr, Modotti C, Nonino CB, Ceneviva R. Anemia
and iron de ciency be ore and a er bariatric surgery. Surg
BIBLIOGRAPHY Obes Relat Dis. 2014;10:49.
Brolin RE, Gorman JH, Gorman RC, et al. Prophylactic iron Singh S, Kumar A. Wernicke encephalopathy a er obesity sur-
supplementation a er Roux-en-Y gastric bypass: a pro- gery: a systematic review. Neurology. 2007;68:807.
spective, double-blind, randomized study. Arch Surg. 1998; T omson AD, Ryle PR, Shaw GK. Ethanol, thiamine and brain
133:740. damage. Alcohol Alcoholism. 1983;18:27.
Cook CC, Hallwood PM, T omson AD. B Vitamin de ciency Wordem, RW, Allen HM. Wernickes encephalopathy a er
and neuropsychiatric syndromes in alcohol misuse. Alcohol gastric bypass that masqueraded as acute psychosis: A case
Alcoholism. 1998;33:317. report. Curr Surg. 2006;63(2):1146.

http://surgerybook.net/
33
Revisional Bariatric Surgery

Hussna Wakily

A 45-year-old emale with a history o type 2 diabetes, D. A 42-year-old male with a history o RYGB with
hypertension, sleep apnea on CPAP, hyperlipidemia, a BMI o 32.
and a body mass index (BMI) o 42 presents to clinic E. A 25-year-old emale with a sleeve gastrectomy,
with ailure to lose weight a er having a laparoscopic a BMI o 35, and poor compliance with diet and
adjustable gastric band placed our years ago. She has exercise.
been compliant with diet and exercise but has not been
able to lose weight with the band. She is interested in 3. An upper gastrointestinal series can be used as part
the Roux-en-Y gastric bypass (RYGB) and would like to of the preoperative work up to evaluate for revision
know her options or revisional surgery. surgery. What will an upper gastrointestinal series
miss that will require a revision?
1. Who is the ideal candidate for bariatric surgery? A. Prolapse or slippage o adjustable gastric band
A. A 50-year-old emale with a BMI o 37 with no B. Marginal ulcers
comorbidities. C. Anastomotic leaks
B. A 34-year-old male with history o GERD and a D. Gastro-gastric stulae
BMI o 34.
C. A 19-year-old emale with a history o type 1 dia- 4. Patients who have revisional bariatric surgery for
betes, and a BMI o 32. weight recidivism:
D. A 48-year-old male with a history o hyperten- A. Have a higher risk o intra-operative and post-
sion, type 2 diabetes, and a BMI o 42. operative complications.
E. An 89-year-old male with a history o pulmonary B. Have a predictable amount o weight loss.
embolus, pulmonary hypertension, and a BMI C. Do not necessarily need consultation with
o 39. behavior health and nutrition care providers.
D. Usually do so due to a technical complication
2. In which scenario would a revisional weight loss rom surgery.
surgery be most indicated?
A. A 45-year-old emale with a history o a lap band
ANSWERS
with poor results, a BMI o 44 and poor compli-
ance with diet and exercise. 1. D. Ideal candidates or bariatric surgery are those
B. A 38-year-old emale with a history o lap band that have a BMI > 40, or a BMI > 35 and at least
with poor results, persistent hypertension, a BMI one weight-related co morbidity such as hyperten-
o 39, and compliance with nutrition and exercise. sion, hyperlipidemia, diabetes, sleep apnea, depres-
C. An 87-year-old male with a history o RYGB sion, arthritis, or pseudotumor cerebri. T e National
with poor results, a BMI o 43, active smoker, and Institutes o Health (NIH) consensus guidelines
drinks alcohol on weekends. or bariatric surgery are the most commonly used

http://surgerybook.net/
118 G EN ERAL S U RG ERY EXAM IN ATIO N AN D BO ARD REVI EW

criteria or identi ying patients that will bene t most excessive weight loss, and nutritional de ciencies. It
rom weight loss surgery. Patient D clearly meets the is crucial to identi y the cause o ailure prior to con-
NIH guidelines or bariatric surgery and represents sidering revision.
the best candidate among these choices. However, it Another source o ailure can be behavioral mal-
should be noted that recent evidence and consensus adaptation or non-compliance. It is critical to have a
opinions support extension o bariatric candidacy in standardized approach to the bariatric patient seek-
lower BMI patients, particularly in patients with a ing revisional surgery, including complete evalua-
BMI o 30 to 35 and type 2 diabetes. tions o their understanding and compliance with the
T e importance o considering weight loss sur- required diet and nutritional strategies to maximize
gery is due to the widespread increase in obesity the success o bariatric surgery. I this is the source
and associated co morbidities in the population. than behavioral adjustment may be more bene cial
T ere have been many studies showing that surgical as opposed to undergoing revision surgery. Patients
therapy is more e ective than medical intervention who are compliant are probably the best candidates
or supervised diet programs at achieving signi cant in which to consider a revision. T ere ore, identi y-
and sustained weight loss and resolution o associ- ing the source o the ailure is key to considering who
ated co morbidities. will bene t rom revision surgery.
T ese patients must undergo preoperative screen-
ing prior to surgery that entails psychiatric evalua- 3. B. T orough evaluation o the source o the ail-
tion, nutritional counseling, and medical clearance. ure is important to decide which surgery would be
T ey must be educated very thoroughly on the realis- most bene cial, or i revisional surgery is indicated
tic outcomes and work that is involved in undergoing at all. An upper gastrointestinal (UGI) series is usu-
weight loss surgery. T e expected weight loss rom ally used preoperatively to help clari y the anatomy
these procedures range rom 40% to 80% o their o prior surgeries and identi y the presence o com-
excess weight (calculated rom ideal body weight). plications. It can reliably help identi y band slippage
or prolapse, esophageal dilation, leaks or stulae. A
2. B. Revisions are necessary or complications o the band that has slipped or caused a prolapse must be
initial bariatric procedure or or ailure o weight removed because the prolapse can cause an incarcer-
loss and/or control o weight-related comorbidities. ation o the stomach leading to a surgical emergency.
Surgery is considered a ailure i the postoperative A gastro-gastric stula may also be seen but not every
BMI remains > 35 less than 50% excess weight loss stula requires a revision. I the stula is causing
achieved, the presence o persistent comorbidities, recurrent marginal ulcerations or weight regain then
or there is signi cant weight regain. Complications the patient can then be considered or a revision. It
rom the initial procedure that might require surgical must always be appreciated that an UGI series may
revision can include strictures, ulceration, stulae, not identi y small leaks or stulae, and many cent-
dysphagia, severe re ux, or nutritional de ciencies. ers now per orm an UGI study ollowed immediately
T ese may require surgical revision, conversion to a by a C scan. T is approach signi cantly increases
di erent bariatric procedure, or in severe cases even the sensitivity or identi ying smaller leaks, stulae,
reversal o the bariatric procedure. or other anatomic complications. Small leaks seen on
Identi ying the source o the complication is criti- UGI can be controlled by drainage, PN, NPO status,
cal to determining candidacy or revision and decid- or with the adjunct o a stent or clip place endoscopi-
ing on the type o revisional procedure. Anatomic cally. An UGI study typically lacks the resolution to
complications can be based on type o surgery. Com- identi y subtle mucosal problems such as a marginal
mon adjustable gastric band complications include ulcer. Diagnosis o marginal ulceration typically
band slippage or gastric prolapse through the band, requires direct luminal visualization with exible
band erosion, and pouch or esophageal dilation. In upper endoscopy.
addition, the adjustable gastric band has the highest
rate o ailure o weight loss or requiring band revi- 4. A. Revisional surgery is signi cantly more complex
sion or removal. Complications o RYGB include than an initial bariatric procedure or multiple reasons,
marginal ulcer, internal hernias, bowel obstructions, including the altered anatomy, presence o signi cant
gastro-gastric stulas, steatorrhea, inadequate or adhesions or in ammatory changes, and the need or

http://surgerybook.net/
C H AP TER 3 3 REVIS I O N AL BARI ATRIC S U RG ERY 119

more complex surgical maneuvers and reconstruc- BIBLIOGRAPHY


tions. T ere have been several studies that have shown Buchs NC, Bucher P, Pugin F, et al. Value o Per orming Rou-
that patients who have undergone revision surgery tine Postoperative Liquid Contrast Swallow Studies Fol-
have a higher short and long term complication rate. lowing Robot-assisted Roux-en-Y Gastric Bypass. Swiss
T ese rates range rom 13% to 50% depending on Med Wkly. 2012;142:w13556.
Buchwald H. Avidor Y, Braunwald E, et al. Bariatric Surgery:
the type o revision involved, most commonly being A Systematic Review and Meta-analysis. JAMA. 2004;
wound in ections or surgical site in ections. Signi - 292(14):172437.
cantly longer operative times and greater blood loss Gastrointestinal Surgery or Severe Obesity. NIH Consensus
have also been noted. T e mortality rate remains at Development Con erence. Nutrition 1996;12(6):397404.
less than 1%, similar to initial weight loss surgeries. Grif th PS, Birch DW, Sharma AM, et al. Managing Com-
plications Associated with Laparoscopic Roux-en-Y
T e reported results o additional weight loss a er
Gastric Bypass or Morbid Obesity. Can J Surg. October
revisional surgery are also signi cantly more variable 2012;55(5):32936.
than a er an initial bariatric procedure. Inabnet WB 3rd, Belle SH, Bessler M, et al. Comparison o
T ere are multiple options or revision o a bari- 30-day Outcomes a er Non-LapBand Primary and Revi-
atric procedure, and the choice will depend on the sional Bariatric Surgical Procedures rom the Longitudinal
current anatomy and the reason or the revisional Assessment o Bariatric Surgery study. Surg Obes Relat Dis.
2010;6(1):2230.
procedure. T ere is also a signi cant current interest Karmali S, Brar B, Shi X, et al. Weight Recidivism Post-bariatric
and developing technology in the eld o endolumi- Surgery: A Systematic Review. Obes Surg. 2013;23(11):
nal approaches to both primary and revisional bari- 192233.
atric surgery, most commonly to achieve reduction Mikami D, Needleman B, Narula V, et al. Natural Ori ce
in the size o the gastric pouch or decrease the size Surgery: Initial US Experience Utilizing the StomaphyX
Device to Reduce Gastric Pouches a er Roux-en-Y Gastric
o a dilated upper anastomosis to produce additional
Bypass. Surg Endosc. 2010;24(1):2238.
restriction. Weight loss results are more variable and Nesset EM, Kendrick ML, Houghton SG, et al. A wo-decade
less predictable compared to primary bariatric sur- Spectrum o Revisional Bariatric Surgery at a ertiary
gery, multiple studies have shown acceptable success Re erral Center. Surg Obes Relat Dis. 2007;3(1):2530.
rates with weight loss a er revisional surgeries. Sjstrm L. Li estyle, Diabetes, and Cardiovascular Risk Fac-
Weight regain a er a primary bariatric proce- tors 10 Years a er Bariatric Surgery. N Engl J Med. 2004;
351(26):268393.
dure can be multi actorial but o en it is due to poor ran , Pauli E, Lyn-Sue JR, et al. Revisional Weight
dietary and exercise habits and not due to a technical Loss Surgery a er Failed Laparoscopic Gastric Banding:
complication rom the surgery. As such, all patients An Institutional Experience. Surg Endosc. 2013;27(11):
who being considered or revision need consultation 408793.
with behavior health and nutrition services. Follow Unick JL, Beavers D, Bond DS, et al. T e long-term e ective-
ness o a li estyle intervention in severely obese individu-
up a er surgery should be li e-long to help with
als. Am J Med. 2013;126(3):23642.
long-term weight management control.

http://surgerybook.net/
34
Adjustable Gastric Band
Complications
Ashley D. Willoughby

A 37-year-old emale presents to the general surgery discharged. Six months a er the operation, the patient
clinic or pre-operative evaluation to undergo a laparo- presents to the emergency room with intractable nausea
scopic adjustable gastric band placement. Her current and vomiting, intolerance to ood intake. T e patient is
weight is 220 lbs (99.8 kg) with a BMI o 35.5 kg/m 2. admitted or urther evaluation and kept NPO with IV
She was diagnosed with type 2 diabetes last year and uids. A swallow study was per ormed, results o which
her primary care physician is currently evaluating her are shown.
or obstructive sleep apnea (OSA). T e patient under-
goes her surgery without intraoperative complications. 1. With regards to outcomes from laparoscopic
Post-operatively she has a ew episodes o emesis; how- adjustable gastric banding (LAGB) placement,
ever, she eventually is able to tolerate clear liquids and is which of the following is true?
A. Excess weight loss at 2 years is about 60% with a
narrow range.
B. Excess weight loss is about 50% with a wide
range.
C. Excess body weight loss is about 45% at 5 years.
D. Inadequate weight loss at 2 years occurs 15% o
the time.
E. Hypertension and hyperlipidemia resolution
is equivalent to the Roux-en-Y gastric bypass
(RYGB).

2. Which of the following is the most common


complication of LAGB placement?
A. Gastric prolapse
B. ubing/access port problems
C. Esophageal dilation
D. IVC inclusion in the band
E. Gastric erosion

3. T e radiology study shown demonstrates:


A. Gastric prolapse
B. Gastric dilation
C. Esophageal ailure

http://surgerybook.net/
C H AP TER 3 4 Ad j u s TAb l E G As TRi C b An d Co m P l i C ATi o n s 121

D. IVC inclusion in the band 3. A. Band prolapse occurs when part o the stom-
E. Gastric erosion ach herniates under the band. Gastric prolapse
symptoms include dysphagia, worsening Gastroe-
4. Which of the following intraoperative or post- sophageal Re lux Disease (GERD), nausea/emesis,
operative strategies described below could have the and ood intolerance. Patients are at an increase
greatest impact on preventing this complication? in risk or gastric prolapse i they have emesis
A. Small portion sizes immediately post-op, which can dislodge sutures
B. A rebuckling procedure that were placed using the gastroplexy technique
C. Pars accida technique to avoid this complication. A contrast or upper
D. Remove all uid rom the band GI study is used to make the diagnosis. he study
E. Gastropexy will likely show contrast pooling in the prolapsed
stomach with the band in a horizontal position or
5. What is the ideal treatment for this complication? an anterior prolapse (as is the case in the above
A. Unbuckling, reduction, and rebuckling pro- scenario) or in a vertical position or a posterior
cedure prolapse. I the gastric wall remains prolapsed it
B. Removal with plans to per orm an alternative can become strangulated and may result in gastric
weight loss procedure incarceration.
C. Removal o all uid in band port
D. Band de ation and begin proton pump inhibitor 4. E. Restricting the patient to small portion sizes
E. Per orm an esophagogastroduodenoscopy will prevent gastric dilation, which can lead to
(EGD) and then remove band i indicated gastric prolapse, but this is not the most e ective
method in preventing this complication. Educat-
ing patients on the importance o portion size does
ANSWERS play a large role in their post-op weight loss success.
1. B. T e Centers or Disease Control and Preven- An unbuckling, reduction, and rebuckling proce-
tion (CDC) developed the BMI chart comparing an dure is per ormed by some surgeons to f x a gastric
individuals height and weight to determine i they prolapse; however, recurrence is common with this
all within a healthy range. T e excess weight can procedure. Removing uid rom the port to a zero
be calculated by calculating the individuals current f ll volume is the initial treatment or gastric pro-
BMI and then determining what their weight should lapse and will not necessarily prevent gastric pro-
be at that height and subtracting it rom the ormer. lapse nor would it assist in weight loss with zero
Expected excess weight loss a er undergoing a gas- f ll. Per orming a pars accida technique has been
tric band operation at 2 years and 5 is 52% and 56% shown to decrease the incidence o gastric prolapse
respectively with standard deviation that ranges when compared to the older perigastric technique;
rom 2% to 15%. however placing gastrogastric sutures or per orm-
ing a gastropexy is currently considered the most
2. A. In a review o 2283 patients, a total o 238 (8.5%) e ective technique.
patients experienced a complication a er placement
o a LAGB. T e most requent was proximal pouch 5. B. T e unbuckling, reduction, and rebuckling pro-
dilation and gastric prolapse (4.2%). T e incidence cedure is used by some surgeons as a technique to
o gastric prolapse has decreased since changing repair gastric prolapse; however, recurrence rate has
rom the perigastric technique to the pars accida been ound to be quite high. Removal o all uid
technique and adding a gastrogastric suture or gas- in the band port would be considered the non-
troplexy technique. ubing/access port problems operative management or gastric prolapse. T e
were next at 1.2% as well as explantation. Erosion patient may be able to consume ood easier when
into the gastric lumen was the least requent in this uid is removed completely rom the band. Remov-
study at 0.5%. Inclusion o the IVC into the band can ing all uid in the band port would be the treatment
result in an intraabdominal or intrathoracic emer- o choice or gastric pouch dilation, or which the
gency and is a complication that is rare, but reported band would be in normal position on the barium
and described. swallow i this was the diagnosis. Band de ation and

http://surgerybook.net/
122 G En ERAl s u RG ERY EXAm i n ATi o n An d b o ARd REVi EW

beginning a proton pump inhibitor would be the BIBLIOGRAPHY


recommended treatment or a patient experienc- Coburn, Chris; Chapman, Mary Ann; Ali, Arlene; Amrhein,
ing an exacerbation o GERD symptoms a er band John. Five-year weight loss experience o outpatients
placement and may ultimately need a laparoscopic receiving laparoscopic adjustable gastric band surgery.
roux-en-y gastric bypass. Obesity Surgery. 2013;23:90310.
OBrien PE, and Dixon JB. Lap-Band: Outcomes and results.
An EGD would be indicated to rule out the pres- Journal of Laparoscopic and Advanced Surgical Techniques.
ence o a gastric erosion; and i one is seen then 2003;13(4):26570.
removal o the band and repair o the erosion is indi- Schirmer B, Schauer PR. T e Surgical Management o Obe-
cated. Gastric prolapse can result in a surgical emer- sity. In: Brunicardi FC, Andersen DK, Billiar R, Dunn DL,
gency i strangulation or incarceration occurs. I the Hunter JG, Matthews JB, et al. eds. Schwartzs Principles
of Surgery. 9e. 2010. Accessed rom http://accesssurgery.
patient is hemodynamically stable, then an elective
mhmedical.com/content.aspx?
procedure can be per ormed. T is patient would Snow J, Severson PA. Complications o adjustable gastric band-
benef t rom band removal initially to prevent the ing. Surgical Clinics of North America. 2011;91:124964.
incarceration and plan or a uture weight loss sur- Walker G. Devine ormula or ideal body weight. MDCalc.
gery i desired by the patient. March 03 2014. Accessed rom http://www.mdcalc.com/
ideal-body-weight/

http://surgerybook.net/
35
Gallbladder Mass

Douglas Farmer

A 66-year-old Japanese emale is re erred by her o the gallbladder wall on the portal phase o a
primary physician or long-standing biliary colic symp- multi-detector C scan (MDC ).
toms. She describes 10 to15 years o intermittent right C. Gallbladder polyp 1020 mm on ultrasound.
upper quadrant (RUQ) pain with nausea that typically D. Gallbladder wall calci cations; porcelain gall-
resolves a er 1 to 2 hours. She went to the emergency bladder.
room once 6 years ago and had an ultrasound that E. Asymptomatic gallstone greater than 3 cm in size.
showed gallstones. Her medical history is signi cant
or hypertension and osteoporosis. Her vitals and exam 3. Review o all imaging studies shows a gallbladder
are unremarkable. A repeat RUQ ultrasound ordered tumor invades into the muscularis propria. T ere
by her primary care physician now shows a large 3 cm is no lymphadenopathy or distant metastateses
gallstone, as well as a xed mass in the undus, 2 cm seen. What is the most appropriate next step in
in diameter, that appears to originate rom the gallblad- management?
der wall. T e immediate surrounding gallbladder wall is A. Re erral or de nitive treatment with chemo-
thickened to 8 to 11 mm. A CBC, basic chemistry, and therapy and radiation.
liver unction tests are all within normal limits. B. Re erral or neoadjuvant chemotherapy and
radiation.
1. Which o the ollowing is a risk actor or C. Schedule or cholecystectomy.
gallbladder carcinoma? D. Schedule or cholecystectomy with removal o
A. Hemolytic anemia regional lymph nodes and en-bloc hepatic resection.
B. Biliary dyskinesia
C. Clonorchis sinensis in ection 4. T e same patient with the same history o present
D. Anomalous union o the pancreaticobiliary ductal illness and past medical history instead presents
system or pancreaticobiliary maljunction (PBM) to the emergency room with RUQ pain. T e
E. Auto-immune diseases ultrasound in this case is read as 2 large gallstones
with di use gallbladder wall thickening up to
2. Which o the ollowing radiographic ndings is 11 mm, pericholecystic f uid, and a normal
associated with the highest incidence o gallbladder common bile duct. Labs show white blood cell
carcinoma? count o 13 and normal liver unction. You take the
A. Pancreaticobiliary maljunction (PBM) without patient or laparoscopic cholecystectomy.
biliary dilatation identi ed on magnetic reso- T ere was no concern or malignancy during the
nance cholangiopancreatography (MRCP). procedure. T e gallbladder was removed without
B. Strongly enhancing thick inner layer and a spillage and with a retrieval bag. You see her 2 weeks
weakly enhancing or nonenhancing outer layer later in clinic, and review o the pathology report

http://surgerybook.net/
12 4 G EN ERAL S U RG ERY EXAM I N ATI O N AN D BO ARD REVIEW

shows 2 adenocarcinoma. All surgical margins, T e progression to GBC is likely mediated through
including cystic duct margin, are reported as clear. an epithelial hyperplasia with resultant papillary or
What is the most appropriate course o action? villous epithelial changes progressing to GBC. Fur-
A. No additional surgery, surveillance with imaging ther evidence that this is a distinct pathway rom
every 6 months. chronic in ammation is that the gene alterations o
B. Staging with imaging ollowed by radical cho- cancers arising in the setting o cholelithiasis di er
lecystectomy to include: liver resection with at rom anomalies o the duct system associated can-
least 3 cm o margin around gallbladder bed and cers. Adenomas do occur in the gallbladder and can
regional lymphadenectomy. progress to cancer. However, this likely occurs much
C. Staging with imaging ollowed by radical cho- less commonly than the other two pathways, given
lecystectomy and in addition, excision o the a lack o cancer-related molecular changes in most
previous laparoscopic port sites. o these lesions. In ammatory bowel disease is also
D. Staging with imaging ollowed by radical cho- associated with increased risk or gallbladder carci-
lecystectomy and in addition, excision o the noma. Clonorchis in ection is associated with risk
common bile duct. or cholangiocarcinoma, but it has not been linked
E. Staging with imaging ollowed by radical chole- to carcinoma o the gallbladder. Chronic Salmonella
cystectomy and in addition, excision o both the typhi or paratyphi in ection is, however, associated
common bile duct and the laparoscopic port sites. with increased risk or gallbladder cancer. Hemolytic
anemia may be a cause o bilirubin type gallstones
5. Which o the ollowing is true regarding the but is not a risk actor or gallbladder cancer, neither
surgical management or 2 or 3 gallbladder is biliary dyskinesia or auto-immune diseases.
cancers?
2. B. A recent retrospective study o ndings on
A. Formal segmentectomy (4b + 5) improves over-
MDC associated with gallbladder cancer ound
all survival over wedge resection.
two patterns most associated with nding malig-
B. Formal segmentectomy reduces local recurrence
nancy at the time o surgery. A strongly enhancing
rates compared to wedge resection, but overall
thick inner layer and a weakly enhancing or non-
survival is the same.
enhancing outer layer o the gallbladder wall on
C. Formal segementctomy improves disease ree
multi-detector C scan was shown to have a 52% to
survival, but not overall survival.
55% incidence or gallbladder cancer. A single thick
D. A clear survival bene t or ormal segmentectomy
layer with heterogenous enhancement on MDC
over wedge resection has not been demonstrated.
had an incidence o 35% to 38% or gallbladder can-
cer. Pancreaticobiliary mal ormation (PBM) without
ANSWERS
bile duct dilatation have an incidence o biliary tract
1. D. A history o gallstones is common, and 65% to cancer o 37.9%, o which 93.2% o these were gall-
90% o those with biliary carcinoma, have a history bladder cancer. Based on this data, prophylactic chol-
o gallstones. T e relation o gallstones to gallbladder ecystectomy is recommended or these patients. T e
cancer (GBC) is thought to be mediated by chronic incidence rates o malignancy in gallbladder polyps
in ammation. T ere is a relatively well-de ned varies widely in published reports but ranges rom
sequence o at-epithelial premalignant changes 9.6% to 40% or polyps 10 to 20 mm. T e wide vari-
leading to GBC. Chronic in ammation leads to ance is related to various imaging modalities used
intermediate low-grade dysplastic changes. Dysplas- and populations studied. Polyps 10 mm, sessile
tic progression over time leads to carcinoma-in-situ polyps and rapidly growing polyps are all recom-
and nally invasive carcinoma. Anomalous union o mendations or gallbladder removal.
the pancreaticobiliary ductal system, where the pan- T e nding o gallbladder calci cations or a
creatic duct and common bile duct merge outside porcelain gallbladder was ound to associated with
the wall o the duodenum and orm a long common a malignancy in 6% o cases in a recent systematic
channel, is also associated with an increased risk o literature review. T is is a ar lower number than
GBC. T is pancreaticobiliary maljunction (PBM) the historically quoted gure o approximately 25%.
leads to chronic re ux o pancreatic enzymes. Attempts in the review were made to limit inherent

http://surgerybook.net/
C H AP TER 3 5 G ALLBLAD D ER MAS S 125

biases in a review o retrospective studies that avor such as resection o the common bile duct, and the
overestimation but this value likely still overestimates need or port site excision. Excision o the common
the true incidence. Given the incidence, the decision bile duct is only necessary or a positive cystic duct
to per orm a prophylactic cholecystectomy should margin or direct invasion o the hepatoduodenal
not be absolute and should be weighed against the ligament.
risks o surgery or the individual patient. T e pres- Regarding routine common bile duct excision
ence o gallstones is associated with an increased with radical cholecystectomy unless there is direct
risk o gallbladder cancer. T e size and volume o invasion o the hepatoduodenal ligament and/or o
stone burden have been identi ed as potential risk the cystic duct, bile duct resection does not result in
actors or developing gallbladder cancer. However, decreased recurrence or better overall survival and
there is no direct evidence o a causal relationship does not increase the number o nodes in the speci-
between gallstones and gallbladder cancer. T e risk men. Peritoneal involvement with gallbladder cancer
or gallbladder cancer development with a 3 cm or is common and there is theoretical adverse impact
greater stone has been estimated to be a 2% risk over on this with pneumoperitoneum. However, the risk
a 20 year period. o port site recurrence is based on per oration o
the gallbladder or extraction without a retrieval bag
3. D. T e patient has a 1b lesion or early gallbladder rather than the pneumoperitoneum. Port-site exci-
cancer. Surgical resection is the only curative therapy sion does not improve overall or disease- ree sur-
or gallbladder cancer. T e general consensus is or vival in large retrospective series. Port site excision
radical cholecystectomy or 1b or greater lesions. does not need be routinely per ormed during sec-
1b tumors have been shown to have lymph node ondary procedures or gallbladder cancer discovered
metastasis in 24% o cases. Lower recurrence rates a er laparoscopic cholecystectomy.
and improved survival have been observed with
radical resection including lymph nodes when com- 5. D. Although some studies have reported anatomi-
pared to simple cholecystectomy. A review o the cal resection improves survival and R0 resection rate
SEER database showed that the evaluation o even when compared to wedge resection, other reports
a single lymph node improved overall survival and have not demonstrated a bene t. T e majority o
that radical resection without lymph node assess- these studies dealt with liver resection or liver metas-
ment was no better than cholecystectomy alone or tases. T ere are a ew studies looking at gallbladder
early stage gallbladder cancer. Neoadjuvant therapy cancer speci cally. Pawlik et al. ound that patients
has been evaluated in the setting o borderline and who underwent a major hepatic resection (e.g., or-
unresectable extra-heaptic biliary malignancies with mal segmentectomy o 4b + 5 or hemi-hepatectomy)
good results in survival and obtaining negative mar- had a similar risk o disease-speci c death compared
gins but none o these addressed gallbladder cancer with patients who underwent a hepatic wedge resec-
speci cally and were con ned to advanced disease. tion. Horiguchi et al. ound the overall survival rate
Adjuvant chemoradiation does have a role in select and disease- ree survival rate at 5 years did not
gallbladder cancer patients, especially with positive di er signi cantly between wedge resection and
nodes or margins to improve local control. 4a + 5 resection group or 2 tumors. T e available
evidence in gallbladder cancer does not show a clear
4. B. Prognosis o gallbladder carcinoma is determined bene t to anatomic resection. As such, the surgeons
by the depth o tumor in ltration and the ability to goal should be to resect all disease with negative his-
obtain a tumor- ree resection margin (R0). For 2 or tologic margins and chose the appropriate operation
greater the de nitive resection should include a min- to achieve this with the ewest complications.
imal hepatic resection centered on the gallbladder
bed and a regional lymphadenectomy. T e majority
BIBLIOGRAPHY
o experts also extend this recommendation to 1b
Araida , Higuchi R, Hamano M, et al. Should the extrahepatic
tumors but some controversy persists. T ere is uni-
bile duct be resected or preserved in R0 radical surgery
orm agreement that cholecystectomy alone is suf - or advanced gallbladder carcinoma? Results of a Japanese
cient or is and 1a tumors. T ere is also general Society of Biliary Surgery Survey: A multicenter study. Surg
agreement regarding several other technical points, Today. 2009;39:7709.

http://surgerybook.net/
126 G EN ERAL S U RG ERY EXAM I N ATI O N AN D BO ARD REVIEW

Azuma , Yoshikawa , Araida , akasaki K. Di erential Lee J, Yun M, Kim KS, et al. Risk strati cation o gallbladder
diagnosis o polypoid lesions o the gallbladder by endo- polyps (12 cm) or surgical intervention with 18 F-FDG
scopic ultrasonography. Am J Surg. 2001;181(1):65. PE /C . J Nucl Med. 2012;53:3538.
Cavallaro A, Piccolo G, Panebianco V, et al. Incidental gall- Maker AV, Butte JM, Oxenberg J, et al. Is port site resection
bladder cancer during laparoscopic cholecystectomy: necessary in the surgical management o gallbladder can-
Managing an unexpected nding. World J Gastroenterol. cer? Ann Surg Oncol. 2012;19:40917.
August 14, 2012;18(30):401927. Marsh R DE W, Alonzo M, Bajaj S, et al. Comprehensive
Diehl AK. Gallstone size and the risk o gallbladder cancer. review o the diagnosis and treatment o biliary tract can-
JAMA. 1983;250:23236. cer 2012. PAR II: Multidisciplinary management. J. Surg.
Fetzner UK, Holscher AH, and Stippel DL. Regional lymph- Oncol. 2012;106:33945.
adenectomy strongly recommended in 1b gallbladder Maringhini A, Moreau JA, Melton LJ, et al. Gallstones, gall-
cancer. World J Gastroent. 2011;17(38):43478. bladder cancer, and other gastrointestinal malignancies.
Fuks D, Regimbeau JM, Pessaux P, et al. Is port-site resection An epidemiologic study in Rochester, Minnesota. Ann
necessary in the surgical management o gallbladder Intern Med. 1987;107(1):3035.
cancer? J Visc Surg. 2013;150:27784. Miyazaki M, akada , Miyakawa S, et al. Risk actors or
Goetze O, Paolucci V. Immediate re-resection o 1 incidental biliary tract and ampullary carcinomas and prophylactic
gallbladder carcinomas: a survival analysis o the German surgery or these actors. J Hepatobiliary Pancreat Surg.
Registry. Surg Endosc. 2008;22:24625. 2008;15:1524.
Horiguchi A, Miyakawa S, Ishihara S, et al. Gallbladder bed Numata K, Oka H, Morimoto M, et al. Di erential diagnosis
resection or hepatectomy o segments 4 a and 5 or p 2 o gallbladder diseases with contrast-enhanced harmonic
gallbladder carcinoma: Analysis o Japanese registration gray scale ultrasonography. J Ultrasound Med. 2007;26:
cases by the study group or biliary surgery o the Japanese 76374.
Society o Hepato-Biliary-Pancreatic Surgery. J Hepatobili- Pawlik M, Gleisner AL, Vigano L, et al. Incidence o nd-
ary Pancreat Sci. 2013;20(5):51824. ing residual disease or incidental gallbladder carcinoma:
Jensen EH, Abraham A, Habermann EB, et al. A critical analysis implications or re-resection. J Gastrointest Surg. 2007;11:
o the surgical management o early-stage gallbladder can- 147886.
cer in the United States. J Gastrointest Surg. 2009;13:7227. Schnelldor er . Porcelain gallbladder: A benign process or con-
Jensen EH, Abraham A, Jarosek S, et al. Lymph node evalua- cern or malignancy? J Gastrointest Surg. 2013;17:11618.
tion is associated with improved survival a er surgery or Sugiyama M, Atomi Y, Yamato . Endoscopic ultrasonography
early stage gallbladder cancer. Surgery. 2009;146:70613. or di erential diagnosis o polypoid gall bladder lesions:
Kim SJ, Lee JM, Lee JY, et al. Analysis o enhancement pattern Analysis in surgical and ollow up series. Gut. 2000;46:
o at gallbladder wall thickening on MDC to di erentiate 2504.
gallbladder cancer rom cholecystitis. AJR. 2008;191:76571. Vijayakumar A, Vijayakumar A, Patil V, et al. Early diagnosis
Kwon W, Jang J-Y, Lee SE, et al. Clinicopathologic eatures o gallbladder carcinoma: An algorithm approach. ISRN
o polypoid lesions o the gallbladder and risk actors o Radiol. 2013; Article ID 239424:16.
gallbladder cancer. J Korean Med Sci. 2009;24:4817.

http://surgerybook.net/
36
Biliary Colic

Alexander Malloy

A 67-year-old emale presents with sharp, burning right cholecystitis. Her pain again resolves and she is
upper quadrant abdominal pain that radiates to her referred to your clinic the following week. You
back. T e pain awakens her rom sleep. She has experi- discuss performing a cholecystectomy with the
enced nausea and three episodes o emesis. She reports patient, which she refuses. Which of the following
burning substernal and epigastric pain. She denies any is true regarding medical therapy of cholelithiasis?
evers or chills. She is hemodynamically normal and A. Ursodiol therapy typically resolves biliary symp-
a ebrile. Laboratory workup reveals a normal hepatic toms within one month.
unction panel, a normal basic chemistry, and a normal B. Complete resolution o cholelithiasis is success-
white blood cell count. Her abdominal exam is remark- ul in less than 40% o patients.
able or mild tenderness in the right upper quadrant C. I biliary symptoms persist while on ursodiol,
without peritoneal signs. A er three hours, her pain cholecystectomy is indicated.
resolves. D. T e mechanism o action o ursodiol includes
reduction in bile synthesis.
1. What is the next best test in this patients scenario? E. Extracorporeal shockwave lithotripsy (ESWL)
A. Plain lms o the abdomen has a symptomatic recurrence rate o 30%.
B. Cholecystokinin stimulated cholescintigraphy
C. Right upper quadrant ultrasound 4. T e patients ultrasound, in addition to
D. Computed tomography (C ) o the abdomen cholelithiasis, is also notable for a 1.5 cm non-
E. Esophagogastroduodenoscopy (EGD) mobile polypoid lesion in the fundus of the
gallbladder. Which of the following is true?
2. T e appropriate imaging is ordered and shows no A. Polyps greater than 5 mm are a risk actor or
abnormality. Which is the next most appropriate gallbladder cancer.
step in management? B. Patients with large (> 2.5 cm) gallstones are
A. Cholecystectomy more likely to develop gallbladder cancer than
B. Esophagogastroduodenoscopy those without.
C. Right upper quadrant ultrasound in two weeks C. Laparoscopic cholecystectomy should be per-
D. Proton pump inhibitor administration ormed, a rozen analysis should be done, and i
E. Bile microscopy positive, an oncologic resection should be done
laparoscopically.
3. T e patient returns to the emergency department D. Multiple pedunculated subcentimeter lesions are
one month later with similar symptoms. A also a risk actor or gallbladder adenocarcinoma.
right upper quadrant ultrasound is performed E. Extended cholecystectomy is not required or gall-
that shows gallstones without signs of acute bladder adenocarcinomas with up to 3 lesions.

http://surgerybook.net/
128 G EN ERAL S U RG ERY EXAM I N ATI O N AN D BO ARD REVI EW

ANSWERS Cholecystokinin stimulated cholescintigraphy can


help identi y a unctional gallbladder problem that
1. C. T is scenario describes symptomatic cholelithia- may warrant cholecystectomy i positive (Choice A).
sis, or biliary colic. Plain lms o the abdomen would Proton pump inhibitor therapy may help with re ux
not be a high yield study as less than 10% o gall- symptoms but does not speci cally address biliary
stones are radio-opaque, nor would it o er speci c symptoms (Choice D).
imaging o the biliary system. Other yields rom a
plain lm would be presence o nephrolithiasis or 3. B. Ursodiol may take up to 3 months to show
ree air (Choice A). A hepatobiliary (HIDA) scan is improvements in biliary symptoms, and can take up to
the gold standard or the diagnosis o acute chole- 3 years to completely dissolve gallstones (Choice A).
cystitisa clinical scenario in which she does not t A meta-analysis o treatment with ursodiol showed
criteria (Choice B). With the cholecystokinin injec- that only 37% o patients had complete resolution
tion, the nuclear imaging study can be diagnostic or o biliary symptoms; cholecystectomy remains the
biliary dyskinesia. C can o en visualize gallstones pre erred choice in surgical candidates (Choice B).
but is not the recommended imaging modality o Many patients who undergo initial medical therapy
choice or cholelithiasis as the sensitivity is 55% to with ursodiol do progress to surgical treatment, but
80% (Choice D); cholelithiasis is o en ound inci- they tend to show initial improvement in symptoms.
dentally on C scan when the imaging is per ormed I biliary symptoms progress while being treated
or other reasons. with ursodiol, other causes must be ruled out, such
Esophagogastroduodenoscopy could be help ul as sphincter o Oddi dys unction or peptic ulcer
to rule out peptic ulcer disease that can some- disease (Choice C). T e mechanism o action o
times mimic symptoms o biliary colic, but this test ursodiol involves reducing absorption in the duode-
would not be the ideal next choice in this situa- num, resulting in disruption o micelles and reduced
tion (Choice E). T e most common initial imaging cholesterol absorption, thereby decreasing choles-
modality in biliary disease is the right upper quad- terol concentration; ursodiol is not involved with
rant ultrasound. Ultrasound has a sensitivity o 84% the synthesis o bile (Choice D). Extracorporeal
and speci city approaching 99% or the diagnosis o shockwave lithotripsy can be utilized as nonsurgical
cholelithiasis (Choice C). Simultaneously, assessment therapy, but has a recurrence rate o approximately
or signs o acute cholecystitis, choledocholithiasis, 20%. Indications include patients with single stones,
and hepatic pathology can be sought. between the sizes o 5 mm and 2 cm. ESWL is not
commonly o ered as medical therapy due to the ef -
2. C. T is patient should have had a right upper cacy and commonplace practice o cholecystectomy
quadrant ultrasound per ormed in the rst ques- (Choice E).
tion above. I this imaging modality shows no
abnormality, but the patient has biliary symptoms, 4. B. T is patient should undergo open cholecystec-
she should have a repeat right upper quadrant tomy with a symptomatic, visualized preoperative
ultrasound per ormed in two weeks with ocus on intraluminal gallbladder lesion that is 1.5 cm in
areas that may have missed stones smaller than size. T e open cholecystectomy is pre erred, because
3 mm (Choice C). gallbladder per oration or bile spillage during lapa-
Esophagogastroduodenoscopy is an appropri- roscopic cholecystectomy can potentially seed the
ate diagnostic modality to rule out peptic ulcer dis- peritoneal cavity. Polypoid lesion size greater than
ease and can be utilized i the repeat ultrasound is 1 cm is an independent risk actor or gallbladder
still negative. Bile microscopy has been shown as an adenocarcinoma, along with stone size greater than
adjunct study to assess or microlithiasis and can be 2.5 cm (Choice A, B). Gallbladder cancer has a 3:1
help ul (Choice E), although is not routinely per- ratio o incidence in women and typically presents
ormed; bile microscopy also requires endoscopy a er age 60. Extended cholecystectomy is not indi-
or collection. T e sensitivity o bile microscopy cated i the lesion is con ned below the muscle layer
or microlithiasis is 65% to 90%. Cholecystectomy o the gallbladder. I the lesion is 2 or greater, resec-
is not recommended at this time as other causes tion usually includes segments IVb and V o the liver
o the patients symptoms have not been ruled out. (Choice E). Choice D is descriptive o cholesterol

http://surgerybook.net/
C H AP TER 3 6 BI LI ARY CO LI C 129

polyps, which are not a risk actor or gallbladder Moskovitz M, Min C, Gavaler JS. T e microscopic examina-
adenocarcinoma. tion o bile in patients with biliary pain and negative imag-
ing tests. Am J Gastroenterol. 1986;81:329.
Maton PN, Iser JH, Reuben A, Saxton HM, Murphy GM,
BIBLIOGRAPHY Dowling RH. Outcome o chenodeoxycholic acid (CDCA)
Barakos JA, Ralls PW, Lapin SA, et al. Cholelithiasis: Evalua- treatment in 125 patients with radiolucent gallstones.
tion with C . Radiology. 1987;162:415. Factors in uencing ef cacy, withdrawal, symptoms and
Ca asso DE, Smith RR. Symptomatic cholelithiasis and unc- side e ects and post-dissolution recurrence. Medicine
tional disorders o the biliary tract. Surg Clin North Am. (Baltimore). 1982;61(2):86.
April 2014;94(2):23356. Okamoto M, Okamoto H, Kitahara F, et al. Ultrasonographic
Coburn NG, Cleary SP, an JC, Law CH. Surgery or gallblad- evidence o association o polyps and stones with gallblad-
der cancer: A population-based analysis. J Am Coll Surg. der cancer. Am J Gastroenterol. 1999;94(2):446.
September 2008;207(3):37182. Available rom http://www. Reddy YP, Sheridan WG. Port-site metastasis ollowing lapa-
sciencedirect.com/science/article/pii/S1072751508002731. roscopic cholecystectomy: A review o the literature and a
Corazziari E, Sha er EA, Hogan WJ, et al. Functional disor- case report. Eur J Surg Oncol. February 2000;26(1):958.
ders o the biliary tract and pancreas. Gut. 1999;45(Suppl 2): Ricardo AE, Feig BW, Ellis LM, et al. Gallbladder can-
1148. cer and trocar site recurrences. Am J Surg. December
Darzi A, Geraghty JG, Williams NN, Sheehan SS, anner AN, 1997;174(6):61922; disc 6223.
Keane FB. T e pros and cons o laparoscopic cholecys- Rubin RA, Kowalski E, Khandelwal M, Malet PF. Ursodiol
tectomy and extracorporeal shock wave lithotripsy in the or hepatobiliary disorders. Ann Intern Med 1994;121:207.
management o gallstone disease. Ann R Coll Surg Engl. Sarkut P, Kilicturgay S, Ozer A, Ozturk E, Yilmazlar . Gall-
1994;76(1):42. bladder polyps: Factors a ecting surgical decision. World J
Diehl AK, Sugarek NJ, odd KH. Clinical evaluation or gall- Gastroenterol. July 2013;19(28):452630.
stone disease; use ulness o symptoms and signs in diagno- Shea JA, Berlin JA, Escarce JJ, Clarke JR, Kinosian BP, Cabana
sis. Am J Med. 1990;89:29. MD, et al. Revised estimates o diagnostic test sensitivity
Friedman GD. Natural history o asymptomatic and symp- and speci city in suspected biliary tract disease. Arch
tomatic gallstones. Am J Surg. 1993;165:399. Intern Med. 1994;154(22):2573.
Lee SP, Nicholls JF. Nature and composition o biliary sludge. omida S, Abei M, Yamaguchi , et al. Long-term ursode-
Gastroenterology. 1986;90:677. oxycholic acid therapy is associated with reduced risk o
Misra S, Chaturvedi A, Misra NC, Sharma ID. Carcinoma o biliary pain and acute cholecystitis in patients with gall-
the gallbladder. Lancet Oncol. 2003;4(3):167. bladder stones: a cohort analysis. Hepatology. 1999;30:6.

http://surgerybook.net/
37
Biliary ractCholangitis

Harry T. Aubin

A 70-year-old male presents to the emergency depart- 3. Regarding the proper diagnostic workup, which
ment with altered mental status. Family reports he was noninvasive test has highest sensitivity and specif city
complaining o right upper quadrant abdominal pain or detecting the most common cause?
prior to becoming altered mentally. On arrival, he is A. Abdominal ultrasound
ound to have a temperature o 102.5F, heart rate o B. Magnetic resonance cholangiopancreatography
112, and systolic blood pressure o 80 despite 2 liters o (MRCP)
crystalloid in usion. On exam, he is visibly jaundiced C. C Scan
with tenderness in the right upper quadrant. He has D. Hepatobiliary (HIDA) scan
3 out o 4 systemic in ammatory response syndrome
(SIRS) criteria. He is started on pipericillin/tazobac- 4. Regarding the management o septic cholangitis
tam and admitted to the intensive care unit or invasive caused by choledocholithiasis, which o the
monitoring and vasopressor support. ollowing is correct?
A. Urgent biliary tract decompression via endoscopic
1. Regarding this patients constellation o symptoms, retrograde cholangiopancreatography (ERCP) is
what is the most common cause? success ul 60% o the time.
A. Gallstones B. Percutaneous transhepatic cholangiography
B. Biliary stricture (P C) is easible or stone extraction and stent
C. Malignancy placement.
D. Genetic disorder C. ERCP with a sphincterotomy is equivalent to
cholecystectomy or reducing recurrence rates.
2. Regarding the pathophysiology o cholangitis, D. Should ERCP and P C ail or are not easible,
which o the ollowing is correct? operative choledochotomy and -tube place-
A. Increased biliary pressure leads to decrease in ment should be avoided because o the risk o
production o IgG in the biliary mucosa lead- surgery.
ing to increased translocation o duodenal E. Broad spectrum antibiotic therapy alone will
bacteria. generally provide adequate treatment.
B. Intra-portal toxins and bacteria can cross
through the biliary system due to stasis leading
ANSWERS
to in ection.
C. Stones do not colonize with bacteria. 1. A. Cholangitis is caused by obstruction o the bil-
D. Biliary stents are not elt to contribute to or cause iary tree eventually leading to bile stasis and bacterial
cholangitis as they help decompress the biliary in ection. T e most common cause being gallstones,
tree. which account or around hal o cases. Other causes

http://surgerybook.net/
C H AP TER 3 7 Bi l i ARy TRAC T C H o l An g i Ti s 1 31

include stenosis/biliary stricture, malignancy and bil- 80% o patients. Routine ERCP can be per ormed
iary stents. Stents can cause obstruction rom migra- in this setting assuming the patient remains stable.
tion, occlusion, or colonization by bacteria leading to T is patient displays Reynolds pentad o ever, right
bacterial overgrowth and translocation into the blood upper quadrant pain, jaundice, altered mental status,
stream. and hypotension, the rst three signs constituting
Charcots triad. T is lends suspicion to suppurative
2. B. Obstruction o the biliary tree via stricture/ cholangitis due to the patients state o septic shock.
stenosis, stones, malignancy, or stent occlusion leads Antibiotics, though required as initial therapy, are
to increased biliary tract pressure. T is pressure unlikely to complete resolve this patients septic
promotes stasis o bile and decreases production o physiology. Emergent/urgent biliary tree decompres-
IgA in the bile tract mucosa. T e lack o continuous sion is warranted and must be per ormed to prevent
bile ow, coupled with decrease mucosal protection excessive morbidity/mortality.
allows or bacterial translocation rom the duode- ERCP with sphincterotomy has shown upwards o
num through the biliary tract. T is static bile and a 95% success rate in stone extraction and decreasing
gallstones provide a healthy growth medium or bac- the rate o recurrence o cholangitis. It is, however,
teria. T e elevated intra-biliary pressure allows or not superior to cholecystectomy in decreasing rates
translocation o these pathogens into the systemic o recurrence and thus cholecystectomy is recom-
circulation causing septicemia. Less commonly, bac- mended a er the ERCP/sphinceterotomy once the
teria and toxins can enter through the portal circula- septic physiology has resolved. Arguments or early
tion into the bile due to increase biliary pressure. cholecystectomy have been made as waiting 6 to
T e most common bacteria are gram negative 8 weeks runs the risk o 20% recurrence rate o a
enteric pathogens: E coli, klebsiella, and enterobac- gallstone related event.
ter. T ey carry LPS that promotes cytokine release Should ERCP ail, P C is warranted as both these
and leads to septic shock. Enterococcus is seen in a procedures decrease the morbidity/mortality risk o
smaller set o cases. a common bile duct exploration. P C can be chal-
3. B. MRCP is the best non-invasive test to con rm the lenging i there is little intra-hepatic ductal dilatation
presence o choledocholithiasis due to its high sensi- and it also does not allow sphincterotomy.
tivity (some studies quoting 100%) and nearly 100% Common bile duct exploration is warranted
speci city. I the test is positive then this con rms should ERCP and P C ail at decompressing the bil-
diagnosis. It also is help ul in evaluating or stricture iary tree. In the setting o a patient who is in septic
and ampullary masses. Ultrasound is a good screen- shock, choledochotomy with stone extraction and
ing tool and can evaluate or common bile duct dila- -tube placement is recommended as this allows
tation to perhaps lend clinical suspicion to presence or decompression o the biliary tree and allows or
o biliary obstruction. It is best at identi ying the sepsis to resolve prior to per orming cholecystec-
presence o cholelithiasis/cholecystitis. However, it tomy to limit the morbidity and mortality associated
has lower accuracy in identi ying the presence o a with both procedures. T e exploration is per ormed
choledocholith, roughly 80%. through a choledochotomy on the common bile duct
A C scan is less use ul than ultrasound in detect- distal to the insertion o the cystic duct. Stay sutures
ing cholecystitis/common bile duct dilatation, but is are placed on either side o the choledochotomy
help ul at evaluating or ampullary masses as a cause and using balloon catheters, uoroscopy with bas-
o a dilated common bile duct. ket retractors, and ushing, stone extraction is per-
HIDA scans are not use ul in the setting o chol- ormed. In general, the choledochotomy should be
angitis as the biliary tract in ection reduces secretion roughly the size o the largest stone. It is best done
o the radio nucleotide labeled marker into the bil- in dilated ducts as the risk o stenosis is high in
iary tree. It may show, however, obstruction with lack the setting o common bile duct size <6 mm.
o ow into the duodenum. A large bore -tube is placed and the choledochot-
omy repaired over the -tube with 4-0 absorbable
4. B. In cases o cholangitis without septic shock, a sutures. T e tube is externalized and bile allowed to
trial o antibiotic therapy is recommended as this drain into an external bag. Due to lack o re-absorption
can resolve symptoms and ensure stability in up to o bile, a patient with this procedure is prone to being

http://surgerybook.net/
132 g En ERAl s U Rg ERy EXAM i n ATi o n An D Bo ARD REVi EW

de cient in at soluble vitamins (A, D, E, and K). It is Leung JW, Sung JY, Costerton JW. Bacteriological and electron
the vitamin K de ciency which is most worrisome as microscopy examination o brown pigment stones. J Clin
it can lead to a coagulopathy. Microbiol. 1989; 27(5):91521.
Schiphorst AH, Besselink MG, Boerma D, et al. iming o
cholecystectomy a er endoscopic sphincterotomy or
BIBLIOGRAPHY common bile duct stones. Surg Endosc. September 2008;
Hui CK, Lai KC, Yuen MF, Ng M, Lai CL, Lam SK. Acute 22(9):204650.
cholangitispredictive actors or emergency ERCP. Aliment Shimizu S, Yokohata K, Mizumoto K, Yamaguchi K,
Pharmacol T er. October 2001;15(10):16337. Chijiiwa K, anaka M. Laparoscopic choledochotomy
Kimura Y, akada , Kawarada Y, et al. De nitions, pathophys- or bile duct stones. J Hepatobiliary Pancreat Surg.
iology, and epidemiology o acute cholangitis and chole- 2002;9(2):2015.
cystitis: okyo Guidelines. J Hepatobiliary Pancreat Surg. Singh A, Mann HS, T ukral CL, Singh NR. Diagnostic Accu-
2007;14(1):1526. racy o MRCP as Compared to Ultrasound/C in Patients
Lai EC, Mok FP, an ES, et al. Endoscopic biliary drainage or with Obstructive Jaundice. J Clin Diagn Res. March 2014;
severe acute cholangitis. N Engl J Med. 1992;326(24):15826. 8(3):1037.
Lai EC, am PC, Paterson IA, et al. Emergency surgery or Sung JY, Costerton JW, Shaf er EA. De ense system in the
severe acute cholangitis. T e high-risk patients. Ann Surg. biliary tract against bacterial in ection. Dig Dis Sci. 1992;
1990;211(1):559. 37(5):68996.

http://surgerybook.net/
38
Cholecystitis in Pregnancy

Alan P. Gehrich

A 32-year-old gravida 2 para 1 emale at 28 weeks gesta- liver enzymes, low platelet) syndrome and gall
tion presents with acute onset o right upper quadrant bladder disease.
and right upper ank pain with associated nausea and D. In a preterm pregnancy, it is considered sa e to
vomiting over the preceding 24 hours. She has no signi - continue the pregnancy in a patient who has
icant medical or surgical history. She has had 1 uncom- been diagnosed with AFLP.
plicated vaginal delivery. At the time o her evaluation,
her temperature is 99.8F, heart rate is 110, and respira- 3. Which of the following statements regarding radio-
tory rate is 24. Her exam documents a positive Mur- graphic imaging of biliary disease in pregnancy is
phys sign and guarding in the right upper quadrant. correct?
Laboratory studies show the ollowing: WBC 20,000, A. Classic sonographic signs o biliary disease are
H/H- 9/29, Platelets 130,000, AL -60, and AS 90. Her altered in pregnancy.
bilirubin, lipase, and amylase levels are normal. Her uri- B. T e risk o radiation exposure to the etus
nalysis is within normal limits. with ERCP (endoscopic retrograde cholangio-
pancreatography) is high throughout pregnancy.
1. Which of the following diagnoses is the least C. T e neuronal development o the etus is most
common disease in pregnancy presenting with sensitive to radiation between 20 to 28 weeks
right upper quadrant pain? gestation.
A. Acute atty liver o pregnancy D. Exposure to less than 5 rad o ionizing radiation
B. Cholecystitis has not been associated with an increased risk o
C. Cholelithiasis etal anomalies or pregnancy loss.
D. HELLP syndrome E. MR imaging has a higher sensitivity and speci c-
E. Appendicitis ity in the diagnosis o cholecystitis than ultraso-
nography.
2. Which of the following statements is correct
concerning Acute Fatty Liver of Pregnancy (AFLP)? 4. In this patient, acute cholecystitis is diagnosed
A. AFLP presents most commonly in the second by ultrasound. Which of the following is correct
trimester o pregnancy. regarding treatment of this patient?
B. AFLP commonly presents with serum amino- A. T e risk o adverse e ects o laparoscopy is high
trans erase levels similar to those ound in gall- even with maximal intra-abdominal pressures
bladder disease. limited to 9 mm Hg.
C. AFLP can present with hypoglycemia and occa- B. I le untreated, the most common complication
sionally renal ailure which can help distinguish o acute cholecystitis in pregnancy is gangrenous
it rom HELLP (hemoconcentration, elevated cholecystitis.

http://surgerybook.net/
134 G EN ERAL S U RG ERY EXAM I N ATI O N AN D BO ARD REVI EW

C. Available studies have shown signi cant di er- disease, hepatitis, and appendicitis, due to a superiorly
ences regarding preterm delivery rates, birth displaced cecum, as well as pyelonephritis, nephro-
weights or neonatal outcomes when comparing lithiasis, right lower lobe pneumonia, peptic ulcer
laparoscopic versus open cholecystectomies. disease, and myocardial in arction. Obstetric speci c
D. Nonsteroidal anti-in ammatory drugs (NSAID) diagnoses must also be included in the di erential to
treatment or pain expected to last more than include preeclampsia, HELLP (Hemolysis, Elevated
48 to 72 hours is the pharmacologic option o Liver enzymes, Low Platelets) syndrome and acute
choice a er 30 weeks gestation to avoid etal atty liver o pregnancy (AFLP). HELLP syndrome
complications. is a severe orm o preeclampsia occurring in up to
E. Beta-lactam antibiotics such as ampicillin- 8 o 1000 pregnancies presenting most commonly
sulbactam or piperacillin-tazobactam are con- in the third trimester o pregnancy. T is syndrome
traindicated in pregnant patients. generally involves the characteristic hypertension
and proteinuria seen with preeclampsia with evi-
5. T is patient undergoes an uncomplicated laparo- dence o liver dys unction and a consumptive coag-
scopic cholecystectomy. On postoperative day 2, ulopathy which can rapidly progress to ulminant
she develops increasing pain in her right upper DIC. Patients with preeclampsia may present with
quadrant (RUQ) with fever and recurrent leukocy- right upper quadrant or epigastric pain due to liver
tosis as well as elevated total bilirubin, transaminase, involvement and in the most severe cases subcap-
lipase and amylase levels. RUQ ultrasound documents sular hemorrhage or hepatic rupture. AFLP is a rare
dilated biliary ducts. Which of the following would diagnosis, seen in 1:20,000 pregnancies.
be the most appropriate next step? T is patient presents with ndings consistent with
A. MRCP (magnetic resonance cholangio-pancre- an in ammatory intra-abdominal process. Chol-
aticogram) ecystitis, choledocholithiasis, and cholangitis lead
B. ERCP the di erential diagnosis. T e physical exam ndings
C. Continued observation with antibiotic therapy are highly suggestive o gallbladder disease. WBC
D. Repeat surgery with bile duct exploration counts and alkaline phosphatase levels are routinely
E. Delivery o the etus elevated during pregnancy and there ore may not be
as speci c or in ammation during the assessment o
the gravid patient.
ANSWERS
1. A. T e most common surgical disease in pregnancy 2. C. T e AFLP syndrome almost always presents in
is appendicitis with an incidence o 1 in 1000 to the third trimester with serum aminotrans erase
2000 pregnancies. Gall bladder disease is the second elevations up to 1000 IU/L, which is generally higher
most common surgical disease in pregnancy with than those ound in gallbladder disease. AFLP can
an incidence o 1 in 1200 to 1 in 10,000. T eoreti- also present with hypoglycemia and renal ailure,
cally, the incidence o gall bladder disease including which is not characteristic o either HELLP or gall-
cholelithiasis, cholecystitis, and cholangitis should be bladder disease. Findings o AFLP can still signi -
increased in pregnancy. T e elevated level o serum cantly overlap with those o HELLP, making it very
estrogen seen in pregnancy increases cholesterol dif cult to distinguish these two syndromes. T e
secretion, whereas the elevated level o progesterone treatment or both is the emergent delivery o the
reduces soluble bile acid secretion and slows emp- etus.
tying o gallbladder. Despite the predilection toward
biliary sludge and stone ormation, cholecystitis does 3. D. Due to the acuity o presentation and the myriad
not occur more requently during pregnancy. Appen- o diagnoses in the di erential, imaging is an essen-
dicitis occurs with equal requency in each trimes- tial component in the diagnostic evaluation. Risks o
ter and the incidence is not increased in the gravid radiologic studies to the etus must there ore be con-
patient. sidered. Sonography is the appropriate rst line diag-
T e di erential diagnosis or RUQ abdominal nostic modality in pregnancy or both biliary disease
pain is expanded in pregnancy. It includes gastro- and appendicitis as this modality has a high diagnos-
intestinal disorders such as pancreatitis, peptic ulcer tic accuracy (90% to 100% or both diagnoses) and

http://surgerybook.net/
C H AP TER 3 8 C H O LEC YS TI TI S IN P REG N AN C Y 135

has no known risk to the etus. Classic ultrasound adverse etal e ects. Use in the third trimester increases
(US) ndings to include wall edema, pericholecystic the risk o premature closure o the patent ductus
uid, calculi, and songraphic Murphys sign main- arteriosis and oligohydramnios. Intravenous antibi-
tain their sensitivity and speci city in pregnancy. I otics to include ampicillin-sulbactam, piperacillin-
ultrasound studies are non-diagnostic, MR imaging tazobactam, and ticarcillin-clavulanate are not con-
without contrast has become the con rmatory test traindicated in pregnant patients who need antibiotics
or appendicitis. For biliary disease, an MRCP can or acute cholecystitis or choledocolithiasis
be used in equivocal cases or in suspected cases o Early surgery has been advocated or all types o
choledocholithiasis or cholangitis. It is not as sensi- biliary disease in pregnancy. I not treated, cholecys-
tive as US or cholecystitis. Intraoperative cholan- titis can lead to li e threatening complications, the
giogram in combination with cholecystectomy is an most common o which is gangrenous cholecystitis
option or diagnostic evaluation a er etal organo- ollowed by abscess ormation, per oration, stula,
genesis is complete in the second trimester and does ileus, or emphysematous cholecystitis. For sympto-
not appear to increase the risk or preterm delivery matic cholelithiasis with no evidence o cholecysti-
or adverse etal outcomes. I MRCP documents stone tis, surgery can be delayed. However the literature
disease in the biliary tree, ERCP is considered a via- reports that surgical management o symptomatic
ble therapeutic option a er the rst trimester. T e cholelithiasis in pregnancy is sa e, decreases hospital
risks to the etus with cholangiogram and ERCP can days, reduces emergency room visits, and the rate o
be reduced with shielding. preterm deliveries. T e second trimester ( rom 13 to
C scan o the abdomen, which is the pre erred 27 weeks gestation) is considered the best time rame
imaging modality or appendicitis in the non- or cholecystectomy as the uterus is not obstructing
pregnant patient, con ers radiation levels o 5 to the view o the surgical eld, and the risk o miscar-
10 rads which approach the maximum permissible riage or preterm birth is lowest. In this patient, with
radiation dose or etal exposure during pregnancy. clear evidence o cholecystitis, surgical intervention
Fetal exposure to ionizing radiation increases risks is warranted to reduce risk o serious complications.
o microcephaly, micropthalmia, mental retarda- I complications such as cholangitis or gallstone pan-
tion, growth restriction, and cataracts. T e concern creatitis develop in a pregnant patient, maternal mor-
o ionizing radiation is greatest during organogen- tality may approach 15% and etal loss up to 60%.
esis which alls between 320 weeks o gestation. Surgical management o biliary disease has been
T e above patient is at 28 weeks and there ore the revolutionized with the advent o laparoscopy. Lapa-
risk o serious complications with ionizing radia- roscopic technique can be utilized sa ely in preg-
tion is limited. C generally remains behind US and nancy across all trimesters depending on the com ort
MR on the imaging algorithm or both appendicitis level o the surgeon. Although data is limited, lapa-
and cholecystitis even in the patient with a etus o roscopy does not con er an increased risk o adverse
advanced gestational age due to the disputed two old pregnancy outcomes to include preterm delivery as
increased risk o carcinogenesis (1:1000) in the etus. compared to laparotomy. Proper positioning in le
C imaging should, however, not be abandoned as a lateral tilt is important to reduce venal caval com-
diagnostic modality, as the risk o delay in diagno- pression and maintain adequate placental blood
sis ar outweighs the risk o radiation. T e consult- ow, and open entry technique is recommended to
ing radiologist can design C protocols to minimize prevent injury to the enlarged gravid uterus. Intra-
the associated risks and counseling can minimize the abdominal pressure with pneumo-peritoneum
associated anxiety o the patient. should be limited to 10 to 12 mm Hg to reduce the
theoretical concern o etal acidosis associated with
4. B. Initial non-surgical management can be considered the e ect o CO2. T ere is no indication or intraop-
in hemodynamically normal pregnant patients experi- erative etal monitoring.
encing cholelithiasis. T is management plan generally Early surgery or appendicitis is also recom-
involves bowel rest, intravenous hydration, and NSAID mended in pregnant patients, as the consequences or
therapy. A short course (< 48 to 72 h) o indomethacin both the mother and etus can be catastrophic. Abbasi
treatment can provide e ective analgesia but is gen- et al., in the largest case series to date involving 7000
erally avoided in late pregnancy due to the potential patients, documents a markedly increased risk o

http://surgerybook.net/
136 G EN ERAL S U RG ERY EXAM IN ATI O N AN D BO ARD REVIEW

severe complications with conservative management use during pregnancy and lactation. Obstet Gynecol. 2008;
to include miscarriage and maternal sepsis. As with 112(2 Pt 1):33340.
cholecystitis, non-operative management o appendi- Dietrich CS, Hill CC, Hueman M. Surgical diseases present-
ing in pregnancy. Surg Clin North Am. 2008;88(2):40319,
citis is contraindicated in pregnancy. viiviii.
Gilat , Koniko F. Pregnancy and the biliary tract. Can J Gas-
5. B. Following cholecystectomy, this patient presents troenterol. 2000; 14 Suppl D:55D9D.
with ndings consistent with choledocholithiasis, Gilo NB, Amini D, Landy HJ. Appendicitis and cholecystitis in
with associated gallstone pancreatitis and possible pregnancy. Clin Obstet Gynecol. 2009;52(4):58696.
cholangitis. T e best option in this case is ERCP Gjelsteen AC, Ching BH, Meyermann MW, et al. C , MRI,
with the option o sphincterotomy to decompress PE , PE /C , and ultrasound in the evaluation o obstetric
and gynecologic patients. Surg Clin North Am. 2008;88(2):
the biliary tract. T is approach appears sa e in preg- 36190, vii.
nant patients with early onset cholangitis with lower Graham G, Baxi L, T arakan . Laparoscopic cholecystectomy
morbidity than conservative management. In this during pregnancy: A case series and review o the litera-
patient, repeat surgery with intraoperative cholan- ture. Obstet Gynecol Surv. 1998;53(9):56674.
giography or bile duct exploration would be a back- Knight M, Nelson-Piercy C, Kurinczuk JJ, Spark P, Brockle-
hurst P, UK Obstetric Surveillance System. A prospective
up option, i the stones cannot be removed via ERCP.
national study o acute atty liver o pregnancy in the UK.
Percutaneous biliary tract decompression would be Gut. 2008;57(7):9516.
another option in a high risk patient. MRCP is an Khandelwal A, Fasih N, Kielar A. Imaging o acute abdomen
excellent and sa e diagnostic test in pregnancy and in pregnancy. Radiol Clin North Am. 2013;51(6):100522.
is a viable option in patients where the diagnosis is Kim YW, Zagorski SM, Chung MH. Laparoscopic common
uncertain. bile duct exploration in pregnancy with acute gallstone
pancreatitis. JSLS. 2006;10(1):7882.
In a case with a high suspicion o cholangitis, this Koren G, Florescu A, Costei AM, Boskovic R, Moretti ME.
step could delay therapy which could have severe Nonsteroidal antiin ammatory drugs during third trimes-
consequences or both etus and mother. More ter and the risk o premature closure o the ductus arterio-
aggressive therapy or cholangitis is there ore indi- sus: A meta-analysis. Ann Pharmacother. 2006;40(5): 8249.
cated in pregnancy. Although conservative treatment Lu EJ, Curet MJ, El-Sayed YY, Kirkwood KS. Medical versus
surgical management o biliary tract disease in pregnancy.
with continued intravenous antibiotics and observa-
Am J Surg. 2004;188(6):7559.
tion may be appropriate in the non-gravid patient, Othman MO, Stone E, Hashimi M, Parasher G. Conserva-
the risks o this non-surgical approach are higher in tive management o cholelithiasis and its complications
the gravid patient and predispose her to grave com- in pregnancy is associated with recurrent symptoms and
plications. Premature delivery o the etus is not indi- more emergency department visits. Gastrointest Endosc.
cated or the treatment o biliary disease. 2012;76(3):5649.
Parangi S, Levine D, Henry A, Isakovich N, Pories S. Surgi-
cal gastrointestinal disorders during pregnancy. Am J Surg.
BIBLIOGRAPHY 2007;193(2):22332.
Augustin G, Majerovic M. Non-obstetrical acute abdomen Reiss R, Nudelman I, Gutman C, Deutsch AA. Changing
during pregnancy. Eur J Obstet Gynecol Reprod Biol. 2007; trends in surgery or acute cholecystitis. World J Surg. 1990;
131(1):412. 14(5):56770, discussion 5701.
Abbasi N, Patenaude V, Abenhaim H. Management and out- Walcher , Haenle MM, Kron M, et al. Pregnancy is not a risk
comes o acute appendicitis in pregnancy-population-based actor or gallstone disease: Results o a randomly selected
study o over 7000 cases. BJOG. 2014 121(12):150914. population sample. World J Gastroenterol. 2005;11(43):
Banks PA, Freeman ML, et al. Practice Guidelines in Acute 68006.
Pancreatitis. Am J Gastroenterol. 2006;101(10):2379400. Wallace GW, Davis MA, Semelka RC, Fielding JR. Imaging
Chen MM, Coakley FV, Kaimal A, Laros RK, Jr. Guidelines or the pregnant patient with abdominal pain. Abdom Imag-
computed tomography and magnetic resonance imaging ing. 2012;37(5):84960.

http://surgerybook.net/
39
Liver Metastasis

Kevin M. Lin-Hurtubise and Robert L. She er

A 60-year-old male presents with 2 months history o D. Ce uximab plus multiagent chemo (FOLFOX
rectal bleeding associated with anorexia and a 10 lbs or FOLFIRI) is best used or patients with the
weight loss. He underwent a diagnostic colonoscopy K-RAS wild type colorectal cancer.
revealing an ulcerated and circum erential right cecal E. 5-FU and leucovorin are adequate.
mass; biopsy returns as invasive adenocarcinoma. Stag-
3. Contraindications to liver surgery or metastatic
ing C scan o chest/abdomen/pelvis shows bilobar and
tumor resection include:
peripheral liver masses ranging rom 2 to 3 cm in diam-
eter with a total o 3 masses, no regional or paraaortic A. Resection that would remove 5 hepatic segments
adenopathy, and no other distant sites o disease. Percu- B. Bilobar liver involvement
taneous biopsy o a right lobar liver mass reveals meta- C. Resection that would require removal o 60% o
static adenocarcinoma, consistent with the colon as the the liver
primary site o malignancy. T e patient has hyperten- D. 4 cm liver lesion
sion and hypercholesterolemia. He has no amily his- E. umor involving the common hepatic artery
tory o colorectal cancer. His unctional status is good. 4. Af er success ul resection o both the primary
colon cancer and its liver metastases, the guidelines
1. T e best initial management in this patient is: or surveillance include:
A. Neoadjuvant chemotherapy A. Occult ecal blood testing every 6 months
B. Concomitant right hemicolectomy and liver B. CEA levels annually or the next 5 years
resection C. C scans o thorax/abdomen/pelvis every
C. Right colectomy then chemotherapy 3 months or the next 5 years
D. Radio requency ablation o liver tumors D. Surveillance colonoscopy one year a er surgery,
E. Hepatic Intraarterial chemotherapy repeat in 3 years i no advanced adenomas ound,
then every 5 years
2. Regarding neoadjuvant chemotherapy that is
E. Annual PE scan
used or metastatic colorectal cancer, which o the
ollowing statements is true? 5. For initially unresectable colorectal liver metastases,
A. XELOX (capecitabine and oxaliplatin) is less what percentage o patients will be resectable
e ective than 5-FU and leucovorin. ollowing neoadjuvant chemotherapy?
B. FOLFOXIRI (leucovorin, 5-FU, oxaliplatin, iri- A. About 5%
notecan) is recommended over FOLFIRI (5-FU, B. About 15%
leucovorin, and irinotecan). C. About 30%
C. FOLFIRI increases overall survival compared to D. About 50%
FOLFOX (5-FU, leucovorin, and oxaliplatin). E. About 70%

http://surgerybook.net/
138 G EN ERAL S U RG ERY EXAM I N ATI O N AN D BO ARD REVI EW

ANSWERS therapy or Stage III colon cancer. Similarly or Stage


II and III rectal cancer, 5-FU and leucovorin along
1. B. Concomitant colon and liver resections would with radiation therapy is recommended. Currently
be pre erable or this patient. I hepatic metasta- or Stage III colon cancer, FOLFOX is considered
ses are resectable on presentation, surgical resec- the superior regimen over 5FU-Leucovorin, with
tion should be considered rather than preoperative improved survival but has additional risk o periph-
chemotherapy or medically t patients with 4 or eral neuropathy. For Stage IV disease, there seems to
ewer metastases. T e de nition o resectable dis- be increased response and better survival with the
ease encompasses both the character o the meta- addition o biologic agents like ce uximab, pani-
static lesions and the patient's overall health status. tumumab, and bevacizumab, or alkylating agents
T is patient has minimal co-morbidities and a good like oxaliplatin, and topoisomerase inhibitors like
unctional status rendering him a potentially good irinotecan to the 5FU-leucovorin regimen. T e
candidate or major abdominal surgery. T e liver table o chemotherapeutic regimens listed below
lesions in this scenario are considered resectable + /- bevacizumab or ce uximab are acceptable
because o their overall number (4 or less), moder- or metastatic colorectal cancer treatment in the
ate size (only 2 to 3 cm), and its ideal peripheral liver neoadjuvant setting. 5-FU and leucovorin alone is
location. Central or large metastatic liver lesions not adequate treatment or patients with metastatic
(> 6 cm) that encroach upon major liver vasculature colorectal disease.
such as the hilum o the main portal vein would pre- FOLFOXIRI when compared to FOLFIRI was
clude liver resection. Un ortunately no more than equivalent in overall survival, response rates, and
20% o patients with isolated hepatic metastases are median time to disease progression. FOLFOXIRI,
amenable to potential curative hepatic resection; but however, was associated with more signi cant
because o clear survival impact, surgical resection side e ects. Furthermore, the addition o biologic
is the treatment o choice or resectable colorectal agents, ce uximab or panitumumab is recom-
metastases to the liver. mended or patients with colorectal tumors exhib-
T ere is an exception i to this rule and that is, iting the K-RAS wild-type, which may increase
i the metastatic tumor response to chemotherapy the number o patients eligible or liver resection
is expected to create a signi cantly less complex post-chemotherapy (pt reevaluated or conversion
liver resection, then neoadjuvant therapy is rec- to resectable disease every 2 months) to improve
ommended. T e next recommended management their outcomes.
option is an initial right hemicolectomy i the patient For colorectal metastatic disease, FOLFOX or
is already exhibiting GI obstructive symptoms, ol- FOLFIRI are considered to be equally e ective with
lowed by our to six cycles o chemotherapy (must di erent toxicity pro le. Capecitabine is an orally
be cognizant o potential hepatotoxicity i too much administered tumor-activated 5-FU prodrug.
chemotherapy is given), and then consideration o
liver resection i still indicated. Radio requency abla-
tion (RFA) is not considered curative or larger liver 3. E. Inadequate post-resection liver reserve is consid-
metastases. Intra-arterial chemotherapy should be ered a contraindication or liver resection o colo-
limited to centers with expertise in this modality. rectal metastases. T at being said, in healthy livers,
up to 70% o the liver or up to 6 hepatic segments
2. D. T e use o 5-FU ( uorouracil) and leucovorin can be resected sa ely. T e location o the tumor is
has historically been the basis o adjuvant chemo- also important as metastatic tumors invading the

Chemotherapeutic
Regimen 5-FU Leucovorin Oxaliplatin Irinotecan Capecitabine
FOLFOX + + +
FOLFOXIRI + + + +
FOLFIRI + + +
XELOX + +

http://surgerybook.net/
C H AP TER 3 9 LIVER M ETAS TAS IS 139

common hepatic artery, common hepatic duct, main only to (liver) metastatic pts with wild-type KRAS,
portal vein, or > 3 hepatic veins would be consid- resectability was increased rom 4% to 10%.
ered contraindications. Extrahepatic involvement o
the celiac, portal, or paraaortic nodes would also be BIBLIOGRAPHY
considered a contraindication. Lesions larger than Adam R, Aloia , Levi F, et al. Hepatic resection a er rescue
6 cm would also be considered a relative contrain- ce uximab treatment or colorectal liver metastases, pre-
dication to resection. Bilobar disease or greater that viously re ractory to conventional systemic therapy. J Clin
Oncol. 2007;25(29):4593-602.
4 metastatic lesions that do not shrink with neoad- Andre , Boni C, Navarro M, et al. Improved overall survival
juvant chemotherapy was deemed inappropriate or with oxaliplatin, uorouracil, and leucovorin as adjuvant
liver resection based on OncoSurge statistical mod- treatment in stage II or III colon cancer in the MOSAIC
eling. trial. J Clin Oncol. 2009;27(19):3109-116.
Blazer D, Kishi Y, Maru DM, et al. Pathologic response to pre-
4. D. Surveillance is an important aspect o cancer care operative chemotherapy: a new outcome end point a er
in order to determine adequate initial management resection o hepatic colorectal metastases. J Clin Oncol.
2008;26(33);5344-51.
and to detect early recurrences. A patient would not Bokemeyer C, Bondarenko I, Hartmann J , et al. Ef -
be considered cured until they have survived at least cacy according to biomarker status o ce uximab plus
5 years a er initial therapy. T e clinical guidelines FOLFOX-4 as rst-line treatment or metastatic colorectal
o surveillance or colon cancer patients have been cancer: the OPUS study. Ann Oncol. 2011; 22(7): 1535-46.
put orth by the National Comprehensive Cancer Bokemeyer C, Bondarenko I, Makhson A, et al. Fluorouracil,
leucovorin, and oxaliplatin with and without ce uximab
Network, version 1.2016. T eir recommendations
in the rst-line treatment o metastatic colorectal cancer.
include clinical assessments every 3 to 6 months or J Clin Oncol. 2009;27(5):663-71.
the rst 2 years, then every 6 months or the next Carpizo D, Are C, Jarnagin W, et al. Liver resection or meta-
3 years. T is includes a serum CEA level at each static colorecatl cancer in patients with concurrent extra-
clinic visit. A C scan o the chest/abdomen/pelvis hepatic disease: Results in 127 patients treated at a single
is recommended every 6 to 12 months or 5 years center. Ann Surg Oncol. 2009;16(8):2138-46.
Colucci G, Gebbia V, Paoletti G, et al. Phase III randomized
or patients at high risk o recurrence. Regarding trial o FOLFIRI versus FOLFOX-4 in the treatment o
colonoscopy, one is recommended 1 year a er sur- advanced colorectal cancer: a multicenter study o the
gery. I no lesions or a simple adenoma is ound (not Gruppo Oncologico DellLtalia Meridionale. J Clin Oncol.
advanced such as villous polyp, polyp > 1 cm, or high 2005;23(22):4866-75.
grade dysplasia), a colonoscopy should be repeated Martin R, Augenstein V, Reuter NP, et al. Simultaneous versus
staged resection or synchronous colorectal cancer liver
in 3 years, and then repeated every 5 years or the
metastases. J Am Coll Surg. 2009;208(5):842-50.
remainder o their lives. I an advanced or large Network NC. (n.d.). NCCN clinical practice guidelines in
adenoma is ound (and subsequently removed com- oncology, version 1.2016. Retrieved Nov 6, 2015, rom
pletely via colonoscopy), then repeat colonoscopy is http://www.nccn.org/pro essionals/physician_gls/ _guide-
recommended in 1 year. A PE scan is reserved or lines.asp
instances were the CEA is rising or lesions are seen Poston G, Adam R, Alberts S, et al. OncoSurge: a strategy or
improving resectability with curative intent in metastatic
on ollow up C scanning. Fecal occult blood testing colorectal cancer. J Clin Oncol. 2005;23(28);7125-34.
is not required or surveillance. Reddy S, Pawlik M, Zorzi D, et al. Simultaneous resections o
colorectal and synchronous liver metastases: a multi-insti-
5. A. About 5% o patients will bene t rom neoadju- tutional analysis. Ann Surg Oncol. 2007;14(12);3481-91.
vant chemotherapy in terms o ability to resect liver Souglakos J, Androulakis N, Syrigos K, et al. FOLFOXIRI v.
metastases. wo randomized studies, the CRYS AL FOLFIRI as rst-time treatment in metastatic colorectal
and OPUS trials, showed modest improvement o cancer: a multicenter, randomized phase III trial rom the
Hellenic Oncology Research Group (HORG). Br J Cancer.
liver resection rates rom 3.7% to 7% and 2.4% to 2006;94(6):798-805.
4.7%, respectively with the use o the biologic agent Van Custem E, Kohne CH, Hitre E, et al. Ce uximab and che-
ce uximab to FOLFOX or FOLFIRI versus FOLFOX motherapy as initial treatment or metastatic colorectal
or FOLFIRI alone. When the analysis was limited cancer. N Eng J Med. 2009;360(14):1408-17.

http://surgerybook.net/
40
Liver Cirrhosis

Richard Smith

A 55-year-old man with a known history o cirrhosis sec- B. Large volume paracentesis (LVP) decreases the
ondary to alcohol (E OH) abuse presents with a several risk o incarceration or umbilical hernias in the
month history o an increasing umbilical bulge. On exam setting o re ractory ascites.
he had a reducible umbilical hernia with a 3 cm ascial C. Non-operative management o patients with
de ect and shi ing dullness and a uid wave complete with umbilical hernia and ascites is the pre erred
moderate ascites. He denies any symptoms other than mild management because o the morbidity and mor-
discom ort at the hernia. He denies any previous episodes tality associated with surgery.
o GI bleed. History, physical exam, and serum laboratory D. Use o synthetic mesh is contraindicated in
evaluation o the above patient shows a Child- urcotte- repair o umbilical hernias in patients with cir-
Pugh classi cation o 9 points or class B and a MELD score rhosis and ascites.
o 10. His platelets are 160,000, HGB is 12 g/dL, sodium E. Complication rates are signi cantly higher in
level is 135 MeQ/L, and albumin is 3.0 ng/dL. elective repair o umbilical hernia in cirrhotics
when compared to non-cirrhotics.
1. Regarding evaluation and management o this
patients ascites, which o the ollowing is true? 3. Regarding morbidity and mortality in non-hepatic
A. Variceal bleeding is the most common complica- surgery in patients with liver cirrhosis, which
tion o cirrhosis. statement is true?
B. Paracentesis is the rst step in the evaluation o A. Open surgery is pre erred to laparoscopic sur-
this patient. gery because o the concern or increased
C. Cessation o E OH once ascites develops will bleeding ollowing laparoscopic procedures in
have little impact on controlling ascites. cirrhotics.
D. Ascites is re ractory in less than 5% o patients B. Elective pancreatic surgery in C P class A and B
with ascites secondary to cirrhosis. has acceptable morbidity and mortality.
E. Use o non-steroidal anti-in ammatory drugs C. Elective inguinal hernia repair is indicated or
(NSAID), angiotensin converting enzyme (ACE) symptomatic inguinal hernias even in advanced
inhibitors, and angiotensin receptor blocking liver disease with decompensated cirrhosis.
(ARB) drugs can prevent the progression to D. Morbidity and mortality are higher or all sur-
renal ailure. gical procedures in patients with liver cirrhosis
when compared to patients without cirrhosis.
2. Regarding umbilical hernia repair in patients with
cirrhosis, which o the ollowing is true? 4. T e above patient presents to the emergency
A. Incidence o umbilical hernia is 20% in patients room several days a er your evaluation with
with cirrhosis and ascites. ever, abdominal pain, and ileus. His vitals are a

http://surgerybook.net/
C H AP TER 4 0 Li v ER C i RRH o s i s 14 1

temperature o 101F, heart rate o 90, and a systolic Paracentesis with appropriate ascitic uid analysis is
blood pressure o 100. On exam he is di usely the rst step in the evaluation o clinically apparent
tender, without exam evidence o peritonitis, ascites. T e serum-ascites albumin gradient (SAAG)
and has increased ascites. Plain f lms o the chest with a value 1.1 g/dL is indicative o portal hyper-
and abdomen show an ileus pattern, without tension. Medical management with salt restriction
obstruction and no ree intraperitoneal air. and diuretics is highly e ective in most patients with
Your working diagnosis is spontaneous bacterial ascites. Re ractory ascites occurs in less than 20% o
peritonitis (SBP). Which o the ollowing is true? patients. T e use o ACE inhibitors and ARB drugs
A. Administration o broad spectrum antibiotics as can induce renal ailure and hypotension increasing
the rst step is necessary to avoid circulatory col- mortality in patients with ascites. NSAIDs impair
lapse and hepatorenal syndrome. glomerular ltration rate due to a reduced renal per-
B. SBP is associated with the nding o multiple usion secondary to inhibition o renal prostaglandin
organisms on paracentesis culture. synthesis leading to renal ailure.
C. Ascitic uid analysis would expect to show neu-
trophils > 250/mm 3 with lactate dehydrogenase 2. A. Umbilical hernia is present in 20% o patients
greater than the upper limit o normal or serum, with cirrhosis and this is 10 times higher than the
glucose less than 50 mg/dL and ascitic protein incidence in the overall population. T e increased
> 1 g/dL. incidence is related to ascites, abdominal wall atten-
D. Administration o 1.5 g/kg o albumin on admis- uation, and malnutrition. A sudden decrease in the
sion and 1 g/kg on day 3 has been shown to amount o ascites, as with transjugular intrahe-
decrease mortality. patic portosystemic shunt ( IPS) or LVP, can lead
E. A positive peritoneal culture is necessary or to strangulation o an incarcerated umbilical her-
diagnosis. nia. Non-operative management is associated with
higher mortality when compared to elective repair.
5. T e above patient undergoes elective umbilical T e higher mortality is related to the development
hernia repair. On the day 3 a er the operation, he o bowel incarceration or spontaneous rupture rom
develops hematemesis and hypotension. Based on necrosis o overlying skin and subsequent peritonitis
the most likely etiology o his upper gastrointestinal developing during watch ul waiting. T ese com-
hemorrhage, which o the ollowing is true? plications orce prompt elective repair in patients
A. Propranalol plays an important role in the acute who are at very high risk or death with emergency
management o this problem by decreasing surgery. A randomized study comparing umbilical
splanchnic blood ow. hernia repair with or without prosthetic material in
B. rans usion to a HGB o 10 is the goal o resusci- cirrhotic patients with symptomatic, or complicated
tation. hernia ound a decreased recurrence rate avoring
C. Antibiotic therapy is an important component. prosthetic repair. T e same study showed a non-sig-
D. rans-jugular intra-hepatic shunt ( IPS) proce- ni cant increase in in ections in the prosthetic group
dures have no role in the acute setting. but no mesh had to be removed in any o the cases.
E. Pharmacologic therapy should not be started Morbidity in elective repair o umbilical hernia is
until varices are con rmed by esophagastroduo- similar between cirrhotics and non-cirrhotics.
denoscopy (EGD).
3. C. Patients with advanced decompensated cir-
rhosis requently have severe symptoms because o
ANSWERS
ascites entering the hernia sac both in the standing
1. B. T e cessation o alcohol usually leads to improve- position and when recumbent making it dif cult to
ment in liver unction with alcohol associated cirrho- relieve symptoms. Patients with decompensated cir-
sis. One study ound improvement o the ChildPugh rhosis have been shown to bene t the most in terms
score was observed within 3 months in 66% o the o improved quality o li e and the procedures can
abstinent patients. Ascites is the most common com- be done with minimal morbidity. Laparoscopic chol-
plication o cirrhosis with 50% o compensated cir- ecystectomy has been shown to be sa e in patients
rhotics developing ascites over the course o 10 years. with Childs- urcotte class A and B cirrhosis and is

http://surgerybook.net/
14 2 G EN ERAL s U RG ERY EXAM i N ATi o N AN D Bo ARD REv i EW

associated with less blood loss and shorter hospital is to a hemoglobin o approximately 78 g/dL. T is
stay when compared to open cholecystectomy. Simi- is based on studies showing aggressive restoration
lar bene ts are seen avoring laparoscopic appendec- o blood volume leads to increases in portal pres-
tomy over open appendectomy. T e data is limited sure and subsequent rebleeding and mortality. IPS
or patients with Childs C classi cation. Pancreatic procedures are very e ective in controlling bleeding
surgery is associated with an increased risk o post- in the 10% to 20% o patients who ail pharmaco-
operative complications in patients with liver cirrho- logic and endoscopic therapy. Early placement o
sis. T e mortality in Childs A patients is similar to IPS within 24 to 48 hours a er admission has been
non-cirrhotics but the mortality or Childs B patients shown to improve survival in patients at the high-
is prohibitive. Although morbidity is higher or cir- est risk o re-bleeding (Childs C and patients with
rhotics when compared to non-cirrhotics or most a hepatic venous pressure gradient (HVPG) over
procedures, some procedures such as inguinal hernia 20 mm Hg). Vasoconstrictor therapy (vasopressin
repair have similar morbidity even or advanced liver nitroglycerin, terlipressin, somatostatin, or octreo-
disease. Mortality in Childs A patients is o en airly tide) is considered rst-line therapy and should be
similar to non-cirrhotics or a variety o procedures started at the time o admission in any patient with
ranging rom pancreatic surgery to cardiac surgery. suspected variceal hemorrhage even prior to EGD.

4. D. T e use o IV albumin in cirrhotics with SBP in BIBLIOGRAPHY


addition to antibiotics has been shown to decrease
Akriviadis EA, Runyon BA. Utility o an algorithm in di er-
in-hospital mortality. T e e ect is more pronounced entiating spontaneous rom secondary bacterial peritoni-
or patients with a creatinine greater than 1 mg/dL or tis. Gastroenterology. 1990;98(1):12733.
a bilirubin greater than 4 mg/dL. Ascitic uid testing Ammar SA. Management o complicated umbilical hernias
should be done prior to the initiation o antibiotics in cirrhotic patients using permanent mesh: Randomized
because 86% o cultures will be negative even a er clinical trial. Hernia. 2010;14(1):3538.
Bernard B, Grange JD, Khac EN, Amiot X, Opolon P, Poynard
a single dose o antibiotics. T e nding o multiple
. Antibiotic prophylaxis or the prevention o bacterial
organisms rom a diagnostic paracentesis should in ections in cirrhotic patients with gastrointestinal bleed-
prompt consideration o a secondary peritonitis ing: A meta-analysis. Hepatology. 1999;29(6):165561.
not SBP. T e ndings o neutrophils > 250/mm 3 Belghiti J, Durand F. Abdominal wall hernias in the setting o
with lactate dehydrogenase greater than the upper cirrhosis. Semin Liver Dis. 1997;17(3):21926.
limit o normal or serum, glucose less than 50 mg/ de Goede B, Klitsie PJ, Lange JF, Metselaar HJ, Kazemier G.
Morbidity and mortality related to non-hepatic surgery in
dL and ascitic protein > 1 g/dL are associated with patients with liver cirrhosis: A systematic review. Best Pract
secondary peritonitis. A cirrhotic patient with signs Res Clin Gastroenterol. 2012;26(1):4759.
and symptoms o SBP should be treated empirically Dokmak S, Aussilhou B, Belghiti J. Umbilical hernias and cir-
regardless o culture results. T e cultures are impor- rhose. J Visc Surg. 2012;149(5 Suppl):e329.
tant or narrowing o antibiotic coverage given the Eker HH, van Ramshorst GH, de Goede B, et al. A prospective
study on elective umbilical hernia repair in patients with
increasing emergence o resistant organisms.
liver cirrhosis and ascites. Surgery. 2011;150(3):5426.
El-Awadi S, El-Nakeeb A, Yousse , et al. Laparoscopic versus
5. C. Variceal bleeding accounts or 70% o all upper open cholecystectomy in cirrhotic patients: A prospective
gastrointestinal bleeding in patients with cirrhosis. randomized study. Int J Surg. 2009;7(1):669.
Antibiotic prophylaxis in cirrhotic patients with gas- European Association or the Study o the Liver. EASL clinical
trointestinal bleeding has been shown to decrease the practice guidelines on the management o ascites, sponta-
neous bacterial peritonitis, and hepatorenal syndrome in
risk o bacterial in ections, re-bleeding and overall cirrhosis. J Hepatol. 2010;53(3):397417.
mortality. Nonselective beta-blockers (propranolol) Gines P, Quintero E, Arroyo V, et al. Compensated cirrhosis:
do reduce portal blood ow by decreasing cardiac Natural history and prognostic actors. Hepatology. 1987;
output (-1 e ect) and, more importantly, by produc- 7(1):1228.
ing splanchnic vasoconstriction (-2 e ect) and play Garcia- sao G, Sanyal AJ, Grace ND, Carey W, Practice
Guidelines Committee o the American Association or the
a role in prevention o variceal bleeding. However,
Study o Liver Diseases, Practice Parameters Committee
they should not be used in the acute setting as they o the American College o Gastroenterology. Prevention
will decrease blood pressure and will blunt a physi- and management o gastroesophageal varices and variceal
ologic response to bleeding. T e goal or trans usion hemorrhage in cirrhosis. Hepatology. 2007;46(3):92238.

http://surgerybook.net/
C H AP TER 4 0 Li v ER C i RRH o s i s 14 3

Garcia-Pagan JC, Caca K, Bureau C, et al. Early use o IPS in Runyon BA, Montano AA, Akriviadis EA, Antillon MR, Irving
patients with cirrhosis and variceal bleeding. N Engl J Med. MA, McHutchison JG. T e serum-ascites albumin gradient
2010;362(25):23709. is superior to the exudate-transudate concept in the di -
Gray SH, Vick CC, Graham LA, Finan KR, Neumayer LA, erential diagnosis o ascites. Ann Intern Med. 1992;117(3):
Hawn M . Umbilical herniorrhapy in cirrhosis: Improved 21520.
outcomes with elective repair. J Gastrointest Surg. 2008; Stanley MM, Ochi S, Lee KK, et al. Peritoneovenous shunting
12(4):67581. as compared with medical treatment in patients with alco-
Liou IW. Management o end-stage liver disease. Med Clin holic cirrhosis and massive ascites. Veterans administra-
North Am. 2014;98(1):11952. tion cooperative study on treatment o alcoholic cirrhosis
Marsman HA, Heisterkamp J, Halm JA, ilanus HW, Metse- with ascites. N Engl J Med. 1989;321(24):16328.
laar HJ, Kazemier G. Management in patients with liver Sort P, Navasa M, Arroyo V, et al. E ect o intravenous albu-
cirrhosis and an umbilical hernia. Surgery. 2007;142(3): min on renal impairment and mortality in patients with
3725. cirrhosis and spontaneous bacterial peritonitis. N Engl J
Patti R, Almasio PL, Buscemi S, Fama F, Craxi A, Di Vita Med. 1999;341(6):4039.
G. Inguinal hernioplasty improves the quality o li e in elem DA, Schiano , Divino CM. Complicated hernia pre-
patients with cirrhosis. Am J Surg. 2008;196(3):3738. sentation in patients with advanced cirrhosis and re rac-
Perez-Ayuso RM, Arroyo V, Planas R, et al. Randomized tory ascites: Management and outcome. Surgery. 2010;
comparative study o ef cacy o urosemide versus spi- 148(3):53843.
ronolactone in nonazotemic cirrhosis with ascites. Rela- rotter JF, Suhocki PV. Incarceration o umbilical hernia ol-
tionship between the diuretic response and the activity lowing transjugular intrahepatic portosystemic shunt or
o the renin-aldosterone system. Gastroenterology. 1983; the treatment o ascites. Liver Transpl Surg. 1999;5(3):
84(5 Pt 1):9618. 20910.
Puggioni A, Wong LL. A metaanalysis o laparoscopic chole- Warnick P, Mai I, Klein F, et al. Sa ety o pancreatic surgery in
cystectomy in patients with cirrhosis. J Am Coll Surg. 2003; patients with simultaneous liver cirrhosis: A single center
197(6):9216. experience. Pancreatology. 2011;11(1):2429.
Runyon BA, AASLD. Introduction to the revised american Veldt BJ, Laine F, Guillygomarch A, et al. Indication o liver
association or the study o liver diseases practice guideline transplantation in severe alcoholic liver cirrhosis: Quan-
management o adult patients with ascites due to cirrhosis titative evaluation and optimal timing. J Hepatol. 2002;
2012. Hepatology. 2013;57(4):16513. 36(1):938.

http://surgerybook.net/
41
Pancreatitis and Pancreatic Cysts

Kiran Lagisetty

A 56-year-old male presents to the emergency depart- 4. Which o the ollowing conditions associated with
ment complaining o acute onset o intense boring epi- acute pancreatitis increases the mortality rate
gastric pain radiating to his back. He is unable to nd a rom 1%to 1020%?
com ortable position in which to lie in bed. His labora- A. Necrosis o greater than 1/3 o the pancreas
tory analysis is signi cant or a serum lipase o 1300, B. Distant organ ailure
serum glucose o 260 mg/dL, LDH o 375 IU/L, AS o C. Development o local complications (hemor-
350 IU/L, and WBC o 18k. rhage, abscess, pseudocyst)
D. All o the above
1. In order to diagnose acute pancreatitis 2 out o 3
which o the ollowing criteria are required? 5. A 7 cm acute peripancreatic uid collection was
A. Epigastric pain, radiologic evidence o pancreati- identi ed on CT scan shortly af er admission. It
tis, serum lipase at least 2 times normal. was managed expectantly and a repeat CT scan
B. Epigastric pain, radiologic evidence o pancreati- was obtained 6 weeks af er admission. Which
tis, serum amylase at least 3 times normal. o the ollowing would be criteria or surgical
C. Cholelithiasis, radiologic evidence o pancreati- management o a pseudocyst?
tis, serum lipase at least 3 times normal. A. T ere is no evidence o malignancy on C and
D. Epigastric pain, cholelithiasis, serum lipase at only a simple 5 cm pseudocyst persists.
least 2 times normal. B. Decrease in size rom 7 cm to only 4 cm or less
than hal its original size.
2. Which o the ollowing enzymes has been C. Decrease in size rom 7 cm to 4 cm with persis-
implicated in the etiology o pancreatitis? tence o abdominal pain.
A. Pepsin D. Cholelithiasis
B. rypsin E. No communication between the pseudocyst and
C. Gastrin a pancreatic duct such that the pseudocyst can-
D. Lipase not drain.

3. Which o the ollowing conditions is most likely


the etiology o our patients condition?
ANSWERS
A. Serum triglyceride level o > 1000 mg/dL
B. Cholelithiasis 1. B. At least two o the ollowing criteria are required
C. HIV in ection to diagnose acute pancreatitis: Characteristic abdom-
D. Pancreatic head mass inal pain, radiologic evidence o pancreatitis, and/or
E. Seat belt sign ollowing MVA serum amylase or lipase level at least 3 times normal.

http://surgerybook.net/
C H AP TER 4 1 PAn C REATi Ti s An d PAn C REATi C C y s Ts 14 5

2. B. rypsin pseudocyst also represents severe acute pancreatitis


and thus, also increases the mortality rate.
3. B. Each o the above conditions is known to cause
pancreatitis. T e most common etiologies are gall- 5. C. Approximately 50% o pancreatic pseudocysts
stone and alcohol related. Hypertriglyceridemia will resolve spontaneously af er about 4 to 6 weeks
> 1000 mg/dL is the third most common. Other and should there ore be managed expectantly.
known causes o acute pancreatitis include HIV Repeat C scan af er 6 weeks should be obtained to
in ection, certain medications, carcinoma, parasites, ensure the pseudocyst is resolving. Indications or
and trauma. intervention are clinical symptoms, cyst enlargement
or persistence over 6 cm, ductal communication, and
4. D. Mild acute pancreatitis is associated with a suspected malignancy. An interval o 6 weeks is gen-
mortality rate o about 1%. Acute pancreatitis in erally recommended to give time or the pseudocyst
conjunction with any o the listed complications is wall to mature into a thick, brous rind. T e pres-
considered severe acute pancreatitis and increases ence o cholelithiasis is not an indication or opera-
the mortality rate to 10% to 20%; necrosis o greater tive intervention o a pancreatic pseudocyst.
than one third o the pancreas, organ ailure to
include a systolic blood pressure 90 mm Hg, serum
creatinine > 2.9 mg/dL, gastrointestinal blood loss BIBLIOGRAPHY
> 500 mL within a 24 hr period o time, or PaO2< or Beger H, Matsuno S, and Cameron J. Diseases o the Pancreas:
= 60 mm Hg. T e development o an abscess or Current Surgical T erapy, Springer, 2008.

http://surgerybook.net/
42
Head o Pancreas Mass

Richard Smith

A 60-year-old male with no signi cant medical history E. Diagnostic laparoscopy to evaluate or occult
except a long history o smoking presents with painless metastases.
jaundice o 1 week duration. He denies any symptoms
o obstruction, weight loss, or atigue. Initial labs show 3. T e above patient is instead noted to have a 3 cm
mild elevation in transaminases to less than 1.5 times mass in the head o the pancreas that abuts greater
normal values and a bilirubin o 8 mg/dL. A right upper than 180 degrees o the superior mesenteric vein
quadrant (RUQ) ultrasound was obtained that shows (SMV) with occlusion o a short segment. T ere is no
dilated intra and extra-hepatic bile ducts and a dilated evidence o metastatic disease. Endoscopic ultrasound
gallbladder without stones. T e pancreas is not well vis- (EUS)-guided f ne needle aspiration (FNA) biopsy
ualized secondary to overlying bowel gas. was obtained showing adenocarcinoma. What is the
most appropriate management or this patient?
1. What would be the next best step in the evaluation A. Initiation o neo-adjuvant chemoradiation ol-
o this patient? lowed be reassessment or resection ollowing
A. Magnetic Resonance Cholangiopancreatigram neo-adjuvant therapy.
(MRCP). B. Pancreaticoduodenectomy with resection and
B. T in slice C scan with arterial and venous reconstruction o the involved SMV ollowed by
phases. adjuvant chemoradiation.
C. Whole body Positron Emission omography C. Initiation o palliative chemotherapy or unre-
(PE ) scan. sectable pancreatic cancer.
D. Endoscopic Retrograde Cholangiopancreatigra- D. Per ormance o prophylactic surgical biliary and
phy (ERCP). gastric bypasses prior to initiating palliative che-
E. CA19-9 laboratory specimen. motherapy.

2. T e above patient is ound to have a 2 cm mass 4. Which o the ollowing is not true o patients
in the head o the pancreas with no evidence undergoing neo-adjuvant therapy or adenocar-
o vascular invasion, adenopathy or metastatic cinoma o the pancreas when compared to patients
disease on C scan. His CA19-9 level is mildly undergoing up ront resection?
elevated to 1.5 times the normal level. What is the A. Signi cantly ewer patients undergoing neo-
next most appropriate step? adjuvant ultimately undergo surgery.
A. PE scan to evaluate or occult disease. B. Signi cantly lower rate o lymph node positivity
B. ERCP and stenting o the bile duct. in patients undergoing resection ollowing neo-
C. EUS and FNA biopsy o the mass. adjuvant therapy.
D. Pancreaticoduodenectomy within the next C. Signi cantly increased overall survival ollowing
2 weeks. resection a er neo-adjuvant therapy.

http://surgerybook.net/
C H AP TER 4 2 H EAd o f PAn C REAs MAs s 14 7

D. Signi cantly increased rate o R0 resections in 2. D. Patients who are suitable operative candidates
patients undergoing neo-adjuvant therapy. with a resectable solid mass in the head o the pan-
E. Signi cantly decreased local recurrence rate in creas and no evidence o metastatic disease on thin
patients undergoing neo-adjuvant therapy. slice C scan can proceed with resection without
additional work up. PE scan is not currently rec-
5. Which statement is true regarding operative ommended as standard preoperative evaluation in
management in patients undergoing pancreati- resectable pancreatic cancer. Preoperative biliary
coduodenectomy? stent drainage is associated with stent related compli-
A. Pancreaticogastrostomy has been shown con- cations and post-operative in ectious complications
sistently to have a lower incidence o pancreatic and is not routinely recommended. Biliary drainage
leak than pancreaticojejunostomy. should be considered when there will be a signi cant
B. External drainage o pancreaticojejunostomy delay in surgery such as in malnourished patients or
has been shown to decrease pancreatic leak in patients undergoing neo-adjuvant therapy. When
rate. a stent is placed, covered metal stents have shown
C. Octreotide has been shown consistently to better patency than plastic stents and are removable
decrease the incidence o clinically signi cant at time o surgery. Routine biopsy o a pancreatic
pancreatic leak rate. mass should be avoided because o a signi cant alse
D. Internal drainage o pancreaticojejunostomy has negative rate, potential complications, and a small
been shown to decrease pancreatic stula rate. but not insigni cant alse positive rate. No prospec-
E. Extended lymphadenectomy has been shown to tive trial has shown bene t in laparoscopic staging
increase overall survival. over thin slice C scanning and it exposes patient to
two procedures under general anesthesia. However,
a selective approach or staging laparoscopy may be
ANSWERS
undertaken in patients known to be at higher risk
1. B. Given the lack o stones, painless presentation or metastases. T ese patients include tumors greater
and age o the patient the most likely etiology is a than 3 cm, tumors o body/tail and very high CA19-9
peri-ampullary neoplasm. Abdominal C scan with levels.
arterial and venous phases and 1 mm slices is the
modality o choice or identi ying pancreatic tumors, 3. A. T e above example is a borderline resectable
hepatic metastases and determining resectability. pancreatic cancer and technically resectable with
Magnetic resonance cholangiopancreatography resection o a segment o the SMV. However this is
(MRCP) is most use ul in evaluating ductal anatomy likely to result in a positive resection margin and
such as in cases o intra-ductal papillary mucinous would potentially bene t rom neo-adjuvant therapy
neoplasm (IPMN) or i choledocholithiasis is sus- to improve the R0 resection chances. Resectability is
pected. MRCP is not as accurate as thin cut C scan determined by the degree o involvement o the sur-
in determining resectability. A PE scan is not stand- rounding vessels by the tumor. T ere are three cat-
ard preoperative evaluation or a resectable lesion egories: resectable, borderline resectable, and locally
and unless combined with a thin slice dual phase C advanced. T e borderline resectable group is usually
scan, is not as good as thin cut C scan in determin- the hardest to de ne but in general represents a lesion
ing resectability. ERCP sensitivity or diagnosis o a that is resectable but runs a high risk o having a mar-
pancreas mass is no better than imaging and it has gin positive resection. wo major pancreatic surgery
largely been replaced by EUS and FNA biopsy when groups, the MD Anderson Cancer Center (MDACC)
tissue is needed because o low sensitivity or brush- and a consortium o the American Hepatopancrea-
ings in pancreatic cancer. CA19-9 level is correlated ticobiliary Association (AHPBA)/Society o Surgical
to the size o a pancreatic adenocarcinoma and can Oncology (SSO)/National Comprehensive Cancer
miss small tumors and is elevated in benign causes. Network (NCCN), have attempted to de ne what
It is correlated with extent o disease and can be used constitutes each category. However, the two groups
to guide diagnostic laparoscopy and thus would are not in agreement in what is resectable and what
be appropriate to draw once a pancreatic mass is is borderline resectable. In general, the MDACC cat-
identi ed. egories require greater vessel involvement to meet

http://surgerybook.net/
14 8 G En ERAL s U RG ERY EXAM I n ATI o n An d Bo ARd REVI EW

borderline resectable criteria. T e involvement o the local recurrence rate is signi cantly decreased.
greater than 180 degrees involvement o the SMV in Un ortunately none o these improvements in local
this case would be considered borderline resectable control has translated to improved survival.
under both de nitions. wo meta-analyses o retro-
spective data show similar survival o patients under- 5 B. Pancreatic stula is the most requent complica-
going portal/SMV resections to patients undergoing tion a er pancreatic surgery. External drainage o the
Whipple without vein resection. main pancreatic duct placed intra-operatively during
However, i the tumor in ltration involves 50% pancreaticoduodenectomy has been shown in several
or more o the vascular circum erence the survival randomized trials to decrease the rate o pancreatic
is decreased when compared to patients not requir- stula and hospital stay. T is is in contrast to a pro-
ing venous resection. T e extent o disease in this spective randomized trial that ound no bene t to
patient is not unresectable and an acceptable opera- placement o an internal stent in the main pancreatic
tive candidate should not be relegated to only pallia- duct ollowing pancreaticoduodenectomy. T e major-
tive therapy. T e use o endoscopic biliary drainage ity o randomized studies, systematic reviews, and
has largely replaced surgical bypass procedures or meta-analyses have shown no di erence in outcome
biliary obstruction. Multiple studies have shown that with respect to the occurrence o postoperative pan-
endoscopic biliary drainage has identical success and creatic stula based on pancreaticojejunostomy versus
short term e cacy as surgical bypass with decreased pancreaticogastrostomy. A single multi-center trial
morbidity, costs, and hospital stay. In addition, metal did show a decrease in the pancreatic stula rate with
stents have increased long term patency when com- pancreaticogastrostomy but no di erence in overall
pared to plastic stents. In contrast to obstruction o complication rate, length o stay or mortality. T e use
the biliary system which occurs in 80% o patients, o octreotide to reduce the rate o pancreatic stula
duodenal obstruction occurs in only 20% o patients has been extensively studied including prospective
with peri-ampullary tumors. In act, when duodenal randomized trials and meta-analyses with conf icting
palliation is necessary endoscopic therapy has been results. A recent Cochrane review (Cochrane Data-
shown to have shorter operative time, shorter hospi- base Syst Rev. 2013; 4: CD008370) did nd an overall
talization with more rapid resumption o oral intake, decrease in pancreatic leak rate and complications or
with no di erence in morbidity or mortality when the use o octreotide but no di erence in length o
compared to surgical bypass. stay or mortality. T e studies in the Cochrane review
that controlled or signi cant pancreatic leak ound
4. C. T ere are no randomized trials showing a superi- no decrease with the use o octreotide.
ority o neo-adjuvant therapy over adjuvant therapy
regarding median or overall survival. When neo- BIBLIOGRAPHY
adjuvant therapy is used, signi cantly ewer patients Brunner B, Grabenbauer GG, Meyer , Golcher H, Sauer R,
will ultimately undergo surgery when compared to Hohenberger W. Primary resection versus neoadjuvant
chemoradiation ollowed by resection or locally resectable
similar patients with borderline resectable disease
or potentially resectable pancreatic carcinoma without dis-
who do not undergo neo-adjuvant therapy because o tant metastasis. A multi-centre prospectively randomised
disease progression. T is is elt to be one o the major phase II-study o the interdisciplinary working group gas-
bene ts o neo-adjuvant therapy by sparing patients trointestinal tumours (AIO, ARO, and CAO). BMC Cancer.
rom major surgery that was not going to be suc- 2007;7:41.
cess ul because o aggressive disease. Neo-adjuvant Conrad C, Fernandez-Del Castillo C. Preoperative evalua-
tion and management o the pancreatic head mass. J Surg
therapy has shown to signi cantly reduce the nding Oncol. 2013;107(1):2332.
o node positive disease at resection when compared Fernandez-Cruz L, Jimenez Chavarria E, aura P, Closa D,
to patients taken directly to surgery. Neo-adjuvant Boado MA, Ferrer J. Prospective randomized trial o the
therapy has also consistently shown an improved e ect o octreotide on pancreatic juice output a er pancre-
R0 resection rate when compared to patients taken aticoduodenectomy in relation to histological diagnosis,
duct size and leakage. HPB. 2013;15(5): 3929.
directly to surgery and is the rationale or applying
Fiori E, Lamazza A, Volpino P, et al. Palliative management o
neo-adjuvant therapy to patients with borderline malignant antro-pyloric strictures. Gastroenterostomy vs.
resectable disease. As expected with an improved endoscopic stenting. A randomized prospective trial. Anti-
R0 resection rate and decreased nodal positivity, cancer Res. 2004;24(1):26971.

http://surgerybook.net/
C H AP TER 4 2 H EAd o f PAn C REAs M As s 14 9

Greer SE, Pipas JM, Sutton JE, et al. E ect o neoadju- resectability o pancreatic cancer. J Nucl Med. 2008;49(9):
vant therapy on local recurrence a er resection o pan- 140813.
creatic adenocarcinoma. J Am Coll Surg. 2008;206(3): opal B, Fieuws S, Aerts R, et al. Pancreaticojejunostomy
4517. versus pancreaticogastrostomy reconstruction a er pan-
Gurusamy KS, Koti R, Fusai G, Davidson BR. Somatostatin creaticoduodenectomy or pancreatic or periampullary
analogues or pancreatic surgery. Cochrane Database Syst tumours: A multicentre randomised trial. Lancet Oncol.
Rev. 2013;4:CD008370. 2013;14(7):65562.
Hartwig W, Werner J, Jager D, Debus J, Buchler MW. Improve- van den Bosch RP, van Eijck CH, Mulder PG, Jeekel J. Serum
ment o surgical results or pancreatic cancer. Lancet Oncol. CA19-9 determination in the management o pancreatic
2013;14(11):e47685. cancer. Hepatogastroenterology. 1996;43(9):71013.
Kang CM, Hwang HK, Choi SH, Lee WJ. Controversial issues van der Gaag NA, Rauws EA, van Eijck CH, et al. Preopera-
o neoadjuvant treatment in borderline resectable pancre- tive biliary drainage or cancer o the head o the pancreas.
atic cancer. Surg Oncol. 2013;22(2):12331. N Engl J Med. 2010;362(2):12937.
Maire F, Sauvanet A. Palliation o biliary and duodenal Wang Q, Gurusamy KS, Lin H, Xie X, Wang C. Preoperative
obstruction in patients with unresectable pancreatic can- biliary drainage or obstructive jaundice. Cochrane Data-
cer: Endoscopy or surgery? J Visc Surg. 2013;150(3 Suppl): base Syst Rev. 2008; (3): CD005444. doi(3):CD005444.
S2731. Warshaw AL, Lillemoe KD, Fernandez-del Castillo C. Pancre-
Moss AC, Morris E, Mac Mathuna P. Palliative biliary stents or atic surgery or adenocarcinoma. Curr Opin Gastroenterol.
obstructing pancreatic carcinoma. T e Cochrane Database 2012;28(5):48893.
o Systematic Reviews 2006; (1): CD004200. doi: 10.1002/ Winter JM, Cameron JL, Campbell KA, et al. Does pancreatic
14651858.CD004200.pub2. duct stenting decrease the rate o pancreatic stula ollow-
Ouaissi M, Giger U, Louis G, Sielezne I, Farges O, Sastre B. ing pancreaticoduodenectomy? Results o a prospective
Ductal adenocarcinoma o the pancreatic head: A ocus on randomized trial. J Gastrointest Surg. 2006; 10(9):128090;
current diagnostic and surgical concepts. World J Gastro- discussion 1290.
enterol. 2012;18(24):305869. Yang SH, Dou KF, Sharma N, Song WJ. T e methods o
Siddiqui AA, Mehendiratta V, Loren D, Hong SK, Kowalski . reconstruction o pancreatic digestive continuity a er
Fully covered sel -expandable metal stents are e ective and pancreaticoduodenectomy: A meta-analysis o random-
sa e to treat distal malignant biliary strictures, irrespective ized controlled trials. World J Surg. 2011;35(10):22907.
o surgical resectability status. J Clin Gastroenterol. 2011; Yeoh KG, Zimmerman MJ, Cunningham J , Cotton PB. Com-
45(9):8247. parative costs o metal versus plastic biliary stent strategies
Strobel K, Heinrich S, Bhure U, et al. Contrast-enhanced or malignant obstructive jaundice by decision analysis.
18F-FDG PE /C : 1-stop-shop imaging or assessing the Gastrointest Endosc. 1999;49(4 Pt 1):46671.

http://surgerybook.net/
43
Spleen/Idiopathic
T rombocytopenic Purpura
Ali Linsk and John Paul Sanders

A 53-year-old emale with a 3-week history o melena 2. Which o the ollowing statements about steroid-
presents to her primary care provider and is ound to re ractory ITP is correct?
have scattered ecchymoses, heme-positive stool, and A. Rituximab therapy can be used as an alternative to
a platelet count o 45,000. Colonoscopy screening was splenectomy and has an equivalent response rate.
per ormed 1 year ago and was negative. She does not B. Approximately 98% o patients who undergo
take any medications. Other than a mild anemia, labo- splenectomy are ound to have a permanent
ratory work-up is otherwise unremarkable. Peripheral response with no need or urther therapy.
smear and bone marrow biopsy are negative which do C. Signi cant splenomegaly is seen in I P patients
not demonstrate an underlying malignancy. She is diag- who do not respond to medical therapy.
nosed with idiopathic thrombocytopenic purpura (I P). D. Plasmapheresis is a rst-line therapy or steroid-
Despite 8 weeks o prednisone therapy and the addi- re ractory I P and should be initiated be ore
tion o intravenous immunoglobulin (IVIG), the patient considering splenectomy.
continues to have platelet counts ranging rom 20,000 to E. Although initial response rate to splenectomy is
40,000 and ongoing mucosal bleeding. She has been sent in the range o 85% to 90%, the relapse rate over
or a surgical consultation to discuss the possibility o the next 5 years can be as high as 25%.
splenectomy in the setting o her steroid-re ractory I P.
3. Regarding the preoperative and intraoperative
1. Which o the ollowing statements characterizes management o patients undergoing splenectomy
ITP? or ITP, which o the ollowing statements is correct?
A. Endothelial damage triggers deposition o plate- A. Platelet trans usion should be initiated once
lets and brin in small arterioles and capillaries, anesthesia is induced and continued into post-
leading to microvascular thrombotic events. operative day 1 until the platelet count rises
B. IgG autoantibodies directed against platelet brin- above 50,000.
ogen receptors cause increased platelet destruction B. Patients should be vaccinated against encap-
via removal by macrophages. sulated organisms (Haemophilus inf uenzae B,
C. Abnormal myeloid precursor cells are hyperplas- polyvalent Pneumococcus, and Meningococcus)
tic resulting in splenic sequestration o platelets in the holding area just prior to surgery.
and associated thrombocytopenia. C. T e splenic hilar vessels are o en divided simul-
D. Peripheral blood smears in patients with I P taneously with suture ligatures or a vascular load
show schistocytes, nucleated red blood cells, and on a linear stapler device.
basophilic stippling. D. Accessory spleens can be present in 10% to 30%
E. Renal ailure and neurologic complications are o patients and are most commonly ound at the
both seen in late stages o I P. splenic hilum, the tail o the pancreas, within the

http://surgerybook.net/
C H AP TER 4 3 S P l EEn / Id I o PATH I C TH Ro m b o C y To P En I C P u RP u RA 151

splenocolic and gastrosplenic ligaments, in the T ese autoantibodies target brinogen receptors on
omentum, or in the paraduodenal area. platelets and result in subsequent clearance o the
E. Laparoscopic splenectomy is less e ective than platelets by phagocytic cells. More recently, the term
open splenectomy in identi ying and removing primary immune thrombocytopenia has also been
accessory spleens. used to describe this condition, ref ecting the immune-
4. Which o the ollowing statements is true regarding mediated nature o the disease and the large propor-
postoperative complications af er splenectomy or tion o cases that do not involve overt signs o bleeding.
ITP? In addition, research has shown that impaired platelet
production and cell-mediated e ects may also play
A. Overwhelming post-splenectomy in ection (OPSI) a role in the pathophysiology o I P. Peripheral blood
is seen in approximately 5% o children and 15% smears in cases o I P tend to be within normal limits.
o adults, regardless o vaccination status. T rombotic thrombocytopenic purpura ( P)
B. Splenectomy is associated with an increased risk is a condition in which endothelial damage triggers
o abdominal venous thromboembolism within deposition o platelets and brin in small arterioles
the rst 90 days a er surgery. and capillaries, leading to microvascular thrombotic
C. Patients with I P who are treated with medical events. A patient with P will mani est a microan-
therapy alone are at an equivalent risk or venous giopathic hemolytic anemia, ever, severe thrombo-
thromboembolism when compared to those cytopenia, and will commonly su er rom neurologic
who undergo splenectomy. complications and renal ailure. A peripheral blood
D. It is standard o care or patients o all ages to smear will demonstrate schistocytes, nucleated red
take daily prophylactic antibiotics or at least a blood cells, and basophilic stippling.
5-year period ollowing splenectomy. Primary myelo brosis (PMF) is a malignant dis-
E. Rates o bleeding and in ectious complications order in which patients su er rom signi cant sple-
are similar in I P cases that show a response to nomegaly and cytopenias. T e underlying cause o
splenectomy and in those that do not. the condition is related to hyperplasia o an abnormal
5. Which o the ollowing statements best characterizes myeloid precursor cell resulting in marrow brosis
the current role o thrombopoietin (TPO) receptor and subsequent extramedullary hematopoiesis.
agonists in the treatment o ITP?
2. E. According to a recent consensus report on the
A. T ese agents are indicated in cases o symp-
investigation and management o primary immune
tomatic thrombocytopenia despite multiple
thrombocytopenia, 80% o patients respond to
rst- and second-line therapies, and commonly
splenectomy and this response persists in 66% o
induce remission o I P.
patients through at least 5 years ollowing surgery.
B. PO receptor agonists are e ective at increasing
Un ortunately, 20% to 30% o initial responders can
platelet counts via stimulation o megakaryocyte
experience relapses requiring additional medical or
production in the bone marrow and can be used
surgical therapy in the short- or long-term, and up to
preoperatively to boost platelet counts in prepa-
20% o patients do not respond at all. T ese numbers
ration or splenectomy.
are relatively consistent in laparoscopic and open
C. PO receptor agonists are commonly used in con-
approaches and across multiple studies.
junction with steroids as rst-line therapy or I P.
Rituximab is used as a second-line therapy in older
D. PO receptor agonists are given in the post-
patients with more co-morbidities who are not ideal
operative period in order to stimulate platelet
surgical candidates. It has also been used or patients
production and increase the chances o a thera-
who pre er non-operative management. Rituximab
peutic response to splenectomy.
is a monoclonal antibody directed against the B-cell
E. Both romiplostim and eltrombopag are com-
sur ace protein CD20. A 40% to 50% response rate
monly used PO receptor agonists that require
is seen initially but relapses are common and the
weekly intravenous in usions.
response rate at 5 years is 20%.
Plasmapheresis is a rst-line therapy or P, but
ANSWERS
is not indicated in I P. Splenomegaly should prompt
1. B. I P is an acquired, immune-mediated disorder that work-up or other diagnoses, as the spleen is usually
involves the splenic production o IgG autoantibodies. normal in size in cases o I P.

http://surgerybook.net/
152 G En ERAl S u RG ERy EXAm In ATIo n An d b o ARd REVI EW

3. D. Accessory spleens can be present in 10% to 30% o cutaneous injection and eltrombopag is given as a
patients and are most commonly ound at the splenic once-daily pill.
hilum, the tail o the pancreas, within the splenocolic PO agonists can be used in the preoperative
and gastrosplenic ligaments, in the omentum, or in setting in order to boost platelet counts in prepara-
the paraduodenal area. Both laparoscopic and open tion or splenectomy. T ey have also been used in
approaches can be e ective in identi ying accessory patients who are not good candidates or rituximab
spleens. therapy secondary to in ectious risks. Patients who
Platelet trans usion should be initiated a er the do not want to have surgery or are not good candi-
splenic artery is ligated in order to prevent platelet dates can also use PO agonists as chronic mainte-
consumption. T e splenic vessels should ideally be nance therapy. It is not standard to use these agents
ligated separately, in order to prevent arteriovenous in the postoperative period unless a relapse occurs
stula ormation. Vaccination against encapsulated and they do not increase the chances o responding
organisms should be per ormed in the 2- to 4-week to a splenectomy.
period prior to surgery or 14 days a er surgery or at
the time o discharge rom the hospital. I patients BIBLIOGRAPHY
were previously on rituximab therapy, vaccinations Beauchamp RD, Holzman MD, Fabian C, et al. T e spleen.
may need to be delayed or up to 3 months to allow In: ownsend CM, Beauchamp RD, Evers M, et al., eds.
or B-cell recovery. Sabiston extbook o Surgery: T e Biological Basis o Mod-
ern Surgical Practice. 18th ed. Philadelphia, PA: WB Saun-
ders; 2008.
4. B. In a retrospective review o 9,976 patients with Boyle S, White RH, Brunson A, Wun . Splenectomy and
I P, 1,762 who underwent splenectomy, the incidence the incidence o venous thromboembolism and sep-
o thrombotic events was evaluated. An increased sis in patients with immune thrombocytopenia. Blood.
risk o portal vein or mesenteric vein thrombosis was 2013;121(23):478290.
seen in the rst 90 days a er splenectomy, but not Mikhael J, Northridge K, Lindquist K, Kessler C, Deuson R,
Danese M. Short-term and long-term ailure o laparo-
a er 90 days when compared to patients who under- scopic splenectomy in adult immune thrombocytope-
went medical therapy alone. T ere was an increased nic purpura patients: A systematic review. Am J Hematol.
risk o venous thromboembolism in both the early 2009;84(11):7438.
and late periods in the splenectomy group. Moulis G, Sailler L, Sommet A, Lapeyre-Mestre M, Derumeaux
No consensus has been reached in terms o the H, Adoue D. Rituximab versus splenectomy in persistent
or chronic adult primary immune thrombocytopenia: an
bene t or duration o antibiotic prophylaxis. Studies
adjusted comparison o mortality and morbidity. Am J
have shown that asplenic patients carry a li elong risk Hematol. 2013;89(1):416.
o OPSI and this risk is highest in the rst 2 years a er Patel NY, Chilsen AM, Mathiason MA, Kallies KJ, Bottner
surgery. Children are at a higher risk than adults (5% WA. Outcomes and complications a er splenectomy or
vs. 0.9%) and OPSI is reported to have a 50% mortal- hematologic disorders. Am J Surg. 2012;204:101420.
ity rate. A retrospective analysis o 233 patients with Provan D, Stasi R, Newland AC, et al. International consensus
report on the investigation and management o primary
I P who underwent splenectomy showed that, in a immune thrombocytopenia. Blood. 2010;115(2):16886.
10-year ollow-up period, a stable response to sple- aghizadeh M, Muscarella P. Splenectomy or hematologic
nectomy was associated with a lower rate o in ec- disorders. In: Cameron JL, Cameron AM, eds. Current Sur-
tious and bleeding complications. gical T erapy. 10th ed. Philadelphia, PA: Elsevier Saunders:
2011:4739.
5. B. PO agonists are used or persistent thrombo- Vecchio R, Marchese S, Intagliata E, Swehli E, Ferla F, Cacciola
E. Long-term results a er splenectomy in adult idiopathic
cytopenia despite rst- and second-line medical thrombocytopenic purpura: comparison between open
and surgical therapy, and they do not induce remis- and laparoscopic procedures. J Laparoendosc Adv Surg
sion. T ey stimulate production o megakaryocytes ech A. 2013;23(3):1928.
and subsequently platelets in the bone marrow by Vianelli N, Palandri F, Polverelli N, et al. Splenectomy as a
activating the PO receptor. Upon cessation o the curative treatment or immune thrombocytopenia: a ret-
rospective analysis o 233 patients with a minimum ollow
medication, platelet counts begin to all again. T ese
up o 10 years. Haematologica. 2013;98(6):87580.
medications are considered maintenance therapy Zeng Y, Duan X, Xu J, Ni X. PO receptor agonist or chronic
and require ongoing use in re ractory cases o I P. idiopathic thrombocytopenic purpura. Cochrane Database
Romiplostim is administered as a once-weekly sub- Syst Rev. 2011.

http://surgerybook.net/
44
Small Bowel Obstruction

Patrick Golden

You are called to the emergency room to see a 66-year- D. Immediate C scan o the abdomen and pelvis
old male with a chie complaint o abdominal pain with with PO and IV contrast.
nausea and vomiting. Reviewing his ER record reveals E. Discharge to home rom the ER with close clinic
he has a heart rate o 115, blood pressure o 145/90, and ollow up within 48 hours.
a normal temperature at 98.6F. On exam the patient
appears to be in mild distress with a distended abdomen. 2. Which o the ollowing is the most common cause
T e patient states that the pain started 48 hours be ore, o bowel obstruction?
is intermittent and crampy, and is now getting worse. He A. Small bowel ileus
states that he has been vomiting now or about the last B. Mechanical small bowel obstruction
24 hours and is unable to keep any uids down. He also C. Mechanical colonic obstruction
states that he cannot remember the last time he passed D. Acute colonic pseudo-obstruction
atus or had a bowel movement. T e patient has been E. Gastric outlet obstruction
in the ER or about hal an hour, his labs are still pend-
ing, and no imaging studies have been done. His past 3. Which o the ollowing is the most common cause
medical history is signi cant or hypertension, hyper- o mechanical small bowel obstruction?
lipidemia, chronic kidney disease, diet controlled type A. Adhesions
2 diabetes, and gout. His past surgical history is signi - B. Strictures
cant or an open appendectomy 50 years earlier and a C. Hernias
laparoscopic converted to open sigmoidectomy second- D. Gallstones
ary to diverticulosis 5 years prior. T e patient denies E. umors
ever having symptoms like this be ore.
4. wenty- our hours has now passed since the above
1. Following f uid resuscitation with two liters o patient was admitted and had an nasogastric tube
crystalloid, the heart rate downtrends to 90 and (NG) and oley catheter placed. Since that time his
the blood pressure remains normal. Which o the NG tube has put out 1.6 liters o bilious f uid and
ollowing is the next best step in management o his urinary output has been around 1.1 cc/kg/hr.
this patient? T e patients pain and physical exam only worsen
A. o the OR immediately or a diagnostic laparos- intermittently. T e mans heart rate is in the 90s
copy with possible exploratory laparotomy. and his blood pressure is holding steady. Which o
B. Admission to the surgical oor, continue IV u- the ollowing is true?
ids and per orm abdominal exams every 4 hours. A. T e presence o small-bowel ecalization on
C. Placement o a nasogastric tube and an acute C scan o the abdomen and pelvis with IV/oral
abdominal series. contrast predicts the need or an operation.

http://surgerybook.net/
15 4 G EN ERAL S U RG ERY EXAM I N ATIO N AN D BO ARD REVI EW

B. I this patient had his sigmoidectomy 4 weeks ago, nal series is ast and with care ul interpretation the
he would still be a candidate or non-operative diagnosis o a small bowel obstruction can still be
management. made. A non-contrast C scan, however, would be
C. An exploratory laparotomy is indicated. a reasonable option as it would not risk an insult to
D. I a laparoscopical approach is chosen within the the kidneys and it may determine an internal hernia,
rst 48 hours o admission, it is more likely to be which would be an indication or surgical interven-
completed without conversion. tion. Answer E is not appropriate as this patient still
E. One can continue observation in this patient or requires work up.
up to 3 days without increasing morbidity.
2. A. Small bowel ileus is, by ar, the most common
5. A decision is made to proceed to surgery 24 hours orm o intestinal dilation as it is seen af er many
later because the patient developed a ever and surgical procedures whether they are per ormed in
a leuokocytosis. Which o the ollowing is true the abdomen or not. T e exact etiology o the ileus
regarding surgical management? is unclear but is probably multi actorial taking into
A. T e laparoscopic approach is associated with account anesthesia and narcotic usage. Viral ileus can
longer operating times. also be seen in cases o gastroenteritis. Ileus is most
B. T e laparoscopic approach may be associated times sel -limited and resolves with or without NG
with decreased mortality. tube decompression and keeping the patient NPO.
C. T e rate o major complications is similar or Answer B, mechanical small bowel obstruction, is
both the laparoscopic and open approaches. the second most common cause o bowel obstruction
D. T e optical view approach is recommended or and, 90% o the time, is due to adhesions, hernias,
this patient. or cancer; though gallstones, bezoars, and parasitic
E. T e laparoscopic approach has no e ect on post- worms have also been reported. Mechanical colonic
operative length o stay. obstruction, while usually more severe in its presen-
tation, only accounts or around 10% o all mechani-
cal obstructions. T e usual causes o this are cancer,
ANSWERS volvulus, or diverticulitis resulting in a stricture.
1. C. T is patient is presenting with symptoms most Answer D, Acute colonic pseudo-obstruction
consistent with some type o bowel obstruction. is much less common and is usually seen in older,
T is patient has received 2 liters o crystalloid with debilitated or institutionalized patients who are on
an appropriate response in his heart rate but he still numerous medications and answer E, gastric outlet
requires an NG tube. Given his lack o peritoni- obstruction is a less common orm o obstruction
tis, imaging studies are appropriate and an acute and is usually seen with gastric/duodenal/pancreatic
abdominal series is adequate, especially given this malignancies or gastic ulcers. T ese would not, how-
patients history o chronic kidney disease and an as ever, result in intestinal dilatation.
yet unknown creatinine level. 3. A. Up until the early to mid-1900s the leading cause
Answer A is not appropriate given that he does o mechanical small bowel obstruction was due
not have peritonitis and his work up is not yet com- to hernias but over the past century that has now
plete. Given that he is responding to resuscitation switched to postoperative adhesions accounting or
this patient should be given the chance to see i his over 75% o all mechanical small bowel obstructions.
symptoms resolve with non-operative management. Hernias are the second most common cause o
Answer B is not appropriate in that this patient still mechanical small bowel obstructions and in the
has a work up to be done with labs and imaging as absence o a surgical history the presence o an incar-
the diagnosis is still unclear at this time. cerated inguinal, umbilical or incisional hernia needs
Answer D is not appropriate in that while C has to be ruled out. T e remaining answer choices have
a high speci city and sensitivity or bowel obstruc- all been shown to cause mechanical bowel obstruc-
tions and can locate points o blockage his creatinine tions though their incidence is low.
is unknown and with his history o kidney disease
and likely kidney injury rom dehydration a con- 4. E. T is patients physical exam and clinical status
trast load would not be advisable. An acute abdomi- is largely unchanged. As such he is a candidate or

http://surgerybook.net/
C H AP TER 4 4 S M ALL BO WEL O BS TRU C TIO N 155

non-operative management or up to 72 hours. small bowel obstructions. When accessing the abdo-
Patients who have had abdominal surgery within six men in a patient with previous abdominal incisions,
weeks be ore the episode o small bowel obstruction, the recommended technique or gaining the pneu-
patients with signs o strangulation ( ever, tachycar- moperitoneum and placing the initial trocar is via
dia, luekocytosis, metabolic acidosis, continuous the Hasson or open technique.
pain), and patients with irreducible hernias are NO
candidates or non-operative management. On C
BIBLIOGRAPHY
scan, the lack o small bowel ecalization, ree intra-
peritoneal uid, and mesenteric edema predict the Arnaoutakis GJ, Eckhauser FE. Small Bowel Obstruction.
In: Cameron L, Cameron AM. Cameron Current Surgi-
need or urgent laparotomy. As he is not exhibiting
cal T erapy. 10th ed. Philadelphia, PA: Elsevier Saunders;
signs o peritonitis, ischemia, or strangulation he is 2011:937.
most appropriately a candidate or continued obser- Di Saverio S, Coccolini F, Galati M, et al. Bologna guidelines
vation. I af er 72 hours, i there is no improvement or diagnosis and management o adhesive small bowel
in his status, an operation is indicated. Until then obstruction (ASBO): 2013 update o the evidence-based
there is no increase in morbidity or mortality with guidelines rom the world society o emergency surgery
ASBO working group. World J Emerg Surg. 2013;8(1):42.
observation. T ere is no data suggesting that ear- Helton WS, Fisichella, PM. Intestinal Obstruction. In: Souba
lier intervention with a laparoscopic approach will WW, Fink MP Jurkovich GJ, Kaiser LP, Pearce WH, Pem-
reduce morbidity, mortality, or conversion rates. berton JH, et al. eds. ACS Surgery Principles and Practice.
6th ed. New York, NY: WebMD Pro essional Publishing;
5. B. In a study published in 2014 on over 9600 patients 2007:51434.
using the National Surgical Quality Improvement Kelly KN, Iannuzzi JC, Rickles AS, Garimella V, Monson JR,
Database, laparoscopic intervention or small bowel Fleming FJ. Laparotomy or small-bowel obstruction:
rst choice or last resort or adhesiolysis? A laparoscopic
obstructions was shown to have decreased opera- approach or small-bowel obstruction reduces 30-day
tive time (77.2 vs. 94.2 min) and decreased length complications. Surg Endosc. 2014;28(1):6573.
o stay (4.7 vs. 9.9 days). Additionally, the laparo- Soper NJ. Access to Abdomen. In: Soper NJ, Scott-Conner,
scopic approach was less likely to develop major Carol EH, eds. SAGES Manual: Volume 1 Basic Laparoscopy
complications, with an odds ratio o 0.7. Additionally and Endoscopy. New York: Springer, Inc., 2012.
Soybel DI, Landman WB. Ileus and Bowel Obstruction. In:
the laparoscopic approach showed a lower 30-day
Mulholland MW, Lillemoe KD, Doherty GM, et al. eds.
mortality rate (1.3% vs. 4.7%). As such i the surgeon Greenf elds Surgery Scientif c Principles and Practice. 5th
eels com ortable, the laparoscopic approach may be ed. Philadelphia, PA: Lippincott Williams & Wilkins;
a sa er alternative or the surgical management o 2011:74872.

http://surgerybook.net/
45
Illicit Drug-Induced
Abdominal Pain
Bonnie B. Hunt

A 26-year-old male with no signi cant past medical his- B. Abdominal washout, debridement o ulcerated
tory presents to the emergency room in distress, com- tissue, and omental patch closure.
plaining o chest discom ort and severe epigastric pain o C. runcal vagotomy with pyloroplasty.
sudden onset three hours prior to presentation. T orough D. runcal vagotomy with antrectomy and Billroth
history reveals cocaine abuse prior to onset o symptoms. I or II reconstruction.
T e patient is tachycardic and tachypneic, physical exam
reveals a di usely tender abdomen with rigidity. 4. In addition to the surgical management o
the per orated ulcer, what, i any, adjunctive
1. Which o the ollowing is true regarding
management should be included in the patients
complications o stimulant drug abuse?
care?
A. Per orated gastroduodenal ulcer is unlikely to
A. Nothing, the pathophysiology is directly related
occur without known history o ulcer disease.
to the stimulant drug and management is
B. Acute abdominal processes are the most com-
directed at repair o the acute per oration.
mon complications o stimulant drug abuse.
B. Empiric triple or quadruple therapy or
C. An ileus is the most well-documented acute
H. pylori.
abdominal complication o cocaine abuse.
C. Ulcer biopsy or H. pylori in ection, and triple
D. Systemic vasoconstriction may cause multiple
or quadruple therapy only with presence o
gastrointestinal complications.
bacteria.
2. Which radiographic study would most readily D. Pharmacologic vasodilation to reverse e ects o
provide the diagnostic nding or the diagnosis in stimulant physiology.
the patient presentation above?
A. C scan 5. I a patient presents with mesenteric ischemia
B. Abdominal ultrasound secondary to stimulant drug abuse along with
C. Upright chest radiograph peritonitis or an acidosis. Af er resuscitation,
D. Angiography which o the ollowing represents appropriate
management?
3. Af er making the diagnosis above, which o A. Exploratory laparotomy with resection o
the ollowing would be the most appropriate necrotic bowel and vasodilation.
intervention? B. Endovascular thrombolysis and arterial stenting.
A. Fluid resuscitation, gastric decompression via naso- C. Operative SMA embolectomy.
gastric tube, and antibiotic therapy or H. pylori D. Aortomesenteric arterial bypass.
in ection.

http://surgerybook.net/
C H AP TER 4 5 I l l IC I T D Ru g - In D u C ED Ab D o m I n Al PAI n 157

ANSWERS patients acute abdomen also presented with hema-


tochezia in the setting o stimulant drug abuse, mes-
1. D. Illicit drugs such as cocaine and methampheta- enteric ischemia would be higher on the di erential,
mine are highly addictive central nervous system diagnosed by angiogram. In the setting o stimulant
stimulants which produce a rapid euphoria sec- drug abuse and an acute abdomen, it is important
ondary to elevated levels o the monoamine trans- or the provider to obtain imaging to evaluate or the
mitters dopamine, serotonin, and norepinephrine. presence o pneumoperitoneum, indicating possible
Immediate e ects o stimulant use include wake ul- per orated peptic ulcer.
ness, increased physical activity, decreased appetite,
tachypnea, tachycardia, hypertension, and hyper-
3. B. In the absence o ree air or shock, a trial o
thermia. T e most common and well-known serious
nonoperative management may lead to spontane-
side e ects o stimulant drug abuse include cardiac
ous closure o approximately 50% o per orations.
and neurologic complications such as myocardial
Non-operative management o a per orated peptic
in arction, arrhythmias, and cerebral vascular acci-
ulcer presenting with ree air and a surgical abdo-
dents, additionally pulmonary and psychiatric com-
men would not be appropriate; surgical intervention
plications are common. T ough less common, acute
is indicated. With the success o medical therapy
abdominal complications related to both cocaine
directed toward H. pylori and gastric acid suppres-
and methamphetamine are well documented in the
sion, surgical goals or per orated ulcers have been
medical and surgical literature. T e two most com-
increasingly narrowed rom the traditional options
monly documented gastrointestinal complications
or acid-reducing operations to management o the
are per orated peptic ulcer and mesenteric ischemia,
acute complication with omental patch closure o the
both thought to be secondary to splanchnic vasocon-
per oration, with or without primary closure. Gas-
striction via activated alpha-1 receptors ollowing
troduodenal ulcerations presenting in conjunction
rapid elevation in norepinephrine. Gastroduodenal
with stimulant drug abuse in particular o en occur
per oration a er stimulant drug abuse may be the
at a younger age and without known history o ulcer
initial presentation o ulcer disease, and the clinician
disease. T ough the prevalence o H. pylori is still
should have a high index o suspicion or such when
high in stimulant-associated per orated ulcers, the
a patient presents with acute abdomen and a history
path to per oration is promoted by vasoconstriction
o stimulant abuse, despite the lack o ulcer disease in
and ocal ischemia. Given this pathophysiology, the
the patients history. Less commonly, methampheta-
indication or acid-reducing surgery remains limited
mines can cause paralytic ileus.
to per orations in patients who have ailed medical
therapy prior to per oration.
2. C. T e diagnosis o per orated gastroduodenal ulcer,
in conjunction with an appropriate history, is con- 4. C. T e traditional approach to per orated gastrodu-
rmed by the nding o pneumoperitoneum, most odenal ulcer, outside o stimulant drug use, presumes
quickly visualized on the upright chest x-ray under that the vast majority o per orated ulcers involve
the hemidiaphragm(s) or on the le lateral decu- H. pylori in ection. As such, standard treatment
bitus abdominal X-ray. In some cases o per orated includes surgical management o the acute compli-
ulcer o the anterior duodenal wall, ree air and the cation as well as empiric treatment or H. pylori with
sh-eye sign may be demonstrated via ultrasound. In triple or quadruple therapy. Standard triple therapy
approximately 25% o per orated peptic ulcer pres- or H. pylori in ection includes a proton pump inhib-
entations, ree air will not be visualized. A C scan itor and dual antibiotic coverage with clarithromycin
will demonstrate inf ammatory changes surround- plus amoxicillin or metronidazole; quadruple ther-
ing the per orated ulcer, and is highly sensitive or apy includes a proton pump inhibitor, dual antibiotic
evidence o micro-per oration ( ree air or f uid). coverage with tetracycline plus metronidazole, and
However, in diagnosing a per orated peptic ulcer or bismuth subsalicylate. Per orated gastroduodenal
which plain lm did not already reveal pneumop- ulcers ollowing stimulant drug abuse may occur
eritoneum, C scan did not demonstrate additional with or without previous symptoms o peptic ulcer
diagnostic utility within the rst 6 hours. Had the disease. Despite lack o previous symptoms, there

http://surgerybook.net/
158 g En ERAl S u Rg ERY EXAm I n ATI o n An D b o ARD REVI EW

is evidence to show that up to 80% o these ulcers subsequent exploration or reassessment o tenu-
are positive or H. pylori per intraoperative biopsy. ous bowel. Additionally, major abdominal vessels
However, despite the potential presence o H. pylori, and mesenteric per usion can be assessed by direct
a key component in the acute per oration is vasocon- vessel palpation.
striction leading to ocal ischemia, ultimately causing
ulcer per oration in patterns di erent rom standard
BIBLIOGRAPHY
ulcer behavior, notably at younger ages. Given the
potential multi actorial ulcer etiology in the set- Albertson E, Derlet RW, Van Hoozen BE. Methamphetamine
and the expanding complications o amphetamines. West J
ting o per oration ollowing stimulant drug use,
Med. 1999;170:2149.
the result o H. pylori testing on intraoperative ulcer Baker RJ. In: Fischer JE, Jones DB, Pomposeli FB, et al. eds.
biopsy can direct the inclusion o triple or quadru- Mastery o Surgery. 6th ed. Philadelphia, PA: Wolters Kluwer
ple therapy ollowing surgical intervention. Despite Health; 2011:892.
the vasoconstrictive physiology in stimulant drug Brannan A, Soundararajan S, Houghton BL. Methamphet-
related per orations, treatment recommendations amine-associated shock with intestinal in arction. Med-
GenMed. 2004;6:6.
are directed at repair o the per oration rather than Carlson , Plackett, , Gagliano RA, et al. Crystal Meth-
vasodilation. amphetamine-Induced Paralytic Ileus. Hawaii Journal
o Medicine & Public Health. February 2012; 71(2):445.
5. A. Acute mesenteric ischemia (AMI) may result Chander B, Aslanian HR. Gastric per orations associated with
rom arterial occlusion via embolus or thrombus the use o crack cocaine. Gastroenterol Hepatol (NY). 2010;
6(11):7335.
(65%), venous occlusion (10%) or non-occlusive Chey WD, Wong BCY. American college o gastroenterology
etiologies (25%). Prompt diagnosis o AMI is guideline on the management o helicobacter pylori in ec-
strongly correlated to outcomes, as a delay in inter- tion. Am J Gastroenterol. 2007;102(8):180825.
vention results in morbidity up to 50%. Selective Dunser MW, Hasibeder WR. Sympathetic overstimulation
mesenteric catheter angiography is the traditional during critical illness: Adverse e ects o adrenergic stress.
J Intensive Care Med. 2009;24:293316.
diagnostic gold standard, however abdominal C
Feliciano DV, Ojukwu JC, Rozycki GS, et al. T e epidemic o
angiography with intravenous contrast can pro- cocaine-related juxtapyloric per orations: with a comment
vide a aster diagnosis with decreased procedural on the importance o testing or Helicobacter pylori. Ann
risk to the patient. Endovascular thrombolysis Surg. 1999;229:8014.
and arterial stenting can be e ective at restor- Grassi R, Romano S, Pinto A, et al. Gastroduodenal per o-
ing splanchnic per usion in acute arterial throm- rations: conventional plain ilm, US and C indings in
166 consecutive patients. Eur J Radiol. 2004;50(1):306.
bosis; however, acute embolic arterial occlusions Herr RD, Caravati EM. Acute transient ischemic colitis a er
typically do not respond to thrombolytic therapy oral methamphetamine ingestion. Am J Emerg Med. 1991;
(which may also result in secondary distal emboli- 9:4069.
zation rom ragmented portions o the embolus), Herskowitz MM, Gilego V, Ward M, et al. Cocaine-induced
and require laparotomy with SMA embolectomy. mesenteric ischemia: reatment with intra-arterial papav-
erine. Emerg Radiol. 2002;9(3):1724.
Splanchnic vasoconstriction secondary to cat-
Kish SJ. Pharmacologic mechanism o crystal meth. CMAJ.
echolamine release in stimulant drug use is a rare 2008;178:167982.
cause o acute non-occlusive mesenteric ischemia Mulholland MW. In: Mulhollond MW, Lillemoe KD, Goherty
(ANOMI), and impacts the major arterial sources GM, et al. eds. Greenf elds Surgery: Scientif c Principles and
o bowel per usion, the SMA and IMA, as well as Practice. 5th ed. Philadelphia, PA: Lippincott Williams &
smaller collaterals. hrombolysis, embolectomy, Wilkins; 2010: Ch44, pp 699703.
Nirula R. Gastroduodenal Per oration. Surg Clin North Am.
and bypass have no role in the management o 2014;94(1):314.
these cases. Intra-arterial vasodilator therapy can Ng EK, Lam YH, Sung JJ, et al. Eradication o Helicobacter
be e ective at restoring bowel per usion. Regard- pylori prevents recurrence o ulcer a er simple closure o
less o the AMI etiology, with evidence or suspi- duodenal ulcer per oration: Randomized controlled trial.
cion o peritonitis or bowel ischemia or in arction, Ann Surg. 2000;231(2):1538.
Nirula R. Acute Mesenteric Ischemia. Surg Clin North Am.
urgent exploratory laparotomy and resection o
2014;94(1):16581.
necrotic bowel is imperative, typically conducted Osorio J1, Farreras N, Ortiz DeZrate L, et al. Cocaine-induced
according to damage control principles with mesenteric ischaemia. Dig Surg. 2000;17(6):64851.

http://surgerybook.net/
C H AP TER 4 5 I l l I C I T D Ru g - In D u C ED Ab D o m I n Al PAI n 159

Pecha RE, Prindiville , Pecha BS, et al. Association o cocaine Shanti CM, Lucas CE. Cocaine and the critical care challenge.
and methamphetamine use with giant gastroduodenal Crit Care Med. 2003;31(6):18519.
ulcers. Am J Gastroenterol. 1996;91:252327. rompeter M, Brazda , Remy C , et al. Non-occlusive mes-
Reginelli A, Iacobellis F, Berritto D, et al. Mesenteric ischemia: enteric ischemia: Etiology, diagnosis, and interventional
the importance o di erential diagnosis or the surgeon. therapy. Eur Radiol. 2002;12(5):117987.
BMC Surg. 2013;13(Suppl 2):S51.

http://surgerybook.net/
46
Carcinoid umors

Christopher Yheulon

A 50-year-old otherwise healthy male presents to the 1. What is the best next step in diagnosis?
emergency room with nausea, vomiting, and abdomi- A. Small bowel ollow through
nal pain. He has had a one month history o chronic B. Abdominal MRI
intermittent abdominal pain. He has no prior history o C. Capsule Endoscopy
abdominal surgeries and had a normal colonoscopy ear- D. Barium Enema
lier this year. His vital signs are within normal limits. On E. Diagnostic Laparoscopy
exam, his abdomen is so , non-distended, with mild di -
use abdominal tenderness. T ere are no hernias present. 2. What is the most likely malignant pathologic
His laboratory exam is unremarkable. He undergoes diagnosis?
an abdominal C scan which demonstrates an area o A. Adenocarcinoma
intussusception in the distal small bowel with thickening B. Carcinoid tumor
and calci cation o the adjacent mesentery (seen below). C. Lymphoma

http://surgerybook.net/
C H AP TER 4 6 C ARC i n o i d Tu m o Rs 161

D. Small bowel sarcoma additional in ormation, surgery remains the de ni-


E. Small bowel gastrointestinal stromal tumor tive modality or diagnosis and treatment o adult
(GIS ) intussusceptions.
T is di ers rom intussusception in a child which
3. A er you complete your diagnostic workup, is typically idiopathic or associated with a viral
you take the patient to the operating room or a in ection. Non-operative enema reduction is the
diagnostic laparoscopy. You identi y a 4 cm solid treatment o choice o uncomplicated intussuscep-
mass in the mid-jejunum. T e adjacent mesentery tion in children and has a success rate o 90%. Sur-
appears thickened and ore-shortened. T ere is gery is reserved or children with evidence o shock,
no evidence o metastatic disease. You per orm peritonitis, or ailed enema reduction.
a resection with primary anastamosis. In the
recovery room, the patient has pro ound f ushing, 2. B. T e most common small bowel malignancy is
altered mental status, and hypotension re ractory carcinoid tumor (37.4%), ollowed by adenocar-
to f uid resuscitation. What is the next best step in cinoma (36.9%), lymphoma (17.3%), and stromal
management? tumor (8.4%). T e most requent location o carci-
A. Initiation o vasopressors noid tumors are the ileum, ollowed by the duode-
B. Broad spectrum antibiotics num, and least requently in the jejunum.
C. IV octreotide 3. C. his patient has developed carcinoid crisis.
D. Stress dose corticosteroids Carcinoid crisis is a phenomenon that occurs a ter
E. rans usion o packed red blood cells (PRBCs) manipulation o tumor masses, during induction
4. T e patient ultimately recovers well and is o anesthesia, and a ter administration o chemo-
discharged. What labs should be ordered or therapy, but can also occur in up to 11% o patients
surveillance o recurrence and metastatic disease? with metastatic disease. Signi icant amounts o
serotonin, histamine, and other mediators cause
A. Chromogranin A and Urine 5-HIAA the characteristics o pro ound lushing, extreme
B. CEA and CA 19-9 changes in blood pressure, bronchoconstriction,
C. Plasma VMA and metanephrines arrhythmias, and mental status changes. I the
D. FSH and LH condition is associated with hypotension, it is usu-
E. AFP and HCG ally re ractory to luid resuscitation but can be
5. T ree years later on surveillance C scan, the treated with the in usion o plasma and the use o
patient is noted to have multiple new liver masses. octreotide.
Biopsy is consistent with metastatic disease. What I the diagnosis o carcinoid is made pre-
is the best treatment option? operatively, steps can be taken to reduce the risk o
carcinoid crisis, including administration o anti-
A. Systemic chemotherapy histamines and octreotide. Octreotide is a long act-
B. Metastatectomy ing somatostatin analogue. In addition to reducing
C. Hepatic artery embolization the release o GI hormones, it reduces the amount
D. External beam radiation o serotonin released rom tumor cells. Regardless,
E. Observation intraoperative carcinoid events are dif cult to pre-
dict and there is no standard octreotide administra-
ANSWERS tion regimen available.
1. E. Intussusception in an adult is typically due to a 4. A. Chromogranin A is a protein associated with
pathologic lead point. Only 8% to 20% o intussus- neuroendocrine cells and tumors. It is use ul or
ceptions are idiopathic. Malignancy accounts or monitoring disease response and progression in
up to 30% o cases o adult intussusception. Benign patients with carcinoid tumors. Plasma Chromogra-
causes or intussusception include Meckels divertic- nin A levels are an independent predictor o survival.
ulum, postoperative adhesions, lipomas, adenoma- 5-HIAA is the primary urinary metabolite o seroto-
tous polyps, and intestinal tubes (i.e., jejunostomy). nin. Measuring these levels are use ul or diagnosis,
Although advanced imaging techniques may provide monitoring, and prognosis o patients with carcinoid

http://surgerybook.net/
162 G En ERAL s u RG ERY EXAm i n ATi o n An d Bo ARd REVi EW

tumors. T ere is also reduced survival among survival (43 vs. 24 months) when compared to those
patients with elevated Urine 5-HIAA levels and the patients undergoing embolization.
degree o elevation is correlated with the degree o
carcinoid symptoms. BIBLIOGRAPHY
CEA is a marker or colon and pancreatic cancer. Bilimoria KY, Bentrem DJ, Wayne JD, et al. Small bowel cancer
CA 19-9 is a marker or pancreatic cancer. Plasma in the United States: Changes in epidemiology, treatment,
VMA and metanephrines are use ul in the diagnosis and survival over the last 20 years. Ann Surge. 2009;249(1):
o pheochromocytoma. FSH and LH are hormones 6371.
released by the pituitary gland that are not particu- Chambers AJ, Pasieka JL, Dixon E, Rorstad O. T e palliative
bene t o aggressive surgical intervention or both hepatic
larly use ul in tumor surveillance. AFP and b-HCG
and mesenteric metastases rom neuroendocrine tumors.
can be used to identi y testicular cancer. Surgery. 2008;144(4):64551.
Kulke MH. Clinical presentation and management o car-
5. B. Many di erent chemotherapy regimens or car- cinoid tumors. Hematol Oncol Clin North Am. 2007;21:
cinoid tumors have been investigated with response 43355.
rates ranging only rom 0% to 33%. Chemotherapy Mancuso K, Kaye AD, Boudreaux JP, et al. Carcinoid syn-
has not been shown to improve survival but is typi- drome and perioperative anesthetic considerations. J Clin
Anesth. 2011;23(4):32941.
cally used or control o symptoms caused by metas-
Marinis A, Yiallourou A, Samanides L, et al. Intussusception o
tases. the bowel in adults: A review. World J Gastroenterol. 2009;
Surgery or metastatic carcinoid disease has 15(4):40711.
been shown to relieve symptoms related to intes- Modlin IM, Kidd M, Latich I, et al. Current status o gastro-
tinal obstruction and ischemia. Multiple studies intestinal carcinoids. Gastroenterology. 2005;128:171751.
have shown that surgery provides an improved con- Musunuru S, Chen H, Rajpal S, et al. Metastatic neuroendo-
crine hepatic tumors: Resection improves survival. Arch
trol o carcinoid syndrome symptoms. In addition, Surg. 2006;141(10):10004.
cytoreductive surgery to include liver metastases has Osborne DA, Zervos EE, Strosberg J, et al. Improved outcome
shown to improve survival when compared to obser- with cytoreduction versus embolization or symptom-
vation, systemic chemotherapy, and hepatic artery atic hepatic metastases o carcinoid and neuroendocrine
embolization. tumors. Ann Surg Oncol. 2006;13(4):57281.
Rorstad O. Prognostic indicators or carcinoid neuroendo-
In 2006, Osborne et al. demonstrated that patients
crine tumors o the gastrointestinal tract. J Surg Oncol.
who underwent cytoreductive resection or meta- 2005;89(3):15160.
static carcinoid tumor had better complete symptom Vaughan DJ, Brunner MD. Anesthesia or patients with car-
relie (69% vs. 59%) and a signi cantly longer mean cinoid syndrome. Int Anesthesiol Clin. 1997;35(4):12942.

http://surgerybook.net/
47
Mesenteric Ischemia

Danielle E. Ca asso

A 67-year-old male with a history o hypertension, C. Supportive care with avoidance o vasopressors
atrial brillation, and obesity presents to the emer- and optimization o uid status
gency department with acute onset abdominal pain, D. Mesenteric bypass
nausea, vomiting, and hematochezia. Previous opera- E. Surgical embolectomy
tions include a laparoscopic appendectomy. On physical
exam the patient appears to be in signi cant distress due 4. During surgical exploration, the bowel is assessed
to pain; however, his abdomen is so , non-tender, and or viability and 50 cm o small bowel is resected.
non-distended. Laboratory analysis is remarkable or a What is the most reliable means o determining
white blood cell count o 21 109/mL and a lactate o bowel viability af er revascularization?
3.5 mmol/L. Abdominal radiography is unremarkable. A. Second-look laparotomy
C angiography is signi cant or absence o ow distal B. Acid-base status
to the origin o the superior mesenteric artery (SMA). C. Intraoperative Doppler ultrasound
D. ranscutaneous oxygen measurement
1. What is the likely etiology or this SMA occlusion?
E. Resection o all necrotic and marginal appearing
A. Splanchnic vasoconstriction bowel during initial exploration
B. Embolic occlusion o the mesenteric circulation
C. Acute thrombosis o the mesenteric circulation 5. Regarding outcomes, which o the ollowing is
D. Hypercoagulable state true?
E. Mesenteric venous thrombosis (MV )
A. Delays in diagnosis contribute to the high
2. Associated risk actors with acute mesenteric morbidity and mortality in mesenteric isch-
ischemia include which o the ollowing? emia.
A. Hypertension B. Mesenteric ischemia-reper usion is an uncom-
B. Malignancy mon cause or postoperative morbidity.
C. Recent cardiac events C. MV has a worse prognosis than other orms o
D. Hypovolemia mesenteric ischemia.
E. All o the above D. Endovascular intervention is the mainstay o
treatment or all orms o mesenteric ischemia
3. What is the optimal management approach or today.
this patient? E. Percutaneous transluminal angioplasty (P A)
A. T erapeutic anticoagulation or chronic mesenteric ischemia is associated
B. Intra-arterial thrombolytics with a low restenosis rate.

http://surgerybook.net/
16 4 G EN ERAL S U RG ERY EXAM I N ATI O N AN D BO ARD REVI EW

ANSWERS include abdominal X-rays, duplex ultrasonography,


computed tomographic angiography (C A), mag-
1. B. T ere are our distinct pathophysiologies or mes- netic resonance angiography (MRA), and contrast
enteric ischemia: arterial thromboembolism, hypop- angiography. Contrast angiography is considered the
er usion syndrome, acute arterial thrombosis on gold standard providing both diagnostic and thera-
chronic ischemia, and venous thrombosis. Arterial peutic options; however, C A and MRA are com-
types are classi ed as acute or chronic. Acute mes- monly used as well.
enteric ischemia (AMI) is due to either thromboem- Abdominal X-rays may show uid- lled loops o
bolism or hypoper usion syndromes, also known as bowel with bowel wall edema. Endoscopy may be use-
non-occlusive mesenteric ischemia (NOMI). Acute ul or acute on chronic ischemia where the duode-
on chronic mesenteric ischemia is due to thrombotic num and right colon may have evidence o ischemia.
occlusion in the setting o atherosclerotic disease. Classic angiographic patterns exist that assist in the
Finally, there is one classi cation or venous mesen- diagnosis o mesenteric ischemia. Firstly, the SMA is
teric ischemia and that is due to MV . the most common site o embolism due to its high
T e most common type is arterial embolism basal ow rate and anatomic angle o takeo rom
comprising approximately 50% o cases. T e classic the aorta. Secondly, the emboli typically lodge distal
presentation is acute onset periumbilical abdominal to the middle colic branch and the jejunal branches.
pain out o proportion to physical exam ndings. A It appears as an oval-shaped clot surrounded by
retrospective study o patients with AMI who under- contrast in a non-calci ed segment. On the other
went surgery ound the most requent presenting hand, the thrombus in acute on chronic mesen-
symptom was abdominal pain (95%), with nausea teric ischemia typically orms at the atherosclerotic
(44%), vomiting (35%), and diarrhea (35%) to ol- plaque, most commonly at the origin o the mesen-
low. Patients with thrombotic occlusion, acute-on- teric vessel causing complete absence o ow. It will
chronic mesenteric ischemia, typically have a history appear as a clot superimposed on a heavily calci ed
o chronic postprandial abdominal pain and weight occlusive lesion at the ostium. NOMI is caused by
loss due to ood ear. It is common in the elderly hypoper usion or a low- ow state, thus angiographic
population with di use atherosclerosis leading to ndings include the absence o large vessel occlusion
relative ischemia a er meals. with evidence o sequential ocal vasospasm. Finally,
Patients with mesenteric hypoper usion, or MV ndings include bowel wall thickening, delayed
NOMI, are usually critically ill patients who pre- passage o contrast into the portal system, and a lack
sent with a slower onset o di use abdominal pain o opaci cation o the portal vein. It is actually better
related to splanchnic vasoconstriction in a low- ow diagnosed with C A with portal phase enhancement.
state. T ose with MV may have various, nonspe-
ci c abdominal complaints insidious in onset which 2. E. Associated risk actors or acute mesenteric
progressively worsen over time. It is also worth men- ischemia (AMI) are essential to the diagnosis. Gen-
tioning median arcuate ligament syndrome (MALS), eral risk actors or AMI include hypertension,
which is a compressive syndrome o the celiac artery tobacco use, peripheral vascular disease, and coro-
on expiration. T e presenting symptoms are similar nary artery disease. Arterial embolic occlusion is
to other orms o mesenteric ischemia and MALS is most commonly associated with a cardiac source
o en associated with recent weight loss. MALS is a such as atrial brillation leading to atrial append-
diagnosis o exclusion and management is outside age thrombus. Other causes include valvular disease
the scope o this chapter. and le ventricular mural thrombus due to previous
Further diagnosis o mesenteric ischemia relies on myocardial in arction (MI) and subsequent hypoki-
associated risk actors, which will be discussed below, nesis. Arterial thrombosis involves a slow progression
as well as laboratory analysis and imaging. One study o atherosclerosis until a critical stenosis and subse-
ound the mean white blood cell (WBC) count to be quent thrombosis develops, thus patients will have
elevated in 98% o patients, and greater than 20 other mani estations o atherosclerotic disease. Low
109/mL in over hal o the patients. Lactate was also ow states can result in acute on chronic ischemia
elevated in greater than 90% o patients and greater and has been associated with MI, recent cardiac sur-
than 3 mmol/L in over 60% o patients. Imaging may gery, and acute viral illness. NOMI is also related to

http://surgerybook.net/
C H AP TER 4 7 M ES EN TERIC I S C H EM I A 16 5

low ow states, but without ocal occlusion. Patients exploratory laparotomy i bowel ischemia is a concern.
in cardiogenic or septic shock, those with vasopres- Revascularization usually precedes bowel resection
sor in usions, or patients with toxic pharmacologic and treatment o the underlying stenosis or occlusive
ingestion have all been implicated as inciting events lesion is typically done during the same procedure.
or NOMI. Dialysis patients with excessive uid Open surgical management involves surgical
removal are also at risk. In contrast, MV is o en bypass o the occlusion, and may be necessary in
due to systemic disease related to hypercoagulable unsuccess ul embolectomy as well. Retrograde
states such as malignancy, trauma, medications, and bypass rom the in rarenal aorta or iliac artery is
hypercoagulable syndromes. pre erred due to easier exposure and less hemody-
namic changes as it avoids supraceliac cross-clamp.
3. E. reatment goals in AMI include restoration o However, retrograde bypass may produce in erior
normal pulsatile ow and resection o nonviable results. Antegrade bypass rom the suprarenal aorta
bowel. Open surgery and endovascular interven- is less susceptible to kinking, however may be more
tion can both restore ow; however, open laparot- di cult due to post-surgical adhesive disease and
omy may be required to assess bowel viability and calci c atherosclerotic disease. A hybrid approach
is mandatory with peritonitis unless palliative with SMA thrombectomy ollowed by retrograde
management is planned. T e progression o endo- stenting o the lesion is also an option.
vascular technology and skill over the last decade NOMI is pre erably managed non-operatively
has made an endovascular or hybrid approach with optimization o uid status and cardiac out-
more common and more accepted. Multiple non- put, and limitation o vasoconstrictors. Interest-
randomized studies have been published but with ingly, one study ound that 40% o patients with
con icting reports. One retrospective study o NOMI had a potentially treatable SMA stenotic
over 4,000 patients compared outcomes or endo- lesion, thus angiography should be per ormed i at
vascular vs. open surgery in the treatment o AMI all possible. Other potential causes o NOMI that
(Beaulieu RJ, et al.) Endovascular intervention had should be addressed include aortic dissection or
a decreased mortality and shorter length o hos- abdominal compartment syndrome (ACS) which
pital stay when compared to open surgery. Other is commonly seen a er ruptured abdominal aor-
studies report decreased morbidity and mortality tic aneurysm (AAA) repair. T e mainstay o MV
or endovascular intervention in acute thrombotic treatment is therapeutic systemic anticoagulation
occlusions while others still report no di erence in and urther workup to identi y the underlying
mortality between the groups. However, no rand- cause. For the ew patients that ail medical man-
omized clinical trials exist. More importantly, endo- agement, endovascular options do exist but there
vascular or a hybrid approach requires a vascular are no studies with comparative data. In all cases,
center with appropriate acilities and support in clinical deterioration with peritonitis or any con-
place to be success ul. cern or bowel ischemia requires surgical explora-
Surgical embolectomy is the standard treatment tion to assess bowel viability.
or arterial embolic occlusion. T e occlusion is not
likely amenable to thrombolytic therapy as it is a 4. A. Regardless o surgical approach or methods to
relatively organized cardiac thrombus. Furthermore, assess bowel per usion at initial exploration, a second-
thrombolytics risks distal embolization and bowel look laparotomy is essential in the management o
in arction. T e standard surgical approach is a trans- AMI. Published criteria or a second-look lapa-
verse arteriotomy with thromboembolectomy using a rotomy include low ow state, bowel resection, and
3- or 4-Fr Fogarty catheter. In patients without perito- mesenteric thromboembolectomy. T e key is to
nitis, endovascular aspiration embolectomy is a treat- plan the return to the operating room 2448 hours
ment option. Further, catheter directed thrombolysis a er initial exploration no matter what the patients
is an alternative option in cases o incomplete aspira- condition. A er resuscitation and correction o
tion embolectomy or distal embolization. T rombotic acid-base imbalances, patients conditions may
arterial occlusion can be managed endovascularly improve drastically; however, there is still the risk or
with stenting and/or thrombolytic therapy as this is necrotic bowel requiring resection. Clearly necrotic
a resh thrombus. However, the patient may require bowel must be resected at the initial operation,

http://surgerybook.net/
16 6 G EN ERAL S U RG ERY EXAM IN ATI O N AN D BO ARD REVIEW

however marginally per used bowel needs urther mesenteric venous thrombosis. Emerg Radiol. 2009;
evaluation to limit the risk o excessive bowel resec- 16(6):47782.
tion and potentially short gut syndrome. Pulse exam, Arthurs ZM, itus J, Bannazadeh M, et al. A comparison o
endovascular revascularization with traditional therapy
intraoperative Doppler ultrasound, uorescein, and or the treatment o acute mesenteric ischemia. J Vasc Surg.
transcutaneous oxygen measurements are all intra- 2011;53(3):698704; discussion 7045.
operative diagnostic options to assess bowel viability Atkins MD, Kwolek CJ, LaMuraglia GM, Brewster DC, Chung
a er revascularization, but second-look laparotomy K, Cambria RP. Surgical revascularization versus endo-
is the most reliable means o determining the via- vascular therapy or chronic mesenteric ischemia: A com-
parative experience. J Vasc Surg. 2007;45(6):116271.
bility o marginally per used bowel a er revascula-
Beaulieu RJ, Arnaoutakis KD, Abularrage CJ, E ron D ,
rization. Schneider E, Black JH, 3rd. Comparison o open and endo-
5. A. As previously discussed, endovascular therapies vascular treatment o acute mesenteric ischemia. J Vasc
Surg. 2014;59(1):15964.
are an option but not currently considered stand-
Belkin M, Owens C, Whittemore A, Donaldson M, Conte
ard o care due to the lack o randomized controlled M, Gravereaux E. Mesenteric ischemia. In: ownsend C,
trials and evidence-based outcomes indicating the Beauchamp R, Evers B, Mattox K, eds. Sabiston textbook
superiority o endovascular intervention. P A with of surgery. 18th ed. Philadelphia, PA: Saunders Elsevier;
or without stent placement is a therapeutic option 2008.
or chronic mesenteric ischemia, however the long- Bobadilla JL. Mesenteric ischemia. Surg Clin North Am. 2013;
93(4): 92540, ix.
term risk or restenosis remains relatively high. T ere Harnik IG, Brandt LJ. Mesenteric venous thrombosis. Vasc
are no large, randomized, controlled trials compar- Med. 2010;15(5):40718.
ing P A with or without stenting and open surgical Kasirajan K, OHara PJ, Gray BH, et al. Chronic mesenteric
revascularization. T us, close surveillance or recur- ischemia: Open surgery versus percutaneous angioplasty
rence o stenosis is mandatory. and stenting. J Vasc Surg. 2001;33(1):6371.
Kibbe M, Hassoun H. Acute mesenteric ischemia. ACS Surgery:
Mesenteric ischemia has a poor prognosis with
Principles and Practice, 6th edition, 2014;10161026.
mortality rates reported at 30% or embolic and Lee H, Ko EH, Lai M, et al. Delineating the relationships
thrombotic ischemia, and 80% or NOMI. Diagnos- among the ormation o reactive oxygen species, cell mem-
tic delays may be the most important prognostic brane instability and innate autoimmunity in intestinal
actor, but comorbidities may place patients at ur- reper usion injury. Mol Immunol. 2014;58(2):15119.
ther cardiac risk and postoperative complications. Moore EE, Moore FA, Franciose RJ, Kim FJ, Bif WL, Banerjee
A. T e postischemic gut serves as a priming bed or cir-
A signi cant proportion o morbidity and mortal- culating neutrophils that provoke multiple organ ailure.
ity is due to the subsequent development o multi- J Trauma. 1994;37(6):8817.
organ dys unction syndrome (MODS) in relation Paladino NC, Inviati A, Di Paola V, et al. Predictive actors
to mesenteric ischemia and reper usion. Intestinal o mortality in patients with acute mesenteric ischemia.
reper usion injury leads to synthesis and release o A retrospective study. Ann Ital Chir. 2014 May-Jun; 85(3):
26570.
in ammatory mediators, the ormation o reactive
Park WM, Gloviczki P, Cherry KJ, Jr, et al. Contemporary
oxygen species, and cell membrane instability which management o acute mesenteric ischemia: Factors associ-
ultimately causes remote organ injury. Overall MV ated with survival. J Vasc Surg. 2002;35(3):44552.
does have a better prognosis than other orms o Rose SC, Quigley M, Raker EJ. Revascularization or chronic
mesenteric ischemia; however, long-term prognosis mesenteric ischemia: Comparison o operative arterial
or MV is based on the underlying pathology. T us bypass gra ing and percutaneous transluminal angio-
plasty. J Vasc Interv Radiol. 1995;6(3):33949.
malignancy is associated with shorter survival. Ryer EJ, Kalra M, Oderich GS, et al. Revascularization
or acute mesenteric ischemia. J Vasc Surg. 2012;55(6):
16829.
BIBLIOGRAPHY Sise MJ. Acute mesenteric ischemia. Surg Clin North Am.
Acosta S, Bjorck M. Modern treatment o acute mesenteric 2014;94(1):16581.
ischaemia. Br J Surg. 2014;101(1):e1008. allarita , Oderich GS, Macedo A, et al. Reinterventions
Acosta S, Ogren M, Sternby NH, Bergqvist D, Bjorck M. Fatal or stent restenosis in patients treated or atheroscle-
nonocclusive mesenteric ischaemia: Population-based rotic mesenteric artery disease. J Vasc Surg. 2011;54(5):
incidence and risk actors. J Intern Med. 2006;259(3): 14229.e1.
30513. Yanar H, aviloglu K, Retaken C, et al. Planned second-look
Acosta S, Alhadad A, Ekberg O. Findings in multi-detector laparoscopy in the management o acute mesenteric isch-
row C with portal phase enhancement in patients with emia. World J Gastroenterol. 2007;13(24):33503.

http://surgerybook.net/
48
Appendicitis

Erik Criman

You are called to the emergency department to evaluate B. Only 25% o adult patients with acute appendici-
a 25-year-old woman complaining o abdominal pain tis will present with a classic history.
starting 12 hours prior to presentation. She initially C. A clinical diagnosis o acute appendicitis is more
characterized the pain as a periumbilical discom ort, but accurate in men than women.
now it is sharp and localizes to her right lower quadrant. D. T e appendiceal obstruction is not the hypoth-
Review o systems is positive or a temperature eleva- esized pathogenesis o acute appendicitis.
tion (measured at 38C at home), anorexia, and nausea E. Delay in diagnosis and/or surgical interven-
without emesis, and the patient denies diarrhea, melena, tion is the most common cause o complicated
and hematochezia. Her past medical history, past surgi- (gangrenous or per orated) appendicitis rather
cal history, social history, and amily history are all unre- than patient delay in seeking medical atten-
markable. T e patients last menstrual period was three tion.
weeks ago. She does not take any scheduled medications
and has no known allergies. Measurement o her vital 3. Regarding the use of imaging in acute appendicitis,
signs demonstrate a temperature o 37.8C, heart rate o which of the following is correct?
88 beats per minute, blood pressure o 118/74 mm Hg, A. Ultrasound is the most speci c imaging study
respiratory rate o 18 breaths per minute, and oxygen available.
saturation o 98% on room air. Physical examination is B. Selective imaging has been used to lower the
normal except or ocal tenderness to palpation most accepted negative appendectomy rate to less
prominent at McBurneys point. than 20%.
C. Con rmatory imaging is not required to make
1. What is the most important laboratory test to the diagnosis o acute appendicitis in all cases
order for this patient? prior to de nitive management.
A. Complete blood count D. T e most accurate ultrasound nding in acute
B. ype and screen appendicitis is the presence o mesenteric
C. Prothrombin time (P ) and partial thrombo- lymphadenopathy.
plastin time (P ) E. For pregnant women, magnetic resonance imag-
D. Human chorionic gonadotropin ing (MRI) is not recommended when ultrasound
E. Complete metabolic panel is non-diagnostic.

2. Regarding the clinical manifestations of this 4. Regarding non-operative management of acute


diagnosis, which of the following is correct? appendicitis, which of the following is correct?
A. T e location o the appendiceal tip has little to A. Non-operative management has been de ni-
do with determining the presenting symptoms. tively shown to be cost e ective.

http://surgerybook.net/
16 8 G EN ERAL S U RG ERY EXAM I N ATI O N AN D BO ARD REVI EW

B. Surgical management is the standard o care or common in men, is 1.5 times more common in whites,
acute appendicitis in the United States. and is 11.3% more common in the summer months.
C. No trials have demonstrated a potential bene t T e estimated li etime risk o acute appendicitis is
to medical management. 8.6% or males and 6.7% or emales. T e hypothesized
D. Non-operative management has consistently pathogenesis o acute appendicitis is that o appendi-
been shown to reduce length o stay. ceal obstruction. T is may be caused by a ecalith, true
E. T e presence o a ecalith on imaging does not calculi, lymphoid hyperplasia, masses (both benign and
predispose a patient to ail non-operative man- malignant), or in ectious processes. When the appendix
agement. becomes obstructed, increased intraluminal pressure
causes localized ischemia leading eventually to per o-
5. A decision is made to take the patient to the ration and localized abscess ormation or generalized
operating room for a laparoscopic appendectomy. peritonitis.
Regarding possibilities that may occur during the
operation, which of the following is correct? 1. D. All women o childbearing age must have a preg-
A. I the appendix appears normal, a diagnostic nancy test or two reasons. First, it narrows the di er-
laparoscopy should be per ormed to look or ential diagnosis. T e di erential diagnosis or acute
other potential causes o the patients right lower right lower quadrant pain is extensive, especially in
quadrant pain. T e appendix should NO be a emale o childbearing age. It includes such enti-
removed in such cases. ties as in ammatory bowel disease (Crohns Disease,
B. I a sub-centimeter mass is identi ed at the Ulcerative Colitis), ileitis, epiploic appendagitis, cecal
appendiceal tip, one should per orm an ileo- diverticulitis, Meckels diverticulitis, and renal colic.
cecectomy. Speci c to women, obstetrical diagnoses (pregnancy,
C. I an enterotomy occurs during trocar place- ectopic pregnancy) and gynecologic diagnoses (rup-
ment, one should per orm a bowel resection and tured ovarian cyst, endometriosis, pelvic in amma-
not attempt primary repair. tory disease, tubo-ovarial abscess, Mittelschmerz,
D. I a ree rupture o the appendix with ecal con- ovarian torsion) are in play. A rectal examination
tamination o the peritoneal cavity has occurred, along with pelvic examination in women or geni-
one should do a washout only at this operation tourinary examination in men should be consid-
and manage the patient post-operatively with ered. Second, the result o this test will most directly
antibiotics. impact the diagnostic workup and management o
E. I one encounters a per orated appendix, leaving the patient. I the patient is ound to be pregnant
a drain has not been shown to reduce the rate o and is still suspected to have acute appendicitis, the
abscess ormation. diagnostic study o choice will change (ultrasound
vs. MRI over computerized tomography (C )), tera-
togenic medications should be avoided, and etal
ANSWERS
heart tones will be documented both be ore and a er
Introduction to Acute Appendicitis. anesthesia.
Acute in ammation o the vermi orm appendix,
rst described by Fitz in 1886, is one o the most com- 2. C. Clinical diagnosis o acute appendicitis is more
mon causes o the acute abdomen encountered by the accurate in men than it is in women. While the over-
general surgeon. Anatomically, this structure is a true all accuracy o a clinical diagnosis o acute appen-
diverticulum o the cecum as it contains all layers o dicitis is given as approximately 80%, this value is
the colonic wall. It receives its arterial supply rom the signi cantly higher in men than women because
appendiceal artery (a terminal branch o the ileocolic acute obstetric and gynecologic pathology may
artery). It is invariably ound at the base o the cecum con ound the diagnosis. T e anatomic location o
at the convergence o the taenia coli. T is structure is the appendix along with the time o the presenta-
histologically distinct rom the cecum in that it contains tion determines the clinical mani estations o acute
lymphoid tissue in the mucosa and submucosa. Epide- appendicitis. Further, the classic presentation o
miologically, acute appendicitis presents most com- appendicitis (vague peri-umbilical discom ort ol-
monly in the second generation o li e, is 1.4 times more lowed by nausea and anorexia ollowed by migration

http://surgerybook.net/
C H AP TER 4 8 AP P EN D I C I TI S 16 9

o the pain to the right lower quadrant) only occurs morbidity, decreased pain, and a reduced amount
in 50% to 60% o presentations. Interestingly, patient o sick leave and disability. Un ortunately, there has
delay in seeking medical attention is cited as the also been evidence that those who ail conservative
most common cause or the intra-operative ndings management more o en present with abscess or per-
o gangrenous or per orated appendicitis. T e other oration (around 30%). Non-operative management
actor that heavily in uences delay in diagnosis is has also not been shown to be cost e ective because
the patients age. Appendicitis o en presents atypi- o an associated longer length o stay along with the
cally in children (particularly those younger than cost o re-admission and cross-over to the operative
three years o age) and the elderly (de ned as those arm. T e presence o a ecalith is considered a con-
patients over sixty). Adult patients who are obese, traindication to non-operative management because
diabetic, or immunologically compromised may this nding has an association with treatment ailure,
also present in an atypical manner. A high index o recurrence, and complicated appendicitis.
suspicion is required to make the diagnosis in these
patient populations. 5. E. T ere is no evidence that drains reduce complica-
tions (e.g., abscess ormation, wound in ection) ol-
3. C. Con rmatory imaging is not required to diag- lowing appendectomy or any stage o appendicitis.
nose acute appendicitis, and there is evidence to In the event that a normal appearing appendix is
suggest that waiting or imaging only serves to delay encountered intra-operatively, care should be taken
de nitive management in cases where there is little to per orm a thorough evaluation o the peritoneal
doubt o the history, physical exam, and laboratory cavity. Most authors recommend incidental appen-
evaluation. While historically the negative appen- dectomy to remove appendicitis rom the di eren-
dectomy rate was accepted as 10% or men and 20% tial going orward. It should be noted that even i the
or women, this value has been reduced to less than appendix appears normal, nal pathology will not
10% with the selective use o pre-operative imag- in requently demonstrate in ammation consistent
ing (primarily C ) in cases where the diagnosis is with a diagnosis o acute appendicitis. Appendiceal
not clinically apparent. Computerized tomography tumors are rare and are only ound in 1% o appen-
is the most speci c imaging modality available, dectomy specimens. O these, carcinoid tumors are
with an estimated speci city o above 95%. Regard- the most common (> 50%) and 90% are located at
ing ultrasound, an appendiceal diameter o 6 mm the appendiceal tip. I the tumor is less than 2 cm,
or greater has been set as the threshold or making an appendectomy is adequate. I the tumor is greater
the diagnosis o acute appendicitis in an individual than 2 cm in size or there is a question o incom-
with coexistent right lower quadrant pain. MRI is plete resection (positive margins, grossly positive
recommended or pregnant women when the trans- lymph nodes), the National Comprehensive Cancer
abdominal ultrasound is non-diagnostic or acute Network (NCCN) recommendation is initial appen-
appendicitis. dectomy ollowed by re-exploration and right partial
colectomy i the staging evaluation is negative or
4. B. Surgical management remains the standard metastatic disease.
o care or acute appendicitis in the United States. An enterotomy may be repaired primarily i it is
Appendectomy, increasingly done using a laparo- identi ed at the time o the original operation. While
scopic approach, has become an extremely sa e pro- operative management o appendiceal phlegmon or
cedure with a reported intra-operative complication abscess at the time o presentation is controversial,
rate o 0.7% and general post-operative complication thorough washout and appendectomy (either lapa-
rate o 1.5%. In the adult literature, there has recently roscopically or via laparotomy) without drain place-
been some promising research including several ment is indicated or cases o ecal peritonitis caused
randomized, prospective trials that collectively have by ree rupture o the appendix.
documented an aggregate treatment ailure rate and
recurrence rate (adjudged as crossing over to opera- BIBLIOGRAPHY
tive management) ranging rom 14% to 32% at 2 years. Addiss DG, Sha er N, Fowler BS, auxe RV. T e epidemiology
For those who are success ully managed non- o appendicitis and appendectomy in the United States.
operatively, reviews have demonstrated decreased Am J Epidemiol. 1990;132(5):910.

http://surgerybook.net/
170 G EN ERAL S U RG ERY EXAM I N ATI O N AN D BO ARD REVIEW

Binenbaum SJ, Gold arb MA. Inadvertent enterotomy in McCutcheon BA, Chang DC, Marcus LP, Inui , Noorbakhsh
minimally invasive abdominal surgery. JSLS. 2006;10(3): A, Schallhorn C, et al. Long-term outcomes o patients
33640. with non-surgically managed uncomplicated appendicitis.
Birnbaum BA, Wilson SR. Appendicitis at the millennium. JACS. 2014; 218:905.
Radiology. 2000;215(2):337348. National Comprehensive Cancer Network. Neuroendo-
Brgger L, Rosella L, Candinas D, Gller U. Improving out- crine umors (Version 2.2014). http://www.nccn.org/
comes a er laparoscopic appendectomy: A population- pro essionals/physician_gls/pd /neuroendocrine.pd .
based, 12-year trend analysis o 7446 patients. Ann Surg. Accessed April 30, 2014.
2011;253(2):309. Parks NA, Schroeppel J. Update on imaging or acute appen-
Chiarugi M, Buccianti P, Decanini L, Balestri R, Lorenzetti L, dicitis. Surg Clin North Am. 2011;91(1):14154.
Franceschi M, Cavina E. What you see is not what you get. Park JS, Jeong JH, Lee JI, Lee JH, Park JK, Moon HJ. Accura-
A plea to remove a normal appendix during diagnostic cies o diagnostic methods or acute appendicitis. Am Surg.
laparoscopy. Acta Chir Belg. 2001;101(5):243. January 2013;79(1):1016.
Connor SJ, Hanna GB, Frizelle FA. Appendiceal tumors: Ret- Petrowsky H, Demartines N, Rousson V, Clavien PA.
rospective clinicopathologic analysis o appendiceal tumors Evidence-based value o prophylactic drainage in gastro-
rom 7970 appendectomies. Dis Colon Rectum. 1998;41(1):75. intestinal surgery: A systematic review and meta-analyses.
Daehlin L. Acute appendicitis during the rst three years o Ann Surg. 2004;240(6):1074.
li e. Acta Chir Scand. 1982;148(3):291. Pittman-Waller VA, Myers JG, Stewart RM, Dent DL, Page CP,
Di Saverio S, Sibilio A, Giorgini E, Biscardi A, Villani S, Gray GA, et al. Appendicitis: Why so complicated? Analy-
Coccolini F, et al. T e NO A Study (Non-Operative reat- sis o 5755 consecutive appendectomies. Am Surg. 2000;
ment or Acute Appendicitis): Prospective study on the 66(6):548.
ef cacy and sa ety o antibiotics (Amoxicillin and Clavu- Shindoh J, et al. Predictive actors or negative outcomes in
lanic Acid) or treating patients with right lower quadrant initial non-operative management o suspected appendici-
abdominal pain and long-term ollow-up o conservatively tis. J Gastrointest Surg. 2010;14(2):30914.
treated suspected appendicitis. Ann Surg. Jul 18, 2014. Varadhan KK, Neal KR, Lobo DN, et al. Sa ety and ef cacy o
Fitz, RH. Per orating in ammation o the vermi orm appen- antibiotics compared with appendectomy or treatment o
dix with special re erence to its early diagnosis and treat- uncomplicated, acute appendicitis: Meta-analysis o ran-
ment. Am J Med Sci. 1886;92:321. domized controlled trials. BMJ. 2012;344:e2156
Horattas MC, Guyton DP, Wu D. A reappraisal o appendi- Wagner PL, Eachempati SR, Soe K, Pieracci FM, Shou J, Barie
citis in the elderly. Am J Surg. 1990;160(3):291. PS. De ning the current negative appendectomy rate: or
Je rey RB Jr, Laing FC, ownsend RR. Acute appendicitis: whom is preoperative computed tomography making an
sonographic criteria based on 250 cases. Radiology. 1988; impact? Surgery. 2008;144(2):276.
167(2):327. Wilms I, de Hoog D, de Visser D, Janzing H. Appendec-
Lee SL, Walsh AJ, Ho HS. Computed tomography and ultraso- tomy versus Antibiotic reatment or Acute Appendi-
nography do not improve and may delay the diagnosis and citis. Cochrane Review of Systematic Reviews. 2011. doi:
treatment o acute appendicitis. Arch Surg. 2001;136(5): 556. 10.1002/14651858.CD008359.pub2.

http://surgerybook.net/
49
Colon Cancer

Robert B. Lim

A 48-year-old male is re erred to your o ce or pain- A. Computed tomography (C ) virtual colono-


less hematochezia, which has been present or 1 year. He graphy
notes some mild atigue but is otherwise asymptomatic B. Fecal immunochemical test
and has no other past medical or surgical history. He is C. Colonoscopy
adopted so his amily history is unknown. On physical D. Stool DNA
exam, he has a BMI o 32 kg/m 2. His rectal and ano- E. Capsule endoscopy
scopic exam show no lesions. His ecal occult blood test
is positive. T e rest o his exam is normal. 3. T e entire colon was able to be evaluated and a
4 cm sessile polyp is noted in the distal descending
colon on the above study. A subsequent biopsy is
1. T e patient is in the process o nding his biological done which shows a moderately di erentiated
amily and is inquiring about risk actors or colon adenocarcinoma with extension past the lamina
cancer. Which o the ollowing is true? propria. T ere are no other lesions in the colon.
A. All newly diagnosed colorectal cancers under Which o the ollowing is the next best step in
age 70 should be evaluated or the mismatch management?
repair de ciency o Lynch syndrome. A. Le colectomy
B. In amilies with suspected amilial adenomatous B. Repeat colonoscopy to determine i the lesion is
polyposis (FAP) syndrome, screening should amenable to endoscopic resection.
start in the third decade o li e and done every C. Rigid proctoscopy
5 years. D. C scan
C. In patients at risk or attenuated FAP (AFAP), the E. C /PE scan
average age o onset o colon cancer is the same
as with FAP. 4. T e patient is prepared or surgery. He is interested
D. In patients with Peutz-Jeghers syndrome, COX-2 in a laparoscopic resection so he can return to work
inhibitors have been shown to slow the progres- aster. Which o the ollowing is true regarding
sion to cancer. laparoscopic approaches?
E. In serrated polyposis syndrome, the polyps are A. T e robotic assisted laparoscopic approach has
typically hyperplastic in nature and thus, this the least amount o blood loss.
syndrome is not associated with an increase risk B. A single-incision laparoscopic approach has less
o malignancy development. pain than other laparoscopic approaches.
C. T e laparoscopic approach has higher rates o
2. For this patient, the next step in management port site/wound recurrences versus an open
should be approach.

http://surgerybook.net/
17 2 G EN ERAL S U RG ERY EXAM IN ATIO N AN D BO ARD REVI EW

D. T e laparoscopic approach has less deep surgical should begin at 20 to 25 years and continued at least
site in ections than does an open approach. every 2 years.
E. A single-incision laparoscopic approach has a Adenomatous Polyposis syndromes: T ere are
shorter length o hospital stay. three variants: milial adenomatous polyposis (FAP),
attenuated FAP (AFAP), and molecular adenoma-
5. A review o your hospitals National Surgical Quality tous polyposis (MAP). 100% o FAP patients will go
Improvement Program (NSQIP) data shows that the on to develop colon cancer with an average onset o
average length o stay or colon resections is 9 days. 39 to 41 years o age. T is condition is hallmarked
T e Chie o Surgery is considering implementing by > 100 synchronous polyps. Colon cancer screen-
an Enhanced Recovery Af er Surgery (ERAS) ing should begin at age 10 to 12 via colonoscopy and
protocol or colon surgery. Which o the ollowing should continue annually; and ortunately, screening
is true? with subsequent surgery can eliminate the mortality
A. Epidural anesthesia is a requirement. rom this disease. In AFAP, the amount o colonic
B. Use o preoperative carbohydrate drink hastens polyps is reduced to about 50% and correspond-
the return o bowel unction. ingly the average age o malignant onset is higher
C. ERAS has been shown to decrease non-surgical at 58 years and occurs in about 69% o patients with
complications. AFAP. Screening is recommended to begin in the
D. ERAS has been shown to decrease length o stay third decade o li e and continued every 1 to 2 years.
but has a higher readmission rate. In MAP patients, cancer develops in 41% to 100% o
E. T ere is no need to start a ERAS protocol patients with average age o onset o 58 years. T ere
because the national standard or length o stay are typically 20 to 99 synchronous polyps. Here the
is 10 days. MU YH is a base excision repair gene and its muta-
tion causes the oncogenesis. In addition to other
malignancies, APSs are associated with desmoids
ANSWERS
tumors.
1. A. Hereditary syndromes o the gastrointestinal Prophylactic colectomy, meaning total colectomy
tract account or 5% to 10% o GI malignancies with ileo-rectal anastomosis or total proctocolec-
including colon cancer. T ere are several known tomy with ileal pouch-anal anastomosis, should be
such syndromes: Lynch syndrome ( ormerly Heredi- considered in the late teens and early 20s and i there
tary non-polyposis colorectal cancer), adenomatous is evidence o high-grade dysplasia, polyps > 10 mm
polyposis syndromes including FAP- amilial poly- in diameter, marked increases in polyp number rom
posis syndrome, AFAP-attenuated amilial polyposis one exam to the next, and i symptoms are present.
syndrome and MAP-MU HY-associated polypo- Peutz-Jegher syndrome (PJS): PJS is hallmarked by
sis, Peutz-Jegher syndrome, Juvenile Polyposis syn- hamartomatous polyps o the GI tract and mucocu-
drome, Cowden syndrome, Serrated/Hyperplastic taneous pigmentation. Colon cancer develops in 39%
Polyposis syndrome, and the conditions o Heredi- o patients with an age o onset o 42 to 44. More
tary Pancreatic Cancer, and Gastric Cancer. All o commonly patients will present with abdominal pain
these syndrome increase the risk o colorectal cancer due to one o the hamartomatous polyps causing an
development and in the case o the FAPs, all o these intussusception. T is typically occurs in younger
patients will progress to malignancy. patients. T ose with a suspected diagnosis or at risk
Lynch syndrome (LS): T is is the most common should have screening colonoscopy and esophago-
hereditary syndrome and these patients have a mis- gastroduodenoscopy (EGD, or duodenal and gastric
match repair gene mutation. T e average age o colo- cancer risk) at age 8. I polyps are ound and can be
rectal cancer diagnosis is 44 to 61 years (sporadic controlled endoscopically, then surveillance is rec-
cancer is 69 years). LS is also hallmarked by right- ommended or every 3 years. I no polyps are ound
sided colon predominance and rapid progression and the patient is asymptomatic, the next screening
rom adenomatous polyp to cancer (35 months). All by endoscopy can be delayed until 18 years o age and
newly diagnosed colorectal cancer patients should then every 3 years. While COX-2 inhibitors are theo-
be genetically screened or LS mutations. For those retically believed to eliminate the polyps, there is no
amily members at risk, screening with colonoscopy conclusive evidence that they are help ul in doing so.

http://surgerybook.net/
C H AP TER 4 9 CO LO N C AN C ER 173

Juvenile Polyposis syndrome (JPS): JPS is de ned Fecal Immunochemical esting (FI ) and Stool
by the presence o 5 or more juvenile polyps in the DNA testing are other screening tests used to detect
GI tract, which can present in the rst decade o li e. the presence o cancer or advanced adenomas. Both
T e average age o diagnosis or JPS is 18 years o are considered to be speci c and sensitive in the
age with the mean age o colon cancer onset being detection and absence o CRC but their exact role
34 years o age. By this age, 17% to 22% o carriers will has yet to be de ned. Likely they would be used as
have developed colon cancer. Endoscopic screening screening tools in average risk patients. Positive FI
with both EGD and colonoscopy should begin at age or stool DNA tests would prompt the per ormance
12 with surveillance done every 1 to 3 years. o a colonoscopy.
Cowden syndrome (CS): 95% o CS patients have As this patient has atigue, albeit mild, the right
colonic polyps, typically hamartomatous in nature, colon still needs to be evaluated. C virtual colonos-
and they can be numerous, even over 100. CS is also copy would not be the rst diagnostic method in this
associated with ganglioneuromas and it carries a patient as it is generally used as a screening tool only
9% to 16% increased risk or colon cancer. Screen- or asymptomatic patients or or patients who would
ing should begin at age 15 and continued every 1 to not tolerate sedation well. Additionally C colonog-
2 years. raphy has not proven to be as accurate as conventional
Serrated Polyposis syndrome (SRS): SRS was pre- colonoscopy or lesions < 10 mm in diameter. Simi-
viously known as hyperplastic polyposis syndrome larly capsule endoscopy is accurate in diagnosing the
when it was elt that the only types o polyps present presence o a polyp but the accuracy decreases with
were serrated hyperplastic ones; but in act, the pol- polyps smaller than 10 mm. Moreover, the number o
yps can be adenomatous also. All o the polyps are studies and the number o patients in these studies is
serrated however. T e diagnosis can be con rmed small compared to the other modalities. At this time
clinically in patients who have > 5 serrated polyps conventional colonoscopy remains the best diag-
proximal to the sigmoid colon with at least 2 o them nostic tool to rule out malignancy or patients with
being > 10 mm, or any number o serrated polyps symptomatic colorectal disease.
proximal to the sigmoid colon and a rst degree rela-
tive with the diagnosis, or > 20 serrated polyps dis- 3. D. An extent o disease work up is recommended
tributed throughout the colon. While SRS is elt to prior to proceeding to surgery in cases o colon can-
increase the risk o colorectal cancer, the exact risk cer. T e NCCN guidelines recommend a C scan o
is unknown. T e screening age is unknown but once the chest, abdomen, and pelvis. Routine preoperative
it is diagnosed, colonoscopy is recommended every imaging has been shown to change the treatment
1 to 3 years. plan or operative plan in up to 14% o patients. In
addition to identi ying metastatic disease, preopera-
2. C. T e initial work up or a patient who presents tive imaging can be help ul with stage, nodal status,
with painless hematochezia should be a colonos- and location o the lesion. Rigid proctoscopy is only
copy. Alternatively a f exible sigmoidoscopy and needed or rectal cancer to better determine the dis-
an air-contrast barium enema (ACBE) together tance o the lesion rom the anal verge to help deter-
could be used to evaluate the hematochezia as most mine i a low anterior resection can be done. One
lesions, including malignancies, will be in the rectum need not repeat the colonoscopy because the lesion
and sigmoid colon. I a lesion is ound in the proxi- has already advanced into the submucosa and de ni-
mal colon on ACBE, then a colonoscopy will need tive removal via endoscopy is not possible. Addition-
to be done in order to biopsy the lesion. An ACBE ally, the entire colon was examined with the use o
alone would likely miss a lesion in the sigmoid colon conventional colonoscopy in the last sentence o
because the redundancy and mobility o the sig- question 2.
moid colon prevents adequate visualization o this PE scan is more routinely used a er resection o
area. Rigid proctoscopy, in such a patient, is gener- colon cancers to determine recurrence but this is not
ally reserved or accurately measuring the distance mandatory. PE scan/C colonography are recom-
the lesion is rom the anal verge. It would, there ore, mended i the proximal colon cannot be examined
not be used or diagnostic purposes to evaluate the due to an obstructing lesion or i metastatectomy is
entire colon. anticipated.

http://surgerybook.net/
174 G EN ERAL S U RG ERY EXAM IN ATI O N AN D BO ARD REVIEW

4. E. Laparoscopic colonic resection continues to site metastases, but this has not proven to be true.
be more popular and it has proven advantages o T ere is no increase in metastatic disease with the
decreased hospital length o stay, less post-operative use o laparoscopy. Furthermore, the laparoscopic
pain, decreased super cial surgical site in ection, and approach is equivalent to the open one in terms o
decreased non-surgical complications. While robot oncologic outcomes.
assisted laparoscopic surgery is touted that it will
decrease blood loss because o superior vision and 5. C. T e Enhanced Recovery A er Surgery (ERAS)
articulation o instruments, there is no proo that any protocols are designed to standardize the post-
surgical outcomes, including operative time, associ- operative course or commonly per ormed surger-
ated injury, post-operative pain, need or reoperation, ies so that surgeons, nursing staf , other providers,
and leak, are superior with use o a robot. Addition- and the patients themselves know what the process
ally, use o the robot is theoretically elt to decrease is or the patients post-operative care. ERAS proto-
discom ort and injury to the surgeon, but this has yet cols have been shown to decrease the length o stay
to be proven. Indeed there is no proven advantage a er colon surgery, as well as many other procedures.
with use o a robot in laparoscopic surgery. In the case o colon surgery, ERAS is only used or
Single incision laparoscopic surgery (SILS), also elective procedures. ERAS protocols or colon sur-
known as Single Port Access surgery or Laparo- gery generally start prior to the surgical procedure.
scopic Single Site surgery, was touted to reduce post- T e basic tenants o the protocol include the patient
operative pain, decrease recovery time, and improve drinking a high-carbohydrate drink 2 hours prior to
cosmetic outcomes, as all o the laparoscopic instru- their surgery, decreased intra-operative intravenous
ments would be placed through a single < 4 cm uid use, non-narcotic pain control, ambulation on
incision. SILS is technically more demanding than the day o surgery, early removal o a urinary cath-
traditional laparoscopic techniques. When com- eter, and starting a clear liquid diet on the evening
pared to standard multi-port laparoscopy, the SILS a er surgery. T ere are many dif erent protocols
approach is associated with a shorter length o stay. amongst hospitals, however, and none o the ten-
Initially the use o laparoscopic surgery or colon ants mentioned have individually proven bene cial
cancer resection was elt to increase the rate o port to decrease length o stay, hasten return o bowel

Infe rior
S upe rior
me s e nte ric a .
me s e nte ric a .

Right colic Le ft colic


flexure flexure

Middle colic a . Acce s s ory to


colic a .

Right colic a . Ma rgina l a .

Ile ocolic a .
Le ft colic a .

Ce ca l a a .

S igmoid a a .
Appe ndicula r a .

S upe rior
re cta l a .

http://surgerybook.net/
C H AP TER 4 9 CO LO N C AN C ER 175

unction, or decrease complications with the excep- at reducing morbidity and mortality rom Lynch Syn-
tion that early urinary catheter removal decreases drome in relatives. Genet Med. 2009;11(1):3541.
post-operative urinary tract in ections. T us, while Greco, M, et al. Enhanced recovery program in colorectal
surgery: a meta-analysis o randomized controlled trials.
an epidural may reduce the need or narcotic World J Surg. 2014;38(6):153141.
pain medications, it is not a requirement or ERAS Jarvinen HJ, et al. en years a er mutation testing or Lynch
protocols. Syndrome: Cancer incidence and outcome in mutation-
ERAS protocols have been shown to reduce post- positive and mutation-negative amily members. J Clin
operative length o stay to about 5 to 7 days and they Oncol. 2009;27:47937.
Kijima S, et al. Preoperative evaluation o colorectal cancer
have shown a reduction in non-surgical complica-
using C colonography, MRI and PE /C . World J Gastro-
tions like pneumonia and myocardial in arction. enterolo. 2014; 20(45):1696475.
However, they have not shown a reduction in surgical Kuhry E, et al. Long-term results o laparoscopic colorec-
complications like bleeding, super cial surgical site tal cancer resection. Cochrane Database Syst Rev. 2008;
in ections, deep surgical site in ections, leaks, and so 16(2):498504.
on. T e readmission rate a er using ERAS protocol is Sali L, et al. Role o preoperative C colonography in patients
with colorectal cancer. World J Gastroenterolo. 2014;20(14):
similar to non-protocol post-operative outcomes in 3795803.
that 10% to 13% o patients are readmitted. Maggiori L, et al. Single-incision laparoscopy or colorectal
T e NSQIP data is used to track 30-day outcomes resection: A systematic review and meta-analysis o more
or all surgeries per ormed at hospitals that partici- than a thousand procedures. Colorectal Dis. 2012;4(10):
pate. A length o stay o 8 days has been determined 64354.
Martin-Lopez, JE, et al. Comparison o the accuracy o C
as the national standard and those outside o 8 days
colonography and colonoscopy in the diagnosis o colorec-
represent the bottom tenth percentile o hospitals tal cancer. Colorectal Dis. 2014;16(3):829.
and are thus considered outliers. Length o stay, how- Mauchley DC, et al. Clinical utility and cost-ef ectiveness o
ever, does not necessarily mean there are poorer out- routine preoperative computed tomography scanning in
comes or patients. patients with colon cancer. Am J Surg. 2005;189(5): 5127.
Stracci F, Zorzi M, Grazzini G. Colorecatl cancer screening:
ests, strategies, and perspectives. Front Public Health.
BIBLIOGRAPHY 2014;2:210.
Bailey CE, Hu C-H, You YN, Kaur H, Ernst RD, Chang GJ. Syngal S, et al. ACG Clinical Guideline: genetic testing and
Variation in positron emission tomography use a er management o hereditary gastrointestinal cancer syn-
colon cancer resection. J Oncol Pract. 2015;11(3): dromes. Am J Gastroenterol. 2015;110: 22362.
36372. rinh BB, Jackson NR, Hauch A , Hu , Kandil E. Robotic
Evaluation o Genomic Applications in Practice and Preven- versus laparoscopic colorectal surgery. JSLS. 2014;18(4):
tion (EGAPP) Working Group. Recommendations rom e00187.
the EGAPP working group: Genetic testing strategies in Vasen HF, et al. Guidelines or the clinical management o
newly diagnosed individuals with colorectal cancer aimed amilial adenomatous polyposis. Gut. 2008;57:70413.

http://surgerybook.net/
50
Crohns Disease

Andrew T. Schlussel

A 28-year-old woman with a history o Crohns disease C. Enterectomy with stapled anastomosis
presents to the emergency room with worsening diar- D. Non-operative management
rhea, >20 bowel movements a day, and a 10% loss in body E. Proximal diversion
weight over the past 6 weeks. She has previously been
managed on 5-aminosalicylic acid and In iximab. Her 3. Surgical management o the ileosigmoid f stula in
vital signs on evaluation are signi cant or a heart rate o the above patient is best approached with which o
105 beats per minute and a temperature o 100.6F. Her the ollowing techniques?
abdomen is so with mild le low quadrant tenderness, A. En bloc resection o all in amed tissue with
and her rectal exam is normal. Her white blood cell count proximal diversion.
is 15 103/L. She undergoes a computed tomographic B. Debridement and primary closure o stula site
(C ) enterography that shows an ileosigmoid stula with on sigmoid colon i there are no secondary signs
a surrounding three-centimeter rim enhancing uid col- o in ammation.
lection that abuts the abdominal wall, and a more proxi- C. Enterectomy, debridement o stula tract, and
mal ileoileal stula. T ere is no ree air or ree uid. segmental colectomy.
D. Small bowel resection with primary anastomosis
1. A surgical intervention should be o ered or which alone.
o the ollowing presentations o Crohns disease? E. No surgical indications.
A. Enteroenteric stulas with mild symptoms.
B. Medically managed disease but on two medica- 4. Which o the ollowing is true regarding the
tions. medical management o enteroenteric f stulas?
C. New in ammatory changes o the terminal A. In iximab has demonstrated some bene t in the
ileum on radiographic ndings. closure o intra-abdominal stulas.
D. Ileosigmoid stula and associated diarrhea and B. In iximab has a higher rate o closure or intra-
malnutrition. abdominal stulas compared to perianal stulas.
E. Intra-abdominal abscess amenable to C -guided C. T ere is no role or anti- umor Necrosis Factor
percutaneous drainage. agents in the setting o Crohns disease.
D. T ere is no risk in postoperative anastomotic
2. In the absence o symptoms, malnutrition and complications with pre-operative use o an anti-
diarrhea, which o the ollowing is the best option umor Necrosis Factor agent.
or the above patient?
A. Ileocecectomy 5. Which o the ollowing is true regarding the
B. Enterectomy with primary hand sewn anasto- development o cancer in Crohns disease?
mosis

http://surgerybook.net/
C H AP TER 5 0 C Ro H n s D i s EAs E 17 7

A. T ere is a higher rate o a Crohns associated malig- only be per ormed back to grossly normal appearing
nancy in a stula tract compared to a stricture. bowel.
B. T e risk o colon cancer in Crohns disease is no
greater than the general population. 4. A. In iximab is a human chimeric antibody directed
C. Surveillance colonoscopy is not indicated in a against the pro-in ammatory cytokine tumor necrosis
patient with an ileosigmoid stula. actor ( NF). T is therapy was one o the original
D. Colon cancer in the setting o Crohns disease monoclonal antibodies to show improvement o symp-
should be managed similar to the general popu- toms, complete endoscopic remission and a decreased
lation. need or surgery in Crohns disease. In iximab has
E. Subtotal colectomy with ileorectal anastomosis been used in the treatment o stulizing Crohns dis-
has better outcomes in Crohns disease than a ease. In a review o 26 patients with stulizing Crohns
segmental colectomy. disease, in iximab alone was able to result in a com-
plete closure; however, only one was intra-abdominal.
ANSWERS T e ACCEN II study published in 2004 evalu-
ated the maintenance dosing o in iximab in patients
1. D. Crohns disease was rst described in 1932. It with stulizing Crohns who responded to induc-
involves in ammation o the entire gastrointestinal tion therapy. T e majority o patients had perianal
tract rom mouth to anus, and 50% o patients present stulas. T is study showed complete stula closure
with ileocolic disease. T e Vienna system, published in in 36% o patients at 54 weeks. When evaluated by
2000, is most commonly used or the classi cation location, approximately 97% o perianal stulas had
o Crohns disease. It includes: 1. Penetrating or s- complete closure, demonstrating the superior out-
tulizing; 2. Stricturing; and 3. Non-penetrating, non- comes compared to intra-abdominal stulas.
stricturing Crohns disease. Surgical management o Anti- NF agents will impede the immune
this disease is reserved or patients who ail medical response ollowing surgery. El-Hussuna and col-
treatment or who present with complications that leagues per ormed a systemic review o post-operative
include: hemorrhage, per oration, abscess, stula, complications ollowing the use o anti- NF alpha
strictures, malignancy, and growth retardation in agents within three months prior to an operation.
children. Percutaneous drainage o an intraabdomi- Fourteen studies were reviewed, and overall there
nal abscess in an otherwise stable patient should be was no dif erence in anastomotic complications
considered prior to an operative intervention. (7.6% vs. 8.2% in the control groups). In a subgroup
analysis, studies determined to have a lower chance
2. D. Patients with Crohns disease o en undergo mul-
o bias identi ed a greater risk o adverse events with
tiple abdominal operations with 40% to 55% requir-
the administration o anti- NF agents.
ing an operation 10 years ollowing their diagnosis,
and 75% requiring surgery in their li etime. Surgery
is not recommended or asymptomatic enteroenteric 5. D. T e risk o colorectal cancer in Crohns disease
stulas. However, exacerbation o diarrhea with asso- is a controversial topic, with some reports o up to
ciated malnutrition is an indication or resection. a 20- old increase in risk compared to the general
population. Lovasz and colleagues in 2013 demon-
3. B. A systematic approach to the surgical manage- strated the rate o colorectal cancer to be 5.5% a er
ment o Crohns disease is critical to the preserva- 5 years, and 7.5% a er 10 years o disease duration,
tion o bowel length. Nutrition should be optimized with a greater prevalence in stricturing disease. Fur-
as tolerated and preoperative studies to include thermore, it was recommended to employ closer
esophagogastroduodenoscopy, colonoscopy, and C endoscopic surveillance compared to the general
enterography should be considered. It is critical to population ollowing diagnosis.
determine the site o in ammation, as o en times Surveillance colonoscopy or Crohns disease is
the colon only needs to be debrided with primary recommended at similar time intervals as ulcerative
closure o the stula site. Only the diseased portion colitis; beginning 8 to 10 years a er disease onset or
o colon or small bowel should be removed. How- pancolitis, 15 years or le -sided disease, and then
ever, healthy bowel should be excised i the stula annually a er 30 years ollowing diagnosis. Surgi-
is compromising the mesentery. Resection should cal management o a Crohns associated malignancy

http://surgerybook.net/
17 8 G En ERAL s U RG ERY EXAM i n ATi o n An D Bo ARD REVi EW

should be treated like the general population with Lovasz BD, Lakatos L, Golovics PA, et al. Risk o colorectal
segmental resection and associated lymphadenec- cancer in crohns disease patients with colonic involvement
tomy. T ere have been reports that recommend sub- and stenosing disease in a population-based cohort rom
Hungary. J Gastrointestin Liver Dis. 2013;22(3):2658.
total colectomy in the setting o a Crohns associated Lu KC, Hunt SR. Surgical management o crohns disease. Surg
colorectal cancer due to the risk o synchronous and Clin North Am. 2013;93(1):16785.
metachronous lesions. Peters CP, Eshuis EJ, oxopeus FM, et al. Adalimumab or
crohns disease: Long-term sustained bene t in a popula-
tion-based cohort o 438 patients. J Crohns Colitis. 2014;
BIBLIOGRAPHY 8(8):86675.
Castellano J, Frank MS, Brandt LJ, Mahadevia P. Metachro- Peyrin-Biroulet L, Lo us EV, Jr, Colombel JF, Sandborn WJ.
nous carcinoma complicating crohns disease. Arch Intern T e natural history o adult crohns disease in population-
Med. 1981;141(8):10745. based cohorts. Am J Gastroenterol. 2010;105(2):28997.
Cooper DJ, Weinstein MA, Korelitz BI. Complications o Poritz LS, Rowe WA, Koltun WA. Remicade does not abolish
crohns disease predisposing to dysplasia and cancer o the the need or surgery in stulizing crohns disease. Dis Colon
intestinal tract: Considerations o a surveillance program. Rectum. 2002;45(6):7715.
J Clin Gastroenterol. 1984;6(3):21724. Sands BE, Anderson FH, Bernstein CN, et al. In iximab main-
Crohn BB, Ginzburg L, Oppenheimer GD. Regional ileitis: A tenance therapy or stulizing crohns disease. N Engl J
pathologic and clinical entity. 1932. Mt Sinai J Med. 2000; Med. 2004;350(9):87685.
67(3):2638. Sands BE. New therapies or the treatment o in ammatory
El-Hussuna A, Krag A, Olaison G, Bendtsen F, Gluud LL. T e bowel disease. Surg Clin North Am. 2006;86(4):104564.
ef ect o anti-tumor necrosis actor alpha agents on post- Steele SR. Operative management o crohns disease o the
operative anastomotic complications in crohns disease: A colon including anorectal disease. Surg Clin North Am.
systematic review. Dis Colon Rectum. 2013;56(12):142333. 2007;87(3):61131.
Gasche C, Scholmerich J, Brynskov J, et al. A simple classi- Strong SA, Koltun WA, Hyman NH, Buie WD, Standards
cation o crohns disease: Report o the working party Practice ask Force o T e American Society o Colon
or the world congresses o gastroenterology, vienna 1998. and Rectal Surgeons. Practice parameters or the surgical
Inf amm Bowel Dis. 2000;6(1):815. management o crohns disease. Dis Colon Rectum. 2007;
Jess , Winther KV, Munkholm P, Langholz E, Binder V. Intes- 50(11):173546.
tinal and extra-intestinal cancer in crohns disease: Follow- Warren S, Sommers S. Cicatrizing enteritis as a pathologic
up o a population-based cohort in copenhagen county, entity; analysis o 120 cases. Am J Pathol. 1948;24(3):475
Denmark. Aliment Pharmacol T ere. 2004;19(3):28793. 501.

http://surgerybook.net/
51
Diverticulitis

Erik Roedel

A 47-year-old obese man presents to the emergency 3. Regarding the indications or elective colon resec-
room with a two day history o sharp, constant, le tion a er an episode o uncomplicated diverticu-
lower quadrant pain that is worse with straining and litis, which o the ollowing is the strongest
is associated with subjective evers. T is is the patients indication?
rst episode o pain. His vital signs are normal and A. A patient with continued chronic symptoms that
he is in no distress. You are able to elicit moderate ail to resolve with antibiotics.
tenderness in the le lower quadrant on exam, and a B. I the rst episode occurs in a patient younger
complete blood count shows a mild leukocytosis o than the age o 50.
13,000/L. C. A er the second episode o uncomplicated
diverticulitis.
1. Regarding the patient described above, what is the D. A er the rst episode in order to prevent uture
single best study to obtain in order to con rm your per oration and abdominal sepsis.
suspected diagnosis? E. A er three documented cases o uncomplicated
A. Plain abdominal X-rays diverticulitis.
B. Magnetic resonance imaging (MRI) o the abdo-
men 4. Regarding a similar patient who presents without
C. Computed tomographic (C ) scan o the abdo- signs o sepsis but on imaging has evidence o
men and pelvis with IV and oral contrast complicated diverticulitis with a 6 cm pelvic
D. C scan o the abdomen and pelvis with rectal abscess, what is the best treatment plan?
contrast A. Discharge with PO antibiotics and close ollow
E. Abdominal ultrasound up.
B. Admission, IV antibiotics, and percutaneous
2. Regarding the treatment o acute, uncomplicated,
drainage.
sigmoid diverticulitis in a reliable patient, what is
C. Admission, IV antibiotics, and Laparoscopic
the most appropriate treatment plan?
washout with drain placement.
A. Discharge with pain medication. D. Admission, IV antibiotics, laparoscopic washout
B. Discharge with oral (PO) antibiotics and close with drain placement, and planned laparoscopic
ollow up. resection in 68 weeks.
C. Admission, IV antibiotics, and bowel rest. E. Admission, IV antibiotics, and resection o the
D. Urgent exploratory laparotomy and resection o diseased segment with end colostomy.
the involved colon.
E. Discharge with PO antibiotics and planned lapa- 5. Regarding a patient who presents with similar sym-
roscopic resection in 6 to 8 weeks. ptoms and early signs o sepsis and an abdominal

http://surgerybook.net/
18 0 G EN ERAL S U RG ERY EXAM IN ATI O N AN D BO ARD REVIEW

exam consistent with dif use peritonitis, what is there is signi cant evidence supporting the sa ety o
the best course o action? outpatient management without antibiotics. o be a
A. Admission, IV antibiotics, and percutaneous candidate or outpatient therapy the patient must be
drainage. stable, non-toxic, reliable, able to maintain an ade-
B. Admission, IV antibiotics, and Laparoscopic quate enteral diet, and have uncomplicated disease.
washout with drain placement. Finally, the patient should have close interval ollow-
C. Admission, IV antibiotics, laparoscopic washout up to ensure their symptoms are improving.
with drain placement, and planned laparoscopic
resection in 68 weeks. 3. A. T e current indications or elective resection
D. Admission, IV antibiotics, and proximal diver- in diverticulitis include chronic symptoms, com-
sion without resection. plicated disease, inability to rule out a malignancy,
E. Admission, IV antibiotics, and operative therapy and patient speci c actors. T e surgical manage-
with resection o the diseased segment. ment o uncomplicated diverticulitis has changed.
Previous retrospective studies, per ormed prior to
the use o C scan, reported recurrence in one third
ANSWERS o patients a er an uncomplicated episode. Recent
1. C. Multi-slice C has become the standard imag- data has shown not only a lower recurrence rate,
ing modality to con rm the diagnosis o sigmoid 13% to 23%, but more importantly, a low risk o
diverticulitis. It has been shown to have a sensitivity complicated disease and need or emergent surgery.
and speci city as high as 98% and 99% respectively Due to these ndings, ASCRS recommends against
with intravenous and intra-luminal contrast. A C routine sigmoid resection or the indications o :
scan is use ul to con rm the diagnosis o diverticu- two or more uncomplicated episodes, age o < 50,
litis, determine disease severity, and guide treatment. or to prevent complicated disease in the uture. T e
Despite the accuracy o cross-sectional imaging in guidelines recommend that each patient be evalu-
the detection o diverticulitis, it may provide a diag- ated individually regarding their overall health, age,
nostic dilemma, as radiographically a colonic neo- access to care, number, severity, and requency o
plasm can appear similar. T ere ore, colonoscopy is episodes, as well as the impact o each episode on
recommended a er the acute process has subsided. the patients quality o li e. T ese multiple actors
Plain radiographs o the abdomen are inexpen- should be weighed against the risks associated with
sive, easily available, and expose the patient to mini- elective resection. T ought ul counseling can be
mal radiation. However, they o er limited diagnostic per ormed with the patient, and an in ormed deci-
in ormation in the evaluation o diverticulitis. Mag- sion can be made.
netic resonance imaging (MRI) and ultrasound may As a subgroup, immune-compromised patients
be a use ul alternative in a patient where a C scan have been ound to be at a higher risk o mortality
or intravenous contrast is contraindicated. Ultra- with non-operative treatment; there ore, surgeons
sound in some studies has diagnostic accuracy up should have a lower threshold or resection in this
to 97%; however, it is limited by operator variability, patient population.
patient discom ort, and inability to accurately make
an alternative diagnosis. MRI is not limited by the 4. B. T e treatment o complicated diverticulitis with
same issues as ultrasound and has a sensitivity and an associated abscess has drastically changed with
speci city as high as 94% and 92% in some studies. the advent o image-guided percutaneous drainage.
A treatment plan o antibiotics with selective drain-
2. B. Recent publications have questioned the patho- age has been shown to allow up to 75% o patients
physiology o uncomplicated diverticulitis and have with a diverticular abscesses to avoid urgent surgery.
disputed the need or antibiotics in these cases. How- Percutaneous drainage is bene cial as a bridge or
ever, current guidelines recommend that the stand- stable patients without signs o peritonitis that would
ard treatment o acute uncomplicated diverticulitis have previously needed an urgent operation. Drain-
in the United States is outpatient antibiotic therapy. age provides patients the opportunity o an elective
T is is supported by the American Society o Colon single stage resection, the avoidance o surgery in
and Rectal Surgeons (ASCRS) and will continue until the acute in ammatory stage, a lower risk o ostomy

http://surgerybook.net/
C H AP TER 5 1 D I VERTI C U LITIS 181

placement, and an overall decrease in morbidity and BIBLIOGRAPHY


mortality. Abbas S. Resection and primary anastomosis in acute compli-
Currently, there is no data to support the outpa- cated diverticulitis, a systematic review o the literature. Int
tient management o acute complicated diverticuli- J Colorectal Dis. 2007;22(4):3517.
tis. Regarding the decision or elective resection a er Alonso S, Pera M, Pares D, et al. Outpatient treatment o
patients with uncomplicated acute diverticulitis. Colorectal
an episode o complicated diverticulitis, ASCRS Dis. 2010;12(10):e27882.
recommends that resection should be considered Ambrosetti P, Chautems R, Soravia C, Peiris-Waser N, errier F.
in all patients that are appropriate surgery candi- Long-term outcome o mesocolic and pelvic diverticular
dates. However, C scan ndings o a phlegmon abscesses o the le colon: A prospective study o 73 cases.
or extraluminal gas alone does not indicate com- Dis Colon Rectum. 2005;48(4):78791.
Ambrosetti P, Jenny A, Becker C, errier F, Morel P. Acute le
plicated disease. Finally, there is insuf cient data to
colonic diverticulitis-compared per ormance o computed
show that laparoscopic lavage is a sa e alternative to tomography and water-soluble contrast enema: Prospec-
resection in patients with purulent or eculent peri- tive evaluation o 420 patients. Dis Colon Rectum. 2000;
tonitis. ASCRS does not currently support opera- 43(10):13637.
tive therapy without resection or the treatment o Anaya DA, Flum DR. Risk o emergency colectomy and colos-
diverticulitis. tomy in patients with diverticular disease. Arch Surg. 2005;
140(7):6815.
5. E. T e treatment or patients with complicated diver- Baker ME. Imaging and interventional techniques in acute
ticulitis with evidence o purulent or eculent perito- le -sided diverticulitis. J Gastrointest Surg. 2008;12(8):
13147.
nitis is an operative approach with resection o the
Broderick-Villa G, Burchette RJ, Collins JC, Abbas MA, Haigh
diseased bowel. T ere is ongoing research in Europe PI. Hospitalization or acute diverticulitis does not man-
to determine i there is a role or laparoscopic lav- date routine elective colectomy. Arch Surg. 2005;140(6):
age in patients with severe complicated diverticulitis. 57681; discussion 5813.
However, as stated earlier, ASCRS does not currently Chabok A, Pahlman L, Hjern F, Haapaniemi S, Smedh K,
support non-resectional therapy except in the rare AVOD Study Group. Randomized clinical trial o antibiot-
ics in acute uncomplicated diverticulitis. Br J Surg. 2012;
circumstance that the abdomen is too hostile or 99(4):5329.
extirpation o the colon. Previous literature has Constantinides VA, ekkis PP, Senapati A, Association o
evaluated diversion without resection, but this tech- Coloproctology o Great Britain Ireland. Prospective mul-
nique was shown to have an increased rate o post- ticentre evaluation o adverse outcomes ollowing treat-
operative peritonitis as compared to resection. ment or complicated diverticular disease. Br J Surg. 2006;
93(12):150313.
In general, the extent o resection should include
Constantinides VA, ekkis PP, Athanasiou , et al. Primary
the entire sigmoid colon with a proximal margin o resection with anastomosis versus Hartmanns procedure
so and pliable descending colon. While not all the in nonelective surgery or acute colonic diverticulitis: A
diverticula need to be removed, it is important to systematic review. Dis Colon Rectum. 2006;49(7):96681.
ensure that none are included in the anastomosis i Destigter KK, Keating DP. Imaging update: Acute colonic
one is per ormed. T e distal aspect o the specimen diverticulitis. Clin Colon Rectal Surg. 2009;22(3):14755.
de Korte N, Ulna C, Boermeester MA, et al. Use o antibiot-
should include a margin o normal rectum in order ics in uncomplicated diverticulitis. Br J Surg. 2011;98(6):
to reduce the risk o recurrence. 7617.
T e decision to per orm a primary anastomo- Dozois EJ. Operative treatment o recurrent or complicated
sis with or without protective ileostomy versus end diverticulitis. J Gastrointest Surg. 2008;12(8):13213.
colostomy in the setting o emergent surgery or Eglinton , Nguyen , Raniga S, et al. Patterns o recurrence
in patients with acute diverticulitis. Br J Surg. 2010;97(6):
diverticulitis is currently being debated. T ere is
9527.
nonrandomized and retrospective data that sup- Etzioni DA, Chiu VY, Cannom RR, et al. Outpatient treatment
ports primary anastomosis. It was ound to be sa e o acute diverticulitis: Rates and predictors o ailure. Dis
and not associated with worse outcomes in the set- Colon Rectum. 2010;53(6):8615.
ting o complicated diverticulitis. T e inherent selec- Feingold D, Steele SR, Lee S, et al. Practice parameters or
tion bias in these studies has prevented the surgical the treatment o sigmoid diverticulitis. Dis Colon Rectum.
2014;57(3):28494.
community rom making broad recommendations Forgione A, Leroy J, Cahill RA, et al. Prospective evaluation o
regarding primary anastomosis in the setting o unctional outcome a er laparoscopic sigmoid colectomy.
emergent colectomy or diverticulitis. Ann Surg. 2009;249(2):21824.

http://surgerybook.net/
18 2 G EN ERAL S U RG ERY EXAM I N ATIO N AN D BO ARD REVI EW

Hall JF, Roberts PL, Ricciardi R, et al. Long-term ollow-up Salem L, Flum DR. Primary anastomosis or Hartmanns pro-
a er an initial episode o diverticulitis: What are the pre- cedure or patients with diverticular peritonitis? A system-
dictors o recurrence? Dis Colon Rectum. 2011;54(3):2838. atic review. Dis Colon Rectum. 2004;47(11):195364.
Holmer C, Lehmann KS, Engelmann S, et al. Long-term out- Sarma D, Longo WE, NDSG. Diagnostic imaging or diver-
come a er conservative and surgical treatment o acute ticulitis. J Clin Gastroenterol. 2008;42(10):113941.
sigmoid diverticulitis. Langenbecks Arch Surg. 2011;396(6): Shabanzadeh DM, Wille-Jorgensen P. Antibiotics or uncom-
82532. plicated diverticulitis. Cochrane Database Syst Rev. 2012;
Hwang SS, Cannom RR, Abbas MA, Etzioni D. Diverticulitis 11:CD009092.
in transplant patients and patients on chronic corticoste- Stollman N, Raskin JB. Diverticular disease o the colon. Lan-
roid therapy: A systematic review. Dis Colon Rectum. 2010; cet. 2004;363(9409):6319.
53(12):1699707. T aler K, Baig MK, Berho M, et al. Determinants o recur-
Janes S, Meagher A, Frizelle FA. Elective surgery a er acute rence a er sigmoid resection or uncomplicated diverticu-
diverticulitis. Br J Surg. 2005;92(2):13342. litis. Dis Colon Rectum. 2003;46(3):3858.
Lameris W, van Randen A, Bipat S, et al. Graded compres- Zeitoun G, Laurent A, Rou et F, et al. Multicentre, random-
sion ultrasonography and computed tomography in acute ized clinical trial o primary versus secondary sigmoid
colonic diverticulitis: Meta-analysis o test accuracy. Eur resection in generalized peritonitis complicating sigmoid
Radiol. 2008;18(11):2498511. diverticulitis. Br J Surg. 2000;87(10):136674.

http://surgerybook.net/
52
Colon, Rectum, and
AnusRectal Cancer
Ronald A. Gagliano, Jr.

A 48-year-old white man presents to your o ce with a Pathologic evaluation returns a diagnosis o
history o seven months o intermittent painless blood moderately dif erentiated adenocarcinoma, about 9
per rectum with bowel movements, which was diagnosed cm rom the anal verge. What i any tests remain to
as internal hemorrhoids by his primary care provider. He complete the pre-treatment staging?
has no other medical problems and has an unknown am- A. ransabdominal liver ultrasound
ily history because he was adopted as an in ant. On your B. KRAS mutation genotype
digital and anoscopic evaluations, you nd scant bloody C. Fusion PE /C scan o the chest, abdomen, and
mucous, no pathologic hemorrhoids, no other anal dis- pelvis
eases, and the visualized area o rectal mucosa is normal. D. No urther evaluation is necessary.
E. Magnetic resonance imaging (MRI) o the pelvis
1. What is the next best step in his evaluation?
A. Immunohistochemical ecal occult blood test 4. Preoperative staging reveals a 4 cm moderately
B. Guaiac ecal occult blood test dif erentiated adenocarcinoma 2N1M0 located
C. Colonoscopy at 9 cm rom the anal verge. What is the pre erred
D. Computed tomographic (C ) colonography treatment strategy?
E. Flexible sigmoidoscopy A. Pre-operative chemotherapy ollowed by chemo-
radiation therapy ollowed by ull thickness
2. A exible endoscopic examination demonstrates transanal excision o the primary lesion.
an endoscopically unresectable mass lesion B. Low anterior resection o the rectum ollowed by
concerning or cancer is located at approximately chemotherapy or chemoradiation therapy based
14 cm rom the anal verge and a biopsy is per or- on pathologic evaluation o the resection speci-
med. T e next best step to evaluate the lesion is? men.
A. C scan o the pelvis C. Pre-operative chemotherapy ollowed by chemo-
B. Rigid proctoscopy radiation therapy ollowed by low anterior
C. Serum carcinoembryonic antigen (CEA) level resection o the rectum.
D. Hepatic unction tests D. Chemoradiation ollowed by low anterior resec-
E. Fusion positron emission tomography (PE )/ tion o the rectum.
C scan o the chest, abdomen, and pelvis E. Pre-operative chemotherapy ollowed by chemo-
radiation therapy ollowed ull thickness trans-
3. A colonoscopy shows no other lesions. High resolu- anal excision o the primary lesion.
tion C scans o the chest, abdomen, and pelvis do
not demonstrate distant disease and recommended 5. Regarding additional testing to evaluate the
blood tests demonstrate only a mild anemia. patient, which is recommended and demonstrated

http://surgerybook.net/
18 4 G EN ERAL S U RG ERY EXAM IN ATIO N AN D BO ARD REVI EW

to be cost ef ective or all rectal cancer patients rectal cancer. However, tumors diagnosed in the rec-
under 50? tum rom 10.1 cm to 11.9 cm rom the anal verge can
A. Somatic APC gene sequencing be treated as colon cancers, as there seems to be no
B. umor gene sequencing bene t or local control rom neoadjuvant or adju-
C. Circulating tumor cells vant chemoradiation therapy. C is part o the stag-
D. Hypermethylation analysis o tumor DNA ing o colorectal cancer, but C o the pelvis alone
E. Mismatch repair protein analysis and/or micro- is incomplete or staging, and both endorectal ultra-
satellite instability analysis sound in skilled hands and 3.0 MRI o the pelvis
are better modalities to evaluate or local disease and
pelvic regional metastasis. Serum CEA is indicated
ANSWERS until a colon or rectal cancer is diagnosed by biopsy,
1. C. T e diagnostic concern in this patient is primar- but it is not the next best step. Further, it does not
ily cancer, as the most likely other causes o painless advance the patients evaluation at this time. Hepatic
rectal bleeding have been ruled out based on in o ce unction tests are unhelp ul and not indicated unless
evaluation. Other likely diagnoses to consider would there is coexistent liver disease. PE /C scan does
be some orm o colitis or bleeding diverticulosis, not replace a high resolution C o the chest, abdo-
but these are less likely due to the lack o abdomi- men, and pelvis with oral and IV contrast, and is gen-
nal symptoms, bleeding only with bowel movements. erally not indicated in the initial evaluation o colon
From the choices listed, the immunohistochemi- and rectal cancer.
cal ecal occult blood test (iFOB ) and Guaiac ecal
occult blood test (gFOB ) are designed to detect 3. E. T e lesion is now clearly a rectal adenocarcinoma
blood in the GI tract; there ore, they do not advance with no evidence o distant metastasis, giving us a
the patients work up as he has visible gross blood in stage o xNxM0. What we need next is the tests that
his bowel movements. Both exible sigmoidoscopy determine the and N status in order to generate
and colonoscopy have the ability to diagnose and our treatment plan, thereby making the choice o no
biopsy cancers, but only colonoscopy can evaluate urther evaluation wrong. T e best test listed is the
the entire colon and possibly treat the source o the MRI o the pelvis. T is test gives the most accurate
bleeding. In a comparison o our diagnostic strat- assessment o tumor depth and presence o nodal
egies or adult rectal bleeding, colonoscopy was the metastasis without the user variability o endorectal
best overall strategy. C colonography is controver- ultrasound. It is also the best test to assess the pos-
sial in its utility in colorectal cancer screening, but it sibility o a threatened circum erential radial margin
is not recommended in the evaluation o the symp- by invasion o or near to the mesorectal ascia. Given
tomatic patient, with sensitivity that is not equivalent the normal C o the abdomen, there is no indication
to beroptic endoscopy. or the liver ultrasound and again there is generally
no indication or the PE /C in the initial staging o
2. B. T e location o the lesion is critical or the next colorectal cancer, especially i a normal high resolu-
phases o the evaluation and management o this tion C scan is normal. KRAS mutation rom wild
tumor, as there are signi cant di erences in the eval- type is use ul or predicting whether or not biologic
uation and management o certain stages o colon therapy will be use ul, but only in patients with meta-
versus rectal cancers, as well as very proximal versus static disease, which this patient does not have.
middle and distal rectal cancers. Flexible endoscopy
is notoriously unreliable with regards to accurate 4. D. T e patient now can be staged as Stage IIIA and
measurement o a distal lesion, and generally over- the lower edge o the tumor is below 10 cm rom the
estimates the distance rom the verge. Rigid proctos- anal verge. Based on current guidelines, this patient
copy is mandatory or any lesion in the rectum or should have neoadjuvant chemoradiation therapy
recto-sigmoid junction, as it is reliable in its results. ollowed by low anterior resection using a total mes-
It ideally should be per ormed by the operating sur- orectal approach with a distal margin 4 cm rom the
geon. umors with the distal most aspect located tumor. T e transanal excision choices are incorrect
above 12 cm rom the anal verge are diagnosed as based on nodal status and tumor size o 4 cm and
colon cancer and below 12 cm are diagnosed as location o the lower edge above 8 cm rom the anal

http://surgerybook.net/
C H AP TER 5 2 CO LO N , REC TU M , AN D AN U S REC TAL C AN C ER 18 5

verge. T e choice or low anterior resection (LAR) o age i the Bethesda or Amsterdam criteria are
rst then chemoradiation is wrong because in met. Somatic APC gene sequencing is used to diag-
patients who can tolerate the neoadjuvant therapy, nose an index mutation in the amilial adenomatous
there is an absolute decrease in local recurrence o polyposis (FAP) syndrome. umor gene sequenc-
about 7% (6% rom 13%) in these patients who get ing may be use ul in some settings, but not be ore
chemoradiation prior to operation compared to af er per orming the more rapid and cheaper testing or
the operation, and is backed by category 1 data. T e Lynch syndrome. T e presence o circulating tumor
choice with chemotherapy then chemoradiation cells are being studied as a diagnostic tool or cancer
then LAR is an option or care, but not pre erred in and cancer recurrence, but currently is not routinely
this situation. used. Hypermethylation analysis o tumor DNA is
not routinely done and is suggested as the cause o
5. E. T e patient has no knowledge o his amily his- cancer or patients with MMR protein abnormality
tory, nor any timely way to get it, thereby making a detected in hMSH1 and a BRAF mutation, rather
preoperative determination o his risk or a hereditary than a somatic mutation.
colorectal cancer syndrome by Bethesda or Amster-
dam criteria impossible. Approximately 2% to 4% BIBLIOGRAPHY
o rectal cancers are known to be due to Hereditary Allen E, Nicolaidis C, Hel and M. T e evaluation o rectal
Non-Polyposis Colorectal Cancer (HNPCC) syn- bleeding in adults. A cost-e ectiveness analysis comparing
drome or Lynch syndrome. T is syndrome is caused our diagnostic strategies. J Gen Intern Med. January 2005;
by germline mutation in any one o a number o genes 20(1):8190. PMID:15693933.
involved with the process o DNA Mismatch Repair Bipat S, Glas AS, Slors FJ, et al. Rectal cancer: Local staging
and assessment o lymph node involvement with endolu-
(MMR). Impairment o MMR results in accumula-
minal US, C , and MR imagingA meta-analysis. Radiol-
tion o length alterations in simple repeated DNA ogy. September 2004;232(3):77383.
ragments called microsatellites. T is accumulation Evaluation o Genomic Applications in Practice and Preven-
leads to a state o genetic instability termed Microsat- tion (EGAPP) Working Group. Recommendations rom
ellite Instability (MSI). T e syndrome is urther char- the EGAPP Working Group: Genetic testing strategies in
acterized by an increased incidence o cancers in the newly diagnosed individuals with colorectal cancer aimed
at reducing morbidity and mortality rom Lynch syndrome
emale genital system and the urinary system, as well in relatives. Genet Med. January 2009;11(1):3541.
as data supporting an increase in brain, pancreatic NCCN Clinical Practice Guidelines in Oncology Rectal
and gastric cancers in certain amilies. T e Evalua- Cancer v3. 2014. www.nccn.org/pro essionals/physicians_
tion o Genomic Applications in Practice and Pre- gls/pd /rectum.pd .
vention (EGAPP) working group o the CDC ound NCCN Clinical Practice Guidelines in Oncology Colorectal
Cancer Screening v2.2013. www.nccn.org/pro essionals/
it to be cost e ective or MMR and/or MSI testing
physicians_gls/pd /colon.pd .
or all colorectal cancers in patients under 50 years Sauer R, Becker H, Hohenberger W, et al. Preoperative versus
o age, and many cancer centers in the United States postoperative chemoradiotherapy or rectal cancer. N Engl
test all patients under 70 and all patients regardless J Med. October 21, 2004;351(17):173140.

http://surgerybook.net/
53
Perirectal Abscess

Alexander Malloy

A 40-year-old woman presents with a ve day history o 3. Regarding the outcomes o treatment or anal
worsening gluteal and rectal pain. Her pain is dull and f stula, which o the ollowing is correct?
constant and she describes a sensation o rectal ull- A. T ere is no di erence in stula closure rate
ness. She has evers and chills. She has had no changes in with brin glue versus standard stulotomy, but
bowel habits and denies constipation. She denies puru- those who underwent brin glue placement have
lence, hematochezia, melena, or incontinence. She has shorter recovery times.
some discom ort with de ecation but no pain. She denies B. Fistulas occur in 66% o patients who have peri-
a history o in ammatory bowel disease, hemorrhoids, or rectal abscesses.
rectal prolapse. She denies trauma to the area. She is not C. Non-cutting setons have an increased risk o
sexually active. Her signi cant past medical and surgi- incontinence.
cal history includes an appendectomy. In the emergency D. All anterior stulas in emales should be man-
department, she is ebrile to 102.3F, a heart rate o 103, aged with a cutting seton.
and a blood pressure o 120/83. She has some induration
and erythema 45 cm laterally and anterior to the anal 4. Regarding perirectal abscesses, which o the
verge on the right with tenderness, no uctuance, normal ollowing is correct?
sphincter tone, no masses, and no induration. No stula
openings visible. Laboratory workup is signi cant only A. Horseshoe abscesses can occur in two planes and
or a white blood cell count o 19.2. usually require imaging or diagnosis.
B. Immunocompromised patients can be managed
1. What would be the expected course o a f stula-in- with bedside incision and drainage and intrave-
ano (i present)? nous antibiotics.
C. Crohns disease is responsible or the majority o
A. Radial to the posterior midline
perirectal abscesses.
B. Curvilinear to the anterior midline
D. Necrotizing sof tissue in ection o the perianal
C. Radial to the anterior midline
and perineal areas has a mortality rate approach-
D. Curvilinear to the posterior midline
ing 50%.
2. What is the pre erred operative management o
horseshoe abscesses? 5. Regarding perirectal abscesses, which o the
A. Intravenous antibiotics ollowing is correct?
B. Presacral drainage A. Supralevator abscesses can be a complication o
C. Incision and drainage with counter incision diverticulitis.
D. Single incision and drainage B. Perianal abscesses always require a counter inci-
E. ransabdominal drainage sion.

http://surgerybook.net/
C H AP TER 5 3 P ERi REC TAl Ab s C Es s 18 7

C. Fistulas encountered at the time o incision and 3. A. Fistula-in-ano represent a common chronic
drainage o a perirectal abscess should be surgi- problem af er perirectal abscess. Fibrin glue and con-
cally repaired. ventional stulotomy have similar outcomes or rates
D. External mani estations o intersphincteric o recurrence and closure o stula tracks, but brin
abscesses generally include a stula track. glue insertion may have shorter return to work times.
Fistulas occur in up to 25% to 50% o patients
who have perirectal abscess. Setons are placed to
ANSWERS promote drainage o stulas and to acilitate bro-
1. D. According to Goodsalls rule, the transverse anal sis o the stula track. Cutting setons are gradually
line generally dictates the course o a stula track. tightened by the surgeon in the outpatient setting,
External ori ces o a stula track posterior to the which eliminates the stula while gradually dividing
transverse anal line course in a curvilinear ashion to the sphincter complexes, putting the patient at risk
the posterior midline. External ori ces anterior to the or incontinence.
transverse anal line generally course in a radial ash- Non-cutting setons have a signi cantly reduced
ion to the anterior midline. However, i a stula is pre- risk o incontinence when compared to cutting
sent greater than 2 to 3 cm rom the anal verge, their setons, with some studies suggestive o preservation
course does not ollow the traditional radial course to o continent unction, and generally are accepted or
the anterior midline. T ey typically produce curvilin- treatment o high intersphincteric stula tracks.
ear tracks that may end at the posterior midline or just
lateral to the posterior midline. T is patients area o 4. D. As stated earlier, horseshoe abscesses can occur
induration and erythema oci is approximately 4 to in the supralevator plane, the ischiorectal plane, and
5 cm rom the anal verge and thus could be considered the intersphincteric plane. Imaging is not necessary
a long anterior stula track; there ore, D is correct. or the diagnosis but does provide a help ul adjunct
to clinical correlation.
2. C. Horseshoe abscesses encompass almost the Immunocompromised patients should gener-
entire circum erence o the rectum and can occur in ally not be managed in the outpatient setting af er
the supralevator plane, the ischiorectal plane, and the incision and drainage o a perirectal abscess. T ey
intersphincteric plane. Each area o concern dictates may or may not mani est systemic or local signs o
the clinical management. Intravenous antibiotics are in ammation and can present with delayed diag-
not adequate treatment or suppurative perirectal noses o sepsis or necrotizing sof tissue in ection.
disease and may predispose the patient to worsening T ese patients are generally managed with inpatient
perirectal sepsis. admission, incision and drainage, and intravenous
Presacral drainage is not the recommended treat- antibiotic therapy. I the abscess is a super cial peri-
ment or horseshoe abscesses and is reserved or rec- anal one, then incision, drainage, and packing may be
tal injuries in the settings o trauma. Single incision per ormed with close ollow-up.
and drainage may be e ective in certain abscesses Crohns disease mani ests in several di erent ways
but is generally not recommended in horseshoe and can certainly be the inciting actor or perirec-
abscesses. Drainage may be inadequate or ine ec- tal abscesses and complex stula disease. However,
tive and counter incisions are pre erred which also the most common cause o perirectal suppuration is
includes unroo ng o the external sphincter com- cryptoglandular ormation o in ection. Other causes
plex. T is same modality is also pre erred in the set- include malignancy, trauma, and hidradenitis suppu-
ting o a horseshoe stula. ritiva.
ransabdominal drainage may be warranted in Necrotizing sof tissue in ection o the perirectal
certain clinical scenarios involving supralevator and perineal region is a li e threatening process, as
abscesses when the origin o the abscess is intra- is the disease in any sof tissue location. Intersphinc-
abdominal in nature. ransabdominal drainage or a teric abscesses can of en go unnoticed or misdiag-
cryptoglandular abscess runs the risk o inadequate nosed and can result in a delayed diagnosis. In areas
drainage and transsphinctertic stula ormation, without access to healthcare, perirectal disease can
along with the risk o systemic sepsis due to inade- progress rapidly as well. In ection can worsen and
quate drainage. spread through sof tissue planes, progressing to a

http://surgerybook.net/
18 8 G EN ERAl s U RG ERY EXAM i N ATi O N AN D b O ARD REVi EW

Pe rire c tal abs c e s s e s

S upra le va tor Le va tor


a bs ce s s a ni mus cle

Is chioa na l (is chiore cta l)


P e ria na l Inte rna l Exte rna l a bs ce s s
a bs ce s s s phincte r s phincte r
Inte rs phincte ric (intra mus cula r
or s ubmucos a l) a bs ce s s

necrotizing sof tissue in ection, requiring wide local Chung W, Kazemi P, Ko D, Sun C, Brown CJ, Raval M, et al.
debridement. In some cases, the debridement is large Anal stula plug and brin glue versus conventional treat-
enough that a diverting colostomy is required or ment in repair o complex anal stulas. Am J Surg. May
2009;197(5):6048. doi:10.1016/j.amjsurg. 2008.12.013.
wound healing. Mortality rates with necrotizing sof Dunn KM, Rothenberger DA. Colon, Rectum, and Anus.
tissue in ection are approximately 40% to 50%. In: Brunicardi, CF, et al. eds. Schwartzs Principles of Sur-
gery. 9th ed. New York, NY: McGraw-Hill Co.: 2010:
5. A. Supralevator abscess can occur as a result o intra- 10604.
abdominal pathology such as diverticular abscesses, Henrichsen S, Christiansen J. Incidence o stula-in-ano
malignancy, or trauma. Perianal abscesses usually are complicating anorectal sepsis: A prospective study. Br J
super cial, simple abscesses without stula tracks Surg. 1986;73:371372.
Huber P Jr, Kissack AS, Simonton C . Necrotising sof -tissue
and are treated with a single cruciate incision with in ection rom rectal abscess. Dis Colon Rectum. 1983; 26(8):
packing, there ore B is incorrect. 50711.
Fistulas encountered at the time o a perirec- Nelson, H. Anus. In: ownsend CM et al. eds. Sabiston Text-
tal abscess are common. However, stulotomy is book of Surgery. 19th ed. Philadelphia, PA: Elsevier: 2012:
generally not per ormed in the acute setting as no 13926.
North JH Jr, Weber K, Rodriguez-Bigas MA, Meropol NJ,
improved clinical outcome has been documented
Petrelli NJ. T e management o in ectious and nonin ec-
with acute management. Intersphincteric abscesses tious anorectal complications in patients with leukemia.
generally have a more insidious clinical course and J Am Coll Surg. October 1996;183(4):3228.
plague the patient with complaints o vague discom- Pezim ME. Success ul treatment o horseshoe stula requires
ort without external mani estations. Induration and deroo ng o deep postanal space. Am J Surg. May 1994;
tenderness can be elicited on a digital rectal exami- 167(5):5135.
Sentovich SM, Fibrin glue or anal stulas: long-term results.
nation but other ndings are of en inconsistent. T is Dis Colon Rectum. April 2003;46(4):498502.
type o abscess requires an exam under anesthesia Vial M, Pars D, Pera M, Grande L. Faecal incontinence af er
or adequate treatment modalities. seton treatment or anal stulae with and without surgi-
cal division o internal anal sphincter: a systematic review.
BIBLIOGRAPHY Colorectal Dis. 2010 Mar;12(3):1728. doi: 10.1111/j.1463-
Cirocco WC, Reilly JC. Challenging the predictive accuracy 1318.2009.01810.x.
o Goodsalls rule or anal stulas. Dis Colon Rectum. June
1992;35(6):53742.

http://surgerybook.net/
Br east Disease
Richard Smith

http://surgerybook.net/
This page intentionally left blank

http://surgerybook.net/
54
Breast Mass

Anita Mamtani

A 57-year-old, otherwise healthy emale presents to has no palpable lesion on physical exam. T e most
your clinic to discuss the results o her recent screen- appropriate next step is:
ing mammogram. She has previously had unremark- A. Repeat mammography in 6 months.
able screening mammograms since the age o 40. She B. Magnetic resonance imaging (MRI) o the breast.
has two aunts with breast cancer, and recently heard C. Genetic screening or inherited mutations.
that her 35-year-old niece was diagnosed with invasive D. Stereotactic core needle biopsy.
ductal adenocarcinoma. She reports no palpable masses
noticed on breast sel -exams, no nipple drainage, no 3. Pathology reveals lobular carcinoma in situ (LCIS).
breast skin changes, and no other concerning constitu- T e patient now seeks advice on her overall risk o
tional symptoms. cancer. Which o the ollowing is true?

1. T e patient requests counseling on various genetic A. LCIS has a 40% li etime risk o development into
syndromes, given the young age o her recently lobular carcinoma.
diagnosed niece. Which o the ollowing is B. LCIS is not considered a precursor itsel , but
correctly matched? has an increased risk o malignancy in either
breast.
A. BRCA1chromosome 13qincreased emale C. Ductal carcinoma in situ (DCIS) is not consid-
and male breast cancer risk. ered a precursor to malignancy.
B. BRCA1chromosome 17qincreased emale D. LCIS and DCIS are both precursors to malig-
breast and ovarian cancer risk. nancy.
C. BRCA2chromosome 17qincreased emale E. LCIS is the most common type o in-situ breast
and male breast cancer risk. lesion.
D. Li-Fraumeni syndromeP EN mutation
increased breast, nervous system, and GI tract
4. T e patient opts to proceed with prophylactic
cancers.
bilateral mastectomies. In discussion o operative
E. Peutz-Jehgers syndromeKRAF mutation
details, she mentions that her niece, who had Stage
increased breast and colon cancer, mucocutane-
I invasive ductal carcinoma with positive estrogen
ous lesions.
receptor status, underwent a lumpectomy with
2. T e patients screening mammogram reveals a 1.1. cm radiation. Which o the ollowing is true?
density in the subareolar region o her le breast. A. All patients bene t rom chemotherapy, regard-
She promptly undergoes diagnostic mammogram less o nodal status.
with ultrasound or urther evaluation o the lesion, B. Adjuvant therapy with tamoxi en results in sig-
which the radiologist classif es as BIRADS 4B. She ni cant prolongation o disease- ree and overall

http://surgerybook.net/
192 G EN ERAL S U RG ERY EXAM I N ATI O N AN D BO ARD REVI EW

survival in patients with HER2/neu gene overex- and male breast cancer risk), Li-Fraumeni syndrome
pression. (p53 mutation, increased breast cancer, sarcoma, leu-
C. T e purpose o radiation is to decrease the risk kemia, brain tumors), Cowdens syndrome (P EN
o distant metastasis. mutation, increased breast, nervous system, and GI
D. Patients with early stage breast cancer who tract cancers), and hereditary non-polyposis colon
undergo lumpectomy with sentinel lymph node cancer (HNPCC, mismatch repair de ect, increased
biopsy ollowed by radiation have similar sur- colon, endometrial, ovarian, and breast cancers).
vival rates as those who undergo ormal axillary
lymph node dissection. 2. D. For BIRADS 4 or 5 lesions detected on screen-
E. Aromatase inhibitors may be help ul or adju- ing studies, stereotactic core biopsy is considered the
vant therapy in pre-menopausal women. pre erred rst step in pathologic diagnosis.
Clinical eatures concerning or breast cancer
5. At her ollow-up visit a er her operation, the include xed/ rm palpable masses, nipple retraction,
patient reports eeling a sensation o numbness in skin changes, and unilateral nipple discharge. Annual
her upper inner arm. She has ull motor strength mammography is valuable as a screening tool in
o the extremity, and the remainder o her arm women 40 years in age, although is limited in
has normal sensation. Her incisions appear to be women younger than 35 due to increased breast den-
healing well. What is the most likely etiology o sity. Concerning mammographic ndings include an
this? architectual distortion, developing asymmetry, mass
A. Injury to cutaneous branches o the brachial (with partially or ill-de ned margins), or a cluster o
plexus. microcalci cations with amorphous, linear, branch-
B. Injury to the thoracodorsal nerve. ing patterns. Screening mammography in women 50
C. Injury to the intercostobrachial nerve. years and older has decreased breast cancer mortal-
D. Disruption o the lymphatic network. ity by 33%. Ultrasound may be used to distinguish
E. Occult post-operative seroma in the axilla. solid and cystic masses. Magnetic resonance imaging
(MRI) is available as an adjunctive tool, but cannot
replace mammography or ultrasound. Fine needle
ANSWERS
aspiration (FNA) yields simple cytological results but
1. B. T e BRCA1 gene is located on chromosome 17q, cannot distinguish carcinoma in situ rom invasive
and con ers an increased risk o emale breast and cancer because it does not include in ormation about
ovarian cancer. architecture. Core biopsy is considered the pre erred
Breast cancer is the most common cancer in rst step in pathologic diagnosis, particularly or
women, and is second to lung cancer as a cause o palpable lesions and or lesions detected on screen-
death in women, with a li etime risk o 1 in 8. Risk ing studies. In general, a two-step approach o initial
actors or developing breast malignancy include biopsy ollowed by de nitive operation, i indicated,
a amily history o breast cancer, early menarche, is the pre erred pathway o management.
late menopause, personal history o breast or uter-
ine cancer, and nulliparity. T e Gail Model is a tool 3. B. LCIS is not considered a pre-cancerous lesion, but
designed to calculate breast cancer risk by assess- is associated with an increased risk o malignancy in
ing risk actors including personal medical history, either breast. amoxi en can decrease this risk, and
reproductive history, and history o breast cancer depending on tumor biology and individual patient
among rst-degree relatives, and translating these risk, management that may be advised includes close
into a score to estimate risk o developing invasive observation with serial monitoring, chemopreven-
breast cancer over set periods o time. tion, or surgery.
Approximately 10% o breast cancers are heredi- DCIS is considered a precursor to ductal adeno-
tary in nature. Genes and syndromes implicated in carcinoma, the most common histologic type o
hereditary BCA include, among others, the BRCA1 breast cancer, and is associated with a signi cant
(chromosome 17q, increased emale breast and increase in risk o ipsilateral cancer. 10% to 20% o
ovarian cancer risk) and BRCA2 mutations (chro- patients with newly-discovered DCIS already have
mosome 13q, increased ovarian cancer, and emale associated invasive carcinoma, and when untreated,

http://surgerybook.net/
C H AP TER 5 4 BREAS T MAS S 193

40% to 60% o these patients will develop ipsilat- ing axillary lymph nodes is sentinel node biopsy,
eral invasive ductal carcinoma. LCIS is not consid- which is per ormed with methylene blue or 99m c
ered a precursor to invasive adenocarcinoma, but is sul ur colloid, a radiopharmaceutical that is taken
associated with a 20% to 30% risk o ductal adeno- up by lymphatic capillaries, transported to the sen-
carcinoma in either breast. Lobular carcinoma com- tinel node, and then phagocytosed by macrophages.
prises a smaller 10% o all invasive breast cancers. A ormal axillary dissection is generally indicated in
In ammatory breast cancer is yet rarer and with the the case o clinically positive nodes, or completion
worst prognosis o all breast cancer types, compris- axillary dissection in the case o clinically negative
ing 5% o cases, with a characteristic rapid onset o nodes with ailed sentinel mapping. T e currently
pain, visible breast edema, and skin changes (peau evolving realm o neaodjuvant chemotherapy, with
dorange), and a median survival o 36 months. Skin potential or downstaging both the breast mass and
biopsy alone o en reveals unique tumor invasion o nodal metastases, continues to change the landscape
lymphatic channels. o breast cancer surgery.
Radiation is used to decrease the risk o local
4. D. Multiple large randomized studies with long- recurrence, particularly when used as adjuvant to
term ollow-up including the National Surgical breast-conserving surgery. Chemotherapy typically
Adjuvant Breast and Bowel Project (NSABP) tri- consists o anthracycline and taxane agents, and may
als and Milan series showed that disease- ree and be help ul or patients with positive nodal status and
overall survival rates are similar in Stage I and II high risk o relapse, but may generally be excluded in
patients treated with lumpectomy with axillary dis- patients with small tumors and negative lymph nodes.
section ollowed by radiation, and those treated by Hormonal therapy has been known to decrease risk
modi ed radical mastectomy. T is led to breast-con- o recurrence and mortality in patients with estro-
serving surgery with radiation to be a sa e and less gen receptor-positive (ER+ ) tumors. amoxi en,
morbid course o treatment or patients with ame- a selective ER modulator, reduces recurrence and
nable, early-stage breast cancer. T is has been the contralateral disease (side e ects include increased
historical approach to early breast cancer. T e Z11 risk o endometrial cancer, thromboembolism, and
trial showed that patients treated with breast conser- stroke). Adjuvant therapy with tamoxi en has been
vation therapy (lumpectomy + SLN biopsy + radia- shown to lead to signi cant prolongation o disease-
tion therapy) and who had a positive sentinel lymph ree and overall survival in ER+ disease, regardless
node had the same survival as those who underwent o nodal status. Anastrazole is an aromatase inhibi-
completion axillary lymph node dissection or the tor that may be used in patients with contraindica-
positive node. T is urther decreased the morbidity tions to tamoxi en, and only in post-menopausal
o breast cancer surgery. T is makes D the correct women, as they do not produce suf cient estrogen
answer. opposition in pre-menopausal women. rastuzumab
Upon discovering a new diagnosis o breast can- is a monoclonal HER2 antibody that may be used as
cer, the stage must clearly be determined be ore adjuvant therapy in both pre- and post-menopausal
initiating treatment. Surgery is indicated or Stage women who have overexpression o the HER2/neu
I to III disease, while Stage IV (metastatic) disease gene.
is generally managed with medical and/or radia- Stage is the best indicator o prognosis. When
tion therapies. T e purpose o surgery is to obtain limited to the breast only, there is a 99% cure rate,
local control by either lumpectomy or mastectomy, however, 5-year survival decreases to 66%87%
and to evaluate nodal burden. Lumpectomy may be based on Surveillance, Epidemiology, and End
done with or without pre-operative mammographic Results (SEER) data with axillary spread. T e tumor,
needle-localization. ypes o mastectomy include node, metastasis ( NM) model is the most widely
simple (removal o all breast tissue), radical (removal accepted staging method. 0 implies no evidence
o all breast tissue, pectoralis major, pectoralis minor, o a primary tumor and it includes in-situ lesions,
and an axillary node dissection), and modi ed radi- 1 is a mass 2 cm, 2 is a mass rom 25 cm, 3
cal (removal o all breast tissue, pectoralis ascia is a mass > 5 cm, and 4 is a mass with chest wall
only, and an axillary node dissection). I nodes are invasion, skin edema or ulceration, or in ammatory
clinically negative, the pre erred method or stag- breast cancer.

http://surgerybook.net/
194 G EN ERAL S U RG ERY EXAM I N ATIO N AN D BO ARD REVI EW

N0 implies negative lymph nodes, N1 means posi- to pectoralis minor), level II (deep to pectoralis
tive mobile ipsilateral axillary lymph nodes, N2 is xed minor), and level III (medial to pectoralis minor).
or matted ipsilateral axillary nodes, and N3 is ipsilat- Nerves present in the axillary at pad, and to be
eral in raclavicular, supraclavicular, or internal mam- mind ul o during dissection, include the long tho-
mary nodes. T ere is also pathologic N-staging, based racic nerve (innervating the serratus anterior, with
upon number o histologically positive lymph nodes. injury resulting in a winged scapula), the thora-
M0 disease implies no metastatic disease, and M1 codorsal nerve (innervating the latissimus dorsi,
means metastatic disease is present. T e most com- with injury resulting in weak adduction and inter-
mon locations or breast cancer metastasis are bone, nal rotation), the intercostobrachial nerve (provid-
lung, and brain. ing sensation to the upper inner arm, with injury
resulting in parasthesias or numbness), and the
5. C. Injury to the intercostobrachial nerve, the lateral medial and lateral anterior thoracic nerves (inner-
cutaneous branch o the second intercostal nerve, vating the pectoralis muscles, with injury resulting
results in numbness o the medial upper arm. in weakness).
T e anatomic borders o the breast are the second
rib superiorly, sixth to seventh rib in eriorly, sternal
BIBLIOGRAPHY
border medially, and midaxillary line laterally. T e
parenchyma o the breast is comprised o glandular American College o Radiology. Practice guideline or the
breast conservation therapy in the management o inva-
lobes in a radial pattern, each with a ductal system
sive breast carcinoma. J Am Coll Surg. 2007;205(2):36276.
ending at the nipple, and a surrounding connective Bao , Prowell , Stearns V. Chemoprevention o breast can-
tissue ramework made by the suspensory ligaments cer: tamoxi en, raloxi ene, and beyond. Am J T er. 2006;
o Cooper. T e most common distribution o cancer 13(4):33748.
is the upper outer quadrant (45%), ollowed by sub- Bartella L, Smith CS, Dershaw DD, et al. Imaging breast can-
areolar (25%), upper inner (15%), lower outer (10%), cer. Radiol Clin North Am. 2007;45(1):4567.
Chan K, Morris GJ. Chemoprevention o breast cancer or
and lower inner (5%). women at high risk. Semin Oncol. 2006;33(6):6426.
T e blood supply o the breast includes per o- Giuliano AE, Hunt KK, Ballman KV, et al. Axillary dissection
rator branches rom the internal mammary artery, vs no axillary dissection in women with invasive breast
branches rom the posterior intercostal arteries, cancer and sentinel node metastasis: a randomized clinical
and axillary artery branches including the lateral trial. JAMA. 2011;305(6):569575.
King A, Morrow M. Breast Disease. In: Mulholland MW,
thoracic and pectoral branches o the thoracoacro-
Lillemoe KD, Doherty GM, et al. eds. Greenf elds Surgery:
mial artery. Venous drainage parallels the arterial Scientif c Principles and Practice. 5th ed. Philadelphia, PA:
supply. T e axillary lymph nodes drain 75% to 85% Lippincot, Williams & Wilins: 2011:26352701.
o lymph rom the breast, with the remainder drain- Patel RR, Sharma CG, Jordan VC. Optimizing the antihor-
ing to parasternal/internal mammary lymph nodes. monal treatment and prevention o breast cancer. Breast
T ere are three levels o lymph nodes: level I (lateral Cancer. 2007;14(2):11322.

http://surgerybook.net/
55
Ductal Carcinoma in Situ

Angela Penn

A 42-year-old woman presents with extensive new cal- A. SLNB does not need to be per ormed i this
ci cations on screening mammography. She undergoes patient undergoes a mastectomy or her DCIS.
stereotactic core biopsy that reveals ductal carcinoma in B. By de nition, DCIS is noninvasive disease, there-
situ (DCIS), intermediate grade, with comedonecrosis. ore SLNB should never be per ormed.
Estrogen receptor status is positive (ER+). She has no C. SLNB has an almost negligible risk o lymph-
signi cant past medical history and there is no amily edema.
history o cancer. D. SLNB should be considered at the time o
lumpectomy or DCIS located in the axillary tail
1. Which of the following is a risk factor for this patient
o the breast.
actually having occult invasion instead of DCIS?
E. SLNB should only be considered i a patient has
A. + ER receptor status DCIS in the setting o a mass.
B. Palpable mass on clinical exam
C. DCIS that spans a diameter larger than 3 cm
4. Regarding imaging in DCIS, which of the following
D. intermediate grade histology
is true?
E. Contralateral disease
A. Approximately 50% o all mammography
2. Which of the following statements is true? detected breast cancers are DCIS.
A. For patients undergoing breast conservation B. MRI estimates o the size and extent o DCIS
therapy (BC ) or DCIS, the optimal margin o en correlate with pathologic evaluation.
width is 10 mm. C. DCIS o en extends beyond the area o calci ca-
B. Adjuvant radiation does not decrease the risk o tion seen on mammography.
recurrence a er lumpectomy or DCIS. D. Magnetic resonance imaging (MRI) is better
C. Adjuvant radiation increases survival in patients than mammography or distinguishing DCIS
undergoing BC or DCIS. rom benign, atypical proli erative lesions or
D. Risk o death a er any treatment or DCIS is less microinvasion.
than 2% a er ten years, and is usually secondary E. MRI is not help ul in identi ying multicentric
to recurrence as invasive disease. disease and synchronous disease in the contra-
E. Approximately 10% o ipsilateral breast recur- lateral breast.
rences a er lumpectomy alone or in conjunction
with radiation are invasive disease. 5. With regard to adjuvant tamoxifen for DCIS,
3. Regarding the role of sentinel lymph node biopsy which of the following statements is true?
(SLNB) in DCIS, which of the following statements A. amoxi en ( MX) improves both local recur-
is true? rence and survival rates in DCIS.

http://surgerybook.net/
196 G EN ERAL S U RG ERY EXAM I N ATI O N AN D BO ARD REVI EW

B. MX has no signi cant e ect on survival in ume ratio is low and lumpectomy with reasonable
invasive breast cancer. cosmesis cannot be achieved, or in patients in whom
C. Side e ects o MX includes an increased risk o lumpectomy ails to achieve negative margins. Most
ovarian cancer. patients, however, can and should be considered or
D. amoxi en is equally e ective in ER+ lesions as BC .
in ER lesions. Optimal margins or lumpectomy remain contro-
E. In women undergoing breast conservation versial. Studies suggest a decrease in recurrence rates
therapy or DCIS, MX decreases invasive and a er lumpectomy alone with increased margin width
non-invasive breast cancer events or both the up to 10 mm with no additional bene t to margins
ipsilateral and contralateral breast. wider than 10 mm. However, this bene t is eradi-
cated when radiation is added to lumpectomy and
the general consensus is that margin width greater
ANSWERS than or equal to 2 mm is adequate with some advo-
1. B. Because o sampling error, a ter surgical exci- cating absence o cancer at the inked margin as the
sion o a lesion diagnosed on core biopsy, DCIS de nition o a negative margin.
is upgraded to invasive cancer in approximately Given the variable behavior o DCIS, there have
10% to 20%. Risk actors or occult invasive dis- been extensive e orts to identi y a subgroup o
ease include presence o a mass, extent o DCIS patients with indolent disease or whom radiation
greater than 5 cm, comedonecrosis, or high grade can be sa ely eliminated a er lumpectomy. Such
histology. e orts have thus ar resulted in con icting data.
DCIS represents a heterogeneous spectrum o T ere ore, radiotherapy remains standard o care or
disease involving the abnormal proli eration o patient with DCIS who undergo BC . T e prospec-
epithelial cells con ned to the breast ducts. Unlike tive National Surgical Adjuvant Breast and Bowel
invasive cancer, DCIS is bounded by the basement project (NSABP) B-06 trial demonstrated signi cant
membranes and does not invade beyond its ductal decreases in recurrence rate with adjuvant radiation
origin or into the neighboring tissues. Although in BC or invasive disease. T ere were some women
studies o the natural progression o DCIS are lack- in this study who were initially thought to have inva-
ing, the general consensus is that in situ disease sive disease. However, upon review o their biopsy
represents an intermediary between normal breast specimens, they were determined to have DCIS
tissue and invasive disease. However, its behavior instead.
is highly variable, ranging rom indolent low-grade T ese women were ound to have a signi cant
lesions that may never progress to invasion, to high- reduction in recurrence rates, including recur-
grade lesions that may harbor oci o invasive cancer. rence as invasive disease. T e NSABP B-17 study
T is non-obligate progression and variation in bio- was subsequently designed to evaluate radiother-
logic behavior has raised dilemmas in clinical man- apy speci cally in patients with DCIS. T is study
agement with some patients being over treated or ound a signi cant reduction in recurrence rates
the disease. with adjuvant radiation or noninvasive disease.
In neither invasive nor in situ disease was there a
2. D. T e mortality or patients with DCIS a er demonstrated survival bene t to adjuvant radio-
excision is low (less than 2% in 10 years based on therapy. T e European Organisation or Research
population studies) but is generally secondary to and reatment o Cancer (EOR C) 10753 trial was
recurrence as invasive disease. Almost hal the recur- a large European study that corroborated similar
rences a er treatment or DCIS occur in the orm o ndings.
invasive disease. T ere ore, the undamental goals
o management are to prevent recurrence and to 3. D. Although by de nition DCIS is non-invasive
minimize treatment related morbidity. DCIS can be disease, given the signi cant chance o upgrading
treated by either mastectomy or breast conservation to invasive disease a er surgical excision, sentinel
therapy (BC ). Mastectomy should be considered lymph node biopsy to evaluate or possible nodal
in patients with extensive or multicentric disease, in metastases has been advocated in certain scenar-
patients in whom the breast volume to tumor vol- ios. Although it is reasonable to consider SLNB in

http://surgerybook.net/
C H AP TER 5 5 D U C TAL C ARC I N O MA IN S I TU 197

patients who have risk actors or occult invasive may be help ul in identi ying multicentric or syn-
disease to avoid a second operation should they be chronous disease in the contralateral breast. How-
upgraded to invasive disease a er excision, many ever, it appears to be no better than mammography
patients would undergo unnecessary axillary sam- or distinguishing DCIS rom benign proli erative
pling. SLNB is not entirely benign and there is about lesions and MRI estimates o lesion size and extent
a 6% risk o lymphedema. Since SLNB can be per- correlate only moderately well with pathological
ormed subsequent to a lumpectomy, SLNB is not ndings, both over and underestimating the size o
advocated at the time o initial excision or most the DCIS.
patients. T e general consensus, however, is to per-
orm a SLNB i a patient is to undergo mastectomy 5. E. Given the role o estrogen in the pathogen-
or DCIS. In this situation, removal o the breast esis o breast cancer, strategies targeting the estro-
precludes subsequent SLNB should the nal pathol- gen pathway have been investigated as possible
ogy reveal invasive disease. T ese patients would adjuncts to breast cancer therapy. amoxi en is a
then require axillary lymph node dissection. For selective estrogen receptor modulator (SERM).
the same reason, patients whose DCIS is located in While it is an estrogen receptor agonist at some
the axillary tail o the breast should be considered sites such as the bone and endometrium, it has
or SLNB at the time o initial excision as lymphatic potent antagonist e ects in breast tissue. Stud-
drainage to the axilla may be disrupted during exci- ies had demonstrated the bene t o tamoxi en as
sion and may decrease the likelihood o success ul adjuvant therapy in invasive disease, including
localization o a SLN subsequently. Most patients, decreased local recurrence and breast cancer mor-
however, can and should undergo SLNB subsequent tality rates. T e NSABP B-24 study was designed
to a nal pathological determination o invasive to investigate tamoxi en in the setting o DCIS.
disease. In this study, patients undergoing lumpectomy
and radiation or DCIS were subsequently rand-
4. C. Most cases o DCIS are discovered upon omized to receive tamoxi en or placebo. amoxi en
screening mammography and DCIS now accounts was administered in the treatment group at 10 mg
or approximately 20% o all screening detected twice a day or 5 years. T e median ollowup was
breast cancers. DCIS most o ten presents mam- 74 months and patients in the tamoxi en arm had
mographically as microcalci ications, sometimes 37% ewer breast cancer events overall and there
with branching corresponding to intraductal was both an ipsilateral and contralateral breast
disease. bene t. T ere was no bene t in terms o overall
Mammography is limited in that DCIS o en survival. Adverse e ects o tamoxi en include an
extends beyond the area o microcalci cations seen increased incidence o thromboembolic events and
on imaging. T us, up to (15% to 20%) o women endometrial cancer. A subsequent subset analysis
undergoing lumpectomy or DCIS may be subjected demonstrated that the bene t o tamoxi en was
to re-excision to obtain negative margins. In addi- limited to those with ER-positive disease.
tion, up to 40% o DCIS lesions grow discontinu- DCIS represents a very heterogenous group o
ously which can make complete excision dif cult. lesions that have a variable clinical course lend-
T is is a postulated reason as to why adjuvant breast ing challenges to the management o this disease.
irradiation is e ective in decreasing recurrence rates Almost all patients with DCIS will be cured o the
a er excision. disease and adjuvant radiation and endocrine ther-
Given the limitations o mammography, MRI has apy have urther improved recurrence rates. With
been studied as an alternate imaging modality. MRI urther understanding o the biology o the subtypes
tends to have high sensitivity but low speci city in o DCIS, we will be able to better tailor therapy to
breast cancer imaging. Given its high sensitivity, it individual patients.

http://surgerybook.net/
198 G EN ERAL S U RG ERY EXAM I N ATIO N AN D BO ARD REVI EW

DCIS LCIS
Non-obligate precursor to invasive cancer. Risk actor or invasive cancer, both lobular and ductal. Risk is
elevated or both the ipsilateral and contralateral breast. Consider
hormone modulators to decrease risk.
O en diagnosed with mammography as O en mammographically occult and ound incidentally on biopsy or
microcalci cations. other reasons. Consider MRI.
Usually unilateral but multi ocality is common. O en bilateral and multi ocal.
Surgical treatment is with lumpectomy or Surgical excision is indicated i ound on core needle biopsy sec-
mastectomy. ondary to risk o sampling error (DCIS or invasive cancer nearby is
requently ound). I ound on surgical excision, no urther excision
necessary since it is not treated as a precursor lesion.
Adjuvant radiotherapy is indicated i treated No adjuvant radiotherapy.
with lumpectomy.

BIBLIOGRAPHY Fisher B, Constantino JP, Wickerham DL, et al. amoxi en


Allred DC, Anderson SJ, Paik S, et al. Adjuvant tamoxi en or the prevention o breast cancer: Current status o the
reduces subsequent breast cancer in women with estrogen National Surgical Adjuvant Breast and Bowel Project P-1
receptor-positive ductal carcinoma in situ: a study based Study. J Natl Cancer Inst. 2005;97:165262.
in NSABP Protocol B-24. J Clin Oncol. 2012;30:126873. Huo L, Sneige N, Hunt KK, et al. Predictors o invasion in
Bijker N, Meijnen P, Peterse JL, et al. Breast-conserving treat- patients with core-needle biopsy-diagnosed ductal car-
ment with or without radiotherapy in ductal carcinoma cinoma in situ and recommendations or a selective
in situ: en-year results o European Organisation or approach to sentinel lymph node biopsy in ductal carci-
Research and reatment o Cancer randomized phase III noma in situ. Cancer. 2006;107:17608.
trial 10853 a study by the EOR C Breast Cancer Coop- Hwang ES, Kinkel K, Esserman LJ, et al. Magnetic resonance
erative Group and EOR C Radiotherapy Group. J Clin imaging in patients diagnosed with ductal carcinoma in
Oncol. 2006;24:33817. situ: value in the diagnosis o residual disease, occult inva-
Burak WE Jr, Owens KE, ighe MB, et al. Vacuum-assisted sion, and multicentricity. Ann Surg Oncol. 2003;10:3818.
stereotactic breast biopsy: histologic underestimation o Kau mann M, Morrow M, von Minckwitz G, et al. Locore-
malignant lesions. Arch Surg. 2000;135:7003. gional treatment o primary breast cancer: consensus rec-
Dunne C, Burke JP, Morrow M, et al. E ect o margin status ommendations rom an international expert panel. Cancer.
on local recurrence a er breast conservation and radia- 2010;116:118491.
tion or ductal carcinoma in situ. J Clin Oncol. 2009;27: Kerlikowske K, Molinaro A, Cha I, et al. Characteristics asso-
161520. ciated with recurrence among women with ductal carci-
Ernster VL, Barclay J, Kerlikowske K, et al. Mortality among noma in situ treated by lumpectomy. J Natl Cancer Inst.
women with ductal carcinoma in situ o the breast in the 2003;95:16921702.
population-based Suveillance, Epidemiology and End King A, Farr GH Jr, Cederbom GJ, et al. A mass on breast
Results program. Arch Intern Med. 2000;160:9538. imaging predicts coexisting invasive carcinoma in patients
Fisher B, Redmond C, Poisson R, et al. Eight-year results o a with a core biopsy diagnosis o ductal carcinoma in situ.
randomized clinical trial comparing total mastectomy and Am Surg. 2001;67:90712.
lumpectomy with or without irradiation in the treatment MacDonald HR, Silverstein MJ, Mabry H, et al. Local
o breast cancer. N Engl J Med. 1989;320:8228. control in ductal carcinoma in situ treated by excision alone:
Fisher B, Dignam J, Wolmark N, et al. Lumpectomy and radia- Incremental bene t o larger margins. Am J Surg. 2005;
tion therapy or the treatment o intraductal breast can- 190:5215.
cer: Findings rom National Surgical Adjuvant Breast and McLaughlin SA, Wright MJ, Morris K , et al. Prevalence o
Bowel Project B-17. J Clin Oncol. 1998;16:44152. lymphedema in women with breast cancer 5 years a er
Fisher B, Constantino JP, Remond C, et al. A random- sentinel lymph node biopsy or axillary dissection: objec-
ized clinical trial evaluating tamoxi en in the treat- tive measurements. J Clin Oncol. 2008;26:52139.
ment o patients with node-negative breast cancer who Silverstein MJ, Lagios M, Groshen S, et al. T e in uence o
have estrogen-receptor-positive tumors. N Engl J Med. margin width on local control o ductal carcinoma in situ
1989;320:47984. o the breast. N Engl J Med. 1999;340:145561.
Fisher B, Dignam J, Wolmark N, et al. amoxi en in treatment Vanderwalde LH, Dang CM, Bresee C, et al. Discordance
o intraductal breast cancer: National Surgical Adjuvant between pathologic and radiologic tumor size on breast
Breast and Bowel Project B-24 randomized controlled MRI may contribute to increased re-excision rates. Am
trial. Lancet. 1999;353:19932000. Surg. 2011;77:13613.

http://surgerybook.net/
56
In ammatory Breast Cancer

Ranjna Sharma

A 42-year-old emale presents to your o ce with C. Modi ed radical mastectomy


an erythematous and edematous le breast that she D. Radical mastectomy
reports as occurring acutely over the past 2 weeks. She
is not breast- eeding. She denies any recent trauma or 2. What impact has the addition o neoadjuvant
source o in ection. She presented to her primary care chemotherapy had on the outcome o patients with
physician last week who administered a trial o antibi- inf ammatory breast cancer?
otics or presumed cellulitis, without any improvement A. It decreases the risk o developing a contralateral
in symptoms. Upon your physical exam, you note an breast cancer in the uture.
erythematous, edematous, le breast with clinical nd- B. It decreases the risk o developing a concurrent
ings o peau dorange and a mass underlying the area o ovarian cancer.
clinical change. T e le breast appears larger than the C. It increases e ectiveness o endocrine therapy.
right breast. You are able to appreciate ullness in the D. It improves survival.
le axilla. You order dedicated breast imaging. She has
a diagnostic mammogram per ormed that shows skin 3. What is the role o radiation therapy in this clinical
thickening and trabecular distortion in the le breast, scenario?
with increased tissue density in the retroareolar region. A. Decreased incidence o local recurrence.
An ultrasound o the le breast showed a 6 cm vague B. It improves response to endocrine therapy.
irregular mass in the retroareolar region, with two C. It improves response to chemotherapy.
enlarged le axillary lymph nodes. A skin punch biopsy is D. It prevents distant metastases.
per ormed in your o ce and it demonstrates cancer cells
in ltrating the dermal lymphatics. A core needle biopsy 4. What is the usual order o treatments delivered?
o the mass reveals invasive ductal carcinoma. A ne A. Surgery, radiation therapy, chemotherapy
needle aspiration o an enlarged le axillary lymph node B. Radiation therapy, surgery, chemotherapy
shows metastases. Based on the clinical presentation, C. Chemotherapy, surgery, radiation therapy
imaging ndings, and histopathologic results, a diagnosis D. Radiation therapy, chemotherapy, surgery
o in ammatory breast cancer (IBC) is made. A staging
work-up is negative or any distant sites o disease. You 5. What is the underlying pathophysiology in
discuss the treatment modalities o surgery, chemotherapy, inf ammatory breast cancer contributing to the
and radiation therapy as part o her treatment plan. skin changes that are noted on clinical exam?
A. In ection (mastitis/abscess)
1. What surgical procedure will this patient have? B. umor emboli obstructing the dermal lymphatic
A. Mastectomy vessels
B. Mastectomy with sentinel lymph node biopsy C. Localized dermatitis

http://surgerybook.net/
200 G EN ERAL S U RG ERY EXAM IN ATI O N AN D BO ARD REVIEW

D. Congestive heart ailure causing skin edema chemotherapy and shows improvement in survival
E. Concurrent non-Hodgkin Lymphoma and prognosis. I Her2/neu is overexpressed, then
trastuzumab is given or 1 year, which contributes
to increased pathologic complete response (pCR).
ANSWERS T e skin changes o erythema and edema in IBC
1. C. T is patient will have a modi ed radical mas- will generally improve when a patient is respond-
tectomy (MRM), which is comprised o a simple ing to neoadjuvant chemotherapy. I a pCR can be
mastectomy and axillary lymph node dissection. achieved, survival is increased. T e improvement
T e borders o the mastectomy portion o the proce- in survival is seen in both disease- ree survival and
dure are superiorly to the clavicle and deltopectoral overall survival.
groove, laterally to the latissimus muscle, in eriorly
3. A. Radiation therapy is administered a er MRM,
to the upper edge o rectus sheath, and medially to
thus it is post-mastectomy radiation therapy (PMR ).
the sternal border. T e posterior border is the ascia
PMR is given to improve local control o the dis-
overlying the pectoralis major muscle. T e mastec-
ease process and decrease the risk o local recurrence
tomy should resect any residual gross disease and
seen in patients with IBC due to high local disease
obtain negative surgical margins. T e surgical plan
burden at the time o diagnosis. Administration o
must remove all skin changes. T e mastectomy skin
PMR is known to decrease the risk o recurrence on
aps must be closed without tension. T e borders o
the chest wall, mastectomy scar, and regional lymph
an axillary lymph node dissection are the axillary
node basins. It is delivered to the chest wall and
vein superiorly, the pectoralis major muscle anteri-
regional lymph node basins in the axilla, in raclavic-
orly, the subscapularis muscle posteriorly, the serra-
ular, supraclavicular, and internal mammary regions
tus anterior muscle medially, and the latissimus dorsi
in standard ractionation, with a boost to the chest
muscle laterally. Within these anatomic boundaries,
wall scar. Any area with pretreatment skin involve-
dissection will occur which will excise the Level 1
ment should receive radiation to decrease the risk o
and Level 2 axillary lymph nodes.
local recurrence.
In eriorly, the dissection should be carried out to
the 4th or 5th rib. T e Level 1 nodes are located lat- 4. C. IBC is treated with a multidisciplinary/multi-
eral to the pectoralis minor muscle and the Level 2 modality approach, combining local and systemic
nodes are located posterior to this muscle. A modi- treatments. Local treatment modalities are surgery
ed radical mastectomy or a patient with IBC with and radiation therapy, whereas chemotherapy and
metastases to the axillary lymph nodes will remove endocrine therapy are utilized as systemic therapies.
the a ected skin, generalized disease process in the However, in regards to endocrine therapy, most IBCs
breast, and involved lymph nodes. T is will contrib- are ER/PR negative, so there would be no role or
ute to improved local control o the disease process. endocrine therapy in those patients. T is combina-
A sentinel lymph node biopsy is not indicated since tion and order o neoadjuvant chemotherapy, MRM,
there is already known disease in the axillary nodes. and PMR will improve local disease control, thus
Consequently, to control regional disease, a complete decreasing risk o local recurrence. MRM is most
axillary lymph node dissection must be done. e ective i a patient has a good clinical response
(decrease in skin erythema and edema) to neoadju-
2. D. T e introduction o neoadjuvant chemother- vant chemotherapy, and particularly so in patients
apy has signi cantly improved clinical outcomes in who have a pathologic complete response (pCR)
patients with IBC. It is used to downstage the tumor to neoadjuvant chemotherapy. T is multimodality
to allow surgical resection to be more success ul by treatment plan has improved survival, particularly
decreasing the likelihood o leaving residual dis- disease- ree survival.
ease behind. Anthracycline-based regimens show a
survival bene t. A regimen containing cyclophos- 5. B. In IBC, tumor emboli invade the dermal lym-
phamide, 5- uorouracil, and either doxorubicin or phatic vessels, causing an obstruction, which leads
epirubicin is generally used. axanes are also admin- to the edema, induration, and peau dorange appear-
istered. T e combination o anthracycline and taxane ance. T e emboli invade vessels in the papillary and
regimens increases the rate o clinical response to reticular dermis.

http://surgerybook.net/
C H AP TER 5 6 I N f LAM M ATO RY BREAS T C AN C ER 2 01

BIBLIOGRAPHY Breast Surgical Oncology. New York, NY: McGraw-Hill;


Dawood S, Merajver SD, Viens P, et al. International expert 2010:92736.
panel on in ammatory breast cancer: consensus statement Neuman HB and Van Zee KJ. Axillary Lymph Node Dissec-
or standardized diagnosis and treatment. Annals of Oncol- tion. In: Kuerer HM, ed. Kuerers Breast Surgical Oncology.
ogy. 2011;22:51523. New York, NY: McGraw-Hill; 2010:67991.
Duskin H, Cristo anilli M. In ammatory Breast Cancer. J Natl Rao R and Leitch AM. Modi ed Radical Mastectomy and
Compr Canc Netw. 2011;9:23341. echniques or Avoiding Skin Necrosis. In: Kuerer HM, ed.
Khan AJ, Haf y BG. Postmastectomy Radiation T erapy. In: Kuerers Breast Surgical Oncology. New York, NY: McGraw-
Kuerer HM, ed. Kuerers Breast Surgical Oncology. New Hill; 2010:69397.
York, NY: McGraw-Hill; 2010:9951008. Recht A, Edge SB, Solin LJ, et al. Postmastectomy radiother-
Li BD, Sicard MA, Ampil F, et al. rimodal therapy or In am- apy: Guidelines o the American society o clinical oncol-
matory Breast Cancer: A Surgeons Perspective. Oncology. ogy. J Clin Oncol. 2001;19(5):153969.
2010;79:312. Rehman S, Reddy CA, endulkar RD. Modern outcomes o
Makower D and Sparano JA. How Do I reat In ammatory in ammatory breast cancer. Int J Radiation Oncol Biol
Breast Cancer? Current Treatment Options in Oncology. Phys. 2012;84(3):61924.
2013;14:6674. Robertson FM, Bony M, Yang, W, et al. In ammatory breast
National Comprehensive Cancer Network Guidelines Version cancer: T e disease, the biology, the treatment. CA Cancer
2.2011 In ammatory Breast Cancer, IBC-1. NCCN breast J Clin. 2010;60:35175.
cancer clinical practice guidelines in oncology. Available Saigal K, Hurley J, akita C, et al. Risk actors or locoregional
at: http://www.nccn.org/pro essionals/physician_gls/PDF/ ailure in patients with in ammatory breast cancer treated
breast.pd . Accessed June 1,2014. with trimodality therapy. Clinical Breast Cancer. 2013;
Negron Gonzalez V, Oh JL, Cristo anilli M, Babiera GV. 13(5):33543.
In ammatory Breast Cancer. In: Kuerer HM, ed. Kuerers Zellars R. Post-mastectomy radiotherapy. Clin Adv Hematol
Oncol. 2009;7(8):53343.

http://surgerybook.net/
57
Breast Reconstruction

Pamela C. Masella and Mark K. Markarian

A 42-year-old otherwise healthy emale recently under- 3. T e patient inquires about a nipple-sparing
went core-needle biopsy o a palpable 4 cm le breast mastectomy. Which of the following is a
mass in the upper outer quadrant that revealed inva- contraindication to this procedure?
sive ductal carcinoma. On physical exam she had no A. 3 cm tumor
urther palpable lesions and no palpable lymph nodes. B. Pagets disease
Metastatic work-up was negative. She would like to dis- C. A large tumor-to-areola distance
cuss mastectomy and reconstruction options prior to D. Positive axillary lymph nodes
her surgery. E. Multi ocal disease

1. Regarding breast reconstruction options, which of 4. T e patient undergoes skin-sparing mastectomy


the following is correct? and sentinel lymph node biopsy with tissue
A. Autologous tissue reconstruction is pre erred expander placement at the time of surgery.
whenever possible over implant reconstruction. Which of the following pathologic criteria
B. I implant reconstruction is desired, tissue would preclude her from completing implant
expanders will be placed at the time o mas- reconstruction?
tectomy and nal implants will be placed 4 to A. A negative sentinel lymph node biopsy
6 weeks later. B. 3 cm primary tumor
C. Reconstruction does not signi cantly inter ere C. 10 mm margins
with the detection o recurrent disease. D. Stage IIA
D. Smoking is an absolute contraindication or E. Stage IB
autologous tissue reconstruction.
E. Autologous and implant reconstruction can sig- 5. Intraoperatively, the patients sentinel lymph
ni cantly delay subsequent adjuvant therapy. node biopsy is positive for invasive ductal
carcinoma, she undergoes axillary dissection
2. Which of the following is an advantage of and her final pathology returns p 2 pN1
autologous tissue reconstruction? MO, Stage IIB invasive ductal carcinoma. She
A. Better symmetry with the contralateral breast undergoes adjuvant chemotherapy and radiation
B. Shorter operating time therapy to the chest wall, infraclavicular,
C. Shorter inpatient hospitalization supraclavicular, and inframammary areas.
D. Lower potential or the need o blood trans usion She may not be an appropriate candidate
when compared with implant reconstruction for which of the following autologous tissue
E. No revisions needed reconstructions?

http://surgerybook.net/
C H AP TER 5 7 BREAs T RECo n s TRu C Ti o n 203

De e p Infe rior Epiga s tric Pe rfora tor fla p


Fla p conne cte d to
re cipie nt blood ve s s e ls

Fla p with de e p infe rior


e piga s tric a rte ry & ve in
Ma s te ctomy
de fe ct

Re ctus a bdominis
mus cle

De e p infe rior
e piga s tric Incis ion
S ca r
a rte ry a nd ve in
Be fore Afte r
Alila Me dica l Me dia - www.Alila Me d ica lMe d ia .c om

Fre e TRAM Fla p S urge ry


Fla p conne cte d to
re cipie nt blood ve s s e ls

Fla p with
Ma s te ctomy blood ve s s e ls
de fe ct

Re ctus a bdominis
mus cle
Blood ve s s e ls Me s h

S ca r

Be fore Afte r
Alila Me dica l Me dia - www.Alila Me d ica lMe d ia .c om

La tis s imus Dors i Fla p

Alila Me dica l Me dia - www.Alila Me d ica lMe d ia .c om

http://surgerybook.net/
204 G En ERAL s u RG ERY EXAM i n ATi o n An D Bo ARD REVi EW

S upe rior Infe rior Epiga s tric Arte ry Fla p


Fla p conne cte d to
re cipie nt blood ve s s e ls

Ma s te ctomy
de fe ct

Re ctus a bdominis
mus cle
De e p infe rior
e piga s tric
a rte ry a nd ve in
Fla p with s upe rficia l S ca r
S upe rficia l infe rior infe rior e piga s tric
e piga s tric a rte ry a nd ve in a rte ry a nd ve in
Be fore Afte r
Alila Me dica l Me dia - www.Alila Me d ica lMe d ia .c om

Tra nsve rs e Re ctus Abdominis Myocuta ne ous (TRAM) Fla p S urge ry

Ma s te ctomy Fla p configure d to


de fe ct form bre a s t mound

Re ctus a bdominis
mus cle
Me s h
Blood ve s s e ls

S ca r

Be fore TRAM fla p pa s s e d Afte r


unde r s kin to che s t
Alila Me dica l Me dia - www.Alila Me d ic a lMe d ia .c om

A. Pedicled transverse rectus abdominis myocuta- whether breast reconstruction delays time to adju-
neous ( RAM) ap vant therapy, primarily surrounding autologous
B. Free transverse rectus abdominis myocutaneous reconstruction due to increase in post-operative
( RAM) ap wound complications, however, this does not appear
C. Latissimus dorsi ap to be signi cant enough to af ect cancer-speci c sur-
D. Deep in erior epigastric per orator (DIEP) ap vival.
E. Super cial in erior epigastric artery (SIEA) ap Additionally, immediate breast reconstruction o
any variety is associated with decreased breast can-
cer-speci c mortality, particularly among younger
ANSWERS
women. Patient pre erence, and li estyle, the avail-
1. C. Autologous or synthetic breast reconstruction ability o autologous tissue, and plans or adjuvant
has not been shown to decrease the ability to detect cancer therapies are variables that can in uence the
local or locoregional breast cancer recurrence. Mul- timing and choice o operative technique. While
tiple studies have shown that reconstruction a er heavy smoking is not an absolute contraindication
mastectomy with a variety o methods does not or autologous tissue reconstruction, it does in u-
adversely af ect the incidence or time to detection ence/limit the type o autologous tissue options that
o recurrent breast cancer. T ere is debate about are available.

http://surgerybook.net/
C H AP TER 5 7 BREAs T RECo n s TRu C Ti o n 205

A er placement o tissue expanders, rst expan- thetic reconstruction because o decreased local
sion is typically approximately 14 days rom the ini- vascularity, the increased incidence o capsular con-
tial surgery. issues may be expanded up to 25% more tracture, and the increased risk o surgical site in ec-
than the desired volume over a variable amount o tion. T e indications or radiation therapy in breast
time, a er which the breast tissue is allowed to rest, cancer according to the 2014 NCCN guidelines on
on average, or 2 to 4 weeks, be ore tissue expand- invasive breast cancer include those patients under-
ers are exchanged with permanent breast implants. going breast conservation therapy, tumors > 5 cm,
T e overall time to complete the expansion process those with positive axillary lymph nodes, or positive
is typically 3 to 6 months depending on the desired (< 1 mm) surgical margins. According to the AJCC
breast size, the thickness o the mastectomy aps, and NM classi cation, a tumor > 5 cm is considered a
the patients ability to tolerate the expansion. 3 lesion while any lymph node involvement is at
least N1. Stage IIA ( 2 N0 M0) disease does not indi-
2. A. Autologous tissue reconstruction involves the cate the need or radiation therapy. Stage IB includes
trans er o vascularized muscle, skin, and subcuta- 0-1, N1 M0 lesions and there ore may require radi-
neous tissue to the mastectomy de ect. issue can ation therapy.
be trans erred on a vascular pedicle or as a ree ap
requiring a microsurgical anastomosis. T is method
5. C. Because the patient underwent axillary dissection
typically allows or a more natural appearing breast
and locoregional radiation therapy to the chest wall
contour, with a so er texture than implant recon-
and axilla, vessels in the area may have been damaged
struction and o entimes eliminates the need or
during treatment. T e latissimus dorsi myocutane-
contralateral symmetry procedures. Autologous
ous ap receives its primary blood supply rom the
reconstruction is also associated with ewer revi-
thoracodorsal vessels, which may be compromised
sion surgeries. Disadvantages include longer opera-
during an axillary dissection or locoregional radia-
tive times, prolonged inpatient recovery time, higher
tion. T e primary blood supply to the RAM ap
need or blood trans usion, morbidity associated
are the superior epigastric vessels, while the primary
with the donor site and in the case o ree aps*, the
blood supply to the DIEP ap is rom deep in erior
increased potential or total ap loss. (*Note: total
epigastric per orators, and the super cial in erior
f ap loss may occur with any autologous reconstruc-
epigastric artery provides the primary blood supply
tionpedicled or ree.)
to the SIEA ap.
3. B. T e literature suggests that the nipple-sparing
mastectomy is oncologically sa e with a clearly BIBLIOGRAPHY
de ned set o pathologic parameters. Contraindica- Bezuhly M, emple C, Sigurdson LJ, et al. Immediate post
tions to nipple-sparing mastectomy include tumors mastectomy reconstruction is associated with improved
4.5 cm or greater, tumors located less than 2.5 cm breast cancer-speci c survival: Evidence and new chal-
rom the areolar edge or less than 4 cm rom the nip- lenges rom the Surveillance, Epidemiology, and End
Results database. Cancer. 2009;115(20):464854.
ple center, gross involvement o the nipple-areola Craigie JE, Allen RJ, Dellacroce FJ, et al. Autogenous breast
complex including bloody nipple discharge or Paget reconstruction with the deep in erior epigastric per orator
disease. umors that are multicentric, multi ocal, or ap. Clin Plast Surg. 2003;30:35969.
contain extensive ductal carcinoma in situ (DCIS) Gucwa A, Harper JG, Lind DS. Section 3: Breast, Skin, and
remain eligible or nipple-sparing mastectomy. Addi- So issue. Chapter 5: Breast Procedures. In: Ashley SW.
ACS Surgery [Online]. Philadelphia, PA: Decker Intellectual
tionally, women who have undergone neoadjuvant
Properties; 2009.
therapy and subsequently meet the criteria or tumor Kronowitz SJ, Kuerer HM. Advances and surgical decision-
size and location can also be considered, but in am- making or breast reconstruction. Cancer. 2006;107:893
matory breast cancer is absolutely excluded. T e 907.
presence o clinically positive axillary lymph nodes Kwon DS, Kelly CM, Ching CD. Invasive Breast Cancer. In:
is not a contraindication. Feig BW, Ching CD. MD Anderson Surgical Oncology
Handbook [Kindle Edition]. 5th ed. Philadelphia, PA: Lip-
pincott Williams & Wilkins; 2012.
4. E. In general, women who have or will undergo Namnoum JD. Breast Reconstruction: RAM Flap ech-
radiation therapy are not ideal candidates or pros- niques. In: T ome, Charles H. Grabb and Smiths Plastic

http://surgerybook.net/
206 G En ERAL s u RG ERY EXAM i n ATi o n An D Bo ARD REVi EW

Surgery. 6th ed. Philadelphia, PA: Lippincott Williams & Reddy S, Colakoglu S, Curtis MS, et al. Breast cancer recur-
Wilkins; 2007:6417. rence ollowing post mastectomy reconstruction com-
Nahabedian MY. Breast Reconstruction a er mastectomy: pared to mastectomy with no reconstruction. Ann Plast
Indications, technique, and results. In: Cameron JL, Cam- Surg. 2011;66(5):46671.
eron AM. Current Surgical T erapy [Online]: Expert Con- Rozen WM, Ashton MW. Improving outcomes in autolo-
sult. 10th ed. Philadelphia, PA: Elsevier Health Sciences; gous breast reconstruction. Aesthetic Plast Surg. 2008;33:
2011. 32735.

http://surgerybook.net/
Endo c r ine Sur ger y/Hea d a nd
Nec k Tumo r s
Robert B. Lim and Richard Smith

http://surgerybook.net/
This page intentionally left blank

http://surgerybook.net/
58
Adrenal Gland umors

Yong Choi

A 35-year-old emale presents with a 3 cm mass in B. Monoamine oxidase inhibitors (MAOIs) can
the le adrenal gland that was incidentally ound alsely elevate plasma ree metanephrine and
on a computed tomography (C ) scan done or di - normetanephrine levels.
use abdominal pain. T e patient reports intermittent C. Pheochromocytoma on a C scan is repre-
increases in blood pressure but she does not take any sented by venous phase enhancement exclu-
anti-hypertensive medications. She has no other com- sively.
plaints. She has no signi cant past medical history and D. Pheochromocytomas represent the majority o
her only past surgery is a laparoscopic appendectomy incidentalomas ound on C scan.
15 years ago. She does have a sister who had a total
thyroidectomy or medullary carcinoma o the thyroid 3. In relation to perioperative considerations
gland. She is re erred to the general surgery clinic by in patients with adrenal tumors, which of the
her primary care physician or evaluation o this le following is true?
adrenal gland. A. In patients with pheochromocytoma, beta block-
ade should be initiated prior to alpha-adrenergic
1. Regarding the workup of this adrenal mass, which blockade.
of the following is correct? B. umor manipulation may stimulate sudden cat-
A. Diagnosis o Cushings syndrome is suggested echolamine release during the surgery.
by suppression o plasma cortisol levels a er an C. Patients with an aldosterone secreting adrenal
overnight low-dose dexamethasone test. tumor will most likely have hyperkalemia.
B. Patients with primary cortisol producing adeno- D. Patients with cortisol producing tumor should not
mas have elevated cortisol levels and low plasma receive heparin due to their hypocoagulability.
adrenocorticotropic hormone (AC H).
C. T e risk o adrenal carcinoma is increased due to 4. Which of the following is correct regarding the
the size o this adrenal mass. natural history/treatment of the adrenal mass in
D. Percutaneous biopsy o this mass is critical in this patient?
making the diagnosis. A. Due to the size o the mass (3 cm), the patient
should be taken or an adrenalectomy even i the
2. Due to the history of intermittent hypertension, mass is non- unctional.
the patient is evaluated for a pheochromocytoma. B. Since the patient has a amily history o med-
Which of the following is correct? ullary carcinoma o the thyroid, she should be
A. Plasma- ree metanephrine levels but not taken or an adrenalectomy.
normetanephrine levels are sensitive markers or C. Metastatic cancer presenting as a true adrenal
pheochromocytoma. incidentaloma is common.

http://surgerybook.net/
210 G EN ERAL S U RG ERY EXAM IN ATIO N AN D BO ARD REVI EW

D. T is patient can be ollowed with serial C scans 2. B. Pheochromocytoma evaluation is necessary in all
since malignancy at 3 cm is uncommon. patients with an adrenal mass, even i they have no
signs or symptoms o tachycardia, severe hyperten-
5. Which of the following statements on operative sion, cardiac palpitations, arrhythmias, anxiety, and
treatment/technique is correct? sweating. Plasma- ree metanephrine and norme-
A. T e right adrenal vein is identi ed as it enters tanephrine levels are sensitive markers or pheochro-
directly into the vena cava. mocytoma and are easier to obtain than a 24-hour
B. Laparoscopic adrenalectomy should not be per- urine collection. T e plasma- ree metanephrine and
ormed i the patient has adrenal carcinoma. normetanephrine levels should be 3 to 4 times the
C. For benign disease, laparoscopic adrenalectomy normal in patients with pheochromocytoma.
is clearly superior over the open adrenalectomy. Some medications can alsely elevate plasma- ree
D. Posterior retroperitoneal approach has a higher metanephrine and normetanephrine levels. T ese
risk o injury to the liver and spleen compared to include MAOIs, tricyclic antidepressants, decongest-
the transabdominal approach. ants, amphetamines, and phenoxybenzamine. I pos-
sible, these medications should be discontinued prior
to testing or pheochromocytoma.
ANSWERS
Pheochromocytomas are hypervascular and they
1. B. T e diagnosis o Cushings syndrome can be have intense enhancement on the arterial phase o the
accomplished by a dexamethasone suppression test. C scan. T ey also have a slow washout on delayed
Patients with Cushings syndrome ail to suppress imaging. With non-contrast C scan, the density o
plasma cortisol levels a er an overnight low-dose pheochromocytoma is similar to that seen in muscle.
dexamethasone suppression test. In addition, these T ey also represent a small percentage o all inciden-
patients have an elevated 24-hr urine ree cortisol level. talomas ound on C scan, comprising 0% to 11%.
Once diagnosis o Cushings syndrome is con rmed, Non- unctioning adrenal adenomas are the most com-
urther tests can be per ormed to identi y the cause. mon incidentalomas ound on C scan examinations.
Patients with primary cortisol producing adeno-
mas have elevated cortisol levels and a low AC H 3. B. All patients with a diagnosis o pheochromocy-
level. In addition, these patients should have a C toma should undergo alpha-adrenergic blockade. T is
scan or MRI nding o an adrenal mass. Approxi- should be initiated approximately 1 to 2 weeks be ore
mately 20% o patients with Cushings syndrome are surgery. Alpha blockade should help with blood pres-
due to a cortisol producing adenoma. T e signs and sure control and control o intraoperative arrhyth-
symptoms o Cushings syndrome include weight mias. A er initiation o alpha blockade, beta-blockers
gain, hypertension, diabetes mellitus, and centrip- can be used in patients with persistent tachycardia or
etal obesity. Other causes o Cushings syndrome hypertension. Some side e ects o alpha blockers are
are aldosteronomas and adrenocortical carcinomas, atigue, loose stools, dizziness, and somnolence.
although these are rare. During the surgery or pheochromocytoma, care-
T is patients adrenal mass is 3 cm in size. Adrenal ul dissection o the adrenal gland must be per ormed
cortical carcinoma should be suspected with adrenal due to the act that tumor manipulation may cause a
masses greater than 6 cm. T e risk o a 3 cm adrenal sudden release o catecholamines. T is may result in
mass being a carcinoma is very low. a sharp increase in blood pressure and bleeding rom
Percutaneous biopsy o an adrenal mass is not the small vessels. Immediate cessation o manipu-
commonly per ormed. One indication or a percu- lation o the adrenal gland will help normalize the
taneous biopsy is obtaining tissue diagnosis prior blood pressure. Good communication with the anes-
to initiating treatment on a patient who has adrenal thesiologist throughout the procedure is necessary.
metastasis on imaging. Biopsy should not be per- Patients with an aldosterone secreting adrenal
ormed on patients suspected to have pheochro- tumor have hypokalemia. T e aldosterone causes
mocytoma due to the possibility o precipitating a the distal renal tubules to increase sodium and water
hypertensive crisis. As a rule, percutaneous biopsy resorption and thereby increasing the potassium loss,
has a minimal role in the diagnosis o an adrenal resulting in hypokalemia. Hypokalemia is associated
mass. with increased risk o arrhythmias. T ere ore, oral

http://surgerybook.net/
C H AP TER 5 8 AD REN AL G LAN D TU M O RS 211

and intravenous supplementation should be given to invasion, an open approach may be advisable. How-
correct the hypokalemia. ever, small lesions suspicious or carcinoma can be
Patients with cortisol producing tumors are per ormed by the laparoscopic approach.
hypercoagulable due to the hyperhomocysteinemia, As the technique o laparoscopic adrenalectomy
increased clotting actors, impaired brinolysis, and has grown in use, it is now considered a stand-
abnormalities in von-Willebrand actor. T ere ore, ard o care or small benign lesions. However, like
these patients should be considered or pre-operative laparoscopic cholecystectomy, there have not been
heparin injection to decrease the risk o venous randomized controlled trials comparing open to lap-
thromboembolic events. aroscopic adrenalectomy.
T ere are two basic approaches to laparoscopic
4. D. Adrenocortical carcinoma is not common and adrenalectomy: the transabdominal and the posterior
the requency ranges rom 1.2% to 12% o all inciden- retroperitoneal. T e laparoscopic posterior retroperi-
talomas ound on C scan. Carcinoma is rare or an toneal adrenalectomy has certain advantages over the
adrenal mass size o less than 4 cm. Malignant trans- standard transabdominal approach. It eliminates the
ormation should be considered i a mass enlarges by need to retract the liver or the right side and
1 to 2 cm over 1 to 3 years. Since this patient has an the spleen or the le side. It also reduces the risk o
adrenal mass o 3 cm, it can be ollowed by serial C injury to organs such as the liver and spleen. However,
scans i the mass is non- unctional. patient selection is important and it can be technically
With a amily history o medullary carcinoma o more demanding than the transabdominal approach.
the thyroid, multiple endocrine neoplasia (MEN)
syndrome must be considered. However, i the
BIBIOGRAPHY
patient has a non- unctioning adrenal adenoma and
workup or pheochromocytoma is negative, there is Bickenbach KA, Strong VE. Laparoscopic transabdominal lat-
eral adrenalectomy. J of Surg Onco. 2012;106:61118.
no need to per orm an adrenalectomy in a 3 cm adre-
Bittner JG, Brunt LM. Evaluation and management o adrenal
nal mass. incidentaloma. J Surg Onco. 2012;106:55764.
Metastatic cancer presenting as a true adrenal inci- Boland GW, Blake MA, Hahn PF, et al. Incidental adrenal
dentaloma is extremely rare. In a study by Lee et al. lesions: Principles, techniques, and algorithms or imaging
only 0.2% o patients had unknown primary cancer characterization. Radiology. 2008;249:75675.
presenting as an adrenal mass. All o these patients Brunt LM, Moley JF. Adrenal incidentaloma. World J. Surg.
2001;25:90513.
had tumor sizes greater than or equal to 6 cm. Lee JE, Evans DB, Hickey RC, et al. Unknown primary can-
cer presenting as an adrenal mass: Frequency and implica-
5. A. T e anatomy o the right adrenal gland is di er- tions or diagnostic evaluation o adrenal incidentalomas.
ent than the le in that the adrenal vein enters directly Surgery. 1998;124:111522.
into the in erior vena cava (IVC). T ere ore, during the Lee J, El- amer M, Schif ner , et al. Open and laparoscopic
adrenalectomy: Analysis o the National Surgical Qual-
operative approach, a plane is developed between
ity Improvement Program. J AM Coll Surg. 2008;206(5):
the lateral aspect o the IVC and the medial border o 9539.
the adrenal gland. T e vein is double clipped or stapled Malayeri AA, Zaheer A, Fishman EK, et al. Adrenal masses:
with an Endo-GIA. On the le side, the adrenal vein Contemporary imaging characterization. J Comput Assist
drains into the le renal vein. Tomogr. 2013;37:52842.
T ere is no clear literature on the use o lapa- Mansmann G, Lau J, Balk E, et al. T e clinically inappar-
ent adrenal mass: Update in diagnosis and management.
roscopic adrenalectomy or adrenal carcinoma. Endocr Rev. 2004;25:30940.
Reviews have been done that show the laparoscopic Phitayakorn R, McHenry, CR. Perioperative considerations
approach to be equal to an open approach; and in a in patients with adrenal tumors. J Surg Onco. 2012;106:
NSQIP review o over 600 patients, the laparoscopic 60410.
approach was associated with shorter operations, askin HE, Siperstein A, Mercan S, et al. Laparoscopic poste-
rior retroperitoneal adrenalectomy. J Surg Onco. 2012;106:
shorter hospital stays, less trans usions, ewer reop-
61921.
erations, and less 30 day mortality. However, the sur- erzolo M, Stigliano A, Chiodini I, et al. AME position state-
geon has to be aware o the possibility o invasion to ment on adrenal incidentaloma. Euro J Endocrinology.
other structures. In cases o large adrenal cancers or 2011;164:85170.

http://surgerybook.net/
59
Hyperparathyroidism

Richard K. Inae

A 30-year-old emale is re erred to your general sur- A. Repeat observation and ollow up.
gery practice or evaluation or hypercalcemia by her B. Bilateral neck exploration with identi cation o
primary care physician. A chemistry panel per ormed all our parathyroid glands and subtotal resec-
or her insurance a month ago shows an elevation o her tion o 3 glands.
serum calcium levels. Her primary care physician reor- C. Bilateral neck exploration with identi cation o
dered the chemistry panel two days ago, which shows all our parathyroid glands and removal o the
a persistent calcium elevation but her urinalysis and abnormal parathyroid adenoma.
CBC are also normal. He is concerned she may have D. Bilateral neck exploration with identi cation o
primary hyperparathyroidism. T e patient states she all our parathyroid glands with total resection o
eels ne. Her past medical history is signi cant or a all our glands and reimplantation o a normal
kidney stone a couple o years ago. Her physical exam gland in the sternocleidomastoid muscle.
is unremarkable.
3. During your parathyroidectomy or the above
1. You decide to check the patients parathyroid
patient, you are unable to locate the right lower
hormone level. I the patient has primary
parathyroid gland despite nding the other three
hyperparathyroidism, the parathyroid hormone
normal appearing glands. T e next step would be:
level would be:
A. Close up and repeat sestamibi scan.
A. Low
B. Right thyroid lobectomy.
B. Normal
C. Explore the retroesophageal space.
C. Elevated
D. Per orm a partial median sternotomy.
D. A and B
E. Selective venous sampling or parathyroid
E. B and C
hormone.
2. T e patients parathyroid hormone level (P H)
and ollowing 24 hour urinary collection or 4. Af er per orming a total thyroidectomy in
calcium and creatinine excretion con rm your a 55-year-old emale, the patient develops
suspicion o primary hyperparathyroidism. You Chvosteks sign and parasthesias on post-operative
obtain a sestamibi scan or preoperative planning day one. She is started on calcium and vitamin D
that shows a likely adenoma o the right lower replacement therapy and discharged home with
parathyroid gland. Your management o this those medications as well as teriparatide shots
patient would be: (parathyroid hormone replacement therapy). Af er

http://surgerybook.net/
C H AP TER 5 9 H y P ERPARATH y Ro i d i s m 213

one year postsurgery, the patient still requires


calcium and vitamin D replacement as well as
teriparatide shots. T e likely structure injured
during the thyroidectomy was the:
A. Superior thyroid arteries
B. Middle thyroid veins
C. Recurrent laryngeal nerves
D. In erior thyroid arteries
E. Superior laryngeal nerves

ANSWERS
1. E. Hypercalcemia or high-normal serum calcium
levels with normal or elevated parathyroid hormone Sestamibi scan at 20 minutes post-injection shows normal
levels con rm primary hyperparathyroidism. Ele- appearing uptake in the thyroid gland and a nodular region
vated calcium levels should normally suppress P H o intense uptake in the in erior pole o the right lobe o the
production. Other associated lab ndings o primary thyroid gland. (Images and descriptions courtesy o Andrew
hyperparathyroidism may include decreased serum Mullins D.O. and John Yasmer M.D.)
phosphate levels, elevated serum chloride levels, and
elevated BUN, Cr, and alkaline phosphatase.
Primary hyperparathyroidism is usually ound
by routine lab testing, and most patients are asymp-
tomatic. I symptoms are present, they usually con-
sist o history o renal calculi, bone pain, pathologic
ractures, bone sha tumors, muscle weakness, and
depression/lethargy and body aches/pains.

2. C. A single parathyroid adenoma causes primary


hyperparathyroidism in 80% to 85% o cases. T e
rest o the cases are caused by parathyroid gland
hyperplasia (10%), double adenomas (4%), and para-
thyroid carcinoma (1%). Observation and medical
management is not cost e ective in the management Sestamibi scan at 2 hours post-injection shows normal wash-
o primary hyperparathyroidism. T e resection o out o the thyroid with a persistent increased uptake in the re-
the 3 parathyroid glands would be most applicable gion o the in erior pole o the right lobe o the thyroid gland
to parathyroid gland hyperplasia. which is consistent with a parathyroid adenoma. (Images
A more recent alternative to the traditional and descriptions courtesy o Andrew Mullins D.O. and John
approach o bilateral neck exploration and our Yasmer M.D.)
gland identi cation is the removal o the parathyroid
adenoma using preoperative/intraoperative gamma- 3. C. Surgical excision o the parathyroid adenoma is
probe localization with or without the use o intraop- curative o primary hyperparathyroidism in 90% to
erative P H assay. T is approach (minimally invasive 97%. A missed parathyroid adenoma accounts or
radioguided parathyroidectomy or MIRP) relies on the majority o surgical treatment ailures. Methods
the adenoma being seen on the technetium-99m ses- described to acquire the missing parathyroid gland
tamibi (MIBI) scan which it does about 80% to 90% via the transcervical incision include examining
o the time. I the MIBI scan is positive, then the long the retroesophageal space, the carotid sheath, the
term success rate o the MIRP (98%) is equivalent submandibular region, and the thymus gland. T e
to conventional our gland identi cation with the most common location or a missing parathyroid
bene ts o a signi cantly decreased hospital stay and adenoma is the retroesophageal space. Additional
avoidance o general anesthesia. adjuncts include intraoperative ultrasound. Partial

http://surgerybook.net/
214 G EN ERAL s U RG ERy EXAm i N ATi o N AN d Bo ARd REVi EW

or complete thyroid lobectomy, bilateral internal the American association o endocrine surgeons position
jugular vein sampling o P H to evaluate or which statement on the diagnosis and management o primary
side o the neck the adenoma is on, and partial or hyperparathyroidism. Endocr Pract. 2005;11:4954.
Goldstein RA, Billheimer D, Martin WH, et al. Sestamibi
complete median sternotomy are additional options scanning and minimally invasive radioguided parathy-
to consider. I the parathyroid gland is still missing, roidectomy without intraoperative parathyroid hormone
a last approach is to ligate the in erior thyroid artery measurement. Ann Surg. 2003 May; 237(5): 72231.
and its arterial branches on the side o the missing Goldstein RE, Blevins L, Delbeke D, et al. E ect o minimally
gland. invasive radioguided parathyroidectomy on ef cacy, length
o stay, and costs in the management o primary hyper-
4. D. Most o the time, the in erior and superior para- parathyroidism. Ann Surg. 2000;231:73242.
Halsted WS, Evans HM. T e parathyroid glandules: T eir
thyroid arteries are derived rom branches o o the blood supply, and their preservation in operation upon the
in erior thyroid artery. Another common variant is thyroid gland. Ann Surg. 1907(46): 489506.
the superior parathyroid arteries arising rom the Heath DA, Heath EM. Conservative management o primary
anastomosing branches o the superior and in erior hyperparathyroidism. J Bone Miner Res. 1991;6:S11720.
thyroid arteries. Jaskowiak N, Norton JA, et al. A prospective trial evaluating a
standard approach to reoperation or missed parathyroid
adenoma. Ann Surg.September 1996;224(3): 30820.
BIBILIOGRAPHY Udelsman R, Donovan PI. Remedial parathyroid surgery:
AACE/AAES ask Force on Primary Hyperparathyroidism. Changing trends in 130 consecutive cases. Ann Surg. Sep-
T e American association o clinical endocrinologists and tember 2006;244(3): 4719.

http://surgerybook.net/
60
T yroid Disease

Richard K. Inae

A 43-year-old emale is re erred to your o ce by her cancer on FNA and no lymphadenopathy on


primary care physician or evaluation o a le thyroid ultrasound?
lesion. T e primary care physician thought the le A. Follow-up with repeat ultrasound in 6 months
side o the neck elt uller than the other and ordered B. Le thyroid lobectomy
an ultrasound o the neck and lab work. A lesion was C. Le thyroidectomy with isthmusectomy
seen on the ultrasound and her primary care physi- D. otal thyroidectomy
cian re erred her to general surgery or evaluation. T e
patient states that she eels per ectly well with no loss 4. What is the recommended initial surgical approach
in weight or hot ashes. She has no amily history o i the 1.1 cm lesion was non-diagnostic on initial
cancer or neck irradiation. A er per orming a thor- and repeat FNA, and no lymphadenopathy is seen,
ough physical exam, you eel a ullness in her le neck and the patient desires a limited surgical procedure?
corresponding with her le thyroid gland but no obvi- A. Repeat FNA again
ous discrete mass. Her lab work shows a normal chem- B. Le thyroid lobectomy
istry panel and normal thyroid panel. T e ultrasound C. Near-total thyroidectomy
reveals a 0.4 cm circular, smooth, solid, intracapsular D. otal thyroidectomy
lesion.
5. A 23-year-old Paci c Islander emale is re erred to
1. What would be your next step in management or you or evaluation or a lef neck mass. She rarely
this lesion? receives any medical care. She has noticed the neck
A. Fine-needle aspiration (FNA) mass or the past 6 months. T e lump is increasing
B. Repeat lab work in size with time. She states that otherwise, she
C. Observation eels normal. On physical exam, she has a palpable
D. Subtotal thyroidectomy lump on her lef thyroid gland as well as lef neck
lymphadenopathy. She is very thin with long limbs
2. What would your next step be i the lesion were and round, rm lumps in her lips. You suspect
1.1 cm on imaging and palpable? she has multiple endocrine neoplasia (MEN) 2B
syndrome. What potential abnormalities would
A. FNA
you suspect and work up?
B. Repeat lab work
C. Follow-up with repeat ultrasound in 6 months A. Papillary thyroid cancer
D. Subtotal thyroidectomy B. Pituitary adenoma
C. Pheochromocytoma
3. What would be your treatment option i the 1.1 cm D. Parathyroid hyperplasia
lesion showed suspicion or a papillary thyroid E. Hirschsprungs

http://surgerybook.net/
216 G EN ERAL S U RG ERY EXAM I N ATIO N AN D BO ARD REVI EW

ANSWERS 4. B. For indeterminate biopsies on FNA such as


ollicular neoplasms or Hurthle cell neoplasms on
1. C. T yroid nodules are common with approximately pathology, the malignancy risk is approximately 20%.
4% o the population having a palpable nodule and T is risk increases with large tumors (> 4 cm) and
more than 50% having a nodule on ultrasound. Per pathology ndings consistent with cellular pleomor-
Durante et al., the majority o patients with asymp- phism or other atypia and suspicion or papillary car-
tomatic, sonographically or cytologically benign thy- cinoma. Other risk actors include amily history o
roid nodules will not undergo any signi cant size thyroid carcinoma and previous radiation exposure.
increase or display cancer at ve years. No FNA or Per the 2015 A A guidelines, or patients with an
surgery is required or this nodule per the Ameri- isolated indeterminate solitary nodule, thyroid lobec-
can T yroid Association (A A) 2015 guidelines and tomy is the recommended initial surgical approach.
repeating lab work is not indicated or this patient. otal thyroidectomy is recommended or indetermi-
T e A A in their 2015 guidelines recommend con- nate nodules which are large (> 4 cm), show marked
sidering repeat ultrasound at 612 months or nod- atypia present on biopsy, in patients with a amily
ules with highly suspicious ultrasound patterns history o thyroid cancer, in patients with a history
(hypoechoic, taller than wide) and 1224 months or o radiation exposure, or or those with FNA biopsy
nodules with low to intermediate suspicion on ultra- results suspicious or papillary carcinoma.
sound pattern (iso or hyperechoic, hypoechoic with
smooth margins and no microcalci cations). 5. C. T e MEN 2B syndrome is characterized by med-
ullary thyroid cancer, pheochromocytomas, and
2. A. T e A A and the AACE recommend biopsy i the mucosal neuromas as well as mar anoid appearance.
nodules are > 1 cm. T e A A recommends biopsy o MEN 2A syndrome is characterized by medullary
subcentimeter nodules larger than 5 mm that are thyroid cancer, pheochromocytomas, and parathy-
suspicious appearing on ultrasound (hypoechoic roid hyperplasia causing hyperparathyroidism as
with microcalci cations) and individuals with a high well as Hirschsprungs and cutaneous lichen amyloi-
risk history (e.g., amily history o papillary thyroid dosis. MEN 1 syndrome is characterized primarily by
cancer, history o irradiation as a child or adolescent, pituitary, pancreatic, and parathyroid tumors.
prior hemithyroidectomy with discovery o thyroid
cancer, and positron emission tomography (PE ) BIBLIOGRAPHY
positive thyroid nodules). T e AACE recommends Durante C, Costante G, Lucisano G, et al. T e natural history
biopsy o nodules o any size or suspicious history o benign thyroid nodules. JAMA. 2015;313(9):92635.
(e.g., history o irradiation in childhood or adoles- Gharib H, Papini E, Baskin HJ, et al. American association o
cence, amily history o thyroid cancer or MEN syn- clinical endocrinologists and associazione medici endocri-
drome, previous thyroid surgery or cancer, increased nologi medical guidelines or clinical practice or the diag-
nosis and management o thyroid nodules. Endocr Pract.
calcitonin levels) or suspicious ultrasound ndings
January-February 2006;12(1): 63102.
(e.g., hypoechogenicity, irregular borders, microcal- Haugen BR, Alexander EK, Bible KC, et al. 2015 American
ci cations, taller than wide, chaotic intranodular vas- T yroid Association Management Guidelines or Adult
cular images). Patients with T yroid Nodules and Dif erentiated T yroid
Cancer: T e American T yroid Association Guidelines
3. D. For patients with FNA biopsies showing thy- ask Force on T yroid Nodules and Dif erentiated T yroid
roid cancer, the initial surgical procedure should be Cancer. T yroid. January 2016;26(1):1133.
Levine RA. Current guidelines or the management o thyroid
near-total or total thyroidectomy or lesions greater
nodules. Endocr Pract. July-August 2012;18(4):5969.
than 1 cm. T yroid lobectomy may be considered or Raue F, Frank-Raue K. Genotype-phenotype relationship in
< 1 cm, low risk, uni ocal, intrathyroidal papillary multiple endocrine neoplasia type 2: Implications or clini-
carcinomas in patients with no history o prior head cal management. Hormones. 2009;8(1):238.
and neck irradiation and no evidence o cervical T akker RV, Newey PJ, Walls GV, et al. Clinical practice guide-
nodal metastases. lines or multiple endocrine neoplasia ype 1 (MEN1).
J Clin Endocrinol Metab. September 2012;97(9):29903011.

http://surgerybook.net/
61
Pancreatic Endocrine umors

Jigesh A. Shah and Omar Yusef Kudsi

1. T e patient from the previous question denies any B. With PNE lesions located in the body or tail
family history of multiple endocrine neoplasia o the pancreas, a distal pancreatectomy can be
(MEN). In regards to MEN syndromes, which of per ormed and even i the lesion is thought to
the following is correct? be malignant, splenic preservation may still be
A. T e most common endocrinopathies in MEN-1 attempted.
are pituitary adenomas, reaching 100% penetra- C. Enucleation and ormal resection o PNE s gen-
tion by age 50. erally lead to equivalent outcomes or the man-
B. T e most common endocrinopathy in MEN-2 is agement o small PNE s less than 2 cm.
pheochromocytoma. D. Most non unctional PNE s are benign, even
C. MEN-2B includes medullary thyroid cancer and i they have local invasion and regional lymph
pheochromocytoma in all patients. node involvement.
D. Prophylactic thyroidectomy is not recom- E. Formal resection with appropriate lymphade-
mended in patients with a con rmed RE germ- nectomy is recommended or patients with insu-
line mutation and a discernable thyroid mass linoma/gastrinoma smaller than 2 cm and other
by ultrasound i the serum calcitonin levels are unctional tumors such as VIPoma, somatostati-
normal. noma, and glucagonoma which demonstrate no
E. During work up or a MEN, i a pheochromo- evidence o metastatic disease.
cytoma is detected it should be resected prior to
surgery on the thyroid. 3. You are evaluating a 50-year-old male with a
history of recurrent duodenal ulcers located in the
2. A 40-year-old male was found to have recurrent second portion of the duodenum. Further work-
hypoglycemic symptoms with a plasma glucose up demonstrates that he is H. pylori negative and
level of 44 mg/dL and a C-peptide level of an upper endoscopy shows markedly hypertrophic
200 pmol/L. He has relief of his symptoms with gastric folds. Which of the following is true
regarding his likely underlying diagnosis?
the administration of glucose. Further workup
A. A asting serum gastrin level o more than
includes a positive 72-hour fast. In regards to
1,000 pg/mL (normal is < 100 pg/mL) is consid-
pancreatic neuroendocrine tumors, which of the ered diagnostic or gastrinoma.
following is true? B. Following intravenous secretin injection, an
A. Pancreatic neuroendocrine tumors (PNE s) idiosyncratic rise in serum gastrin o 10 pg/mL
are common and 75% o cases are ound in is diagnostic.
association with symptoms related to a speci c C. T e majority o gastrinomas are generally ound
hormone. within the gastrinoma triangle, which is bound

http://surgerybook.net/
218 G EN ERAL S U RG ERY EXAM I N ATIO N AN D BO ARD REVI EW

by the junction o the cystic and common bile Hyperparathyroidism occurs in approximately
duct, the junction o the rst and second portion 20% o patients. Medullary thyroid cancer (M C) is
o the duodenum and the junction o the neck typically the rst presenting endocrine tumor mani-
and body o the pancreas. esting between the age o 5 and 25 years. MEN-2B
D. Gastrinomas are commonly ound in locations is relatively rare, accounting or less than 10% o the
such as the gallbladder, renal capsule, splenic cases o amilial medullary thyroid cancers. T is
hilum, mesentery, omentum, ovary, lymph nodes, syndrome includes medullary thyroid cancer in all
and heart. patients and pheochromocytoma in approximately
E. Medical treatment o Zollinger-Ellison Syn- 50%. A ected patients also exhibit a mar anoid body
drome with proton pump inhibitors is very e ec- habitus, enlarged lips, and have mucosal neuromas o
tive, eliminating any malignant potential, thus the eyelids, lips, and tongue.
preventing the progression o malignant disease. MEN-2B is an aggressive orm o MEN-2 in that
M C tends to present earlier than in MEN-2A or
4. A 59-year-old male patient presents with diabetes, amilial medullary thyroid carcinoma (FM C), o en
anemia, and a 30 pound weight loss. Recently, during the rst year o li e. I a carrier state is identi-
he started to develop a rash described as raised ed in a pre-clinical setting, surgery should be done
erythematous plaques that developed central be ore the age o typical onset o cancer or metasta-
bullae that sloughed, leaving necrotic centers sis. In a prophylactic setting, the procedure o choice
and serous crusts. Serum glucagon levels are is a total thyroidectomy. I the patient has an elevated
1100 pg/dL. A C scan of the abdomen and calcitonin level, then a total thyroidectomy and cen-
pelvis demonstrated a 4 cm mass in the tail of the tral neck dissection should be done. Genetic testing
pancreas. Which of the following is true regarding dictates strati ed management o the thyroid. T e
his diagnosis? American T yroid Association (A A) has developed
A. Localization with C and staging with soma- risk groups based on the speci c mutation. Carriers
tostatin receptor scintigraphy are generally with MEN-2B should have a thyroidectomy within
insu cient or obtaining a diagnosis, despite the the rst six months. Patients with MEN-2 should
large size and somatostatin-rich receptor status have a thyroidectomy be ore the age o ve and
o these tumors. patients with the lowest level o risk should have a
B. Necrolytic migratory erythema occurs in thyroidectomy between the ages o 5 and 10 years.
approximately 5% o these patients. Similarly, identi cation o the speci c codon will
C. Deep venous thrombosis and thromboembolism dictate screening and management or pheochromo-
are quite common and patients should be ully cytomas and hyperparathyroidism.
anticoagulated at the time o diagnosis and in e-
rior vena cava lters should be considered pre- 2. C. PNE s are uncommon in the general population,
operatively. accounting or only 3% to 5% o pancreatic malig-
D. Glucagonomas are most commonly located in nancies, a majority o which are sporadic and unc-
the head o the pancreas and are usually sensi- tional. Several important actors must be considered
tive to chemotherapy. when selecting the operative approach to, including
E. When per orming a distal pancreatectomy and the tumors unctional status, benign or malignant
splenectomy or a distal pancreatic glucagonoma, nature, involvement with contiguous structures,
a cholecystectomy should not be routinely per- presence o metastatic disease, and whether the
ormed. tumor is sporadic or associated with a genetic syn-
drome. Clinical symptoms and biochemical evidence
o hormone excess determine the tumors unctional
ANSWERS
status. Preoperative evaluation by cross-sectional
1. E. MEN-1 is mani ested by hyperparathyroidism, and unctional (somatostatin) imaging combined
pituitary adenomas, and pancreatic neuroendocrine with endoscopic ultrasound can help determine the
tumors. MEN-2A is mani ested by medullary thyroid tumors benign or malignant nature by delineating
cancer in nearly all patients but pheochromocytoma the degree o local invasion, lymph node involve-
develops in only 40% to 50% o patients. ment, and metastases to the liver and elsewhere.

http://surgerybook.net/
C H AP TER 6 1 PAN C REATIC EN D O C RIN E TU M O RS 219

Endoscopic ne needle aspiration biopsy can con rm


the presence o neuroendocrine cells and may help
determine the proli erative index (Ki67) which in
turn can help grade the likelihood o malignancy.
An accurate history and genetic testing can deter-
mine whether the tumor is sporadic or associated
with a genetic syndrome and must be considered
during preoperative decision making and planning Right of S MA Le ft of S MA
or resection. T e goal o surgery or primary PNE s
is to resect the primary tumor and associated lymph
nodes while preserving the maximal amount o pan-
creatic mass. T e indications or surgery in patients
with PNE s include systemic symptoms due to hor- S MV S MA
mone overproduction, local compressive symptoms
Figure 61-1. Gastrinoma riangle.
due to mass e ect, and prevention o malignant
trans ormation or dissemination. Formal pancre-
atic resection with appropriate lymphadenectomy is gastrinomas. Gastrinomas can be distinguished rom
recommended or patients with established known these other conditions by virtue o its paradoxical
malignancy, including splenectomy when lesions are e ect on serum gastrin levels in response to a secretin
ound within the tail o the pancreas. With insulino- in usion (secretin stimulation test). A er baseline
mas/gastrinomas larger than 2 cm and other unc- gastrin measurement, secretin is administered and
tional tumors such as VIPoma, somatostatinoma, and gastrin levels are rechecked. An increase in gastrin
glucagonoma who have no evidence o metastatic levels o greater than 120 pg/mL over basal levels has
disease a ormal resection is the pre erred operative 94% sensitivity and 100% speci city or gastrinoma.
management strategy. For those lesions smaller than T e majority o gastrinomas are generally ound
2 cm, the survival o simple enucleation is similar to within the gastrinoma triangle, which is bound by
a ormal resection. the junction o the cystic and common bile duct,
the junction o the second and third portion o the
3. A. Gastrinomas are associated with Zollinger- duodenum and the junction o the neck and body
Ellison syndrome (atypical peptic ulcer disease, o the pancreas. Localization o gastrinomas is best
gastric hyperacidity and hyper-secretion). Proton- per ormed with endoscopic ultrasonography and
pump inhibitors have no e ect on the natural his- somatostatin-receptor scintigraphy. Because o the
tory o the disease. Most patients are male ( 60%) malignant potential o gastrinomas, medical treat-
and the average age at diagnosis is about 60 years. A ment alone does not prevent progression o malig-
asting serum gastrin level o more than 1000 pg/mL nant disease and with the development o excellent
(10 times the upper limit o normal) is diagnostic o pharmacologic therapy or the control o gastric acid

Syndrome Chromosome Gene Associated Pathology


MEN 1 Chromosome 11 MENIN Parathyroid Hyperplasia (4-gland)
Pancreatic Islet Cell umors (Gastrinoma)
Pituitary umors (Prolactinoma)
MEN 2A Chromosome 10 RE proto-oncogene Parathyroid Hyperplasia (4-gland)
Pheochromocytoma
Medullary T yroid Carcinoma
MEN 2B Chromosome 10 RE proto-oncogene Pheochromocytoma
Medullary T yroid Carcinoma
Mucocutaneous Ganglioneuroma, Mar anoid
Body Habitus

MEN Multiple Endocrine Neoplasia, RE REarranged during rans ection.

http://surgerybook.net/
220 G EN ERAL S U RG ERY EXAM IN ATI O N AN D BO ARD REVIEW

hypersecretion, surgery or gastrinomas has shi ed BIBLIOGRAPHY


toward tumor localization and resection. Brandi ML, Gagel RF, Angeli A, et al. Guidelines or diagnosis
and therapy o MEN type 1 and 2. J Clin Endocrinol Metab.
4. C. Glucagonomas have been described as caus- 2001;86:565871.
ing the 4 Ds: dermatitis, diabetes, depression, and Gold nger SE. Zollinger-Ellison syndrome (gastrinoma):
deep vein thrombosis. Weight loss and a character- Clinical mani estations and diagnosis. Available at:
http://www.uptodate.com/contents/zollinger-ellison-syndrome-
istic skin rash (necrolytic migratory erythema) are
gastrinoma-clinical-mani estations-and-diagnosis?source=
the most common presenting eatures, occurring in search_result&search=Gold nger+SE+Zollinger-Ellison+
about 70% o patients, with other symptoms occur- syndrome+%28gastrinoma%29%3A+Clinical+mani estations+
ring less requently at initial presentation. Necrolytic and+diagnosis&selected itle=1~150.
migratory erythema, which is an intensely pruritic Kloos R , Eng C, Evans DB, et al: Medullary thyroid cancer:
rash that usually presents on the lower abdomen, Management guidelines o the American T yroid Associa-
tion. T yroid. 2009;19:565612.
perineum, perioral area, or lower extremities, may Norton JA, Fraker DL, Alexander HR, et al. Surgery increases
present be ore any other signs or symptoms and is survival in patients with gastrinoma. Ann Surg. 2006;44(3):
pathognomonic or glucagonoma and is ound in up 41019.
to 70% o patients. Most glucagonoma patients have ORiordain DS, OBrien , Crotty B, et al. Multiple endo-
glucose intolerance and asting serum glucagon lev- crine neoplasia type 2B: More than an endocrine disorder.
Surgery. 1995;118:93642.
els between 1000 and 5000 pg/mL.
Passaro E, Howard J, Sawicki MP, et al. T e origin o sporadic
T e initial management o a patient with a gluca- gastrinomas within the gastrinoma triangle: A theory. Arch
gonoma should ocus on blocking and treating the Surg. 1998;133(1):1316.
metabolic e ects o glucagon hypersecretion and Phan GQ, Yeo CJ, Hruban RH, et al. Surgical experience with
preventing or treating venous thromboembolism. pancreatic and peripancreatic neuroendocrine tumors:
Octreotide and intravenous f uids should be admin- Review o 125 patients. J Gastrointest Surg. 1998;2:473.
Pitt SC, Pitt HA, Baker MS, et al. Small pancreatic and peri-
istered. Additionally, due to their hypercoagulable ampullary neuroendocrine tumors: resect or enucleate?
state and predisposition to thromboembolic events, J Gastrointest Surg. 2009;13:1692.
patients should be ully anticoagulated. Glucagono- Plockinger U, Rindi G, Arnold R, et al. Guidelines or the diag-
mas are most commonly located in the tail o the nosis and treatment o neuroendocrine gastrointestinal
pancreas and surgical resection usually consists o tumours. A consensus statement on behal o the European
Neuroendocrine umour Society (ENE S). Neuroendocri-
a distal pancreatectomy along with a splenectomy.
nology. 2004;80:394424.
A cholecystectomy should be per ormed prophylac- Stabile BE, Morrow DJ, Passaro E Jr. T e gastrinoma triangle:
tically at the time o the initial operation to elimi- Operative implications. Am J Surg. 1984;147:2531.
nate the potential or gallstone related complications Wermers RA, Fatourechi V, Wynne AG, et al. T e gluca-
which are associated with octreotide therapy. gonoma syndrome. Clinical and pathologic eatures in 27
patients. Medicine. 1996;75(2):5363.

http://surgerybook.net/
62
Salivary Gland umors

Maxwell Sirkin and William V. Rice

A 55-year-old Caucasian male is re erred to your clinic A. otal parotidectomy


or a palpable 2 cm mass located 2 to 3 cm anterior to B. Super cial parotidectomy, sparing the acial
his right tragus. He states that it has been present or nerve
about 9 months and has slowly been getting larger. He C. Super cial parotidectomy, sacri cing any acial
denies any pain, acial weakness or numbness, evers, nerve that may be involved
weight loss, atigue, and dry mouth. He has no medi- D. Enucleation o the mass, sparing nerve and nor-
cal or surgical history and does not take any medica- mal glandular tissue
tions. On physical exam you nd a well-circumscribed, E. Surveillance o the mass with serial computer-
mobile, non-tender 2 cm mass, with no overlying skin ized tomography (C ) scans
changes. T ere is no lymphadenopathy and the rest o
the exam is normal. 4. Tissue and diagnostic studies instead reveal a high
grade mucoepidermoid carcinoma that is clearly
1. Regarding the above patient, which o the ollow- deep to the acial nerve. What is the next best step?
ing is the most likely diagnosis? A. otal parotidectomy with resection o any adja-
A. Mucoepidermoid carcinoma cent acial nerve and immediate interposition
B. Adenoid cystic carcinoma nerve gra
C. Pleomorphic adenoma B. otal parotidectomy with sparing o the acial
D. Papillary cystadenoma lymphomatosum nerve, ollowed with post-operative radiation
E. Sjogrens syndrome related glandular enlarge- C. Radiation and chemotherapy with post-treatment
ment surveillance
D. Radiation alone with post-treatment surveil-
2. Which o the ollowing studies is the next best step lance
in the diagnostic workup? E. Super cial parotidectomy, sparing the acial
A. Core needle biopsy nerve, ollowed with post-operative radiation
B. Incisional biopsy
C. Positron emission tomography (PE ) scan 5. A month a er the operation the patient returns to
D. Fine needle aspiration (FNA) your clinic complaining o f ushing and sweating
E. Ultrasound on the right side o his ace when he eats. What is
the cause o this?
3. Tissue and imaging studies reveal a pleomorphic A. Cross innervation between para-sympathetic
adenoma that appears to be con ned to the bers rom the superior salvatory nucleus with
super cial parotid gland. What is the next best sympathetic bers rom the superior cervical
step? ganglion to the skin

http://surgerybook.net/
222 G EN ERAL S U RG ERY EXAM IN ATIO N AN D BO ARD REVI EW

B. Cross innervation between bers in the auricu- MRI and C scans are being used more requently
lotemporal nerve with sympathetic bers to the to help assess or regional lymphadenopathy, extent/
skin depth o tumor and tumor location in relationship
C. Cross innervation between bers in branches o to other important structures. Core needle biopsy in
the acial nerve with sympathetic bers to the the ace and neck is usually not recommended due
skin and sweat glands to the proximity o sensitive structures. Ultrasound
D. Cellulitis alone, without an FNA, will yield little additional
E. Changes in the skin caused by congestion o value than what a C , MRI, and FNA will provide
dermal lymphatics and may be considered a waste o time and resources.
PE is help ul in tumors that are PE avid, such as
Warthins tumors, or when there is concern or dis-
ANSWERS tant disease. PE is not recommended as part o the
1. C. Most salivary gland tumors arise in the parotid routine workup.
gland (70%) with ewer being ound in the subman- Incisional biopsy o the tumor or tissue diagno-
dibular gland (20%). T e larger the gland, the more sis is not recommended due to high recurrence rates,
likely the tumor will be benign. In the parotid gland concerns or seeding, and the potential to turn a sin-
80% o tumors will be benign. Submandibular glands gle lesion into multiple lesions. T e thought is that
have a ratio o 50:50 and minor salivary glands have an incisional biopsy will not entirely remove a single
a ratio o 25:75 benign to malignant, respectively. lesion and when the disease recurs it will recur in
Signs and symptoms including pain, rapid growth, multiple locations due to the tumor le behind rom
nerve weakness, paresthesias, cervical lymphad- the initial biopsy.
enopathy, and xation to underlying tissue are all
concerning or malignancy. Skin changes, purulent 3. B. Pleomorphic adenomas rarely extend into the
drainage, evers, chills, and tenderness to palpation deep space and a super cial parotidectomy should
are more indicative o an in ectious cause or granu- be the rst surgical intervention or most o these
lomatous disease. Dry mouth or mucus membranes, benign tumors. Pleomorphic adenomas do not
with bilateral swelling is associated with Sjogrens invade into the nerve and all attempts should be made
disease. to identi y and spare the acial nerve and its branches
It is important to realize that pleomorphic adeno- during the surgery. For a benign tumor, resecting the
mas, a benign tumor, are the most common tumors least amount o tissue will result in superior cosmetic
in the parotid gland and account or 40% to 70%. T e outcomes and results in less potential or injury to
second most common tumor is a papillary cystad- the acial nerve. I the tumor is benign and has no
enoma lymphomatosum, also known as a Warthins obvious invasion into the deep lobe then all that is
tumor. T is tumor is also benign with a higher pro- required is a super cial parotidectomy.
pensity to be bilateral. Mucoepidermoid carcinoma otal parotidectomy is not necessary in the
is the most common malignant tumor o the parotid patient unless intraoperative ndings show involve-
gland. Adenoid cystic carcinomas, which are malig- ment o the deep lobe. Enucleation o the tumor is
nant, are the most common tumors o the subman- not recommended as there is a high risk or recur-
dibular and minor salivary glands. rence when this is per ormed. Surveillance o parotid
gland tumors with serial examination and C is rec-
2. D. T is patients signs and symptoms suggest a ommended or certain Warthins tumors in patients
benign tumor, however malignancy may be present. that have contraindications to surgery, but is oth-
T ere ore, tissue diagnosis is recommended prior erwise not recommended or either malignant or
to proceeding with surgical intervention. T e next benign tumors o the salivary glands.
best step or surgical planning and patient coun-
seling would be to obtain tissue preoperatively with 4. B. In this patient there is involvement o the deep
a ne needle aspiration. FNA has excellent sensitivity lobe, which is an indication or total parotidectomy.
(92%), speci city (100%), and accuracy (98%). FNA Since the patient has no neurologic symptoms prior
with or without ultrasound guidance would be the to the operation, i the nerve is not being visibly/
initial study o choice. directly invaded then you should attempt to dissect

http://surgerybook.net/
C H AP TER 6 2 S ALI VARY G LAN D TU M O RS 223

the tumor o o the nerve ollowed by post-operative sweat glands rom signals in the auriculotemporal
radiation. You should still plan or the potential nerve, meant to trigger salivation in a gland that no
need to do a nerve harvest and gra , should you longer exists or has had part o it removed.
nd that there is nerve invasion during the opera- Option A describes a phenomenon that is ound
tion. T e sural nerve and greater auricular nerve when submandibular glands or minor salivary glands
are commonly used conduits. Some sources suggest are injured and is similar to Freys syndrome in the
that malignant tumor con ned to the super cial sense that it is caused by aberrant nerve regeneration.
lobe with no obvious peri-neural involvement can However, it involves di erent nerves and would not
be treated with a super cial parotidectomy, dissec- present as a result o parotid surgery.
tion o the tumor o o the acial nerve, ollowed by Cellulitis is usually accompanied by pain and con-
post-operative radiation. Non-surgical management, tinuous redness, not sweating, and is usually sooner
preoperative chemotherapy and radiation only o a er the operation. Congestion o dermal lymphatics
malignant salivary tumors are not currently recom- would create a peau dorange appearance o the skin
mended methods o treatment. and is usually associated more with inf ammatory
Malignant salivary tumors are staged according to breast cancer not salivary gland tumors.
size and degree o local invasion. T ey are also classi-
ed according to histological grade which separates BIBLIOGRAPHY
them into either high or low, based on the number Gordon AB, Fiddian RV. Freys syndrome a er parotid sur-
o mucoid cells and epidermoid components. High gery. Am J Surg. 1976;132(1):548.
grade tumors indicate a more aggressive tumor and Kaplan M, Abemayor E. Major salivary glands. In: Fu YS,
thus exhibit a worse prognosis. High grade histol- Wenig BM, Abemayor E, et al., eds. Head and Neck Pathol-
ogy is typically an indication or an ipsilateral neck ogy: With Clinical Correlations. New York, NY: Churchill-
Livingstone; 2001:231.
dissection. Indications or post-operative radiation Lorenz RR, Couch ME, Burkey BB. Head and Neck. In: Beau-
include: high grade, extra-glandular involvement, champ DR, ownsend CM, Evers MB, Mattox KL. Sabiston
peri-neural invasion, direct invasion o surrounding extbook o Surgery: T e Biological Basis o Modern Surgi-
structures, dissection o tumor o the acial nerve cal Practice. 19th ed. Philadelphia, PA: Elsevier Saunders;
and distant metastatic disease. 2012:81113.
Miller MC, Moyer JS, eknos N. Head and Neck. In: Mulhol-
5. B. Freys syndrome is a rare complication o parotid land MW, Lillemoe KD, Doherty GM, et al. Greenf elds Sur-
gery: Scientif c Principles & Practice. 5th ed. Philadelphia,
tumor surgery and is the result o necessary injury PA: Lippincott Williams & Wilkins; 2011:6157.
to the auriculotemporal nerve when resecting the Rahbar R, Grimmer FJ, Vargas SO, et al. Mucoepidermoid
parotid gland. T e auriculotemporal nerve has par- carcinoma o the parotid gland in children: A 10-year
asympathetic bers that trigger secretion o saliva experience. Arch Otolaryngol Head Neck Surg. 2006;132:
rom the parotid gland. When these bers are injured 37580.
Stewart CJ, MacKenzie K, McGarry GW, et al. Fine-needle
during parotidectomy and undergo healing, they can
aspiration cytology o salivary gland: A review o 341 cases.
sometimes cross innervate with sympathetic nerve Diagn Cytopathol. 2000;22:13946.
bers that travel to the skin and sweat glands. T e Witt RL: T e signi cance o the margin in parotid surgery or
result is gustatory sweating or triggering o the pleomorphic adenoma. Laryngoscope. 2002;112:214154.

http://surgerybook.net/
This page intentionally left blank

http://surgerybook.net/
Pediat r ic Sur g er y
Mary J. Edwards

http://surgerybook.net/
This page intentionally left blank

http://surgerybook.net/
63
Head and Neck Pediatric umors

Margaret E. Clark

A 4-year-old girl is brought in to see her primary care C. When a child presents with an in ected bran-
provider (PCP) by her parents. T ey have noticed a chial cle cyst, unlike a thyroglossal duct cyst, it
small mass on the anterior portion o her neck, but is imperative to per orm resection immediately
recently started noticing that it was spontaneously to control the in ection and prevent urther com-
draining and has become erythematous. Her PCP has plications.
the girl stick out her tongue and notices that the mass D. I a cyst is located on the anterior border o the
moves with tongue protrusion. T e physician correctly sternocleidomastoid muscle it is a type II bran-
identi es that this is likely an in ected thyroglossal duct chial cle cyst.
cyst. She appropriately places the child on antibiotics,
covering or oral ora, and then re ers the child to sur- 3. Which o the ollowing statements is true regarding
gery or operative intervention, delayed until the in ec- the presentation, diagnosis, and management o a
tion clears. lymphatic mal ormation?
1. What is the most appropriate treatment or a A. Lymphatic mal ormations never present with
thyroglossal duct cyst? airway compromise given their position in the
posterior neck.
A. Surgical resection o the cyst, taking care to only B. Ninety percent o lymphatic mal ormations are
remove the cyst and not disturb any o the criti- apparent at birth, with most diagnosed during
cal surrounding neck structures prenatal ultrasound.
B. Resection o the thyroglossal duct and the com- C. Fine needle aspiration (FNA) is necessary to
plete hyoid bone con rm the diagnosis o a lymphatic mal orma-
C. Removal o the cyst en bloc, excising a core o tion prior to resection.
tissue around the tract to the base o the tongue, D. Given that lymphatic mal ormations are benign,
and including the central 1/3rd o the hyoid radical resection should never be per ormed, due
bone to the lesions close association or proximity to
D. otal thyroidectomy vital structures.
2. Regarding branchial clef cysts, which o the E. All o the above are true.
ollowing statements is correct?
A. T e rst branchial cle extends to the audi- 4. What is the most common childhood malignancy
tory canal, placing both the acial nerve and the presenting as a head or neck mass in children over
parotid at risk or injury. the age o 5?
B. ype I cysts are the most common, accounting A. T yroid carcinoma
or 80% to 90% o all lesions. B. Lymphoma

http://surgerybook.net/
228 G EN ERAL S U RG ERY EXAM IN ATI O N AN D BO ARD REVIEW

C. Rhabdomyosarcoma presenting with a thyroglossal duct cyst. Papillary


D. Neuroblastoma carcinoma can arise in adulthood rom the dysge-
E. Melanoma netic ectopic thyroid tissue and is reason to per orm
complete excision o the lesion in a child with a thy-
5. Which o the ollowing is true regarding treatment roglossal duct cyst or sinus. T ough these cysts, simi-
or torticollis? larly to branchial cle cysts, o en present with an
A. Division o the sternocleidomastoid muscle is acute in ection during childhood, up to 40% present
only indicated i the in ant has plagiocephaly or a er the age o 20. Dermoid cysts are o en usually
hemihypoplasia. ound midline on the neck, potentially being con-
B. Botulinum toxin injection should be the rst used with a thyroglossal duct cyst. However, they
treatment, ideally preventing the in ant rom are not associated with the thyroid and will not move
needing to undergo physical therapy. when the patient swallows or with protrusion o the
C. Physical therapy, with passive range o motion tongue, as opposed to a thyroglossal duct cyst. It is
and neck-stretching, is the primary treatment recommended that the presence o normal thyroid
modality. tissue outside o the thyroglossal duct cyst is con-
D. Ocular imbalance causing torticollis will be xed rmed prior to excision. Ectopic thyroid tissue is
by corrective lenses. identi ed in surgical specimens in approximately
25% to 35% o cases, though it is actually rare that
this is the childs only thyroid tissue. However, i thy-
ANSWERS
roid tissue is noted in the resected specimen thyroid
1. C. T yroglossal duct cysts are ound in the midline scanning is still indicated to identi y the child that
o the neck. T ese are the second most common con- may require li elong thyroid hormone replacement.
genital pediatric neck masses, though the most com-
mon midline anomaly. T ough classically described 2. A. T e di erential diagnosis o a lateral neck mass is
as midline, up to 40% may actually lie just lateral to extensive and includes branchial cle remnants, lym-
the midline. T ese cysts, based on their embryological phatic mal ormation, dermoid cyst, hemangioma,
descent, are ound to have a tract extending rom the lymphadenitis, torticollis, neuro broma, lipoma, or
hyoid bone up to the base o the tongue. T e normal metastatic malignancy to the cervical lymph nodes.
thyroglossal duct involutes by the eighth etal week, Branchial cle cysts are the most common congeni-
with remnants leading to cyst ormation or ectopic tal neck lesion, accounting or approximately 20%
thyroid tissue anywhere along the tract. Approxi- to 30% o all pediatric neck masses. Sinuses, stulas,
mately 60% o these cysts lie adjacent to the hyoid and remnants are usually noticed at birth or early
bone, 24% lie above, and 13% lie below the hyoid in li e, opposed to branchial cle cysts which are
bone. Cysts may also present intralingually, with more likely to present later in childhood. T ere are
2% occurring at the base o the tongue, which may six paired branchial arches, containing ectodermal,
place a neonate at risk or airway obstruction. Sur- mesodermal, and endodermal components.
gery involves removal o the middle 1/3 o the hyoid Understanding the relationships o these arch
bone, called the Sistrunk procedure, with removal o components is help ul to conceptualize and catego-
the hyoid necessary to prevent recurrence. rize congenital neck masses. T e dorsal portion o
T is procedure was rst described in 1920, and he the rst cle becomes the external auditory canal.
described the importance o an en bloc dissection, Given this tracts location, it places the acial nerve
excising a core o tissue around the tract, necessary at risk or injury during resection. ype II is the most
given the possible existence o multiple tracts. Recur- common abnormality, with 90% to 95% o cases.
rence rates are less than 10% and usually related to T ey are ound along the anterior border o the ster-
incomplete excision or intraoperative rupture. T ere nocleidomastoid muscle (SCM) and may be bilat-
is a possible association between preoperative in ec- eral in 10% o cases. Anomalies o the third cle are
tion and risk o recurrence, though not con rmed extremely rare, but are also located along the ante-
in recent analyses. T ese congenital lesions have the rior border o the SCM. However, they are usually
potential or malignant trans ormation, with papil- lower in the neck and pass lateral to the carotid
lary adenocarcinoma ound in up to 10% o adults bi urcation rather than through it. Sinus tracts o the

http://surgerybook.net/
C H AP TER 6 3 H EAD AN D N EC k P ED IATRI C TU M O RS 229

second branchial anomalies pass between the exter- between the rst, second, and third branchial cysts
nal and internal carotid arteries, entering the lateral and stulas.
wall o the pharynx at the tonsillar ossa. Congenital
lesions may present while acutely in ected, and prior 3. D. Lymphatic mal ormations, previously re erred
to operative intervention their close relationships to to as cystic hygromas or lymphangiomas, are most
major nerves and vessels needs to be understood. commonly located in the head and neck, speci cally
Injury to the parotid gland and acial nerve may in the posterior triangle o the neck. However, they
occur rom recurrent in ection o a ype I branchial can occur in other locations, and when they involve
cle cyst, or rom a complication o an incision and the tongue or the mediastinum they may result in
draining. In ection o the third and ourth branchial airway compromise, or in dysphagia and ailure to
cle cysts are rare and more challenging to diagnose thrive. T ese are vascular mal ormations, occurring
and treat. Suppurative thyroiditis may be due to an in 1 out o every 2000 to 4000 births. T ey occur due
in ected third or ourth arch sinus. ype III and IV to sequestrations o lymphatic tissue that dont com-
cysts may cause airway edema and dif culty swal- municate with the normal lymphatic system. Ninety
lowing, which can o en be controlled by controlling percent o these lesions appear be ore the age o two,
the acute in ection. with 50% to 65% appearing at birth. T ese lesions are
It is important that the cysts and their stula tracts more commonly located on the le side o the neck
be excised a er the acute in ection has been cleared and i large enough may be noted on prenatal ultra-
in order to prevent recurrence. T e initial treatment sound.
is antibiotics and not surgery. Surgery is usually However, lymphatic mal ormations can be ound
6 weeks later, and the entire tract, to include the skin anywhere in the so tissues o the ace or oral cav-
punctum i present, needs to be removed as recurrent ity, with other locations including the axilla, chest,
in ections are common with incomplete removal. In extremities, retroperitoneum, or perineum. T e
an in ant, surgery may be delayed until age 3 to 6 majority o these lesions are asymptomatic, though
months. Figure 63-1 demonstrates the relationships large lesions can invade the oor o the mouth and
cause upper airway obstruction. Fine needle aspi-
ration (FNA) should not be used to diagnose a
lymphatic mal ormation due to the possibility o
hemorrhage into the lesion and subsequent rapid
expansion, causing airway compromise. T e only
indication or aspiration o the lymphatic mal orma-
tion is i there is a need to emergently decompress
the cyst to relieve airway obstruction. Spontaneous
hemorrhage within the mass may also occur, caus-
ing rapid enlargement with swelling and echymo-
sis. Hemorrhage predisposes the child or in ection
and antibiotics should be started. Complete excision
Firs t bra nchia l
cys t a nd fis tula
may be dif cult without involving vital structures, so
S e cond bra nchia l multiple debulking procedures may be per ormed.
fis tula C with intravenous contrast or magnetic resonance
angiography can be used when planning or resec-
tion to evaluate the extension rom one body space
to another and delineate associated vital structures.
Third bra nchia l
fis tula
Radical resection is not indicated. Sclerotherapy
has also been studied and is an option particularly
Figure 63-1 A child with a cle cyst and remnants o the rst
three branchial systems. Note the important relation to the in macrocystic lesions. Picinabil (OK-432), bleomy-
sternocleidomastoid muscle and stulas origin. (Reprinted cin, doxycycline, acetic acid, alcohol, and hypertonic
with permission rom Welch KJ, Randolph JG, Ravitch MM, saline have all been used as sclerosing agents. T ere
et al. editors. Pediatric Surgery. 4th ed. Chicago: Year Book is a small possibility o spontaneous regression in
Medical; 1986, p. 543.) low-stage macrocystic lymphatic mal ormations, so

http://surgerybook.net/
230 G EN ERAL S U RG ERY EXAM IN ATI O N AN D BO ARD REVIEW

observation and monitoring may also be an option T e only indication to operate, by dividing the
in some o these patients. muscle, is acial hemihypoplasia. Here, the ipsilat-
eral side o the ace grows more slowly than the nor-
4. B. Malignancies o the head and neck account or mal side, causing progressive asymmetry. Persistent
approximately 12% o pediatric malignancies, with an symptoms a er 1 year o conservative treatment
increasing incidence. Head and neck malignancies are may also be a possible indication or surgery. T e
more common in 15 to 18 year olds, ollowed by age etiology o torticollis is usually rom trauma, though
group < 4 years. Females aged 11 to 18 are most likely there are other possible etiologies such as ocular
to have thyroid carcinoma. However, lymphoma is the imbalance, cervical hemivertebrae, posterior ossa
number one cause o a malignant head or neck mass, tumors, deep cervical in ections, or atlanto-occipital
with up to 80% o pediatric patients with Hodgkins subluxation. An ocular imbalance is usually due to
disease having cervical lymph node involvement. superior oblique palsy, which is corrected surgically.
T ough non-Hodgkins lymphoma is more common Congenital muscular torticollis is thought to be due
in the pediatric population, it is less likely to have cer- to etal head descent or abnormal etal positioning
vical lymph node involvement. Rhabdomyosarcoma during the third trimester. rauma during delivery
is the most common pediatric solid tumor in the head is another potential cause. reatment or torticol-
or neck and typically causes symptoms through local- lis is primarily physical therapy with passive range
ized compression or in ltration. o motion and neck-stretching exercises. I torticol-
In children under the age o 5, neuroblastoma lis persists a er 6 months o physical therapy, then
is the most common solid tumor. Neuroblastoma additional workup needs to be pursued, assessing or
occurs anywhere along the sympathetic chain and other potential causes such as a congenital vertebral
may present with Horner syndrome. On imaging, mal ormation, or ocular imbalance. Botulinum toxin
calci cations will be evident. Melanoma should also (botox) can also be used to assist in treatment. T e
be on the di erential o either a primary tumor in the botox enhances the e ectiveness o stretching on the
head and neck, or as a cause or a malignant lymph a ected side, potentially making physical therapy
node. Lastly, teratomas are a rare and usually benign more bene cial.
tumor in children, o which only 0.47 to 6% present
in the head and neck. T ough rare they are important BIBLIOGRAPHY
to be aware o , given that the majority present at birth Albright J , opham AK, Reilly JS. Pediatric head and neck
with respiratory distress. Despite being a benign dis- malignancies: U.S. incidence and trends over two decades.
Arch Otolaryngol Head Neck Surg. 2002;128(6):6559.
ease, head and neck teratomas carry a 12.5% to 23% Allard RH. T e thyroglossal cyst. Head Neck Surg. 1982;5(2):
risk o intrauterine or neonatal death. 13446.
Alexander VRC, Manjaly JG, Pepper CM, et al. Head and
5. C. orticollis occurs rom brosis and shortening o neck teratomas in children: A series o 23 cases at great
the SCM and has an incidence as high as 16% in the ormond street hospital. Int J Pediatr Otorhinolaryngol.
2015;79(12):200814.
newborn population. T is brosis and shortening
Armstrong D, Pickrell K, Fetter B, Pitts W. orticollis: An anal-
causes a mass in the muscle that causes the in ant ysis o 271 cases. Plast Reconstr Surg. 1965;35:1425.
to turn his ace toward the contralateral side and tilt Chen A, Otto KJ. Head and neck surgery and oncology. Di er-
the head towards the ipsilateral shoulder. In children ential diagnosis o neck masses. In: Haughey BH, Robbins
who can walk, they may compensate or their torti- K , eds. Otolaryngology: Head & neck surgery. 5th ed.
collis by elevating one shoulder to keep their head Philadelphia, PA: Mosby Elsevier; 2003:163642.
Geddes G, Butterly MM, Patel SM, Marra S. Pediatric neck
and eyes level. Facial and cranial asymmetries can masses. Pediatr Rev. 2013;34(3):11524; quiz 125.
result rom the abnormal positioning o the childs Goins MR, Beasley MS. Pediatric neck masses. Oral Maxil-
head i the torticollis is not treated. When the in ants lo ac Surg Clin North Am. 2012;24(3):45768.
head is turned towards the contralateral side, the Hutson JM, Beasley SW. T e neck. In: T e surgical exami-
contralateral occiput presses against the bed when nation o children. 2nd ed. Heidelberg: Springer; 2013:
10121.
they are supine, and attening o the occiput then
Kekunnaya R, Isenberg SJ. E ect o strabismus surgery
leads to secondary attening o the ipsilateral ore- on torticollis caused by congenital superior oblique palsy
head, re erred to as plagiocephaly. As the torticollis in young children. Indian J Ophthalmol. 2014;62(3):
resolves the plagiocephaly also typically resolves. 3226.

http://surgerybook.net/
C H AP TER 6 3 H EAD AN D N EC k P ED I ATRI C TU M O RS 2 31

Kuint J, Horowitz Z, Kugel C, oper L, Birenbaum E, Linder N. Upchurch GR, eds. Greenf elds surgery: Scientif c principles
Laryngeal obstruction caused by lingual thyroglossal duct and practice. 5th ed. Philadelphia, PA: Lippincott Williams
cyst presenting at birth. Am J Perinatol. 1997;14(6):3536. & Wilkins; 2011:184353.
Kuo AA, ritasavit S, Graham JM, Jr. Congenital muscular Perkins JA, Manning SC, empero RM, et al. Lymphatic mal-
torticollis and positional plagiocephaly. Pediatr Rev. 2014; ormations: Review o current treatment. Otolaryngol Head
35(2):7987; quiz 87. Neck Surg. 2010;142(6):795803, 803.e1.
Laberge J, Shamberger RC, Caldamone AA, eds. Pediatric sur- Sistrunk WE. T e surgical treatment o cysts o the thyroglos-
gery. 7th ed. Philadelphia, PA: Elsevier;2012. sal tract. Ann Surg. 1920;71(2):12122.
Lillehei C. Chapter 59. Neck cysts and sinuses. In: Coran AG, Stellwagen L, Hubbard E, Chambers C, Jones KL. orticollis,
Adzick NS, Krummel M, Laberge J, Shamberger RC, acial asymmetry and plagiocephaly in normal newborns.
Caldamone AA, eds. Pediatric surgery. 7th ed. Philadelphia, Arch Dis Child. 2008;93(10):82731.
PA: Elsevier; 2012:75361. Woo RK, Albanese C . Pediatric surgery. In: Norton JA, Barie
Oldham K , Aiken JJ. Pediatric head and neck. In: Mulholland PS, Bollinger RR, et al., eds. Surgery: Basic science and clini-
MW, Lillemoe KD, Doherty GM, Maier RV, Simeone DM, cal evidence. 2nd ed. New York, NY: Springer;2008:64996.

http://surgerybook.net/
64
Pyloric Stenosis

Margaret E. Clark

A one-month old Caucasian boy is brought to his pri- D. Hypochloremic, hypokalemic metabolic alkalo-
mary care provider (PCP) by his parents due to di - sis with aciduria
culty eeding. T is is their rst child and they are very E. Hyochloremic, hyperkalemic metabolic alkalosis
concerned. T e child has been bottle- ed since birth and with aciduria
almost every time he eats he has a bout o emesis. T ey
state the emesis has gotten worse; it has remained non- 3. When is an adequate pyloromyotomy achieved?
bilious, but increasingly projectile. T e parents have
A. When the submucosa bulges into the myotomy
tried changing ormulas, but without relie . He contin-
site and both edges o the divided pyloric muscle
ues to show interest in eating, as his PCP watches him
are reely mobile.
vigorously drink rom his bottle.
B. Division through the outer longitudinal muscle
On exam the in ants vitals are normal or his age,
ollowed by division through 2/3 o the inner cir-
and he is a ebrile. He is well-appearing, though small
cular muscle. Approximately 1/3 o the circular
or his age. T e PCP attempts to do an abdominal exam
muscle should remain intact.
and thinks he eels a mass in his right upper quadrant,
C. Longitudinal incision through the pyloric
but the in ant begins to cry, making the exam more di -
sphincter ollowed by a transverse closure.
cult.
D. When you can pass an appropriately sized bou-
1. What is the most appropriate method for diagnosis gie, based on the patients weight, through the
of infantile hypertrophic pyloric stenosis (IHPS)? pylorus without any resistance.
A. Endoscopy
B. Upper gastrointestinal contrast study (UGI) 4. What is thought to be the pathogenesis of this
C. Physical exam syndrome?
D. Ultrasound A. Congenital abnormality
B. A de ciency in neuronal nitric oxide
2. What is the usual acid-base dysfunction associated C. Bottle/ ormula eeding
with this condition? D. Increased testosterone levels in the in ant
A. Hyperchloremic, hyperkalemic metabolic alka-
losis with alkalotic urine 5. What is the most common complication to occur
B. Hypochloremic, hypokalemic metabolic alkalo- with this operation, whether the operation is
sis with alkalotic urine performed open or laparoscopically?
C. Hyperchloremic, hypokalemic metabolic acidosis A. T e in ant continues to vomit or 1 to 3 days a er
with alkalotic urine the operation.

http://surgerybook.net/
C H AP TER 6 4 P y l o Ri C S TEn o S i S 233

B. Intra-operative per oration


C. Incomplete pyloromyotomy
D. Wound in ection

ANSWERS
1. D. In antile hypertrophic pyloric stenosis (IHPS) is
most common in Caucasian rst-born males. T e
di erential diagnosis o non-bilious vomiting in an
in ant includes gastroenteritis, ood allergies, gas-
troesophageal ref ux, pyloric duplication, antral web,
or increased intracranial pressure. T ough a de ni-
tive diagnosis can be made in 75% o in ants with
IHPS by care ul physical examination, it is becoming
Figure 64-1 Surgeon-per ormed US o IHPS. T e muscle
a lost skill and is technically di cult in cases present-
thickness (5.4 mm) and channel length (22.6 mm) are mea-
ing early. I the stomach is distended, aspiration with sured. GB indicates gallbladder; St = stomach; L = liver; D =
a nasogastric tube may assist in success ul palpation duodenum. Reprinted rom Journal o Pediatric Surgery
o the olive, or enlarged pylorus on physical exam. vol 44, Copeland D.R. et al., Return o the surgeon in the
However, ultrasound (US) has now become not only diagnosis o pyloric stenosis 11891192, Copyright 2009, with
the most common initial imaging technique or permission rom Elsevier.
diagnosis, but the gold standard. T e speci city and
sensitivity o US reaches 98% and 100%, respectively.
T ere continues to be some debate over the exact been described in the literature. ypically a 5% dex-
pathological pyloric measurements. trose in 0.45 normal saline with 20 mEq/L o potas-
ypically a muscle thickness o > 4.0 mm and sium has been most commonly used ollowing a
a pyloric channel length > 15 mm are consistent saline bolus o 2030 mL/kg. Indirect hyperbiliru-
with the diagnosis. However, when in ants present binemia also occurs in a small percentage o in ants
younger than 22 days, a 3.5 mm thickness may be a with IHPS. However, this invariably resolves post-
more use ul or a more accurate cuto point. A mus- operatively, so urther evaluation at presentation is
cle thickness measuring between 3.0 and 4.0 mm unnecessary. It is important to note that the majority
may be considered borderline and require repeat US o in ants now present with normal laboratory values,
a ew days later i the clinical suspicion remains high. and earlier diagnosis has led to ewer in ants with
Figure 64-1 shows that the gallbladder is an impor- electrolyte abnormalities.
tant anatomic landmark, with the pylorus o en 3. A. T e Ramstedt pyloromyotomy remains the opera-
ound nearby. T e most common reason or inabil- tion o choice, whether open or laparoscopic. An
ity to visualize the pylorus is gastric overdistention, adequate pyloromyotomy is achieved when the sub-
which displaces the pylorus dorsally. mucosa bulges into the myotomy site and the edges
o the pyloric muscle are reely mobile. Figure 64-2
2. D. In ants, i they present with electrolyte abnor-
demonstrates an adequate pyloromyotomy, with vis-
malities, present with hypochloremic, hypokalemic,
ualization o bulging submucosa.
metabolic alkalosis, and paradoxical aciduria. T ese
in ants have gastric acid loss rom emesis, renal loss 4. B. T e exact etiology o IHPS is unknown and is
o potassium, and renal retention o bicarbonate. thought to be multi actorial. T e relative contributions
It is important to correct their abnormalities, spe- o genetic and environmental actors and how they
ci cally dropping bicarbonate levels to less than 30 interact to cause the pylorus to hypertrophy during
mEq/L prior to going to the operating room in order early in ancy is unknown. T ough environmental
to avoid intraoperative and postoperative apnea. T e actors are associated with IHPS, the most commonly
chloride level should be corrected to at least 90 to accepted pathogenesis is decreased neuronal nitric
95 mEq/L. Various intravenous f uid regimens have oxide. Nitric oxide is a mediator o relaxation in the

http://surgerybook.net/
234 G En ERAl S U RG ERy EXAM i n ATi o n An D Bo ARD REVi EW

to pediatric surgeons (4.18% vs. 2.58%), though not


statistically signi cant.
T ere was a small (0.87%) but statistically sig-
ni cant increase in the incidence o incomplete
pyloromyotomy in the laparoscopic vs. open group.
In order to avoid an incomplete pyloromyotomy an
incision length o 2 cm should be used. In a rand-
omized control trial 200 patients underwent laparo-
scopic versus open pyloromyotomy. 2 patients in the
laparoscopic group and 4 patients in the open group
had post-operative wound in ections, not statistically
signi cant. Post-operative emesis is expected and is
not a complication. Greater than 90% o patients in
a series o 778 vomited or 1 to 3 days a er surgery,
and this is considered to be normal post-operatively.
Figure 64-2 Intraoperative photograph o a completed pylo- T e vomiting is thought to be secondary to gastroe-
romyotomy. T e submucosa can be seen in the depths o the sophageal ref ux, discoordination o gastric peristal-
myotomy. sis, or gastric atony. However, i requent vomiting
persists past 3 days, then an incomplete myotomy or
digestive tract. It is thought that the lack o nitric a per oration needs to be considered.
oxide synthase in the pyloric tissue causes pylor-
spasm and subsequent hypertrophy. T is has been BIBLIOGRAPHY
demonstrated in pyloric biopsies. Almost all patients
Copeland DR, Cosper GH, McMahon LE, et al. Return o the
with pyloric stenosis are not born with a hyper- surgeon in the diagnosis o pyloric stenosis. J Pediatr Surg.
trophic pylorus. T ough amilial aggregation studies 2009;44(6):118992; discussion 1192.
and twin reports point to a genetic involvement in Ein SH, Masiakos P , Ein A. T e ins and outs o pyloromy-
the etiology o IHPS, the exact heritability remains otomy: What we have learned in 35 years. Pediatr Surg Int.
unclear. Males are a ected 4 to 5 times more than 2014;30(5):467480.
Hall NJ, Simon E, Seims A, et al. Risk o incomplete pyloromy-
emales, but there is no clear explanation or this di -
otomy and mucosal per oration in open and laparoscopic
erence. No di erences in testosterone levels between pyloromyotomy. J Pediatr Surg. 2014;49(7):10836.
IHPS patients and age-matched controls were ound Krogh C, Biggar RJ, Fischer K, Lindholm M, Wohl ahrt J,
in a study o umbilical cord blood testing, and Melbye M. Bottle- eeding and the risk o pyloric stenosis.
X-linked transmission patterns have not been shown Pediatrics. 2012;130(4):e9439.
to explain why IHPS occurs more in males. Breast- Langer JC, o . Does pediatric surgical specialty train-
ing a ect outcome a er ramstedt pyloromyotomy? A
eeding has been indicated as a potentially protective population-based study. Pediatrics. 2004;113(5):13427.
actor but bottle- eeding is not a direct cause or etiol- Leaphart CL, Borland K, Kane D, Hackam DJ. Hypertro-
ogy o IHPS. phic pyloric stenosis in newborns younger than 21 days:
Remodeling the path o surgical intervention. J Pediatr
5. B. T e most common intra-operative complication Surg. 2008;43(6):9981001.
is per oration. A distal per oration can be prevented McAteer JP, Ledbetter DJ, Goldin AB. Role o bottle eeding in
by not carrying the distal part o the myotomy past the etiology o hypertrophic pyloric stenosis. JAMA Pedi-
atr. 2013;167(12):11439.
the pyloric vein o Mayo. T e incidence o duodenal Niedzielski J, Kobielski A, Sokal J, Krakos M. Accuracy o
per oration when pyloromyotomy was per ormed sonographic criteria in the decision or surgical treatment
by general surgeons was ound to be almost 4 times in in antile hypertrophic pyloric stenosis. Arch Med Sci.
that o pediatric surgeons (relative risk 3.65; 95% CI 2011;7(3):50811.
1.439.32). However, when adjusted or surgical vol- Ostlie DJ, Woodall CE, Wade KR, et al. An e ective pyloromy-
otomy length in in ants undergoing laparoscopic pyloro-
ume this was not signi cant. In a large series o 1777
myotomy. Surgery. 2004;136(4):82732.
in ants only 4 required repeat surgery because o an Peters B, Oomen MW, Bakx R, Benninga MA. Advances in
incomplete pyloromyotomy. General surgeons have in antile hypertrophic pyloric stenosis. Expert Rev Gastro-
an overall higher complication rate when compared enterol Hepatol. 2014;8(5):53341.

http://surgerybook.net/
C H AP TER 6 4 P y l o Ri C S TEn o S i S 235

Sato , Oldham K . Chapter 107. Pediatric abdomen. In: prospective, randomized trial. Ann Surg. 2006;244(3):
Mulholland MW, Lillemoe KD, Doherty GM, Maier RV, 36370.
Simeone DM, Upchurch GR, eds. Greenf elds surgery: Scien- utay GJ, Capraro G, Spirko B, Garb J, Smithline H. Electrolyte
tif c principles and practice. 5th ed. Philadelphia, PA: Lippincott pro le o pediatric patients with hypertrophic pyloric ste-
Williams & Wilkins; 2011:18851946. nosis. Pediatr Emerg Care. 2013;29(4):4658.
Schwartz MZ. Chapter 78. Hypertrophic pyloric stenosis. Vanderwinden JM, Mailleux P, Schi mann SN,Vanderhaeghen JJ,
In: Coran AG, Caldamone A, Adzick NS, Krummel M, De Laet MH. Nitric oxide synthase activity in in antile hyper-
Laberge J, Shamberger R, eds. Pediatric surgery. 7th ed. trophic pyloric stenosis. N Engl J Med. 1992;327(8):51115.
Philadelphia, PA: Elsevier; 2012:10218. Ward E, Easley D, Pohl J. Previously unsuspected in antile
St Peter SD, Holcomb GW, Calkins CM, et al. Open ver- hypertrophic pyloric stenosis diagnosed by endoscopy. Dig
sus laparoscopic pyloromyotomy or pyloric stenosis: A Dis Sci. 2008;53(4):9468.

http://surgerybook.net/
65
Hirschsprungs Disease

Mary J. Edwards

1. A newborn male in ant has di culty eeding the C. Cystic brosis


rst day o li e. T is progresses to abdominal D. Duodenal atresia
distension and bilious vomiting. T e ollowing E. Intestinal malrotation
plain X-ray is obtained on the second day o li e.
2. T e ollowing contrast enema is obtained:

T is diagnosis:
A. Requires tissue biopsy
B. Requires surgery prior to resumption o oral
eeds
T e di erential diagnosis includes which o the C. Is not associated with chromosomal anomalies
ollowing: D. ypically has a amily history o similar prob-
A. Situs inversus lems on the athers side
B. Pyloric stenosis E. Is cured with a success ul surgery

http://surgerybook.net/
C H AP TER 6 5 H i Rs C H s P Ru n g s D i s EAs E 237

3. echnical considerations o the corrective surgery tion begins with a good physical exam. Patients with
or Hirschprungs disease include: imper orate anus or hernias should be diagnosed
A. aking multiple biopsies to determine exactly readily at this time. Patients with Hirschsprungs dis-
when the rst ew ganglion cells are seen, in ease will o en have explosive output o stool ollow-
order to resect as little colon as possible ing rectal exam. T e next diagnostic test should be a
B. Resection rom the level o the dentate line to the contrast enema with either isotonic or mildly hyper-
ganglionated intestine tonic water soluble contrast. T is is typically thera-
C. Routinely per orming a protective colostomy or peutic and diagnostic or small le colon syndrome,
ileostomy meconium plug, and meconium ileus.
D. A tension ree anastomosis It is o en (75%) diagnostic o Hirschsprungs
disease with correct identi cation o the transition
4. A 5-year-old patient who underwent a pull zone. Findings o a microcolon without obstruction
through or Hirschprungs as a newborn and did is very consistent with an ileal atresia. Plain lm nd-
well or years, now presents with a 6 month history ings o malrotation can be normal, consistent with an
o abdominal distension and ecal incontinence. ileus, or a proximal obstruction. Duodenal atresia on
T e most likely diagnosis is: a plain lm classically presents with a double bub-
A. Sphincter injury at the time o surgery ble sign that shows air in the duodenum and stom-
B. ransition zone pull through at the time o ach. Pyloric stenosis would most likely look normal
surgery on plain lm.
C. Hirschprungs associated enterocolitis
D. Constipation 2. A. T e enema is consistent with a diagnosis o
E. Anastomotic stricture Hirschprungs disease with a transition zone in the
rectosigmoid area. While contrast enema is accurate
5. A 6-month-old patient with Downs syndrome approximately 75% o the time, tissue diagnosis or
and Hirschprungs disease underwent a leveling anorectal manometry are the only ways to de ni-
colostomy (at the level o normal ganglionated tively make the diagnosis. Anorectal manometry is
bowel) shortly af er birth, ollowed by pull- not practical in the newborn period, but is a use ul
through one month ago. He presents now with study in older children or adults. A suction rectal
ever, lethargy, abdominal distension, and bilious biopsy at the bedside can sample the submucosa in
vomiting. Plain lms reveal no ree air, but multiple an in ant and does not require anesthesia or sedation.
dilated loops o bowel. Rectal exam results in an Hirschsprungs disease is due to arrest o the caudal
explosive output o bloody stained, oul smelling descent o ganglion cells which coordinate intestinal
stool. Which o the ollowing is required or the relaxation. Aganglionic bowel does not relax properly
management o this patient? to allow propagation o intestinal contents and thus
A. C scan to evaluate or intrabdominal abscess causes a unctional obstruction. T e biopsy should
B. Immediate surgical intervention or a revision o sample the submucosa 1 to 2 cm above the dentate
the pull-through line.
C. Lavage o saline through the anus Findings o a lack o ganglion cells and nerve
D. Blood trans usion trunk hypertrophy are diagnostic. Patients with tran-
E. Nasogastric decompression sition zones in the rectosigmoid can usually be tem-
porized by rectal irrigations which allows or delayed
surgical correction at several months o age, although
ANSWERS
surgery in the newborn period is also an acceptable
1. C. T e lm is consistent with a distal intestinal treatment strategy. Most cases o Hirschprungs dis-
obstruction. T e di erential diagnosis or this in the ease are sporadic, although some are amilial. Several
newborn period includes cystic brosis, incarcerated genes have been implicated, most notably the ret
inguinal hernia,, small le colon syndrome, meco- proto-oncogene. Hirschprungs has a strong associa-
nium ileus, meconium plug syndrome, Hirschsprungs tion with Downs syndrome, Waadenburg-Shah, and
disease, imper orate anus, and ileal or colonic atresia. others. Even patients who have success ul surgeries
In a stable patient without peritonitis the evalua- have a high likelihood o ecal soiling (60%) and

http://surgerybook.net/
238 g En ERAL s u Rg ERY EXAM i n ATi O n An D BO ARD REVi EW

constipation requiring chronic laxative use (30%) in cations. T is is one o the most eared complications
the long term. Surgery essentially converts the dis- o Hirschsprungs, as i treatment is not initiated
ease to ultrashort segment Hirschsprungs while tak- quickly patients may become severely ill and die. T e
ing away the rectal reservoir. etiology is bacterial translocation through the wall o
the colon, presumably due to an ine ective mucosal
3. D. Multiple intraoperative biopsies are required, but barrier in the colon. reatment involves uid resus-
in order to avoid pull-through o the transition zone citation, antibiotics, nutritional support and most
(where the bowel transitions rom ganglionated to importantly, rectal irrigations. T is is done transan-
aganglionated), a level where plenti ul ganglion cells ally with warm saline. Unlike an enema, a catheter is
are seen and no nerve trunk hypertrophy is present used to evacuate stool and gas, while gently irrigat-
is the goal. Some authors advocate resection up to 10 ing stool out o the colon. In severe cases, colostomy
to 12 cm above this level to more de nitively ensure may be required. Patients can present with HEC
normal bowel is pulled through. It is critical to begin both be ore and up to 2 years a er pull-through. T is
the dissection at least 0.5 to 1.0 cm above the dentate clinical presentation is much more consistent with
line in a newborn or 1.0 to 2.0 cm in an older child to enterocolitis than an abscess. A C scan is not typi-
preserve the anal canal and allow uture continence. cally utilized to evaluate or intra-abdominal abscess
A protective ileostomy or colostomy is not typically in in ants. T e lack o intra-abdominal at makes
done unless there is a technical concern about the it dif cult to discern an abscess cavity rom a uid
anastomosis. T e surgical principles o per orming lled loop o bowel. I imaging is needed to evaluate
a tension ree, well vascularized anastomosis should or this, ultrasound is typically a better study. Neither
always be ollowed. blood trans usions or nasogastric decompression is
required or treatment but they may be utilized.
4. D. Persistent obstructive symptoms ollowing a
pull-through are not uncommon and all o the
answers listed are possibilities. However, a patient BIBLIOGRAPHY
that has done well or years and suddenly presents DeLa orre L, Langer JC. ransanal pullthrough or
with distension and ecal soiling is likely to be con- Hirschsprung Disease: echnique, controversies, pearls,
stipated. T irty to eighty- ve percent o patients with pit alls and an organized approach to the management o
postoperative obstructive symptoms. Sem Ped Surg. 2010;
Hirschprungs disease will have constipation ollow-
19:96106.
ing pull-through that may require chronic laxatives, Engum SA, Gros eld JL. Long- erm results o treatment o
and 60% to 75% will have intermittent ecal soiling. Hirschsprungs Disease. Sem Ped Surg. 2004;13;27385.
Fecal soiling tends to become worse during periods Haricharan RN, Georgeson KE. Hirschprung disease. Sem Ped
o severe constipation. T ese patients are best evalu- Surg. 2008;17:26675.
ated with a physical exam and a plain X-ray. I these Kapur RP, Kennedy AJ. ransition Zone pull through: surgical
pathology considerations. Sem Ped Surg. 2012;21:291301.
are consistent with constipation then a clean out with Langer JC. Hirschsprungs Disease. In: Oldham K , Colom-
enemas or oral laxatives can be initiated and inconti- bani PM, Foglia RP, Skinner MA, eds. Principles and
nence will likely improve. Persistent obstruction a er Practice of Pediatric Surgery, 2nd ed. Philadelphia, PA:
appropriate management or no signi cant improve- Lippincott Williams and Wilkins: 2005:134764.
ment a er pull-through should be evaluated with Levitt MA, Dickie BA, Pena A. Evaluation o the patient with
Hirschprung disease who is not doing well a er a pull
contrast enema, exam under anesthesia and repeat
through procedure. Sem Ped Surg. 2010;19:14653.
rectal biopsy. Mundt E, Bates MD. Genetics o Hirschprung disease and
anorectal mal ormations. Sem Ped Surg. 2010;19:10117.
5. C. T is patient has Hirschprungs associated enter- Mychaliska GB. Introduction to Neonatal Intestinal Obstruc-
ocolitis (HEC). Risk actors or this condition are tions. In: Oldham K , Colombani PM, Foglia RP, Skinner
Downs syndrome, diagnosis a er one week o li e, MA, eds. Principles and Practice of Pediatric Surgery, 2nd ed.
anatomic obstruction, and a history o previous HEC. Philadelphia, PA: Lippincott Williams and Wilkins: 2005:
12221226.
Obstruction can be a result o an anastomotic stric-
Rintala RJ, Pakarinen MP. Long term outcomes o
ture, swelling o the anastomosis in the early postop Hirschprungss disease. Sem Ped Surg. 2012;21:33643.
period and transition zone pull-through. However, Vieten D, Spicer R. Enterocolitis ollowing Hirschprungs Dis-
HEC can also occur in the absence o these compli- ease. Sem Ped Surg. 2004;13:26372.

http://surgerybook.net/
66
Omphalocele/Gastroschisis

Paul Wetstein

You are called to the delivery room to assist in man- 3. Which of the following is the most important
agement o the delivery o a child with a known aspect regarding the management of a newborn
anterior abdominal wall de ect and maternal polyhy- with omphalocele?
dramnios. A. Protection o the sac to help prevent heat loss
and insensible uid loss
1. Regarding the pathophysiology of pediatric B. Establishing IV access
abdominal wall defects, which of the following is C. Gastric (OG) tube placement or bowel decom-
correct? pression
A. Polyhydramnios and intestinal atresia are not D. Airway stabilization
associated with omphalocele or gastroschisis. E. Attempting to primarily repair all de ects
B. Gastroschisis is commonly associated with cardiac
and chromosomal anomalies. 4. Which of the following is correct regarding the
C. A giant omphalocele is more likely to be associ- management of a newborn with gastroschisis?
ated with etal aneuploidy than gastroschisis. A. An associated mid-gut volvulus is not a concern.
D. Intestinal nonrotation is not associated with gas- B. Sequential closures should be avoided.
troschisis or omphalocele. C. T e bowel tends to be less damaged than that o
E. Herniated liver indicates the de ect is most likely the omphalocele.
a omphalocele. D. Surgical management is urgent.
E. T e associated abnormalities are the bigger
2. Which of the following associations is correct concern.
regarding congenital abdominal wall defects?
A. Since gastroscisis is commonly ound o to the 5. Omphalocele is associated with many syndromes
midline, liver is o en seen herniating through like Beckwith-Wiedemann, the pentalogy of
the de ect. Cantrell, and CHARGE. Which of the following is
B. T e most important prognostic actor or a component of the pentalogy of Cantrell?
gastroscisis is the amount o bowel herniated. A. Coloboma
C. T e most important prognostic actor or ompha- B. Gigantism
locele is presence o other associated anomalies. C. Ectopic cordis
D. Omphaloceles are usually located at the umbili- D. Genital hypoplasia
cus. E. Ear abnormalities

http://surgerybook.net/
240 G EN ERAL S U RG ERY EXAM I N ATIO N AN D BO ARD REVI EW

ANSWERS sequential reduction and staged closure with a silo or


other alternative.
1. E. Be ore 12 weeks o gestation, the di erentiation
o an omphalocele and a physiologic midgut hernia- 4. D. While one should be cautious in preserving the
tion cannot be made in the absence o herniated liver. omphalocele sac, extreme caution should be main-
Herniated liver appreciated on prenatal ultrasound tained when handling the bowel in gastroschisis
is not a normal physiologic f nding. Intestinal atre- because there is no sac. In gastroschisis patients,
sia is associated with gastroschisis in 10% to 20% the bowel may be extremely edematous and riable.
o cases. Omphalocele has also been reported to be It may also be malrotated and prone to becoming a
associated with intestinal atresias. Omphalocele has volvulus. Under most circumstances, gastroschisis
been associated with structural anomalies, chro- intestines are placed in a warm saline f lled plas-
mosomal anomalies, and several syndromes. Small tic organ bags, secured at the nipple line. An initial
omphalocele sacs with an intracorporeal liver have care ul attempt and reduction and operative closure
a stronger association with chromosomal anomalies is warranted by a trained surgeon, but a sequential
than larger de ects. reduction and staged closure may be necessary.

5. C. T e pentalogy o Cantrell is comprised o ompha-


2. D. Giant omphalocele is associated with herniated locele, ectopia cordis, intracardiac de ect, sternal cle ,
liver. Di erentiating characteristics between gastro- and an anterior diaphragmatic hernia. Beckwith-
schisis and omphalocele are summarized in able 66-1 Weidemann syndrome composes o an umbilical de ect
below. which could be either a hernia or an omphalocele, mac-
roglossia and gigantism. Coloboma (C) is a component
3. A. Initial resuscitation centers around ensuring o the CHARGE syndrome which also includes (H)
a stabilized airway, uid resuscitation, intestinal Heart Disease, (A) atresia choanae, (R) retarded growth,
decompression via gastric tube, IV antibiotics, and development and/or CNS anomalies, (G) genital hypo-
preventing heat and insensible uid loss. Protection plasia, (E) ear anomalies and/or dea ness.
o the sac is imperative. Some advocate or wrapping
the bowel in thermoneutral, sterile saline soaked
gauze and covering with a bowel bag. Primary clo- BIBLIOGRAPHY
sure o small and medium sized de ects is pre erred Chung DH. Pediatric Surgery. In: ownsend Jr. CM, Beau-
but attempts at closure o larger de ects will need a champ RD, Evers BM, Mattox KL. Sabiston extbook of
Surgery: T e Biological Basis of Modern Surgical Practice.
19th ed. Philadelphia, PA: Elsevier Saunders;2012:
Curtis JA, Watson L. Sonographic diagnosis o omphalocele
Table 66-1 DIFFERENTIATING CHARACTERISTICS in the f rst trimester o etal gestation. J Ultrasound Med.
BETWEEN OMPHALOCELE AND GASTROSCHISIS 1988;7:97.
Dono rio M , Moon-Grady AJ, Hornberger LK, Copel JA,
Characteristic Omphalocele Gastroschisis Sklansky MS, Abuhamad A, et al. on behal o the American
Heart Association Adults with Congenital Heart Disease
Herniated Bowel liver Bowel only Joint Committee o the Council on Cardiovascular Disease
viscera in the Young and Council on Clinical Cardiology, Coun-
Sac Present Absent cil on Cardiovascular Surgery and Anesthesia, and Coun-
cil on Cardiovascular and Stroke Nursing. Diagnosis and
Associated Common (50%) Uncommon
treatment o etal cardiac disease: A scientif c statement
anomalies (< 10%)
rom the American Heart Association. Circulation. May 27,
Location o Umbilicus Right o 2014;129(21):2183242. doi: 10.1161/01.cir.0000437597.
de ect umbilicus 44550.5d. Epub April 24, 2014.
Mode o delivery Vaginal/cesarean Vaginal Kitchanan S, Patole SK, Muller R, Whitehall JS. Neonatal out-
come o gastroschisis and exomphalos: A 10-year review.
Surgical Nonurgent Urgent J Paediatr Child Health. 2000;36(5):428.
management Klein MD. Congenital De ects o the Abdominal Wall. In:
Prognostic Associated Condition o Coran AG, Caldamone A, Adzick NS, Krummel M,
actors anomalies bowel Laberge JM, and Shamberger R, eds. Pediatric Surgery.
7th ed. Philadelphia, PA: Elsevier Saunders; 2012:

http://surgerybook.net/
C H AP TER 6 6 O M P H ALO C ELE/ G AS TRO S C H IS I S 2 41

Nyberg DA, Fitzsimmons J, Mack LA, Hughes M, Pretorius anomalies: CHARGE association. J Pediatr. August 1981;
DH, Hickok D, et al. Chromosomal abnormalities in etuses 99(2):2237.
with omphalocele. Signif cance o omphalocele contents. Snyder CL, Miller KA, Sharp RJ, Murphy JP, Andrews WA,
J Ultrasound Med. 1989;8(6):299. Holcomb III GW, et al. Management o intestinal atre-
Pagon RA, Graham JM Jr, Zonana J, Yong SL. Coloboma, con- sia in patients with gastroschisis. J Pediatr Surg. October
genital heart disease, and choanal atresia with multiple 2001;36(10):1542154.

http://surgerybook.net/
This page intentionally left blank

http://surgerybook.net/
r auma Sur ger y a nd Cr it ic a l
Ca r e
Matthew J. Martin

http://surgerybook.net/
This page intentionally left blank

http://surgerybook.net/
67
rauma and Critical
CareRoadside Bomb
M h wE k M h w J. M

During the annual Memorial Day parade o your home-


town, a large explosion occurs along the parade route, with
multiple injured persons and atalities at the scene. Pre-
hospital EMS providers, rst responders, and bystanders
begin to administer rst aid to those injured and start
trans er o patients to the local hospital. As an on-call
general surgeon, you and your colleagues stand ready in
the emergency department while preparing or a potential
mass casualty event. Hospital administrators have acti-
vated the disaster/mass casualty plan, triage o cers are
designated, and the incident command plan is in e ect.

1. A p -hosp p ov o s j
w h b ow k p o
p wo s w h
bo b h (F 67-1). T f s p o
h o h sp sho b ?
A. Airway
B. Breathing
C. Circulation/control o li e-threatening hemor-
rhage
D. Disability
E. Evacuation
Figure 67-1 Representative injury pattern o roadside bomb
2. Wh h o h o ow p so h o h so or improvised explosive device.
p s o ? E. Intra-cranial hemorrhageReversal o hyperco-
A. Junctional hemorrhage ourniquet placed aguable state and neurosurgical intervention
directly over the injury
B. Extremity hemorrhageHemostatic dressing 3. T p p so o o s
and direct pressure wh h o h o ow ?
C. runcal hemorrhagePermissive hypotension A. Primary ocus upon control o hemorrhage only
and hemostatic resuscitation B. Avoiding the use o temporary abdominal clo-
D. Pelvic hemorrhageEarly use o rFactor VIIa sures

http://surgerybook.net/
246 G EN ERAL S U RG ERY EXAM IN ATI O N AN D BO ARD REVIEW

C. Appropriate triage based upon injuries and avail- or li e-threatening issues such as airway obstruction,
able resources tension pneumothorax, or an open pneumothorax
D. Colostomy or all colon injuries (sucking chest wound).
E. De nitive repair o all abdominal injuries
2. C. Major extremity hemorrhage can be e ectively
4. T o p o o o o h os controlled in both the pre-hospital and hospital set-
s s o s wh h o h o ow ? ting using an e ective proximal tourniquet. Appli-
A. Limited use o colloid solution resuscitation cation o tourniquets to control major hemorrhage
B. 3:1:1 PRBC:FFP:Platelet trans usion strategy should be placed as distal as possible yet proximal to
C. Empiric trans usion o cryoprecipitate the site o injury, to control hemorrhage and limit the
D. Administration o tranexamic acid ( XA) or extent o tissue ischemia. Hasty tourniquets applied
treatment o hyper brinolysis in combat under re or prior to moving a patient to
a sa er location may be placed expediently anywhere
5. P b s j s wh h o h proximal on the injured extremity and later moved
o ow ? distally. Approximately one-third o bleeding deaths
A. Damage to gas lled organs due to extreme pres- a er explosive events are due to extremity hemor-
sure changes rhage, with the remaining two-thirds attributed
B. runcal injury sustained as victim is thrown to junctional bleeding or non-compressible trun-
through the air by blast cal hemorrhage (Figure 67-2). Junctional injuries
C. Crush injuries rom structural collapse around are de ned as injuries to the groin, axilla, neck, or
the patient perineum. T ese injuries present major challenges
D. Penetrating injury rom debris and ragments to hemorrhage control as damaged deep vascular
propelled by the blast orce structures transition rom the extremities to major
cavities beyond the e ective reach o a proximal
tourniquet. Direct pressure and topical hemostatic
ANSWERS adjuncts are o en the only currently available treat-
1. C. raditional teaching o advanced cardiac li e ments o potential bene t be ore proximal surgical
support (ACLS) and advanced trauma li e support control can be achieved. Non-compressible trun-
(A LS) concepts stress the ABCDE mantra (air- cal hemorrhage is the most eared and atal type o
way, breathing, circulation, disability, and exposure), bleeding as it is not amenable to pre-hospital hem-
in which airway control is paramount. However, in orrhage control measures, and requires higher-level
the setting o combat injuries and civilian catastro- interventions such as surgery or angioembolization.
phes such as blast injuries, immediate control o Signi cant pelvic hemorrhage may result rom
li e-threatening hemorrhage takes precedence as complex bony injuries and damage to the pelvic vas-
atal hemorrhage can rapidly occur while addressing culature. reatment o pelvic ractures may include
other elements o the primary survey. Hemorrhage pelvic sheeting, commercial or improvised binders,
remains the leading cause o potentially preventable and orms o external xation, that aim to stabilize
morbidity and mortality in both military and civilian the pelvic ring. Surgical control o pelvic hemorrhage
trauma, rein orcing the importance o pre-hospital
e orts to control bleeding and prevent exsanguina-
tion a er severe injury. T e experiences gained dur-
ing combat operations in Iraq and A ghanistan have
led to a change in the priorities o treatment that are
taught to combat medics and rst-responders.
As the most likely cause o potentially preventable
death in these scenarios is hemorrhage, the standard
A LS approach o ocusing rst on the airway has
been changed to a primary ocus on C, or the control
o hemorrhage, as the rst priority. T e next priori- Figure 67-2 Distribution o preventable bleeding deaths
ties would be on assessing the airway and breathing rom battle eld wounds by the site o hemorrhage.

http://surgerybook.net/
C H AP TER 6 7 TRAU M A AN D C RI TI C AL C ARE RO AD S ID E BO M B 247

includes open and catheter based therapies as well obtained, as well as replacing shed-blood volume
as pre-peritoneal pelvic packing to tamponade hem- with a balanced blood product based resuscitation
orrhage. Permissive hypotension is the concept that that restores the capacity to carry and deliver oxygen
avoiding normal or elevated blood pressure in a and addresses derangements o the coagulation sys-
bleeding patient prior to surgical hemorrhage con- tem. DCR research has ocused upon improved mor-
trol limits blood loss. bidity and mortality with limitation o crystalloid
T e goal is to maintain an adequate arterial pres- in usion to prevent hemodilution (colloid solutions
sure or critical organ per usion while limiting over- are, in general, not advocated or trauma resusci-
pressurization rom vigorous f uid resuscitation that tations), acidosis and worsening o coagulopathy,
may exacerbate hemorrhage. Although there was in avor o trans usion ratios approaching 1:1:1 o
initial interest in the use o activated recombinant packed red blood to resh rozen plasma and plate-
actor VII (rFactor VIIa) as an adjunctive therapy lets. Additional targeted treatment o speci c coagu-
or bleeding, subsequent studies have ound little to lation abnormalities based upon traditional studies
no bene t and it also would not be used as a pre- (P /INR, P , brinogen) and point o care test-
hospital therapy or pelvic bleeding. T e treatment ing such as thromboelastography is recommended,
o traumatic intracranial hemorrhage begins with to avoid unnecessary trans usion and appropriate
reversal o any identi ed coagulopathy, not hyperco- resource utilization.
aguble state, treatment o associated cerebral edema, Permissive hypotension is the concept that avoid-
and possible neurosurgical interventions. ing normal or elevated blood pressure in a bleeding
patient prior to surgical hemorrhage control limits
3. C. Damage control surgery (DCS) is a concept that blood loss. T e goal is to maintain an adequate arterial
includes the principles o limiting urther physi- pressure or critical organ per usion while limiting
ologic insult to a severely injured patient through the over-pressurization rom vigorous f uid resuscitation
rapid surgical control o li e-threatening hemorrhage that may exacerbate hemorrhage. Previous studies in
and enteric spillage, to minimize urther bleeding, civilian trauma patients have con rmed a avorable
restore tissue per usion, and prevent sepsis. T e goal survival advantage by limiting pre-hospital resuscita-
is to avoid prolonged, initial operative procedures tion but heterogeneity in clinical trials such as opti-
that may urther exacerbate hypothermia, coagu- mal blood pressure target and f uid resuscitation type
lopathy, and/or acidosis. emporary closures o the have yet to yield de nitive conclusions.
abdomen and thorax may be employed to shorten Current research has identi ed early derange-
the procedure length and allow or urgent or planned ments o the coagulation system that appear to con-
re-operation in this population. Once bleeding and er an increased risk o morbidity and mortality a er
gastrointestinal spillage are controlled, there are no trauma, termed the acute coagulopathy o trauma.
urther attempts to address other non-li e-threatening Hyper brinolysis is a key component o this condi-
abdominal injuries at the initial surgery. tion, and targeted pharmacologic intervention with
T e operation is terminated and the patient is the anti brinolytic agent, tranexamic acid ( XA),
returned to the ICU or resuscitation and restoration has been linked to a survival advantage in several
o normal physiology and coagulation. Following studies o bleeding trauma patients. T e use o XA
physiologic restoration these patients are taken back in the setting o major hemorrhage and massive
to the operating room or de nitive treatments and trans usion situations is now recommended in the
closure. In mass casualty situations and resource- early treatment (< 3 hours rom injury) in civilian
constrained environments, DCS allows limited and combat injuries.
surgical personnel to provide critical surgical proce-
dures to severely injured patients rapidly, and then 5. A. Blast injuries are traditionally classi ed based
either evacuate them to higher levels o care or clear upon the discrete mechanism o injury involved.
the operating room or additional patients. Primary blast injuries involve damage to gas- lled
organs such as the intestine, lungs, and middle ear
4. D. Damage control resuscitation (DCR) or hemo- as well as the brain. T ese injuries result rom over-
static resuscitation is the concept o limiting urther pressurization due to the blast orce. Rupture o the
blood loss be ore surgical hemorrhage control is tympanic membrane is a requently encountered

http://surgerybook.net/
248 G EN ERAL S U RG ERY EXAM I N ATIO N AN D BO ARD REVI EW

primary blast injury, but the absence does not rule out Cooper GJ, Maynard RL, Cross NL, et al. Casualties rom ter-
other types o blast injury. Secondary blast injuries rorist bombings. J Trauma. 1983;23(11):95567.
result rom injury due to f ying debris and ragments Cothren CC, Osborn PM, Moore EE, et al. Preperitoneal pel-
vic packing or hemodynamically unstable pelvic ractures:
that are propelled by the blast. T ese are the most re- A paradigm shi . J Trauma. 2007;62(4):83439.
quently sustained type o blast injuries. ertiary inju- Cotton BA, Reddy N, Hatch QM, et al. Damage control resus-
ries occur when a person is thrown through the air by citation is associated with a reduction in resuscitation vol-
the blast, while Quaternary injuries include other blast umes and improvement in survival in 390 damage control
e ects such as burn injury, inhalation o toxic gases, or laparotomy patients. Ann Surg. 2011;254(4):598605.
CRASH-2 trial collaborators, Shakur H, Roberts I,
injury due to collapse o structures around the person.
Bautista R, et al. E ects o tranexamic acid on death, vas-
Lastly, quinary injuries are the result o contamination cular occlusive events, and blood trans usion in trauma
by chemical, radiologic, or biologic materials rom the patients with signi cant haemorrhage (CRASH-2): A ran-
blast itsel or other injured persons. domised, placebo-controlled trial. Lancet. 2010;376:2332.
In addition to the injury classi cation described DePalma RG, Burris DG, Champion HR, et al. Blast injuries.
above, the severity o injury sustained a er a blast N Engl J Med. 2005;352(13):133542.
Eastridge BJ, Mabry RL, Seguin P, Cantrell J, ops , Uribe P,
mechanism depends upon the patients proximity to et al. Death on the battle eld (20012011): Implications or
the blast, the energy o the blast itsel , and whether the uture o combat casualty care. J Trauma Acute Care
or not the blast occurred in a closed space. Persons Surg. 2012;73:S4317.
injured by a blast in an enclosed space such as a Kauvar DS, Le ering R, Wade CE. Impact o hemorrhage on
vehicle or building requently sustain a higher rate trauma outcome: An overview o epidemiology, clinical
presentations, and theraputic considerations. J Trauma.
o lethal primary blast injury as well as increased re-
2006;60:S3S11.
quency o secondary penetrating injuries. Kwan I, Bunn F, Chinnock P, et al. iming and volume o f uid
administration or patients with bleeding. Cochrane Data-
BIBLIOGRAPHY base Syst Rev. 2003;(3):CD002245.
Beekley AC. Damage control resuscitation: a sensible Morrison CA, Carrick MM, Norman MA, et al. Hypotensive
approach to the exsanguinating surgical patient. Crit Care resuscitation strategy reduces trans usion requirements
Med. 2008;36(7):S26774. and severe postoperative coagulopathy in trauma patients
Blackbourne LH. Combat damage control surgery. Crit Care with hemorrhagic shock: Preliminary results o a random-
Med. 2008;36(7):S30410. ized controlled trial. J Trauma. 2011;70(3):65263.
Borgman MA, Spinella PC, Perkins JG, et al. T e ratio o Morrisson JJ, Dubose JJ, Rasmussen E, et al. Military applica-
blood products trans used a ects mortality in patients tion o tranexamic acid in trauma emergency resuscitation
receiving massive trans usions at a combat support hospi- (MA ERs) study. Arch Surg. 2012;147:1139.
tal. J Trauma. 2007;63(4):80513. Rommens PM, Ho mann A, Hessmann MH, Management
Bulger EM, Snyder D, Schoelles K, et al. An evidence-based o acute hemorrhage in pelvic trauma: An overview. Eur J
guideline or external hemorrhage control: American col- Trauma Emerg Surg. 2010;2:9199.
lege o surgeons committee on trauma. Prehosp Emerg Rotondo MF, Schwab CW, McGonigal MD, et al. Damage
Care. 2014;18(2):16373. control: An approach or improved survival in exsangui-
Butler FK Jr, Holcomb JB, Giebner SD, et al. actical combat nating penetrating abdominal injury. J Trauma. 1993;35(3):
casualty care 2007: Evolving concepts and battle eld expe- 37582.
rience. Mil Med. 2007;172(11):S1S19.

http://surgerybook.net/
68
Kni e Wounds

Laura Mazer

A 35-year-old man is brought to the emergency depart- D. ake the patient immediately to the operating
ment a er an altercation at a bar. He has a 3 cm stab room or thoracotomy.
wound to his right chest, approximately 5 cm lateral to E. Initiate a massive trans usion protocol to stabi-
and just superior to his nipple. He is able to tell you that lize the patient or computerized tomography
the wound is rom a kni e. He is oriented and appropri- (C ) scan to identi y the source o the hypoten-
ate, although intoxicated, and complains loudly o pain sion.
in his right chest. His initial vital signs are as ollows:
HR 96, BP 110/63, RR 20, 98% saturation on room air. 3. Which o the ollowing is a correct indicator or
operative intervention paired with an appropriate
1. What is your rst step in the management o this operative approach or this patient?
patient? A. Greater than 500 cc blood rom the chest tube
A. Per orm a tube thoracostomy. upon initial placementright posterolateral
B. Per orm the primary survey and examine or any thoracotomy in the operating room.
other wounds. B. Cardiac arrest in the trauma bayle anterolat-
C. Per orm a FAS ( ocused assessment with eral thoracotomy in the emergency room.
sonography or trauma) scan to identi y intra- C. Persistent trans usion requirements to maintain
abdominal injury. stable blood pressuremedian sternotomy in
D. Obtain a chest radiograph to identi y presence o the emergency room.
pneumothorax or hemothorax. D. Second entry wound identi ed below the cos-
tal margintransverse anterior thoracotomy
2. Despite luid resuscitation, the patients blood (clamshell incision) in the operating room.
pressure suddenly becomes 60/palpable. His E. An entry wound medial to the midclavicular
trachea is midline and his breath sounds are linemedian sternotomy in the emergency room.
reduced over the right hemithorax, with dullness
to percussion. What is the best next step in 4. I this patients wound were located in the neck
management? above the cricoid cartilage (but below the angle o
A. Place a 36F chest tube into the right pleural cav- the mandible), which o the ollowing diagnostic
ity at the 2nd interspace in the midaxillary line. tests would be de nitely indicated?
B. Place a 36F chest tube into the right pleural cav- A. None; manage expectantly with admission or
ity at the 5th interspace in the mid- or anterior serial exams.
axillary line. B. Computed tomography (C ) o the neck with
C. Place an 18 g angiocath into the 2nd intercostal C angiography (C A) o the cervical vessels.
space at the midclavicular line. C. C A or angiography plus bronchoscopy.

http://surgerybook.net/
250 G EN ERAL S U RG ERY EXAM I N ATIO N AN D BO ARD REVI EW

D. C A or angiography, bronchoscopy, and esopha- sounds with distended neck veins. However, these
gogastroduodenoscopy (EGD). can be late ndings and may be di cult to appreciate
in a busy and noisy trauma bay. I a tension pneumo-
5. T e patient has 800 cc initial drainage rom the thorax is suspected, the rst step would be placing
right-sided chest tube, which then slows down an large bore (14 or 18 gauge) angiocath to needle
over the next ew hours. He is admitted to the f oor decompress the thoracic cavity. T e diagnosis would
on telemetry or monitoring, and remains stable. be con rmed by a rush o air rom the needle, and a
A ollow up chest radiograph the next morning chest tube should then be placed.
demonstrates signi cant residual basilar f uid. In this patient, the absence o tracheal deviation
What is the most appropriate next step? along with dullness to percussion makes a hemotho-
A. Go to the operating room or a video-assisted rax the more likely diagnosis, and a chest tube the
thoracoscopic surgery (VA S) to evacuate the appropriate next step. T e pre erred method or tube
retained hemothorax. thoracostomy is to enter the sa e triangle bounded
B. Go to the operating room or a right antero- by the anterior border o the latissimus, the lateral
lateral thoracotomy. border o the pectoralis major, and a horizontal line
C. Place a pigtail catheter to drain the residual at the level o the nipple (males) or in ra-mammary
hemothorax. crease ( emales). T is positioning minimizes the
D. Observation and serial chest X-rays. likelihood o entering the abdominal cavity, injur-
ing muscle or breast tissue, or underlying structures
such as the internal mammary artery, and avoids the
ANSWERS major chest wall musculature that can cause signi -
1. B. In any trauma patient, the rst priority is to ensure cant pain with chest tube insertion.
stable A,B,Cs, airway, breathing, and circulation,
which may include emergent interventions (e.g., tube 3. B. Accepted indications or emergency department
thoracostomy). In this patient with stable vital signs, thoracotomy in the setting o penetrating thoracic
the initial step is to per orm a primary and secondary trauma are loss o pulses with previously witnessed
surveya brie head to toe physical exam. A common cardiac activity and unresponsive, persistent, hypo-
mistake in the setting o penetrating trauma is ail- tension. In blunt trauma, indications are rapid exsan-
ure to identi y multiple injuries. Common locations guination rom the chest tube (> 1500 cc on initial
or missed penetrating injuries can be in the back, placement) with unresponsive hypotension. Survival
axilla, or perineum, so a complete head to toe survey rates a er emergency thoracotomy are up to 30%
is critical. Most patients can be stabilized enough to or penetrating trauma, but closer to 1% or blunt
undergo a secondary survey, but any problems iden- trauma patients. T e goals o an emergency depart-
ti ed on the primary survey should be immediately ment thoracotomy are to control hemorrhage (which
addressed. Once all injuries are identi ed, they can be may require cross-clamping the descending thoracic
prioritized or diagnostic imaging or operative repair. aorta), allow access or internal cardiac massage, and
Both chest X-ray and FAS scan can be used as treat potential cardiac tamponade. Access to the heart,
adjuncts to the secondary survey, but they do not descending aorta, and mediastinum requires a le
replace the need or a head-to-toe assessment o the thoracotomy incision. Regardless o the side o injury,
patient. In the initial evaluation o a trauma patient, traumatic arrest warrants a le thoracotomy or access
priorities are: 1. identi ying all wounds, 2. determin- to the mediastinal structures. I necessary, the incision
ing i urgent li esaving intervention is indicated, and can be extended to the right chest (clamshell thora-
3. determining i additional testing is needed. cotomy) or access to the right thoracic cavity.
Stable patients with penetrating trauma may still
2. B. In the setting o penetrating trauma to the chest, require exploration in the operating room. radi-
the di erential diagnosis or hypotension includes tionally, anterior abdominal stab wounds warranted
tension pneumothorax, cardiac tamponade, and mandatory laparotomy to rule out intraabdominal
hemothorax. Classically, a tension pneumothorax injury, although recent evidence has raised the possi-
will present with deviation o the trachea away rom bility o expectant management or the stable patient
the injury, increased percussion and decreased breath with penetrating injuries. T oracic injury to the

http://surgerybook.net/
C H AP TER 6 8 KN I f E WO U N D S 251

box, the area de ned by nipples, sternal notch, and 5. A. Retained hemothorax is a relatively common
xiphoid process generally warrants operative explo- problem among patients who present with a moder-
ration as well. ate or large volume hemothorax with either blunt or
In a stable patient, however, it is reasonable to penetrating trauma. Although observation only is an
proceed with additional diagnostic studies. Although option, this carries the risk o developing an in ected
some have advocated an extensive workup includ- hematoma (empyema) or brothorax. I there is still
ing esophagoscopy, bronchoscopy, and a pericardial a signi cant amount o clotted blood in the thoracic
window, more recent data indicates that a high-qual- cavity that is not adequately drained by the initial chest
ity C scan provides excellent imaging o the tho- tube, then there are several options or management.
rax and mediastinum, and additional studies can be T ese include placement o a second chest tube
per ormed based on the clinical exam and C nd- (or removal and replacement o the initial chest tube)
ings. In a patient who undergoes tube thoracostomy, in a better position to evacuate the uid collection or
indications to proceed to the operating room are: administration o local lytic therapy ( PA or uroki-
> 1500 cc on initial placement, > 150200 cc/hr or nase administered through the chest tube) to attempt
> 3 hours, or hemodynamic instability requiring per- to break up the clot and allow drainage through the
sistent blood trans usions. chest tube. Although these can be e ective in select
cases, they have been associated with relatively high
4. B. Zone I o the neck extends rom the clavicles to ailure rates. Since the retained hemothorax likely
the cricoid cartilage, Zone II is rom the cricoid car- consists o clotted blood, a small-bore pigtail cath-
tilage to the angle o the mandible, and Zone III is eter is unlikely to achieve adequate drainage. T ere
rom the angle o the mandible to the skull base. T e has been a trend toward the increased use o early
majority o penetrating injuries, as with the patient in VA S to evacuate the hematoma and place a well-
this case, occur in Zone II. For unstable patients with positioned chest tube under direct visualization. T is
signs o major vascular or airway injury, manage- approach can also be use ul i there is suspicion or
ment o penetrating neck trauma involves securing an associated diaphragm injury that can be repaired
an airway and proceeding directly to the operating simultaneously. Median sternotomy would not be
room or neck exploration. T ese signs include stri- indicated or evacuation o a retained hemothorax.
dor, massive subcutaneous air, gurgling or bubbling
through the wound, hemoptysis/hematemesis, and BIBLIOGRAPHY
pulsatile bleeding or rapidly expanding hematoma. Bif WL, Moore EE. Management guidelines or penetrating
Historically, all Zone II injuries that breached the abdominal trauma. Curr Opin Crit Care. December 2010;
platysma mandated operative exploration. T is is 16(6):60917.
no longer widely practiced, and stable patients with Brohi K. Emergency Department T oracotomy. Available
no hard signs o vascular or airway imaging can be at: http://www.trauma.org/index.php/main/article/361/.
Accessed June 9, 2016.
managed expectantly with appropriate diagnostic Burgess CA, Dale O , Almeyda R, Corbridge RJ. An evidence
tests. C A has similar sensitivity to operative explo- based review o the assessment and management o pene-
ration or identi ying vascular and so tissue injuries. trating neck trauma. Clin Otolaryngol. February 2012;37(1):
C imaging can also provide valuable in ormation 4452.
about potential injuries to the esophagus or trachea Houshian S, Larsen MS, Holm C. Missed injuries in a level I
trauma center. J rauma. April 2002;52(4):71519.
that can guide the selective use o additional stud-
Laws D, Neville E, Du y J. B S guidelines or the insertion o
ies to evaluate these structures. Esophageal injuries a chest drain. T orax. May 2003;58(Suppl 2):ii539.
are requently asymptomatic, and morbidity signi - Moore FA, Moore EE. Initial Management o Li e-T reatening
cantly increases i repair is delayed beyond 24 hours. rauma. In: Souba et al., eds. ACS Surgery: Principles
Contrast esophagography or esophagoscopy is rec- and Practice, 6th ed. Ontario, Canada: B.C. Decker Inc;
ommended or all Zone II injuries that breach the 2007:121130.
Mowery N , Gunter OL, Collier BR, et al. Practice manage-
platysma and have either physical exam or C scan ment guidelines or management o hemothorax and occult
ndings concerning or an esophageal injury. Simi- pneumothorax. J rauma. February 2011;70(2):51018.
larly, exible bronchoscopy can be used selectively isherman SA, Bokhari F, Collier B, et al. Clinical practice
based on any exam or C imaging ndings concern- guideline: penetrating zone II neck trauma. J rauma. May
ing or airway injury. 2008;64(5):13921405.

http://surgerybook.net/
69
Penetrating raumaMultiple
Gunshot Wounds
Erik Criman and Matthew J. Martin

You wake to the sound o your pager and the amiliar with prompt return o air and 1800 mL blood, and
re rain overhead, rauma team to the trauma room. has the remainder o the two liters o crystalloid
rauma team to the trauma room. Your pager reads: in used via EMS. You remove the tourniquet and
23YOM GSW O CHES /ABD/R HIGH. GCS immediately appreciate pulsatile bleeding. Distal
15 HR 100 BP 90/P RR30 98%/NRB. E A 3MIN. right lower extremity pulses are not palpable
As you reach the trauma bay, your patient arrives in prior to the re-application o this device and
extremis. He is unresponsive, his breathing is agonal with resultant hemostasis. T e patients heart
with oxygen saturation in the low 80s despite ventila- rate has increased to 120 beats per minute with
tion with a bag valve mask, and his radial pulse is weakly a blood pressure o 86/44. With your primary
palpable with a rate o 105 beats per minute. Paramedics survey completed, which o the ollowing injuries
have established two large bore peripheral IVs and have necessitates urgent operative intervention?
begun in using two liters o normal saline. T ey report A. Gunshot wound to the chest only.
that the patient was shot three times at close range B. Gunshot wound to the abdomen only.
with an unknown rearm. T e rst wound in the right C. Right lower extremity injury only.
upper chest was characterized as sucking on the scene D. Right lower extremity injury and the chest injury.
and has been dressed with an occlusive dressing, taped E. Right lower extremity injury, abdominal injury,
on three sides. T ey have already per ormed a needle and chest injury.
decompression. T e second wound is in the le lower
quadrant o the abdomen and is hemostatic. T e nal
wound is located in the right mid-thigh and is presently 3. Regarding penetrating chest trauma, which o the
hemostatic. Paramedics tell you that the bleeding was ollowing is correct?
pulsatile prior to the application o a tourniquet. A. amponade physiology requires accumula-
tion o at least 150cc o blood in the pericardial
1. Your rst priority or this patient should be: space.
A. Establishing a de nitive airway. B. 85% o injuries can be managed with tube thora-
B. Per orming a chest X-ray. costomy alone.
C. Removing the tourniquet on the patients right C. Great vessel injuries are common in penetrating
lower extremity. chest trauma.
D. Placing a le -sided tube thoracostomy. D. Lung injury that requires operative intervention
E. Placing a central venous catheter. is more common ollowing blunt injury.
E. Prophylactic antibiotic use reduces the incidence
2. Your patient undergoes a rapid sequence intuba- o post-traumatic empyema in the setting o
tion, has a lef -sided tube thoracostomy placed retained hemothorax.

http://surgerybook.net/
C H AP TER 6 9 P En ETRATi n g TRAu m A m u l Ti P l E g u n s H o T Wo u n d s 253

4. Regarding this patients penetrating abdominal gunshot wounds are no exception. Evaluation and
wound, which o the ollowing is correct? stabilization o the patients airway to ensure ade-
A. Injuries to the bowel may be primarily repaired i quate oxygenation and ventilation represents the rst
less than 75% o the bowels circum erence being step in resuscitation. T is sort o vignette is ubiqui-
involved. tous on board examinations and the admonition is to
B. T e most common organ injured is the large Keep it Simple, by ollowing the algorithm deline-
bowel. ated by Advanced rauma Li e Support. Remember
C. FAS ( ocused assessment with sonography or the acronym ABCDE or the primary survey. T is
trauma) examination is poor at detecting hollow involves assessing the patients airway and maintain-
viscous injuries. ing in-line cervical spine immobilization, ensur-
D. All patients with penetrating abdominal injuries ing adequacy o the patients breathing by assessing
that violates the posterior ascia must undergo both oxygenation (via pulse oximetry) and ventila-
an exploration but a laparoscopic one may tion (evaluating respiratory rate and e ort), circula-
su ce. tion (pulse examination, addressing li e-threatening
E. T e operation o choice in an unstable patient bleeding), disability (neurologic examination), and
having sustained penetrating abdominal injury exposure (strip patient and per orm rapid scan or
is a limited laparotomy with extension only i injuries). It is important to be mind ul that with a
injuries are suspected. team approach in a working trauma, much o the
above can be per ormed simultaneously.
For our patient, his unresponsiveness, agonal
ANSWERS
breathing, and oxygen desaturation are concerning
Gunshot wounds all under the broad classi cation o and a de nitive airway is indicated. While a chest
penetrating trauma and comprise up to 10% o all major X-ray will be per ormed and the le -sided needle
trauma in the United States. T e energy imparted to tis- decompression will need to be supplanted by a tube
sue can be calculated using the kinetic energy equation: thoracostomy, these are o subsidiary importance to
Ke = m(v)2, where m is the mass o the projectile and securing the patients airway. A central venous cath-
v is the change in velocity be ore and a er contact. eter is not necessary at this time given the presence o
Simplistically, it can be in erred that higher caliber re- two large bore peripheral IVs. While it will be impor-
arms and those projecting higher velocities will cause tant to take down the patients tourniquet to per orm
more grievous injury. T is assumes, o course, that the a detailed extremity evaluation, the report o pulsatile
entirety o the projectiles kinetic energy is imparted bleeding in the eld is suggestive o vascular injury
to the tissue. T ose bullets that enter and exit do not and given present hemostasis, this can be de erred
expend all o their kinetic energy on the body. Further- until the initial resuscitation has been per ormed.
more, the projectiles behavior in the body is a unction
o its relative density. Lower density projectiles (e.g., 2. E. Knowing the indications or operative interven-
lead or so-called hollow-point bullets) will tend to tion is essential to caring or patients having sus-
expand, creating a progressively enlarging wound tract tained traumatic injuries. For penetrating thoracic
with a comparatively small entrance wound and a large trauma, immediate resuscitative thoracotomy (the
exit wound. High density and/or high velocity projec- so-called Emergency Department T oracotomy)
tiles tend to pass directly through tissue but can cause is indicated or witnessed pulseless electrical activ-
signi cant indirect injury via cavitation. Even urther, ity (overall survival 4% to 5% or gunshot wounds
high density tissue like bone can dramatically alter the vs. 18% to 24% or stab wounds). Urgent thoracot-
initial trajectory, de ecting the projectile in nearly any omy (within 1 to 4 hours o admission) is indicated
direction. It is imperative that patients who have sus- or: initial chest tube output > 1500 mL, evidence o
tained gunshot wounds undergo a systematic evalu- ongoing bleeding at a rate o 200300 mL/h, massive
ation with complete anatomic exposure and physical air leak, or cardiac tamponade.
examination. For abdominal trauma, indications or laparot-
1. A. T e primary survey on trauma patients ollows omy include hemodynamic instability, peritonitis on
an algorithmic approach to prevent overlooking examination, or evisceration. Strong consideration
potentially li e-threatening pathology. Patients with should also be given to exploration or those patients

http://surgerybook.net/
254 g En ERAl s u Rg ERY EXAm i n ATi o n An d Bo ARd REVi EW

with an abdomen that cannot be evaluated clinically trauma. T e majority o these injuries can be man-
(e.g., due to altered mental status, distracting injury, aged with pulmonary tractotomy ( or penetrating
paralytic/sedative administration, etc.). non-hilar injuries) or nonanatomic stapled resec-
Hard signs o arterial injury include pulsatile tions. Inadequately evacuated hemothorax can result
hemorrhage, expanding/pulsatile hematoma, bruit in secondary in ection (post-traumatic empyema)
or thrill over wound, absent distal pulses, or evidence or brothorax (entrapped lung). Un ortunately,
to suggest extremity ischemia (pallor, poikilother- prophylactic antibiotics do not appear to prevent
mia, pain, paralysis). So-called so signs include the secondary development o in ection in this set-
non-expanding hematoma, peripheral nerve de - ting. Chest tubes are o en unsuccess ul in remov-
cit, history o pulsatile hemorrhage at the time o ing clotted blood and consideration should be given
injury, and unexplained hypotension. Any patient to operative exploration or patients with retained
presenting with hard signs o vascular injury should hemothorax.
undergo prompt exploration as the positive predic-
tive value o physical examination or arterial injury 4. C. While a FAS examination is a use ul bedside
approaches 100%. In the absence o hard signs, an adjunct to detect the presence o ree intra-abdominal
alternate means o per orming a bedside evalua- or pericardial uid, it is operator dependent and has
tion is with the injured extremity index (analogous a poor sensitivity with respect to hollow viscous, ret-
to an ankle-brachial index, also known as an arterial roperitoneal, and diaphragmatic injuries. A negative
pressure index) with a normal value o > 0.9 having FAS examination does not rule out intra-peritoneal
a reported sensitivity/speci city o 95/97% respec- injury. T ere is evidence to suggest that hemody-
tively or major vascular injury. A normal physical namically stable patients with abdominal gunshot
examination and injured extremity index virtually wounds and no evidence o peritonitis on exam may
exclude major arterial injury. undergo evaluation via computerized tomography o
the abdomen to determine whether urther operative
3. B. Most thoracic injuries can be managed with tube intervention is required. Even i the posterior ascia
thoracostomy; only 10% to 15% o thoracic trauma is violated, elective non-operative management with
requires operative intervention. amponade physi- observation and serial examinations is a sa e alterna-
ology classically only requires 50 cc o blood in tive to re ex operative exploration in a major trauma
the pericardial space and is characterized clinically center with in-hospital surgical support.
by Becks riad (jugular venous distention, muf ed In general, primary repairs can be done on both
heart sounds, hypotension), occasionally with a nar- small and large bowel i 50% or less o the circum-
rowed pulse pressure, and the presence o an e usion erence is damaged. T e most commonly injured
on FAS examination. abdominal organ with penetrating trauma is the
Patients with tamponade physiology should small bowel. For unstable patients with penetrating
receive aggressive volume administration because abdominal injuries, a ull laparotomy incision should
they will be dependent on their preload to generate be made so that all o the injuries can be identi ed
cardiac output. Decompressive pericardiocentesis quickly.
may be per ormed though in the setting o trauma,
but this is o en a temporizing measure. De ni-
BIBLIOGRAPHY
tive surgical management in this setting o trauma
involves the creation o a pericardial window. American College o Surgeons Committee on rauma.
Advanced Trauma Life Support (ATLS) Student Course Man-
T e reported incidence o great vessel injury in ual. 9th ed. Chicago: American College o Surgeons; 2012.
penetrating chest trauma is only 4%, as the major- Boulanger BR, Kearney PA, suei B, Ochoa JB. T e routine
ity o these patients exsanguinate prior to presenta- use o sonography in penetrating torso injury is bene cial.
tion. Gunshot wounds to the mediastinum should J Trauma. 2001;51(2):320.
raise suspicion and the diagnosis can be con rmed Champion HR, Copes WS, Sacco WJ, Lawnick MM, Keast
SL, Bain LW Jr, et al. T e major trauma outcome study:
with C angiography. I present, urgent exploration
Establishing national norms or trauma care. J Trauma.
is warranted. 1990;30(11):1356.
Lung injury that requires surgical intervention Cohn SM. Pulmonary contusion: Review o the clinical entity.
is more commonly encountered with penetrating J Trauma. 1997;42(5):9739.

http://surgerybook.net/
C H AP TER 6 9 P En ETRATi n g TRAu m A m u l Ti P l E g u n s H o T Wo u n d s 255

Como JJ, Bokhari F, Chiu WC, Duane M, Holevar MR, Karmy-Jones R, Jurkovich GJ, Shatz DV, et al. Management o
andoh MA, et al. Practice management guidelines traumatic lung injury: A western trauma association mul-
or selective nonoperative management o penetrating ticenter review. J Trauma. 2001;51(6):104953.
abdominal trauma. J Trauma. 2010;68(3):721. Maxwell RA, Campbell DJ, Fabian C, et al. Use o presump-
Demetriades D. Penetrating injuries to the thoracic great ves- tive antibiotics ollowing tube thoracostomy or trau-
sels. J Card Surg. 1997;12(2):1739. matic hemopneumothorax in the prevention o empyema
Demetriades D, Hadjizacharia P, Constantinou C, Brown C, and pneumonia: A multi-center trial. J Trauma. 2004;
Inaba K, Rhee P, et al. Selective nonoperative management 57(4):7428.
o penetrating abdominal solid organ injuries. Ann Surg. Meredith JW, Hoth JJ. T oracic trauma: When and how to
2006;244(4):6208. intervene. Surg Clin North Am. 2007;87(1):95118.
Demetriades D,Velmahos GC. Penetrating injuries o the chest: Natarajan B, Gupta P , Cemaj S, Sorensen M, Hatzoudis
indications or operation. Scand J Surg. 2002;91(1):415. GI, Forse RA. FAS scan: Is it worth doing in hemo-
Frykberg ER, Dennis JW, Bishop K, Laneve L, Alexander RH. dynamically stable blunt trauma patients? Surgery.
T e reliability o physical examination in the evaluation o 2010;148(4):695.
penetrating extremity trauma or vascular injury: Results Pryor JP, Reilly PM, Dabrowski GP, Grossman MD, Schwab
at one year. J Trauma. 1991;31(4):502. CW. Nonoperative management o abdominal gunshot
Hines MH, Meredith JW. Special problems o thoracic trauma. wounds. Ann Emerg Med. 2004;43(3):344.
In: Ritchie WP, Steele G Jr., Dean RH, eds. General surgery. Velmahos GC, Constantinou C, illou A, Brown CV, Salim
Philadelphia, PA: JB Lippincott; 1995:85972. A, Demetriades D. Abdominal computed tomographic
Hoth JJ, Burch P , Bullock K, et al. Pathogenesis o post- scan or patients with gunshot wounds to the abdo-
traumatic empyema: T e impact o pneumonia on pleural men selected or non-operative management. J Trauma.
space in ections. Surg Infect (Larchmt). 2003;4(1):2935. 2005;59(5):1155.
Isenhour JL, Marx J. Advances in abdominal trauma. Emer- Velmanos GC, Demetriades D, outouzas KG, et al. Selective
gency Clinics of North America. 2007;25:713. nonoperative management in 1856 patients with abdomi-
Johansen K, Lynch K, Paun M, Copass M. Non-invasive vas- nal gunshot wounds: Should routine laparotomy still be
cular tests reliably exclude occult arterial trauma in injured the standard o care? Ann Surg. 2001;234(3):395.
extremities. J Trauma. 1991;1(4):5159. Wall MJ Jr, Hirschberg A, Mattox KL. Pulmonary tractotomy
Karmy-Jones R, Jurkovich GJ. Blunt chest trauma. Curr Probl with selective vascular ligation or penetrating injuries to
Surg. 2004;41:211380. the lung. Am J Surg. 1994;168(6):6659.

http://surgerybook.net/
70
Cervical Spine Clearance

Matthew R. Fusco, Ajith J. T omas, and Christopher S. Ogilvy

A 60-year-old male restrained driver is involved in a 3. Which of the following therapeutic measures is
high-speed head-on motor vehicle collision. He is heav- considered a current standard treatment option
ily intoxicated and uncooperative, there ore he is intu- for this patients spinal shock?
bated at the scene and brought to the nearest Level I A. Limited boluses o normal saline or LR to pre-
trauma center. Upon arrival he is able to shrug his vent spinal cord edema
shoulders to questions. Initial vital signs indicate a pulse B. Bed rest
o 48 bpm and a blood pressure o 78/39 mm Hg. His C. Selective vasopressor treatment to keep MAPs
respiratory status appears stable on the current ventila- > 85
tor settings and initial primary trauma evaluation ails D. Administration o methylprednisolone bolus
to reveal any major external signs o injury. at 30 mg/kg ollowed by continuous in usion at
Subsequent secondary trauma survey reveals the 5.4 mg/kg/hr or 23 hours
patient is able to shrug his shoulders but demonstrates
no motor or sensory unction below his deltoids. Ques- 4. Imaging reveals a fracture dislocation of the mid
tionable rectal tone is present. A cervical collar is in cervical spine with a resultant spinal cord injury.
place, but the patient demonstrates tenderness to palpa- Computed tomographic angiography (C A) reveals
tion in the midline o the cervical spine. a traumatic vertebral artery dissection. Which
of the following is associated with a markedly
1. What is the most likely cause of this patients increased chance of cervical vascular injury and
hypotension? thus requires an evaluation with C A?
A. Splenic rupture A. Neurological examination out o proportion with
B. Acute blood loss computerized tomography (C ) head ndings
C. Flail chest B. Seat belt sign
D. Spinal shock C. C5 spinous process racture
E. Beta-blockade D. Glascow coma scale < 10
2. Which of the following is not a component E. Le Fort I racture
evaluated by the NEXUS criteria for clearing a
patients cervical spine from injury such that the ANSWERS
cervical collar can be removed?
1. D. Spinal cord injury ollowing a traumatic cervical
A. Pain with passive motion o the patients neck spine injury typically results rom blunt compres-
B. History o Ambien use sion injury to the cord itsel . ypically the central grey
C. Any other injury matter is a ected rst while the peripherally located
D. Altered mental status white matter ber tracts may be relatively spared. I
E. High velocity mechanism o injury

http://surgerybook.net/
C H AP TER 7 0 C ERv i C Al S P i n E C l EARAn C E 257

the spinal cord injury is severe enough, neurogenic or 3. C. Current treatment protocols or spinal shock
spinal shock may occur. T e most basic de nition o involve the mainstays o aggressive uid resuscita-
this is inadequate tissue per usion due to paralysis o tion and vasopressors. Vasopressor choice is lef to
vasomotor input, most commonly due to loss o sym- the discretion o the treating physician based on the
pathetic tone and signi cant disruption o the vaso- patients co-morbid actors. A goal MAP o > 85 sus-
dilator and vasoconstrictor balance. It is commonly tained or 7 days has demonstrated some promise or
characterized by bradycardia, hypotension, decreased improving a patients outcome. Usage o high dose
peripheral vascular resistance, and decreased car- steroids in acute spinal cord injury is a controversial
diac output. Common physical examination ndings topic and has been or quite some time. Various large
demonstrate accid paralysis o the extremities with trials (NASCIS I, II, III) have demonstrated mild
no sensory unction, lack o rectal tone, lack o oley bene ts with the administration o methylpredniso-
catheter sensation, and priapism in males. I the spi- lone bolus o 30 mg/kg ollowed by 23 hours o con-
nal cord injury occurs at C5 or above then respiratory tinuous in usion at 5.4 mg/kg/hour. However, these
depression may occur. As this patient demonstrates studies have also demonstrated signi cant deleteri-
some deltoid unction, this indicates a lower cervical ous side e ects with high dose steroids leading to the
cord injury. Presence o a shoulder shrug should not Congress o Neurosurgeons to declare that high dose
ool the examiner. T is motor unction is provided by steroids risks outweigh their bene ts, and are thus
the 11th cranial nerve. T e most common classi ca- not recommended.
tion o spinal cord injuries occurs via the ASIA system:
4. A. Blunt carotid or vertebral artery injuries can be a
A = Complete No motor or s e ns ory function is pre s e rve d in potentially lethal injury i missed upon initial trauma
the s a cra l s e gme nts S 4S 5
evaluation. Level II evidence exists neck C A screen-
B = Incomplete S e ns ory but no motor function is pre s e rve d
be low the ne urologica l leve l a nd include s ing in trauma patients with an exam out o proportion
the s a cra l s e gme nts S 4S 5 to their cranial imaging (i.e., a comatose patient with
C = Incomplete Motor function is pre s e rve d be low the
ne urologica l leve l, a nd more tha n ha lf of
minimal traumatic intracranial damage). Level III
key mus cle s be low the ne urologica l leve l evidence exists or such screening in trauma patients
have a mus cle gra de le s s tha n 3
D = Incomplete Motor function is pre s e rve d be low the
with GCS 8 or below, petrous bone ractures, di use
ne urologica l leve l, a nd a t le a s t ha lf of key axonal injury, C13 ractures, any cervical spine rac-
mus cle s be low the ne urologica l leve l have
a mus cle gra de of 3 or more ture with subluxation, cervical spine racture through
E = Normal Motor a nd s e ns ory function a re norma l the oramen transervsarium, or Le Fort II/III injuries.
Surprisingly, despite its widely believed association
with cervical vascular injuries, presence o an isolated
2. E. Any patient suspected o a cervical spine injury
seat belt sign with no other above listed injuries is
and there ore a potentially unstable cervical spine
only associated with a 1% yield or blunt cervical
should be placed in cervical immobilization with a
vascular injuries. reatment o such blunt injuries
rigid collar. Maintenance o in-line cervical align-
attempt to prevent intracranial ischemic injuries.
ment during intubation as well as log-roll precautions
Previously aggressive treatment with open surgical
must be upheld during trans ers. Per the NEXUS cri-
or endovascular repair, of en in the orm o stent-
teria, a patient can be cleared with a 99.8% negative
ing or coiling, provided the mainstay o treatment.
predictive value or cervical spine injury i the ol-
However, current emerging evidence suggests these
lowing criteria are met:
lesions, i diagnosed prior to evidence o ischemia,
Meets all low-risk criteria? can be quite benign i treated with antithrombotic
1. No pos te rior midline ce rvica l-s pine te nde rne s s medications. Currently, either anticoagulation via
2. No evide nce of intoxica tion heparin/Coumadin or anti-platelet medications via
3. A norma l leve l of a le rtne s s
4. No foca l ne urologic de ficit aspirin 325 mg daily are widely used.
5. No pa inful dis tra cting injurie s

YES NO BIBLIOGRAPHY
Aarabi B, Hadley MN, Dhall SS, Gelb DE, Hurlbert RJ,
No Radiography Radiography Rozzelle CJ, et al. Management o acute traumatic central
cord syndrome (A CCS). Neurosurgery. 2013;72(2):195204.

http://surgerybook.net/
258 G En ERAl S U RG ERY EXAM i n ATi O n An D BO ARD REv i EW

Dumont RJ, Okonkwo DO, Verma S, Hurlbert RJ, Boulos P , Hurlbert RJ, Hadley MN, Walters BC, Aarabi B, Dhall SS, Gelb
Ellegala DB, et al. Acute spinal cord injury, part I: patho- DE, et al. Pharmacological therapy or acute spinal cord
physiologic mechanisms. Clin. Neuropharmacol. 2001;24: injury. Neurosurgery. 2013;72(2);93105.
25464. Michale ZA, Maher CG, Verhagen AP, Rebbeck , Lin C-WC.
Harrigan MR, Hadley MN, Dhall SS, Walters BC, Aarabi B, Accuracy o the Canadian C-spine rule and NEXUS to
Gelb DE, et al. Management o vertebral artery injuries screen or clinically important cervical spine injury in
ollowing non-penetrating cervical trauma. Neurosurgery. patients ollowing blunt trauma: A systematic review. CMAJ
2013;72(2),23443. Can. Med. Assoc. J. J. Assoc. Medicale Can. 2012;184:E86776.

http://surgerybook.net/
71
Management o Elevated
Intracranial Pressure
Matthew R. Fusco, Ajith J. T omas, and Christopher S. Ogilvy

A 32-year-old male, unrestrained driver, is involved in 3. Which o the ollowing should be implemented in
a high-speed motor vehicle collision. A er a prolonged this patient ollowing external ventricular drain
extraction, the patient is ound to be somnolent and is placement and admission to the ICU?
intubated on the scene. A er arrival in the trauma center, A. Elevation o the head o bed to up to 10 degrees
a complete evaluation reveals a non-displaced linear so measurement o the intracranial pressure
skull racture and multiple areas o intracranial contu- (ICP) remains calibrated
sions with di use edema but no large mass lesions. No B. Maintain hemoglobin levels > 10 g/dL
other systemic injuries are discovered. His neurologic C. Maintenance o a cerebral per usion pressure o
exam demonstrates small but reactive pupils, presence 60 or above
o corneal, gag, and cough re exes, lack o eye opening D. Allow hypercapnia
to voice or pain ul stimulation, brisk withdrawal o both E. Administration o high levels o positive end-
legs and his right arm to pain ul stimulation as well as expiratory pressure (PEEP) to aid in oxygena-
localization o his le arm to pain ul stimulation. tion
1. What is the patients Glasgow Coma Scale (GCS)?
A. 5 4. On post-admission day 2, the patients intracranial
B. 6 pressures rise to the low 30s. Which o the ollowing
C. 7 maneuvers can be used to reduce his intracranial
D. 8 pressure?
E. 9 A. Increased sedation with propo ol only because
other sedatives can increase cerebral metabolic
2. Which o the ollowing are indications or the use demand
o intracranial pressure monitoring in traumatic B. 3% hypertonic saline
brain injury? C. Permissive hypotension
A. Intracranial injury demonstrated on computer- D. ransient periods o hypoventilation
ized tomography (C ) scan and GCS o 8 or below. E. Administration o methylprednisolone
B. Lack o intracranial injury on C scan, but age
> 40, SBP < 90, and motor posturing on exam.
ANSWERS
C. Minor neurologic injury but lack o neurologic
examination due to need or systemic paralytics 1. C. T e Glasgow Coma Scale (GCS) is the most basic
in treatment o other injuries. method o quickly communicating a patients neuro-
D. All o the above. logic examination. It is comprised o three compo-
E. None o the above. nents:

http://surgerybook.net/
260 G EN ERAL S U RG ERY EXAM IN ATI O N AN D BO ARD REVIEW

Eye Opening (E) Verbal Response (V) Motor Response (M)


4= opens spontaneously 5= normal conversation 6= normal
3= opens to voice 4= disoriented conversation 5= localizes pain
2= opens to pain 3= words, incoherent 4= withdraws rom pain
1= none 2= incomprehensible sounds 3= decorticate posturing
1= none 2= decerebrate posturing
1= none

T us, a non-responsive comatose patient has a and is commonly used to guide therapy. In uninjured
GCS o 3 while an awake, oriented patient would patients, the brain will maintain cerebral per usion
demonstrate a GCS o 15. Intubated patients auto- via autoregulation over estimated CPP ranges o 50
matically receive just 1 point or verbal response. I a to 150.
patients motor exam is asymmetric, the best response However, patients with severe brain injury will
is used or grading. T us this patients exam is E1, V1, commonly exhibit loss o autoregulation, which
M5 or a total GCS o 7. For intubated patients, many means that their cerebral blood ow will be directly
providers use the convention o adding a ollow- dependent on an adequate mean arterial pressure
ing the GCS score, so this patient would be described (MAP) and a normal intracranial pressure (ICP).
as a GCS 7 . T is is why there is so much attention paid to main-
taining a normal to slightly elevated blood pressure
2. D. Insertion o an intracranial pressure monitor can and aggressive measures to avoid ICP elevations in
be used to evaluate and treat elevated ICP or as a sur- patients with severe brain injury.
rogate or a neurologic examination in those who All attempts at treating elevated intracranial pres-
require sedation or chemical paralysis or other inju- sure keep this Monro-Kellie hypothesis and its rela-
ries (such as a trauma patient with an open abdomen tionship with cerebral per usion pressure in mind.
on paralytics). T e most common indication or an T e rst steps toward the treatment o a patient with
ICP monitor is a patient with a traumatic intracra- elevated intracranial pressure begin with many sim-
nial injury on imaging and a GCS o 8 or less. T ese ple bedside maneuvers. Initial steps should include
patients will have an elevated ICP 60% o the time. elevation o the head o bed to aid cerebral venous
Patients with a normal head C scan will only dem- out ow, seizure prophylaxis, maintenance o hemo-
onstrate elevated ICP 13% o the time. T is occurs globin o at least 7 g/dL or above, avoidance o
most commonly in patients over 40 years o age who elevated intra-adominal pressure, and adequate pul-
demonstrate posturing on examination and hypo- monary support. Pulmonary support should ocus
tension. o avoidance o hypoxia, hypercapnia, and exces-
sive PEEP. Elevated levels o PaCO2 lead to intrac-
3. C. T e Monro-Kellie doctrine states that the skull is
ranial vasodilation and cerebral blood volume, thus
a xed compartment with 3 basic components: brain
elevated ICP. Excessive PEEP may lead to increased
(80%), cerebrospinal uid (10%), and blood (10%).
intra-thoracic pressure and thus impaired cerebral
An increase in any one o these components requires
venous out ow.
an equal decrease in one or both o the other two to
prevent an increase in intracranial pressure. ypical 4. B. I elevated ICP occurs, more aggressive methods
cerebral blood ow compromises 1520% o the car- o treatment include increased sedation (via propo-
diac output; a decrease in this may result in unmet ol or barbituates to decreased cerebral metabolic
cerebral metabolic demands. T e primary goal o demands), paralytics, mannitol or both rheologic
treatment o severe brain injury is to maintain ade- and osmotic e ects, and hypertonic saline. Steroids
quate cerebral blood ow and oxygen delivery, but it should be avoided in intracranial trauma. Despite
is dif cult to readily measure these parameters. How- the usage o dexamethasone with other causes
ever, cerebral blood ow can be roughly estimated by o cerebral edema, steroids are associated with
cerebral per usion pressure (CPP = MAP ICP), elevated mortality in trauma patients with severe

http://surgerybook.net/
C H AP TER 7 1 MAN AG EM EN T O f ELEVATED I N TRAC RAN I AL P RES S U RE 2 61

brain injury due to the myriad o side ef ects. Hypo- Brain rauma Foundation, American Association o Neu-
tension should be avoided to help maintain the CPP. rological Surgeons, Congress o Neurological Surgeons,
Joint Section on Neurotrauma and Critical Care, AANS/
CNS, Bratton SL, Chestnut RM, et al.: Guidelines or the
BIBLIOGRAPHY management o severe traumatic brain injury. XIII. Anti-
Brain rauma Foundation, American Association o Neuro- seizure prophylaxis. J. Neurotrauma. 2007;24(1):S8386.
logical Surgeons, Congress o Neurological Surgeons, Joint doi: 10.1089/neu.2007.9983
Section on Neurotrauma and Critical Care, AANS/CNS, Brain rauma Foundation, American Association o Neuro-
Bratton SL, Chestnut RM, et al.: Guidelines or the man- logical Surgeons, Congress o Neurological Surgeons, Joint
agement o severe traumatic brain injury. I. Blood pressure Section on Neurotrauma and Critical Care, AANS/CNS,
and oxygenation. J. Neurotrauma. 2007;24(1):S713. doi: Bratton SL, Chestnut RM, et al.: Guidelines or the man-
10.1089/neu.2007.9995 agement o severe traumatic brain injury. XIV. Hyperven-
Brain rauma Foundation, American Association o Neu- tilation. J. Neurotrauma. 2007;24(1):S8790. doi: 10.1089/
rological Surgeons, Congress o Neurological Surgeons, neu.2007.9982
Joint Section on Neurotrauma and Critical Care, AANS/ Edwards P, Arango M, Balica L, Cottingham R, El-Sayed H,
CNS, Bratton SL, Chestnut RM, et al.: Guidelines or the Farrell B, et al. Final results o MRC CRASH, a randomised
management o severe traumatic brain injury. VII. Intra- placebo-controlled trial o intravenous corticosteroid in
cranial pressure monitoring technology. J. Neurotrauma. adults with head injury-outcomes at 6 months. Lancet.
2007;24(1):S4554. doi: 10.1089/neu.2007.9989 2005;365:19579. doi: 10.1016/S0140-6736(05)66552-X

http://surgerybook.net/
72
Ortho- raumaPelvic Fracture

William J. Jordan

A 27-year-old emale presents to the emergency depart- A. < 5%


ment a er a motorcycle crash. She was hemodynami- B. 10% to 15%
cally unstable on arrival and is currently receiving 2L o C. 50%
crystalloid via large bore peripheral IVs. Glasgow Coma D. 60% to 75%
Scale (GCS) is 12. She is breathing spontaneously and E. > 85%
has no obvious extremity trauma. Initial radiographs
demonstrate normal cervical alignment without rac- 3. Regarding the hemodynamic instability o pelvic
ture, right side ribs 4 through 8 ractured, a small right ractures, what is the racture pattern associated
side pulmonary contusion, no ree air under the dia- with the highest severity o hemorrhage?
phragm, and a pelvis lm that shows widening o the A. Anteroposterior compression (APC III) racture
pubic symphasis by 4 cm along with diastasis o the le patterns
sacroiliac joint. Abdominal ultrasound is negative or B. Lateral compression (LC III) racture patterns
ree uid. T e patient is currently on her menses. C. Vertical sheer (VS) racture patterns
D. Combined APC/VS patterns
1. With regards to the pelvic injury, the patients E. Open pelvic ractures
hemodynamic instability is most likely related to:
A. Disruption o the anastomis o the external 4. Regarding the mortality o displaced pelvic
iliac/deep epigastric and obturator vessels (the ractures, the highest mortality rates are seen in:
Corona Mortis) A. Anteroposterior compression (APC III) racture
B. Arterial injury o the superior gluteal artery and patterns
vein at the level o the greater sciatic notch B. Lateral compression (LC III) racture patterns
C. Post-traumatic closed so tissue degloving C. Vertical sheer (VS) racture patterns
injury in which the skin and subcutaneous tissue D. Combined APC/VS patterns
separate rom the ascia super cial to the under- E. Open pelvic ractures
lying musculature (Morel-Lavallee lesion)
D. Disruption o the anterior sacral venous plexus. 5. Identi cation and management o open pelvic
E. Rupture o the pelvic oor structures (sacrospi- ractures depends on a high index o suspicion by
nous and sacrotuberrous ligaments) with vaginal the initial treating provider. Although this patient
laceration by the anterior pubic symphasis rag- was on her menses and blood was to be expected
ments on examination, a speculum examination o the
vaginal mucosa demonstrates a 2 cm laceration
2. What percentage o vascular injuries rom high- on the lef wall o the vagina and a proctoscopic
energy pelvic trauma are arterial in nature? exam showed a rectal laceration. To decrease the

http://surgerybook.net/
C H AP TER 7 2 O RTH O - TRAu m A P El v i C F RAC Tu RE 263

risk or post-injury in ection and sepsis, which o and does not respond to the initial maneuvers listed
the ollowing should occur early in the treatment above. T e remaining 15% o vascular injuries are
o this injury? arterial and may best be addressed via angiography
A. Urgent gynecological consultation with primary in the event o continued hemodynamic instability
closure o vaginal laceration ollowing pelvic packing.
B. Exploratory laparotomy or laparoscopy with
repair o rectal laceration and diverting colos- 3. A. In an antero-posterior compression racture
tomy (APC), the racture propagates rom anterior to poste-
C. Exploratory laparotomy with repair o rectal lac- rior. T is widens the symphasis pubis and depending
eration without diversion on the level o orce, can widen the anterior sacroiliac
D. Urgent urological consultation with repair o joint or disrupt it entirely. In an APC III injury, the
bladder and urethral injuries pelvic oor ligaments (sacrospinous and sacrotuber-
E. Immediate placement o a suprapubic catheter ous) are disrupted and can leading to instability and
increased volume in the pelvic cavity. Disruption and
displacement o the sacroiliac joint has the potential
ANSWERS to lead to massive hemorrhage rom the anterior
1. D. Hemodynamic instability a er blunt trauma is sacral venous plexus that is closely approximated to
due to ongoing hemorrhage until proven otherwise, the joint. Major hemorrhage in patients with pelvic
with the common locations being the chest, abdo- ractures can reliably be predicted based on a pulse
men, pelvis, or extremity/external bleeding. T e greater than 130, a hematocrit o 30 or less, and wide
normal chest X-ray and abdominal ultrasound in diastasis o the pubic symphasis.
conjunction with the abnormal pelvis X-ray make Although pelvic binders are now routinely applied
the pelvis the most likely source o bleeding in this or suspected or proven pelvic ractures, they may
patient. Although arterial injuries in association with have no bene t or may even cause additional harm in
major pelvic trauma can occur, the majority o bleed- select types o ractures. T e AP pelvic radiograph will
ing occurs at the venous plexus located on the ante- identi y injuries that may bene t rom provisional sta-
rior aspect o the sacrum and sacroiliac joints. T is bilization with external compression with a sheet or
historically has responded best to pelvic packing or pelvic binder. In general, LC injuries will not respond
racture reduction, rather than immediate angiog- to binder placement, whereas APC and VS injuries
raphy. Injuries to the superior gluteal arteries pos- will. Compression o an LC injury is potentially dam-
teriorly and the Corona Mortis anteriorly have been aging as it may induce additional lateral compression
reported and can be devastating, but they are ar out- and racture dislocation or bleeding. Occasionally, an
numbered by the venous lacerations o the presacral LC injury in an elderly patient may have a hemor-
venous plexus. Morel-Lavallee lesions are common rhage associated with vascular or visceral disruption;
a er blunt orce trauma, but do not typically lead to these patients will not bene t rom wrapping or bind-
hemodynamic instability seen rom the deep pelvic ing, but may be candidates or angiography.
venous lacerations.
4. E. Open pelvic ractures are potentially lethal inju-
2. B. T e current management o pelvic racture ries with a reported mortality rate o 30% to 50%.
patients who are hemodynamically unstable consists Open ractures o the pelvis by de nition communi-
o aggressive resuscitation, mechanical stabilization, cate with the rectum, the vagina, or the outside envi-
and angioembolization. Despite this multidiscipli- ronment by disruption o the skin. T ey are o en
nary approach, mortality rates o these high-risk associated with disruption o the pelvic oor, lead-
patients can exceed 40%. Initial maneuvers to reduce ing to loss o tamponade and persistent bleeding. An
the pelvic racture and decrease the pelvic volume additional concern is pelvic in ection and sepsis due
include placement o a pelvic binder, wrapping the to the open communication with the rectum, vagina,
pelvis with a sheet, or application o an external xa- or skin. Clinical suspicion o an open racture and
tion device. Preperitoneal pelvic packing (PPP) via any rectal or vaginal bleeding mandates a thorough
laparotomy can directly address the venous bleeding examination o the pelvis, perineum, rectum/anal
that compromises 85% o pelvic racture hemorrhage complex, and vagina in emales.

http://surgerybook.net/
264 G EN ERAl S u RG ERY EXAm i N ATi O N AN D BO ARD REv i EW

In addition to visual inspection and digital exami- the actual location, nature, and size o the cutaneous
nation, ull evaluation should include rigid proc- wound. For open ractures with an associated rectal
toscopy or exible sigmoidoscopy, and speculum injury, immediate diverting colostomy and repair o
examination in emales. Management o major open the rectal laceration (i possible) are indicated. Ante-
pelvic ractures includes administration o intrave- rior wounds o the groin, anterior thigh, iliac crest, or
nous antibiotics, washout and debridement o open pubis do not require diversion.
wounds, and consideration or a possible diverting
colostomy to prevent ecal contamination o the BIBLIOGRAPHY
open racture. Blackmore CC, Cummings P, Jurkovich GJ, Linnau KF, Ho -
er EK, Rivara FP. Predicting major hemorrhage in patients
5. B. Early diagnosis o an open pelvic racture is essen- with pelvic racture. J Trauma. August 2006;61(2):34652.
Cothren CC, Osborn PM, Moore EE, Morgan SJ, Johnson JL,
tial and a thorough examination must be done so that
Smith WR. Perperitoneal pelvic packing or hemody-
no such injuries are missed. While it is not dif cult to namically unstable pelvic ractures: a paradigm shi .
diagnose an open racture when massive wounds o J Trauma. 2007;62(4):83442.
the skin and perineum are present, a small vaginal or Fallinger MS, McGanity PL. Unstable ractures o the pelvic
rectal tear that communicates with and contaminates ring. J Bone Joint Surg Am. June 1992;74(5):78191.
the racture may be missed unless it is sought speci - Lang ord JR, Burgess AR, Liporace FA, Haldukewych GJ.
Pelvic Fractures: Part 1: Evaluation, classi cation, and resus-
cally. Rectal injuries must be sought, particularly in citation. J Am Acad Orthop Surg. August 2013;21(8):448
patients with a sacral racture, because the ragments 57.
o bone requently traverse the rectal wall. Mohanty K, Musso D, Powell JN, Kortbeek JB, Kirkpatrick
Classically, an open pelvic racture prompts rec- AW. Emergent management o pelvic ring injuries: An
ommendations or colostomy to prevent so -tissue update. Can J Surg. February 2005;48(1):4956.
al DK, et al. Retroperitoneal pelvic packing in the manage-
sepsis in an expanded perineum. It has recently
ment o hemodynamically unstable pelvis ractures: A
been suggested that ecal diversion in an open pel- level 1 trauma center experience. J Trauma. October 2011;
vic ractures can be applied selectively, according to 71(4):E7986.

http://surgerybook.net/
73
Head Injuries

Ahmed B. Bayoumi, Fares Nigim, and Ekkehard M. Kasper

CASE 1
A 44-year-old emale patient presented with a history o
pro use bleeding rom a scalp wound. She was brought
to the emergency room by an attendant. She had direct
trauma caused by alling and hitting her head on a large
rock while hiking. She denied any loss o conscious-
ness or seizures. T e bleeding was controlled by a gauze
dressing and compression bandage at the site o the
injury. She was vitally stable with a Glasgow Coma Scale
(GCS) o 15/15. On neurological examination, she had
no motor de cit except or right sided lower motor neu-
ron (LMN) acial palsy. A large, parietal lacerated scalp
wound was visible 3 cm away rom the midline on the
le , measuring about 2 cm in length. A computerized
tomography (C ) o her head was done a er she com-
plained o having an abnormal salty taste in her mouth.
On leaning orward she had drops o clear uid coming
rom her nose.
T e head C is shown in Figure 73-1. Figure 73-1

1. Which o the ollowing is an indication or surgical 2. Regarding the management o this type o racture,
intervention? which o the ollowing is true?
A. Depression that does not exceed the calvarial A. Early surgical intervention may decrease the risk
thickness traumatic brain injury.
B. Associated intracranial hematoma without a B. Antibiotics are recommended in almost all cases.
mass e ect C. Even i there is evidence o a wound in ection,
C. An isolated clean racture overlying a dural primary bone repair will have a avorable out-
venous sinus in an asymptomatic patient come.
D. Dural injury with persistent cerebrospinal uid D. Elevation o depressed racture usually lowers
(CSF) leak the risk o post-traumatic seizures.
E. Because this is an open skull racture then opera- E. Severe cosmetic de ormity is an indication or
tive intervention is mandated. urgent surgery.

http://surgerybook.net/
266 G EN ERAL S U RG ERY EXAM I N ATI O N AN D BO ARD REVI EW

3. Four hours later, another patient with the same type about 3 hours prior to arrival. He was brought in by his
o head injury came to the ER but he deteriorated riend, who said that he had transient loss o consciousness
immediately af er admission. His head C or about one minute a er which he woke up complain-
(Figure 73-2) showed the ollowing image: ing o headache and blurry vision. He vomited once be ore
becoming somnolent and again about 30 minutes later.
On general examination, his vital signs were as ol-
lows: BP = 140/85 mm Hg, RR = 18 cycle/min, Pulse =
68 beat/min, and GCS = 10/15. Bruises were seen
over the right temple. On neurological examination,
the patient had a le hemiparesis with round, equal,
and reactive pupils. T e patient was mildly agitated
with good localization o pain ul stimuli. Deep tendon
re exes (D Rs) were mildly exaggerated with a positive
Babinskis sign on the le side.
A thorough examination was done to exclude associ-
ated injuries o the spine, chest, and abdomen. A rigid
cervical collar had been placed in the eld by EMS. Brain
imaging via C scan revealed a large, right parieto-
temporal epidural hematoma. Cervical spine images
were unremarkable. Emergent surgical intervention was
per ormed with postoperative ICU admission or strict
monitoring o vital signs and neurological status.

Figure 73-2 1. Regarding the etiology o cranial epidural


hematoma (EDH), which o the ollowing is correct?
T e main cause o deterioration o this patient is: A. T e most common source o bleeding in this case
A. ension pneumocephalus o EDH is an injured anterior division o middle
B. Acute obstructive hydrocephalus meningeal artery running beneath the pterion.
C. Hydrocephalus ex vacuo B. EDH cannot be caused by injury o dural venous
D. Subdural hygroma sinuses.
E. Subdural hematoma C. An underlying racture is not commonly ound
in cases o post-traumatic EDH.
4. Regarding the CSF rhinorrhea/otorrhea, which o
D. T e bridging cortical veins are usually torn in
the ollowing is true?
cases o EDH with associated extensive brain
A. CSF has a lower concentration o glucose than lacerations.
does mucus. E. T e source o bleeding in traumatic EDH and
B. Magnetic resonance imaging (MRI) is the most acute subdural hematoma (aSDH) is usually
sensitive procedure to localize the site o CSF arterial.
stula.
C. Meningococci are the most common pathogen 2. Regarding the clinical presentation o patients with
that may cause meningitis in this condition. cranial EDH, which o the ollowing is correct?
D. Post-traumatic CSF rhinorrhea is more likely to A. EDH is more common in older patients with a
persist compared to spontaneous CSF rhinorrhea. 1:3 male to emale ratio.
E. Post-traumatic CSF otorrhea is usually sel - B. A lucid interval is reported in more than 80% o
limited. traumatic EDH.
C. All cases o EDH present with altered level o
CASE 2
consciousness.
A 30-year-old male A rican patient presents to the emer- D. Anisocoria occurs in cases o rapidly expanding
gency department with altered level o consciousness temporal EDH secondary to central transtento-
ollowing direct trauma to the head by a heavy object rial herniation.

http://surgerybook.net/
C H AP TER 7 3 H EAD I N j U RI ES 267

E. Motor de cit (e.g., hemiparesis) may occur ipsi- C. Bradycardia and hypotension are both part o
lateral to the side o hematoma due to compres- Cushings triad in markedly elevated ICP.
sion against Kernohans notch. D. Preoperative decerebration does not predict
F. Symptoms o increased intracranial pressure morbidity in closed head injuries.
(ICP; e.g., vomiting) are o en more pro ound in E. Presence o a skull racture is a risk actor or the
supratentorial EDH compared to the posterior development o a delayed EDH.
ossa EDH.

3. Regarding the diagnosis o cranial EDH, which o CASE 3


the ollowing is correct? A 21-year-old male Caucasian patient was brought into
A. C scan o the head is the standard imaging the emergency room by his girl riend. She described
modality that reveals cranial EDH without the an episode o a le acio-brachial seizure about 1 hour
need or IV contrast. a er he had a motorbike accident. T e patient had
B. EDH is usually crescentic in shape on C imag- shown a transient loss o consciousness at the scene.
ing, while aSDH is more lenticular with biconvex Upon regaining consciousness he complained only o
borders. a headache be ore he developed the seizure. On general
C. EDH is usually di use beneath the cranial bone examination, vital signs were ound to be within nor-
and appears limited by the alx. mal limits. On neurological examination, he was ound
D. T e resh, undiluted blood o an EDH appears as to still be in post-ictal status but without any motor
a hypodense mass on head C scan. de cit. Intravenous Fosphenytoin was administered
E. A positive swirl sign on a C scan o the head and a non-contrast head C was requested to exclude
indicates a long-standing EDH, which can be intracranial bleeding. T e axial head C can be seen in
treated conservatively. Figure 73-3.

4. Regarding the management o cranial EDH, which


o the ollowing is correct?
A. Lumbar puncture is recommended to allevi-
ate elevated ICP symptoms in cases o temporal
EDH.
B. In general, an epidural hematoma exceeding
30 cm 3 should be surgically evacuated regardless
the conscious level o the patient.
C. EDH maximal thickness > 3 mm on axial C
slices o the head is an absolute indication or
surgical intervention.
D. An acute rapidly expanding EDH is usually
evacuated in the operating room by two widely
spaced bur holes rather than a craniotomy.
E. Dural tenting/tack-up/hitching stitches (that
holds the dura to bone) should be avoided
because it may increase the risk o dural stripping
and post-operative EDH re-accumulation.

5. Regarding the complications and outcome o


Figure 73-3
cranial EDH, which o the ollowing is correct?
A. Patients with aSDH have a better prognosis and
outcome compared to those with EDH. 1. What type o intracranial hemorrhage did he
B. Rapidly expanding temporal EDH may cause have?
uncal herniation where the uncus compresses A. Epidural hemorrhage
the medulla oblongata medially. B. Subdural hemorrhage

http://surgerybook.net/
268 G EN ERAL S U RG ERY EXAM I N ATIO N AN D BO ARD REVI EW

C. Subarachnoid hemorrhage CASE 4


D. Hemorrhagic brain contusions A 26-year-old Hispanic male was brought to the emer-
E. Intraventricular hemorrhage gency room a er being involved in a motor vehicle acci-
dent 2 hours prior. He complained o an intense headache,
2. Regarding this type o intracranial hemorrhage, nausea, and was ound to have memory troubles. He was
which o the ollowing is correct? a known cocaine addict. He was brought in by his room-
A. It occurs most commonly in the occipital and mate whom he could not identi y during examination.
parietal poles o the brain. T e patient had unremarkable vital signs and a GCS o
B. It occurs rom an impact o the brain on bony 14/15. On neurological examination, he was agitated and
prominences. con used with post-traumatic amnesia. He had no appar-
C. Surgical evacuation is only indicated in case o ent motor or sensory de cit except or right sided pupil-
progressive neurological deterioration. lary dilation and ptosis since the accident. A thorough
D. emporal involvement has a lower threshold or examination was done to exclude associated injuries and
surgery than the other sites. a non-contrast C study o the head was done, which
E. IV Dexamethasone is part o the medical man- revealed the image seen (Figure 73-4).
agement or this type o head injury.

3. Regarding elevated intracranial pressure (= ICP)


and cerebral hemodynamics, which o the ollowing
is true?
A. Normal ICP o adults should always be less than
10 mm Hg.
B. reatment o high ICP should be initiated or
patients with sustained ICP 15 mm Hg.
C. T e goal o ICP management is to keep mean
ICP < 20 mm Hg and cerebral per usion pres-
sure (CPP) 70 mm Hg.
D. T e cerebral blood ow (CBF) to white matter is
higher than that o grey matter.

4. Which o the ollowing will increase ICP?


A. Mannitol 20% Figure 73-4
B. Saline 3%
C. Saline 23.4%
1. What is the most common cause o this type o
D. Dextrose 10%
intracranial hemorrhage?
E. Furosemide
A. rauma
B. Aneurysmal rupture
5. Which o the ollowing is considered to be routine C. Ruptured arteriovenous mal ormation (AVM)
measures to control ICP? D. Coagulopathy
A. Elevation o the head to 75 E. Pituitary apoplexy
B. Avoidance o hypotension, hypertension, or F. Cocaine abuse
hypoxia
C. Hyperventilation to keep PCO2 25 mm Hg 2. Regarding the clinical presentation o this type o
D. Heavy sedation and paralysis intracranial hemorrhage, which o the ollowing is
E. Endotracheal intubation or patients with GCS correct?
12 A. T e patient usually presents with high ICP symp-
toms secondary to hemorrhagic mass e ect and
not cytotoxic di use brain edema.

http://surgerybook.net/
C H AP TER 7 3 H EAD IN j U RI ES 269

B. Seizures are not a part o the course o the disease. nausea, speech di culties, or visual changes. T e patient
C. Nuchal rigidity is mandatory to con rm the reports several episodes o transient orthostatic light-
diagnosis. headedness and had had several alls; she ell last about
D. Patients do not present with ocal neurological 2 months prior to admission and had a minor head
de cits. trauma which was cared or without medical attention;
E. Sudden onset unilateral ptosis with pupillary dila- the patient denies loss o consciousness be ore or a er
tion may raise the suspicion o cerebral aneurysm. the alls. During history taking, the patient reported that
she had a stroke 2 years ago which le her with slight
3. Regarding the diagnosis o this type o hemorrhage, weakness in the le upper and lower extremities.
which o the ollowing is correct? On physical examination her vital signs show BP
A. Lumbar puncture is a sa e procedure to con rm 135/90 mm Hg; temperature o 98F (36.7C); RR o 16/
the diagnosis by CSF analysis. minute; HR is 76/minute and regular. Head, Eye, Ear,
B. C Head requires IV contrast to increase the sen- Nose, and T roat Exam (HEEN ) is normocephalic
sitivity to 95% o cases within the rst 24 hours. but showing a large bruise on her le temple. PERRLA
C. Contrast enhanced head C does not mimic the shows no undoscopic abnormalities. Her neck is supple,
radiological image o this type o intracranial with no carotid bruits and her heart rate and rhythm is
bleeding. normal S1/S2, with no murmurs, rubs, or gallops.
D. Fluid-attenuated inversion recovery (FLAIR) T e neurological examination shows mental status
MRI is the most sensitive MRI sequence to is alert and oriented 3. T e patient could spell words
detect bleeding. backwards, but recalled only 2/3 objects. Her GCS is
E. MRI is a sensitive radiological study to detect the 11/15 and the cranial nerves are 212 grossly intact. Her
acute stage (within the rst 24 hours) o this type motor:strength is 5/5 in all muscle groups except 3/5 in
o bleed. the le arm throughout. Her D Rs are asymmetric 3+
in le upper and lower extremities and 1+ on the right.
4. I this patient had no obvious history o head T ere was no Babinski sign bilaterally. T e cerebellar
trauma, what would be the gold standard study to exam was negative, as was Rombergs test. Her gait is nor-
identi y the underlying cause o hemorrhage? mal and her sensation is intact to pinprick and light touch.
A. Magnetic resonance angiography (MRA) A thorough physical examination was done to exclude
B. Magnetic resonance venography (MRV) skull bone ractures and spine injuries. Head C -scan
C. Catheter cerebral angiography imaging showed the images provided below, revealing
D. MRI di usion study (DWI) a crescent shaped, hypo-dense extra-axial collection as
E. C per usion study seen in Figure 73-5:
F. ranscranial doppler study

5. I the patients conscious level deteriorates, what


could be the underlying cause o this deterioration?
A. Hydrocephalus
B. Cerebral vasospasm
C. Hyponatremia
D. Rebleeding
E. Seizures
F. Any o the above

CASE 5
A 75-year-old emale was brought to the emergency
room by her daughter. T e patient has been complain-
ing o con usion, orget ulness, and headaches over the
last month and her symptoms got worse over the last
week. Her daughter, who lives with her, said that she has
not been hersel or about a month. She denies vomiting, Figure 73-5

http://surgerybook.net/
270 G EN ERAL S U RG ERY EXAM IN ATIO N AN D BO ARD REVI EW

T e patient was brought to the operating room or ANSWERS


evacuation o a hematoma. She did well post-opera-
tively and was discharged to rehabilitation at day-2 a er Answers to Case 1
surgery.
1. D. Image 73-1 is an axial cut o C head (bone win-
1. Which o the ollowing is considered to be the dow) showing a le parietal compound depressed
highest risk actor or chronic subdural hematoma bone racture. T is is a case o an open skull racture
af er head trauma? or compound depressed skull racture. It is recom-
mended to manage patients with skull ractures over-
A. Seizure history
lying a dural venous sinuses conservatively provided
B. Anticoagulation/antiplatelet therapy
that there is no neurological de cit or CSF leak. T e
C. Gender
high probability o vessel wall laceration with subse-
D. Alcohol abuse
quent pro use bleeding during surgery is the main
E. Age > 65
cause o perioperative mortality. Meier and his col-
leagues reported 100% mortality rate o cases with
2. Which o the ollowing statements is correct with skull ractures lacerating the posterior third o the
regard the options or management o chronic superior sagittal sinus and a 50% mortality rate in
subdural hematoma? cases in which the middle third was af icted. T e
A. Conservative treatment (e.g., corticosteroids and study showed an overall mortality rate o 41% or
bed rest) is suitable in lesions with less than 1 cm cases with major dural sinus injury. In a retrospec-
thickness even with the presence o ocal neuro- tive review o 27 patients with depressed compound
logic signs. skull ractures overlying but not disrupting a venous
B. Burr-hole drainage with the placement o subdu- sinus, a management recommendation was made that
ral drain is not as e ective as a craniotomy and wound washout with antibiotics is an adequate treat-
should be avoided. ment as long as there is no mass e ect or contamina-
C. Craniotomy is a suitable surgical intervention tion deep to bone. A depressed skull racture greater
or loculated lesions. than the thickness o the calvarium, an intracranial
D. Percutaneous twist-drill bedside drainage is not hematoma with mass e ect are considered possible
indicated because o the risk o in ection. indications o surgery.
E. Operating room twist drill craniostomy requires
intubation or heavy sedation. 2. B. T ere is no evidence that bone elevation ollow-
ing open skull ractures necessarily decreases the
3. Which o the ollowing correlates with the outcome incidence o post-traumatic seizures which probably
o chronic subdural hematoma? do occur as a direct result o the initial brain injury.
A. Hematoma density on C Early surgery has been recommended to reduce
B. raumatic etiology the risk o in ection but it does not e ect the extent
C. T e male sex o a traumatic brain injury. Antibiotics are part o
D. Hematoma location the management strategies o compound depressed
E. Age ractures. Same session cranioplasty or repair o
autologous bone is accepted only i there is no evi-
dence o in ection. Marked cosmetic de ormity may
4. Which o the ollowing risk actors is associated
warrant elective but not urgent surgical skull recon-
with higher recurrence rate o chronic subdural
struction.
hematoma (CSDH)?
A. Bilateral hematomas with poor re-expansion 3. A. Image 73-2 is an axial cut o C head (so tis-
rate a er surgery sue) shows a case o ension Pneumocephalus
B. Etiology with Mount Fuji sign due to collection o bi ron-
C. Interval rom trauma to rst operation tal subdural air compressing the brain. T is image
D. C -characteristics (density) depicts a case o tension pneumocephalus. Pneumo-
E. Age cephalus is de ned as the presence o air within the

http://surgerybook.net/
C H AP TER 7 3 H EAD IN j U RI ES 271

intracranial cavity and may include one or more o could help in identi ying CSF due to its high glu-
the ollowing compartments: (a) epidural, (b) sub- cose concentration ( 40 mg/dl) when compared
dural, (c) subarachnoid, (d) intraparenchymal, and/ to low glucose levels (< 5 mg/dl) ound in mucus
or (e) intraventricular. ension pneumocephalus is secretions.
a term used when a considerable amount o air or
gas is detected on a C scan o the head with rapid Answers to Case 2
clinical deterioration. T is scenario usually war- 1. A. earing or avulsion o the bridging cortical veins
rants immediate surgical intervention in order to is one o the causes o aSDH. Hemorrhagic brain
avoid atal brain herniation. T e C here shows contusions or lacerations might be another source
accumulation o trapped air in subdural and inter- o aSDH. EDH is commonly caused by an arterial
hemispheric space with some resulting compress- injury o the middle meningeal artery which may
ing o the brain. T is radiological nding is called progressively strip the dura rom the inner table o
Mount Fuji sign. T e air usually appears black on the skull. EDH is sometimes caused by bleeding rom
C scans with a density measuring 1000 Houns- dural veins, venous sinus or diploic channel injury
eld units. No hydrocephalus is seen as there is no ollowing skull ractures. In a case series o 40 in ants
ventricular enlargement. T e latter usually occurs with EDH, Leggate and colleagues could identi y the
due to obstruction o the CSF- ow resulting in CSF bleeding source in 31 cases. In 42.5% o those cases
accumulation and subsequent ventriculomegaly it was rom the middle meningeal artery. Skull rac-
rom obstructive hydrocephalus. Hydrocephalus tures were seen in 45% o cases in another series o
ex vacuo is a term that alsely describes relative 210 traumatic EDH cases.
ventricular enlargement that may occur in elderly
patients secondary to brain parenchymal atrophy 2. E. T e alse localizing sign, or Kernohans notch
rom the natural aging process. phenomenon, is a compression o the contralat-
T e presence o a subdural hygroma is a post- eral cerebral peduncle against the edge o tentorial
traumatic condition that develops due to a separation incisura which can lead paradoxically to an ipsilat-
o the dura-arachnoid inter ace ollowed by a passive eral hemiparesis. It is an important motor localizing
uid accumulation secondary to decreased ICP or sign that appears alse in topography, but may occur
rom excessive dehydration. Such a uid collection with any supratentorial mass during uncal hernia-
overlying the brain convexity rarely shows any mass tion. Yoo and colleagues explored this state o the
e ect which makes surgery usually unnecessary. T e corticospinal tract in a patient with chronic SDH
dynamics o alternating absorption and expansion is and ipsilateral hemiparesis by using di usion tensor
thought to be responsible or the occasional trans or- imaging (D I) and transcranial magnetic stimula-
mation o some SDHG cases into chronic subdural tion ( MS).
hematomas. EDH occurs less requently in children and
elderly patients likely due to adherence o the dura
4. E. Most post-traumatic CSF leaks subside within to the inner table o the skull, thus sealing rmly the
72 hours to 1 week without any need or surgical epidural space. In an epidemiological analysis o 210
intervention. Daele and colleagues recommended cases o traumatic EDH, 89.2% o cases were in males
surgery or spontaneous CSF stula cases since and the observed presenting GCS was between 13
they are more likely to persist. Post-traumatic CSF and 15. T ere ore, it is not uncommon or patients
otorrhea on the other hand usually stops within 5 to with EDH to be ully conscious at the time o clinical
10 days. Detection o 2-trans errin in rhinorrhea- assessment.
uid corroborates the presence o a CSF stula. A classic lucid interval is seen in only 21% o
ahir and colleagues reported that C -cisternog- traumatic EDH cases. Ipsilateral pupillary dilation
raphy has the highest sensitivity and speci city mostly occurs ollowing oculomotor nerve compres-
when compared to C - and MR-scanning to detect sion by the medially displaced uncus and hippocam-
the site o a CSF leak. Pneumococci are the most pal gyrus (called: uncal herniation).
common causative organisms in cases o menin- In 27 posterior ossa EDH cases reported in a
gitis ollowing a CSF stula. Glucose urine strips study by Su and colleagues, headache and vomiting

http://surgerybook.net/
272 G EN ERAL S U RG ERY EXAM I N ATI O N AN D BO ARD REVI EW

Figure 73-6 Axial cuts o C head. On the Right image: Rt acute subdural hematoma (crescentic and di use). On the Le
image: Rt acute epidural hematoma (lenticular and localized).

the most requently seen symptoms. Rapid brain- radiological sign o brain compression which usu-
stem compression with respiratory depression and ally indicates the need or surgical intervention. A
death may occur in posterior ossa EDH cases i not patient with a GCS o more than 8 and a hematoma
treated promptly. less than 30 cm 3, less than 15 mm in thickness, with
less than 0.5 cm midline shi , and without ocal
3. A. C scanning o the head usually shows a classic de cit can be managed by close observation with
appearance o EDH which is a homogeneous hyper- serial scans and placement in a monitored neuro-
dense extra-axial lenticular mass with convex bor- surgical unit. Acute intracranial hematomas are
ders. IV post-contrast C is very rarely needed in evacuated by cranitomies or craniectomies as the
cases o isodense EDH. EDH is usually localized to resh blood is almost clotted. On the other hand,
a limited part o the calvarium since it is con ned by chronic SDH are usually liqui ed and can be surgi-
the skull sutures (not the alx). On the other hand, cally drained by one or two burr holes. Dural tent-
a SDH is characterized on head C as a crescentic ing/tack-up/hitching stitches (that holds the dura
mass with biconcave shape, di usely spanning a large to bone) are advocated by many surgeons at the
area o the brain convexity and is not limited by the end o the surgical procedure as a routine to close
sutures. T is is illustrated in Figure 73-6. the epidural space in order to prevent EDH reac-
A swirl sign is a distinct radiographic sign o cumulation. However, hemostasis is still the most
active bleeding which may occur in di erent types relevant aspect to prevent rebleeding and there is
o intracranial hemorrhage. It presents as an area o no compelling evidence to support the role o dural
irregular density in the hyperdense clot and is typi- tenting or all intracranial operations.
cally o low attenuation (radiolucency). Its occur- Lumbar puncture may lead to li e threatening cen-
rence is associated with worse clinical outcomes and tral or tonsillar herniation in cases in which there is
bigger sizes o intracerebral hematomas as reported high ICP and a pressure gradient between the intrac-
by Selariu et al., as it represents an area o active ranial and intraspinal compartments.
ongoing bleeding.
5. E. Uncal herniation usually occurs with mid-
4. B. An EDH measuring more than 30 cm 3 in volume dle cranial ossa masses where the uncus herniates
must be surgically evacuated regardless the GCS. A medially compressing the occulomotor nerve, cer-
midline shi o more than 5 mm is also a critical ebral peduncles o mid brain, and posterior cerebral

http://surgerybook.net/
C H AP TER 7 3 H EAD I N j U RI ES 273

arteries which may cause unilateral ptosis with improve the outcome or lower the ICP in patients
mydriasis, hemiparesis, and hemianopia, respectively. with severe traumatic brain injury.
In a case series o 161 consecutive patients oper-
ated on or post-traumatic EDH, the overall mortal- 3. C. ICP is one o the most power ul predictors o
ity was 12%, which could be urther lowered by early neurological outcome in patients with traumatic
diagnosis and proactive management. T e overall brain injury. Normal values are variable among di -
mortality in another study o aSDH was 66%. T e erent age groups and are shown in the ollowing
Cushings triad is a re ex triggered by markedly table:
elevated ICP causing hypertension, bradycardia, and
respiratory irregularities. Delayed EDH is de ned as Age Group Normal Range (mm Hg)
a radiologically evolving EDH during sequential neu-
roradiologic examination and which was not present Adults <1015
initially. Detection o a skull racture is an indication Children 37
or hospital admission under neurologic observation erm in ants 1.56
or early detection o possible delayed EDH.

Answers to Case 3 Variable thresholds have been suggested or the


initiation o treatment or raised ICP in patients
1. D. Image 73-3 is an axial cut o C head shows mul- with head injury. Most neurosurgical centers use a
tiple hemorrhagic brain contusions (le rontal and sustained ICP 2025 mm Hg. An intraventricu-
bitemporal). T is is a case showing a right temporal lar catheter is the most accurate way o monitor-
hemorrhagic brain contusion. It is also called di use ing. T e goal o ICP management is to keep it below
post-traumatic intracerebral hemorrhage and usu- 20 mm Hg provided that the CPP is not lowered
ally appears on C scanning during the immediate below 50 mm Hg (not 70 mm Hg) to guard against
post-traumatic phase as an intra-axial hyperdensity cerebral ischemia.
indicating resh blood within the brain parenchyma.
CBF = CPP / CVR = (MAP - ICP) / CVR
2. D. T e degree o temporal involvement o en plays
a critical role due to its proximity to the brain stem, CBF is the cerebral Blood Flow
whereas mass e ect is better tolerated in a ected CVR is the Cerebrovascular Resistance
supratentorial brain areas. T ere ore, the development MAP is the Mean Arterial Pressure
o temporal hematomas carries a lower threshold or CPP is the Cerebral Per usion Pressure
surgical intervention. T e indications or surgery in ICP is the Intracranial Pressure
cases o traumatic brain contusions include hema- T e average cerebral blood ow usually ranges
toma causing progressive neurological deteriora- rom 50 to 60 mL/min/100 g o brain tissue during
tion; signs o signi cant mass e ect on C ; medically resting state. Normal white matter receives 1825 mL/
re ractory intracranial hypertension; patients with min/100 g, while normal gray matter receives 67
GCS scores o 6 to 8 with a rontal or temporal contu- 80 mL/min/100 g.
sion greater than 20 cm 3 in volume and with a midline
shi o at least 5 mm and/or cisternal compression on 4. D. Hyperglycemia aggravates cerebral edema. Brain
C scan, and patients with any lesion greater than 50 cell swelling is increased by hyperglycemia and keto-
cm 3 in volume. Patients with such eatures should be sis especially during hypocapnia. T ere ore, Dextrose
treated operatively with decompression and possible 10% is not considered appropriate in cases o head
resection o the a ected area. trauma as it will urther increase the ICP impeding
Cerebral contusions most requently develop a er both the CPP and CBF. Osmotic medical therapy
a mechanical impact o distinct skull areas onto the includes mannitol and/or hypertonic saline (used
rather so brain substance in moments in which to control the ICP in traumatic brain injury). Furo-
the brain bounces. Frontal and temporal lobe tips semide is a loop diuretic that can result in greater
are most commonly a ected by such ocal hemor- reduction o brain water content i administered in
rhagic contusions ollowing translational accel- combination with mannitol as it enhances its e ect
eration. Glucocorticoids are not recommended to on plasma osmolality.

http://surgerybook.net/
2 74 G EN ERAL S U RG ERY EXAM I N ATIO N AN D BO ARD REVI EW

5. B. It is not recommended to use hyperventilation ormation which may determine the source o bleed-
vigorously to lower PaCO2 to less than 25 mm Hg ing in patients with multiple cerebral aneurysms. T e
as a PaCO2 o less than 30 mm Hg may lower the FLAIR sequence has the highest sensitivity in detect-
CBF or distort cerebral autoregulation without con- ing the cases with low-grade SAH. 2* / Gradient echo
sistently lowering ICP. T e goal is to keep PaCO2 at sequence or Susceptibility Weighted Image (SWI)
the lower end o eucapnia (35 mm Hg). T e other may be done as a complementary sequence. Post-
options that are considered routine measures to con- contrast C o the head may be misdiagnosed as SAH
trol ICP are head o bed elevation to 30, light seda- especially i the contrast is injected intrathecal.
tion, intubation or a GCS < 8.
4. C. T e our-vessel cerebral angiogram or catheter
Answers to Case 4 cerebral angiography is the gold standard study or
1. A. Image 73-4 is an axial cut o C head shows di - the evaluation o cerebral aneurysms. C angiog-
use post-traumatic subarachnoid hemorrhage. T is raphy (C A) with 3-dimensional reconstruction
is a case o subarachnoid hemorrhage (SAH) shown imaging may be used instead o catheter angiogra-
in the C in the orm o a hyperdense blood lm in phy in some centers as the sole imaging diagnostic
the subarachnoid cisterns. rauma is still the most study be ore major surgical intervention. C A is a
common cause o SAH. Aneurysmal rupture is the non-invasive tool that may provide prompt, accurate,
most common cause o spontaneous SAH. Ruptured and versatile diagnostic and anatomical in ormation
AVM usually presents in the orm o intracerebral on which clipping can be based. However, unneces-
hemorrhage or intraventricular hemorrhage. SAH sary craniotomies based exclusively on C A ndings
occurs in about 5% o AVM cases. Other causes were also reported.
may include pituitary apoplexy, coagulopathy, and MRA can also detect cerebral aneurysms, but it
cocaine abuse. has lower sensitivity and speci city compared to
the catheter cerebral angiogram especially or aneu-
2. E. Nuchal rigidity may occur in some cases o SAH rysms less than 3 mm diameter. MRA is also use ul in
due to meningeal irritation but it is not a must screening high-risk patients. MRI Di usion (DWI)
or diagnosis. Patients with SAH may present with is the best imaging study or detection o acute brain
increased ICP symptoms, seizures and/or neurologi- ischemia within the rst hours especially when it
cal de cit. Unilateral ptosis with pupillary dilation may involves the brain stem or cerebellum. C per usion
indicate occulomotor nerve compression secondary identi es the area surrounding the in arction called
to a posterior communicating artery (P-comm) aneu- potentially salvageable penumbra that allows or
rysm. Nine percent o P-comm aneurysms present better outcomes i interventional treatment modali-
with unilateral occulomotor nerve palsy, which may ties are available. ranscranial doppler ( CD) is a
indicate impending rupture and prompt intervention non-invasive semi-quantitative technique o ultra-
by a specialized neurovascular team. Cytotoxic brain sound derived blood velocity measurements within
edema is usually seen in head trauma patients (vs. major cerebral vessels (e.g., MCA) through thin
vasogenic brain edema in brain tumor cases). regions o skull bones providing a window. CD is
usually used in SAH to detect cerebral vasospasm
3. D. Lumbar puncture always carries the risk o re- early prior to the clinical delayed ischemic neuro-
bleeding in case o aneurysmal SAH. T ere ore, LP logical de cit. T eoretically, the higher the velocity
is only needed to con rm the diagnosis o clinically o blood ow, the narrower the lumen o the cerebral
suspicious cases o SAH with negative or equivocal blood vessel.
C scans. Sidman et al. reported that C scans o the
head had 100% sensitivity in detecting SAH within 5. F. SAH has numerous possible complications that
the rst 12 hours. Morgenstern and his colleagues may cause neurological deterioration. It is believed
stated that 97.5% o patients with SAH present- that proteinaceous blood products may occlude the
ing to the emergency room with worst headache arachnoid granulations causing secondary commu-
symptoms were adequately diagnosed by C imag- nicating hydrocephalus in 20% o SAH survivors.
ing alone. MRI is valuable in demonstrating subacute Angiographic cerebral vasospasm is a contributing
and chronic hemorrhages due to Methemoglobin actor o delayed neurological de cit in patients with
C H AP TER 7 3 H EAD IN j U RI ES 275

SAH. Broderick and his colleagues reported on the 2. C. Conservative treatment including bed-rest and
causes o morbidity and mortality ollowing aneu- corticosteroids have been shown to be very e cient
rysmal SAH. In their study, re-bleeding was the most in the treatment o patients with hematoma less
important preventable cause o death. than 1 cm without ocal neurologic de cit or mental
It is also well-known that seizures may occur early status changes. Corticosteroids have been posited
ollowing SAH mostly due to the presence o blood as both an anti-in ammatory and anti-angiogenic
in the cisterns which may irritate the cerebral cor- agents. Answer B, Burr-hole drainage with the
tex, but the explanation o late onset seizures remains placement o subdural drain, was shown to decrease
unclear. T e prevalence rate o hyponatremia ollow- signi cantly the recurrence rate o CSDH a er
ing SAH is about 30% to 55%. Hyponatremia may treatment. Answer C, Craniotomy with excision o
lead to seizures, vasospasm, altered mental status, subdural membranes, is indicated when the hema-
or death. It may occur due to syndrome o inappro- toma persistently recurs despite the treatment with
priate ADH (SIADH) secretion or glucocorticoid twist-drill craniostomy or burr hole drainage. It has
de ciency. However, correction o hyponatremia been shown to be sa e and valid technique. Answer
should always be gradual and monitored, as rapid D, Percutaneous twist-drill bedside drainage, is the
corrections may cause central pontine myelinolysis least invasive technique over the above mentioned
(osmotic demyelinating syndrome). techniques. It requires a 0.5 cm incision in the scalp
and has been shown to be a sa e technique. Operat-
ing room twist drill craniostomy, answer E, is a valid
Answers to Case 5 option or the management o CSDH, although, it
1. B. Image 73-5 is an Axial cut o C head shows right does not o er any advantage over the percutaneous
rontoparietal chronic subdural hematoma. Patients twist-drill drainage in terms o recurrence. How-
with chronic anticoagulation/antiplatelet therapies ever, given that CSDH is more requent among the
are considered to be at high risk or chronic subdural elderly, the treatments diverge regarding the periop-
hematoma rom trivial traumas. T ose patients are erative risks related to general anesthesia; burr holes
prone to develop subdural bleeding within days or are requently per ormed under general sedation,
weeks a er small head trauma. T e mechanism is whereas a local anesthetic is usually su cient with
not yet very well understood, although alteration percutaneous drainage, so patients comorbidities
in the blood-hematoma components was ound in have to be taken in consideration.
most anticoagulation treated patients.
For answer A, although patients with uncon- 3. A. Brain atrophy, hydrocephalus, midline shi , and
trolled seizure episodes are more prone to requent hematoma density on C (e.g., high, iso, or low) all
head traumas rom potential recurrent seizure epi- in uence the outcome o CSDHs. raumatic causes,
sodes, those patients dont have higher incidence o sex, hematoma location and age have never been
subdural hematomas a er trivial trauma compared shown to correlate with CSDHs outcome.
to patients with chronic anticoagulation therapy. For
4. A. T e presence o bilateral e usion and poor re-
answer C, there is higher incidence o chronic sub-
expansion is associated with high rate o recurrence.
dural hematoma in male compared to emale, but
For answers B, C, D, E, etiology (e.g., alcoholism,
gender does not seem to in uence the development
head injury, neurosurgery, anticoagulant, coagulopa-
o CSDH a er slight head trauma. In answer D, his-
thy, subarachnoid cyst), interval rom trauma to rst
tory o alcohol abuse associated with coordination
operation, C - ndings (e.g., high, iso, or low den-
and motor problems, consequently more alls, that
sity), and age are not associated with higher recur-
put the patient at risk or recurrent head trauma. But
rence o rate o CSDH a er surgery.
history o alcohol abuse does not show a strict corre-
lation with CSDH a er trauma as coagulation thera- BIBLIOGRAPHY
pies do. For answer E, although old people (e.g., age
Agid R, Andersson , Almqvist H, Willinsky RA, Lee SK,
> 65 years) have a higher incidence o CSDH com- terBrugge KG, et al. Negative C angiography ndings
pared to those with age < 65, age is not considered in patients with spontaneous subarachnoid hemorrhage:
a risk actor or the development CSDH a er head When is digital subtraction angiography still needed?
trauma. AJNR Am J Neuroradiol. 2010;31:696705.
276 G EN ERAL S U RG ERY EXAM I N ATI O N AN D BO ARD REVIEW

Ajlan A, Marcoux J. Intracranial Pressure Management. In: Edlow JA, Caplan LR. Avoiding pit alls in the diagnosis
Nader R, Sabbagh AJ, eds. Neurosurgery Case Review. New o subarachnoid hemorrhage. N Engl J Med. 2000;342:
York, NY: T ieme; 2010:1747. 2936.
Araujo JL, Aguiar Udo P, odeschini AB, Saade N, Veiga JC. Ford LE, McLaurin RL. Mechanisms o extradural hemato-
Epidemiological analysis o 210 cases o surgically treated mas. Journal o neurosurgery. 1963;20:7609.
traumatic extradural hematoma. Revista do Colegio Gade GF, Becker DP, Miller JD, Dwan PS. Pathology and
Brasileiro de Cirurgioes. 2012;39(4):26871. pathophysiology o head injury. In: Youmans JR, ed. You-
Baechli H, Nordmann A, Bucher HC, Gratzl O. Demographics mans Neurological Surgery. 6th ed. Philadelphia, PA:
and prevalent risk actors o chronic subdural haematoma: Elsevier Saunders; 1990:19652016.
Results o a large single-center cohort study. Neurosurg Germanwala AV, Huang J, amargo RJ. Hydrocephalus a er
Rev. 2004;27:2636. aneurysmal subarachnoid hemorrhage. Neurosurg Clin N
Binder DK, Lyon R, Manley G . ranscranial motor evoked Am. 2010;21:26370.
potential recording in a case o Kernohans notch syn- Gharaibeh KA, Craig MJ, Koch CA, Lerant AA, Fulop ,
drome: case report. Neurosurgery. 2004;54(4):9991002; Csongradi E. Desmopression is an e ective adjunct treat-
discussion 10023. ment or reversing excessive hyponatremia overcorrection.
Boon JM, Abrahams PH, Meiring JH, Welch . Lumbar punc- World J Clin Cases. 2013;1:1558.
ture: Anatomical review o a clinical skill. Clinical anatomy. Glaser N, Bundros A, Anderson S, ancredi D, Lo W, Orgain M,
2004;17(7):54453. et al. Brain cell swelling during hypocapnia increases with
Brain rauma Foundation. Steroids. Journal o Neurotrauma, hyperglycemia or ketosis. Pediatr Diabetes. 2014;15(7):484
2007;24(Suppl 1):S-91-S-95. doi:10.1089/neu.2007.9981 93.
Brain rauma Foundation. Cerebral per usion thresholds. Greenberg MS. Handbook o Neurosurgery. New York, NY:
Journal o Neurotrauma, 2007;24(Suppl 1):S-59-S-64. doi: T ieme Publishers; 2010.
10.1089/neu.2007.9987 Greenwood R. Head injury or neurologists. J Neurol Neuro-
Brain rauma Foundation. Intracranial pressure monitoring surg Psychiatry. 2002;73(1):i816.
technology. Journal o Neurotrauma, 2007;24(Suppl 1): Gurdjian ES. Cerebral contusions: re-evaluation o the mech-
S-45-S-54. doi:10.1089/neu.2007.9989 anism o their development. J rauma. 1976;16:3551.
Brain rauma Foundation. Intracranial pressure thresholds. Hackney DB, Lesnick JE, Zimmerman RA, Grossman RI,
Journal o Neurotrauma. 2007;24(Suppl 1):S-55-S-58. doi: Goldberg HI, Bilaniuk L . MR identi cation o bleeding
10.1089/neu.2007.9988 site in subarachnoid hemorrhage with multiple intracra-
Bricolo AP, Pasut LM. Extradural hematoma: oward zero nial aneurysms. J Comput Assist omogr. 1986;10:87880.
mortality. A prospective study. Neurosurgery. 1984;14(1): Hannon MJ, Behan LA, OBrien MM, ormey W, Ball SG,
812. Javadpur M, et al. Hyponatremia ollowing mild/moderate
Broderick JP, Brott G, Duldner JE, omsick , Leach A. Ini- subarachnoid hemorrhage is due to SIAD and glucocorti-
tial and recurrent bleeding are the major causes o death coid de ciency and not cerebral salt wasting. J Clin Endo-
ollowing subarachnoid hemorrhage. Stroke. 1994;25: crinol Metab. 2014;99:2918.
13427. Hasan D, Schonck RS, Avezaat CJ, anghe HL, van Gijn J, van
Bullock MR, Chesnut R, Ghajar J, Gordon D, Hartl R, Newell der Lugt PJ. Epileptic seizures a er subarachnoid hemor-
DW, et al. Surgical management o depressed cranial rac- rhage. Ann Neurol. 1993;3:28691.
tures. Neurosurgery. 2006;58:S5660; discussion Si-iv. Hamilton MG, Rashidi A, Rezaii J, et al. T e role o postopera-
Chen PR, Amin-Hanjani S, Albuquerque FC, McDougall C, tive patient posture in the recurrence o traumatic chronic
Zabramski JM, Spetzler RF. Outcome o oculomotor subdural hematoma a er burr hole surgery. Neurosurgery.
nerve palsy rom posterior communicating artery aneu- 2007;61(4):7947; discussion 797.
rysms: Comparison o clipping and coiling. Neurosurgery. Heula A-L, Ohlmeier S, Sajanti J, Majamaa K. Characteriza-
2006;58:10406; discussion 10401046. tion o chronic subdural hematoma uid proteome. Neu-
da Rocha AJ, da Silva CJ, Gama HP, Baccin CE, Braga F , rosurgery. August 2013;73(2):31731.
Cesare Fde A, et al. Comparison o magnetic resonance Hoen B, Viel JF, Gerard A, Dureux JB, Canton P. Mortality in
imaging sequences with computed tomography to detect pneumococcal meningitis: A multivariate analysis o prog-
low-grade subarachnoid hemorrhage: Role o uid- nostic actors. Eur J Med. 1993;2:2832.
attenuated inversion recovery sequence. J Comput Assist Jamieson KG, Yelland JD. Surgically treated traumatic subdural
omogr. 2006;30:295303. hematomas. Journal o Neurosurgery. 1972;37(2):13749.
Daele JJ, Go art Y, Machiels S. raumatic, iatrogenic, and Jennette B. Epilepsy Af er Non-missile Head Injuries. London:
spontaneous cerebrospinal uid (CSF) leak: Endoscopic Heinemann Medical;1975:179.
repair. B-EN 7. 2011;17:4760. Kentaro M, Minoru M. Surgical treatment o chronic subdu-
Dorai Z, Hynan LS, Kopitnik A, Samson D. Factors related ral hematoma in 500 consecutive cases: clinical charac-
to hydrocephalus a er aneurysmal subarachnoid hem- teristics, surgical outcome, complications, and recurrence
orrhage. Neurosurgery. 2003;52:7639; discussion 769 rate. Neurol Med Chir. August 2001;41(8):37181.
71. Kim HS, Hur JW, Lee JW, Lee HK. Extraordinarily long-term
Dunn L . Raised intracranial pressure. J Neurol Neurosurg post-traumatic cerebrospinal uid stula. J Korean Neuro-
Psychiatry. 2002;73(1):i2327. surg. 2007;42:403405.
C H AP TER 7 3 H EAD IN j U RI ES 277

Kolias AG, Sinha R, Park H, et al. Surgical management o Phonprasert C, Suwanwela C, Hongsaprabhas C, Prichayudh P,
chronic subdural hematomas: In need o better evidence. OCharoen S. Extradural hematoma: Analysis o 138 cases.
Acta Neurochir. 2013;155:1834. T e Journal o trauma. 1980;20(8):67983.
Lackner P, Vahmjanin A, Hu Q, Kraf PR, Rolland W, Zhang Rahman M, Friedman WA. Hyponatremia in neurosurgical
JH. Chronic hydrocephalus a er experimental subarach- patients: clinical guidelines development. Neurosurgery.
noid hemorrhage. PLoS One. 2013;8:e69571. 2009;65(5):92535; discussion 9356.
Leggate JR, Lopez-Ramos N, Genitori L, Lena G, Choux M. Riesgo P, Piquer J, Botella C, Orozco M, Navarro J, Cabanes J.
Extradural haematoma in in ants. British journal o neuro- Delayed extradural hematoma a er mild head injury:
surgery. 1989;3(5):5339. report o three cases. Surgical neurology. 1997;48(3):22631.
Lee KS. T e pathogenesis and clinical signi cance o trau- Rivas JJ, Lobato RD, Sarabia R, Cordobes F, Cabrera A, Gomez P.
matic subdural hygroma. Brain Inj. 1998;12:595603. Extradural hematoma: Analysis o actors in uencing the
Lee KS, Bae WK, Doh JW, Bae HG, Yun IG. Origin o chronic courses o 161 patients. Neurosurgery. 1988;23(1):4451.
subdural haematoma and relation to traumatic subdural Ross JS, Masaryk J, Modic M , Ruggieri PM, Haacke EM,
lesions. Brain Inj. 1998;12:90110. Selman WR. Intracranial aneurysms: evaluation by MR
LeFeuvre D, aylor A, Peter JC. Compound depressed angiography. AJNR Am J Neuroradiol. 1990;11:44955.
skull ractures involving a venous sinus. Surg Neurol. Ryall RG, Peacock MK, Simpson DA. Use ulness o beta
2004;62:1215; discussion 1256. 2-trans errin assay in the detection o cerebrospinal uid
Lind CR, Lind CJ, Mee EW. Reduction in the number o leaks ollowing head injury. J Neurosurg. 1992;77:7379.
repeated operations or the treatment o subacute and Saramma P, Menon RG, Srivastava A, Sarma PS. Hyponatre-
chronic subdural hematomas by placement o subdural mia a er aneurysmal subarachnoid hemorrhage: Implica-
drains. J Neurosurgery. 2003;99:446. tions and outcomes. J Neurosci Rural Pract. 2013;4:2428.
Lindegaard KF, Nornes H, Bakke SJ, Sorteberg W, Nakstad P. Sawiris N, Venizelos A, Ouyang B, Lopes D, Chen M. Current
Cerebral vasospasm a er subarachnoid haemorrhage utility o diagnostic catheter cerebral angiography. J Stroke
investigated by means o transcranial Doppler ultrasound. Cerebrovasc Dis. 2014;23(3):e14550.
Acta Neurochir Suppl. 1988;42:8184. Selariu E, Zia E, Brizzi M, Abul-Kasim K. Swirl sign in intrace-
Lindvall Peter, Lars-Owe D. Koskinen. Anticoagulants and rebral haemorrhage: De nition, prevalence, reliability and
antiplatelet agents and the risk o development and recur- prognostic value. BMC neurology. 2012;12:109.
rence o chronic subdural haematomas. Journal o Clinical Shapiro SA, Scully . Closed continuous drainage o cerebro-
Neuroscience. 2009;16:128790. spinal uid via a lumbar subarachnoid catheter or treat-
Macdonald RL. Delayed neurological deterioration a er sub- ment or prevention o cranial/spinal cerebrospinal uid
arachnoid haemorrhage. Nat Rev Neurol. 2014;10:4458. stula. Neurosurgery. 1992;30:2415.
Malik NK, Makhdoomi R, Indira B, Shankar S, Sastry K. Pos- Shen YS, ai CH, Chen JH, Chen WL, Yao C , Wu YL, et al.
terior ossa extradural hematoma: Our experience and Aneurysmal rebleeding episode a er lumbar puncture. Am
review o the literature. Surgical neurology. 2007;68(2):155 J Emerg Med. 2007;25(8):984.e13.
8; discussion 158. Sidman R, Connolly E, Lemke . Subarachnoid hemorrhage
Meier U, Gartner F, Knop W, Klotzer R, Wol O. T e trau- diagnosis: Lumbar puncture is still needed when the
matic dural sinus injury: A clinical study. Acta Neurochir. computed tomography scan is normal. Acad Emerg Med.
1992;119:9193. 1996;3:82731.
Morgan M, Sekhon L, Rahman Z, Dandie G. Morbidity o Stanisic M, Lyngstadaas SP, Pripp AH, et al. Chemokines
intracranial hemorrhage in patients with cerebral arterio- as markers o local in ammation and angiogenesis in
venous mal ormation. Stroke. 1998;29:20012. patients with chronic subdural hematoma: A prospective
Morgenstern LB, Luna-Gonzales H, Huber JC, Jr., Wong SS, study. Acta Neurochir. 2012;154:11320.
Uthman MO, Gurian JH, et al. Worst headache and sub- Su M, Lee H, Lee C, Cheng CH, Lu CH. Acute clinical
arachnoid hemorrhage: prospective, modern computed deterioration o posterior ossa epidural hematoma: Clini-
tomography and spinal uid analysis. Ann Emerg Med. cal eatures, risk actors and outcome. Chang Gung medical
1998;32:297304. journal. 2012;35(3):27180.
Nabavi DG, Cenic A, Craen RA, Gelb AW, Bennett JD, Kozak R, Sweni S, Senthilkumaran S, Balamurugan N, T irumalaikolun-
et al. C assessment o cerebral per usion: Experimen- dusubramanian P. ension pneumocephalus: A case report
tal validation and initial clinical experience. Radiology. with review o literature. Emerg Radiol. 2013;20:5738.
1999;213:1419. ahir MZ, Khan MB, Bashir MU, Akhtar S, Bari E. Cerebro-
Obrist WD, Lang tt W, Jaggi JL, Cruz J, Gennarelli A. Cere- spinal uid rhinorrhea: An institutional perspective rom
bral blood ow and metabolism in comatose patients with Pakistan. Surg Neurol Int. 2011;2:174.
acute head injury. Relationship to intracranial hyperten- sai FY, eal JS, Hieshima GB. Neuroradiology o Head
sion. J Neurosurg. 1984;61:24153. rauma. Baltimore, MD: University Park Press; 1984.
Ozer FD, Yurt A, Sucu HK, ektas S. Depressed ractures over van Gijn J, Kerr RS, Rinkel GJ. Subarachnoid haemorrhage.
cranial venous sinus. J Emerg Med. 2005;29:1379. Lancet. 2007;369:30618.
Perry JJ, Stiell IG, Sivilotti ML, Bullard MJ, Hohl CM, Suther- Wilberger JE, Jr., Harris M, Diamond DL. Acute subdural
land J, et al. Clinical decision rules to rule out subarachnoid hematoma: Morbidity, mortality, and mortality, and opera-
hemorrhage or acute headache. JAMA. 2013;310:124855. tive timing. Journal o neurosurgery. 1991;74(2):2128.
278 G EN ERAL S U RG ERY EXAM I N ATI O N AN D BO ARD REVIEW

Winston KR. E cacy o dural tenting sutures. Journal o neu- and transcranial magnetic stimulation. Journal o
rosurgery. 1999;91(2):1804. neurology, neurosurgery, and psychiatry. 2008;79(11):
Yose Laviv, Zvi Harry Rappaport. Risk actors or develop- 12957.
ment o signi cant chronic subdural hematoma ollowing Zhao J, Liu Z, Liu Y, et al. [E ect o epidural drainage and
conservative treatment o acute subdural hemorrhage. Br J dural tenting suture on epidural hematoma in 145 cases
Neurosurg. May 2014;19:16. o craniotomy.] Zhong nan da xue xue bao Yi xue ban =
Yoo WK, Kim DS, Kwon YH, Jang SH. Kernohans notch Journal o Central South University Medical sciences (J Cent
phenomenon demonstrated by di usion tensor imaging S Univ Med Sci). March 2010;35(3):2736.
74
NeurosurgeryHyponatremia/
Hypernatremia
Albert Jesse Schuette

A 38-year-old male presents to the trauma bay with a A. Fluid bolus


Glasgow coma scale (GCS) o 15 ollowing a motor vehi- B. Hypertonic saline
cle crash. T e patient undergoes a computed tomogra- C. Fluid restriction
phy (C ) scan o the head that shows a small traumatic D. Vaptan therapy
subarachnoid hemorrhage over the right rontal lobe E. IV steroid dose
and tiny contusions in the right rontal and le occipi-
tal lobes. Overnight the patient becomes slightly more 3. Which o the ollowing statements is true regarding
agitated but his GCS remains stable at 15. A C scan the the neurological signs and symptoms associated
morning a er admission shows no change in his intrac- with severe hyponatremia?
ranial injuries. Additionally, his chemistry panel the ol- A. Usually not apparent until Na drops below
lowing morning shows a sodium level o 130 mEq/L. 115 mEq/L.
His admission sodium was noted to be 139 mEq/L. B. I the hyponatremia is chronic (> 48 hours in
duration), the patient will be symptomatic.
1. A er determining that the ef ective serum C. I symptoms are present, the physician should
osmolality was within the normal range and rapidly correct the sodium to near normal levels
urine osmolality was greater than 100 mOsm/kg, at a rate o 57 mEq/L/hr.
the next step in determining the cause o the D. Symptoms typically include headache and poor
hyponatremia is: concentration.
A. Assess the patients volume status. E. Hyponatremia can be atal due to transtentorial
B. Determine the liver unction and cardiac output. herniation.
C. Check pituitary unction assays or cortisol
4. T e patients neurological exam has now deteriora-
derangements.
ted requiring intubation, external ventricular drain
D. Draw an arterial blood gas to determine the pH
placement, and aggressive intracranial pressure
o the blood.
(ICP) management. T e patients urine output has
E. Determine the urine sodium level.
increased to over 300 cc/hr and the serum sodium
2. Your team has determined that the patient is has increased to over 150 mOsm/hr. What diagnosis
euvolemic and has given him the diagnosis o would you assign the patient at this time and what is
syndrome o inappropriate antidiuretic hormone the likely cause?
secretion (SIADH). His sodium on last draw was A. Nephrogenic diabetes insipidus due to medica-
128. On last exam, the patient shows no symptoms tions
o hyponatremia. What treatment would you begin B. Neurogenic diabetes insipidus due to undiag-
to correct the low serum sodium level? nosed skull racture
280 G EN ERAL S U RG ERY EXAM I N ATI O N AN D BO ARD REVIEW

C. Neurogenic diabetes insipidus due to herniation rologic symptoms o mild hyponatremia or chronic
D. Neurogenic diabetes insipidus due to underlying hyponatremia involve headache, poor concentra-
meningitis tion, anorexia, and muscle weakness. T e more acute
E. Nephrogenic diabetes insipidus due to underly- and severe hyponatremia cases can lead to cerebral
ing kidney disease edema, nausea, seizures, respiratory arrest, coma,
and death rom transtentorial herniation. Symptoms
5. Which o the ollowing is part o the best treat- generally are easily diagnosed below 125 mEq/L.
ment option or this patient? Symptoms are more apparent with acute onset
A. Monitor uid intake and output hourly and draw (< 48 hours in duration) as there is less time or
the serum sodium every 24 hours. mechanisms o the brain to compensate. For that rea-
B. Place on a basal uid rate with D5 NS with son, chronic hyponatremia (> 48 hours) can be rela-
replacement or urine output above the base rate. tively asymptomatic even at low Na levels. T e rate
C. Administer DDAVP rst and assess impact on o correction even in symptomatic and acute cases
urine output. should not exceed 1 to 2 mEq/L/hr and 810 mEq/L
D. Allow the patients thirst mechanism to control in 24 hours to avoid central pontine myelinolysis.
the Na level and treat with DDAVP as needed.
4. C. Hypernatremia is de ned as a serum sodium
ANSWERS > 150 mEq/L. In patients with neurologic injuries,
this is commonly due to diabetes insipidus (DI). DI
1. A. T e diagnosis or the cause o hyponatremia is is due to low levels o ADH or renal insensitivity to
generally reached in an algorithmic ashion. A er ADH. T e diagnosis can be reached by high output o
a lab measurement shows serum sodium less than inappropriately dilute urine with high serum osmo-
135 mEq/L, you must determine the ef ective serum lality and high serum sodium. T e two major eti-
osmolality. T is rules out hyperglycemia, hypertri- ologies are neurogenic DI and nephrogenic DI. T e
glyceridemia, and hyperproteinemia (so called pseu- primary causes o nephrogenic DI are medications,
dohyponatremia). T e next step is to evaluate urine renal disease, hypokalemia, and hypercalcemia. Neu-
osmolality to ensure that the patient is not suf ering rogenic DI can be idiopathic, posttraumatic, tumors,
rom water intoxication. A er these three steps are in ections, vascular, and autoimmune. In a posttrau-
complete, the patients volumes status must be ascer- matic patient who has worsening mental status, the
tained. T is is extremely important in a neurologic cause is most likely due to herniation rom pituitary
patient as it will help dif erentiate the most com- stalk injury or hypothalamic injury rom impending
mon neurologic causes o hyponatremia: syndrome brain death.
o inappropriate antidiuretic hormone secretion
(SIADH) and cerebral salt wasting (CSW). 5. B. A er arriving at the diagnosis o central diabe-
2. C. As with the diagnosis, the treatment or SIADH tes insipidus, it is essential to begin treatment o the
also runs as an algorithm. Again, the patient must be patient. T e treatment o DI is based around anti-
con rmed to be euvolemic as the treatment or CSW diuretic hormone (ADH) analogues. T ese include
involves uid resuscitation and or SIADH o en Pitressin and the longer acting, more potent orm,
involves uid restriction. In the case o a patient with DDAVP. In a conscious and ambulatory patient,
mild hyponatremia (125135 mEg/L) with no symp- o en their thirst mechanisms will compensate or
toms, treatment should involve uid restriction. I the hypernatremia. However, the patient in this
there are symptoms, patients should receive saline scenario is unable to drink or compensate with his
in usion with 0.9% saline and Lasix. Finally, in cases thirst mechanism, so D is clearly incorrect. I the
with Na levels less than 125 mEq/L and symptoms, patient is not able to drink, they should be monitored
patients are treated 3% saline and Lasix. Newer med- or Is and Os q1h with serum sodium draws every
ications such as the vaptan class, which act as a vaso- 6 hours. o deal with the uid loss, the patient should
pressin antagonist, can be used in re ractory cases. be placed on a basal uid rate with D5 NS with
replacement or urine output above the base rate. I
3. E. T is question relates to the symptoms and gen- the uid replacement continues to all behind and
eral treatment options o hyponatremia. T e neu- the hypernatremia worsens, Pitressin or DDAVP
C H AP TER 7 4 N EU RO S U RG ERY H YP O N ATREM I A/ H YP ERN ATREM IA 2 81

may be given. T ough all the possible answers are Greenberg MS. Handbook of neurosurgery. New York, NY:
treatments or DI, only the rst three could be uti- T ieme Publishing Company; 2010.
lized or this patient. Maghnie M. Diabetes insipidus. Hormone research. 2003;
59(1):4254.
Peri A, Giuliani C. Management o euvolemic hyponatremia
BIBLIOGRAPHY attributed to SIADH in the hospital setting. Minerva endo-
Adrogue HJ, Madias NE. Hyponatremia. T e New England crinologica. March 2014;39(1):3341.
journal of medicine. May 25, 2000;342(21):15819. Petersen RC. Acute con usional state. Dont mistake it or
Chanson P, Salenave S. reatment o neurogenic diabetes dementia. Postgraduate medicine. December 1992;92(8):
insipidus. Annales dendocrinologie. December 2011;72(6): 1418.
4969.
75
Ophthalmology rauma

Morohunranti O. Oguntoye and Robert A. Mazzoli

T ere is an ammonium nitrate explosion at a local er- suspect a ruptured globe and begin management
tilizer plant, with multiple injured persons and atalities until the ophthalmologist arrives. Which o the
at the scene. Pre-hospital EMS providers, rst respond- ollowing would be contra-indicated in the initial
ers, and bystanders begin to administer rst aid to the management o this patient?
injured and start trans er o patients to the local hos- A. Ultrasound
pital. As the on-call emergency medicine physician, B. Fox Shield/Rigid Eye Shield
you and your colleagues stand ready in the emergency C. Antibiotics
department while preparing or a potential mass casu- D. X-Ray/C
alty event. As part o the report or incoming injuries, E. etanus shot
you are noti ed that there are multiple ace and eye
injuries. T e on-call ophthalmologist is on his way, but 3. A worker is hit in the le eye/orbit with a large
he is being delayed by the tra c that was caused by the piece o ying debris. T e patient is in severe pain,
explosion. You have been designated to manage oph- and is unable to open his eye. Gentle palpation o
thalmic injuries until he arrives. the le eye reveals a markedly swollen orbit that
is dif cult to retropulse. T e eyelids are tense
1. A 35-year-old male involved in the explosion was but you are able to gently open the lids with a
splashed in the ace with ammonia. His ace was retractor. Vision is light perception only. You note
irrigated with water at the scene, but he complains the pupil dilates when you shine light in it, but the
o persistent eye pain and decreased vision. contralateral pupil then constricts as you swing the
T e rst priority in the initial care o the patient light to it. Extraocular motility is severely limited.
should be? What is your diagnosis and plan or management?
A. Obtaining visual acuity A. Open globelateral canthotomy and cantholysis
B. Neutralization o pH with an acidic solution B. Orbital blowout ractureorbital decompres-
C. Copious irrigation with non-caustic solution sion
D. Obtaining eye pressure C. Retrobulbar hemorrhagepressure patch in
E. Pain control anticipation o surgical intervention
D. Retrobulbar hemorrhagelateral canthotomy
2. You examine a 45-year-old emale worker who and cantholysis
ell during the explosion, striking her eye on the E. Periorbital ecchymosisice packs and pain con-
side o a work bench. On gently retracting the trol
lids, the globe appears so and irregular with
di use subconjunctival hemorrhage and swelling. 4. T ere is an elementary school next to the ertilizer
She can only count ngers with that eye. You plant. A 7-year-old child was struck in the right ace
C H AP TER 7 5 O P H TH Al m O l O g y TRAu m A 283

and orbit with ying debris, resulting in a brie loss o I there is any suspicion or an open globe, irri-
consciousness. He has minimal bruising on his ace gation should be gentle but thorough and tap water
and he has no complaints other than mild discom ort should be avoided because o potential intraocular
around that eye. As you conduct your exam, you note damage. Never use acidic solutions to neutralize
that he avoids looking at you, pre erring to keep his alkalis or vice versa, as these can generate harm ul
eyes shut. His vision, intraocular pressure (IOP), and substrates. opical anesthetic can be used to acilitate
pupil exam appear normal, but when you instruct the irrigation process and provide the patient some
him to look up as part o the exam, he immediately com ort, but it should not delay care. A lid speculum
vomits. You continue your exam and note that his may be necessary to keep eyes open or irrigation.
heart rate drops dramatically whenever you ask him Upper and lower eyelid ornices should be everted
to move his eyes, prompting you to discontinue the and irrigated. Particulate matter should be ushed
exam. You suspect an orbital racture but the eye out or manually removed. T e conjunctival ornices
and lid are otherwise unremarkable. What is your should be swept with moistened cotton-tipped appli-
concern and urgency? cators to remove any particles or caustic materials.
A. Intracranial hemorrhage: emergent Check the pH in the ornices 5 to 10 minutes af er
B. Orbital oor (blowout) racture: emergent irrigation and resume irrigation i the pH is not
C. Orbital oor (blowout): non-urgent within the normal range. Retained material should
D. Retrobulbar hemorrhage: emergent be suspected i the pH remains abnormal.
E. Carotid dissection: non-urgent T e pH should be rechecked 15 to 30 minutes af er
stabilization. It is also important to identi y the chem-
5. T e on-call ophthalmologist has called to get an ical agent and the time o the injury. Alkali materials
update on the patients you have seen. Which o the (e.g., sodium hydroxide/lye, ammonia, calcium
ollowing physical exam or imaging techniques hydroxide/lime, industrial/chemical detergents)
would be indicated or the given pathology? cause more damage to the eye than acidic materi-
A. Best corrected visual acuity in a retrobulbar als due to deeper penetration. Once the pH has been
hemorrhage. neutralized, a ull eye exam should be conducted.
B. Relative a erent pupillary de ect (RAPD) with Worrisome ndings or extensive damage include:
an open globe injury. signi cantly decreased visual acuity (irrigation itsel
C. Intraocular pressure exam in a chemical burn. may cause a temporary keratitis and mild decrease
D. Ultrasound when computerized tomography in vision), increased IOP, partial and ull-thickness
(C ) scan is not available in a suspected globe burns to the surrounding skin, conjunctival swelling,
rupture. conjunctival blanching, ull thickness penetration/
E. C scan o orbit and brain with intraorbital or- open globe, and an opaque cornea. A shield should
eign body. be applied and the patient trans erred to an ophthal-
mologist immediately or urther care.
ANSWERS
2. A. A history suspicious or open globe will of en
1. C. A chemical injury is a true ophthalmic emer- include a blunt or penetrating trauma to the eye,
gency. While visual acuity and IOP are the vital a eeling o loss o uid rom the eye, pain, and
signs o ophthalmology, in the case o a chemical decreased vision. Determining whether there is an
injury, diluting and removing the chemical agent open globe is one o the most important aspects o
takes priority. Copious but gentle irrigation with any the ocular trauma evaluation, as this requires metic-
non-caustic uid should begin in the prehospital set- ulous initial care to prevent additional injury and an
ting and continue at the hospital until the pH o the emergent evaluation by an ophthalmologist. Signs
ocular sur ace has normalized to between 7.0 and that are concerning or open globe include a sof or
7.2. In an emergency department setting, Lactated irregular-appearing globe, an irregular or peaked
Ringers or normal saline are the pre erred uids or pupil, hemorrhagic swelling o the conjunctiva (espe-
irrigation, but tap water can be used, i necessary. cially i 360 degrees), positive Seidels sign on the
Irrigation should last or at least 30 minutes and may cornea (leakage o uid seen upon staining with uo-
require several liters o irrigant. rescein), hyphema (blood in the anterior chamber),
284 g EN ERAl S u Rg ERy EXAm IN ATI O N AN D BO ARD REVIEW

shield is not available, alternatives include the bottom


o a paper cup, moldable splints (e.g., SAM splints),
or sunglasses/eye protection. Do not apply patches
or gauze pads to an open globe. Do not use topical
anesthetics or ointments. Anti-nausea medications
should be given with any pain medications to pre-
vent Valsalva and/or vomiting. Imaging (pre erably
an axial ace and orbit C scan with coronal recon-
structions) should be obtained to rule out concomi-
tant injuries or unidenti ed retained oreign bodies.
Antibiotics should be given within 6 hours o the
injury (4th generation uoroquinolones IV have
excellent vitreous penetration are pre erred in adults;
Ce azolin 25 to 50 mg/kg/day IV in 3 divided doses
Figure 75-1 Signs o open globe af er blunt trauma to the eye: and Gentamicin 2 mg/kg IV q8h or children). A tet-
irregular pupil, at anterior chamber, and leakage o intraocu- anus shot should be given i necessary. Remember-
lar uids rom the eye. Note the examiners nger positioning,
ing how to deal with an open globe can be acilitated
minimizing pressure on the globe. Source: Dr. Morohunranti
by using the acronym FAC : Fox shield, Antibiotics/
O. Oguntoye.
Anti-emetics/Analgesia, C scan, and etanus shot.
T e eye should be shielded and the patient should be
a shallow or deep anterior chamber (compared to trans erred immediately to the nearest ophthalmolo-
the uninjured eye), decreased extraocular motility, gist or urther care.
oreign body tract, and severe vision loss.
Figure 75-1 shows a patient who presented to the 3. D. In a patient who has had recent blunt trauma
emergency department af er being hit in the eye with or surgery to the eye/orbit, ndings o intense pain,
a branch. He complained o uid leakage rom his decreased vision, an RAPD (the a ected pupil para-
eye, mild to moderate eye pain, and decreased vision. doxically dilates to light, while the normal pupil con-
Note the irregular, cloudy pupil, at anterior cham- stricts to it), inability to open eyelids due to swelling,
ber, and the leakage o intraocular uids (vitreous) at and loss o color vision should raise a strong sus-
the 5 oclock position o the eye. Also note the exam- picion or retrobulbar hemorrhage, which is a true
iners nger positioning, with the ngers at the brow ocular emergency. In contrast to the open globe, the
and maxilla used to hold open the eye. T is allows exam will reveal a tense, proptotic globe that is resist-
or a proper examination without applying pressure ant to retropulsion on gentle palpation. In the case
to the globe, which could lead to urther extrusion o o an orbital blowout racture, the globe is usually
intraocular contents. enophthalmic (sunken in) and there is pain on eye
Once the diagnosis o an open globe is made, ur- movement, but the visual acuity is usually normal
ther examination should be de erred to the ophthal- or only slightly decreased. A patient with periorbital
mologist, who can address the injury at the time o ecchymosis (black eye) may have some periorbital
surgical repair. A visual acuity may be obtained (light tenderness and swelling but should not have globe
perception, hand motion, count ngers are appro- proptosis, signi cantly elevated eye pressure, or vis-
priate measures i the patient cannot see letters on ual acuity changes.
a near card) but measurement o IOP should not Retrobulbar hemorrhage is an orbital compart-
be attempted. No pressure should be applied to an ment syndrome that can rapidly and permanently
eye with a suspected open globe, to include an ultra- damage the optic nerve and retina leading to perma-
sound probe, especially when other clinical ndings nent vision loss i not identi ed and treated immedi-
strongly point to the diagnosis and disposition. Ask ately. T e retinas ischemic tolerance time is roughly
the patient not to strain or squeeze their eyelids. Do 90 minutes. Retrobulbar hemorrhage is a clinical
not remove any penetrating objects. diagnosis and does not require any additional imag-
A rigid eye shield should be applied over the ing. T e key to e ective management o a retrobulbar
patients eye to prevent urther injury. I a rigid eye hemorrhage is timely and aggressive decompression
C H AP TER 7 5 O P H TH Al m O l O g y TRAu m A 285

Figure 75-2 Steps to per orm a lateral canthotomy and cantholysis. (A) injection o local anesthe-
sic, (B) Crush the lateral canthus with straight hemostat, (C) Divide the lateral canthon tendon,
(D) Pull the lower eyelid away rom ace and divide the in erior crus o the canthal tendon, (E)
Divide any remaining attachments so lower eyelid pulls away reely rom ace (Modi ed and
Reproduced rom Department o De ense. Ocular injuries. In: Emergency War Surgery Manual.
4th ed. San Antonio, exas: Borden Institute; 2013:14.8).

with a lateral canthotomy and cantholysis by the rst dure to preserve vision rom sight threatening orbital
provider that is able to per orm the decompression hemorrhage, by Ballard, et al. Following a success-
(see Figure 75-2). T e goal is to disinsert the lower ul lateral canthotomy and cantholysis, additional
eyelid sling rom its periosteal attachments at the measures to reduce the orbital pressure should be
lateral canthus. T e only two instruments needed are instituted, including applying ice packs to the orbit,
tissue orceps, and blunt scissors. I available, local elevation o the head, and administration o systemic
anesthetic and a hemostat can acilitate a smoother analgesia and anti-emetics.
procedure.
A detailed description o lateral canthotomy and 4. B. T is is what is known as a white-eyed blow
cantholysis is ound within the article Emergency out racture or trapdoor racture with oculocar-
lateral canthotomy and cantholysis: A simple proce- diac re ex, and represents an ocular emergency.
286 g EN ERAl S u Rg ERy EXAm I N ATIO N AN D BO ARD REVI EW

Childrens bones are exible, and their orbital bones An eye exam can be broken down into a primary
can break and bend, allowing the rectus muscle to survey (identi ying sight-threatening injuries) and
pass below the broken but non-displaced bone. T is a secondary survey (the complete ophthalmic
can result in a racture in which the rectus muscle is exam). T e primary survey is per ormed using a
trapped below what appears to be an intact orbital muscle light/penlight, uorescein with Woods lamp,
oor. T e external signs can be minimal: T ere may a visual acuity card, an intraocular pressure (IOP)
be no periorbital edema but the patient will exhibit instrument, and an ophthalmoscope. T e secondary
signi cant extraocular muscle restriction. A child survey is per ormed using a slit lamp and an oph-
will of en not complain o double vision, rather he thalmoscope.
may simply close one or both eyes. Key ndings on the primary survey include:
T e oculocardiac re ex can occur when extraocu-
Best corrected visual acuity: Count ngers (CF),
lar muscles are pulled. Immediate signs and symp-
hand motion (HM), and no light perception (NLP)
toms can include nausea, vomiting, and bradycardia
rom best to worst i the patient is unable to visualize
with eye movement. Bradycardia can be so pro ound
the ar or near charts.
as to cause syncope or asystole. T e mani estation o
IOP: not elevated (> 21) or decreased (< 10) (Note:
this re ex is an ocular emergency and an ophthal-
do not per orm i suspecting open globe).
mologist should be consulted immediately or de ni-
Globe: well formed, no leakage of uid, free extraoc-
tive management.
ular motility, no proptosis, not tense to retropulsion,
Adults with orbital oor ractures will complain o
no oreign bodies, enophthalmos/exophthalmos, no
pain on attempted eye movement, binocular vertical
oculocardiac re ex.
diplopia, and hypoesthesia in the distribution o the
Cornea: uorescein exam without evidence of up-
in raorbital nerve V2 (ipsilateral cheek and upper lip).
take or leakage, no corneal clouding, no oreign bod-
Signs may include enophthalmos, crepitus, subcutane-
ies [NB: contact lenses should be removed].
ous emphysema, restricted eye movement, nosebleeds,
Conjunctiva: no swelling, no blanching, no subcon-
bony point tenderness, and step-o de ormities o the
junctival hemorrhage.
orbital rim. I the patient had an intracranial hemor-
Anterior chamber: well formed, no in ammation, no
rhage, dilated pupils and rapidly declining mental sta-
blood (hyphema).
tus should be expected. A retrobulbar hemorrhage or
Pupil: reactive to light and accommodation, not
carotid dissection would cause an RAPD. Periorbital
peaked/irregular/torn.
ecchymosis should not cause vomiting and/or brady-
Posterior segment: no optic nerve swelling, no vitre-
cardia. While orbital blowout ractures in children can
ous hemorrhage.
be an ocular emergency, in adults without an RAPD
or oculocardiac re ex, ophthalmic re erral can be Key ndings on the secondary survey:
delayed up to 24 to 48 hours. An RAPD, ashes, oat- Orbit/Globe: crepitus, step-o s/deformities, numb-
ers, photophobia, or decreased vision should prompt ness on cheek, teeth, or orehead.
an immediate ophthalmic consult. Lids: laceration, medial canthus rounding, canalicu-
lar lacerations, epiphora, tense lids, ecchymosis, or-
5. E. Ultrasound is contraindicated in an open globe nix evaluation, singed lashes.
because pressure on the globe may lead to extrusion Conjunctiva: laceration.
o intraocular contents. T e initial visual acuity in Cornea: corneal edema.
an open globe correlates with trauma severity and is Iris: iris tears, photophobia.
highly predictive o nal visual outcome. T e RAPD Anterior segment: lens dislocation.
in a patient with a retrobulbar hemorrhage indicates Posterior segment: retinal detachment.
ocular compartment syndrome, requiring emergent
decompression. Fluorescein exam in a chemical burn
indicates the extent o the corneal damage. In the BIBLIOGRAPHY
case o penetrating ocular or orbital injury, a C scan
Ballard S, Enzenauer R, et al. Emergency lateral canthotomy
should always be obtained to determine the location and cantholysis: A simple procedure to preserve vision
o intraorbital oreign bodies and to rule out optic rom sight threatening orbital hemorrhage. Journal o
nerve or central nervous system involvement. Special Operations Medicine. 2009;9:2632.
C H AP TER 7 5 O P H TH Al m O l O g y TRAu m A 287

Gerstenblith A , Rabinowitz MP. T e Wills eye manual: Of ce Joint T eater rauma System Clinical Practice Guide-
and emergency room diagnosis and treatment o eye disease. line. Initial care o ocular and adnexal injuries by non-
Philadelphia, PA: Lippincott Williams & Wilkins, 2012. ophthalmologists at Role 1, Role 2, and non-ophthalmic
Hemmati HD, and Colby KA. Ophthalmic pearls: Cornea. Role 3 Facilities. CEN COM. November 2014.
treating acute chemical injuries o the cornea. San Francisco, Lima Vanessa, et al. Orbital compartment syndrome: the
CA: American Academy o Ophthalmology EyeNet Maga- ophthalmic surgical emergency. Survey o Ophthalmology.
zine; October 2012. July-August 2009;54:441441.
76
T oracic rauma

Booker . King

1. A 22-year-old male who sustained a gunshot with a heart rate o 65 bpm. What is the most
wound to the chest arrives in the emergency room appropriate management o this patient?
at your acility. He is hemodynamically stable and A. Admission to the intensive care unit with serial
has an entry wound just below his right nipple and cardiac panel and repeat chest lm in 24 hours.
probable exit wound just below the tip o the le B. Echocardiogram per ormed in the emergency
scapula. Supine chest lm shows no retained bullet room to evaluate cardiac wall motion.
ragments, contusion o the lung and moderate le C. Discharge home with instructions to return to
apical pneumothorax. A pericardial ultrasound the hospital i the symptoms worsen.
shows no ef usion or other abnormality. T e initial D. Consultation with cardiologist or suspected
diagnostic work-up and management o this injury acute coronary syndrome.
should include: E. Repeat C scan o the chest in 12 hours.
A. Computerized tomography angiogram (C A) o
the chest a er placement o le chest thoracos- 3. A 33-year-old emale was admitted a er being
tomy tube. involved in a motor vehicle collision ve days ago.
B. Placement o le thoracostomy tube and obser- She had a le chest tube placed upon admission
vation on the ward. or a hemothorax. Initially, 150cc o blood was
C. rans-esophageal echocardiogram, non-contrast evacuated but a er 24 hours, only additional 100cc
C scan o the chest, and le chest thoracostomy came out. A repeat C scan o the chest now shows
tube. a retained hemothorax and there has been minimal
D. Bronchoscopy, esophagoscopy, esophagography, drainage rom the chest tube or the past 48 hours.
a catheter angiogram o the thoracic aorta, and T e most appropriate management o this patient
le chest thoracostomy tube. at this time would be:
E. Immediate surgical exploration via a median A. Video assisted thorascopic surgery (VA S) to
sternotomy. evacuate the hemothorax.
B. VA S i hemothorax does not resolve in three
2. A 50-year-old male is brought to a local more weeks o observation.
emergency room a er a motor vehicle collision. C. Instill thrombolytic agent into the chest cavity
He is complaining o mild mid-sternal chest pain via the chest tube to break up the clot.
and has some bruising over the sternum. He is a D. T oracotomy or decortication and evacuation
healthy male with no cardiac history. C scan o o hemothorax.
the chest, abdomen, and pelvis are unremarkable. E. C scan guided aspiration o retained hemo-
Electrocardiogram shows normal sinus rhythm thorax.
C H AP TER 7 6 TH o RAC i C TRAu m A 289

4. A 21-year-old male sustained multiple injuries ANSWERS


a er he struck a tree while riding a motorcycle
traveling at 60 mph. He was intubated at the scene 1. A. Although the majority o thoracic gunshot wounds
or unresponsiveness. He was transported to will be localized to one side o the thoracic cavity, it
your medical center and trauma evaluation was is critical to identi y the smaller subgroup o patients
signi cant or a 2 cm right-sided rontal cerebral with a trans-mediastinal trajectory. rans-medias-
contusion, racture o ribs 2 through 6 on the tinal gunshot wounds carry a signi cantly higher
right, right clavicle racture and moderate right mortality and morbidity, and can present a diagnos-
pulmonary contusion. A right-sided chest tube was tic and management dilemma due to the number o
placed or large pneumothorax. Repeat chest lm critical structures at risk o injury. Unstable patients
shows a persistent pneumothorax so a second chest require immediate intervention and surgical explo-
tube was placed. A third set o chest lms show that ration based on the initial examination and imaging
the lung has re-expanded slightly but a moderate ndings. Stable patients require urther radiologic
apical pneumothorax remains. T e two chest tubes workup, with the goal o excluding injury to the heart,
have a persistent large air leak present. T e next lungs, great vessels, esophagus, and tracheobronchial
step in this patients management should be: tree. Historically this mandated an exhaustive series
o imaging studies including chest X-ray or C scan,
A. Oxygenate with 100% oxygen or 24 hours to
echocardiogram, esophagoscopy and esophagogra-
allow pneumothorax to resolve spontaneously.
phy, thoracic angiography, and bronchoscopy.
B. Repeat chest lm in six hours and planned VA S
With the improved availability and image quality
i pneumothorax persists.
obtained with modern C scanners, this exhaustive
C. Urgent bronchoscopy.
workup has largely been replaced by a single high-
D. Continued observation as long as the patient is
quality C scan o the chest with intravenous con-
stable and oxygenating well.
trast in most trauma centers. In addition to imaging
E. T oracotomy or repair o probable large paren-
the critical mediastinal structures o interest, the
chymal lung laceration.
C scan can be used to re-create the missile tract
5. An 80-year-old male sustained multiple bilateral and assess the proximity to these structures and to
rib ractures and a mild pulmonary contusion a er identi y any secondary signs o injury (Figure 76-1).
a motor vehicle collision. He is hemodynamically Patients with a negative C scan can be sa ely
stable and oxygenating well in the emergency observe, while those with any concern or a potential
room but complains o chest pain upon deep
inspiration. rauma evaluation did not reveal any
other injuries. T e appropriate management o
this patient should be:
A. Discharge to home on oral pain medications and
instructions to return to the hospital i his condi-
tion worsens.
B. Admission to medical-surgical ward with sup-
plemental oxygen and orders or repeat chest
lms in 12 hours.
C. Admission to intensive care unit, pain control
with either a thoracic epidural or patient con-
trolled analgesia (PCA) with ketorolac, and
incentive spirometry.
D. Admission to intensive care unit and schedule
operative xation o rib ractures or the ollow- Figure 76-1 Chest C scan o a transmediastinal gunshot
ing day. wound shows the key mediastinal structures as well as the
E. Admission to intensive care unit and electively trajectory and tract o the missile. (Reprinted courtesy o
intubate the patient as he is at signi cant risk or Aydemir NA, Bakir I, Atin F et al. A magic bullet through the
respiratory ailure. heart. Circulation. 2007;115:e4678.)
290 G EN ERAL S u RG ERY EXAm i N ATi o N AN D Bo ARD REVi EW

injury can then undergo additional ocused evalua- the injury. Observation is particularly dangerous in
tion based on the C scan results. T is patient has a this situation since even a brie period o hypoxia is
le pneumothorax and thus a le tube thoracostomy not tolerated well in a patient with traumatic brain
should be placed prior to obtaining the C scan. injury. 100% oxygen therapy is usually reserved or
Immediate operative exploration is not indicated non-traumatic pneumothoraces.
in the stable patient with no other obvious signs o
injury to a mediastinal organ or structure. 5. C. T e patient is an elderly male who sustained chest
trauma resulting in multiple rib ractures and pulmo-
2. C. T is patient presents with mild symptoms a er nary contusion. T is patient is at a high risk or pul-
motor vehicle collision. Cardiac contusion should monary complications, pneumonia, and death i not
be suspected but unlikely given clinically presen- managed appropriately and aggressively. T e patient
tation and normal electrocardiogram. Serial car- should be admitted to the intensive care unit or close
diac panel could yield mildly elevated enzymes but monitoring and pain management. Elderly patients
not clinically relevant data. Echocardiogram would o en have limited physiologic reserve and unrecog-
not be indicated either given normal electrocardio- nized decompensation on a medical-surgical ward
gram. Cardiology consult is not necessary since the could have tragic implications. Pain control with
patients symptoms are not related to myocardial either a thoracic epidural in usion or intravenous/
ischemia. A repeat C scan o the chest will likely oral narcotics supplemented with a non-steroidal
show no changes. T is patient can be sa ely dis- anti-in ammatory agent such as ketorolac should be
charged home. initiated, and titrated to allow ull inspiratory e ort
without pain. Intubation should not be done until the
3. A. T is patient has a retained hemothorax. T ere is patient has ailed conservative management (includ-
a signi cant risk or this patient to develop a compli- ing in selected cases a trial o non-invasive ventilator
cated uid collection, brothorax, or empyema i the support). Operative xation o rib ractures is usually
hemothorax is not evacuated in a timely manner. T e reserved or patients who are dif cult to wean rom
instillation o thrombolytic agents into the chest cav- mechanical ventilation, and would not be indicated
ity or C scan guided aspiration o hemothorax may this early a er the initial injury.
not result in complete evacuation o the hemotoma
since much o the hemotoma is probably solidi ed BIBLIOGRAPHY
clot at this point. Patients who received early VA S Aydemir NA, Bakir I, Atin F, et al. A magic bullet through the
(within 5 days) had lower incidence o developing heart. Circulation. 2007;115:e4678.
empyema in several studies. T oracotomy would not Dubose J, Inaba K, Demetriades D, et al. Management o post-
be warranted at this time since the hemothorax can traumatic retained hemothorax: A prospective, observa-
tional, multicenter AAS study. J rauma Acute Care Surg.
be evacuated less invasively with VA S. C guided Jan 2012;72(1):1122.
drainage is an option but a signi cant portion o the Embrey R. Cardiac trauma. T orac Surg Clin. Feb 2007;
hematoma has clotted; there ore, this will not likely 17(1):8793, vii.
be an e ective therapy. Harrington D , Phillips B, Machan J, et al. Factors associ-
ated with survival ollowing blunt chest trauma in older
4. C. T e patient has sustained signi cant chest patients: results rom a large regional trauma cooperative.
trauma but tracheobronchial injury must be ruled Arch Surg. May 2010;145(5):4327.
Kiser AC, OBrien SM, Detterbeck FC. Blunt tracheobron-
out given persistent pneumothorax. T is is particu- chial injuries: treatment and outcomes. Ann T orac Surg.
larly true in the presence o a large and persistent 2001;71:205965.
air leak and inability to re-expand the lung despite Martin RS, Meredith JW. Management o acute trauma. In:
adequate mechanical ventilation and chest tube ownsend CM, Beauchamp RD, Evers BM, Mattox K, eds.
drainage. An urgent bronchoscopy will be needed Sabiston extbook of Surgery: T e Biological Basis of Mod-
ern Surgical Practice. 19th ed. Philadelphia, PA: Elsevier
to evaluate or a tracheobronchial injury that may
Saunders;2012:449.
require intervention or surgical repair. It would not Okoye O , alving P, eixeira PG, et al. ransmediastinal
be wise to per orm a blind thoracotomy until bron- gunshot wounds in a mature trauma centre: Changing per-
choscopy is per ormed rst to identi y and localize spectives. Injury 2013; 44(9):11981203.
77
Sepsis and Multi-Organ Failure

Robert Shawhan, Matthew Eckert, and Matthew J. Martin

A 68-year-old emale is brought into the emergency C. Goals o early goal directed therapy in sepsis are
department by her son or evaluation o altered mental to achieve a central venous pressure (CVP) 8 to
status. T e son reports that his mother rarely goes to the 12 mm Hg; urine output 0.5 mL/kg/hr; and
doctor, is an alcoholic, and has been a smoker as long ScvO2 o 70% or greater.
as he can remember. She has occasionally had intermit- D. In intubated patients, a higher CVP (12 to
tent bouts o diverticulitis that have been treated with 15 mm Hg) should be considered pathologic.
oral antibiotics. Her most recent bout was 2 weeks ago E. Early initiation o targeted goal directed ther-
or which she recently nished a course o antibiot- apy in sepsis has not been de nitively shown to
ics. For the past two days she has had requent stools improve survival.
and increasing abdominal pain over the past 48 hours.
oday she was noted to be somnolent and di cult to 2. Despite receiving 3 liters o crystalloid, the patient
arouse by her son. remains hypotensive with a systolic blood pressure
On exam, her vitals show a temperature o 101.8F, o 75 and a ScvO2 o 60. Her hematocrit is 41%.
with a heart rate o 121. Her blood pressure is 83/54 and Which o the ollowing is the appropriate next
respiratory rate o 24, O2: 83% on RA. She is lethargic, step?
lungs are coarse bilaterally; her abdomen is distended, A. Switch to using a hydroxyethyl starch solutions
and she grimaces with palpation o her lower abdomen. as they are considered equivalent to isotonic
Her skin is pale and cool, and she has at neck veins. crystalloid in the resuscitation o septic patients,
Labs are signi cant or a white blood cell count (WBC) and are use ul in patients in uid re ractory
o 24, creatinine o 3.2, and lactate o 5.8. An arterial shock.
blood gas is obtained that pH o 7.30, pCO2 o 80, and B. Start a high dose phenylephrine in usion and
pO2 o 67 with a base de cit o 10. Her central venous titrate to a MAP o 65 mm Hg.
oxygen saturation (ScvO2) is 55%. C. Initiate a Levophed (norepinephrine) in usion
and i the patient remains hypotensive consider
1. T e patient is intubated and trans er to the starting a low dose vasopressin in usion.
intensive care unit (ICU) is being arranged. Which D. Start an in usion o intravenous hydrocortisone.
o the ollowing is true regarding the initiation o E. Obtain an echocardiogram to determine i the
goal directed therapy? patient is in cardiogenic shock prior to initiating
A. Antibiotic therapy should be delayed until all a vasopressor.
cultures are obtained and a causative organism is
identi ed. 3. Initial cultures were sent on admission. Because
B. Normalization o blood lactate is a poor guide o a recent history o antibiotic use and diarrhea,
or resuscitation. a polymerase chain reaction (PCR) based
292 G EN ERAL S U RG ERY EXAM I N ATIO N AN D BO ARD REVI EW

clostridium dif cile toxin test was sent as well. o multiple organ systems. Regarding outcomes
Despite initiating broad spectrum antibiotics o multi-organ dys unction syndrome (MODS),
the patient remain in shock. welve hours a er which o the ollowing is true?
admission the labs calls and tells the nurse that the A. For septic patients that develop agitation and
test is positive or clostridium dif cile (C. dif cile) delirium, the use o benzodiazepines has been
toxin. Which o the ollowing statements is true shown to reduce length o ICU stay, making
regarding the management o this patient? them pre erred over other agents.
A. Repeat the test or C. dif cile because PCR based B. Acute kidney injury is an independent risk actor
tests have a high alse positive rate. or death in patients with multi-organ dys unc-
B. In addition to broad-spectrum antibiotics, anti- tion syndrome.
ungal agents should be routinely started on C. Because septic patients are hypermetabolic,
admission o septic patients. parental nutrition has been shown to be bene -
C. A er a patient is asymptomatic, low pro brain cial when initiated early.
natriuretic peptide can help the decision to dis- D. Stress ulcer prophylaxis with proton-pump
continue antibiotics. inhibitors is not associated with any adverse
D. Surgical intervention or this pathology would events or complications.
include a total abdominal colectomy and end E. Critical illness polyneuropathy does not result
ileostomy. in di culty weaning a patient rom a ventilator
E. Once a source o in ection is identi ed, source because it results in peripheral nerve dys unc-
control should be initiated a er antibiotics have tion but preservation o the diaphragm.
taken ef ect and the patient is no longer septic.
ANSWERS
4. Upon arrival to the ICU a chest X-ray is obtained
and is shows bilateral inf ltrates. T e ventilator is 1. C. T e main principles o sepsis therapy are to begin
set to assist control mode with a peep o 5, tidal both treatment and resuscitation immediately upon
volume o 8 cc/kg, and FiO2 o 60%. On these identi cation o sepsis or suspected sepsis. reat-
setting her most recent PaO2 is 55. You suspect ment should consist o early initiation o empiric
acute respiratory distress syndrome (ARDS). broad spectrum antibiotics and initial antibiotic
Which o the ollowing is true regarding ARDS? administration does not require identi cation o
A. ARDS requires the presence o a ocal and uni- the exact causative organism. When sepsis is sus-
lateral in ltrate or consolidation on chest X-ray. pected early initiation o resuscitation with the goal
B. In patients with ARDS, routine corticosteroid o reversing tissue induced hypoper usion has been
use has been shown to reduce the overall mortal- shown to improve patient survival in patients pre-
ity rate compared to placebo. senting with septic shock. T e goals o resuscitation
C. Use o high tidal volumes and low positive end- should be to achieve a CVP 812 mm Hg, MAP
expiratory pressure (PEEP) is the traditional 65 mm Hg, urine output 0.5 mL/kg/hr, and ScvO2
ventilator strategy used to ventilate patients with o 70% or SvO2 o 65% within 6 hours. In mechani-
ARDS. cally ventilated patients venous return can be
D. reatment options or re ractory ARDS that has impeded and as a result a higher target or CVP (12
ailed conventional ventilation include airway to 15 mm Hg) is recommended. Note that there is no
pressure release ventilation (APRV), high re- arbitrary systolic blood pressure as a primary goal o
quency ventilation, and extra-corporeal mem- resuscitation.
brane oxygenation (ECMO). Lactate is also a marker o metabolic acidosis and
E. Airway pressure release ventilation has been tissue hypoper usion, and is seen in a majority o
shown to improve mortality in patients with patients presenting in septic shock. When a patient
ARDS. presents with hypotension and an elevated lactate
(4) they have an increased mortality over elevated
5. In patients treated or septic shock, there are lactate or hypotension alone. I a patient presents
consequences o a prolonged ICU stay and can with an elevated lactate it can be monitored until it
include progressive dys unction and even ailure reaches normal levels and can be used along with
C H AP TER 7 7 S EP S I S AN D M U LTI - O RG AN FAILU RE 293

other markers, to guide resuscitation. Base de cit therapy then corticosteroid administration can be
is another measure o metabolic acidosis that may considered.
be used to gauge the severity o illness and to guide Based on the presentation, there is a high suspicion
resuscitation. It is o en used as a surrogate or lac- o septic shock and no clinical or physical exam signs
tate, but there are many other actors that can impact o cardiogenic shock. While an echocardiogram may
the base de cit (renal ailure, alcohol, bicarbonate be use ul in certain situations, the treatment or sep-
losses), making it a less speci c measure o tissue tic shock should not be delayed. Delaying treatment
hypoper usion and lactic acidosis. o septic shock and can result in increased patient
In addition to restoring tissue hypoper usion, a mortality.
source o the patients sepsis should be sought and
appropriate antibiotics should be administered. Each 3. D. T is patient has a C. dif cile in ection and likely
hour antibiotics administration is delayed results in a toxic megacolon secondary to the in ection. PCR
measurable increase in patient mortality. Ideally, cul- based C. dif cile toxin as well as most current test or
tures should be obtained prior to giving antibiotics C. dif cile are extremely sensitive and speci c or the
with the goal o administering antibiotics within one diagnosis o an active C. dif cile in ection and repeat
hour o presenting with sepsis. However i obtain- testing is not necessary. T e severity o C. dif cile
ing appropriate cultures would delay the administra- in ections can range rom mild to severe. In very
tion o antibiotics beyond one hour, then antibiotics severe cases, patients can present with shock and
should be given prior to obtaining cultures. sepsis known as ulminant C. dif cile colitis or toxic
megacolon. Over the past two decades this diagno-
2. C. Many studies have been conducted comparing sis has become more common, and more and more
synthetic starches to isotonic crystalloid based solu- patients are requiring surgical intervention.
tions. T e results o these studied have varied. Some While we do not know rom the description i
studies have shown no mortality dif erence, others there is per oration or peritoneal signs, this patient
have shown increased mortality or increased rates is clearly in shock. Antibiotic therapy should be
o renal replacement therapy. None have shown a targeted at the suspected diagnosis and the patient
bene t, and recent data suggest increased renal ail- should be immediately evaluated or surgical inter-
ure and mortality with administration o hydroxy- vention. Surgical intervention or toxic megacolon
ethyl starch solutions. T ere is also a concern or the rom any cause typically mandates resection o the
potential impact o these solutions on platelet unc- entire colon (total abdominal colectomy) and place-
tion and coagulation. As a result, starch-based solu- ment o an end ileostomy. Anti- ungal agents should
tions are not recommended or resuscitation. not be routinely started in septic patients. T ey
Vasopressor therapy is recommended or use should be used only in patients that are at risk o or
in hypotensive septic patients a er adequate vol- suspected o having a ungal in ection. Patients that
ume resuscitation to maintain per usion pressures. are immunocompromised, neutropenic, or live in at
Norepinephrine (Levophed) is currently the initial risk areas should be considered or the implementa-
vasopressor o choice or septic shock, as it pro- tion o anti- ungal agents. However, there is nothing
vides a balanced pressor and cardiac inotrope ef ect. in this patients history that would lead us to suspect
Dopamine is associated with increased short-term she is at an increased or a ungal in ection.
mortality and serious adverse events compared to Once a source o in ection is identi ed then anti-
norepinephrine. Pure vasopressors such as phenyle- microbial therapy should be targeted at the suspected
phrine should be avoided as the unopposed vasocon- source. I there a speci c anatomical source o in ec-
striction can o en urther worsen the ongoing tissue tion such as an abscess, cholangitis, and so on, that
hypoper usion and lead to severe extremity or bowel is driving a patients sepsis, then emergent-targeted
ischemia. I the patient remains hypotensive a er therapy should be initiated within 12 hours; not a er
starting norepinephrine then starting a low dose antibiotics have taken ef ect. However, i no source o
in usion o vasopressin at 0.03 U/min can be used in ection is identi ed and the patient improved with
to decrease the dose o norepinephrine and enhance empiric anti-biotic therapy, then procalcitonin levels
organ per usion. I the patient remains in septic (not pro BNP levels) can be used to help a clinician
shock despite uids and appropriate vasopressor determine when to stop anti-microbial therapy.
294 G EN ERAL S U RG ERY EXAM I N ATI O N AN D BO ARD REVIEW

4. D. In 2012, an updated clinical de nition o ARDS whenever possible. Rather the primary treatment
(known as the Berlin De nition) was published. T e or agitation and delirium is non-pharmacologic
presence o bilateral chest opacities is needed to make and includes reorientation and maintenance o
the diagnosis o ARDS, and either a chest radiograph sleep-wake cycles. I this is in-ef ective then anti-
or a C scan is an acceptable way to evaluate or this. psychotics such as haloperidol or the newer atypical
A ocal unilateral in ltrate or consolidation on chest antipsychotics are pre erred to benzodiazepines.
X-ray would be more consistent with lobar pneumo- Another area that can challenge the manage-
nia than ARDS. According to the most recent ARDS ment o septic patients is predicting which patients
de nition outlined by the ARDS task orce the diag- are likely to have a worse outcome. T ere are many
nosis o ARDS must be made within one week o a scoring systems that are utilized to predict out-
known clinical insult, and typically the diagnosis is come in these patients, including the well-described
made within 72 hours. A er the diagnosis is made, APACHE score. One such system that is speci c or
the use o low tidal volumes (4 to 6 mL/kg) should be multi-organ ailure is the Sequential Organ Failure
initiated as this is the only mode o ventilation shown Assessment or SOFA score. T is system indepen-
to improve mortality in randomized controlled trials dently grades the severity o organ dys unction or
o patients with ARDS. As part o this strategy, high 6 dif erent organ systems: respiratory, coagulation,
PEEP is o en utilized to increase alveolar recruit- liver, cardiovascular, central nervous system, and
ment and improve oxygenation, but has not been renal. T e individual scores are then added to give
shown to improve overall mortality. a total score.
Many other strategies are utilized to try and Higher overall SOFA scores directly correlate with
improve outcomes in patients with re ractory ARDS. a worse outcome (Figure 77-1). Mortality has been
One treatment that has been studied is the use o cor- shown to directly increase with the number o organ
ticosteroids. T e ARDSNet trial showed that while systems that are ailing, and approaches 100% when
corticosteroids reduced then mean number o days 4 or more organ systems have ailed. Among individ-
on a ventilator and increased the number o shock- ual organ systems, one o the strongest predictors o
ree days, they had no ef ect on overall survival and ICU morbidity and risk o mortality is the develop-
were harm ul when given more than two weeks a er ment o acute kidney injury and/or renal ailure.
the diagnosis o ARDS was made. Meduri et al. used Many critically ill patients are at increased risk
low dose corticosteroids, and while ICU mortality o upper gastrointestinal bleeding, and stress ulcer
was signi cantly reduced overall mortality was not. prophylaxis is indicated or patients with prolonged
For the patient in ARDS who is re ractory to stand- mechanical ventilation (>48 hours) or other risk ac-
ard mechanical ventilation, salvage or rescue options tors. T e risk o bleeding rom stress ulceration or
include switching to APRV, high requency or jet gastritis in this patient population must be weighed
ventilation, and even ECMO. Other rescue therapies against the risk o treatment with either proton pump
include prone positioning, neuromuscular blockade, inhibitors (PPI) or histamine receptor blockers (H 2
and inhaled nitric oxide. While these treatments are blockers). Multiple studies have shown that patients
used as rescue therapies in severe ARDS and have that receive stress ulcer prophylaxis are at increased
been shown to produce improved oxygenation, they risk o adverse events and complications such as
have not been shown to improve mortality in con- C. dif cile in ections, ventilator associated pneumo-
trolled trials. nia, and thrombocytopenia (H 2 blockers).
Although hypermetabolism and a prolonged cat-
5. B. T ere are many challenges that are encountered abolic state are characteristic o sepsis and MODS,
when managing a critically ill patient, and delirium there has been no demonstrated survival bene t
is a commonly encountered complication in the ICU o administration o parenteral nutrition. Multi-
setting. Severely ill and older patients are at particu- ple series have demonstrated that overall in ectious
larly high risk or developing delirium, and this has complications are lower among patients given enteral
been shown to adversely impact morbidity and mor- nutrition versus total parenteral nutrition, and the
tality. While benzodiazapenes are ef ective or delir- gut should be the pre erred route or nutritional sup-
ium acutely, their use has been shown to increase ICU port whenever possible. Another important problem
length o stay and as a result they should be avoided that is seen in patients with septic shock is critical
C H AP TER 7 7 S EP S I S AN D M U LTI - O RG AN FAILU RE 295

Figure 77-1 Graph demonstrating the association o higher sequential organ ail-
ure assessment (SOFA) scores with mortality at all time points during ICU admis-
sion. (Reprinted with permission rom Fueglistaler P, Amsler F, Schuepp M, et al.
Prognostic value o sequential organ ailure assessment and simpli ed acute physi-
ology II score compared with trauma scores in the outcomes o multiple-trauma
patients. Am J Surg. 2010;200:204214.)

illness polyneuropathy. T e highly morbid condi- De Backer D, Aldecoa C, Njimi H, Vincent JL. Dopamine
tion is more common in patients with prolonged versus norepinephrine in the treatment o septic shock: a
meta-analysis. Crit Care Med. 2012;40(3):72530.
ICU stays and among patients who received steroids De Backer D, Biston P, Devriendt J, et al. Comparison o dopa-
and/or neuromuscular blocking agents. It can pre- mine and norepinephrine in the treatment o shock. N Engl
sent with muscle atrophy, limb weakness, peripheral J Med. 2010;362(9):779789.
sensory de cits and di culty weaning patients rom Delaloye J, Calandra . Invasive candidiasis as a cause o sep-
mechanical ventilation. However, cranial nerve unc- sis in the critically ill patient. Virulence. 2014;5(1):1619.
Dellinger RP, Levy MM, Rhodes A, et al. Surviving Sepsis
tion is usually spared.
Campaign: international guidelines or management o
severe sepsis and septic shock, 2012. Intensive Care Med.
BIBLIOGRAPHY 2013;39(2):165228.
T e Acute Respiratory Distress Syndrome Network. Venti- Ferreira FL, Bota DP, Bross A, Mlot C, Vincent JL. Serial eval-
lation with lower tidal volumes as compared with tradi- uation o the SOFA score to predict outcome in critically ill
tional tidal volumes or acute lung injury and the acute patients. JAMA. 2001;286(14):17548.
respiratory distress syndrome. N Engl J Med. 2000;342(18): Friedrich O. Critical illness myopathy: Sepsis-mediated ail-
13018. ure o the peripheral nervous system. Eur J Anaesthesiol
Adhikari NKJ, Dellinger RP, Lundin S, et al. Inhaled nitric Suppl. 2008;42:7382.
oxide does not reduce mortality in patients with acute Girard D, Bernard GR. Mechanical ventilation in ARDS:
respiratory distress syndrome regardless o severity: Sys- A state-o -the-art review. Chest. 2007;131(3):9219.
tematic review and meta-analysis. Crit Care Med. 2014; doi:10.1378/chest.06-1515
42(2):40412. Guidet B, Martinet O, Boulain , et al. Assessment o hemody-
Brower RG, Lanken PN, MacIntyre N, et al. Higher versus namic e cacy and sa ety o 6% hydroxyethylstarch 130/0.4
lower positive end-expiratory pressures in patients with versus. 0.9% NaCl uid replacement in patients with severe
the acute respiratory distress syndrome. N Engl J Med. 2004; sepsis: T e CRYS MAS study. Crit Care. 2012;16(3):R94.
351(4):32736. Heyland DK, Johnson AP, Reynolds SC, Muscedere J. Procal-
Cohen SH, Gerding DN, Johnson S, et al. Clinical practice citonin or reduced antibiotic exposure in the critical care
guidelines or Clostridium di cile in ection in adults: setting: A systematic review and an economic evaluation.
2010 update by the society or healthcare epidemiology Crit Care Med. 2011;39(7):17929.
o America (SHEA) and the in ectious diseases society Marik PE, Vasu , Hirani A, Pachinburavan M. Stress ulcer
o America (IDSA). In ect Control Hosp Epidemiol. 2010; prophylaxis in the new millennium: A systematic review
31(5):43155. and meta-analysis. Crit Care Med. 2010;38(11):22228.
296 G EN ERAL S U RG ERY EXAM IN ATIO N AN D BO ARD REVI EW

Maung AA, Kaplan LJ. Airway pressure release ventilation in Rivers E, Nguyen B, Havstad S, et al. Early goal-directed
acute respiratory distress syndrome. Crit Care Clin. 2011; therapy in the treatment o severe sepsis and septic shock.
27(3):5019. doi:10.1016/j.ccc.2011.05.003 N Engl J Med. 2001;345(19):136877.
Meduri GU, Golden E, Freire AX, et al. Methylprednisolone Russell JA. Bench-to-bedside review: Vasopressin in the man-
in usion in early severe ARDS results o a randomized con- agement o septic shock. Crit Care. 2011;15(4):226.
trolled trial. 2007. Chest. 2009;136(5 Suppl):e30. Sprung CL, Annane D, Keh D, et al. Hydrocortisone therapy or
McConnell KW, Coopersmith CM. Organ ailure avoidance patients with septic shock. N Engl J Med. 2008;358(2):11124.
and mitigation strategies in surgery. Surg Clin North Am. Stanley JD, Bartlett JG, Dart BW, Ashcra JH. Clostridium di -
2012;92(2):30719, ix. cile in ection. Curr Probl Surg. 2013;50(7):30237.
Myburgh JA, Fin er S, Bellomo R, et al. Hydroxyethyl starch or Steinberg KP, Hudson LD, Goodman RB, et al. E cacy and
saline or uid resuscitation in intensive care. N Engl J Med. sa ety o corticosteroids or persistent acute respiratory
2012;367(20):190111. distress syndrome. N Engl J Med. 2006;354(16):167184.
Perner A, Haase N, Guttormsen AB, et al. Hydroxyethyl starch Vincent JL, de Mendona A, Cantraine F, et al. Use o the
130/0.42 versus Ringers acetate in severe sepsis. N Engl J SOFA score to assess the incidence o organ dys unction/
Med. 2012;367(2):12434. ailure in intensive care units: Results o a multicenter, pro-
Ranieri VM, Ruben eld GD, T ompson B , et al. Acute respi- spective study. Working group on sepsis-related problems
ratory distress syndrome: the Berlin De nition. JAMA. o the European Society o Intensive Care Medicine. Crit
2012;307(23):252633. Care Med. 1998;26(11):1793800.
78
Major Burns and Smoke Inhalation

ovy Haber Kamine, Stephen R. Odom, and Booker . King

A 25-year-old man arrives in the emergency depart- or 2 hours. Select the next most appropriate step
ment a er being pulled rom a house re. On initial in management.
evaluation in the emergency room, he is noted to be A. Continue lactated ringers (LR) at 800 cc/hr or
agitated, with singed hair around his mouth and cough- 8 hours total, then decrease to 400 cc/hr or the
ing up black sputum. T e right hal o his torso and ollowing 16 hours.
back are covered with thick leathery skin, as are his B. Increase LR to 1000 cc/hr then monitor urine
right arm, thigh, and leg. His right arm is burned cir- output over the next hour.
cum erentially. T e le hal o his torso and back are C. Bolus LR and titrate to a urine output o 30 to
erythematous, with no blisters. His vital signs are show- 50 cc/hr.
ing a temperature o 95.3F with a heart rate o 120/ D. Bolus LR and titrate to a urine output o 100 to
min. Blood pressure is 110/50 mm Hg with a respiratory 200 cc/hr.
rate o 20, and Sat 89% on 6 L/min by nasal cannula. T e
patient is intubated and placed on mechanical ventila- 3. Af er 4 hours in ICU, the patients right hand
tion. A secondary survey is per ormed, which reveals no is noted to be cold. You are unable to palpate
additional injuries. T e patients weight is 70 kg. a radial or ulnar pulse. His vitals at this point
are: emperature: 97.5; HR: 140/min; BP: 90/40
1. T e patient is placed on 100% FiO2. An arterial (mean arterial pressure (MAP): 57) mm Hg; Sat:
blood gas is per ormed with the ollowing results: 100% on assist control/volume control (AC/VC)
pH: 7.24; pCO2: 32 mm Hg; pO2: 460 mm Hg; mechanical ventilation: FiO2: 60%; RR: 20/min;
HCO3: 16 mEq/L; Base Excess: 12 mEq/L; Lactate: tidal volume ( V): 400 cc; positive end-expiratory
8 mmol/L. Which o the ollowing is the most pressure (PEEP): 5 mm Hg. Which is the next most
appropriate next step in management? appropriate step in management?
A. Intravenous methylene blue A. Bedside escharotomy o right arm.
B. Hyperbaric oxygen B. Immediate trans er to the operating room or
C. Activated charcoal asciotomy.
D. Hydroxocobalamin C. Initiation o vasopressor therapy to target a MAP
E. Decrease FiO2 o 65 mm Hg.
D. Excision o right arm burns with split thickness
2. Following the appropriate intervention in skin gra rom le thigh.
Question 1, the patient is started on 800 cc/hr o
lactated ringers and trans erred to the intensive 4. Af er ICU admission, topical ma enide acetate
care unit (ICU). A oley catheter is placed and af er (Sul amylon) is placed on the burn wounds. On
4 hours he is noted to have 20 cc o urine output hospital day 3, labs are per ormed and enteral
298 G EN ERAL S U RG ERY EXAM IN ATI O N AN D BO ARD REVIEW

eeding is started. Labs reveal a normal white obtundation, tachyarrthymia, and cardiac arrest.
blood cell (WBC) count at 10 K/L, normal glucose Because o the di culty with diagnosis, a high index
at 110 mg/dL, and a chemistry panel remarkable o suspicion is necessary and treatment should be
or a HCO3 o 12 mEq/L. An arterial blood gas started immediately. Hydroxocobalamin should be
demonstrates pH: 7.22; pCO2: 24 mm Hg; pO2: administered to any patient who has su ered smoke
200 mm Hg; HCO3:12 mEq/L; Base Excess: 16 inhalation and demonstrates signs o cardiovascular
mEq/L, and Lactate: 2 mmol/L instability.
Carbon monoxide poisoning is the leading cause
Which o the ollowing is the next most appropriate o death due to unintentional poisoning in the United
step in management? States. Carbon monoxide binds to hemoglobin to
A. Decrease tube eeding rate. make carboxyhemoglobin with an a nity 200 times
B. Initiate systemic antibiotics. higher than oxygen. T e most common symptom o
C. Discontinue any topical antimicrobial treatment. carbon monoxide poisoning is headache, however,
D. Change topical antimicrobial to silver nitrate. similar to cyanide poisoning, the symptoms may
E. No change in management is indicated. be nonspeci c, but severe poisoning leads to coma,
arrhythmias, and death. reatment involves 100%
5. On hospital day 4, the patient is doing well and the oxygen at normal atmospheric pressure with a tight-
decision is made to per orm excision and graf ing tting mask (normobaric oxygen) or via ventilator
o his burns. Which o the ollowing statements in the intubated patient. Hyperbaric oxygen (HBO)
about excision and graf ing is true? may be indicated or severe carbon monoxide
A. Excision and gra ing o deep burns should be poisoning.
delayed at least 2 to 3 weeks to ensure that the
2. B. T e rule o nines is used to calculate the percent-
burns have ully demarcated.
age body sur ace area (BSA) burned in adults. Only
B. T e ideal gra or burn excision wounds is a
second and third degree burns are included in the
widely meshed ull thickness skin gra .
calculation o total BSA burned; rst degree burns
C. Split-thickness skin gra sites may be re-used or
are not.
additional gra harvesting on a weekly basis.
T e most commonly used ormula or calculating
D. Negative pressure wound therapy (NPW ) has
crystalloid resuscitation needs in the rst 24 hours is
no place in the management o burn wounds.
the Parkland ormula:
E. Human cadaver allogra may be gra ed to a
burn wound to protect it until an appropriate First 24 resuscitation = 4 cc tBSA% burn
autogra can be placed i there is not enough body weight (kg).
donor skin is available.
Hal o this f uid is given in the rst 8 hours a er
injury, and hal in the next 16 hours.
ANSWERS It is important to note that the Parkland or-
mula is only a guideline and intravenous f uid
1. D. Smoke inhalation injury is primarily a chemical needs should be titrated to urine output o 30 to
injury to the tracheobronchial tree. Most thermal 50 cc/hr in most adults. I this urine output range
energy is dissipated in the upper airway. Inhala- is not achieved, the IV f uid rate should be adjusted
tion injury increases mortality in burns by 20% and /+ 25% each hour until the desired urine output
increases pneumonia risk by 40%. Cyanide poisoning is reached. Fluid boluses should be avoided unless
in particular is requently atal and there is no easy the patient is hypotensive as it could lead to over-
method o diagnosis. Cyanide binds to cytochrome resuscitation.
oxidase and stops cellular respiration. T e diagnosis
is suggested in any enclosed space re where patients 3. A. Full thickness or deep partial thickness burns
have evidence o smoke inhalation and su er rom that are circum erential around an extremity or chest
altered mental status, and have a lactic acidosis; it will prevent normal elastic movement o the skin.
may present with variable signs or symptoms rom Edema a er f uid resuscitation can cause tissue pres-
mild con usion and hyperventilation to complete sure to rise underneath this leathery skin and impair
C H AP TER 7 8 MA j O R BU RN S AN D S M O k E I N H ALATIO N 299

circulation. T e treatment is escharotomy, which substitutes may be used. Cadaver allogra is a good
involves a ull thickness incision through the burned option or temporary coverage.
skin (epidermis and dermis down to subcutaneous
at). T e incision is made along the medial or lat- BIBLIOGRAPHY
eral aspect o the limb, or, in the chest, in the mid-
Albright JM, Davis CS, Bird MD, et al. T e acute pulmonary
axillary line, to decompress underlying structures. inf ammatory response to the graded severity o smoke
T e incisions should only be through the eschar, as inhalation injury. Crit Care Med. 2012;40(4):111321.
they should not involve the deeper ascia. Although Asmussen S, Maybauer DM, Fraser JF, et al. Extracorporeal
escharotomy is o en con used with asciotomy, the membrane oxygenation in burn and smoke inhalation
majority o circum erential extremity burns resulting injury. Burns. 2013;39(3):42935.
Barajas-Nava LA, Lpez-Alcalde J, Roqu i Figuls M, Sol I,
in signs o elevated compartment pressures require Bon ll Cosp X. Antibiotic prophylaxis or preventing
only an escharotomy, and not a asciotomy. burn wound in ection. Cochrane Database Syst Rev. June
6, 2013;6:CD008738.
4. D. T is patient has a severe metabolic acidosis that Blumetti J, Hunt JL, Arnoldo BD, et al. T e Parkland or-
mula under re: Is the criticism justi ed? J Burn Care Res.
is likely due to treatment with Ma enide acetate 2008;29(1):1806.
(Sul amylon). Ma enide acetate is a weak carbonic Cartotto RC1, Innes M, Musgrave MA, et al. How well does
anhydrase inhibitor and can produce a non-gap met- the Parkland ormula estimate actual f uid resuscitation
abolic acidosis. Silver sul adiazine has broad spec- volumes? J Burn Care Rehabil. 2002;23(4):25865.
trum activity and some activity against pseudomonas, Cortazo JA, Lichtman AD. Methemoglobinemia: A Review
and recommendations or management. J Cardiothorac
but has poor eschar penetration. Ma enide acetate has
Vasc Anesth. August 13, 2013;ii:S10530770(13)000438.
good eschar penetration. Silver nitrate has broad spec- Gibran N. Burns. In: Mulholland MW, Lillemoe KD, Doherty
trum activity and good eschar penetration, but stains GM, et al., eds. Greenf elds Surgery Scientif c Principles and
skin and sheets black, and can cause hyponatremia by Practice. 5th ed. Philadelphia, PA: Lippincott Williams &
leaching sodium rom tissues. Silver nitrate however Wilkins; 2011:21436.
is the best alternative topical agent in this situation. Guzman JA. Carbon Monoxide Poisoning. Crit Care Clin.
2012;28(4):53748.
Systemic antibiotics should not be given prophylacti- Hettiaratchy S, Papini R. Initial management o a major burn:
cally, only or treatment o proven in ections. II Assessment and resuscitation. BMJ. 2004;329(7457):
1013.
5. E. One o the greatest achievements in burn care was Leversedge F, Moore J, Peterson BC, Seiler JG. Compart-
ment syndrome o the upper extremity. J Hand Surg.
the adoption o early excision and gra ing o deep 2011;36(3):54459.
burns. Early excision and gra ing o burn wounds Lin JJ, Chung XJ, Yan CY, Lau HL. A meta-analysis o trials
is associated with decreased mortality and decreased using the intention to treat principle or glutamine sup-
hospital length o stay. plementation in critically ill patients with burn. Burns.
Ideally, all devitalized tissue should be removed 2013;39(4):56570.
Milner SM, Asuku ME. Burn Wound Management. In: Cam-
in the rst week. Excision can be per ormed either
eron JL, Cameron AM, eds. Current Surgical T erapy. 11th
by tangential excision (layers o burned tissue are ed. Philadelphia, PA: Elsevier Saunders; 2014:112831.
excised until a bleeding wound bed is reached) or by OBrien DJ, Walsh DW, erri CM, Hall AH. Empiric man-
ascial excision (excision to the layer o the ascia). agement o cyanide toxicity associated with smoke inhala-
Fascial excision results in less bleeding but can cause tion. Prehosp Disaster Med. 2011;26(5):37482.
signi cant cosmetic de ormities. Ong YS, Samuel M, Song C. Meta-analysis o early excision o
burns. Burns. 2006;32(2):145150.
Skin coverage o excised areas is best done with Orgill DP. Exicision and Skin Gra ing o T ermal Burns.
meshed split thickness skin gra s but should be N Eng J Med. 2009;360(9):893901.
avoided in cosmetically sensitive areas (e.g., the ace). Perel P, Roberts I. Colloids versus crystalloids or f uid resus-
Meshing allows a larger sur ace area to be covered citation in critically ill patients. Cochrane Database Syst
as well as allowing space or blood and f uid to leak Rev. 2007;(4):CD000567.
Piccolo NS, Piccolo MS, Piccolo PD, et al. Escharotomies,
through the gra lessening the risk o hematoma or
asciotomies and carpal tunnel release in burn patients
seroma ormation under the gra . Donor sites may review o the literature and presentation o an algorithm
be recropped a er they have healed, usually around or surgical decision making. Handchir Mikrochir Plast
2 to 3 weeks. I there is a shortage o donor skin, skin Chir. 2007 Jun;39(3):1617.
300 G EN ERAL S U RG ERY EXAM I N ATIO N AN D BO ARD REVI EW

Rousseau AF, Losser MR, Ichai C, Berger MM. ESPEN Shadgan B, Menon M, Sanders D, et al. Current thinking about
endorsed recommendations: Nutritional therapy in major acute compartment syndrome o the lower extremity. Can
burns. Clin Nutr. 2013;32(4):497502. J Surg. 2010;53(5):32934.
Royall D, Fairholm L, Peters WJ, Jeejeebhoy KN, Allard JP. Spodaryk M, Kobylarz K. T e Usability o Harris-Benedict
Continuous measurement o energy expenditure in ven- and Curreri Equations in Nutritional Management o T er-
tilated burn patients: an analysis. Crit Care Med. 1994; mal Injuries. Ann Burns Fire Disasters. 2005;18(3):117
22(3):399406. 121.
79
Surgical Nutrition

Julia B. Greer

A 49-year-old woman who has medically re ractory C. 30% IV at emulsion


Crohns disease and high ileostomy output (2200 mL/ D. Omega-3 atty acids
day) is re erred to your clinic. She has undergone sev- E. Insulin
eral abdominal surgeries or complications o her
Crohns and she currently has a diverting ileostomy and 3. Af er 8 weeks at home receiving parenteral
approximately 120 cm o her small bowel. O note, she nutrition, your patient develops hair loss, a
has been unable to tolerate any in ammatory bowel dis- pustular rash around her mouth, and darkening o
ease medications aside rom corticosteroids. her skin creases. T e most likely cause is:
T e patient was admitted to an outside hospital twice A. Copper de ciency
in the past 3 months or rehydration and repletion o B. Hyperkalemia
sodium, potassium, and magnesium. She now comes to C. Hyperglycemia
your institution complaining o increased ostomy out- D. Magnesium de ciency
put, lightheadedness, atigue, and nausea. She reports a E. Zinc de ciency
recent weight loss o approximately 20 pounds (approx-
imately 15% o total body weight). A Hickman catheter 4. All individuals who receive total parenteral
is placed in her right subclavian vein and she receives nutrition or >13 weeks will develop:
3 days o parenteral nutrition without complications. A. Venous thrombosis
Insurance coverage or teduglutide is pending and she B. Steatohepatitis
is discharged home on parenteral nutrition. You would C. Gallbladder sludge
like to per orm an ileostomy takedown but would like D. Cholelithiasis
to improve her nutritional status rst. E. Re eeding syndrome

1. Compared to enteral nutrition, parenteral 5. I your patient on parenteral nutrition suddenly


nutrition (PN): spikes a ever, the most important entity to rule
A Is less expensive out is:
B. Does not su er rom product shortages A. A Crohns disease are
C. Preserves immunologic unction o gut B. A catheter line in ection
D. Is not associated with metabolic bone dys unction C. An in ection at the ostomy site
E. Is less likely to cause diarrhea D. Clostridium dif cile colitis
E. A urinary tract in ection
2. Basic parenteral nutrition ormulations include:
A. Sucrose
B. Amino acids
302 G EN ERAL S U RG ERY EXAM IN ATI O N AN D BO ARD REVIEW

ANSWERS spasms, restless leg syndrome, and general agitation;


additional signs and symptoms o low magnesium
1. E. Compared to enteral nutrition, PN is less likely to include nausea, vomiting, insomnia, and con usion.
cause diarrhea. Enteral nutrition is delivered directly A pathognomonic sign o zinc de ciency is hair
to the GI tract and its hyperosmolarity may result in loss. Skin lesions, including acne, a perioral pus-
diarrhea, especially in patients with an underlying tular and darkening o skin olds, are requently
condition that causes malabsorption. Diarrhea has observed. Loss o appetite, decreased motor skills,
been shown to occur in as many as 95% o patients and decreased immunity may characteristic o low
who receive enteral eeds. PN is considerably more dietary zinc. PN must be ormulated to address de -
expensive than enteral nutrition. Shortages o many ciencies at initiation as well as those that may occur
orms o product, especially vitamin and trace min- over the course o hyperalimentation. While elec-
eral components, has been causing delays in initiation trolyte levels are routinely monitored, one should be
o PN as well as inconsistent mixing and matching aware o the potential or vitamin and trace mineral
o di erent brands o product, which may result in de cits. Individual with high output stula or ostomy
certain micronutrient de ciencies i the provider can develop metabolic disturbances. T is patient
does not have expertise with PN. Because it com- may have had a zinc de ciency prior to receiving PN
pletely bypasses the GI tract, PN does not preserve which should have been addressed and monitored.
the immunologic unction o gut. PN has been asso-
ciated with metabolic bone dys unction and abnor- 4. C. Catheter-related venous thrombosis is a airly rare
mal bone metabolism and some patients have been complication o PN, occurring in 1% to 3% o indi-
shown to develop osteoporosis and osteomalacia. viduals per catheter-year. Deleterious e ects o PN
2. B. Standard components o PN ormulas include on the liver and gallbladder are well known to clini-
amino acids, dextrose, a 10% or 20% IV at emulsion cians. Hepatic steatosis, which may mani est as atty
to provide essential atty acids, electrolytes (sodium liver in ltration, may occur in PN patients within 1
phosphate, sodium chloride, sodium acetate, potas- to 2 weeks o initiating PN. It is reversible and can be
sium phosphate, potassium chloride, potassium managed by limiting the at content. Liver unction
acetate, magnesium sul ate, and calcium gluconate), tests (LF s) should be checked weekly or individu-
multi-component vitamins and multi-component als on PN and i they are elevated, lipids should be
trace minerals. Some potential additives include minimized to < 1 g/kd/day and total or peripheral PN
cysteine, regular insulin, and additional trace vita- should be cycled over 12 hours to rest the liver.
mins or elements as required. Although omega-3 I total bilirubin is > 510 mg/dL due to hepatic
atty acid-enriched PN ormulations have been stud- dys unction, trace elements should be discontinued
ied as a potential means o decreasing in ammation due to the potential or toxicity o manganese and
and increasing immune unction in certain subsets copper. Cholestasis is inevitable during PN because
o patients, standard PN ormulations do not contain there are no intestinal nutrients to stimulate hepatic
them at this time. bile ow. Cholestasis typically occurs 2 to 6 weeks a er
starting PN and is indicated by progressive increases
3. E. Because copper works with iron to orm red in total bilirubin and elevated serum alkaline phos-
blood cells, an early sign o copper de ciency is ane- phatase. While cholelithiasis is not uncommon dur-
mia. Low body temperature, osteoporosis, low white ing PN, it is certainly not ubiquitous. Re- eeding
blood cell count, irregular heartbeat, loss o skin pig- syndrome is a complication that begins rapidly a er
mentation, and thyroid problems may also occur due starting PN in a severely malnourished individual,
to a de ciency o copper. Hyperkalemia is associ- typically a person who has been in a starvation state
ated with a slow or irregular heartbeat and weakness. or > 710 days. T is syndrome is characterized by
Signs o hyperglycemia include weakness, nausea, a severe shi in uid and serum electrolyte levels,
excessive thirst/urination/appetite, headache, irrita- especially hypophosphatemia, resulting rom intra-
bility, and abdominal pain. Severe hyperglycemia can cellular electrolyte movement. Severe systemic com-
lead to unconsciousness. A magnesium de ciency plications, and even death, can result rom re- eeding
o en mani ests as cardiac and muscle irregularities, syndrome. Correcting electrolyte abnormalities
including arrhythmia, weakness, muscle cramps or prior to initiating PN is preventative or re- eeding
C H AP TER 7 9 S U RG IC AL N U TRITIO N 303

syndrome. Although the previous complications may Chang SJ, Huang HH. Diarrhea in enterally ed patients: Blame
occur or some individuals on PN, all PN patients the diet? Curr Opin Clin Nutr Metab Care. September
will develop gallbladder sludge a er receiving PN or 2013;16(5):58894.
Clare A, eubner A, Sha er JL, et al. What in ormation should
13 weeks. lead to a suspicion o catheter sepsis in HPN? Clin Nutr.
August 2008;27(4):5526.
5. B. Any o the entities listed in question 5 may cause Dibb M, eubner A, T eis V, et al. Review article: the manage-
your patient to become ebrile but catheter-related ment o long-term parenteral nutrition. Aliment Pharm &
bloodstream in ections (CR-BSI) are the most com- T er. March 2013;37(6):587603.
mon and most serious complication o PN. Adequate Dimick JB, Swoboda S, alamini MA, et al. Risk o coloniza-
nutrition is a cornerstone or strength preservation tion o central venous catheters: Catheters or total paren-
teral nutrition versus other catheters. Amer J Crit Care. July
and immune system unction in patients with serious 2003;12(4):32835.
gastrointestinal illnesses and proper training o am- Gastmeier P, Weist K, Ruden H. Catheter-associated primary
ily members and ancillary health personnel or home bloodstream in ections: Epidemiology and preventive
PN is essential. Sterile technique when manipulating methods. In ection. 1999;27(Suppl 1):S16.
the catheter is imperative. Not all patients with a CR- Guenter P, Holcombe B, Mirtallo JM, et al. Parenteral nutri-
tion utilization: Response to drug shortages. JPEN. January
BSI will present with pyrexia but a sudden increase in
2014;38(1):1112.
body temperature and an elevated C-reactive protein Hamilton C, Seidner DL. Metabolic bone disease and
provide a high index o suspicion. High white blood parenteral nutrition. Curr Gastroenterol Rep. August
cell count, low albumin, and/or elevated total biliru- 2004;6(4):33541.
bin may be present. Catheter maintenance or home Lee JW. Fluid and electrolyte disturbances in critically ill
parenteral nutrition patients and repeated removal/ patients. E & BP. December 2010;8(2):7281.
Maroulis J, Kal arentzos F. Complications o parenteral
reinsertions can result in loss o venous access. nutrition at the end o the century. Clin Nutr. October
Whenever possible, salvage o an in ected tunneled 2000;19(5):295304.
catheter, such as the Hickman catheter used in this McWhirter D. Parenteral nutrition line sepsis: T e dif culty
patient, should be attempted. T e most e ective way in diagnosis. Proc Nutr Soc. November 2010;69(4):508510.
to prove the existence o a CR-BSI is to simultane- Messing B. Gallbladder sludge and lithiasis: Complication o
bowel rest. Nutrition. MarchAprril 1990;6(2):1901.
ously draw blood cultures rom the central catheter
Messing B, Bories C, Kunstlinger F, Bernier JJ. Does total par-
and rom a peripheral source. Other sources o ever enteral nutrition induce gallbladder sludge ormation and
should be investigated. lithiasis? Gastroenterology. May 1983;84(5 Pt 1):101219.
Mirtallo JM. Consensus o parenteral nutrition sa ety issues
and recommendations. JPEN. March 2012;36(2 Suppl):62S.
BIBLIOGRAPHY Radrizzani D, Bertolini G, Facchini R, et al. Early enteral
Ayers P, Adams S, Boullata J, et al. A.S.P.E.N. Parenteral nutri- immunonutrition versus parenteral nutrition in critically
tion sa ety consensus recommendations: ranslation into ill patients without severe sepsis: A randomized clinical
practice. Nutr Clin Practice. June 2014;29(3):27782. trial. Intensive Care Med. August 2006;32(8):11918.
Buchman AL, Howard LJ, Guenter P, et al. Micronutrients Sabiston D. Jr, Lyerly HK. Metabolism in Surgical Patient. In:
in parenteral nutrition: oo little or too much? T e past, Sabiston D. Jr, Lyerly HK, eds. extbook o Surgery: T e Bio-
present, and recommendations or the uture. [Erratum logical Basis o Modern Surgical Practice. 15th ed., Philadel-
appears in Gastroenterology. April 2010;138(4):1633. Note: phia, PA: Elsevier Saunders; 1997:13775.
Dosage error in article text.] Gastroenterology. November Seres DS, Valcarcel M, Guillaume A. Advantages o enteral
2009;137(5 Suppl):S16. nutrition over parenteral nutrition. T er Adv Gastroenterol.
Chang A, Enns R, Saqui O, et al. Line sepsis in home par- March 2013;6(2):15767.
enteral nutrition patients: Are there socioeconomic risk Whelan K, Schneider SM. Mechanisms, prevention, and man-
actors? A Canadian study. JPEN. NovemberDecember agement o diarrhea in enteral nutrition. Cur Opin Gastro-
2005;29(6):40812. enterol. March 2011;27(2):1529.
80
Blood Products and rans usion

Stephen R. Odom

A 33-year-old emale (G3P2) 31 1/7 weeks pregnant, 3. Which o the ollowing non-trans usion strategies
with severe vaginal bleeding is taken emergently to the is o proven benef t in reducing blood product
operating room or cesarean section. Intraoperatively, a trans usion in stable trauma patients?
percreta is identi ed with involvement o the bladder A. Minimize blood loss, use o autologous blood
and pelvic wall. A er delivery o the baby, an emergent salvage, avoidance o coagulopathy and hypo-
hysterectomy is per ormed or massive bleeding. Blood thermia, and damage control techniques
loss was estimated at 15 liters. T e patient was given B. Pelvic binders
7000 mL o crystalloid, along with 40 units o packed C. Blood substitutes
red blood cells, 10 units o resh rozen plasma, and D. Factor V concentrate
20 units o platelets. E. ranexamic acid, i used more than 12 hours
a er the traumatic event
1. Which o the ollowing is an indication or blood
trans usion? 4. Which o the ollowing statements regarding
A. Serum hemoglobin < 10 g/dL heparin-induced thrombocytopenia (HIT) is true?
B. Central venous oxygen concentration < 55% A. HI is an IgA antibody mediated response to
C. rauma score > 14 heparin-platelet actor IV complexes.
D. Loss o 20% o total body blood volume B. Procine un ractionated heparin (UFH) is more
E. Ongoing bleeding with tachycardia, systolic likely to cause HI than bovine UFH.
blood pressure < 90 mm Hg, oliguria, acidosis, C. T e combined sensitivity and speci city o ELISA
and/or elevated base de cit and serotonin unctional assay is about 70%.
D. reatment o con rmed HI involves stopping
2. Which o the ollowing is true o trans usion-
heparin and initiation o war arin therapy.
related acute lung injury (TRALI)?
E. Patients with remote history o HI can be sa ely
A. Improvement when well-resuscitated, takes treated with heparin.
> 2 weeks.
B. It is caused by anti-HLA-antibodies in blood 5. Which o the ollowing statements regarding
products that activates recipient leukocytes. complications o blood trans usion is true?
C. Acute respiratory illness arises within 2 days o A. Risk o hepatitis C in ection per unit o packed
trans usion o blood products. red blood cells is 1 in one million.
D. Is easy to di erentiate rom acute respiratory dis- B. Hyperkalemia is a rare but requently atal disor-
tress syndrome (ARDS). der seen in massive blood trans usion.
E. It is not associated with the number o trans u- C. Citrate can lower serum calcium and magnesium.
sions or the age o blood.
C H AP TER 8 0 Bl o o d P Ro d u C Ts An d TRAn s f u s i o n 305

D. Massive trans usion requently causes acidosis. theories, including the roles o non-polar lipids and
E. ABO incompatible blood trans usion is a rare activated platelets are less well supported in the lit-
cause o hemolytic reaction. erature. Resolution in a well resuscitated patient nor-
mally occurs within 48 hours. Mortality is around
6% (as compared to much higher mortalities in
ANSWERS
ARDS). Risk actors or the development o RALI
1. E. Speci c triggers or blood trans usions have include increased number o trans usions, possibly
been the subject o a large amount o research, and age o blood trans used, emale plasma donor, and
the decision to trans use is not always clear. Data anti-HLA-antibody complement. Patient risk actors
indicate that in a patient without cardiac or lung include higher IL-8 level, shock, liver surgery, cirrho-
disease and hemodynamic stability, a hemoglobin sis, positive uid balance, elevated peak airway pres-
7 g/dL is satis actory to minimize blood trans u- sures, and current smoking.
sion without adversely a ecting mortality. What is
clear rom these studies is that a liberal trans usion 3. A. T ere are multiple non-trans usions options in
strategy is not help ul or oxygen delivery but has a surgical patients, many o which have proven bene -
negative e ect on the immune system. In addition, cial in decreasing number o trans usions required.
class o hemorrhage by itsel is not an indication or Initially, care ul surgical technique (minimize blood
trans usion unless systemic hypotension, re ractory loss, avoid hypothermia and coagulopathy, damage
tachycardia, oliguria, lactic acidosis are present with control, etc.) and care ul treatment o preoperative
evidence o ongoing bleeding. anemia are clearly associated with decreased need
Systemic venous oxygen saturation (SvO2) is an or trans usion. While pelvic binders are recom-
attractive measure o oxygen consumption, but can- mended by the American College o Surgeons there
not be associated with parameters o blood loss or are no clinically relevant data to support their use.
severity o injury and so, by itsel , cannot represent a Blood substitutes, including human products, bovine
reason to trans use a patient. products, and genetically engineered hemoglobin
T e trauma score is a physiologic score that repre- are available. O these products, the genetically engi-
sents the sum o scores or respiratory rate and e ort, neered hemoglobin is promising, in that the cross-
capillary re ll, systolic blood pressure, and Glasgow match is avoided (timely administration), osmotic
coma score. In this scoring system, a higher number is pressure is increased (resuscitative uid) and blood
a less severe injury. Recent data suggest that over 90% pressure is increased (vasopressor activity), but it
o patients with a trauma score > 14 did not need a remains to be determined that these products are
blood trans usion. Both the revised trauma score and bene cial in reducing blood trans usions in trau-
the ISS have been shown, interestingly, to predict the matically injured or critically ill patients.
need or trans usion is severe pelvic trauma. Recombinant actor VII concentrate, not actor V,
T e only reliable criteria or the need to trans- enhances thrombin generation and platelet activation
use include clinical criteria, like ongoing bleeding, and has been shown to decrease the need or massive
re ractory tachycardia, decreased systolic blood pres- trans usion o the severely injured trauma patients. A
sure, oliguria, lactic acidosis, and elevated base de cit. multicenter Phase III trial demonstrated a decrease in
A single criterion has been elusive. blood product usage, but did not demonstrate a mor-
tality di erence rom placebo. It seems reasonable to
2. B. RALI is an acute respiratory complication o avoid its use in elderly patients (> 75 years old) as they
blood trans usion that occurs within 4 to 6 hours have increased risk o arterial thrombosis. ranexamic
o trans usion o product and cannot be traced to acid is an anti brinolytic agent that blocks binding o
another acute lung injury risk actor. T e syndrome plasmin to brin. In the CRASH-2 trial, over 20,000
is very similar to ARDS o other etiologies, and o en major trauma patients were evaluated. Early use o
cannot be di erentiated. Overlapping syndromes tranexamic acid decreased death rate without dem-
include trans usion associated circulatory overload onstrated major side e ects. Use a er 3 hours is not
( ACO). RALI is an anti-HLA-antibody mediated bene cial. ranexamic acid has also been shown to
reaction rom recipient leukocytes. Once comple- decrease the size o intracranial bleeding lesions in
ment is activated, acute lung injury ensues. Other trauma patients (CRASH-2 collaborators).
306 G En ERAl s u RG ERY EXAM i n ATi o n An d Bo ARd REVi EW

4. E. HI is an IgG mediated disease directed at (1:250,000), Hepatitis C (1:150,000), and HIV


heparin-platelet actor IV (PF-4) complexes. Bovine (1:2 million). Cytomegalovirus (CMV) is present in
UFH increases risk o HI compared to porcine 50% o the population, and is transmitted in 5% in
derived UFH. Orthopedic surgery/injury, cardiac the in ected units. T e rate o transmission o prion
surgery, especially heart transplant, the use o UFH disease is not known, but is presumably very, very low.
as compared to low molecular weight heparin all Hyperkalemia a er massive trans usion can be seen
increase the risk o HI . Obstetric patients are at very in up to 38.5% in trauma patients but is rarely associ-
low risk o developing HI . T e ELISA test or HI ated with clinical sequelae. A er trans usion, new red
detects antibodies that react with the heparin-P-F-4 blood cell counts can take up potassium resulting in
epitope while unctional platelet assays (like the ser- post-trans usion hypokalemia, so electrolytes should
otonin assay) use radiolabeled platelets mixed with be closely monitored be ore, during, and a er trans-
patients serum and heparin. usion. Citrate (3 g/unit) is used in stored blood, and
T e supernatant is then evaluated or radiolabeled can bind calcium in the blood, resulting in hypocal-
material. Speci city and sensitivity o the ELISA cemia a er massive trans usion.
test is 50% to 70% and 90%, respectively. Speci c- Clinical signs o hypocalcemia include wid-
ity and sensitivity o unctional assays are 95% and ened Q interval, decreased ventricular contractil-
90%, respectively. Used in conjunction, the sensitivity ity, hypotension (related to a decrease in peripheral
and speci city or HI approaches 100%. T e results vascular resistance), muscle tremors and even PEA
must be put into clinical context because between arrest. Hypomagnesemia can also be responsible
20% to 60% o patients will orm the heparin-PF-4 or prolonged Q seen a er trans usion. T e pH o
complexes that can be seen on the EISA test with- stored blood o en decreases to 6.6 to 6.8 because
out the clinical syndrome. Once the diagnosis has o increasing CO2. However, the most requent
been made, all heparin must be stopped. Even a er acid-base problem seen a er massive trans usion is
stopping heparin, patients are at increased risk or alkalosis because o the large amount o citrate in
thrombotic complications or up to 100 days while stored blood. Acidosis seen a er massive trans usion
the complexes persist, and require treatment with should be concerning or ongoing tissue hypoper-
a direct thrombin inhibitor (only argatroban and usion rather than a result o the trans usion itsel .
lepirudin are FDA approved). Factor Xa inhibitors ABO incompatible trans usion and the resultant
(dancparoid) are options, but not available in the hemolytic reaction is the most common preventable
United States. potentially atal complication o blood trans usion.
Ancrod (pit viper venom), a glycoprotein IIb/IIIa T e most common cause is error along the chain o
inhibitor, has been evaluated in HI , and was ound identi cation o the unit, the patient, or both.
to be without ef cacy and may increase thrombotic
risk. Vitamin K antagonists are contraindicated in the BIBLIOGRAPHY
acute phase o HI , but have a role in the long-term Aboudara MC, Hurst FP, Abbott KC, Perkins RM. Hyperka-
management o thrombotic complications. A er lemia a er packed red blood cell trans usion in trauma
about 100 days ree rom heparin, the heparin-PF-4 patients. J rauma. February 2008;64(2 Suppl):S8691.
complexes have cleared and patients with previous Bannon MP, ONeill CM, Martin M, Ilstrup DM, Fish NM,
episodes o HI can be sa ely treated with heparin Barrett J. Central venous oxygen saturation, arterial base
de cit, and lactate concentration in trauma patients. Am
provided ELISA testing is negative. T e IgG response
Surg. August 1995;61(8):73845.
is not amnestic and previous HI does not increase Baron JF. Blood substitutes. Haemoglobin therapeutics in
risk or uture HI . clinical practice. Crit Care. 1999;3(5):R99102.
Bo ard KD, Riou B, Warren B, Choong PI, Rizoli S, Rossaint
5. C. ransmission o in ectious diseases by trans u- R, et al. Recombinant actor VIIa as adjunctive therapy or
sion in the United States is rare. Bacterial in ection bleeding control in severely injured trauma patients: wo
is clinically apparent in 1:80,000 cases. Storage o parallel randomized, placebo-controlled, double-blind
clinical trials. J rauma. July 2005;59(1):815.
platelets at room temperature may increase risk o
CRASH-2 Collaborators, Intracranial Bleeding Study. E ect
bacterial in ection a er in usion o platelets. Current o tranexamic acid in traumatic brain injury: A nested ran-
rates o viral transmission are as ollows (per unit domised, placebo controlled trial (CRASH-2 Intracranial
trans used): Hepatitis A (1:1 million), Hepatitis B Bleeding Study). BMJ. July 1, 2011;343:d3795.
C H AP TER 8 0 Bl o o d P Ro d u C Ts An d TRAn s f u s i o n 307

CRASH-2 trial collaborators, Shakur H, Roberts I, Bautista R, Looney MR, Gilliss BM, Matthay MA. Pathophysiology o
Caballero J, Coats , et al. E ects o tranexamic acid on trans usion-related acute lung injury. Curr Opin Hematol.
death, vascular occlusive events, and blood trans usion in September 2010;17(5):41823.
trauma patients with signi cant haemorrhage (CRASH-2): Perkins RM, Aboudara MC, Abbott KC, Holcomb JB. Resuscita-
A randomised, placebo-controlled trial. Lancet. July 3, tive hyperkalemia in noncrush trauma: A prospective, obser-
2010;376(9734):2332. vational study. Clin J Am Soc Nephrol. March 2007;2(2):3139.
Davenport RD. Pathophysiology o hemolytic trans usion Popovsky MA, Moore SB. Diagnostic and pathogenetic con-
reactions. Semin Hematol. July 2005;42(3):1658. siderations in trans usion-related acute lung injury. rans-
Francis JL, Palmer GJ 3rd, Moroose R, Drexler A. Compari- fusion. November-December 1985;25(6):5737.
son o bovine and porcine heparin in heparin antibody Starr AJ, Grif n DR, Reinert CM, Frawley WH, Walker J,
ormation a er cardiac surgery. Ann T orac Surg. January Whitlock SN, et al. Pelvic ring disruptions: Prediction o
2003;75(1):1722. associated injuries, trans usion requirement, pelvic arteri-
Hauser CJ, Bo ard K, Dutton R, Bernard GR, Croce MA, ography, complications, and mortality. J Orthop rauma.
Holcomb JB, et al. Results o the CON ROL trial: Ef cacy September 2002;16(8):55361.
and sa ety o recombinant activated Factor VII in the man- Sayah DM, Looney MR, oy P. rans usion reactions: newer
agement o re ractory traumatic hemorrhage. J rauma. concepts on the pathophysiology, incidence, treatment,
September 2010;69(3):489500. and prevention o trans usion-related acute lung injury.
Hbert PC, Wells G, Blajchman MA, Marshall J, Martin C, Crit Care Clin. July 2012;28(3):36372.
Pagliarello G, et al. A multicenter, randomized, controlled Sihler KC, Napolitano LM. Complications o massive trans u-
clinical trial o trans usion requirements in critical care. sion. Chest. January 2010;137(1):20920.
rans usion Requirements in Critical Care Investiga- oy P, Gajic O, Bacchetti P, Looney MR, Gropper MA, Hubmayr
tors, Canadian Critical Care rials Group. N Engl J Med. R, et al. rans usion-related acute lung injury: Incidence
February 11, 1999 Feb;340(6):40917. and risk actors. Blood. February 16, 2012;119(7):175767.
Hirsh J, Raschke R. Heparin and low-molecular-weight hepa- Vincent JL, Baron JF, Reinhart K, Gattinoni L, T ijs L, Webb A,
rin: T e seventh ACCP con erence on antithrombotic et al. Anemia and blood trans usion in critically ill patients.
and thrombolytic therapy. Chest. September 2004;126(3 JAMA. September 25, 2002;288(12):1499507.
Suppl):188S203S. West HC, Jurkovich G, Donnell C, Luterman A. Immediate
Lanzarotti S, Weigelt JA. Heparin-induced thrombocytopenia. prediction o blood requirements in trauma victims. South
Surg Clin North Am. December 2012;92(6):155972. Med J. February 1989;82(2):1869.
Levi M, Levy JH, Andersen HF, rulo D. Sa ety o recom- Zentai C, Grottke O, Spahn DR, Rossaint R. Nonsurgical tech-
binant activated actor VII in randomized clinical trials. niques to control massive bleeding. Anesthesiol Clin. March
N Engl J Med. November 4, 2010;363(19):1791800. 2013;31(1):4153.
81
Necrotizing Fasciitis

Allyson L. Berglund and John M. Giurini

A 33-year-old emale with a 19 year history o type 2 A. Plain radiographs demonstrating subcutaneous
insulin dependent diabetes complicated by peripheral gas is present in about 25% o cases.
neuropathy, peripheral vascular disease, and chronic B. A computerized tomography (C ) scan has a
kidney disease presents to the emergency room com- sensitivity o about 50%.
plaining o worsening right oot pain and swelling over C. A laboratory risk indicator or necrotizing asci-
the past 3 days. She has a history o Charcot de ormity itis (LRINEC) score o 4 indicates an over 50%
to her right oot with a chronic right oot ulceration that chance o necrotizing asciitis.
has been treated with weekly debridements and wound D. A good physical exam will di erentiate amongst
care. She endorses subjective evers and chills. cellulitis, an abscess and necrotizing asciitis.
On examination she has a 1 cm 1 cm 3 cm deep
plantar mid oot ulceration with probing and tracking 3. When necrotizing fasciitis is suspected, immediate
noted dorsally to the mid oot. T ere is a strong malo- operative debridement is indicated. Which of the
dor and her entire oot is signi cantly edematous and following is true regarding surgical exploration?
erythematous with signi cant pain on palpation o the A. Surgical exploration should be delayed until vas-
mid oot and lower leg. She has biphasic dopplerable cular status can be evaluated and optimized.
pulses. B. In dubious cases, surgical exploration should be
avoided until the diagnosis is con rmed to limit
1. Regarding clinical presentation, which of the unnecessary large incisions and tissue debride-
following is true? ment.
A. Necrotizing asciitis has the same incidence in C. Repeated surgical debridements are not neces-
both adult and pediatric populations. sary i the patient is on appropriate intravenous
B. Initial signs o necrotizing asciitis include skin antibiotics.
necrosis with a blue or purple discoloration, D. Macroscopic ndings include gray necrotic tis-
crepitus and bullae. sue, dishwater pus, and a positive nger test.
C. T e most constant clinical eature is pain, out o
proportion, to physical ndings. 4. Which of the following is true regarding isolated
D. When the borders o in ection appear ill-de ned, organisms and antibiotic therapy?
erysipelas diagnosis is more likely than necrotiz- A. Broad spectrum intravenous antibiotics are
ing asciitis. enough to stop the spread o in ection.
B. T e most commonly isolated organism is Clos-
2. With regards to the diagnostic tools that are tridium, and antibiotic therapy should always
available, which of the following is true? include coverage or this organism.
C H AP TER 8 1 N EC Ro Ti z i N g FAs C i i Ti s 309

C. Long term intravenous antibiotics or > 4 weeks 2. A. Subcutaneous gas seen on plain radiographs is a
is standard practice when necrotizing asciitis speci c, but not sensitive nding. It has been docu-
has been diagnosed and the patient is ree rom mented that subcutaneous gas on radiographs is ound
systemic symptoms. in as low as 25% o cases, and its absence should not
D. Clindamycin may be use ul in controlling exo- exclude the diagnosis o necrotizing asciitis.
toxin production especially in cases complicated T e diagnosis o necrotizing asciitis can of en be
by streptococcal toxic shock syndrome. delayed and mistaken or cellulitis or abscess. Given
many o the initial signs and symptoms are non-
5. In regards to prognosis, which of the following is speci c, both clinical ndings and laboratory values
true? in addition to advanced imaging i needed should be
A. Patients in ected with Clostridia have lower used in conjunction to help assist in early diagno-
mortality and limb loss rates compared to those sis. In 2004, Wong et al. developed a scoring system
with a polymicrobial or other monocrobial LRINEC, which classi es patients into risk catego-
in ection. ries that determine necrotizing asciitis probability,
B. T e single most important actor that negatively as seen in able 81-1 below. LRINEC scores greater
in uences prognosis is delayed surgical debride- than or equal to 6 have a positive predictive value
ment. (PPV) o 92% and a negative predictive value (NPV)
C. Hyperbaric oxygen therapy (HBO) has been o 96% or necrotizing asciitis.
shown to dramatically decrease limb loss. Magnetic resonance imaging (MRI) has been
D. Diabetes, even in those presenting in diabetic documented to be the most use ul imaging tech-
ketoacidosis, does not have higher mortality or nique when di erentiating necrotizing rom non-
longer hospital stays. necrotizing in ections. Speci c ndings on an MRI
include thickening o the sof tissue and a hyperin-
tense signal on -2 weighted images at the level o
the deep ascia and muscle. C imaging has been
ANSWERS
reported to be more sensitive than plain radiographs,
1. C. Pain, out o proportion to physical ndings, has
been well documented as a constant recurring ea-
ture o necrotizing asciitis. When present, necrotiz- Table 81-1 LABORATORY RISK INDICATORS FOR
NECROTIZING FASCIITIS (LRINEC) SCORE
ing asciitis should always be considered in the
di erential diagnosis. Keys that lead to diagnosis Variable, Units Score
include pain out o proportion to the degree o der- C-Reactive Protein, mg/L
mal involvement as well as severe pain that appears < 150 0 0
to extend beyond the apparent borders o in ection. 150 3.5 4
Necrotizing asciitis is more common in the adult otal white cell count, per mm 2
population, with a reported incidence o 0.40 cases < 15 0 0
per 100,000 as compared to 0.08 per 100,000 pediat- 1525 0.5 1
ric cases per year. While skin necrosis with a blue or > 25 2.1 2
purple discoloration, crepitus and bullae are perhaps Hemoglobin, g/dL
> 135 0 0
the more well recognized signs and symptoms o
11135 0.6 1
necrotizing asciitis, these are late eatures o the dis- < 11 1.8 2
ease, not initial presenting symptoms. Initial present- Sodium, mmol/L
ing signs are of en less speci c and include erythema, 135 0 0
warmth, myalgia, edema, and pain out o portion. As > 141 1.8 2
such, this of en can lead to delayed diagnosis and Creatinine, mol/L
delayed surgical debridement. Erysipelas is di erent 141 0 0
rom necrotizing asciitis in that erysipelas involves > 141 1.8 2
in ection o the super cial layers o the skin and Glucose, mmol/L
10 0 0
cutaneous lymphatics, leading to a well-demarcated
> 10 1.2 1
and of en raised border.
310 g EN ERAL s U Rg ERY EXAM i N ATi o N AN D Bo ARD REVi EW

with the ability to identi y abscesses and other the bacterial cell wall. However, various studies have
in ammatory changes, with up to 80% sensitivity. ailed to show any bene ts, including mortality or
hospital stay, with the addition o HBO treatment.
3. D. Gross intraoperative ndings include thin, watery As previously mentioned, delay in surgical
gray necrotic uid of en described as dishwater pus, debridement is the single most important actor
a oul smelling odor, and necrotic muscle which ails with regards to a negative prognosis; a delay o over
to respond to electrocautery. A positive nger test is 24 hours is an independent predictor o mortality. It
also characteristic, in which there is ease o dissect- is well known that diabetes in itsel is a risk actor
ing the subcutaneous layer o o the deep ascia with or necrotizing asciitis. T ose with a history o type
the surgeons ngers. 2 diabetes and who present in diabetic ketoacido-
Surgical debridement should never be delayed or sis have been shown to have longer rates o hospital
optimization o vascular per usion, nor until there stays and higher rates o mortality. T is is likely due
is a de nitive diagnosis as delay in surgical manage- to poor glycemic control which contributes to the
ment has been shown to be both limb and li e threat- pathogenesis and has been correlated to the extent
ening. A delay in surgical debridement more than o disease and poor outcomes. Similarly, in a large
24 hours has been shown to be an independent risk retrospective cohort study, patients with isolated
actor or mortality. Furthermore, repeated surgical Clostridium in ections have been shown to have
debridement may be necessary in order to remove increased mortality rates, up to our times that when
all necrotic tissue, lessen the bacterial load, expose compared to other monocrobial or polymicrobial
the tissues to oxygen to aid in reduction o anaerobic in ections.
bacteria, all helping to acilitate more rapid recovery.

BIBLIOGRAPHY
4. D. Clindamycin is a protein synthesis inhibitor that
inhibits M protein and exotoxin production. As such, Anaya DA, McMahon K, Nathens AB, et al. Predictors o mor-
tality and limb loss in necrotizing sof tissue in ections.
administration o clindamycin has been shown to be
Arch Surg. 2005;140:1517.
especially use ul in cases with severe in ammatory Andreasen J, Green SD, Childers BJ. Massive in ectious
responses, which include cases complicated by strep- sof -tissue injury: Diagnosis and management o necrotiz-
tococcal toxic shock syndrome. Initial management ing asciitis and purpura ulminans. Plast Recontstr Surg.
o suspected necrotizing asciitis cases include broad 2001;107:102535.
spectrum IV antibiotics. Duration o IV antibiotics Bisno AL, Stevens DL. Streptococcal In ections o Skin and
Sof issues. N Engl J Med. 1996;334(4):2406.
af er the source has been controlled and the patient is Brown DR, Davis NL, Lepawsky M, Cunningham J, and
ree o systemic signs o in ammation and in ection, Kortbeek J. A multicenter review o the treatment o major
is typically 14 days. truncal necrotizing in ections with and without hyperbaric
It should be noted that IV antibiotics alone are oxygen therapy. Am J Surg. 1994;167(5):485489.
not enough to control in ection. Using only IV Childers BJ, Potyondy LD, Nachreiner R, et al. Necrotizing
asciitis: A ourteen-year retrospective study o 163 con-
antibiotics, mortality approaches 100% because the
secutive patients. Am Surg. 2002;68:10916.
antibiotics may not reach the a ected area due to Edlich RF, Cross CL, Dahlstrom JJ, Long WB III. Modern con-
the thrombosis o vessels. As a result o improved cepts o the diagnosis and treatment o necrotizing asci-
sanitation, Clostridium species are now a rare cause itis. J Emerg Med. 2010;89:736.
o necrotizing asciitis. Rather, Streptrococcus is Fustes-Morales A, Gutierrez-Castrellon P, Duran-McKinster
the number one isolated organism, ollowed by the C, et al. Necrotizing asciitis: Report o 390 pediatric cases.
Arch Dermatol. 2002;138:8939.
staphylococcus species. Hassan Z, Mullins RF, Friedman BC, et al. reating necrotiz-
ing asciitis with or without hyperbaric oxygen therapy.
5. B. HBO therapy has been considered as an adjunct Undersea Hyperb Med. 2010;37:11523.
therapy in the treatment o necrotizing asciitis. T is Lancerotto L, occo I, Salmaso R, Vindigni V, and Bassetto F.
is based on the theory that necrotizing in ections Necrotizing asciitis: Classi cation, diagnosis and manage-
ment. J Trauma. 2011;72(3):5606.
are associated with decreased oxygen tension and
Lim YJ, Yong FC, Wong CH, and an ABH. Necrotizing as-
ischemia. As such, it is believed that HBO treatment ciitis and traditional medical therapy a dangerous liai-
can reverse these e ects by increasing oxygen ten- son. Annals of the Academy of Medicine Singapore. 2006;
sion and helping to deliver antibiotic therapy across 35(4):2703.
C H AP TER 8 1 N EC Ro Ti z i N g FAs C i i Ti s 311

Oncul O, Erenoglu C, op C, et al. Necrotizing asciitis: a Wang YS, Wong CH, ay YK. Staging o necrotizing asciitis
li e threatening clinical disorder in uncontrolled type 2 based on the evolving cutaneous eatures. Int J Dermatol.
diabetic patients. Diabetes Research and Clinical Practice. 2007;46:103641.
2008;80(2):21823. Wong CH, Khin LW, Heng KS, an KC, and Low CO. he
Ozalay M, Ozkoc G, Akpinar S, Hersekli MA, andogan RN. LRINEC (laboratory risk indicator or necrotizing
Necrotizing sof -tissue in ection o a limb: Clinical pre- asciitis) score: A tool or distinguishing necrotizing
sentation and actors related to mortality. Foot Ank Int. rom other so t tissue in ections. Crit Care Med. 2004;32:
2006;27(8):598605. 153541.
Shupak AO, Shoshani I, Goldenber A, et al. Necrotizing asci- Wong CH, Chang HC, Pasupathy S, et al. Necrotizing asciitis:
itis: an indication or hyperbaric oxygen therapy? Surgery. Clinical presentation, microbiology, and determinates o
1995;118(5):8738. mortality. J Bone Joint Surg Am. 2003;85:145460.
Vijayakumar A, Pullagura R, T immappa D. Necrotizing as- Wysoki MG, Santora A, Shah RM, and Friedman AC. Nec-
ciitis: Diagnostic challenges and current practices. ISRN rotizing asciitis: C characteristics. Radiology. 1997;
Infectious Diseases. 2014; Article ID 208072. 2003:85963.
82
Genitourinary rauma

Peter L. Steinberg

A 35-year-old man presents to the emergency depart- E. C scan o the abdomen with and without IV
ment as a trauma alert, a er alling 40 eet rom a sca - contrast
old. On primary survey he has no li e threatening
injuries. His vital signs are stable, with a heart rate o 85, 3. What is the appropriate management o a simple
blood pressure o 126/84, and a room air oxygen satura- extraperitoneal bladder rupture?
tion o 99%. A. Suprapubic tube placement or 7 days
On secondary survey he is noted to have blood at the B. Large bore Foley catheter placement or 10 to
urethral meatus. In addition, he has bruising over the 14 days
le ank. He has pain over the pubic symphysis, but his C. Open exploration and cystorraphy
pelvis is grossly stable. His abdomen is mildly distended. D. Laparoscopic exploration and cystorraphy
His rectal examination reveals no blood, normal rectal E. Bilateral percutaneous nephrostomy tube place-
tone, and a normal prostate exam. He has not voided ment
since the incident.
4. C scan o the abdomen and pelvis with IV
1. What is the next step in diagnosis and management contrast is indicated to rule out a renal injury in
o blood at the urethral meatus? which clinical scenario(s)?
A. Passage o a coudee tipped Foley catheter A. Blunt trauma with gross hematuria
B. Passage o a three-way Foley catheter B. Blunt trauma with microscopic hematuria and
C. Placement o a suprapubic catheter normal blood pressure
D. Retrograde urethrography C. Blunt trauma with microscopic hematuria and
E. Voiding cystourethrogram shock (SBP < 90)
D. High energy mechanism o injury
2. Af er concluding his urethra is uninjured, a Foley E. A, C, and D
catheter is placed and grossly bloody urine is
obtained. Which imaging test is most appropriate in 5. T is patient has a Grade 3 lef renal injury and is
evaluating his other possible genitourinary injuries? hemodynamically stable. What is the appropriate
A. Fluoroscopic cystogram initial management o that injury?
B. FAS ( ocused assessment with sonography or A. Open renal exploration
trauma) scan B. Renal angiography
C. Computerized tomography (C ) scan o the C. Serial examination, serial vital signs, and serial
abdomen and pelvis with IV contrast and C CBC monitoring
cystogram D. Ureteral stent placement
D. C cystogram E. Nephrectomy
C H AP TER 8 2 G En i To u Ri n ARy TRAu m A 313

ANSWERS o two kidneys and to provide a pyelogram o each


renal unit.
1. D. Retrograde urethrography is essential in any case
o blood at the urethral meatus, as blood at the mea- 3. B. A simple extraperitoneal bladder injury can be
tus may be a sign o a urethral disruption or lacera- readily managed with Foley catheter drainage alone.
tion. Passage o a Foley catheter blindly in the setting T e catheter must be large enough to allow the egress
o a urethral injury may exacerbate a laceration or o clots and be unlikely to occlude. Most clinicians
lead to placement o the catheter outside o the blad- would allow 2 weeks o drainage or healing to occur
der. I the retrograde urethrogram shows the urethra and would per orm a cystogram prior to catheter
is intact, then Foley placement can be attempted. removal.
Operative management is only needed i the
2. C. T is patient is at risk or both bladder and renal patient is otherwise being explored or i there are
injury, so imaging will be required to assess those mitigating actors that will preclude healing, such as
organs. T e bladder is best imaged with either a ret- ragments o the bone in the bladder injury. In addi-
rograde gravity cystogram or a C cystogram. T e tion, i there are lacerations to the rectum, vagina,
kidneys are best imaged with a contrast enhanced C or the bladder injury involves the bladder neck, oper-
scan o the abdomen and pelvis, as well as with delayed ative repair is best per ormed to prevent a f stula and
images o the ureters to assess or ureteral injury. allow appropriate healing.
A gravity cystogram can also be done with a c-arm
4. E. T e guidelines or renal imaging in blunt trauma
in the operating room i the patient needs operative
are very well established. In the absence o gross
intervention. It is critical that gravity cystography
hematuria, with an insignif cant mechanism o injury,
involve oblique views as well as a post drainage f lm,
and without a systolic blood pressure < 90 mm Hg
to assess or leakage posterior to the bladder. Also,
and microscopic hematuria (> 35 RBCs per HPF),
the patient should be f lled via gravity to a volume
there is a 99.7% chance the patient does not have a
o at least 300 to 400 mL, to adequately distend the
signif cant renal injury. A contrast enhanced C with
bladder. Awake patients should be f lled to a sense o
delayed images is the optimal imaging test in this
bladder ullness.
setting.
Roughly 90% o all patients with a bladder injury
in a blunt trauma setting will have both gross hema- 5. C. Most renal injuries can be managed with non-
turia and a pelvic racture. Additionally, nearly 30% o operative intervention. Operative intervention,
patients with gross hematuria and a pelvic racture will including open exploration with repair o a renal
have a bladder injury, so imaging the bladder is essen- injury or nephrectomy, ureteral stent or percutaneous
tial in those patients. A minority o patients with a nephrostomy placement or urinary extravasation, or
bladder injury will have only hematuria or only an iso- angiography, is required or patients with hemody-
lated pelvic racture, so the decision to image the blad- namic instability or or urinary leaks that do not heal
der in those situations is based on clinical judgment. or are complicated by issues such as in ection.
Certainly any signs or symptoms o bladder per ora- T e majority o renal injuries, especially grade
tion, such as low urine output, abdominal distension, 1 to 3 injuries can be managed without surgical
or acute kidney injury would warrant cystography. intervention.
T e indications or renal imaging in blunt trauma
patients include gross hematuria, microscopic hema- BIBLIOGRAPHY
turia (3 to 5 RBCs per high powered f eld on urinaly-
Broghammer JA, Fisher MB, Santucci RA. Conservative
sis) with systolic BP < 90 mm Hg, and a mechanism management o renal trauma: a review. Urology. October
with high energy that could lead to renal injury. A 2007;70(4):6239.
contrast enhanced C scan with delayed images o Miller KS, McAninch JW. Radiographic assessment o renal
the ureters is the best imaging test in a trauma set- trauma: Our 15-year experience. J Urol. 1995;154:3525.
ting. In the event o urgent exploration, a 1 shot Morey AF, Brandes S, Dugi DD III, Armstrong JH, Breyer BN,
et al. Urotrauma: AUA Guideline. J Urol, 2014;116.
intravenous pyelogram (IVP) can be per ormed with Wessells H, Suh D, Porter JR, et al.: Renal injury and operative
2 cc/kg o IV contrast and a single plain abdomi- management in the United States: Results o a population-
nal f lm 10 minutes later, to assess or the presence based study. J Trauma. 2003;54:423.
83
Foley rauma

Katherine Carlisle

A 64-year-old male with a history o spinal cord injury, 3. T e next best step for the nurse would be to:
hypertension, DM2, and tobacco abuse is scheduled to A. Slowly orce the catheter past the resistance.
undergo a video-assisted thoracoscopic surgery (VA S) B. Dilate a possible stricture by slowly in ating the
or resection o a suspicious lung mass. He voids using catheter balloon.
clean intermittent catheterization (CIC) and has been C. Assume the positioning is correct and in ate the
doing so or the past 8 years. Prior to the start o the catheter balloon.
case, the nurse attempts to insert a Foley catheter. She D. Remove the catheter.
meets resistance while advancing the catheter and does
not get return o urine. 4. T e nurse repeatedly tries to force the catheter
without success before removing the catheter. T e
1. Which of the following is true? urology team is called for consultation. Which of
A. A history o prolonged CIC does not increase the the following is the next best step?
risk o Foley trauma. A. Attempt to blindly dilate the urethra.
B. Voiding symptoms help identi y patients who B. Per orm cystoscopy.
are more likely to have di cult catheter place- C. Attempt to pass a larger size catheter.
ment. D. Per orm a retrograde urethrogram.
C. T e incidence o urethral stricture as a compli-
cation o chronic catheterization decreases with
ANSWERS
duration.
D. Women experience similar rates o urethral 1. B. Urethral catheterization is not a benign procedure
stricture as men. and can result in numerous complications to include
hematuria, urinary tract in ection, and urethral stric-
2. Which of the following is true? ture, per oration, or erosion. T e incidence o ure-
A. In patients with known stricture disease, catheter thral stricture in women is signi cantly less than that
placement requires direct visualization. in men and is primarily related to the shorter length
B. Using CIC to void rather than an indwelling o the urethra, 3.5 to 4 cm in women, versus 18 to
catheter does not increase risk o alse passage. 20 cm in men. It is important to do a thorough his-
C. Indwelling catheters have a lower risk o devel- tory and physical to identi y patients at risk or di -
oping a urethral stricture. cult catheter placement. Urologic procedural history
D. Resistance or obstruction encountered less than should be reviewed along with any documentation
16 cm rom the meatus may suggest urethral o prior di cult catheter placement. In men, voiding
stricture or other anterior urethral pathology. symptoms such as decreased or intermittent urinary
C H AP TER 8 3 F o l Ey TRAu m A 315

stream, valsava to initiate urination, and nocturia 3. D. T e most appropriate course o action would be
may indicate obstructive pathology such as benign to remove the catheter. Con rmation o appropriate
prostatic hypertrophy. History o repeated urethral catheter placement requires ow o urine and the bal-
manipulation with CIC or indwelling catheter such loon should not be in ated until this time. Continu-
as the patient in our scenario should prompt suspi- ing to advance the catheter af er meeting resistance
cion or prior urethral trauma and possible di cult could result in creation o a alse passage. You should
Foley placement. never in ate the balloon intentionally within the ure-
Incidence o urethral injury and chronic com- thra. I a stricture is identi ed it may require dilation
plications increases with prolonged duration o by a urologist with urethral sounds or dilators.
CIC. Alternate catheter types can provide advan-
tages over straight-tipped catheters such as Foleys 4. B. T e sa est option would be to examine the urethral
catheter. For example, a Coud catheter which has anatomy using cystoscopy. T is acilitates placement
a curved tip assists in traversing the prostatic ure- o a guidewire beyond any identi ed stricture over
thra in gentlemen with benign prostatic hyperplasia which a catheter can be placed or dilation o a stric-
(BPH) whereas a Councill open-tipped catheter can ture under direct vision. Attempting to pass a larger
be placed over a guidewire in patients with urethral catheter in a patient who is suspected to have a stric-
stricture. ture would likely result in urther trauma. A smaller
bore catheter would be the appropriate choice. Retro-
2. B. Considering the patients known history o pro- grade urethrogram is a contrasted X-ray study to
longed CIC, urethral stricture would be high on evaluate the urethra that is of en ordered when ure-
the di erential or the etiology o the encountered thral trauma is suspected. Although this study could
resistance during the nurses attempt at Foley place- show evidence o urethral stricture, it would not be
ment. Studies comparing long term CIC use versus the best choice while in the operating room. T is
indwelling catheter or urinary drainage reveals an study may be obtained as a ollow up study to urther
increased risk or development o urethral stricture assess the anatomy o the urethra particularly i surgi-
in patients with an indwelling catheter, but interest- cal intervention is planned to treat urethral stricture.
ingly no signi cant di erence in the risk or alse
passage between the two groups. T e rst 16 cm BIBLIOGRAPHY
o the urethra is the anterior portion and would Mndez-Probst CE, Razvi H, Denstedt JD. Fundamen-
be the most likely location or urethral stricture tals o Instrumentation and Urinary ract Drainage. In:
related to Foley trauma. T e remaining 2 to 4 cm Wein AJ, Kavoussi LR, Novick AC, Partin AW, Peters CA,
corresponds to the posterior urethra and resistance eds. Campbell-Walsh Urology. 10th ed. Philadelphia, PA:
Elsevier Saunders; 2011:17782.
at this distance may re ect BPH. Had the patient Perrouin-Verbe B, Labat J, Richard I, et al. Clean intermittent
been previously diagnosed with urethral stricture catheterisation rom the acute period in spinal cord injury
or i the providers suspicion or stricture was high, patients. Long term evaluation o urethral and genital tol-
they may choose to use a smaller silicone catheter erance. Paraplegia. 1995;33:61924.
to more easily traverse the narrowed segment with- Singh R., Rohilla RK, Sangwan K, et al. Bladder management
methods and urological complications in spinal cord injury
out direct visualization. T is technique would be
patients. Indian Journal of Orthopaedics. 2011;45:1417.
most e ective in the hands o a urologist or other Villanueva C, Hemstreet III GP. Di cult male urethral cath-
provider well trained in urinary tract pathology and eterization: A review o di erent approaches. International
anatomy. Brazilian Journal of Urology. 2008;34:40112.
This page intentionally left blank
SECTION 2

Surgical Subspecialties
This page intentionally left blank
Tr a nspl a nt at io n Sur ger y
Ronald A. Gagliano, Jr.
This page intentionally left blank
84
ransplant SurgeryKidney

J S k

A 61-year-old patient with insulin dependent diabetes shows lymphocytes within the renal tubules and
mellitus and end-stage renal disease on hemodialy- vascular endothelium. Immunophenotyping o the
sis presents or evaluation or kidney transplantation. biopsy demonstrates -cell preponderance.
A suitable cadaveric donor is ound, and the patient
undergoes transplantation at another hospital; the 2. Wh ch f h f w d h
immediate post-operative course is uncomplicated. en - v c m c x c d b h
days later, the patient presents to your hospital and is ?
noted to have an abrupt decrease in urine output with a A. T is complication is mediated by anti-HLA anti-
creatinine elevation to 1.5 mg/dL, and has gra tender- bodies and is likely related to a clerical error dur-
ness on exam. A ever o 100.9F is noted. ing the pre-transplant cross-match.
B. T e presence o Epstein-Barr virus (EBV) is
1. Wh ch f h f w b h strongly associated with the development o this
m m f h d v d neoplasia, and treatment involves decreasing
mc ? immunosuppressive therapy.
A. Renal biopsy should rst be per ormed to di er- C. Failure to treat this condition will increase the
entiate between acute rejection and acute inter- risk o gra loss. Management consists o pulse
stitial nephritis. corticosteroids and changing maintenance ther-
B. Doppler ultrasound should be per ormed to apy rom cyclosporine to tacrolimus.
evaluate vessel patency and rule out ureteral D. Risk actors or this complication include peri-
obstruction. operative hypovolemia, ischemia-reper usion
C. T e dose o cyclosporine should be increased to injury, and the use o an extended-criteria donor.
treat possible acute rejection. E. T is viral in ection was likely transmitted via
D. Angiography is indicated to evaluate the vascu- leukocytes within the donor; treatment includes
lar supply to the gra . ganciclovir and IntraVenous ImmunoGlobulin
E. T e patient should be promptly taken back to (IVIG).
the operating room or re-exploration in order
to salvage the gra . 3. Wh ch f h f w c c d -
Cyclosporine levels are drawn on the previous ch c c m c ?
patient and ound to be at therapeutic levels. A Dop- A. T rombosis o the transplanted renal artery
pler ultrasound is obtained and shows patent arterial in the early post-operative period should be
and vascular f ow with no perinephric f uid collec- urgently treated with heparin in usion and
tion. Subsequently, a percutaneous biopsy o the gra angioplasty.
322 G EN ERAL S U RG ERY EXAM IN ATI O N AN D BO ARD REVIEW

B. Renal vein thrombosis may mani est with hema- hyperacute or acute rejection, renal artery throm-
turia in the rst week ollowing transplantation. bosis, renal vein thrombosis, urine leak, or ureteral
C. On Doppler ultrasound, identi cation o a obstruction (e.g., by stricture or lymphocele). T e
round, sonolucent, septated mass medial to the most requent cause o an elevated creatinine level
renal allogra with associated ureteral compres- post-transplant is calcineurin inhibitor toxicity. A er
sion is most consistent with a urine leak. this is ruled out, the next step is to evaluate blood
D. Initial management o a lymphocele involves f ow and rule out structural problems with Doppler
intra-peritoneal marsupialization (i.e., creation ultrasound. I the ultrasound is normal, it is then
o peritoneal window). appropriate to per orm a renal biopsy, which can di -
E. Presence o multiple strictures in the transplanted erentiate rejection, post-transplant lymphoproli er-
ureter during the early post-operative period is ative disorder (P LD), acute tubular necrosis (A N),
associated with polyoma BK viral in ection. among other diagnoses. I arterial/venous thrombo-
sis, arterial stenosis, or aneurysm is seen on the ultra-
4. R d h m ch m d f v sound, angiography is then indicated to con rm the
mm v , wh ch f h f w diagnosis. I no blood f ow is seen to the transplanted
c c? kidney on ultrasound or a nuclear medicine renal
A. Cyclosporin binds cyclophilin protein and scan, then re-exploration is indicated.
blocks IL-2 production.
B. Azathioprine binds antigens on cells, causing 2. C. -lymphocytes within the renal tubules and vas-
altered cell unction as well as cell depletion. cular endothelium are characteristic o acute cell-
C. Rituximab binds to FK binding protein, and is mediated rejection. Each rejection episode decreases
used in maintenance immunosuppression. the long-term unction o the gra , and should be
D. Sirolimus inhibits the unction o NF-B, dimin- treated with pulse corticosteroids. Also, i the epi-
ishing the response to cytokines. sode o rejection occurred on therapeutic calcineu-
E. A GAM binds to CD20, leading to initial cyto- rin inhibitor levels, the maintenance therapy should
kine release ollowed by B cell depletion. be changed. In contrast, hyperacute rejection occurs
within minutes to hours o reper usion, mani ests
5. W h d c m c f m with rapid mottling and o en gra rupture, and is
d mm , wh ch f h mediated by pre- ormed anti-HLA antibodies due
f w c c? to ABO incompatibility. Hyperacute rejection is rare
A. T e most common malignancy associated with due to pre-transplant cross-matching, and is usually
post-transplantation immunosuppression is due to clerical error. Post-transplant lymphoproli -
lymphoma. erative disorder (P LD) is associated with EBV, and
B. Mycophenolate mo etil and azathioprine are can be di erentiated rom acute rejection on biopsy
associated with diarrhea and leukopenia. by the preponderance o B lymphocytes; treatment
C. Antithymocyte globulin is associated with gingi- is to decrease immunosuppression. Acute tubular
val hyperplasia and hirsutism. necrosis (A N) may cause oliguria and elevated
D. Post-transplant activation o cytomegalovirus creatinine, and risk actors include hypovolemia,
(CMV) occurs most commonly in patients who reper usion injury, and use o an extended-criteria
tested seropositive or CMV IgG preoperatively. donor. Biopsy would show injury o tubular cells and
E. acrolimus and sirolimus are associated with casts within the tubule lumen. CMV in ection may
nephrotoxicity and post-transplant diabetes. be transmitted within donor leukocytes and also can
cause oliguria with creatinine elevation; treatment is
with ganciclovir. On biopsy, intranuclear or cytoplas-
ANSWERS mic inclusions are characteristic o viral in ections
including CMV.
1. B. Post-transplant oliguria with elevated creatinine
has multiple possible etiologies to include hypov- 3. B. Renal artery thrombosis is a rare complication
olemia, urinary catheter occlusion, calcineurin inhib- occurring in 1% o cases, and necessitates return to
itor toxicity, viral in ection, delayed gra unction, the operating room or urgent exploration. New onset
C H AP TER 8 4 TRAN S P LAN T S U RG ERY KI D N EY 323

hematuria may be the rst sign o a renal vein throm- most commonly in patients who tested sero-negative
bosis. Lymphoceles are the result o intra-operative or CMV pre-operatively and received a gra rom
lymphatic disruption and are identi able on ultra- a CMV-seropositive donor. Mycophenolate mo etil
sound as a round, multi-septated mass medial to the and azathioprine are both antiproli erative agents
gra that may compress the ureter. It is di erentiated with associated GI toxicity and leukopenia. Antithy-
rom urine leak sonographically in that a urine leak mocyte globulin is associated with cytokine release
would be a non-septated f uid collection in the pel- syndrome, leukopenia, and serum sickness; gingival
vis. Lymphocele is initially managed with percutane- hyperplasia and hirsutism are both associated with
ous drainage; creation o a peritoneal window can be cyclosporine. Like cyclosporine, tacrolimus is a cal-
help ul or drainage i percutaneous drainage does cineurin inhibitor associated with nephrotoxicity, but
not resolve symptoms. Presence o multiple ureteral sirolimus is not, and tacrolimus is more strongly asso-
strictures in the late post-operative period may be ciated with post-transplant diabetes than sirolimus.
due to polyoma BK virus, but early ureteral stenosis
is usually due to ischemia or extrinsic compression. BIBLIOGRAPHY
Adams AB, Kirk AD, Larsen CP. ransplantation immunobi-
4. A. Cyclosporine is a calcineurin inhibitor which ology and immunosuppression. In: ownsend CM, Beau-
unctions by binding cyclophilin protein and block- champ D, Evers BM, Mattox KL, eds. Sabiston extbook o
ing IL-2 production. Azathioprine unctions by Surgery: T e Biological Basis o Modern Surgical Practice.
converting 6-mercaptopurine to 6-thioinosine- 19th ed. Philadelphia, PA: Elsevier; 2012:61754.
Becker Y. Kidney and Pancreas ransplantation. In: ownsend
5-monophosphate, which inter eres with DNA CM, Beauchamp D, Evers BM, Mattox KL, eds. Sabiston
and purine synthesis; -cell antigens are bound extbook o Surgery: T e Biological Basis o Modern Surgical
and blocked by antilymphocyte globulins such as Practice. 19th ed. Philadelphia, PA: Elsevier; 2012:66681.
A GAM, as well as OK -3 and several monoclonal Freise C, Stock P. Renal ransplantation. In: Mulholland MW,
antibodies (e.g., basiliximab, dacluzimab). Rituximab Lillemoe KD, Doherty GM et al., eds., Greenf elds Surgery:
Scientif c Principles and Practice. 5th ed. Philadelphia, PA:
binds to CD20 on B cells, causing depletion; tacroli-
Lippincott Williams & Wilkins; 2011:53141.
mus and sirolimus (a.k.a. rapamycin) both bind FK Racusen LC, Solez K, Colvin RB, et al. T e Ban 97 working
binding protein. Corticosteroids bind a nuclear recep- classi cation o renal allogra pathology. Kidney Interna-
tor, inhibiting NF-B and blocking -cell activation. tional. 1999;55:71323.
Veale JL, Singer JS, Gritsch HA. T e transplant operation and
5. B. Immunosuppression is associated with both its surgical complications. In: Danovitch GM. Handbook o
drug-speci c toxicity as well as susceptibility to in ec- Kidney ransplantation. 5th ed. Philadelphia, PA: Lippin-
cott Williams & Wilkins; 2009:18197.
tions and malignancy. T e most common malignancy
Wilkinson A. T e First Quarter: T e First T ree Months
associated with post-transplant immunosuppression A er ransplantation. In: Danovitch GM. Handbook o
is skin cancer, speci cally squamous cell carcinoma. Kidney ransplantation. 5th ed. Philadelphia, PA: Lippin-
Post-transplant activation o CMV actually occurs cott Williams & Wilkins: 2009:198216.
85
ransplant SurgeryLiver

Joy Sarkar

A 52-year-old patient with a long history o hepatitis C D. Solitary HCC nodule in peripheral lung measur-
(HCV) presents with vague epigastric pain and reports ing 1 cm
weight loss o 10 pounds over the past 4 months with a E. HIV in ection
serum AFP level o 600 mcg/L. Contrast computerized A previously healthy 24-year-old patient is admit-
tomography (C ) scan o the liver shows multiple nod- ted with a two-day history o malaise, nausea, vom-
ules in the hepatic parenchyma. iting, jaundice, and epigastric pain. Over the next
three days, she becomes increasingly con used, then
1. Which of the following is true regarding obtunded, with associated marked elevation in liver
liver transplantation in a patient with chronic enzymes, bilirubin, and ammonia levels. INR rises to
hepatitis C? 7.1 and Cr is 4.3 mg/dL.
A. Preoperative treatment with lamivudine may
decrease the rate o HCV recurrence a er trans- 3. Which of the following is true with regards to liver
plantation. transplantation in this patient?
B. HCV is second to hepatitis B (HBV) as the most A. Alcohol-induced hepatitis is the most likely
common indication or liver transplantation underlying etiology o this patients condition.
worldwide. B. Without transplantation, the mortality rate asso-
C. Up to 30% o patients with active hepatitis C at ciated with this condition approaches 80%.
the time o transplant will experience recurrence C. Emergent transplantation is contraindicated due
o HCV in the transplanted liver. to severe coagulopathy.
D. T e Model or End-Stage Liver Disease (MELD) D. T e MELD score assessment will be used to
score assessment predicts perioperative mortal- determine the patients priority on the transplant
ity o transplant recipients. waiting list.
E. Post-transplantation, progression o HCV in ec- E. One-year survival is higher a er transplantation
tion to cirrhosis is more aggressive than the orig- or ulminant hepatic ailure than or chronic
inal in ection. liver ailure.
T e above patient undergoes orthotopic liver
2. Which of the following is a contraindication to liver transplantation. On the rst post-operative day, the
transplantation in a patient with hepatocellular patient demonstrates no improvement in mental
carcinoma (HCC)? status. Laboratory analysis is notable or acidosis,
A. Portal vein thrombosis marked elevation in liver enzymes, elevated INR,
B. Presence o hepatorenal syndrome and hyperkalemia. Minimal output is noted rom the
C. T ree tumors within hepatic parenchyma mea- -tube biliary drainage catheter.
suring 2.7 cm, 2 cm, and 1.5 cm
C H AP TER 8 5 TRAn s P l An T s u Rg ERy l i v ER 325

4. T e most likely etiology for this clinical presenta- a contraindication to OL because the thrombus
tion is: can be extracted, or a jump gra can be placed to
A. Acute cell-mediated rejection. the superior mesenteric vein (SMV). Patients with
B. Bacterial sepsis. hepatorenal syndrome may experience recovery o
C. Primary non- unction o gra . renal unction ollowing liver transplantation; even
D. Biliary anastomotic leak. in advanced cases the patient may be a candidate or
E. Acute viral hepatitis. combined liver-kidney transplantation. HIV is no
longer considered an absolute contraindication to
5. T e best treatment for the above condition is: OL provided it is well controlled with antiretroviral
A. Increase in tacrolimus dose. therapy, but active sepsis remains a contraindication.
B. High-dose intravenous corticosteroids.
3. B. Fulminant hepatic ailure is de ned as the pres-
C. Inter eron with ribavirin.
ence o encephalopathy within 8 weeks o the devel-
D. ERCP with biliary stenting.
opment o jaundice in the absence o previous liver
E. Re-transplantation.
disease. In the United States and Europe, the most
common cause o ulminant hepatic ailure is aceta-
ANSWERS
minophen overdose. T e Kings College criteria are
1. E. Worldwide, HCV is the most common indication the most widely accepted guideline or transplan-
or liver transplantation. Following liver transplan- tation o patients with ulminant hepatic ailure.
tation, the majority o patients with positive HCV According to the Kings College criteria, a patient
titers will experience recurrence o hepatitis C in the with ulminant hepatic ailure secondary to aceta-
transplanted liver. Pre-operative therapy with inter- minophen overdose quali es or transplantation i
eron and ribavirin is help ul in managing symp- either pH < 7.3, or all three o the ollowing crite-
toms o early HCV in ection, but will only achieve ria are met: grade 34 encephalopathy, P > 100 s or
viral clearance rom serum in a small percentage INR > 6.5 (choice C is alse), or Cr > 3.4 mg/dL. In
o patients. Lamivudine is used in post-transplant non-acetaminophen overdose, the criteria are either
patients with HBV and has been shown to signi - P > 100 s (i.e., INR > 6.5), or any three o the ollow-
cantly improve survival rates and overall outcomes. ing: age < 10 or > 40 y, non-A/non-B/drug-induced/
When hepatitis C recurs in the transplanted liver, Wilson disease hepatitis, greater than 7-day transi-
the progression to cirrhosis is much more aggressive tion rom jaundice to encephalopathy, P > 50 s (i.e.,
than the original in ection, and patients can progress INR > 3.5), or total bilirubin > 17.5 mg/dL. Regard-
to end-stage liver ailure in 6 months. T e MELD less o etiology, the patient in this scenario meets
ormula is calculated using logarithms o the serum Kings College criteria or transplantation based on
creatinine, bilirubin, and INR. T e MELD score pre- her severe encephalopathy, INR, and creatinine level.
dicts the likelihood o death i the patient does not Without transplantation, the mortality rom ul-
receive liver transplantation, but does not correlate minant hepatic ailure approaches 80%. While the
with non-transplant post-operative survival rates. MELD score can be additive to the Kings College
criteria in predicting mortality, and is used to assign
2. D. T e majority o HCC tumors develop in patients priority on the transplant waiting list or patients
with cirrhosis. While liver resection is the treat- with chronic liver ailure, patients with ulminant
ment o choice or HCC in patients without cir- hepatic ailure are listed as status 1A (highest prior-
rhosis, orthotopic liver transplantation (OL ) has ity), which supersedes the MELD score assessment.
evolved as the pre erred treatment or HCC in the Survival a er transplantation or ulminant hepatic
setting o advanced cirrhosis as OL treats both the ailure is lower at 1 year compared to transplantation
tumor and underlying liver dys unction. T e Milan or chronic liver ailure: 73% versus 85%.
criteria guides patient selection or liver transplan-
tation; patients with a solitary tumor up to 5 cm or 4. C. Primary non- unction o the transplanted
three tumors up to 3 cm each are eligible or OL . liver occurs in 2% to 10% o cases and is character-
Metastatic HCC is an absolute contraindication ized by absence o bile production, severe acidosis,
to transplantation. Portal vein thrombosis is not elevation o liver enzymes, hyperkalemia, hepatic
326 g En ERAl s u Rg ERy EXAM i n ATi O n An D BO ARD REv i EW

encephalopathy, and eventually multi-organ ailure. patients with underlying hepatitis C. T ese patients
Possible etiologies are early hepatic arterial throm- should minimize steroids as much as possible to
bosis, prolonged ischemia o the donor liver, poor decrease viral replication, and should instead receive
preservation, advanced donor age, or allogra stea- an increase in their tacrolimus or mycophenolate
tosis. Acute cell-mediated rejection a er liver trans- mo etil dose. Patients with recurrent hepatitis C may
plantation is a less common cause o gra loss than be treated with a combination o inter eron and riba-
primary non- unction or hepatic artery thrombosis. virin. ERCP with biliary stenting is the treatment o
Rejection may be asymptomatic, or may present with choice or a biliary leak or stricture.
mild symptoms mimicking hepatitis. In act, recur-
rent hepatitis C may occur in the early post-operative BIBLIOGRAPHY
period due to the immunosuppressive regimen and be Ascher NL. Liver transplantation. In: ownsend CM, Beau-
dif cult to distinguish rom acute rejection, although champ D, Evers BM, Mattox KL, eds. Sabiston extbook o
a biopsy can be help ul in this regard. Intra-abdominal Surgery: T e Biological Basis o Modern Surgical Practice.
19th ed. Philadelphia, PA: Elsevier; 2012:65565.
sepsis may occur, mani esting with ever and peritoni- Locke JE, Cameron AM. reatment or hepatocellular carci-
tis, and is most o en due to biliary anastomotic leak. noma: Resection versus transplantation. In: Cameron JL,
Biliary leak or stricture is a common post-operative Cameron AM, eds. Current Surgical T erapy. 11th ed. Phil-
complication occurring in 10% to 30% o cases, and adelphia, PA: Elsevier; 2014:3345.
is diagnosed via cholangiography. Roberts MS, Angus DC, Bryce CL, Valenta Z, Weiss eld L.
Survival a er liver transplantation in the United States:
5. E. Patients with primary non- unction o the gra A disease-speci c analysis o the UNOS database. Liver
ranspl. 2004;10(7):88697.
are essentially anhepatic, and a majority require re- Welling H, Pelletier SJ. Hepatic transplantation. In: Mulhol-
transplantation; patients are relisted as Status 1A. land MW, Lillemoe KD, Doherty GM, et al., eds. Greenf elds
High-dose intravenous corticosteroids are the treat- Surgery: Scientif c Principles and Practice. 5th ed. Philadel-
ment or acute cell-mediated rejection, except in phia, PA: Lippincott Williams & Wilkins; 2011:54264.
Vasc ul a r Sur ger y
Dwight C. Kellicut
This page intentionally left blank
86
Carotid Occlusion

ony Katras

A 62-year-old male is re erred to your o ce or evalu- C. Obtain a C angiogram. I the internal carotid is
ation o an asymptomatic le carotid bruit. He has no patent, even i 99% stenotic, per orm an urgent
history o transient ischemic attack ( IA) or stroke. le carotid endarterectomy.
Carotid duplex scanning reveals an occluded right D. Obtain a C angiogram. I the le internal
internal carotid artery, and elevated ow velocities in carotid is indeed occluded, wait 4 weeks and per-
the le internal carotid (peak systolic o 510 cm/sec and orm a le carotid endarterectomy.
end diastolic o 185 cm/sec). Computerized tomogra- E. Obtain a C angiogram. I the le internal
phy (C ) angiography con rms occlusion o the right carotid is indeed occluded, per orm a le extra-
internal carotid artery and > 80% stenosis o the le cranial to intra-cranial bypass.
internal carotid artery. An EKG and a nuclear medicine
cardiac stress test are both normal. 3. T e patient eventually undergoes carotid
endarterectomy (CEA) with prosthetic patch
1. T e best treatment option or this patient is: closure. He is initially awake and alert in the
A. Right carotid endarterectomy recovery room but 1 hour later he develops new
B. Le carotid endarterectomy onset aphasia and right arm weakness. What do
C. Staged bilateral carotid endarterectomy you recommend as the next step?
D. Bilateral carotid stents A. Observation
E. Medical treatment without surgery will give pt B. Systemic anticoagulation administration
the best chance or avoiding a stroke C. Systemic thrombolytic administration
D. Immediate return to the operating room
2. T e patient did not want surgery and opted E. C angiogram
or medical management. One week later he
presents to the emergency room ten hours a er 4. During surgical exposure o the carotid artery
experiencing a le hemispheric IA. He now has the patient becomes severely bradycardic. Only
no neurological def cits. C scanning shows no occasional beats with long pauses are visible on
evidence o cerebral in arction. Duplex scan now the cardiac monitor. Which o the ollowing is the
shows occlusion o both internal carotid arteries. most appropriate next step?
T e best treatment plan is: A. Abort procedure
A. reat the patient with anti-platelet therapy and B. Administer IV atropine
manage non-operatively. C. Electrical cardioversion
B. Fully anti-coagulate the patient with heparin/ D. Administer norepinephrine
Coumadin and manage non-operatively. E. ransect the vagus nerve
330 G EN ERAL S U RG ERY EXAM I N ATIO N AN D BO ARD REVI EW

5. Which o the ollowing is an indication or scans are generally reserved or urther evaluation
emergency carotid endarterectomy? a er con rmation o a patent carotid repair. Most
A. Asymptomatic patient with an 80%99% inter- surgeons would not recommend systemic thrombo-
nal carotid artery stenosis. lytic agents due to the risk o hemorrhage.
B. Severe stroke with an 80%99% stenosis.
4. B. T e carotid sinus, located in the distal common
C. ransient monocular blindness (amaurosis
carotid artery, is a collection o baroreceptor tissue
ugax) with an 80%99% stenosis.
that, when stimulated by increase pressure, produces
D. Crescendo transient ischemic attacks ( IA) with
bradycardia and hypotension. T e carotid sinus is
a 60%79% stenosis.
innervated by the carotid sinus nerve (nerve o Her-
E. IA due to tandem lesions o the carotid bi urca-
ing), which arises rom the glossopharyngeal nerve.
tion and intracranial carotid siphon.
During a carotid endarterectomy, particularly dur-
ing dissection near the bulb or with manipulation
ANSWERS o the plaque, the carotid sinus may be stimulated
1. B. T e correct answer is a carotid endarterectomy leading to bradycardia and even asystole. Some sur-
(CEA) on the side with high grade stenosis. CEA geons routinely inject the tissue near the sinus with
is not indicated on occluded carotid arteries. Even lidocaine. I bradycardia occurs in a bulb that has not
i CEA is possible on total occlusions, the patient been anesthetized, lidocaine can be employed. I the
would be at a higher risk or perioperative stroke due patient experiences severe bradycardia and loses a
to reper usion injury, thus a CEA should not be per- pulse, atropine should be administered immediately
ormed on a chronic occlusion. T e same holds true along with epinephrine as needed.
or carotid stenting. Due to the severity o disease,
5. D. Crescendo IAs represent unstable plaque and
CEA is a better option than medical management
call or an emergent carotid endarterectomy. T e ret-
alone as many randomized clinical trials have shown.
inal in arction rom carotid stenosis is certainly an
2. C. Because it is o en di cult to dif erentiate indication or carotid endarterectomy but not nec-
between occlusion and near occlusion (string sign) essarily on an emergent basis. Asymptomatic high
with carotid duplex scanning, best practice is to con- grade carotid artery stenosis does not require emer-
rm with angiography (usually C A) whether the gent surgery. A er a severe stroke, there is usually an
ICA is patent or not. T us i there is a string sign with interval o recovery be ore endarterectomy should
trickle carotid ow, urgent carotid endarterectomy be considered, the exact length o which remains
is indicated. I the carotid is indeed occluded, most debated. Most now believe that waiting longer than
vascular surgeons would choose antiplatelet therapy 2 to 4 weeks places the patient at an unnecessary
or long term treatment. CEA is not per ormed on risk o recurrent stroke. IAs with a carotid stenosis
occluded ICAs due to increased risk or stroke sec- should be treated aggressively whether it is associ-
ondary to reper usion injury. Extra-cranial to intra- ated with an intracranial stenosis or not, but this is
cranial bypass has no bene t over medical therapy not considered an indication or emergent surgery.
alone. T is procedure is rarely per ormed today, with
BIBLIOGRAPHY
only very ew indications.
Moore WS. Extracranial cerebrovascular disease. In: Moore
3. D. Most surgeons recommend immediate re- WS. Vascular and Endovascular Surgery. 7th ed. Philadel-
operation or patients with new onset neurological phia, PA: Elsevier Saunders; 2006:61758.
Naylor AR. Surgical treatment o carotid disease. In: Hallet
events in the recovery room ollowing CEA. T e JW, Mills JL, Earnshaw JJ, eds. Comprehensive Vascular and
purpose o operative exploration is to correct any Endovascular Surgery. 2nd ed. Philadelphia, PA. Mosby;
surgical de ect and to restore blood ow as soon as 2009:60629.
possible i thrombotic occlusion is ound. Intraop- Pine R, Avellone JC, Hof man M, Plecha FR, Swayngim DM,
erative angiography is pre erred by many surgeons Urban J. American Journal of Surgery. June 1984;147(6):
7635.
either be ore or a er surgical re-exploration. Intraop-
Ricotta JJ, AbuRhama A, Ascher E, Escandan M, Faries P, Lai
erative duplex ultrasound may also be used to exam- BK. Updated society or vascular surgery guidelines or
ine the operative site or abnormal ow or intimal management o extracranial carotid disease. Journal of
aps. Magnetic resonance imaging (MRI) and/or C Vascular Surgery. September 2011;54(3):e131.
87
Abdominal Aortic Aneurysm

Dwight C. Kellicut

A 63-year-old male with past medical history signi cant 4. Regarding survival af er endovascular repair
or well controlled hypertension, hyperlipidemia, and a (EVAR), compared with open repair, which o the
remote history o smoking is re erred or evaluation o ollowing is true?
a 5.7 cm in rarenal abdominal aortic aneurysm (AAA) A. EVAR demonstrates equal survival both in the
ound incidentally on a C scan o the abdomen during rst 30 days and long term.
a recent emergency room visit or abdominal pain. B. EVAR demonstrates better long term survival
T e patient recently underwent a cardiac stress test and equal 30 day survival.
by his primary care physician, which was normal and C. EVAR demonstrates better 30 day survival and
is now being considered or open versus endovascular no di erence in long term survival.
repair. D. In patients with minimal, well-controlled comor-
bidities, EVAR provides the better long-term
1. Which o the ollowing is considered a major risk
results.
actor in the development o AAA?
E. In patients with poorly controlled co-morbidities,
A. Diabetes mellitus the open repair has a 15% mortality rate.
B. Hypertension
C. Smoking
D. Collagen vascular disease ANSWERS
E. Obesity 1. C. T e major risk actors associated with increased
likelihood o aneurysm ormation include: age > 65,
2. For patients with major risk actors, what would be
male gender, history o smoking, and amily history
the best method to screen or AAA?
o AAA in a rst degree relative.
A. Physical exam
B. Abdominal ultrasound 2. B. For patients with major risk actors, the ques-
C. C scan tion becomes what is the best methodology to detect
D. MRI aneurysms in regard to screening? In the U.S. Preven-
E. Angiography tative Services ask Force (USPS F) January 2014
bulletin, the ollowing recommendations were made:
3. Regarding the size o his aneurysm, when should an the ask Force ound that one-time AAA screening
elective abdominal aortic aneurysm be repaired? can be e ective and recommends it or men ages
A. 4.5 cm 65 to 75 who have ever smoked. T is is a B recom-
B. 5 cm mendation. For men ages 65 to 75 who have never
C. 5.5 cm smoked, the ask Force recommends that these men
D. 6.0 cm talk to their doctor or nurse about whether one-time
332 G EN ERAL S U RG ERY EXAM IN ATI O N AN D BO ARD REVIEW

AAA screening might be right or them based on symptoms (back/abdominal pain/tenderness on


their health history and the potential bene ts and exam), and patients located in a remote area.
harms o screening.
For women, the ask Force ound that the ben- 4. C. As a general rule, patients may be considered or
e ts and harms o screening are di erent. In the dra open aortic surgery i they are under the age o 65,
recommendation statement, the ask Force calls or provided they are an acceptable surgical risk. EVAR
more research to determine i AAA screening is ben- may also be considered, but multiple anatomic con-
e cial or women ages 65 to 75 who smoke or have siderations must be taken into account to include:
smoked in the past. Based on the lack o evidence, the neck diameter, neck length, angulation o neck,
ask Force determined it could not recommend or reverse taper o neck, calci cation, thrombus, diam-
or against screening older emale smokers and issued eter o common emoral/iliac arteries, distal seal
an I statement (issued when evidence is insu cient length, iliac tortuosity, and renal/splanchnic blood
to ully assess bene ts and harms). Research is criti- f ow. Given all o the technical considerations associ-
cally needed in this area to determine i AAA screen- ated with EVAR, there are signi cant immediate and
ing could be bene cial or women who smoke or late complications that include device ailure, arterial
who have ever smoked. Among nonsmoking women, embolization, limb thrombosis, gra migration, gra
the chance o developing AAA is extremely low in ection, and endoleaks.
(well under 1%), and the ask Force ound that AAA wo trials (EVAR I and DREAM) have com-
screening is very unlikely to bene t these women pared open aortic surgery versus EVAR in patients
and may even cause harm. T e ask Force recom- considered t or open surgery. In both studies
mends against screening or AAA in these women; (EVAR I/DREAM), open surgical repair was ound
this is a D recommendation. to have increased 30 day mortality (4.7%/4.6% vs.
Physical examination is highly dependent upon 1.7%/1.2%), increased blood loss, longer operative
the experience o the examiner and on patient body time, and increased blood trans usion. T e long term
habitus. As a general rule, patients with a 5.0 cm AAA survival between open surgery and EVAR have been
are palpable 75% o the time and those measuring shown to be equal. EVAR demonstrates signi cant
3.0 to 3.9 cm are palpable 25% o the time. Abdomi- gra related complications (41% vs. 9%) and higher
nal ultrasound has been evaluated in numerous revision rates (20% vs. 6%) making open surgery in
studies and has been ound to be available, accurate, this case a better long term option. T e EVAR 2 trial
and low cost. C and MRI are excellent modalities ound patients deemed un t, de ned somewhat neb-
or evaluating the aorta regarding accuracy, but ail ulously, or open surgery had the substantial opera-
on cost e ectiveness and availability. Angiography tive mortality o 7%, indicating that those patients
actually underestimates the size o a AAA because it that are high risk will remain high risk regardless o
only outlines the arterial lumen. It is also not cost- what procedure is chosen. O note, both EVAR 1 and
e ective as a screening tool. DREAM excluded emergent cases.

3. C. T e natural history o AAA is such that most BIBLIOGRAPHY


aneurysms expand at a rate o 0.3 cm/year. wo ran- Abdominal Aortic Aneurysm: Screening. U.S. Preventative
domized, prospective trials have demonstrated that Services ask Force Web site. http://www.uspreventiveser-
there is no survival bene t to repairing AAA < 5.5 cm, vicestask orce.org/uspst 14/abdoman/abdomandra trec.
Released June 2014. Accessed May 15, 2015.
even in well selected patients who are considered to
Endovascular Aneurysm rial Participants. Comparison o
have a avorable operative risk. All aneurysms greater endovascular aneurysm repair with open repair in patients
than 4.0 cm should be ollowed on a yearly basis. with abdominal aortic aneurysm (EVAR I rial): Random-
For patients who have an aneurysm that measures ized controlled trial. Lancet. 2004;365:1179.
greater than 4.0 cm, but less than 5.5 cm, their aneu- Endovascular Aneurysm rial Participants. Endovascular
rysm should be ollowed every six months as approx- Aneurysm Repair and outcome in patients un t or open
repair o abdominal aortic aneurysm (EVAR trial 2): Ran-
imately 25% o those aneurysms will expand at a rate domized controlled trial. Lancet. 2005;365:9478.
greater than or equal to 0.5 cm in six months, an indi- Fink HA, Lederle FA, Roth CS, et al. T e accuracy o physical
cation or repair. Other indications to repair an AAA examination to detect abdominal aortic aneurysm. Arch
less than 5.5 cm include embolization, occlusion, Int Med. 2000;160:8336.
C H AP TER 8 7 ABD O M I N AL AO RTI C AN EU RYS M 333

Fleming C, Whitlock EP, Beil , Lederle FA. Screening or Lederle FA, Johnson GR, Wilson SE, et al., or the Aneurysm
abdominal aortic aneurysm: A systematic review and Detection and Management (ADAM) Veterans A airs
meta-analysis or the U.S. Preventative Services ask Force. Cooperative Study Investigators. Relationship o age, gen-
Ann Intern Med. 2005;142:20311. der, race, and body size on in rarenal aortic diameter. J Vasc
Kim LG, Scott RAP, Ashton HA, T ompson SG. A sustained Surg. 1997;26:595601.
mortality bene t rom screening or abdominal aortic Nevitt MP, Ballard DJ, Hallett JW. Prognosis o abdominal
aneurysm. Ann Intern Med. 2007;146(10):699706. aortic aneurysms: A population based study. N Engl J Med.
Lederle FA, Johnson GR, Wilson SE, et al., or the Aneu- 1989;321:100914.
rysm Detection and Management Study screening pro- Prinssen M, Verhoeven ELG, Buth J, et al. A randomized trial
gram: Validation cohort and nal results. Arch Int Med. comparing conventional and endovascular repair o aortic
2000;160:142530. aneurysms. N Engl J Med. 2004;351:1607.
Lederle FA, Wilson SE, Johnson GR, et al., or the Aneurysm U.K. Small Aneurysm rial Participants. Mortality results or
Detection and Management (ADAM) Veterans A airs randomized controlled trial o early elective surgery or
Cooperative Study Investigators. Immediate repair com- ultrasonographic surveillance or small abdominal aortic
pared with surveillance o small abdominal aortic aneu- aneurysms. Lancet. 1998;352:164955.
rysms. N Engl Med. 2002;346:11437444.
88
Vascular Access

Booker . King

1. A 65-year-old male with end-stage renal disease C. Placement o a long term central venous dialysis
has recently had an autogenous right radial- catheter (Permacath).
cephalic arteriovenous (AV) stula per ormed. T e D. Placement o a emoral-saphenous AV stula in
patient will require temporary dialysis access until the le groin.
the stula matures. T e best option or temporary E. rans er the dialysis catheter to the ipsilateral
access is: internal jugular vein and placement o a radial-
A. A cu ed, tunneled central venous dialysis cath- cephalic AV stula in the contralateral arm.
eter in the ipsilateral internal jugular vein.
B. A cu ed, tunneled central venous dialysis 3. A 67-year-old male with end stage renal disease
catheter in the contralateral subclavian vein. returns to your vascular clinic six weeks af er he
C. A cu ed, tunneled central venous dialysis cath- had a lef autogenous radial-basilic AV stula
eter in the contralateral internal jugular vein. per ormed. He complains o intractable pain in his
D. An uncu ed central venous dialysis catheter in lef hand and that hand is colder. He also has dark,
the right emoral vein. dry ulcers o the distal end o ngers 2 to 4. What
E. An uncu ed central venous dialysis catheter in is the most likely cause o this condition?
the contralateral internal jugular vein. A. Emboli that has travelled rom plaque that has
ormed in the radial artery.
2. You are evaluating a patient or an autogenous B. Arterial ow is being shunted away rom the
AV stula or long-term dialysis access. T e radial artery into the venous out ow o the AV
patient has no Doppler signal in the right palmar stula.
arch and urther work-up con rms stenosis o C. An air embolus inadvertently ormed in the
the ulnar artery on that side. T e patient has a radial artery rom air trapped in the dialysis cir-
lef subclavian central venous dialysis catheter cuit that has travelled to the arterial system o
currently. Vascular evaluation o the lef arm the hand.
and hand was unremarkable. What is the best D. An unrecognized arterial stenosis in the radial
option or long term dialysis access in this artery that has become symptomatic.
patient? E. Intermittent vasospasm o the radial artery
A. Placement o a radial-cephalic AV stula in the brought on by hemodialysis.
contralateral arm.
B. Placement o a brachial-basilic AV stula in the 4. T e most appropriate treatment or the condition
ipsilateral upper arm as the more proximal loca- in question # 3 is:
tion will not be a ected by the ulnar artery ste- A. Angioplasty and possible stenting o any stenosis
nosis. in the arterial limb o the AV stula.
C H AP TER 8 8 VAs C u l AR ACC Es s 335

B. Intra-arterial injection o papaverin into the AV dialysis catheter placed in the contralateral internal
stula and a three month course o an oral cal- jugular vein. Catheter placement in the emoral vein
cium channel blocker post-procedure. is not ideal especially in patients who are ambulatory.
C. Ligate the radial artery distal to the AV stula. Femoral venous catheters have been associated with
D. T erapeutic heparinization and then War arin higher rates o venous thrombosis and blood stream
therapy or 3 months or treatment o embolic in ections in several reports.
disease.
E. Reassure the patient that the condition is sel - 2. E. AV stula placement anywhere in the ipsilateral
limiting and will improve over the next ew arm is not prudent given the stenosis o the ulnar
weeks. artery on that side. T is option has a high risk o gra
ailure. Femoral-saphenous AV stula is an in erior
5. A 50-year-old male presents with ever to 102.5F option due to increased risk o gra thrombosis and
and malaise two months af er placement o a in ection in this location. AV stula placement in the
prosthetic AV stula in his right orearm. T e contralateral arm is the best option but the tempo-
patient has erythema around the AV stula rary dialysis should be trans erred to the right inter-
incision site and the AV stula is warm and pain ul nal jugular vein to maximize long term patency.
upon palpation. Blood cultures were obtained
and preliminary Grams stain show large amount 3. B. T e arterial insuf ciency that is seen in this
o Gram positive cocci in clusters. A course o patient is an example o ischemic steal syndrome.
intravenous Vancomycin and Ce epime was Arterial ow is being diverted away rom the hand
started. What is the next step in the management into the venous out ow (in this case the basilic vein).
o this patient? T is phenomenon occurs in all AV stulas to some
A. Schedule surgery urgently or the removal o degree but in rare cases the arterial insuf ciency is
prosthetic material rom the right orearm. to such a degree that the condition becomes symp-
B. Continue intravenous Vancomycin or a our tomatic. Patients usually present with severe pain in
week course in an attempt to sterilize the AV the hand (or oot in the case o emoral-saphenous
stula. AV stulas). T is can progress to ischemic ulcers
C. Inject Vancomycin and Ce epime into the AV orming on the distal extremity. T e diagnosis can be
stula directly and place a tourniquet above the con rmed by Doppler imaging or venograpy.
stula or 30 minutes.
D. Start an eight day course o Vancomycin and 4. A. Be ore ligating the radial artery, standard o care
Ce epime and then repeat blood cultures. No is or an arteriogram o the in ow artery with pos-
additional therapy is needed i blood cultures are sible balloon angioplasty and/or stenting. I treat-
negative. ment o the arterial in ow with angioplasty and/or
E. Surgically remove prosthetic material rom the stenting does not correct the problem, then either a
right orearm and placement o an autogenous distal revascularization and interval ligation (DRIL)
ipsilateral brachial-basilic AV stula concur- procedure or Proximalization procedure would be
rently during this operation. indicated. Ligation o the radial artery should only
be considered a last resort. Ligation will allow arte-
rial ow to be restored to the palmar arch and hand.
ANSWERS
Alternatively, or more proximal upper extremity
1. C. Cu ed, tunneled dialysis catheter can be main- and lower extremity AV stulas, distal revasculariza-
tained or 6 weeks or more while uncu ed catheters tion with interval ligation or DRIL procedure can be
will need to be replaced at 3 weeks. An autogenous per ormed. In this procedure, a conduit (usually the
AV stula will usually take 4 to 6 weeks to mature. saphenous vein gra ) is created rom the proximal
Catheters placed in the ipsilateral internal jugu- portion o the arterial limb o the AV stula to an
lar vein or either subclavian vein are associated area on the artery distal to the stula. A erwards, the
with higher rates o AV stula ailure due to par- artery is ligated between the stula and the bypass
tial venous out ow obstruction. T ere is negligible gra . T is procedure will usually allow preservation
risk o venous out ow obstruction with temporary o the AV stula as well as treat the arterial steal. I
336 G EN ERAl s u RG ERY EXAM I N ATIO N AN D BO ARD REVI EW

these measures ail to correct the problem, the AV o dialysis access in ischemic steal syndrome. J Vasc Surg.
stula will need to be ligated. Banding or plication o 1997;26(3):393404.
the venous out ow limb are less invasive procedures Jennings WC, Kindred MG, Broughan A. Creating radioce-
phalic arteriovenous stulas: echnical and unctional suc-
that have been used to treat this condition. cess. J Am Coll Surg. 2009;208:41925.
Joynt GM, Kew J, Gomersall CD, et al. Deep venous thrombo-
5. A. T e patient in question #5 has an in ected AV s- sis caused by emoral venous catheters in critically ill adult
tula gra . T is patient also has bacteremia as result patients. Chest. 2000;117:17883.
o the gra in ection and is at risk to develop severe La rance JP, Rahme E, Lelorier J, et al. Vascular access-related
sepsis which can lead to death i not treated appro- in ections: De nitions, incidence, and risk actors. Am J
priately and expeditiously. Antibiotic therapy should Kidney Dis. 2008;52(5):98293.
Malik J, uka V, Kasalova Z, Chytilova E, Slavikova M, Clagett
be introduced immediately and empiric broad spec- P, et al. Understanding the dialysis access steal syn-
trum antibiotics are advised since the patient has drome. A review o the etiologies, diagnosis, prevention
likely been exposed to resistant hospital pathogens. In and treatment strategies. J Vasc Access. JulySeptember
this case methicillin-resistant Staphylococcus Aureus 2008;9(3):15566.
(MRSA) is the most likely pathogen and Vancomy- Merrer J, De Jonghe B, Golliot F, et al. Complications o
emoral and subclavian venous catheterization in criti-
cin is directed against it. T e prosthetic material in
cally ill patients: a randomized controlled trial. JAMA.
the gra is the source o the in ection and there ore 2001;286(6):700707.
must be removed. Any attempt to eradicate the gra Schwab SJ, Beathard G. T e hemodialysis catheter conun-
in ection with antibiotic therapy alone will ail. T e drum: Hate to live with them but cant live without them.
placement o a new autogenous AV stula should be Kidney Int. 1999;56:117.
postponed until the in ection is adequately treated. ashjian DB, Lipkowitz GS, Madden RL, et al. Sa ety and ef -
cacy o emoral- based hemodialysis access gra s. J Vasc
Surg. 2002;35:6913.
BIBLIOGRAPHY Yaghoubian A, de Virgilio C. Plication as a primary treatment
Berman SS, Gentile A , Glickman MH et al. Distal revascular- o steal syndrome in arteriovenous stulas. Ann Vasc Surg.
ization: Interval ligation or limb salvage and maintenance 2009;23(1):1037.
89
Complications o Hemodialysis

Alexander Malloy

A 66-year-old male with end-stage renal disease, coro- D. Both A and C


nary artery disease, diabetes, and hypertension presents E. All o the above
to his outpatient clinic or regularly scheduled hemodi-
alysis. He has a unctioning le brachiocephalic arterio- 4. Another patient who undergoes peritoneal dialysis
venous stula that his nurse accesses without di culty. or end-stage renal disease develops peritonitis.
His dry weight is 80 kg. T e patient is otherwise hemodynamically normal,
without volume overload or signif cant electrolyte
1. A er initiating hemodialysis, the patient begins abnormality. What is the recommended treatment?
to complain o muscle cramps in his legs. Which A. Laparotomy and peritoneal washout
o the ollowing is an e ective treatment or this B. Peritoneal dialysis catheter removal
condition? C. Empiric gram negative and gram positive cover-
A. Having the patient drink water during hemodi- age with intraperitoneal and oral antibiotics
alysis D. Empiric gram negative, gram positive, and anti-
B. Hypertonic saline ungal intravenous coverage
C. A low-sodium dialysate E. Emergent temporary dialysis catheter placement
D. Increasing the ultra ltration rate and initiation o hemodialysis
E. Permissive hypotension
5. T is patients peritonitis does not improve, and
2. T e patient continues his hemodialysis. He yeast is isolated on the gram stain. What is the
becomes acutely hypotensive. Which o the recommended management o this patient?
ollowing could be a cause o hypotension? A. Peritoneal lavage, intraperitoneal uconazole,
A. Fasting in the peri-hemodialysis period and catheter removal
B. Increased sympathetic tone B. Catheter removal and intravenous uconazole
C. Hemolysis C. Intraperitoneal amphotericin B ollowed by
D. Low ultra ltration rate catheter removal
E. Cooling o the dialysate solution D. Peritoneal lavage ollowed by catheter removal
E. Catheter removal and immediate hemodialysis
3. T e patients hypotension has resolved, but he is
now experiencing dyspnea and chest pain. What is
the appropriate management or this patient? ANSWERS
A. Electrocardiogram and serum cardiac enzymes 1. B. Dialysis-associated muscle cramps is a common
B. Continuation o dialysis problem that af ects up to 86% o patients. T e true
C. Reduction o ultra ltration rate etiology o the muscle cramps is not well understood,
338 G EN ERAL S U RG ERY EXAM IN ATIO N AN D BO ARD REVI EW

but is likely related to electrolyte and uid uctua- ully understood but lower temperatures are thought
tions. T ese reactions can occur when a low-sodium to increase sympathetic activity, thereby maintaining
dialysate is used resulting in hyponatremia or hypo- adequate systemic vascular resistance (Choice E).
osmolar plasma states. T is can be avoided by using
a dialysate with a higher concentration o sodium. 3. E. T is patient has coronary artery disease so angina
Cramps o en occur due to rapid increase o inter- or myocardial in arction should be ruled out but all
dialytic weight gains leading to intravascular volume o the above treatments are appropriate. Cardiac
contraction and hypo-osmolality as a result o high enzymes and electrocardiograms should be obtained
ultra ltration rates. Hypertonic saline or hypertonic on an individual basis based on clinical scenario
dextrose in a non-diabetic patient have been shown as and this patient meets this criteria. Reduction in
ef ective treatments to counteract the hypo-osmolar ultra ltration will reverse a hypovolemic state, thus
state o the intravascular uid, and also be help ul in reducing myocardial demand ischemia (Choice C).
hypovolemic states that may precipitate cramping or Continuation or cessation o dialysis (Choice B) are
other serious reactions. Drinking water would not both acceptable, but are dependent on clinical actors
be ef ective in reversing a hypo-osmolar state and such a hemodynamic stability, physical ndings, and
would not alleviate muscle cramps. degree o patient com ort.

2. C. Intradialytic hypotension is one o the most com- 4. C. Peritonitis is a common complication o perito-
mon complications o hemodialysis, occurring in up neal dialysis, and is the primary reason or cessation
to 30% o patients. T ere are many causes o hypo- o peritoneal dialysis with transition to hemodialy-
tension in the dialysis patient that can lead to li e- sis. Peritonitis in patients who undergo peritoneal
threatening scenarios such as arrhythmia, bleeding, dialysis has signi cant morbidity but are managed
in ection, and myocardial in arction, among others. with empiric outpatient antibiotics in over 75% o
Hypotension can occur rom aggressive ultra ltra- cases. Current guidelines state that empiric gram
tion or hypo-osmolar dialysate resulting in weight negative and gram positive coverage with antibiot-
gain. Low ultra ltration rates do not typically cause ics are indicated. Outpatient antibiotics typically
hypotension (Choice D). Ingestion o a large meal include intraperitoneal and oral administration,
immediately prior to or during dialysis can direct such as intraperitoneal ce azolin and oral cipro oxa-
blood ow to splanchnic circulation and result in cin, but inpatient regimens can include intravenous,
peripheral hypotension; as a result, eating in the intraperitoneal, or oral modalities (Choice C). Mon-
peri-dialysis period is not recommended (Choice otherapy is also available in intravenous orm with
A). Autonomic dys unction and neuropathy related imipenem/cilastatin. Empiric anti ungal coverage
to long-standing diabetes mellitus o en results in is not recommended due to emerging resistance to
hemodynamic instability during dialysis, which has anti ungals (Choice D) and a large review revealed
been reported in up to 50% o patients (Choice B). that only 2% o peritoneal dialysis-associated perito-
Hemolysis can occur i the dialysate solution is con- nitis were rom ungal elements.
taminated with bacteria or other minerals, as well Laparotomy would not be recommended even i
as i the solution is hypotonic. I hemolysis occurs, the patient were hemodynamically unstable, unless
the pathognomonic sign is a port-wine appearance there was another precipitating cause o peritonitis
o blood in venous lines (Choice C). Hemolysis is such as bowel per oration or hemorrhage (Choice A).
usually accompanied by other symptoms such as Peritoneal dialysis catheter removal is not a rst line
nonspeci c chest pain and dyspnea. Hemolysis can treatment or microbial peritonitis. Indications or
lead to hyperkalemia and arrhythmia i not properly catheter removal are relapsing peritonitis, lack o
identi ed and treated. Hypotension can be precipi- antimicrobial response within ve days o initiation,
tated by warm dialysate and cooling o the solu- or tunnel or insertion sites that involve in ection.
tion is known to prevent such a complication. Fixed Placement o a temporary dialysis catheter and initi-
reduction o body temperature by 1C has shown a ation o hemodialysis is not indicated in this patient
statistically signi cant reduction in systolic blood as he is hemodynamically normal without any gross
pressure in several studies. T e mechanism is not electrolyte disturbance (Choice E).
C H AP TER 8 9 CO M P LI C ATIO N S O f H EM O D IALYS I S 339

5. A. I ungal elements are isolated on gram stain Hernando P, Caramelo C, Lpez Garcia D, Hernando L. Mus-
or culture, prompt removal o the dialysis catheter cle cramps: A cause o elevated creatine kinase levels in
and administration o intraperitoneal or intrave- hemodialysis patients. Nephron. 1990;55(2):231.
Li PK, Szeto CC, Piraino B, Bernardini J, Figueiredo AE, Gupta
nous anti ungals should be initiated. Intraperitoneal A, et al. Peritoneal dialysis-related in ections recommenda-
amphotericin B is generally not recommended as it tions: 2010 update. Perit Dial Int. 2010;30(4):393.
causes additional pain to the patient and is known Port FK, Held PJ, Nolph KD, urenne MN, Wol e RA. Risk
or developing signi cant intraperitoneal adhesions. o peritonitis and technique ailure by CAPD connection
ypically the peritoneal uid has a grossly cloudy technique: a national study. Kidney Int. 1992;42(4):967.
Sands JJ, Usvyat LA, Sullivan , Segal JH, Zabetakis P, Kotanko
appearance and should be lavaged with normal saline
P, et al. Intradialytic hypotension: Frequency, sources o
until clear. T e patient should undergo hemodialysis variation and correlation with clinical outcome. Hemodial
in the near uture or a minimum o six weeks. Imme- Int. April 2014;18(2):41522. Epub January 27, 2014.
diate hemodialysis is not necessary unless clinically Selby NM, McIntyre CW. A systematic review o the clinical
indicated via signs o acidosis, electrolyte imbalance, ef ects o reducing dialysate uid temperature. Nephrol
uremia, or uid overload. A er the six week period, Dial Transplant. 2006;21(7):1883.
Sherman RA, Goodling KA, Eisinger RP. Acute therapy o
a new peritoneal catheter can be placed i the patient hemodialysis-related muscle cramps. Am J Kidney Dis.
desires. 1982;2(2):287.
Sherman RA, Daugirdas J , Ing S. Complications during
BIBLIOGRAPHY hemodialysis. In: Daugirdas J , Ing S. eds. Handbook of
Barakat MM, Nawab ZM, Yu AW, Lau AH, Ing S, Daugir- Dialysis. 4th ed. Philadephia, PA: Lippincott Williams and
das J . Hemodynamic ef ects o intradialytic ood inges- Wilkins; 2007:170.
tion and the ef ects o caf eine. J Am Soc Nephrol. May Stablein DM, Nolph KD, Lindblad AS. iming and charac-
1993;3(11):18138. teristics o multiple peritonitis episodes: A report o the
Canzanello VJ, Burkart JM. Hemodialysis-associated muscle National CAPD Registry. Am J Kidney Dis. 1989;14(1):44.
cramps. Semin Dial. 1992;5:299. Wang AY, Yu AW, Li PK, Lam PK, Leung CB, Lai KN, et al. Fac-
Ewing DJ, Winney R. Autonomic unction in patients with tors predicting outcome o ungal peritonitis in peritoneal
chronic renal ailure on intermittent haemodialysis. Neph- dialysis: Analysis o a 9-year experience o ungal peritoni-
ron. 1975;15(6):424. tis in a single center. Am J Kidney Dis. 2000;36(6):1183.
90
Lymphatic Disease

Robert C. McMurray

A 65-year-old man presents to your clinic with com- 1. T is patient is most likely su ering rom which
plaints o increasing right lower extremity heaviness disease process?
and swelling over the last 2 to 3 months. He states his A. Chronic venous insu ciency
symptoms worsen throughout the day and his shoes eel B. Filariasis
tight by the time he takes them o at night. He denies C. Nephrotic syndrome
any skin changes, wounds, or recent trauma to the area. D. Lymphedema
He also denies any evers, chills, atigue, weight loss, or
weight gain. His past medical history is only signi cant 2. T e most common cause o secondary lymphedema
or hypertension, obesity, and a recent history o mela- worldwide is:
noma o the right lower extremity or which he under- A. Malignancy or a result o its treatment
went resection o the lesion with regional lymph node B. Lymphedema praecox
dissection and has been recovering well. His social his- C. Wuchereria bancro i
tory and amily history are unremarkable and he denies D. Lymphedma tarda
any recent travel.
He has never smoked and denies any alcohol use. He 3. T e best test or diagnosing this patient with
does not exercise regularly and takes an ACE inhibi- suspected lymphedema is:
tor and a multivitamin daily. His vital signs are a tem- A. Lymphangiography
perature o 38C, heart rate o 72 beats per minute, B. Computerized tomography (C ) scan
blood pressure o 135/74 mm Hg, respiratory rate o 14 C. Magnetic resonance imaging (MRI)
breaths per minute, and oxygen saturation o 99% on D. Detailed history and physical exam
room air. Physical examination demonstrates di use E. Lymphoscintigraphy
non-pain ul pitting edema over his right lower extrem-
ity that extends over the dorsum o his oot including 4. T e f rst line treatment o lymphedema consists
his toes. T ere is no evidence o skin discoloration or o:
open wounds. He has easily palpable pulses and good A. Diuretics
capillary re ll. His right lower extremity measures 4 cm B. Lymphatic reconstruction or excision o diseased
greater in circum erence than his le lower extremity. channels
T ere are no palpable masses in his groin. His other C. Compressive therapy
extremities are normal in appearance and the rest o his D. Benzopyrones (Coumarin) and long-term anti-
physical exam is unremarkable. biotics
C H AP TER 9 0 Ly m P H ATi C D i s EAs E 341

5. You diagnose this patient with lymphedema and seen in women with a ratio o 10:1 compared to men.
start him on the appropriate therapy. He returns to Lymphedema tarda is an uncommon cause o pri-
your clinic a ew years later with a pain ul, purplish mary lymphedema with an age o onset o 35 years
ulcerating macule on his right lower extremity. old or greater and accounts or less than 10% o cases.
T is presentation is most concerning or:
3. D. Diagnosis o lymphedema is relatively simple
A. Lymphangitis
in the second and third stages o the disease. When
B. Lymphangiosarcoma
lymphedema is in the rst stage (i.e., pitting, mild,
C. Hematoma
and relieved by elevation) diagnosis can be more di -
D. T rombophlebitis
cult. In patients with suspected secondary orms
o lymphedema, C scan and MRI are use ul in
excluding an underlying malignant etiology. In most
ANSWERS
patients, the diagnosis o lymphedema can be made
1. D. T is patient is most likely su ering rom based on a thorough history and physical exam
lymphedema. He is su ering rom unilateral lower alone. Also, patients with previous known lymph
extremity edema, which excludes systemic etiolo- node excision, such as the patient in the vignette,
gies o his edema, such as nephrotic syndrome, liver do not require additional studies or diagnosis o
disease, and heart ailure. Filariasis is the most com- lymphedema ( urther imaging is only necessary as
mon cause o secondary lymphedema worldwide, needed or ollow-up or each underlying malig-
although this patient denies any travel history. nancy). In cases where the diagnosis o lymphedema
Venous pathology, such as chronic venous insu - is suspected but the etiology is unclear, lymphoscin-
ciency is by ar the most common cause o unilat- tigraphy is the diagnostic test o choice.
eral lower extremity edema. Lower extremity edema Lymphoscintigraphy has a sensitivity around 70%
caused by venous pathology typically presents with to 90% and a speci city o nearly 100%. Lymphangi-
pitting edema that spares the eet. T e prolonged ography provides the best detail o the lymphatic sys-
venous disease the skin can become atrophic with tem, but due to complications such as lymphangitis,
hemosiderin pigmentation. pulmonary embolism, allergic reaction to the con-
T is patient has a history o lower extremity mela- trast dye and urther damage to the lymphatic ves-
noma with regional lymph node dissection, which is sels, it is reserved or patients being considered or
a risk actor or developing lymphedema. Other risk lymphatic reconstruction.
actors include increased body mass index, tumor
location, post-operative in ection or hematoma, 4. C. T ere is no curative treatment or lymphedema.
and radiation therapy. Additional malignancies that T e goals o treatment are to minimize swelling and
have been associated with the development o lym- prevent in ections to the a ected limb. Patients are
phadema are sarcomas, gynecologic cancer, geni- at increased risk o developing recurrent in ections
tourinary cancer and head and neck cancer, but the such as cellulitis, erysipelas and lymphangitis. A
most common malignancy to be associated with the combination o physical therapies (CP ) is the pri-
development o lymphedema is breast cancer, with mary approach to the management o lymphedema.
an incidence o 17% among survivors. CP involves good skin care ollowed by manual
lymphatic drainage through massage, bed rest with
2. C. T e most common cause o secondary leg elevation, application o graded compression
lymphedema worldwide is Filariasis rom in ec- stockings and sequential pneumatic compression
tion by the nematode Wuchereria banchro i. In devices. When worn daily, compression stockings
developed nations, almost all cases o secondary have shown long-term maintenance o reduced limb
lymphedema are caused by a sequela o malignancy circum erence, may protect against external trauma
or as a result o its treatment. Lymphedema praecox is and the development o skin and subcutaneous
the most common orm o primary lymphedema and tissue thickening.
accounts or around 94% o cases. T e onset is usually No drug therapy or the treatment o lymphedema
seen in children and teenagers and is predominantly has been shown to be e ective. Diuretics can be
342 G EN ERAL s U RG ERy EXAm i N ATi O N AN D BO ARD REVi EW

use ul in an acute exacerbation secondary to in ec- BIBLIOGRAPHY


tion or i seen with coexisting venous disease, but is Burnand KG, McGuinness CL, lagattolla NR, et al. Value o
not use ul or long-term care o lymphedema. Cou- isotope lymphography in the diagnosis o lymphedema o
marin is still under investigation as a potential treat- the leg. BR J Surg. 2002;89:748.
ment or lymphedema, but currently has no role in Cambria RA, Gloviczki P, Naessens JM, et al. Noninvasive
evaluation o the lymphatic system with lymphoscintig-
treatment in the United States. raphy: A prospective semiquantitative analysis in 386
Although patients with lymphedema are at extremities. J Vasc Surg. 1993;18:77382.
increased risk o developing recurrent in ections, Cormier JN, Askew RL, Mungovan KS, et al. Lymphedema
prophylactic use o antibiotics has no role in treat- beyond breast cancer: a systematic review and meta-
ment. Instead, patients should be prescribed antibi- analysis o cancer-related secondary lymphedema. Cancer.
2010;116:5138.
otics that they can keep with them and take at the
Disipio , Rye S, Newman B, Hayes S. Incidence o unilateral
rst signs or symptoms o an in ection. arm lymphoedema a er breast cancer: A systemic review
Patients with lymphedema can be managed non and meta-analysis. Lancet Oncol. 2013;14:500.
operatively 95% o the time. Operative intervention Freischlag JA, Heller JA. Venous disease. In: Beauchamp RD,
may be considered or patients with Stage II or Stage Evers BM, Mattox KL, et al. Sabiston extbook of Surgery:
III disease who have severe unctional disability. Sur- T e Biological Basis of Modern Surgical Practice. 19th ed.
Philadelphia, PA: Elsevier Saunders; 2008: 2017.
gical intervention is made up o two categories: exci- Liem K, Moneta GL. Venous and lymphatic disease. In: Bru-
sional or reconstructive. Long-term ollow-up data nicardi FC, Andersen DK, Billiar R, et al. Schwartzs Prin-
or surgical care o lymphedema is not available cur- ciples of Surgery. 9th ed. New York City, NY: McGraw-Hill
rently, and is there ore not well accepted as a main- Pro essional; 2009:147.
stay o treatment throughout the world. Marotel M, Cluzan R, Ghabboun S, et al. ransaxial computer
tomography o lower extremity lymphedema. Lymphology.
5. B. T is clinical presentation is most concerning or 1998;31:1805.
Pipinos II, Baxter B . T e Lymphatics. In: Beauchamp RD,
lymphangiosarcoma. Lymphangiosarcoma is a rare Evers BM, Mattox KL, et al., eds. Sabiston extbook of Sur-
malignant tumor that can occur in patients su ering gery: T e Biological Basis of Modern Surgical Practice. 19th
rom chronic lymphedema. T e tumor originates in ed. Philadelphia, PA: Elsevier Saunders; 2008: 202027.
vascular endothelial cells and mani ests clinically as Rockson SG. Lymphedema. Curr reat Options Cardivasc
a reddish-blue or purple skin lesion with a macular Med. 2006;8:12936.
omita K, Yokogawa A, Oda Y, erahata S. Lymphangiosar-
shape, an ulcer, a poor healing eschar or a rm pain-
coma in postmastectomy lymphedema (Stewart- reves
ul nodule. Syndrome): Ultrastructural and immunohistologic char-
Lymphangitis is usually caused by group A beta- acteristics. J Surg Oncol. 1988;38:275.
hemolytic streptococcal or staphylococcal in ections iwari A, Cheng KS, Button M, et al. Di erential diagnosis,
and presents in an area o cellulitis. It clinically pre- investigation, a current treatment o lower limb lymph-
sents with pain and multiple linear, long, red streaks edema. Arch Surg. 2003;138:15261.
Warren AG, Brorson H, Borud LJ, Slavin SA. Lymphedema:
toward the regional lymph nodes and can even man- A comprehensive review. Ann Plast Surg. October 2007;
i est as a systemic response with evers, chills, sepsis, 59(4):46472.
or death. reatment consists o warm compresses Wake eld W, Rectenwald JR, Messina LM. Veins and Lym-
and intravenous antibiotics. phatics. In: Doherty GM. Current Diagnosis and reatment
T rombophlebitis clinically presents with an ery- Surgery. 13th ed. New York, NY: McGraw-Hill Medical;
2009:804.
thematous palpable cord along a super cial vein.
Werner G , Scheck R, Kaiserling E. Magnetic resonance
Causes o thrombophlebitis are rom an indwelling imaging o peripheral lymphedema. Lymphology. 1998;31:
catheter, venous stasis, intravenous drug use, or an 3436.
occult hypercoagulable state. reatment includes Weissleder H, Weissleder R. Interstitial lymphangiography:
compression stockings and anti-inf ammatory medi- Initial clinical experience with a dimeric nonionic contrast
cations with surgery being reserved or clusters o agent. Radiology. 1989;170:3714.
Yasuhara H, Shigematsu H, Muto . A study o the advan-
varicosities or cases o suppurative septic thrombo- tages o elastic stockings or leg lymphedema. Int Angiol.
phlebitis. 1996;15:272.
91
Venous Stasis Disease

John illou

A 48-year-old emale presents with a 10-month his- 2. Which o the ollowing statements regarding
tory o le , lower extremity swelling with worsening chronic venous insuf ciency (CVI) is true?
swelling and crampy pain over the last two days. A. Venous stasis ulcers are o en located over the
She has a history o hypertension or which she takes lateral malleolus.
hydrochlorothiazide. Her medical history is otherwise B. Ultrasound is o limited utility in the diagnosis
unremarkable. She has no amily history o thrombo- o venous insu ciency.
philia. On physical exam her distal pulses are easily C. Distal extremity ulcerations associated with arte-
palpable. She is noted to have multiple bilateral lower rial disease are twice as common as venous stasis
extremity varicosities as well as a brawny appearing, ulcers.
well-granulated ulcer over the le medial malleolus. D. T e rendelenberg test can be used to distin-
T e circum erence o her le thigh is noted to be 5 cm guish between super cial and deep venous sys-
greater than that o the right thigh and her le cal is tem ref ux.
4 cm greater in circum erence than that o the right E. Cal muscle dys unction plays no role in venous
cal . C scan o the pelvis is consistent with May- insu ciency.
T urner Syndrome.
3. Which o the ollowing is true regarding the
1. Which o the ollowing statements regarding May- medical management o CVI?
T urner Syndrome (M S) is true? A. Systemic antibiotics are recommended as part o
A. M S related thrombus accounts or 15% to 20% the routine treatment o venous stasis ulcers.
o all lower extremity deep venous thromboses B. Hydrocolloid dressings have not been shown to
(DV ). de nitively improve ulcer wound healing over
B. Compression o the right common iliac vein by simple non-adherent dressings.
the le common iliac artery is the most common C. In terms o venous ulcer healing, inelastic
cause o M S. compression systems are more e ective than
C. A common cause o M S related mortality is multi-layer compression systems with an elastic
acute pulmonary embolism. component.
D. Duplex ultrasound is the best imaging modality D. Moderate exercise may limit venous ulcer wound
with which to assess or iliac thrombus. healing.
E. In the setting o M S associated thrombosis, E. Intermittent pneumatic compression devices are
anticoagulation alone is the most e ective rst considered rst line treatment in the majority o
line treatment. patients.
344 G EN ERAL S U RG ERY EXAM I N ATI O N AN D BO ARD REVI EW

4. Which o the ollowing is true regarding the involving compression o the in erior vena cava and
surgical management o CVI? right common iliac vein do exist. Over time chronic
A. T e risk o greater saphenous nerve injury is compression o the iliac vein leads to endothelial
increased with greater saphenous vein stripping injury and thrombus ormation. Acute M S related
distal to the knee. thrombosis is o en heralded by the sudden onset
B. Radio requency ablation o the greater saphe- o le lower extremity pain and swelling and may
nous vein is not e ective at preventing long term be precipitated by pregnancy or recent abdominal
recurrence o venous ref ux. surgery.
C. umescent solution is not commonly used when Duplex ultrasound lacks the sensitivity to detect
per orming endovenous laser ablation. most cases o ileo emoral thrombus. T e pre erred
D. With regard to endovenous laser ablation, longer imaging modalities in cases o M S include C with
laser wavelengths are better absorbed by hemo- venous phase contrast or magnetic resonance (MR)
globin and there ore exert less o an ablative venogram.
e ect on the vein wall. Catheter directed thrombolyses as well as percu-
E. In the management o venous stasis ulcers, taneous mechanical thrombectomy with the addition
allogra placement in conjunction with com- o endovascular stent placement are the treatments o
pression has not been shown to improve wound choice or the management o acute M S associated
healing. thrombosis. Anticoagulation alone or with mechani-
cal thrombectomy without stent placement results in
5. Which o the ollowing is true regarding varicose re-thrombosis rates upwards o 70%. Endovascular
veins? stent placement is also considered the rst line treat-
A. Varicose veins are more prevalent in men than ment o M S without evidence o iliac thrombus.
women.
B. Liquid sclerotherapy is superior to oam in the 2. D. T e rendelenberg test can be used to distinguish
treatment o varicose veins in the setting o between super cial and deep venous system incom-
greater saphenous vein ref ux. petence. o per orm the test, the patient is placed in
C. Ischemic stroke has been reported as a complica- the supine position and the a ected leg is elevated in
tion o the use o oam sclerotherapy in the man- order to empty the super cial venous system. Either
agement o varicose veins. manual compression or a tourniquet is applied
D. Varicose veins are never pain ul. proximally and the patient is moved to an upright
E. T e use o compression stockings has been position. Individuals with deep venous system insu -
de nitively ound to prevent the progression o ciency will have rapid lling o the super cial veins
uncomplicated varicose veins. whereas patients with ref ux that is limited to the
super cial venous system will not have lling or
greater than 20 seconds a er changing position.
Venous stasis ulcers are o en located over the
ANSWERS
distal-medial aspect o the lower extremities, com-
1. C. Causes o mortality in the acute setting include monly in the vicinity o the medial malleolus. In con-
pulmonary embolism and iliac vein rupture resulting trast, ulcers associated with arterial disease tend to
in a retroperitoneal hemorrhage. Individuals with occur over the distal digits or over distal-lateral bony
chronic M S may develop chronic le lower extrem- prominences. Both orms o ulceration can be pain-
ity edema as well as venous stasis ulcers. M S related ul. Venous stasis ulcers are typically shallow with
thrombosis accounts or 2% to 3% o lower extrem- irregular boarders and commonly have a granulated
ity DV s and is most o en seen in emales 20 to or brinous base.
40 years old. Autopsy studies indicate that anatomic Evaluation with duplex ultrasonography is recom-
eatures consistent with M S are ound in 22% to mended or most patients with a new clinical diagno-
32% o individuals in the general population. sis o venous insu ciency. Ultrasound is use ul in the
M S is most commonly the result o compression assessment o proximal venous compression, deep
o the le common iliac vein by an overlying right vein thrombosis, and or determining the direction o
common iliac artery, although anatomic variants venous f ow. By using a rapid cu inf ation-def ation
C H AP TER 9 1 VEN O U S S TAS I S D I S EAS E 345

maneuver while the patient is upright, the extent o 4. A. T e classic greater saphenous vein (GSV) strip-
the reversal o venous f ow in the super cial system ping procedure involves stripping o the vein rom
can be determined. Reversal o f ow or greater that the ankle to the groin with high ligation at the
0.5 seconds in duration is consistent with incompe- sapheno- emoral junction. T e greater saphenous
tent super cial valves whereas reversal o f ow or nerve, which is a sensory cutaneous branch o the
greater than one second is consistent with ref ux orig- emoral nerve, is at risk o injury as it emerges rom
inating in the deep venous system. the adductor canal ( ormed by the tendons o the
Venous stasis ulcers are present in around 1% o gracilis and sartorius muscles) to run with the GSV
the population in westernized countries and account just posterior-medial to the tibia. Injury to the nerve
or around 80% o lower extremity ulcers. results in loss o cutaneous sensation over the medial
Normal cal muscle unction is thought to improve leg and can be avoided by not stripping the GSV dis-
venous return via a pump-like action during ambu- tal to the knee.
lation and lower extremity exercise. Impaired cal Radio requency ablation has been shown to result
muscle unction has been shown to positively cor- in long term GSV occlusion with occlusion rates
relate with the severity o venous insu ciency as well upwards o 88% at our years in one large trial. Another
as with the ormation o stasis ulcers. recent study reports GSV and small saphenous vein
occlusion rates o up to 94.6% and 94.5% respectively
3. B. A meta-analysis o 42 randomized trials compar- at 14 months a er radio requency ablation.
ing the use o di erent dressing types in individuals umescent solution is used in both radio re-
with venous stasis ulcers demonstrated no signi - quency ablation as well as endovenous laser ablation
cant di erence in wound healing rates when using a to anesthetize the tissue surrounding the vein as well
hydrocolloid dressing verses a simpler, less expensive as to protect the area rom thermal injury.
non-adherent dressing. Shorter laser wavelengths are better absorbed by
Bacterial colonization o stasis ulcers is common. hemoglobin whereas longer wavelengths are better
A meta-analysis that reviewed 22 trials involving the absorbed by water allowing them to have greater abla-
use o topical antibiotics, antiseptics or systemic anti- tive e ects on vein walls. Commonly used endovenous
biotics ound no clear evidence to support the routine laser wavelengths are between 810 and 1470 nm.
use o systemic antibiotics in individuals with venous A Cochrane review rom 2013 ound that allo-
stasis ulcers. T ere was some data to suggest the gra when used in conjunction with compression
use ulness o topical cadexomer iodine (not readily resulted in superior healing o longstanding stasis
available in the United States); however, the authors ulcers when compared to treatment with compres-
concluded that more data was needed to deter- sion alone. O note, the authors concluded that cur-
mine the e cacy o alternative topical antibiotics/ rent data is insu cient to determine whether the use
antiseptics such as povidone iodine and mupirocin. o autogra or xenogra results in increased rates o
Systemic antibiotics are recommended in cases o healing.
ulcers with surrounding cellulitis.
A recent Cochrane review ound that the use o 5. C. Although uncommon, ischemic strokes as well
compressive dressings promotes increased ulcer as transient ischemic attacks have been reported
healing rates when compared to dressings without a er oam sclerotherapy. Additional complications
an element o compression (i.e., Unna boot). Single include DV , anaphylactic reactions, and local tissue
layer dressings were less e ective than multi-layered necrosis.
dressings and multi-layered dressings utilizing an Varicose veins are approximately twice as com-
elastic component were ound to be more e ective mon in women with a reported prevalence o 15% to
than those without. Increased cal muscle unction as 30.1% in men and 28% to 50.5% in women. Risk ac-
the result o structured exercise is likely to improve tors associated with the ormation o varicose veins
venous return and thereby promote ulcer healing. include prior pregnancy, standing occupations, prior
Intermittent pneumatic compression devices are not episodes o thrombophlebitis, DV , amily history,
considered rst line treatment or most individuals with and obesity.
chronic venous insu ciency as they are both expensive Foam sclerotherapy, typically using 1% or 3%
and require patient immobilization while in use. polidocanol in a gas to liquid ratio o 4:1, has been
346 G EN ERAL S U RG ERY EXAM IN ATI O N AN D BO ARD REVIEW

ound to be superior to liquid polidocanol in the Kim JY, Choi D, Guk Ko Y, et al. Percutaneous treatment o
treatment o varicose veins due to greater saphen- deep vein thrombosis in May-T urner syndrome. Cardio-
ous vein ref ux. Lower extremity varicosities can be vasc Intervent Radiol. 2006;29(4):5715.
Merchant RF, Pichot O, Myers KA. Four-year ollow-up on
pain ul with standing. In addition to cosmetic com- endovascular radio requency obliteration o great saphe-
plaints, patients requently report eelings o heavi- nous ref ux. Dermatol Surg. 2005;31(2):12934.
ness, aching or itching sensations. Molloy S, Jacob S, Buckenham , Khaw K , aylor RS. Arterial
Despite numerous studies and randomized con- compression o the right common iliac vein; An unusual
trolled trials, compression stockings have not been anatomical variant. Cardiovasc Surg. 2002;10(3):2912.
Nelson EA, Mani R, T omas K, Vowden K. Intermittent pneu-
clearly shown to halt the progression or slow the
matic compression or treating venous leg ulcers. Cochrane
recurrence o uncomplicated varicose veins. Database Syst Rev. 2011;16(2): CD001899.
OMeara S, Cullum N, Nelson EA, Dumville JC. Compres-
BIBLIOGRAPHY sion or venous leg ulcers. Cochrane Database Syst Rev.
Cavezzi A, Parsi K. Complications o oam sclerotherapy. 2012;14(11): CD000265.
Phlebology. 2012;27(1):4651. OMeara S, Al-Kurdi D, Ologun Y, et al. Antibiotics and anti-
Choi JH, Park HC, Joh JH. T e occlusion rate and patterns septics or venous leg ulcers. Cochrane Database Syst Rev.
o saphenous vein a er radio requency ablation. J Korean 2014;10(1):CD003557.
Surg Soc. 2013;84(2):10713. OSullivan GJ, Semba CP, Bittner CA, et al. Endovascular
Coleridge SP. Sclerotherapy and oam sclerotherapy or vari- management o iliac vein compression (May-T urner)
cose veins. Phlebology. 2009;24(6):2609. syndrome. J Vasc Interv Radiol. 2000;11(7):82336.
Criqui MH, Jamosmos M, Fronek A, et al. Chronic venous OSullivan GJ. T e role o interventional radiology in the
disease in an ethnically diverse population: the San Diego management o deep venous thrombosis: Advanced ther-
Population Study. Am J Epidemiol. 2003;158(5):44856. apy. Cardiovasc Intervent Radiol. 2011;34(3):44561.
Evans CJ, Fowkes FG, Ruckley CV, Lee AJ. Prevalence o vari- Padberg F , Jr, Johnston MV, Sisto SA. Structured exercise
cose veins and chronic venous insu ciency in men and improves cal muscle pump unction in chronic venous
women in the general population: Edinburgh vein study. insu ciency: a randomized trial. J Vasc Surg. 2004;39(1):
J Epidemiol Community Health. March 1999;53(3):14953. 7987.
Fernando RR, Koranne KP, Schneider D, Fuentes F. May- Pal reyman SJ, Michaels JA. A systematic review o compres-
T urner Syndrome. Texas Heart Institute Journal. 2013; sion hosiery or uncomplicated varicose veins. Phlebology.
40(1):827. 2009;24(1):1333.
Fretz V, Binkert CA. Compression o the in erior vena cava by Pal reyman S, Nelson EA, Michaels JA. Dressings or venous
the right iliac artery: A rare variant o May-T urner syn- leg ulcers: Systematic review and meta-analysis. BMJ.
drome. Cardiovasc Intervent Radiol. 2010;33(5):10603. 2007;335(7613):244.
Gillespie DL, Writing Group III o the Paci c Vascular Sympo- Rabe E, Otto J, Schliephake D, Pannier F. E cacy and sa ety
sium 6, Kistner B, et al.Venous ulcer diagnosis, treatment, and o great saphenous vein sclerotherapy using standardised
prevention o recurrences. J Vasc Surg. 2010;52(5):8S14S. polidocanol oam (ESAF): A randomised controlled mul-
Jones JE, Nelson EA, Al-Hity A. Skin gra ing or venous leg ticentre clinical trial. Eur J Vasc Endovasc Surg. February
ulcers. Cochrane Database Syst Rev. 2013;1: CD001737. 2008;35(2):23845.
Kibbe MR, Ujiki M, Goodwin AL, et al. Iliac vein compres- White-Chu EF, Conner-Kerr A. Overview o guidelines or
sion in an asymptomatic patient population. J Vasc Surg. the prevention and treatment o venous leg ulcers: A U.S.
2004;39(5):93743. perspective. J Multidiscip Healthc. 2014;11(7):1117.
92
T oracic Outlet Syndrome

Courtney E. Barrows

A 34-year-old emale nurse, previously healthy, pre- D. Anterior scalene, subclavian vein, subclavius
sented to the emergency department with a 2-day his- muscle, brachial plexus, subclavian artery, mid-
tory o right shoulder pain that came on suddenly when dle scalene muscle, phrenic nerve.
she was li ing a heavy trash can. In addition, over the
past 24 hours she reports persistent right arm swelling 3. Which o the ollowing is true with regards to the
and bluish discoloration that is more severe with her diagnosis/workup o thoracic outlet syndrome?
arm in the dependent position. A. Electromyography and nerve conduction tests
alone exhibit a strong degree o speci city in
1. Which o the ollowing is true regarding the diagnosing.
pathophysiology o the condition this patient most B. Scalene muscle injection are not help ul in diag-
likely has? nosing n OS but serve as a use ul temporizing
A. Scalene muscle brosis is the major causative measure prior to de nitive surgery.
actor. C. Venography is considered the gold standard or
B. It is due to compression o the subclavian artery, the diagnosis o v OS, but the diagnosis can be
most likely rom a cervical rib. missed i the brachial veins are used or access.
C. It is due to repeated compression o the subcla- D. In v OS, ndings on dynamic ultrasound scan
vian vein in the costoclavicular space, resulting include presence o subclavian or axillary throm-
in intimal injury and thrombus ormation in the bus and decrease in venous velocity by 50% with
axillary and subclavian veins. abduction.
D. Hypercoaguable state leading to spontaneous
subclavian vein thrombosis. 4. Regarding the management o Paget-Schroetter
Syndrome, which o the ollowing is true?
2. Which o the ollowing is the most common order A. Most patients can be managed with anticoagula-
o anatomic structures in the thoracic outlet, tion alone and experience low recurrence rates.
moving rom anteromedial to posterolateral? B. It is best managed initially with angioplasty and
A. Subclavius muscle, subclavian artery, subclavian stenting prior to de nitive surgical decompres-
vein, phrenic nerve, anterior scalene muscle, bra- sion.
chial plexus, middle scalene muscle. C. Catheter-directed thrombolysis ollowed by
B. Subclavius muscle, subclavian vein, phrenic irst rib resection is the pre erred management
nerve, anterior scalene muscle, subclavian artery, strategy.
brachial plexus, middle scalene muscle. D. Primary patency rates exceed 90% i thrombol-
C. Anterior scalene, subclavian vein, subclavius ysis is per ormed within 6 weeks o symptom
muscle, subclavian artery, brachial plexus, mid- onset.
dle scalene muscle, phrenic nerve. E. None o the above.
348 G EN ERAL S U RG ERY EXAM I N ATIO N AN D BO ARD REVI EW

5. Which o the ollowing is the most common costoclavicular space. It accounts or approximately
complication o a f rst rib resection? 5% o all OS cases. It is classically seen in young,
A. lymphatic leakage healthy patients with a history o repetitive motion
B. pneumothorax (swimmers, baseball pitchers, manual laborer, nurse)
C. intercostal brachial cutaneous nerve injury or those with an active li estyle. Symptoms include
D. long thoracic nerve injury arm swelling (unique to v OS), cyanosis, pain/
E. subclavian vein injury aching, and occasionally paresthesias in the ngers
or hands (o en secondary to hand swelling).

ANSWERS 2. B. Moving rom anteromedial to posterolateral, the


subclavius muscle is the rst anatomic structure,
1. C. T e thoracic outlet is de ned as the anatomic which attaches rom the rst rib to the in erior por-
area bound by the clavicle superiorly, rst rib in e- tion o the clavicle and can be a source o compression
riorly, subclavius muscle anteriorly, and the middle o the next structure, which is the subclavian vein. T e
scalene muscle posteriorly. It is through this region phrenic nerve usually runs immediately posterior to
that the subclavian artery, vein, and brachial plexus the subclavian vein along the anterior scalene muscle.
pass as they exit the chest. T e thoracic outlet syn- However, in approximately 5% to 7% o individuals it
drome ( OS) re ers to the compression o one or can run anterior to the subclavian vein, and can be a
more o the neurovascular structures as they exit rare cause o subclavian vein compression.
this region, can be subdivided into 3 major classes T e subclavian artery is separated rom the vein
depending upon the structure that is compressed (in via the anterior scalene muscle. Posterior and lateral
order o decreasing incidence): neurogenic, venous, to the artery are the brachial plexus and nally the
and arterial. Neurogenic OS (n OS) is by ar the middle scalene muscle.
most common type, comprising greater than 90% o
all cases, and is due to compression o the brachial 3. D. T e diagnosis o n OS can be challenging and is
plexus as it passes over the rst rib between the ante- made with a combination o physical examination
rior and middle scalene muscles (through the scalene maneuvers as well as adjuncts such as nerve conduc-
triangle). tion studies, EMG, and even anterior scalene mus-
Scalene muscle brosis as the result o a traumatic cle injections o local anesthetics or botox. T ere is
event such as an MVC (classically whiplash injury) a growing body o evidence to suggest that anterior
or engaging in repetitive upper extremity activity scalene injections are reliable means to not only diag-
is a major causative actor, although occasionally nose n OS but also predict which patients will ben-
an anomalous rst rib or cervical rib can result in e t rom surgical decompression. In one study, 122
n OS. T e most common presenting symptom is patients being evaluated or n OS underwent ante-
arm paresthesias, present in approximately 90% rior scalene block using electrophysiologic guidance.
o patients. Other classical symptoms are pain and O the patients who were ultimately diagnosed with
weakness in the upper extremity, neck pain, and a condition other than OS, only 5% had a positive
occipital headaches. Pain and paresthesias are most response to the block, compared to 92% o patients
commonly noted in the C8- 1 (ulnar distribution). with con rmed OS. In addition, o the patients who
Arterial OS (a OS) is the least common variant underwent surgical decompression or OS, 94% o
with an incidence o approximately 1%. Patients patients with a positive response to scalene muscle
typically present with ispilateral hand and/or digit block preoperatively had success ul surgery while
ischemia rom distal embolization o a subclavian only 50% o those with negative response preopera-
artery stenosis or aneurysmal dilation, almost always tively had good outcomes.
as a result o extrinsic compression rom a cervical or In addition to standard chest radiographs to
anomalous rst rib. T e patient described above su - detect the presence o a cervical or anomalous rst
ers rom venous OS (v OS), also known as Paget- rib, dynamic duplex ultrasonography is used to iden-
Schroetter Syndrome, which is thrombosis or severe ti y axillosubclavian thrombus as well as evaluate or
narrowing o the subclavian and/or axillary vein venous compression by having the patient abduct the
rom chronic extrinsic compression at the level o the ipsilateral arm, which should result in a decrease in
C H AP TER 9 2 TH O RAC IC O U TLET S YN D RO M E 349

venous velocity by 50%. Ultrasound is recommended seal the de ect. Post operative causalgia requiring
by the American College o Radiology as the best rst sympathectomy occurred in two patients. One
approach or direct evaluation o arm veins, although patient experienced lymphatic leak.
venography is still considered the gold standard or In another series examining 334 surgeries or n OS,
making the diagnosis o v OS (Paget-Schroetter 13 complications (3.9%) occurred. T e most common
Syndrome). However, ultrasound-guided access to the was pneumothorax (n = 7), or which 6 required chest
deep system (true brachial veins) at the antecubital tubes and 1 patient required additional surgery. T ere
ossa or upper arm is needed to obtain optimal visu- were 5 instances o vascular injury (4 minor subcla-
alization and also per orm intervention. I the cephalic vian vein injuries and 1 transection o the internal
vein is used or contrast injection, the diagnosis can mammary artery resulting in a 2 L blood loss). T ere
be missed. was also one minor injury to the long thoracic nerve.
T e intercostal brachial nerve is encountered dur-
4. D. Because o poor long-term outcomes with anti- ing the transaxillary approach as it exits between the
coagulation alone, which has been associated with 1st and 2nd ribs, and should be preserved i possible.
symptom recurrence in up to 70% o patients, the Damage to this nerve would result in loss o sensa-
standard accepted treatment or Paget-Schroetter tion over the medial aspect o the arm.
Syndrome is catheter-directed thrombolysis ollowed
by rst rib resection. However, success o thromboly- BIBLIOGRAPHY
sis decreases as the time between symptom onset and Adelman MA, Rosen RJ, et al. A multidisciplinary approach
presentation increases. Although the exact time rame to the treatment o Paget Schroetter syndrome. Ann Vasc
is up or debate, the best results have been achieved Surg. 1997;11:14954.
in patients who present within 1014 days. Certainly Altobelli GG, Kudo , Ahn SS, et al. T oracic outlet syndrome:
a er 6 weeks, long term patency results are quite low. Pattern o clinical success a er operative decompression.
J Vasc Surg. 2005;42:1228.
Angioplasty with or without venous stent is not Brooke BS, Freischlag JA. Contemporary management o tho-
recommended as treatment or Paget Schroetter racic outlet syndrome. Curr Opin Cardiol. 2010;25:53540.
Syndrome due to the rate o complications and re- Desai SS, Azizzadeh A, et al. Outcomes o surgical paracla-
stensosis or thrombosis. Stenting o the vein run- vicular thoracic outlet decompression. Ann Vasc Surg.
ning through the non-decompressed costoclavicular 2014;28:45764.
Doyle A, Saad WE, et al. Management o ef ort thrombosis
junction has been shown to be complicated by stent
o the subclavian vein: todays treatment. Ann Vasc Surg.
racture in some, de ormation in nearly all, and 2007;21:7239.
rethrombosis rates as high as 40%. Hempel GK, Shutze WP, Anderson JF, Bukhari HI. 770 con-
secutive supraclavicular rst rib resections or thoracic
5. B. All o the above have been reported as compli- outlet syndrome. Ann Vasc Surg. 1996;10:45663.
cations a er rst rib resection, with pneumothorax Illig KA, Doyle AJ. A comprehensive review o Paget-Schroetter
being the most common. syndrome. J Vasc Surg. 2010;51:153847.
Jordan SE, Machleder HI. Diagnosis o thoracic outlet syn-
Entry into the pleural space with resultant pneu-
drome using electrophysiologically guided anterior scalene
mothorax and/or pleural ef usion is a known hazard blocks. Ann Vasc Surg. 1998;12:2604.
o rst rib resection. At the completion o the pro- Lee J , Karwowski JK, Olcott C, et al. Long-term thrombotic
cedure, it is generally recommended to instill irriga- recurrence a er nonoperative management o Paget-
tion into the eld and per orm a Valsalva maneuver Schroetter syndrome. J Vasc Surg. 2006;43:123643.
in order to check or pneumothorax. I present, a Meier GH, Pollack JS, Gusberg RJ, et al. Initial experience with
venous stents in exertional axillary-subclavian vein throm-
small chest tube can be placed. In a large retrospec- bosis. J Vasc Surg. 1996;24:97481.
tive series o 770 patients who underwent supra- Propper BW, Freischlag JA. T oracic Outlet Syndromes. In:
clavicular rst rib resection and scalenectomy, very Cameron JL and Cameron AM. Current Surgical T erapy.
ew operative complications were noted. Although 11th ed. Philadelphia, PA: Elsevier Saunders; 2014: 9247.
pneumothorax is a known complication o this pro- Sanders RJ, Hammond SL, Rao NM. Diagnosis o thoracic
outlet syndrome. J Vasc Surg. 2007;46:6014.
cedure, no patients developed post operative pneu-
Sanders R. T oracic Outlet Syndrome: General Consider-
mothorax because or the 20% o patients in whom ations. In Cronenwett JL and Johnson KW, ed. Rutherfords
the pleural space was inadvertently entered during Vascular Surgery. 7th ed. Philadelphia, PA: Elsevier Saun-
the procedure, a closed suction drain was placed to ders; 2010:186577.
350 G EN ERAL S U RG ERY EXAM I N ATI O N AN D BO ARD REVI EW

Schroeder WE, Green FR. Phrenic nerve injuries: Report o a Urschel Jr HC, Razzuk MA. Paget-Schroetter Syndrome:
case, anatomical and experimental researches, and critical What is the best management? Ann T orac Surg. 2000;69:
review o the literature. Am J Med Sci. 1902;123:196220. 16638.
Urschel HC, Patel AN. Surgery remains the most ef ective Urschel HC, Patel AN. Paget-Schroetter syndrome therapy:
treatment or Paget-Schroetter Syndrome: 50 yearsexperi- Failure o intravenous stents. Ann T orac Surg. 2003;75:
ence. Ann T orac Surg. 2008;86:25460. 16936.
Gynec o l o gy a nd Obst et r ic s
Alan P. Gehrich
This page intentionally left blank
93
Ovarian Masses

Charles S. Dietrich III and Bradford P. Whitcomb

Disclaimer: T e views expressed in this manuscript are A. BRCA1 mutation


those o the authors and do not re ect the o cial policy B. BRCA2 mutation
or position o the Department o the Army, Department C. Lynch syndrome
o De ense, or the United States Government. D. Mother af ected with ovarian cancer
E. Mother and sister af ected with ovarian cancer
A 52-year-old post-menopausal emale presents to her
primary physician with abdominal bloating and uri- 3. For the above patient, what is the best management
nary requency over the past several weeks. She denies strategy?
abdominal or pelvic pain but con rms mild constipa-
A. Observation
tion with reported bowel movements every three days.
B. Percutaneous core biopsy o the adnexal mass
She has had no vaginal or rectal bleeding. Her past med-
C. Surgical exploration with staging and cytoreduc-
ical and surgical history is signi cant or mild hyper-
tion
tension and a prior cesarean delivery. Her mother was
D. Neoadjuvant chemotherapy
diagnosed with breast cancer at age 45.
E. Palliative care
On examination, her vital signs are normal. Abdomi-
nal exam reveals normal bowel sounds and mild dis- 4. Which of the following procedures should be
tention. No abdominal pain is elicited and there are no included in the surgical staging of an apparent
palpable abdominal masses. On pelvic exam, a nodular early ovarian malignancy?
mass is noted in the le adnexal region with decreased
A. Pelvic washings
mobility. Rectal exam is unremarkable. Laboratory
B. Removal o the ovarian mass
assessment shows a normal CEA, but her CA125 level is
C. Omentectomy
elevated at 430 U/mL.
D. Pelvic and para-aortic lymph node sampling
1. What is the best initial imaging study for a patient E. All o the above
with a suspected ovarian mass?
5. Which of the following adjuvant treatment options
A. Abdominal X-rays is associated with the greatest overall survival?
B. Ultrasound
C. Computerized tomography (C ) scan A. Intravenous cyclophosphamide and cisplatin
D. Magnetic resonance imaging (MRI) B. Intravenous paclitaxel and carboplatin
E. Positron emission tomography-computed C. Intravenous paclitaxel, carboplatin, and bevaci-
tomography (PE -C ) zumab
D. Intraperitoneal and intravenous paclitaxel and
2. Which of the following confers the highest risk for cisplatin
lifetime development of ovarian cancer? E. Intravenous cyclophosphamide and doxorubicin
354 G EN ERAL S U RG ERY EXAM I N ATI O N AN D BO ARD REVIEW

ANSWERS planning phase. PE -C has poor sensitivity and


speci city or adnexal masses and is expensive. Its
1. B. T e best initial imaging study or evaluating an utility is best served in a patient with a known ovar-
ovarian mass is the trans-vaginal ultrasound ( VS). ian malignancy that has a suspicion o recurrence.
Morphologic characteristics, volume o the mass, MRI is a rather speci c modality when investigating
and assessment o vascular resistance through color the adnexa, and is usually reserved or indeterminate
Doppler assists in dif erentiating benign rom malig- ultrasound ndings.
nant ovarian processes. T e addition o abdominal
ultrasound is important in delineating larger ovar- 2. A. Obtaining a thorough, and accurate, amily history
ian and uterine masses extending outside the pelvis. is mandatory in evaluating the patient with a suspi-
T e sensitivity o VS with color Doppler is approxi- cious adnexal mass. T e average womens li etime risk
mately 92% to 99% or identi cation o malignant or developing ovarian cancer is 1.4%. However, in a
tumors o the ovary. Findings on ultrasound sugges- meta-analysis published in 1998, the li etime relative
tive o a malignant process include complex masses risk o developing ovarian cancer with one af ected
with solid and cystic components, internal or exter- rst degree relative was 3.1 (95% CI 2.63.7), con er-
nal nodular excrescences, and surrounding pelvic ring a 5% li etime risk. I more than one relative was
ascites (Figure 93-1). Although there is no universal af ected, the estimated relative risk increased to 11.7
test or combination o tests that are recommended (95% CI 5.325.9) (approximate 20% li etime risk).
or ovarian cancer screening, there is data to support Overall, approximately 5% to 10% o women diag-
the use o the morphology index (based on tumor nosed with ovarian cancer have an inherited syn-
characteristics and volume) derived rom VS nd- drome that increases risk. BRCA1 and 2 are tumor
ings to better risk strati y the ovarian abnormality as suppressor genes coding or a protein involved in the
benign versus malignant. repair o double-stranded DNA breaks. Mutations in
Plain lm X-rays are not very use ul in the evalu- these genes are the most common deleterious amil-
ation o ovarian masses as they are typically com- ial mutations in ovarian cancer and con er the high-
posed o uid and so tissue; however, a plain lm est risk o penetrance compared to other risk actors.
may reveal incidental calci cations, one o the hall- Patients with BRCA1 mutations have a li etime ovar-
mark ndings in ovarian teratomas. C per ormance ian cancer risk o approximately 40% to 53%, while
dif erentiating the so tissue structures within the BRCA2 penetrance is 20% to 30%. Hereditary Non-
ovary is also limited. Radiation exposure and IV Polyposis Colorectal Cancer (HNPCC), or Lynch
contrast administration pose additional risk to the Syndrome, represents a de ect in the mismatch repair
patient. C is most use ul i malignancy is suspected (MMR) genes. T ese patients carry a very high risk
to evaluate or metastatic disease in the preoperative o developing colon and endometrial cancer, as well
as additional risk or other extracolonic malignan-
cies. Ovarian cancer risk varies based on the actual
genetic de ect: MLH1/MSH2424%; MSH6
111%; PMS26%. Other amilial syndromes, such
as Li-Fraumeni ( P53 mutation) and Peutz-Jeghers
(S K11 mutation) also carry increased risks o
developing ovarian cancer.

3. C. T e menopausal patient described has ndings


concerning or an ovarian malignancy based on her
constitutional symptoms, the adnexal mass, and her
elevated CA-125 level. In 2016, an estimated 22,280
new ovarian cancer cases will occur in the United
States, and there will be an estimated 14,240 deaths
Figure 93-1 A trans-vaginal ultrasound o an ovarian high (http://seer.cancer.gov/stat acts/html/ovary.html).
grade serous carcinoma demonstrating a complex mass with Ovarian cancer is the h leading cause or cancer
solid and cystic regions. deaths among women. While symptoms occur in
C H AP TER 9 3 O VARI AN MAS S ES 355

women with ovarian malignancies, they are o en apparent malignancy con ned to the ovary ranges
vague and nonspeci c. As such, almost 80% o patients rom 22% to 50% when ormal staging procedures
are diagnosed with metastatic disease. Persistent are per ormed. Formal staging usually includes the
symptoms that should prompt evaluation o the ollowing: removal o the af ected ovary; removal o
ovaries include pelvic pain, changes in bowel habits, the uterus and contralateral ovary (i ertility is not
bloating, and urinary requency. T e overall 5-year desired); care ul evaluation o all peritoneal sur aces
survival or all patients diagnosed with ovarian can- with biopsy o any suspicious areas; peritoneal wash-
cer is currently around 45%. ings or cytologic evaluation; in racolic omentec-
T e American College o Obstetricians and tomy; random peritoneal sampling rom the right
Gynecologists (ACOG) and Society o Gynecologic hemidiaphragm, paracolic gutters, pelvic sidewalls,
Oncology (SGO) recommend re erral o the meno- bladder, and posterior cul-de-sac; and removal o
pausal patient with an adnexal mass to a gyneco- pelvic and para-aortic lymph nodes. Lymphadenec-
logic oncologist i any o the ollowing is ound: any tomy probably is the most important and technically
elevation in the CA-125, associated ascites, the mass challenging component o adequate staging. In a
is xed or nodular, evidence o metastatic disease is review by Powless et al., 13% o apparent early stage
noted, or i amilial risk is determined. In the pre- ovarian cancers were ound to have positive nodes.
menopausal patient with an adnexal mass, re er- While still recommended, several investigators have
ral should occur or CA-125 > 200 U/mL, ascites, questioned the utility o omentectomy and random
evidence o metastatic disease, or hereditary risk. peritoneal biopsies when no gross disease is visible.
Observation has signi cant risk or delay in diagno- In their reviews, upstaging based on omentectomy
sis and is not recommended. Percutaneous biopsy o occurred in less than 4% o patients. Similarly, less
the mass carries signi cant risk or peritoneal seed- than 5% o patients were ound to have microscopic
ing or upstaging o the disease. Furthermore, it is not disease on random peritoneal biopsies.
accurate, with sensitivity or malignancy detection T e AC ION trial con rmed the importance o
ranging rom 25% to 82%. adequate staging. In this study, patients with Stage IA
Neoadjuvant chemotherapy (chemotherapy to IIA ovarian epithelial carcinoma were randomized
given be ore surgical debulking o disease) is a er surgery to observation or adjuvant chemo-
another option or patients who have had a con r- therapy. Groups were urther strati ed into optimal
mation o ovarian cancer by paracentesis or other versus non-optimal staging categories. In a recent
biopsy. Patients with comorbid conditions making long-term analysis, overall survival was improved in
them poor surgical candidates or those with bulky patients with optimal surgical staging, even among
upper abdominal or stage IV disease may bene t those receiving adjuvant chemotherapy [HR = 1.89,
most rom this approach. Recent trials have shown p = 0.05]. Considerable debate is ongoing regarding
that neoadjuvant chemotherapy of ers a signi cant the route o surgical staging. Historically, laparotomy
improvement in optimal cytoreduction at interval was elt to of er the best exposure or ull evaluation
surgery, diminished surgical morbidity, and similar o all peritoneal sur aces, and this approach is still
progression ree and overall survival rates in Stage standard today. However, as advances in minimally
IIIC and IV ovarian cancer. However, there is much invasive surgery have occurred in the past decade,
debate internationally regarding this approach, and many investigators have concluded that operative
up ront surgery remains the most common strategy outcomes with a laparoscopic approach could be
in the United States. Palliative care is an option once compatible with those o laparotomy.
a pathologic diagnosis and accurate stage or prog- When an advanced ovarian malignancy is pre-
nosis has been determined, but it is usually reserved sent, the extent o surgical cytoreduction directly
or patients with progressive disease or or those with correlates with overall survival. Cytoreduction is a
signi cant comorbidities. concept that has until recently been unique to the
management o ovarian malignancies. T e theoretic
4. E. I an ovarian malignancy is con rmed on intra- bene ts o cytoreduction include improved patient
operative assessment, accurate staging is para- com ort, increased tumor per usion, increased
mount to determine the extent o the disease and tumor growth raction, and improved immunologic
appropriate adjuvant treatments. Upstaging o an response. Removal o all individual tumor implants
356 G EN ERAL S U RG ERY EXAM IN ATIO N AN D BO ARD REVI EW

to less than one centimeter is currently considered chemotherapy penetration into tumor nodules
an optimal cytoreduction. However, the de nition o exceeding one cm is diminished. IP ports can be
optimal cytoreduction is rapidly changing, and most easily placed at the time o a cytoreductive surgery,
gynecologic oncologists now consider reduction to but this requires preoperative anticipation o nd-
microscopic disease to be the goal. In a review o sev- ings and a lengthy discussion o adjuvant treatment
eral Gynecologic Oncology Groups (GOG) studies, options. T e IP catheter can also be placed at a sec-
patients with Stage IIIC disease had an overall sur- ond operation which can typically be accomplished
vival o 71.9 months when cytoreduction to micro- laparoscopically. A 9.6 French single lumen medport
scopic disease was achieved versus 42.4 months is positioned over a rib in the midclavicular line near
when disease was reduced to < 1 cm. Achieving an the in erior costal margin. T e catheter is then tun-
optimal cytoreductive status o en requires bowel neled subcutaneously to the level o the umbilicus
resection and extensive upper abdominal procedures be ore entering the peritoneal cavity. Approximately
such as diaphragmatic stripping, splenectomy, and 10 cm o catheter length should be contained intra-
distal pancreatectomy. abdominally. Port complications are the most likely
reason or discontinuation o therapy and include
5. D. With the exception o Stage IA grade 1 tumors, in ection, catheter occlusion, and intestinal erosion.
adjuvant chemotherapy is recommended or all epi- Postoperative adhesions can also develop limiting
thelial ovarian cancers ollowing surgical staging and circulation o chemotherapy.
tumor cytoreduction. Platinum based therapy has Signi cant ef orts are ocusing on biologic tar-
remained the cornerstone o therapy since the 1970s, geted therapies in the treatment o ovarian cancer.
with the current standard incorporating paclitaxel Bevacizumab, a monoclonal antibody that prevents
with carboplatin delivered intravenously every three angiogenesis by inhibiting vascular endothelial
weeks or six to eight cycles. With platinum based growth actor (VEGF), has received considerable
therapy, almost 75% o patients with advanced disease attention or its activity in a number o solid tumors.
will achieve remission. Un ortunately, recurrences are Several studies using bevacizumab in combination
likely, usually occurring within two years o primary with cytotoxic chemotherapy have demonstrated a
therapy, and are rarely curable. Intense ef orts are con- progression- ree survival advantage in ovarian can-
tinually ongoing to discover novel treatment options. cer; however, none have realized an overall survival
Intraperitoneal (IP) chemotherapy has long inter- advantage to date.
ested gynecologic oncologists or its theoretic advan-
tages. However, due to the challenges in delivery and
increased toxicities, this route o administration was BIBLIOGRAPHY
not avored until recently. In 2006, GOG 172 dem- American College o Obstetricians and Gynecologists. ACOG
onstrated one o the largest survival advantages or Practice Bulletin. Management o adnexal masses. Obstet
any ovarian cancer study ever published. In this trial, Gynecol. 2007;110(1):20114.
Anthoulakis C and Nikoloudis N. Pelvic MRI as the gold
a 16 month overall survival advantage was ound or
standard in the subsequent evaluation o ultrasound-
an IP regimen (Day 1, intravenous paclitaxel; Day intederminate adnexal lesions: a systematic review. Gynecol
2, intraperitoneal cisplatin; Day 8, intraperitoneal Oncol. 2014;132(3):6618.
paclitaxel) when compared to intravenous paclitaxel Armstrong DK, Bundy B, Wenzel L, et al. Intraperitoneal
and cisplatin delivered every three weeks. oxicities cisplatin and paclitaxel in ovarian cancer. N Engl J Med.
were signi cantly increased in the IP group, with 2006;354(1):3443.
Burger RA, Brady MF, Bookman MA, et al. Incorporation o
most patients only completing an average o 4 out bevacizumab in the primary treatment o ovarian cancer.
o 6 planned cycles. However, quality o li e assess- N Engl J Med. 2011;365(26):247383.
ments one year ollowing completion o therapy DePriest PD, Varner E, Powell J, et al. T e e cacy o a sono-
were equivalent between the two groups. Following graphic morphology index in identi ying ovarian can-
this publication, the NCI issued a rare clinical alert cer: A multi-institutional investigation. Gynecol Oncol.
1994;55(2):1748.
recommending that ovarian cancer patients be con-
Grabowski JP, Harter P, Buhrmann C, et al. Re-operation
sidered or IP treatment. outcome in patients re erred to a gynecologic oncology
Only patients who achieve an optimal cytore- center with presumed ovarian cancer FIGO I-IIIA a er
ductive status are candidates or IP therapy, as sub-standard initial surgery. Surg Oncol. 2012;21:315.
C H AP TER 9 3 O VARI AN M AS S ES 357

Harris RD, Javitt MC, Glanc P, et al. ACR Appropriateness Powless CA, Aletti GD, Bakkum-Gamez JN, et al. Risk actors
Criteria clinically suspected adnexal mass. Ultrasound Q. or lymph node metastasis in apparent early-stage epithe-
2013;29(1):7986. lial ovarian cancer: implications or surgical staging. Gyne-
Kehoe S, Hook J, Nankivell M, et al. Chemotherapy or up ront col Oncol. 2011;122(3):53640.
surgery or newly diagnosed advanced ovarian cancer: Powless CA, Bakkum-Gamez JN, Aletti GD, et al. Random
Results rom the MRC CHORUS trial. J Clin Oncol. (Meet- peritoneal biopsies have limited value in staging o appar-
ing Abstracts.) 2013;31(15 suppl):5500. ent early stage epithelial ovarian cancer a er thorough
Lee JY, Kim HS, Chung HH, et al. T e role o omentectomy exploration. Gynecol Oncol. 2009;115(1):869.
and random peritoneal biopsies as part o comprehensive Ramus SJ and Gayther SA. T e contribution o BRCA1 and
surgical staging in apparent early-stage epithelial ovarian BRCA2 to ovarian cancer. Mol Oncol. 2009;3(2):13850.
cancer. Ann Surg Oncol. 2014;21(8):27626. Siegal R, Ma J, Zou Z, et al. Cancer statistics, 2014. CA Cancer
National Comprehensive Cancer Network. Genetic/Familial J Clin. 2014;64(1):929.
High-Risk Assessment: Colorectal (Version 1.2014). http:// Stratton JF, Pharoah P, Smith SK, et al. A systematic review and
www.nccn.org/pro essionals/physician_gls/pd / genetics_ meta-analysis o amily history and risk o ovarian cancer.
colon.pd . Accessed May 14, 2014. Br J Obstet Gynaecol. 1998;105(5):4939.
Ozols RF, Bundy BN, Greer BE, et al. Phase III trial o carbopl- rimbos B, immers P, Pecorelli S, et al. Surgical staging and
atin and paclitaxel compared with cisplatin and paclitaxel treatment o early ovarian cancer: long-term analysis rom
in patients with optimally resected stage III ovarian can- a randomized trial. J Natl Cancer Inst. 2010;102:9827.
cer: A Gynecologic Oncology Group study. J Clin Oncol. Vergote I, rop CG, Amant F, et al. Neoadjuvant chemother-
2003;21(17):3194200. apy or primary surgery in stage IIIC or IV ovarian cancer.
Park HJ, Kim DW, Yim GW, et al. Staging laparoscopy or the N Engl J Med. 2010;363(10):94353.
management o early-stage ovarian cancer: A metaanalysis. Young R, Decker D, Wharton J . Staging laparotomy in early
Am J Obstet Gynecol. 2013;209:58e18. ovarian cancer. JAMA. 1983;250:30726.
Perren J, Swart AM, P sterer J, et al. A phase 3 trial o beva- Winter WE, Maxwell GL, ian C, et al. Prognostic actors or
cizumab in ovarian cancer. N Engl J Med. 2011;365(26): stage III epithelial ovarian cancer: A Gynecologic Oncol-
248496. ogy Group Study. J Clin Oncol. 2007;25(24):36217.
94
Ectopic Pregnancy

Charles S. Dietrich III and Brad ord P. Whitcomb

Disclaimer: T e views expressed in this manuscript are C. Computerized tomography (C ) scan


those o the authors and do not re ect the o cial policy D. Magnetic resonance imaging (MRI)
or position o the Department o the Army, Department E. Abdominal X-rays
o De ense, or the United States Government.
2. What single quantitative hCG level distinguishes
A 24-year-old emale presents to the emergency depart- between ectopic or intrauterine pregnancy
ment with worsening right pelvic pain over the past 12 (viable or nonviable) when no intrauterine uid
hours associated with vaginal spotting. She denies evers collection and normal adnexa are identif ed on
and chills and has had no nausea or vomiting. Her bowel ultrasonography?
movements have been regular, and she denies any urinary
A. 500 mIU/mL
symptoms. She is sexually active and not using contra-
B. 1000 mIU/mL
ception. Her last menstrual period was 7 weeks prior to
C. 2000 mIU/mL
presentation. Past medical and surgical history is remark-
D. 3000 mIU/ML
able only or a chlamydial in ection two years ago.
E. T ere is no single hCG value that can reliably
Her vital signs show a temperature o 98.6F with a
distinguish between ectopic or intrauterine
heart rate o 101 and blood pressure o 95/55. Focused
pregnancy.
examination shows mild abdominal distention with
right lower quadrant tenderness to deep palpation. No 3. Which o the ollowing is a contraindication to
peritoneal signs are elicited. Her pelvic exam reveals medically managing an ectopic pregnancy with
a small amount o bleeding rom a closed cervical os. methotrexate?
T ere is no cervical motion tenderness, but exquisite
A. Limited to no cardiac activity noted on ultraso-
tenderness with a slight ullness is appreciated in the
nography
right adnexal region.
B. Absence o pelvic pain
Laboratory assessment is notable or a positive urine
C. hCG = 4500 mIU/mL
hCG test. Quantitative hCG returns at 2200 mIU/mL.
D. Hemodynamically stable patient
Her white blood count is 6.5 109/L, hemoglobin
E. Unreliable ollow-up
9.8 g/dL, hematocrit 29.2%, and platelets are 230
109/L. Chemistries are unremarkable. 4. In a patient demonstrating hemodynamic
instability, what is the best management option or
1. What is the BEST initial imaging modality or this an ectopic pregnancy in the distal allopian tube?
patient? A. Expectant management
A. rans-vaginal ultrasound B. Methotrexate
B. rans-abdominal ultrasound C. Salpingostomy
C H AP TER 9 4 EC To P i C P REg n An C y 359

D. Salpingectomy
E. Salpingo-oophorectomy

5. Once an ectopic pregnancy has been diagnosed,


what is the uture risk or another ectopic
pregnancy?
A. 1%
B. 10%
C. 20%
D. 50%
E. 75%

ANSWERS Figure 94-1 rans-vaginal ultrasound image o the uterus


demonstrating a thin endometrial lining with no intrauterine
1. A. When a women presents with pain and bleeding gestational sac.
in early pregnancy, the di erential diagnosis includes
a viable intrauterine pregnancy, a nonviable intrau- pregnancy when expected based on hCG levels and/
terine pregnancy, or an ectopic pregnancy. As ultra- or gestational age, identi cation o an adnexal mass
sound technology has improved, this modality, along adjacent to the uterus and medial to the ovary, and
with serum human chorionic gonadotropin (hCG) heterogeneous pelvic uid (Figures 94-1 and 94-2).
measurement, has become the cornerstone in the T e sensitivity o trans-vaginal ultrasound to detect
evaluation o early-pregnancy complications. Early ectopic pregnancy ranges rom 0.73 to 0.93 in
pregnancy development ollows a relatively predict- reported series.
able path as seen on trans-vaginal ultrasonography. A
gestational sac is rst seen around 5 weeks o gesta- 2. E. Human chorionic gonadotropic measurement
tional age when measured rom the beginning o the is the second diagnostic pillar in the evaluation o
last menstrual period. T e yolk sac then appears at early-pregnancy complications. Un ortunately, iso-
5 weeks. T e embryo can usually be visualized adja- lated hCG levels of en overlap the three most com-
cent to the yolk sac at approximately 6 weeks, and a vis- mon possibilities in patients with rst-trimester
ible heartbeat presents concurrently with the embryo bleeding: viable intrauterine pregnancies, nonviable
in most cases. Any deviations rom this expected intrauterine pregnancies, and ectopic gestations. A
pattern o development are worrisome or an abnor- single hCG value, there ore, cannot be used to reli-
mal gestation. Imaging studies with ionizing radia- ably di erentiate between these di erent possibilities
tion should be avoided in early pregnancy i possible. despite intensive e orts to determine a discrimi-
Furthermore, modalities such as C scan, abdominal natory level. As ultrasound capabilities improved,
X-rays, and MRI add little to the evaluation.
Review o managed care databases suggests that
ectopic pregnancies occur in 2.6% o all pregnan-
cies. Risk actors include a history o a prior ectopic
pregnancy, prior pelvic in ections, previous surger-
ies, and smoking. Six percent o maternal deaths
in early pregnancy are caused by ruptured ectopic
pregnancies. However, improved diagnostic modali-
ties which allow or earlier identi cation, have kept
the maternal case- atality rate low at 3.8 deaths per
10,000 ectopic pregnancies. While trans-abdominal
ultrasonography can identi y an intrauterine preg-
nancy, it lacks the capability to ully assess the adnexa.
Findings on trans-vaginal ultrasound suggestive o an Figure 94-2 rans-vaginal ultrasound image demonstrating
ectopic pregnancy include absence o an intrauterine an ectopic mass adjacent to the lef ovary (OV).
360 g En ERAL S U Rg ERy EXAM i n ATi o n An D Bo ARD REVi EW

previous investigators reported a high accuracy in intramuscularly on Day 1. Serial hCG levels are
documenting an intrauterine gestation with hCG lev- trended and a 15% decrease is expected between Day
els between 1000 to 2000 mIU/mL. However, recent 4 and 7. Additional doses can be administered or ris-
studies have questioned these levels. Among women ing or plateaued values. Reliable ollow-up is critical
with a pregnancy o unknown location and hCG lev- when treating patients with methotrexate, and suc-
els between 2000 to 3000 mIU/mL, 19 ectopic preg- cess rates range rom 63% to 96.7% in the literature.
nancies and 38 nonviable intrauterine pregnancies T is wide range is due to the many de nitions o suc-
will be ound or each viable pregnancy, making the cess and variations in patient characteristics.
likelihood o a viable pregnancy approximately 2%.
T e lower reliability o an hCG discriminatory zone 4. D. Active ectopic pregnancies by de nition have
in the diagnosis o an intrauterine pregnancy may be either ruptured or have impending rupture. Patients
due to the higher rate o multiple gestations seen in can present with hemodynamic instability, hemop-
the obstetric population today. T e levels o hCG in eritoneum, and associated severe pain. Surgical man-
multiple gestations when compared to singleton preg- agement is necessary or either patient stabilization
nancies are higher or every stage o development. or because o the high ailure rate with a medical
In a patient with a pregnancy o unknown loca- approach. T e degree o hCG elevation usually is not
tion, obtaining at least one ollow-up hCG measure- help ul in these cases as management is dictated by
ment is recommended be ore initiating treatment to the patients overall condition.
avoid misdiagnosing a potential viable intrauterine T e vast majority o ectopic pregnancies are
pregnancy. Ninety-nine percent o symptomatic located in the distal allopian tube, accounting or
women with a viable pregnancy will demonstrate up to 97% o cases. Other ectopic implantation sites
at least a 53% increase in hCG levels over a 48 hour include the interstitial portion o the allopian tube,
interval. Serial evaluation should only be done i the cervix, ovary, abdomen, and previous cesarean scar.
patient is hemodynamically stable and can be reli- Heterotopic pregnancies (synchronous ectopic and
ably tracked. intrauterine pregnancies) are increasing in incidence
secondary to assisted reproductive technologies,
3. C. Once an ectopic pregnancy has been con rmed, occurring in 1% to 3% o this population.
there are our main treatment options: expectant Surgical options or managing an ectopic in the
management, medical treatment with methotrexate, distal allopian tube include conservative salpin-
conservative surgery with salpingostomy, and sal- gostomy versus the more radical salpingectomy.
pingectomy. Categorizing the activity o the ectopic Both procedures are easible though a laparoscopic
pregnancy into very less active, less active, and active approach, although patient stability and surgeon
can guide subsequent therapy. In general, less active experience will dictate the ultimate modality. Dur-
ectopic pregnancies can be managed medically with ing salpingostomy, a linear incision is made over
high success rates. Active ectopic pregnancies usu- the antimesenteric bulging portion o the tube. A
ally require surgical intervention. Not surprisingly, suction irrigator is then used to hydrodissect and
the de nition o activity has not been uni ormly remove the ectopic mass. T e tubal incision is not
established. One author has proposed to de ne a less reapproximated with suture and lef to close on its
active ectopic pregnancy (one that can be treated own. Salpingectomy includes complete removal o
with methotrexate) as one with a pretreatment hCG the allopian tube with the contained products o
< 5000, with no cardiac activity in the embryo, in conception.
women with no symptoms, and in those who are T e choice between conservative salpingostomy
hemodynamically stable. versus salpingectomy is usually made intraopera-
Methotrexate is an antimetabolite that inhibits tively. Factors in uencing management include the
dihydro olate reductase. Proli erating tissues, includ- patients history (prior ectopic), patient pre erence,
ing trophoblastic cells, are readily a ected. Its use the appearance o the contralateral tube, and bleed-
in treating ectopic pregnancies was rst reported ing. In patients demonstrating hemodynamic insta-
in 1982 by anaka et al. While several regimens bility, salpingectomy is usually pre erred and has a
are e ective, the most commonly used one today 100% e cacy rate. Salpingostomy carries a ailure
is a single-dose regimen using 50 mg/m 2 injected rate o 6.6% to 17.5%, leading to persistent ectopic
C H AP TER 9 4 EC To P i C P REg n An C y 361

tissue in the pelvis. In these cases, the hCG level ails Capmas P, Bouyer J, Fernandez H. reatment o ectopic preg-
to normalize, and subsequent treatment with metho- nancies in 2014: new answers to some old questions. Fertil
trexate may be necessary. Steril. 2014;101(3):61520.
Crochet JR, Bastian LA, Chireau MV. Does this woman have
an ectopic pregnancy? T e rational clinical examination
5. B. T e rate o recurrent ectopic pregnancy ranges
systematic review. JAMA. 2013;309(16):17229.
rom 6% to 10% regardless o the treatment modal- Doubilet PM, Benson CB, Bourne , Blaivas M. Diagnostic
ity selected. Patients of en question their uture er- criteria or nonviable pregnancy early in the rst trimester.
tility chances af er having an ectopic pregnancy. T e N Engl J Med. 2013;369:144351.
DEME ER trial compared medical management Doubilet PM, Benson CB. Further evidence against the reli-
with conservative salpingostomy, and ound no di - ability o the human chorionic gonadotropin discrimina-
tory level. J Ultrasound Med. 2011;30:163742.
erence in the 2-year ertility rate (67% vs. 71%, p = Fernandez H, Capmas P, Lucot JP, et al. Fertility af er ecto-
0.37). T is trial also compared the 2-year ertility rate pic pregnancy: T e DEME ER randomized trial. Hum
between salpingostomy and salpingectomy and dem- Reprod. 2013;28:124753.
onstrated no di erence (70% vs. 64%, p = 0.78). anaka , Hayashi H, Kutsuzawa , et al. reatment o intersti-
tial ectopic pregnancy with methotrexate: report o a suc-
BIBLIOGRAPHY cess ul case. Fertil Steril. 1982;37:8512.
Stovall G, Ling FW. Ectopic pregnancy: diagnostic and
Barnhart K , Sammel MD, Rinaudo PF, et al. Symptomatic therapeutic algorithms minimizing surgical intervention.
patients with an early viable intrauterine pregnancy: hCG J Reprod Med. 1993; 38: 80712.
curves rede ned. Obstet Gynecol. 2004;104(1):505.
Benson CB, Doubilet PM, Peters HE, Frates MC. Intrauterine
uid with ectopic pregnancy: a reappraisal. J Ultrasound
Med. 2013;32:38993.
95
Pelvic In ammatory Disease

Alan P. Gehrich

A 24-year-old gravida 0 caucasian emale with no sig- D. Acute abdominal series


ni cant medical or surgical history presents to the emer- E. Magnetic resonance imaging (MRI) o the abdo-
gency room with complaints o acute onset right lower men and pelvis with contrast
quadrant pain increasing over the preceding 24 hours.
She also endorses evers, nausea, and vomiting. 3. For the above patient, imaging documents a right
Her vital signs show a temperature o 100.8F, blood adnexal cystic structure 8 cm in greatest diameter
pressure 113/67 with respirations 18 and pulse 85. Per- consistent with an abscess in the right adnexa. She
tinent positives on physical exam include tenderness in is admitted or treatment. Which o the ollowing
the right lower quadrant with voluntary guarding but is the LEAST appropriate initial treatment option
no rebound and hypoactive bowel sounds. Pelvic exam or this patient?
reproduces exquisite right pelvic tenderness with a poorly A. Operative laparoscopy with drainage o the abscess
de ned pelvic mass. Laboratory evaluation includes a with intravenous ce otetan and doxycycline
negative HCG, WBC 15,000, hemoglobin and hematocrit B. Intravenous ce otetan plus doxycycline
o 13 and 39 with platelet count o 350,000. She has nor- C. Intramuscular ce riaxone and IV cipro oxacin
mal renal and liver unction tests. Imaging studies show an D. Intravenous ampicillin/sulbactam
in ammatory mass in the right lower quadrant and pelvis. E. C guided drainage with intravenous ce oxitin
and clindamycin
1. Which o the ollowing bacteria is the most
common inciting organism in the development o 4. Further treatment is dependent on the patients
acute pelvic in ammatory disease? response to therapy. Which o the ollowing
A. Staphylococcus aureus statements is correct?
B. Gonorrhea A. Sepsis is not commonly associated with a rup-
C. Escherishia Coli tured tubo-ovarian abscess but does not require
D. Peptostreptococcus sp. surgical intervention.
E. Bacteroides sp. B. Bacteroides is the most requent cause o Gram-
negative sepsis associated with tubo-ovarian abscess.
2. Which diagnostic radiologic study is the f rst line C. Antibiotics should be switched rom paren-
imaging modality to evaluate or a gynecologic teral to oral route o administration only a er
pathology in this patient? 72 hours o clinical improvement, resolution o
A. Computed tomography (C ) o the abdomen nausea, vomiting, and severe pain.
and pelvis with contrast D. I this patient does not respond to intravenous
B. rans-abdominal ultrasound antibiotics, a hysterectomy with bilateral sal-
C. rans-vaginal ultrasound pingo-oophorectomy is required or cure o a
C H AP TER 9 5 P El v i C i n f l Am m ATo Ry D i s EAs E 363

tubo-ovarian abscess ( OA) resulting rom pel- T is brinoid exudate can be seen laparoscopically
vic in ammatory disease. on any intra-abdominal organ extending to the liver
E. T e most common sequelae o OA are in ertil- edge. Necrosis inside the complex in ammatory
ity, ectopic pregnancy, and chronic pelvic pain. mass may lead to one or more polymicrobial abscess
cavities. Interestingly, C. trachomatis has never been
ANSWERS isolated rom an abscess cavity.
Classic symptoms or PID include abdominal
1. B. Pelvic in ammatory disease (PID) is a polymi- pain, abnormal discharge, inter-menstrual or post-
crobial ascending in ection that causes in ammation coital bleeding, ever, urinary urgency, low back pain,
o the upper genital tract, including endometritis, sal- nausea, and vomiting. Symptoms o OA can be
pingitis, pelvic peritonitis and in some cases leading indistinguishable rom acute salpingitis and appen-
to OA ormation. T e general surgeon can be con- dicitis. OAs generally mani est with symptoms to
sulted in cases in which an in ectious process involv- include abdominal and pelvic pain (> 90%), ever
ing the bowel such as diverticulitis or appendicitis, or (50%), vaginal discharge (28%), nausea (26%), and
an extirpative surgery is being considered with severe abnormal vaginal bleeding (21%). wenty percent o
pelvic adhesive disease involving the bowel. T e gen- OA cases, however, have normal white blood cell
eral surgeons expertise may be required in the sur- counts. T ere is no single complaint, physical exam
gical drainage o a OA, or to assist in managing a nding or laboratory nding that is highly sensitive
patient suf ering rom li e threatening sepsis arising or speci c or the diagnosis o PID, and the clinical
rom a suspected OA. diagnosis o PID has a positive predictive value o
PID is diagnosed in more than 800,000 women only 65% to 90%.
annually in the United States. Ninety percent o these
in ections are treated as outpatients. T e vast majority 2. C. T e diagnosis o OA requires the nding o
o cases are ound in the reproductive age emale less an in ammatory mass on exam or imaging. rans-
than 25 years o age. Many cases o PID, however, are vaginal ultrasound is considered the rst-line imag-
asymptomatic and there ore elude diagnosis. PID typ- ing modality or gynecologic pathology, because it
ically originates with an in ection o either Chlamydia provides excellent imaging o the upper genital tract,
trachomatis or Neisseria gonorrheoea in the lower uro- is relatively inexpensive and does not expose the
genital tract. T ese pathogens ascend the genital tract patient to radiation. C scan o the abdomen/pelvis
causing cervicitis, endometritis and salpingitis. With with contrast is pre erred in cases where the diagno-
the accompanying damage to the lower genital tract, sis is uncertain, and in which there is a concern or a
bacterial organisms, generally ound in the perianal coexisting malignancy or gastrointestinal pathology
region, can ascend through the cervix. As a result, PID such as appendicitis or diverticulitis. T e C scan has
trans orms into a poly-microbial in ection which may increased sensitivity to detect a OA (78% to 100%
ultimately mani est as a OA and sepsis. Occasionally, vs. 75% to 82%) and improved speci city (100% vs.
patients with chlamydia in ection develop perihepati- 91%) as compared to ultrasound. MRI has limited
tis (Fitzhugh-Curtis Syndrome), an in ammation o utility in the evaluation o acute pelvic pain.
the liver capsule and adjacent peritoneal sur aces.
Anaerobic and acultative aerobic bacteria, with 3. D. reatment choice or OA is dependent on many
or without N. gonorrheae and C. trachomatis, have actors. At a minimum, all patients with OA require
been isolated rom the upper genital tract o women admission and treatment with intravenous antibi-
with PID. OAs are caused by endothelial damage otics. Multiple regimens are considered acceptable
and associated edema within the allopian tube. T e and should be implemented based on the resistance
intrinsic de ense mechanisms o the upper genital pro le in the community. T ese regimens must pro-
tract become compromised leading to rapid bacte- vide broad coverage or not only N. Gonorrheae and
rial growth. As the in ection progresses, the ovary C. trichomatis, but also or Mycoplasma genitalium,
becomes involved with invasion o the ovarian Gram positive, and Gram negative acultative aer-
epithelium. Classically peritonitis is characterized obes, as well as anaerobic organisms (see able 95-1).
by brinoid exudate on the serosal sur aces o the T e cephalosporin based regimens are pre-
uterus, tubes, and ovaries leading to agglutination. erred due to improved tolerability. Clindamycin,
364 G En ERAl s U RG ERy EXAm i n ATi o n An D Bo ARD REv i EW

Table 95 1 INPATIENT PARENTERAL ANTIBIOTIC vomiting, evers, and severe pain. Patients should
REGIMENS FOR TREATMENT complete a 14 day course with oral antibiotics. T ese
OF SEVERE PELVIC INFLAMMATORY DISEASE regimens include doxycycline (100 mg BID) com-
AND TUBO-OVARIAN ABSCESS
bined with metronidazole (500 mg BID). Other regi-
Recommended Regimen: mens include amoxicillin/clavulante (875 to 2000 mg
Clindamycin 900 mg intravenously every 8 hours BID) or trimethoprim/sul amethaxazole (800/160 mg
PLUS BID) and metronidazole (500 mg BID). Drainage is
not necessary as a rst line treatment unless there is
Ce riaxone 1 g intravenously every 12 hours
concern or sepsis, which would indicate a ruptured
Substitute gentamicin or ce riaxone in patients with abscess.
Beta-lactam allergy: gentamicin loading dose 2 mg/kg
intravenously or intramuscularly ollowed by
maintenance dose (1.5 mg/kg) every 8 hours. Single 4. E. When clinical response is not achieved within
daily dosing may be substituted. 48 hours a er initiation o antibiotics, surgical man-
Alternative Regimens: agement or drainage should be strongly considered.
When drainage o a OA is indicated, options include
Ce oxitin 2 g intravenously every 6 hours
a percutaneous approach by interventional radiology,
OR or a surgical approach via either a laparoscopic or
Ce otetan 2 g intravenously every 12 hours open procedure. Unilateral salpingo-oophorectomy
PLUS may also be necessary depending on the clinical
scenario. Pelvic abscesses have been drained using
Doxycycline 100 mg intravenously or orallya every
12 hours ultrasound or C guidance with a transabdominal,
transgluteal, transrectal, or transvaginal approaches
OR
depending on the location o the abscess with suc-
Ampicillin/sulbactam 3 g intravenously every 6 hours cess rates o 77.8 to 100%.
PLUS T e decision or surgical management o a OA
Doxycycline 100 mg orally or intravenouslya every with either drainage or excision depends on the sta-
12 hours tus o the patient and the size o the abscess. When
rupture is suspected based on presentation with acute
a
May be initiated when patient is able to tolerate oral therapy to avoid abdomen or septic shock, prompt surgical interven-
phlebitis associated with parenteral doxycycline.
Source: Soper DE. Pelvic in ammatory disease. Obstet Gynecol. tion is required because o the high morbidity and
2010;116:419428. risk o mortality associated with a ruptured OA. E.
Coli is the most common isolate in women with rup-
tured OAs and a requent cause o Gram-negative
metronidazole and ce oxitin have been shown to have sepsis. Large case series have shown that antimicro-
superior abscess wall penetration and activity within bial therapy alone is usually ef ective in 70% o all
the cavity in animal models. Amnioglycosides have OAs and, in a ew o these studies, abscess size has
reduced activity in acidic, anaerobic environments been shown to be predictive o treatment success
such as those ound in abscesses. Adding ampicillin with antibiotics alone. Reed et al. showed that 35% o
to gentamicin and clindamycin nearly doubles cure abscesses 7 to 9 cm in size required surgery as com-
rates or OA rom 47% to 87.5%. Due to the high pared to almost 60% o abscess > 9 cm. DeWitt et al.
rates o resistance o E. Coli, ampicillin-sulbactam showed that abscesses > 8 cm more o en required
is no longer recommended as a single agent or the drainage or surgery. It is reasonable to observe the
treatment o community acquired intra-abdominal ef ect o IV antibiotic therapy without immediate
in ections. T e resistance o community acquired surgery in women who are not developing signs o
N. Gonorrheae to uoroquinolones has also elimi- sepsis and whose abscess is 8 cm or less in diameter.
nated this class o antibiotics as a rst or second line When clinical response is not achieved with
treatment option or PID or OA. Antibiotic therapy antibiotic therapy, surgical management should be
can be switched rom a parenteral to an oral route considered. Hysterectomy with bilateral-salpingo-
o administration a er 24 to 48 hours o clinical oophorectomy is generally not necessary but can be
improvement including resolution o nausea and considered i patient has extensive disease and has
C H AP TER 9 5 P El v i C i n f l Am m ATo Ry D i s EAs E 365

completed child-bearing. A post-menopausal patient Jaiyeoba O, Soper DE. A practical approach to the diagnosis
with a OA should be consented or total abdominal o pelvic in ammatory disease. Infect Dis Obstet Gynecol.
hysterectomy with bilateral salpingo-oophorectomy 2011;2011: Article ID 753037.
Joiner KA, Lowe BR, Dzink JL, Bartlett JG. Antibiotic levels in
due to the strong association with either gynecologic in ected and sterile subcutaneous abscesses in mice. J Infect
or intestinal malignancy. Hysterectomy with bilateral Dis. 1981;143(3):48794.
salpingo-oophorectomy has the least risk o recur- Landers DV, Sweet RL. ubo-ovarian abscess: Contemporary
rence but also the highest surgical morbidity. T e approach to management. Rev Infect Dis. 1983;5(5):87684.
long-term sequelae o PID are in ertility or ectopic McNeeley SG, Hendrix SL, Mazzoni MM, Kmak DC, Ran-
som SB. Medically sound, cost-ef ective treatment or pel-
pregnancy due to tubal scarring and chronic pelvic
vic in ammatory disease and tuboovarian abscess. Am J
pain attributed to pelvic adhesive disease. Obstet Gynecol. 1998;178(6):12728.
Mingeot-Leclercq MP, Glupczynski Y, ulkens PM. Amino-
BIBLIOGRAPHY glycosides: Activity and resistance. Antimicrob Agents Che-
Boardman LA, Peipert JF, Brody JM, Cooper AS, Sung J. Endo- mother. 1999;43(4):72737.
vaginal sonography or the diagnosis o upper genital tract Patton DL, Moore DE, Spadoni LR, Soules MR, Halbert SA,
in ection. Obstet Gynecol. 1997;90(1):547. Wang SP. A comparison o the allopian tubes response to
Chappell CA, Wiesen eld HC. Pathogenesis, diagnosis, and overt and silent salpingitis. Obstet Gynecol. 1989;73(4):622
management o severe pelvic in ammatory disease and 30.
tuboovarian abscess. Clin Obstet Gynecol. 2012;55(4):893 Reed SD, Landers DV, Sweet RL. Antibiotic treatment o tubo-
903. ovarian abscess: Comparison o broad-spectrum beta-
Dewitt J, Reining A, Allsworth JE, Peipert JF. uboovarian lactam agents versus clindamycin-containing regimens.
abscesses: Is size associated with duration o hospitaliza- Am J Obstet Gynecol. 1991;164(6 Pt 1):155661; discussion
tion and complications? Obstet Gynecol Int. 2010;2010: 15612.
Article ID 847041. Soper DE. Early recognition o serious in ections in obstetrics
Gaitan H, Angel E, Diaz R, Parada A, Sanchez L, Vargas C. and gynecology. Clin Obstet Gynecol. 2012;55(4):85863.
Accuracy o ve dif erent diagnostic techniques in mild- Soper DE. Pelvic in ammatory disease. Obstet Gynecol. 2010;
to-moderate pelvic in ammatory disease. Infect Dis Obstet 116(2 Pt 1):41928.
Gynecol. 2002;10(4):17180. Solomkin JS, Mazuski JE, Bradley JS, et al. Diagnosis and
Granberg S, Gjelland K, Ekerhovd E. T e management o management o complicated intra-abdominal in ection in
pelvic abscess. Best Pract Res Clin Obstet Gynaecol. 2009; adults and children: Guidelines by the surgical in ection
23(5):66778. society and the in ectious diseases society o America. Surg
Horrow MM. Ultrasound o pelvic in ammatory disease. Infect (Larchmt). 2010;11(1):79109.
Ultrasound Q. 2004;20(4):1719. Sutton MY, Sternberg M, Zaidi A, St Louis ME, Markowitz LE.
Hiller N, Sella , Lev-Sagi A, Fields S, Lieberman S. Computed rends in pelvic in ammatory disease hospital discharges
tomographic eatures o tuboovarian abscess. J Reprod and ambulatory visits, united states, 1985-2001. Sex Transm
Med. 2005;50(3):2038. Dis. 2005;32(12):77884.
96
Endometriosis

Alan P. Gehrich

A 26-year-old gravida 0 emale with history o chronic A. Endometriosis presents with a uni orm type o
low grade right pelvic pain presents to the emergency lesion.
room with onset o acute right lower quadrant pain over B. T e number and size o the lesions directly cor-
the course o the preceding 12 hours. T e gynecology relates with patient symptoms.
team has taken patient to the operating room to address C. Endometriosis is commonly ound outside o the
her pain with the preliminary diagnosis o ruptured pelvis.
ovarian cyst. On diagnostic evaluation, they nd severe D. Deep endometriotic lesions can become retro-
endometriosis with a large right sided endometrioma peritonealized and be di cult to appreciate lapa-
involving the appendix. T e ovary with endometrioma roscopically.
is xed to the right pelvic side wall and posterior uterus. E. Endometriosis is not associated with in ertility.
Endometriosis has completely obliterated the posterior
cul-de-sac with thick adhesions between the recto- 4. An oophorectomy plus appendectomy is
sigmoid colon and the uterus. T e gynecology service performed in the above patient. If this patient
has requested assistance rom the general surgery ser- has persistent pain post operatively consistent
vice to extricate the ovary and per orm appendectomy. with endometriosis, which therapy would be least
appropriate?
1. Which of the following is the LEAS common
A. GnRH (gonadotropin releasing hormone) ago-
symptom for endometriosis?
nist therapy
A. Dyschezia B. Oral contraceptive therapy
B. Dyspareunia C. Medroxy-progesterone acetate intramuscular
C. In ertility injection therapy
D. Dysmenorrhea D. Le oophorectomy
E. Hematochezia E. Non-steroidal anti-in ammatory therapy
2. T e most common site of intestinal endometri-
osis is: 5. If endometriosis involves the recto-vaginal septum
or bowel, which of the following statements is
A. Rectum and sigmoid
correct?
B. Appendix
C. Cecum A. Colonoscopy or rigid proctoscopy are o high
D. Small bowel yield in diagnosing bowel endometriosis.
B. ransvaginal ultrasound has a < 50% sensitivity
3. Which of the following statements is true concern- and speci city in diagnosing recto-vaginal endo-
ing the laparoscopic evaluation of endometriosis? metriosis.
C H AP TER 9 6 En d o m ETRi o s i s 367

C. Gross endometriotic lesions along the appendix lesions are highly variable in size, texture, and color.
and small bowel are common. Endometriosis can present as brotic scar tissue,
D. Excision o recto-vaginal endometriosis hemorrhagic or clear vesicles, at yellow-brown
improves ertility. lesions or the classic raised reddish-blue islands
E. Pain outcomes with bowel resection o endo- which are typically ound on the peritoneal lining
metriosis are similar to those with rectal nodule o the pelvis. Although visual inspection by an expe-
excision without bowel resection. rienced surgeon can have sensitivity o 94% to 97%
and speci city o 77% to 85%, histopathologic con-
rmation remains the standard or diagnosis. Inter-
ANSWERS estingly, neither the severity nor location o disease
correlates directly with the severity o symptoms.
1. E. T e dif erential diagnosis or acute as well as
Super cial lesions can cause pain symptoms, how-
chronic pelvic pain is broad involving the gyneco-
ever are rarely associated with adhesions. In contrast,
logical, urologic, gastrointestinal, musculoskeletal,
deep in ltrative endometriosis lesions invade into
and neurologic systems. Most commonly the general
neighboring tissue leading to signi cant pelvic adhe-
surgeon is consulted in cases o acute pelvic pain in
sive disease which can involve bowel and bladder
cases in which a gastrointestinal diagnosis such as
such as depicted in the above scenario. In rare cases
appendicitis, diverticulitis, or hernia is entertained.
this invasion is transmural leading to endometriosis
However, as in the above scenario, the general sur-
in the lumen o the bladder or bowel with associ-
geon may also be consulted to assist in the treatment
ated cyclic hematuria, dysuria, hematochezia, or dys-
o chronic pelvic pain with endometriosis generally
chezia. Endometriotic lesions, particularly DIE, can
due to severe adhesive disease involving the bowel.
become reperitonealized making it di cult to appre-
Endometriosis af ects 6% to 10% o reproductive-
ciate the extent o the lesions when observing them
aged women. T e most common presentations
laparoscopically.
include dysmenorrhea (79%), generalized pelvic
Endometriomas are accumulations o ectopic
pain (69%), and dyspareunia (45%). Bowel symp-
endometrial tissue on the ovary and have character-
toms such as constipation, diarrhea, dyschezia, and
istic ultrasound ndings. Although endometriosis is
tenesmus are present in up to 36% o patients with
associated with in ertility, the impact o endometrio-
severe endometriosis. Hematochezia originating
mas on reproductive unction is uncertain. Multiple
rom endometriosis is rare.
large, prospective studies demonstrate that asympto-
2. A. Endometriosis is de ned as hormonally respon- matic endometriomas do not negatively impact the
sive endometrial tissue ound outside o the uterus. It live birth rates associated with assisted reproductive
is ound most commonly on the ovary, uterine serosa, technologies. However, according to most studies,
in the posterior cul-de-sac along the uterosacral liga- excision o endometriomas may result in a deleteri-
ments, and in the ovarian ossae, but can be ound ous impact on surrogate markers o ovarian reserve.
throughout the peritoneal cavity. Cases o lesions in T us, endometrioma excision may be associated with
the thorax presenting as catamenial pneumothorax a worse prognosis or uture reproductive interven-
and in central nervous system have also been reported. tions compared to medical management or obser-
Rectovaginal or intestinal involvement is estimated to vation. However, this distinction, is probably only
be present in 5% to 12% o women with endometrio- critical with emale patients older than 35 years o age.
sis. Intestinal endometriosis involves the rectum and
sigmoid colon in 76% o cases, the appendix in 18% 4. D. Endometriosis can be treated both medically
and the cecum in 5%. Small bowel is rarely involved. and surgically. A patients desire or uture ertility
and the magnitude o her symptoms will help deter-
3. D. Laparoscopy is considered the gold standard mine the primary mode o therapy. Medical therapy
or the diagnosis o endometriosis with visual iden- is either analgesic or hormonal. All orms o hormo-
ti cation o endometriotic lesions. T ree histologic nal birth control have been shown to be ef ective in
subtypes o endometriotic lesions are ound in the suppressing endometriosis-associated pain. T ese
peritoneal cavity: super cial lesions, deep in ltrative include oral contraceptive pills, patches, rings, inject-
lesions (DIE), and endometriomas. Endometriotic able or implantable progestins, or the levonorgestrel
368 G En ERAL s U RG ERY EXAm i n ATi o n An d Bo ARd REVi EW

intrauterine device. T ese modalities prevent the with reasonable accuracy in making diagnosis. Gross
cyclic stimulation o the endometriosis tissue by endometriotic lesions along the appendix are rare
either inhibiting ovulation or by local endocrine but i ound should prompt appendectomy. Decision
ef ect. Second line therapy involves gonadotropin on the type o surgery or deeply invasive recto-vag-
releasing hormone agonist (GnRH) which eliminates inal endometriosis should be decided on a case-
the pulsatile GnRH signals o the hypothalamus on by-case basis. Bowel resection has been reported in
the pituitary. T is process suppresses gonadotropin numerous trials with improvement o 70% o pain
secretion rom the pituitary which in turn mark- symptoms however all report complications. Pain
edly decreases production o gonadal steroids by the outcomes rom rectal nodule excision without seg-
ovary. A potential complication o long term unop- mental bowel resection seem to be similar with less
posed therapy with a GnRH agonists is a reduction associated post-operative complications.
in bone mineral density.
T e goal o surgery is to extirpate as much o the BIBLIOGRAPHY
endometriosis as possible while reestablishing nor-
Benschop L, Farquhar C, van der Poel N, Heineman MJ. Inter-
mal pelvic anatomy. T e risks and bene ts o aggres- ventions or women with endometrioma prior to assisted
sive surgical treatment or endometriosis need to be reproductive technology. Cochrane Database Syst Rev.
care ully weighed. T e de nitive surgery or endo- 2010;(11):CD008571. doi(11):CD008571.
metriosis involves the complete removal o all endo- Berker B, Hsu HS, Lee KL, Nezhat C, Nezhat F, Nezhat C.
metriosis lesions as well as the uterus and ovaries. In Section 10.3 laparoscopic treatment o endometriosis. In:
Nezhat C, Nezhat F, Nezhat C, eds. Nezhats operative gyne-
a patient with severe, symptomatic endometriosis cologic laparoscopy and hysteroscopy. 3rd ed. Cambridge
who has completed child-bearing, a hysterectomy University Press; 2008:263303.
with bilateral oophorectomy should be considered. Duepree HJ, Senagore AJ, Delaney CP, Marcello PW, Brady
T e treatment or the patient with uture ertility KM, Falcone . Laparoscopic resection o deep pelvic
desires have to be individualized. T e most recent endometriosis with rectosigmoid involvement. J Am Coll
Surg. 2002;195(6):7548.
large scale review in the Cochrane database does not
Falcone , Lebovic DI. Clinical management o endometrio-
demonstrate that endometriosis cysts have a delete- sis. Obstet Gynecol. 2011;118(3):691705.
rious ef ect on pregnancy or live birth rates. T us, Giudice LC, Kao LC. Endometriosis. Lancet. 2004;364(9447):
surgical treatment o an endometrioma should be 178999.
reserved or those patients who are symptomatic or Hudelist G, Ballard K, English J, et al. ransvaginal sonogra-
have ndings concerning or cancer. phy versus clinical examination in the preoperative diag-
nosis o deep in ltrating endometriosis. Ultrasound Obstet
Recommended treatment involves either excision Gynecol. 2011;37(4):4807.
o the cyst wall or unilateral oophorectomy, both o Leone Roberti Maggiore U, Scala C, Venturini PL, Remorgida
which generally improve symptoms and lower risk V, Ferrero S. Endometriotic ovarian cysts do not negatively
o symptomatic recurrence as compared to simple af ect the rate o spontaneous ovulation. Hum Reprod.
drainage and electrocoagulation. Most patients with 2015;30(2):299307.
Ra F, Metwally M, Amer S. T e impact o excision o ovar-
symptomatic endometriosis, who wish to maintain
ian endometrioma on ovarian reserve: A systematic review
their ovaries, bene t rom post-operative hormonal and meta-analysis. J Clin Endocrinol Metab. 2012;97(9):
suppression in the treatment o chronic pelvic pain. 314654.
Roman H, Vassilief M, Gourcerol G, et al. Surgical manage-
5. E. T e general surgeon will most likely be involved ment o deep in ltrating endometriosis o the rectum:
in endometriosis surgery with bowel involvement as Pleading or a symptom-guided approach. Hum Reprod.
2011;26(2):27481.
depicted in the above scenario. Colonoscopy is o low Sinaii N, Plumb K, Cotton L, et al. Dif erences in character-
value in diagnosing rectovaginal endometriosis as istics among 1000 women with endometriosis based on
endometriotic lesions may invade through the serosa extent o disease. Fertil Steril. 2008;89(3):53845.
into the muscularis, but rarely into the bowel lumen. Walter AJ, Hentz JG, Magtibay PM, Cornella JL, Magrina JF.
Based on a meta-analysis by Hudelist involving 1100 Endometriosis: Correlation between histologic and visual
ndings at laparoscopy. Am J Obstet Gynecol. 2001;184(7):
patients, trans-vaginal ultrasound has a sensitiv-
140711; discussion 14113.
ity o 91% and a speci city o 98% or rectovaginal Wolthuis AM, omassetti C. Multidisciplinary laparoscopic
endometriosis. Magnetic resonance imaging (MRI) treatment or bowel endometriosis. Best Pract Res Clin
and barium enema may also be used preoperatively Gastroenterol. 2014;28(1):5367.
Neur o sur g er y
Matthew J. Martin
This page intentionally left blank
97
Back Pain/Sciatica

Albert Schuette

A 53-year-old emale presents to the emergency room C. C myelogram o the lumbar spine
with the acute onset o le leg pain while walking her dog. D. No imaging in the acute setting in the absence
T e patient did not all at the time o presentation nor is o signs o cancer, racture, or cauda equina syn-
there a history o recent trauma. T e pain starts in the drome
le buttock and radiates down the le leg to the posterior E. Plain PA and lateral X-ray o the lumbar spine
cal and oot. T e patient also describes some numbness
in the le cal and mild plantar- exion weakness. She 3. T e patient is seen in clinic 2 weeks a er the
gives a past medical history o well-controlled hyperten- initial presentation to the emergency room. She
sion and does not smoke cigarettes. A er the emergency continues to have severe pain in the le leg with
room physician provides some pain relie in the orm o a positive straight leg raise and a decreased ankle
narcotics, a consult to the surgical team is made. jerk. An MRI has been completed, which shows
a disc herniation at L5-S1 impinging on the le
1. When a physician is evaluating a patient with signs S1 nerve root. What is the typical next step in
and symptoms o sciatica, what would be picked management o the patients sciatica?
up in the history and physical that would alert the A. In orm the patient o the risks and bene ts o
physician o a more emergent disease process? surgery and schedule her or a discectomy at
A. Acute onset o urinary or bladder and bowel L5-S1.
incontinence with saddle anesthesia B. In orm her that 85% o patients improve with-
B. Steroid use out surgery within the rst 6 weeks and give her
C. Immunosuppression more time to recover.
D. IV drug use C. Recommend activity modi cation with bed rest
E. All o the above or 1 to 2 weeks.
D. Prescribe oral steroids to reduce the in amma-
2. A er the history and physical are completed, what tion around the nerve root.
should the initial imaging be in the emergency E. Recommend antidepressant therapy as there is a
room with the radicular pain and mild numbness psychological component to sciatica that should
and no history o trauma as described in the be treated.
question stem?
A. Magnetic resonance imaging (MRI) o the lum- 4. Which o the ollowing acts concerning the
bar spine without gadolinium in the emergency anatomy and prevalence o lumbar herniated discs
room is true?
B. Computerized tomography scan (C ) without A. L4-5 and L5-S1 account or about 50% o herni-
contrast o the lumbar spine ated lumbar discs.
372 G EN ERAL S U RG ERY EXAM I N ATIO N AN D BO ARD REVI EW

B. T e nerve root involved in a typical lateral recess a diagnosis and imaging already included with them.
herniated disc between L4-L5 is the nerve root When the patient rst comes to the ER on symptom
that exits at the higher level (L4). onset, the correct answer is No imaging in the f rst
C. T e nerve root involved in an extreme lateral month o symptoms. T is remains true only in the
herniated disc between L4-L5 is the nerve root absence o the red ag symptoms discussed as these
that exits at the lower level (L5). red ags aid the physician in ruling out cancer, in ec-
D. Cauda equina syndrome rom a herniated lum- tion, spinal racture, and cauda equina syndrome.
bar disc accounts or only 1% to 2% o lumbar Some red ags that should tip of a surgeon to a pos-
disc surgeries. sible malignancy are a history o cancer (the most
E. Herniated lumbar discs in the pediatric age predictive), age over 50, unexplained weight loss, and
group are quite common but in requently require a ailure to improve a er 1 month. Possible malig-
surgery. nancies include primary tumors such as osteoblas-
toma, osteosarcoma, and lymphoproli erative tumors
5. T e patient undergoes an unevent ul micro- and metastatic bone disease rom lung, thyroid, pros-
discectomy or her L5-S1 herniated disc. A er tate, kidney and colon cancer among others. I pain
surgery her strength is noted to be intact in persists, MRI is generally the imaging modality o
bilateral lower extremities throughout all muscle choice per ormed without gadolinium. In patients
groups. T e night o surgery the patient complains with a contraindication or an MRI (pacemakers,
o severe back pain, new weakness in her le etc.), a C myelogram can provide adequate visuali-
plantar- exion and urinary retention. What is the zation.
most likely cause o these new symptoms?
A. Reherniation o a disc ragment at the operative 3. B. T e patient in the question stem is only 2 weeks
level out rom onset o pain. Barring a neurologic de -
B. In ammation o the nerve root rom surgery cit or signs o cauda equina syndrome, patients are
C. Spinal epidural hematoma given time to recover rom the herniation. Over 85%
D. Damage o the nerve root rom retraction at the o herniated disc patients will improve without sur-
time o surgery gical intervention in approximately 6 weeks. Bed rest
E. Unintended durotomy causing arachnoiditis is an option or 2 to 3 days maximum in patients with
radicular complaints, as more time can be harm ul
ANSWERS or recovery. Additionally, antidepressants are more
1. E. T is is an extremely important question in evalu- indicated or chronic back pain and oral steroids
ating a patient with back pain and radiculopathy. T e have not been shown to improve symptoms.
physician must know the red ag symptoms that
4. D. T is question addresses the prevalence and
would alert him/her as to a more severe process. All
anatomy o herniated discs in the lumbar spine.
o the answers in this section provide a clue to those
L4-5 and L5-S1 account or over 90% o herniated
syndromes. Answer A, acute onset o urinary or blad-
lumbar discs. When examining the lumbar spine,
der and bowel incontinence with saddle anesthesia,
the nerve root that exits between L4 and L5 is the
re ers to cauda equina syndrome (CES). T ough
L4 root. Most discs herniate into the lateral recess,
there is controversy as to whether the patient should
which is medial to the exiting nerve root. In this way,
go to surgery within 24 or 48 hours, it remains a sur-
the disc impinges on the nerve root on deck (see
gically urgent case. Answer B, steroid use, could put a
Figure 97-1). T is is an image o a L5-S1 disc hernia-
patient at risk or both an in ectious or a minor trau-
tion. T e vertical arrow indicates the disc herniation
matic process. Answer C, immunosuppression, puts
in the lateral recess compressing the S1 nerve root.
the patient at risk or an in ectious process such as
T e horizontal arrow indicates the L5 nerve root in
osteomyelitis or epidural abscess. Finally, answer D,
the neuro oramen.
IV drug use, would also put the patient at high risk
An L4-5 disc herniation in the lateral recess would
or epidural abscess or osteomyelitis.
cause pain in an L5 distribution. An extreme lateral
2. D. T is can be a di cult answer to arrive at or sur- disc at L4-5, on the other hand, would impinge on
geons. In most cases, patients come to the clinic with the exiting nerve root, L4. It is important to realize
C H AP TER 9 7 BAC k PAI N / S C I ATI C A 373

are o en related to reherniation or instability in the


spinal segment. T ese complications can present
with recurrence o the patients leg pain, pain in the
back or buttocks, and weakness/ numbness in the dis-
tribution o the compressed nerve root.

BIBLIOGRAPHY
Allen C, Glasziou P, Del Mar C. Bed rest: A potentially harm-
ul treatment needing more care ul evaluation. Lancet.
October 9, 1999;354(9186):122933.
Chau AM, Xu LL, Pelzer NR, Gragnaniello C. iming o
surgical intervention in cauda equina syndrome: A sys-
tematic critical review. World neurosurgery. March-April
2014;81(3-4):64050.
Davis RA. A long-term outcome analysis o 984 surgically
treated herniated lumbar discs. Journal of neurosurgery.
March 1994;80(3):41521.
Deyo RA, Rainville J, Kent DL. What can the history and
physical examination tell us about low back pain? JAMA.
Figure 97-1 L5-S1 disc herniation: the vertical arrow indi- August 12, 1992;268(6):7605.
cates disc herniation in the lateral recess compressing the S1 Downie A, Williams CM, Henschke N, et al. Red ags to screen
nerve root. T e horizontal arrow indicates the L5 nerve root or malignancy and racture in patients with low back pain:
in the neuro oramen. systematic review. Bmj. 2013;347: 7095.
Ebersold MJ, Quast LM, Bianco AJ, Jr. Results o lumbar dis-
cectomy in the pediatric patient. Journal of neurosurgery.
that cauda equina syndrome is uncommon account- November 1987;67(5):6437.
ing or only 1% to 2% o all herniated disc surger- Fager CA. Observations on spontaneous recovery rom inter-
ies. Also, herniated lumbar discs in the pediatric age vertebral disc herniation. Surgical neurology. October
1994;42(4):2826.
group are quite rare as stated. Graves JM, Fulton-Kehoe D, Jarvik JG, Franklin GM. Early
imaging or acute low back pain: One-year health and dis-
5. C. When per orming or providing care or a patient ability outcomes among Washington State workers. Spine.
who underwent a discectomy, it is extremely impor- August 15, 2012;37(18):161727.
tant to per orm thorough neurologic tests. Compli- Greenberg MS, ed. Handbook of Neurosurgery. 7th ed. New
cations associated with a discectomy can include a York, NY: T ieme Medical Publishers; 2010.
reherniation o the disc (up to 4% over 10 years), Henschke N, Maher CG, Ostelo RW, de Vet HC, Macaskill P,
Irwig L. Red ags to screen or malignancy in patients
temporary worsening o motor unction (1% to 8% o with low-back pain. T e Cochrane database of systematic
patients3), and unintended durotomy (0% to 14% o reviews. 2013;2:CD008686.
cases). All o these may be a cause o temporary wors- Hodges SD, Humphreys SC, Eck JC, Covington LA. Man-
ening neurologic unction. Any new neurologic de - agement o incidental durotomy without mandatory bed
cit in a patient ollowing surgery should be assumed rest. A retrospective review o 20 cases. Spine. October 1,
1999;24(19):20624.
to be a spinal epidural hematoma until proven oth-
Webster BS, Bauer AZ, Choi Y, Ci uentes M, Pransky GS. Iat-
erwise. Work-up should include an immediate MRI rogenic consequences o early magnetic resonance imag-
and return to the operating room. Long-term com- ing in acute, work-related, disabling low back pain. Spine.
plications o microdiscectomy are uncommon and October 15, 2013;38(22):193946.
This page intentionally left blank
Or t h o pedic Sur ger y
Matthew J. Martin
This page intentionally left blank
98
Calcaneus Fractures

Justin T. Fowler and Justin Robbins

A 27-year-old pro essional painter sustained a all rom C. Non-contrast computerized tomography (C )
a een oot sca old while on the job. By his report, scan with reconstructions
he landed on both eet in a standing position and had D. riple phase bone scan
immediate severe oot and leg pain and was unable to E. None o the above. Adequate in ormation is pro-
ambulate. His primary survey is unremarkable and his vided by the radiographs.
secondary survey is normal except or the examination
o his right oot. He presents to the emergency depart-
ment with a swollen and de ormed right oot and com-
plains o severe pain at rest and with any palpation
or manipulation o the oot. Radiographs o the right
oot reveal a comminuted, intra-articular calcaneus
racture.

1. Plain radiographs o the patients right oot


are shown (Figure 98-1). What other associated
injury is classically described rom this type o
mechanism ( all rom height landing on both eet)
and associated with the presence o this injury?
A. Closed head injury
B. Hollow viscous injury
C. Pelvic racture
D. Lumbar spine racture
E. Concomitant oot ractures

2. Anteroposterior, lateral, and oblique radiographs


are per ormed and reveal a displaced calcaneus
racture. What other imaging modality should be
ordered to urther delineate the injury?
Figure 98-1 Lateral oot radiograph revealing a displaced,
A. Non-contrast magnetic resonance imaging intra-articular calcaneus racture with depression o the
(MRI) posterior acet. Radiographs courtesy o MAJ Justin Fowler
B. Weight-bearing oot lms MD.
378 G EN ERAL S U RG ERY EXAM I N ATI O N AN D BO ARD REVI EW

3. Several hours have gone by while awaiting urther T e other injuries listed can occur with any signi -
work up and disposition o the patient and you cant trauma mechanism, but are all less commonly
notice that the skin overlying the Achilles tendon associated with calcaneus and other ractures o the
is becoming dark and appears to be tented by the oot. Any patient with this type o mechanism should
racture. What is the next most appropriate course be suspected o having a lumbar spine racture until
o action? proven otherwise, and the optimal diagnostic study
A. Emergent reduction in the emergency room is a dedicated C scan o the lumbar spine.
B. Emergent orthopaedic consultation
2. C. Initial diagnosis o a calcaneus racture can be
C. Placement o a well-padded splint and strict
readily made with conventional plain X-rays o the
elevation
oot (Figure 98-1). T e simplest broad classi cation
D. C angiography o the lower extremity
o calcaneus ractures is based on whether there is
E. Emergent compartment pressure check o the
articular involvement, with 75% o ractures being
oot
intra-articular, and approximately 25% being extra-
4. What is the pre erred treatment or a displaced, articular. Although the initial diagnosis can o en
intra-articular racture o this bone in a young, be made with standard X-rays, a high-quality C
active patient? o the hind oot is the de nitive imaging modality
o choice or diagnosis, classi cation, and operative
A. Well-padded splint with conversion to a short
planning. T e C scan should be per ormed with 2
leg cast
to 3 mm interval slices in the sagittal, axial, and 30
B. Closed reduction and casting
degree semi-coronal planes (Figure 98-2). T is imag-
C. Percutaneous screw xation
ing modality allows or accurate characterization o
D. Emergent open reduction internal xation
the racture pattern and acilitates planning or the
E. Delayed open reduction internal xation
patients de nitive treatment. A C scan is use ul in
5. Which o the ollowing is predictive o a better assessing the amount and location o joint impaction,
outcome a er open reduction and internal f xation lateral wall blowout, involvement o calcaneocuboid
o a displaced, intra-articular racture o this type? joint, and the number o primary racture lines in the
posterior acet. An MRI has little additional utility
A. Female gender
compared to a high-quality C scan or diagnosing,
B. Signi cant articular impaction
characterizing, or directing treatment plans or cal-
C. Workers compensation
caneus ractures. Similarly, additional X-rays includ-
D. Age > 50
ing weight-bearing lms will not provide su cient
E. Heavy laborer

ANSWERS
1. D. T is patient has a calcaneus racture. T e associ-
ated injuries, speci cally lumbar spine injuries, have
been historically reported in up to 50% o patients.
T e most common mechanism o injury or calca-
neus ractures has been a all rom a height with the
patient initially striking the ground with their eet
in a standing or semi-standing position. T is results
in the calcaneus racture, but also the transmis-
sion o impact orces up the spine, with the lumbar
region bearing most o the burden. As motor vehi-
cle collisions have become an increasingly prevalent
mechanism, newer data has suggested that roughly
10% o patients with calcaneus racture will have a
concomitant lumbar spine injury and approximately Figure 98-2 Coronal C scan showing the articular involve-
25% have other additional lower extremity injuries. ment o this impacted calcaneus racture.
C H AP TER 9 8 C ALC AN EU S F RAC TU RES 379

anatomic details, and will also likely be poorly toler- surgical site complications, poor wound healing, so
ated by the patient with a calcaneus racture. tissue loss, and hardware in ections.

3. B. Displaced avulsion ractures o the posterior 5. A. A prospective, randomized, controlled trial o


tuberosity with impending skin necrosis require non-operatively treated versus operatively treated
immediate reduction to prevent ull thickness skin calcaneus ractures rom our Canadian trauma
loss over the posterior ankle and Achilles tendon. centers revealed that certain patient demograph-
Emergent orthopaedic surgical consultation should ics were predictive o better versus worse outcomes
be obtained as closed reduction typically is not a er treatment o displaced intra-articular calcaneus
adequate to remove tension rom the skin. T is is ractures. Women treated with operative xation had
typically treated as an orthopaedic emergency and signi cantly higher quality o li e scores than women
usually requires surgical reduction and stabilization who were managed non-operatively. T e study also
in the operating room in an acute ashion. showed that heavy laborers, workers compensation
T e ankle can be splinted in plantar-f exion in cases, older age, higher degree o articular commi-
an attempt to take pressure o the overlying skin nution, and smoking are all predictors o worse out-
while awaiting surgical consultation. Placement o a comes a er operative xation. T e exact reason or
splint will not provide any bene t, and in act could this gender disparity remains unknown, but these
produce additional pressure necrosis on the area i actors should be taken into consideration when plan-
not tted well or i there is additional swelling a er ning a management strategy and in counseling the
the splint is placed. C angiography is only use ul i patient regarding the risks versus bene ts o operative
there is a suspicion o a concomitant vascular injury versus non-operative management.
in the ractured extremity, and has no role or evalu-
ation o oot ractures with a normal vascular exam.
Emergent compartment pressures are indicated i BIBLIOGRAPHY
there is suspicion o a compartment syndrome, but Benirschke SK, Kramer PA. Wound healing complications
in this scenario the primary problem is the displaced in closed and open calcaneal ractures. J Orthop Trauma.
racture and not an elevated compartment pressure. 2004;18(1):16.
Buckley R, ough S, McCormack R, Pate G, Leighton R, Pet-
4. E. Intra-articular displacement o the calcaneus has rie D, Galpin R. Operative compared with non-operative
treatment o displaced intra-articular calcaneal ractures:
been shown to lead to subtalar arthrosis, sub bular a prospective, randomized, controlled multicenter trial.
impingement, di culty with shoe wear, hind oot J Bone Joint Surg Am. 2002;84(10):173344.
sti ness, and limited unction. I the articular sur ace Buckley RE, ough S. Displaced intra-articular calcaneal rac-
is amenable to reconstruction in a young, healthy tures. J Am Acad Orthop Surg. 2004;12(3):1728.
patient, then surgical xation should be attempted. Cave EF. Fracture o the oscalcisthe problem in general. Clin
Orthop Relat Res. 1963;30:646.
T e status o the so tissue envelope dictates the tim-
Gilmer PW, Herzenberg J, Frank JL, Silverman P, Martinez S,
ing o surgical intervention. raditionally, delayed x- Goldner JL. Computerized tomographic analysis o acute
ation through an extensile lateral approach has been calcaneal ractures. Foot Ankle. February 1986;6(4):18493.
the mainstay treatment or these ractures. T ese Koski A, Kuokkanen H, ukiainen E. Postoperative wound
patients will typically have a signi cant amount o complications a er internal xation o closed calcaneal
swelling, bruising, and so tissue edema o the oot ractures: a retrospective analysis o 126 consecutive
patients with 148 ractures. Scand J Surg. 2005;94(3):2435.
which can result in poor wound healing i surgery is Protheroe K. Avulsion ractures o the calcaneus. J Bone and
per ormed during the acute phase. Open reduction Joint Surg Br. 1969;51(1):11822.
and internal xation is typically delayed to allow this Sanders RW, Clare MP. Fractures o the Calcaneus. Surgery of the
so tissue swelling to subside and mitigate the risk o Foot and Ankle. Philadelphia, PA: Mosby; 2007: 20172075.
99
Shoulder rauma

Kelly G. Kilcoyne

A 24-year-old male presents as an acute trauma a er C. Femur racture


injury in a motorcycle crash. On initial presentation D. Arterial injury
he is a GCS 11 (E3V3M5). Because o severe pain and E. Brachial plexus injury
labored breathing, he is intubated and sedated. During
your secondary survey, you note the patient has sig- 4. Upon further clinical evaluation you note that
ni cant edema, swelling, and abrasions about the right the patient has a prominence along the anterior
shoulder girdle. aspect of his shoulder and a sulcus adjacent to
the posterior acromion. He is still not following
1. What serious, and potentially devastating, commands, but examination with gentle range
diagnosis is characterized by lateral displacement of motion demonstrates limitations in internal
of the scapula? rotation and abduction. What is your next step in
A. Pnuemothorax management?
B. Multiple rib ractures A. Attempt a closed reduction.
C. Scapulothoracic dissociation B. Obtain a chest C .
D. Shoulder dislocation C. Schedule an outpatient magnetic resonance
E. Sternoclavicular dislocation image (MRI).
D. Obtain orthogonal X-rays.
2. What study can you evaluate or obtain to further E. Diagnosis should be made based on physical
investigate and diagnose the suspected injury in exam.
Question 1?
A. Ventilationper usion (VQ) scan 5. What is the most common neurovascular injury
B. External rotation anterior-posterior (AP) shoul- associated with an anterior shoulder dislocation?
der X-ray A. Musculocutaneous nerve
C. Axillary X-ray B. Axillary nerve
D. Internal rotation AP shoulder X-ray C. Axillary vein
E. Chest X-ray D. Subclavian artery
E. Radial nerve
3. While reviewing a chest CT obtained as part of the
initial workup, you note a displaced right scapular
fracture. What additional injuries are most ANSWERS
commonly associated with scapular fractures? 1. C. Scapulothoracic dissociation results rom com-
A. Pulmonary injury plete disruption o the scapulothoracic articulation
B. Head Injury with associated lateral translation o the scapula
C H AP TER 9 9 S H o u l d ER TRAu m A 381

without associated partial or complete amputation o or scapular-index, can be measured. T e distance


the so tissue, with the overlying skin typically intact. rom the sternal notch or midline o the spine to
Scapulothoracic dissociation typically results rom the glenoid or medial border o the scapula can be
a high energy, traction orce, to the upper extrem- measured and compared with the contralateral side.
ity. Motorcycle accidents are the most commonly T e injured side measurement is divided by the non-
described mechanism in the literature. Considered injured side measurement. T e normal scapular-
analogous to a closed orequarter amputation, it is index is about 1.09, and in one o the largest series,
associated with a myriad o concomitant injuries the ratio in patients with a scapulothoracic dissocia-
including: dislocation o the acromioclavicular and tion averaged 1.29.
sternoclavicular joints, clavicle racture, vascular
injury to the subclavian and axillary vessels, brachial 3. A. In a review o 58 scapula ractures as a result o
plexus injuries, and complete or partial disruption o blunt trauma T ompson et al ound a high rate o
the surrounding musculature. concomitant injury to the ipsilateral lung, chest wall,
T e clinician must maintain a high index o sus- and chest contents. Fi y-three percent o patients
picion as patients are o en obtunded, sedated, or had a pulmonary contusion, 53% rib ractures, 26.8%
intubated during initial evaluation, preventing a clavicle ractures. Neurovascular injury was not
complete neurological examination. Additionally, uncommon, with a incidence o brachial plexus
because these injuries are associated with a high injury o 12.5% and a 10.7% incidence o subclavian,
energy mechanism and severe and potentially li e brachial, or axillary artery injury.
threatening concomitant injuries, scapulothoracic Baldwin et al. controlled or injury severity score
dissociation can be missed or overlooked, without in order to determine i commonly associated inju-
appropriate and complete evaluation. ypically these ries with scapula ractures were simply a result o an
patients will have signi cant swelling and edema o increased injury severity score. T ey showed the ol-
the entire shoulder girdle rom neck to axilla rom lowing injuries had increased requency in patients
underlying hematoma. Evaluation and comparison with a scapula racture compared to those without:
o pulses o the upper extremity should be promptly rib ractures, pneumothorax, lung injury, ipsilateral
per ormed, and i absent or uneven, urther studies extremity injury, and spine injury.
including arterial pressure indexes, and i abnormal A scapula racture, with a high energy blunt
(< 0.9), arteriogram should be obtained. mechanism o injury, should alert the clinician to the
With the low incidence o injury, outcomes studies possibility o additional injury to the ipsilateral lung,
are limited. However, previous studies have suggested chest wall, shoulder girdle, and surrounding neuro-
a mortality rate o roughly 20% (3 o 15 patients), vascular structures.
though the true rate is likely higher as a result o
death rom associated injuries in these patients. 4. D. Based on the clinical exam ndings present, the
Associated neurovascular injury is extremely com- patient has a suspected shoulder dislocation. Anterior
mon, with neurologic injury in 94% o patients and shoulder dislocations are by ar the most common,
vascular injury in 88%. In addition to a mortality rate accounting or approximately 98% o all gleno-
between 10% to 20%, outcomes are universally poor humeral dislocations. T e typical mechanism or an
with 52% rate o ail extremity and early amputation anterior dislocation is with the arm in an abducted
rate o 21%. Early recognition o scapulothoracic and externally rotated position. Posterior disloca-
injury, and likely associated neurovascular injury, is tions are relatively rare, and result rom axial load to
paramount as diagnosis guides treatment and e ects an adducted, exed and internally shoulder, and can
long term outcomes. also be associated with electrical shocks or seizures.
An acutely dislocated shoulder is typically very
2. E. T e diagnosis o scapulothoracic dissociation can pain ul. T e patient with an anterior dislocation will
be made based on a non-rotated chest X-ray. Lat- typically hold the arm in slight abduction and range
eral displacement o the scapula is pathognomonic. o motion will be limited secondary to pain.
Comparison o the scapular position can be made Radiographs are an essential part o the ini-
with the contralateral side, and di erences and asym- tial assessment, ensuring appropriate diagnosis
metry noted. T e amount o scapular lateralization, including the direction o dislocation, presence o
382 G EN ERAl S u RG ERY EXAm I N ATIo N AN d Bo ARd REVI EW

associated ractures, and potential blocks to reduc- BIBLIOGRAPHY


tion. T e standard shoulder trauma series must Baldwin KD, Ohman-Strickland P, Mehta S, et al. Scapula
include orthogonal X-rays including a true AP, axil- ractures: A marker or concomitant injury? A retrospec-
lary lateral and scapular Y view. Obtaining only a tive review o data in the National rauma Database.
single view o the shoulder can lead to missed injury J rauma. 2008;65:4305.
Brucker PU, Gruen GS, Kau mann RA. Scapulothoracic
and inaccurate diagnosis. C can help aid in diagno- dissociation: Evaluation and management. Injury.
sis i appropriate imaging cannot be obtained. 2005;36:114755.
A er appropriate diagnosis is made, a reduction Damschen DD, Cogbill H, Siegel MJ. Scapulothoracic disso-
can be per ormed. T ere are several classic meth- ciation caused by blunt trauma. J rauma. 1997;42:53740.
ods or relocation including the Kocher, traction- de Laat EA, Visser CP, Coene LN, et al. Nerve lesions in pri-
mary shoulder dislocations and humeral neck ractures. A
counter traction, Stimson and Milch to name a ew.
prospective clinical and EMG study. J Bone Joint Surg Br.
Newer studies and techniques ocus on limiting pre- 1994;76:3813.
medication. Recent meta-analysis have shown equiv- Eachempati KK, Dua A, Malhotra R, et al. T e external rota-
alent e ectiveness using an intra-articular lidocaine tion method or reduction o acute anterior dislocations
injection instead o intravenous analgesia and seda- and racture-dislocations o the shoulder. J Bone Joint Surg
tion or manual acute closed reduction o anterior Am. 2004;86-A:24314.
Ebraheim NA, An HS, Jackson W , et al. Scapulothoracic dis-
shoulder dislocation with decreased risk o post pro- sociation. J Bone Joint Surg Am. 1988;70:42832.
cedure complications. Flanagin BA, Leslie MP. Scapulothoracic dissociation. Orthop
Clin North Am. 2013;44:17.
5. B. T e axillary nerve (C5,C6) is a direct continuation Jiang N, Hu YJ, Zhang KR, et al. Intra-articular lidocaine ver-
o the posterior cord o the brachial plexus. Because sus intravenous analgesia and sedation or manual closed
it is tethered, anterior and posterior, to the gleno- reduction o acute anterior shoulder dislocation: An
humeral joint with limited excursion, it is susceptible updated meta-analysis. J Clin Anesth. 2014;26:3509.
Kelbel JM, Jardon OM, Huurman WW. Scapulothoracic dis-
to injury rom shoulder dislocation. In a study o 105 sociation. A case report. Clin Orthop Relat Res. 1986;2104.
patients with acute primary anterior shoulder dislo- Krieg JC, Green A, Cole PA. In: Schmidt AH, eague DC, eds.
cations, 21% o patients (22/107) were ound to have Orthopedic Knowledge Update 4: Shoulder rauma. 1st ed.
a nerve injury, the most common (n = 13) involving Rosemont, IL: American Academy o Orthopedic Sur-
injury to the axillary nerve. T e authors o this study geons; 2010:18194.
Nagi ON, Dhillon MS. raumatic scapulothoracic dissociation.
note that the incidence o nerve injury may have been
A case report. Arch Orthop rauma Surg. 1992;111:3489.
underestimated as it was diagnosed by clinical exam- Oreck SL, Burgess A, Levine AM. raumatic lateral displace-
ination alone, not with electrophysiologic testing. ment o the scapula: A radiographic sign o neurovascular
In a prospective study o 101 consecutive patients disruption. J Bone Joint Surg Am. 1984;66:75863.
with anterior shoulder dislocation or humeral neck Robinson CM, Aderinto J. Posterior shoulder dislocations and
racture, de Laat et al. ound electrophysiologic evi- racture-dislocations. J Bone Joint Surg Am. 2005;87;639
50.
dence o nerve injury in 45% o patients. T e axillary Rockwood CA, ed. T e Shoulder. Philadelphia, PA: Saunders
nerve was most commonly injured (37%) ollowed Elsevier; 2009.
by the suprascapular nerve (29%) and the muscu- Rubin SA, Gray RL, Green WR. T e scapular Y: A diagnostic
locutaneous nerve (22%). aid in shoulder trauma. Radiology. 1974;110:7256.
Initial diagnosis o a nerve injury can be dif cult te Slaa RL, Wij els MP, Brand R, Marti RK. T e prognosis
ollowing acute primary glenohumeral dislocation. J Bone
secondary to pain. Detailed initial exam and assess-
Joint Surg Br. 2004;86:5864.
ment, additional ollow up or repeat examination, T ompson DA, Flynn C, Miller PW, et al. T e signi cance o
or electrophysiologic testing, can help detect injury scapular ractures. J rauma. 1985;25:9747.
early, and potentially prevent long term poor unc- Zelle BA, Pape HC, Gerich G, et al. Functional outcome ol-
tional outcome. lowing scapulothoracic dissociation. J Bone Joint Surg Am.
2004;86(1):28.
100
Pediatric Musculoskeletal rauma

Kelly G. Kilcoyne

While on your pediatric rotation, you are called to a D. Cervical spine radiographs are indicated as a
trauma code in the emergency room. T e patient is a result o the mechanism o injury and potential
6-year-old emale who was restrained in the back seat distracting injuries o her le leg and right elbow.
o a car when it lost control and went over an embank- E. Cervical spine radiographs are not indicated, but
ment. When you arrive, the child is conscious and a magnetic resonance imaging (MRI) o her cer-
breathing spontaneously. Initial vitals are stable and she vical and thoracolumbar spine is.
is ollowing commands. T e child is in signi cant pain,
but is able to communicate to you that her right elbow 3. Upon urther evaluation o her right elbow, you
and le leg hurt. notice an obvious de ormity. What is your initial
step in management o this potential injury?
1. With regards to pediatric trauma surgery patients, A. Obtain labs to rule out in ection.
compared to adult trauma patients, the ormer B. Obtain a computerized tomography (C ) scan
has: to evaluate potential racture.
A. Small head:body ratio C. Per orm a care ul neurovascular examination.
B. Larger total blood volume D. Per orm a closed reduction, then obtain X-rays.
C. Decreased capacity or plastic de ormation E. Provide sedation to allow evaluation o stability
D. Increased baseline metabolic rate combined with and range o motion.
a larger physiologic reserve
E. Smaller sur ace area to body volume 4. Radiographs o the le emur demonstrate a
displaced midsha racture. How would the
2. A er your initial assessment you begin to order workup o this emur racture dif er, i at all, i the
labs and imaging. In consideration o cervical child was a non-ambulatory 11 month old, injured
spine evaluation, you make the ollowing decision: as a result o a all?
A. Cervical spine radiographs are not indicated but A. Obtain labs to rule out in ection.
lumbar spine radiographs are, because lumbar B. Obtain a C scan to urther evaluate the racture.
spine injuries are more common in children. C. Obtain inlet and outlet pelvic X-rays.
B. Cervical spine radiographs are indicated because D. T e child would need to be evaluated or pos-
she may have lost consciousness. sible child abuse.
C. Cervical spine radiographs are not indicated E. T e workup would be unchanged.
because children are unlikely to sustain injury to
the cervical spine as a result o their unique osse-
ous anatomy.
384 G EN ERAL S U RG ERY EXAM IN ATIO N AN D BO ARD REVI EW

ANSWERS Lee et al. developed a multidisciplinary approach


to pediatric cervical spine injury evaluation and
1. D. T ere are many important di erences in the clearance. T e authors use the presence o any o
anatomy, physiology, mechanism, and characteristics the ollowing as a reason or immobilization and
o injuries that make pediatric trauma unique. T e radiographic evaluation; child is unconscious or
proportions o a childs body are di erent rom those inconsolable, mechanism o injury suggestive o
o an adult, predisposing to certain injuries. Children possible cervical spine injury (motor vehicle crash,
have a large head to body ratio; the younger the child all rom height, pedestrian struck, etc.), neck pain
the more disproportionate the ratio. T eir relatively or ocal neck tenderness, presence o a distracting
larger heads predispose them to head, neck, and injury, abnormal neurologic exam ndings or his-
upper cervical spine injuries. tory o transient neurologic symptoms that suggest
Pediatric bone growth and development is incom- c-spine injury, physical signs o neck trauma, sig-
plete, with bone more de ormable and able to rac- ni cant trauma to head and or ace, or an unreliable
ture with (relatively) less orce. T ese characteristics exam secondary to substance abuse. In short, in the
can lead to internal organ damage without apparent multiply injured patient, a cervical spine is presumed
injury or racture o the thoracic rib cage. Addition- present and must be ruled out by physical exam and
ally, the immature cage leaves the spleen and liver radiographic evaluation.
exposed and vulnerable to injury.
Children have a smaller total blood volume 3. C. Pediatric elbow injuries present a challenge to the
(80 mL/kg) than adults.As a result, equivalent or small- treating physician because diagnosis, physical exam-
volume blood loss can have more signi cant physi- ination, radiographic evaluation and interpretation,
ologic consequences, with hypovolemia developing and surgical treatment can all be dif cult. Addition-
more rapidly. Additionally, because o a higher base- ally, complications, both acute and chronic, are not
line metabolic rate and increased physiologic reserve, uncommon. Un ortunately, while ractures around
the apparent hemodynamic response to trauma may the elbow can be dif cult, they are also quite com-
be minimal. Additionally, children have a more mini- mon. Elbow injuries in children occur more com-
mal metabolic response to trauma, and hypotension monly than in adults, and account or approximately
is o en a late sign o shock in these patients. T ey 30% o all extremity ractures in patients age 0 to
are also more prone to hypothermia because o an 7 years. Injuries to the upper extremity account or
increased ratio o body sur ace area to volume. 65% o all ractures and dislocations in children, with
ractures o the distal end and orearm being most
2. D. T e pediatric spine, especially in children under common, ollowed by ractures and dislocations o
8 years old, has many important anatomic and bio- the elbow. O all pediatric elbow ractures, supracon-
mechanical di erences that predispose it to cervical dylar ractures are the most common.
injury. As previously stated, children have a larger While physical examination in the uncoopera-
head to neck ratio. Additionally, in early childhood, tive child with a swollen elbow can be challenging,
the ulcrum o exion is at the C2-C3 level, com- it is absolutely essential, and should be the rst
pared to C5-C6 by age 11. T is too, predisposes to step in treatment. T e two most important aspects
upper cervical spine injury. Pediatric cervical spine o the exam are the neurovascular assessment and
trauma may present as racture, racture with sub- the presence o any so tissue injury. Examination
luxation, subluxation alone (without racture), or and documentation o neurovascular status must
spinal cord injury without radiographic abnormal- be completed prior to any reduction, and ollow
ity (SCIWORA). T e incidence o SCIWORA and and reduction maneuver, to assess or any interval
isolated so tissue injury alone are more common in change. For supracondylar distal humerus ractures
children < 9 years old. As a result o these, and other, speci cally, neurologic injury is relatively common,
complicating actors, the assessment and clearance ranging rom 10% to 18%.
o pediatric patients with potential spine injuries is Supracondylar humerus ractures requiring xa-
challenging. Incomplete and inaccurate assessment tion were previously thought to be a surgical emer-
can result in missed cervical spine injuries while pro- gency. However, there has been a shi in thinking
longed can be associated with signi cant morbidity. that those ractures without vascular compromise,
C H AP TER 1 0 0 P ED IATRIC M U S C U LO S k ELETAL TRAU M A 385

severe so tissue swelling, or pressure on the skin Dormans JP, Squillante R, Shar H. Acute neurovascular com-
can wait (i.e., wait until morning) until an OR team plications with supracondylar humerus ractures in chil-
amiliar with this operation is available. Again, this dren. J Hand Surg Am. 1995;20:14.
Eleraky MA, T eodore N, Adams M, et al. Pediatric cervical
decision hinges and underlines the importance o spine injuries: report o 102 cases and review o the litera-
accurate and complete physical examination. How- ture. J Neurosurg. 2000;92:1217.
ever, these patients must be splinted and admitted or Green NE, Van Zeeland N. Fractures and Dislocations About
observation with continued neurovascular checks the Elbow. In: Green N, Swiontkowski MF, eds. Skeletal
until surgery can sa ely be per ormed. Trauma in Children. 4th ed. Philadelphia, PA: Elsevier;
2009:207282.
4. D. Pediatric diaphyseal emur racture rates continue Hanlon CR, Estes WL, Jr. Fractures in childhood: A statistical
analysis. Am J Surg. 1954;87:31223.
to increase, accounting or 1.4% to 1.7% o all pedi- Hinton RY, Lincoln A, Crockett MM, et al. Fractures o the
atric ractures. T ey are the most common reason or emoral sha in children. Incidence, mechanisms, and
hospitalization in a pediatric patient with orthopedic sociodemographic risk actors. J Bone Joint Surg Am.
injury. reatment o these ractures is largely depend- 1999;81:5009.
ent on patient age: in ants, 6 months to 5 years, 5 to 11 Jones M, Anderson PA, Noonan KJ. Pediatric cervical spine
trauma. J Am Acad Orthop Surg. 2011;19:60011.
year olds, and patients age 11 to skeletal maturity. T e
Kocher MS, Sink EL, Blasier RD, et al. reatment o pediat-
child in our example would likely be treated opera- ric diaphyseal emur ractures. J Am Acad Orthop Surg.
tively with exible intramedullary nails. 2009;17:71825.
T e treating physician o a child < 36 months old Lee SL, Sena M, Greenholz SK, et al. A multidisciplinary
with a emur racture has to consider, and rule out, approach to the development o a cervical spine clearance
non-accidental trauma as the cause o injury, accord- protocol: Process, rationale, and initial results. J Pediatr
Surg. 2003;38:35862; discussion 35862.
ing to the Academy o Orthopedic Surgeons clinical Lichtenberg RP. A study o 2,532 ractures in children. Am J
guidelines. T e recommendation is based on Level Surg. 1954;87:3308.
II studies which reported that 14% o emur rac- McCartney D, Hinton A, Heinrich SD. Operative stabiliza-
tures were the result o child abuse in children aged tion o pediatric emur ractures. Orthop Clin North Am.
0 to 1 year, and 12% in patients aged 0 to 3 years. 1994;25:63550.
Mohseni S, alving P, Branco BC, et al. E ect o age on cervi-
T is evaluation should include a complete history
cal spine injury in pediatric population: a National rauma
and physical, direct communication with the childs Data Bank review. J Pediatr Surg. 2011;46:17716.
pediatrician or physician, and i available, consulta- Osenbach RK, Menezes AH. Pediatric spinal cord and verte-
tion with a child abuse team. Additionally, a selective bral column injury. Neurosurgery. 1992;30:38590.
skeletal survey should be ordered i elt appropriate Otsuka NY, Kasser JR. Supracondylar Fractures o the Humerus
by the treating physician. in Children. J Am Acad Orthop Surg. 1997;5:1926.
Rewers A, Hedegaard H, Lezotte D, et al. Childhood emur rac-
T e true incidence o child abuse and related rac- tures, associated injuries, and sociodemographic risk actors:
tures is likely underestimated as a result o underre- A population-based study. Pediatrics. 2005;115:e543552.
porting. T e consequences o missing a case o abuse Sahlin Y. Occurrence o ractures in a de ned population: A
can result in serious complications including contin- 1-year study. Injury. 1990;21:15860.
ued abuse or death. Skaggs DL. Elbow Fractures in Children: Diagnosis and Man-
agement. J Am Acad Orthop Surg. 1997;5:30312.
Weisman DS, Frick SL. CH 44 Problematic Pediatric Frac-
BIBLIOGRAPHY
tures in textbook Orthopaedic Knowledge Update: trauma
Cheng JC, Lam P, Shen WY. Closed reduction and percu- 4 1st ed. Schmidt A, eague D eds. Rosemont, IL: American
taneous pinning or type III displaced supracondylar Academy o Orthopaedic Surgeons; 2010:57987.
ractures o the humerus in children. J Orthop Trauma. Wilber JH, T ompson GH, Son-Hing J. T e Multiply Injured
1995;9:51115. Child. In: Green N, Swiontkowski MF, eds. Skeletal
dAmato C. Pediatric spinal trauma: injuries in very young Trauma in Children. 4th ed. Philadelphia, PA: Elsevier;
children. Clin Orthop Relat Res. 2005;(432):3440. 2009:5783.
101
Knee Injuries

Jeremy McCallum and Douglas Rowles

SCENARIO 1 C. Immediate portable X-Rays o his right hip, knee,


A 27-year-old healthy male is transported to the emer- and lower extremity.
gency department a er a motor vehicle crash. He was a D. Admit or observation, antibiotics, and serial
restrained driver in a head on collision with another car examinations.
and was reportedly traveling about 55 mph. His vehicles E. ransport to the OR or irrigation, exploration,
airbag deployed. T e responding medics report he was debridement o the open wound, and other pro-
responsive at the scene and complained o isolated right cedures as indicated.
knee pain with numbness and tingling to his right lower 2. X-ray assessment o the knee and leg reveals a
extremity below the knee. posteromedial knee dislocation without obvious
A head to toe assessment by the on call trauma team ractures. X-rays o the hip, emur, and below the
in the emergency department con rms injury is local- knee are normal. Regarding the neurovascular
ized to the right lower extremity. Physical examination status o the right lower extremity, which o the
reveals signi cant so tissue swelling around the right ollowing should be the next step in management?
knee with de ormity but so and supple leg muscular
compartments, a 2 cm bleeding laceration along the A. Immediate transport to the OR or open reduc-
anterolateral aspect o the knee, and di use numbness tion and vascular repair.
o the right oot and lateral leg. Both the dorsalis pedis B. Obtain immediate vascular studies including an
and posterior tibial pulses are palpable but noticeably angiography.
weaker compared to his contralateral extremity. His C. Conscious sedation ollowed by closed reduction
motor examination reveals signi cant weakness with and reassessment o distal pulses and neurologic
oot and great toe dorsi exion but the exam is limited exam.
due to pain. D. Operative washout o the wound and knee joint
reduction ollowed by ice, elevation, and obser-
vation o neurovascular status.
1. A er applying a saline-soaked dressing and ace E. Immediate re erral to orthopedic surgery.
wrap to the wound, initial management in the
emergency department should include which o 3. Regarding vascular injury associated with knee
the ollowing? dislocations, which o the ollowing statements is
A. A computerized tomography (C ) scan o his true?
head, abdomen, pelvis, and C angiography o A. T e incidence o injury to the popliteal artery is
the right lower extremity. low (< 2%).
B. Immediate right leg compartment pressure B. Vascular repair can be delayed up to 24 hours, a er
testing. which amputation rates dramatically increase.
C H AP TER 1 0 1 Kn EE In j u RI Es 387

C. Physical exam alone is suf cient in detecting all have a pneumothorax, multiple acial contusions, and a
vascular injuries a er knee dislocation. closed injury to her right knee. Focused examination o
D. A er reduction and return o pulses, serial the knee reveals severe so tissue swelling, mild blister-
examinations at least every 4 to 6 hours or a ing o the skin over the anterior portion o her knee but
minimum o 48 hours is necessary to monitor no open wounds and signi cant di use pain with active
or late-developing complications. and passive motion. Her distal neurovascular exam is
E. Normal pulses rule out an arterial or venous intact. X-rays o the knee reveal a displaced tibial pla-
injury. teau racture.

4. Assuming injury to the popliteal artery is conf r- 1. Management o her knee injury should include
med preoperatively, which statement regarding which o the ollowing?
surgical management is correct? A. Urgent surgical treatment with open reduction
A. Vascular repair should be completed ollowed by and internal xation.
splinting prior to orthopedic consultation. B. Urgent de nitive surgical treatment with exter-
B. Orthopedic re erral or immediate ligament nal xation.
repair or reconstruction is indicated prior to vas- C. Long leg casting until bony union.
cular repair. D. Splint versus external xation until so tissue
C. Vascular repair and management o the open swelling resolves ollowed by open reduction
wound should be completed ollowed by bracing and internal xation.
and outpatient rehabilitation o the ligamentous E. Splint versus knee immobilizer until so tissue
knee injury. swelling resolves ollowed by use o a bone stim-
D. Vascular repair and management o the open ulator, unctional bracing, and physical therapy.
wound should be completed. Reconstructive
knee surgery is typically not needed due to post 2. Which o the ollowing may be a concomitant knee
traumatic sti ness. injury to a tibial plateau racture?
E. Vascular repair and orthopedic surgical manage- A. Meniscus tear
ment should occur simultaneously. B. Ligament tear
C. Osteochondral or chondral injury to the articu-
5. Regarding neurovascular injuries associated lar sur ace
with knee dislocations, which o the ollowing D. Knee capsule tear
statements is correct? E. All o the above
A. T e incidence o neurovascular compromise
increases proportionately with increasing energy 3. Which o the ollowing actors most strongly
o injury. predicts an increased risk o leg compartment
B. Delayed recognition o an associated vascular syndrome in this patient?
injury signi cantly increases the chance o poor A. Severity o so tissue swelling
unctional outcomes or limb loss. B. Integrity o the knee joint capsule
C. Neurovascular injury is unlikely in low energy C. ime since injury
injuries. D. Amount o displacement o the tibial racture
D. T e most common neurologic injury associated E. ourniquet time during surgery
with knee dislocation is tibial nerve injury.
E. All o the above statements are correct. SCENARIO 3
A 19-year-old male patient presents to the emergency
SCENARIO 2
department a er sustaining a le knee injury earlier
A 62-year-old emale is transported to the emergency that day playing basketball. He reports a similar injury
department due to injuries incurred as a restrained to this knee a year previously, a er which he has had re-
ront seat passenger in a roll-over motor vehicle acci- quent episodes o his knee giving way. Physical exami-
dent. She has a past medical history signi cant or type nation reveals an e usion and so tissue swelling about
2 diabetes, hypertension, hyperlipidemia, and hypothy- the knee. He has pain but is able to ully ex and extend
roidism. During the trauma assessment, she is noted to the knee. X-ray examination o the knee is normal but
388 G En ERAL s u RG ERY EXAM I n ATI O n An D BO ARD REVI EW

an magnetic resonance imaging (MRI) scan obtained ANSWERS TO SCENARIO 1


the next day reveals a complete tear o the anterior cru-
ciate ligament. 1. C. A er evaluation and stabilization o the patient
radiographs should be obtained to con rm the diag-
1. Which o the ollowing physical examination tests nosis. Radiographs should not unnecessarily delay the
is most reliable at diagnosing an anterior cruciate reduction o the knee. Given up to 50% o knee dis-
ligament (ACL) tear? locations can undergo a spontaneous reduction prior
A. Anterior drawer test to radiographs, normal radiographs do not rule out a
B. Lachman test knee dislocation. Rim ractures, joint asymmetry, avul-
C. Pivot shi test sion ractures, and mild subluxation o the tibio emoral
D. McMurrey test joint may be the only radiographic nding. An MRI
E. Apley compression test o the knee can be obtained on an elective basis once
the patient is acutely stabilized and is the diagnostic
2. What is the most likely associated knee injury with imaging modality o choice. T e question stem states
an acute ACL tear? this is an isolated injury making option A incorrect.
A. Medial collateral ligament tear Compartment syndrome in a knee dislocation is not
B. Medial meniscus tear uncommon and may be secondary to the initial trauma,
C. Lateral eniscus tear hemorrhage, or reper usion o an ischemic limb.
D. Medial patello emoral ligament tear Answer B, a low threshold to measure compart-
E. Medial joint impaction racture (bone contu- ment pressures is appropriate; however, obtaining
sion) radiographs and attempting reduction o the knee
are the more appropriate answer. T e patient needs
3. Initial management o this patient should include urther workup be ore D or E would be applicable.
all o the ollowing except?
A. A course o anti-in ammatory medications 2. C. Once a knee dislocation is identi ed, emergent
(NSAIDS). closed reduction with sedation should be attempted.
B. Re erral to orthopedic surgery or reconstructive T e reduction maneuver depends on the direction
knee surgery. o dislocation and generally involves gentle traction-
C. Re erral to physical therapy. countertraction. T e reduction should be per ormed as
D. Knee immobilization in a brace until swelling is atraumatically as possible with sedation (i needed to
resolved. avoid urther damage). Occasionally the injuries can-
E. Re erral or a well- tted ACL brace. not be reduced in a closed ashion. In a posteriolateral
dislocation, the medial emoral condyle buttonholes
4. Which o the ollowing is NOT an acceptable through the joint capsule and/or the medial collateral
alternative as a gra option or ACLreconstruction ligament,causing a puckering o the skin and preventing
in this patient? a closed reduction I closed reduction ails, the patient
A. Autologous Ipsilateral Patellar endon Gra . is indicated or emergent surgical reduction. Answers
B. Autologous Quadrupled Hamstring endon A, B, and D are incorrect because a closed reduction
Gra t. has not been attempted. A er reduction, the injured
C. Contralateral Patellar endon Gra . knee is requently unstable and should be placed into a
D. Allogra (Donor) endon Gra . long leg splint (above the knee splint) in approximately
E. All o the above are acceptable gra options. 20 degrees o exion to prevent posterior subluxa-
tion o the tibia. A circum erential splint/cast should
5. Which o the ollowing is the most likely be avoided to help prevent compartment syndrome.
complication o ACL reconstructive surgery? Post-reduction radiographs should be obtained to con-
A. Sti ness rm adequate reduction. A er reduction the vascular
B. Intra-articular in ection assessment should be repeated. Answer E, re erral to an
C. Deep venous thrombosis (DV ) orthopedic surgeon, is recommended or a knee dis-
D. Iatrogenic neurovascular injury location however reduction should not be needlessly
E. Gra ailure delayed while waiting or a consultation.
C H AP TER 1 0 1 Kn EE I n j u RI Es 389

3. D. Vascular injuries associated with ractures/ all knee dislocations should be assumed to have a
dislocations are relatively uncommon, but there is a vascular injury until it has been ruled out. Several
strong association with posterior knee dislocations studies have also demonstrated an increased inci-
and popliteal artery/vein injuries. Answer A, the rate dence o these injuries among obese patients versus
o popliteal artery injury, has been reported to range non-obese. Answers A and C are incorrect because in
rom 14% to 65%. Multiple algorithms have been a review o low energy dislocations, 41% were ound
published or assessment o the vascular status o the to have a vascular injury. T is is comparable to the
lower extremity in the incidence o suspected knee dis- vascular injury rates o cohorts with combined (low
location. Initially, a physical exam should be per ormed and high energy and high energy injuries). T e most
assessing both posterior tibial and dorsalis pedis pulses commonly injured nerve in a knee dislocation is the
in comparison to the contralateral extremity. Answer peroneal nerve, not the (D) tibial nerve. Neurologic
C/E, normal pulses, do not rule out a vascular injury. injury occurs in 16% to 40% o knee dislocations.
In the presence o hard signs o ischemia (cool, Less than 50% o patients with peroneal nerve inju-
pulseless, obviously dysvascular extremity) vascular ries have nerve recovery.
surgery consultation should be obtained immediately.
However, when a patient presents with so signs o
ischemia (asymmetric pulses and/or warmth), urther ANSWERS TO SCENARIO 2
assessment is warranted. Measurement o the ankle- 1. D. Displaced tibial plateau ractures greater than
brachial index (ABI) can augment the physical exam. 0.5 to 1 cm are managed with operative reduction
An ABI > /= 0.9 is reassuring that there is no clini- and xation (answers C and E). Protected mobi-
cally signi cant vascular injury. However, choice D is lization can be used or ractures that are non- to
correct because delayed thrombus is a continued risk, minimally displaced with a stable ligamentous exam.
making reassessment every 4 to 6 hours important. High energy injuries can lead to severe injury to the
I the ABI is abnormal, urther studies are indicated. overlying so tissue as demonstrated by swelling
Either an arterial ultrasound, which is technician and racture blisters. Bicondylar ractures, racture-
dependent and may not always be available, or C dislocations, and sha dissociated ractures have
angiography versus conventional angiography should worse so tissue injury. De nitive surgical reduction
be completed. Emergent vascular repair is indicated i and treatment should be delayed in high energy inju-
a signi cant injury is identi ed, with the (B) amputa- ries until the so tissue envelop allows (answer A).
tion rate as high as 86% in cases where vascular repair In high energy trauma, it may take 8 to 21 days or the
was delayed greater than 8 hours. swelling to subside and skin conditions to improve.
Interval treatment can range rom a well applied
4. E. I a vascular injury is identi ed both vascular and splint to temporary spanning external xation. T e
orthopedic surgery should be urgently consulted. goal is to maintain length and alignment until de ni-
T e key to this question is that both teams will need tive xation can be pre ormed and post-reduction
to work simultaneously in stabilizing and restor- radiographs should be obtained. raction radio-
ing blood ow to the lower extremity. Answers A graphs and/or post reduction C scan can be help ul
and C are incorrect because vascular repair without in evaluation and planning o treatment. Even with
stabilizing the leg will likely ail. Answer B is incor- staged management o these ractures. in ection rates
rect because restoring vascular ow to the leg is the still range rom 8.4% to 18%.
rst priority and should be done within 8 hours and
answer D is incorrect because reconstructive knee 2. E. Regarding answers A and B, meniscal tears occur
surgery will be needed to protect the vascular repair in up to 50% o plateau ractures. Ligamentous inju-
and regain unctional range o motion in the short ries occur in approximately 30% to 77% o ractures.
term as well as to prevent long term knee instability. Answers C and D are incorrect because capsular tears
and chondral injury can both be present as part o
5. B. Multiple series have demonstrated that sub- the injury pattern. Segond ractures, reverse Segond
optimal unctional outcomes and even amputa- ractures, anteromedial tibial margin ractures, and
tion can result when there is delayed recognition semimembranosus tendon insertion site ractures
o an associated injury to the popliteal vessels, and are all evidence o associated injuries.
390 G En ERAL s u RG ERY EXAM I n ATI O n An D BO ARD REVIEW

3. D. T e rate o compartment syndrome in tibial Answer D is correct because prolonged immobili-


plateau ractures is 10% to 15%. Presenting symptoms zation should be avoided as it may lead to arthro -
include pain out o proportion to injury, swelling, brosis. Regarding choice E, the use o an ACL brace
pain on passive stretching, pallor, absence o pulses, continues to be controversial and has become an indi-
hyperesthesia, and motor weakness. Physical exam vidualized and optional part o the treatment plan.
can be unreliable (answer A) and i suspected, Patients should be re erred to an orthopedic surgeon
leg muscular compartmental pressures should be or counseling o treatment options. T e ideal candi-
assessed. T e amount o displacement (correct date or surgical reconstruction is the patient with an
answer D) as well as a higher Shatzker and/or OA/ acute ACL de ciency and an active li estyle as well as
O A classi cation are associated with increased risk one with a chronic injury and unctional instability
o compartment syndrome. Repeat examination o with normal daily activities.
the leg or compartment syndrome should be contin-
ued at regular intervals because it can occur 24 hours
or more a er injury (answer C).

ANSWERS TO SCENARIO 3
P oplite a l a .
1. B. T e Lachman test is the most sensitive physical
Me dia l s upe rior La te ra l s upe rior
exam maneuver to test or ACL laxity. T e knee is ge nicula r a . ge nicula r a .
placed in 20 to 30 degrees o exion, the emur is sta- S ura l a a .
bilized, and an anteriorly directed orce is applied to La te ra l infe rior
ge nicula r a .
the tibia. T e examiner should estimate the distance Me dia l infe rior
o translation as well as whether there is an endpoint ge nicula r a . P os te rior tibia l
re curre nt a .
or not. T e pivot shi test and anterior drawer test Ante rior tibia l a .
are other physical examination maneuvers testing Ante rior tibia l
Tibia l-pe rone a l trunk re curre nt a .
the anterior cruciate ligament, however, they are not
as reliable as the Lachman exam. T e Mcmurray test
and Apley compression test are a physical exams Fibula r a .
evaluating the meniscus not the ACL.
P os te rior tibia l a .
2. C. Meniscal tears are the most common associated
injury seen with an ACL tear. T ey occur in 65%
to 75% o patients. Lateral meniscal tears are more
common in acute ACL injuries. Answer B is incorrect
because medial meniscus tears are more requently
seen in chronic injuries but less prevalent in acute
injuries. Bone bruises or trabecular micro ractures Communica ting
bra nch
occur in just over hal the patient with an ACL injury,
however, these are typically located along the lateral
Me dia l ma lle ola r La te ra l ma lle ola r
emoral condyle and the posterolateral proximal bra nche s bra nche s
tibia. T ese are a result o the subluxation and spon-
taneous reduction o the knee joint that occurs dur- Me dia l pla nta r a . Ca lca ne a l
bra nche s
ing an injury that leads to knee ligament disruption.
Approximately 33% o patients with an ACL tear will
have at least one additional ligament injured. Regard- Posterior view of leg vasculature
ing choice A, the medial collateral ligament is injured
in about 7% o patient with an ACL injury. 4. E. Gra selection depends upon patient actors and
surgeon pre erence. Multiple options o gra s are avail-
3. D. Answers A and C are incorrect because the able or ACL reconstruction. Both autogra and allo-
treatment or a suspected ACL tear includes pain gra are utilized. Autogra options include quadrupled
control, MRI, and physical therapy or mobilization. hamstring tendons, bone-patellar tendon-bone, and
C H AP TER 1 0 1 Kn EE I n j u RI Es 391

quadriceps tendon. Allogra options are varied and Brogadir SP, Schimmer BM, Myers RA. Spectrum o the
include bone-patellar tendon-bone, achilles tendon, gonococcal arthritis-dermatitis syndrome. Semin Arthritis
hamstring tendons, anterior tibialis tendon, and poste- Rheum. 1979;8:17783.
Canadian Orthopaedic rauma Society. Open reduction and
rior tibialis tendons. Generally speaking, the results o internal xation compared with circular xator application
ACL reconstruction with any o these gra options are or bicondylar tibial plateau ractures. Results o a Multi-
acceptable, although there are speci c clinical scenarios center, prospective, randomized clinical trial. J Bone Joint
that can lend themselves to certain choices. A lengthy Surg Am. December 2006;88(12):261323.
discussion with the patient, outlining the positives and Cash JD, Hughston JC. reatment o acute patellar disloca-
tion. Am J Sports Med. 1988;16:2449.
negatives o the gra choices is requently needed.
Chan KK, Resnick D, Goodwin D, Seeger LL. Posteromedial
tibial plateau injury Including avulsion racture o the
5. A. Sti ness ollowing ACL reconstruction or arthro - semimembranous tendon insertion site: Ancillary sign o
brosis is the most common complication a er surgi- anterior cruciate ligament tear at MR imaging. Radiology.
cal reconstruction. It is likely related to in ammation 1999; 211: 7548.
a ecting the synovial lining leading to thickening o Chang YH, u YK, Yeh WL, Hsu RW. ibial plateau racture
the capsule and loss o the normal space within the with compartment syndrome: A complication o higher
incidence in aiwan. Chang Gung Med J. 2000;23:14955.
joint. Proper surgical technique and rehabilitation can
Cohen AP, King D, Gibbon AJ. Impingement racture o the
help reduced the risk o joint sti ness. Loss o exten- anteromedial tibial margin: A radiographic sign o com-
sion is reported to be as high as 59% in some popu- bined posterolateral complex and posterior cruciate liga-
lations and other complications are rare. Gra ailure ment disruption. Skeletal Radiol. 2001;30:1146.
occurs and varies with di erent populations and di - Cosgarea AJ, DeHaven KE, Lovelock JE T e surgical treat-
erent gra types. It ranges rom 2% to 5%. T e rate o ment o arthro brosis o the knee. Am J Sports Med.
1994;22:18491.
deep venous thrombosis is approximately 1% to 1.5%, Cullison R, Muldoon MP, Gorman JD, Go WB. T e inci-
iatrogenic neurovascular injury incidence is about 1%, dence o deep venous thrombosis in anterior cruciate liga-
and postoperative intra-articular in ection has been ment reconstruction. Arthroscopy. 1996;12:6579.
reported as low as 0.3% in a large cohort. Egol KA, ejwani NC, Capla EL, Wolinsky PL, Koval KJ. Staged
management o high-energy proximal tibia ractures ota
types 41: T e results o a prospective, standardized proto-
BIBLIOGRAPHY col. J Orthop Trauma. 2005;19:44855; discussion 56.
Alberty RE, Good ried G, Boyden AM. Popliteal Artery Fanelli GC, Stannard JP, Stuart MJ, MacDonald PB, Marx RG,
injury with ractural dislocation o the knee. Am J Surg, Whelan DB, et al. Management o complex knee ligament
1981;142:3640. injuries. J Bone Joint Surg Am. 2010;92:223546.
Almekinders LC, Logan C. Results ollowing treatment o Freed JF, Nies KM, Boyer RS, Louie JS. Acute monoarticular
traumatic dislocations o the knee joint. Clin Orthop Relat arthritis. A diagnostic approach. JAMA. 1980;243:23146.
Res. 1992;284:2037. Freedman KB, DAmato MJ, Nede DD, Kaz A, Bach Jr BR.
Andrews JR, edder JL, Godbout BP. Bicondylar tibial plateau Arthroscopic anterior cruciate ligament reconstruction:
racture complicated by compartment syndrome. Orthop A metaanalysis comparing patellar tendon and hamstring
Rev. 1992;21;3179. tendon autogra s. Am J Sports Med. 2003;31:211.
Anglen JO, Aleto . emporary transarticular external xation Gable DR, Allen JW, Richardson JD. Blunt popliteal artery
o the knee and ankle. J Orthop Trauma. 1998;12:4314. injury: Is physical examination alone enough or evalua-
Azar FM, Brandt JC, Miller 3rd RH, Phillips BB. Ultra-low- tion? J Trauma. 1997;43:5414.
velocity knee dislocations. Am J Sports Med. 2011;39;21704. Gardner MJ, Yacoubian S, Geller D, Suk M, Mintz D, Potter H,
Baer PA, enenbaum J, Fam AG, Little H. Coexistent septic et al. T e incidence o so tissue injury in operative tibial
and crystal arthritis. Report o our cases and literature plateau ractures: A magnetic resonance imaging analysis
review. J Rheumatol. 1986;13;6047. o 103 patients. J Orthop Trauma. 2005;19:7984.
Bardin . Gonococcal arthritis. Best Pract Res Clin Rheumatol. Goldenberg DL, Cohen AS. Acute in ectious arthritis. A review
2003;17;2018. o patients with Nongonococcal joint in ections with empha-
Barei DP, Nork SE, Mills WJ, Henley MB, Benirschke SK. sis on therapy and prognosis. Am J Med. 1976;60:3677.
Complications associated with internal xation o high- Green NE, Allen BL. Vascular injuries associated with disloca-
energy bicondylar tibial plateau ractures utilizing a two- tion o the knee. J Bone Joint Surg Am 1977;59:2369.
incision technique. J Orthop Trauma. 2004;18;64957. Greis PE, Bardana DD, Holmstrom MC, Burks R . Meniscal
Berkson EM, Virkus WW. High-energy tibial plateau ractures. injury: I. basic science and evaluation. J Am Acad Orthop
J Am Acad Orthop Surg. 2006;14:2031. Surg. 2002;10:16876.
Boring H, ODonoghue DH. Acute patellar dislocation: Gupta MN, Sturrock RD, Field M. A prospective 2-year study
Results o immediate surgical repair. Clin Orthop Relat Res. o 75 patients with adult-onset septic arthritis. Rheumatol-
1978;136:1825. ogy Oxford. 2001;40:2430.
392 G En ERAL s u RG ERY EXAM I n ATIO n An D BO ARD REVI EW

Heckman JD, ornetta P. Rockwood and Greens fracture in OBrien JP, Goldenberg DL, Rice PA. Disseminated gonococ-
Adults. Vol. 2, Philadelphia, PA: Lippincott Williams & cal in ection: A prospective analysis o 49 patients and a
Wilkins; 2010. review o pathophysiology and immune mechanisms.
Holmes KK, Weisner PJ, Pedersen AH. T e gonococcal arthritis- Medicine Baltimore. 1983;62:395406.
dermatitis syndrome. Ann Intern Med. 1971;75:4701. Petsche S, Hutchinson MR. Loss o extension a er recon-
Hoover NW. Injuries o the popliteal artery associated with rac- struction o the anterior cruciate ligament. J Am Acad
tures and dislocations. Surg Clin North Am. 1961;41:1099112. Orthop Surg. 1999;7:11927.
Hughes JG, Vetter EA, Patel R, Schleck CD, Harmsen S, ur- Prodromos CC, Fu FH, Howell SM, Johnson DH, Lawhorn K.
geant L , et al. Culture with bactec peds plus/ bottle com- Controversies in so -tissue anterior cruciate ligament
pared with conventional methods or detection o bacteria reconstruction: Gra s, bundles, tunnels, xation, and har-
in synovial uid. J Clin Microbiol. 2001;39:446871. vest. J Am Acad Orthop Surg. 2008;16:37684.
Johnson ME, Foster L, DeLee JC. Neurologic and vascular Reckling FW, Peltier LF. Acute knee dislocations and their
injuries associated with knee ligament injuries. Am J Sports complications. J Trauma. 1969;9:18191.
Med. 2008;36:244862. Rice PA. Gonococcal arthritis disseminated gonococcal in ec-
Jones RE, Smith EC, Bone GE. Vascular and orthopedic tion. Infect Dis Clin North Am. 2005;19:85361.
complications o knee dislocation. Surg Gynecol Obstet. Rihn JA, Gro YJ, Harner CD, Cha PS. T e acutely dislocated
1979;149:5548. knee: Evaluation and management. J Am Acad Orthop Surg.
Koval KJ, Hel et DL. ibial plateau ractures: Evaluation and 2004;12:33446.
treatment. J Am Acad Orthop Surg. 1995;3:8694. Sallay PI, Poggi J, Speer KP, Garrett WE. Acute dislocation o
LaPrade RF, Wentor FA, Fritts H, Gundry C, Hightower CD. the patella. A correlative pathoanatomic study. Am J Sports
A prospective magnetic resonance imaging study o the Med. 1996;24:5260.
incidence o posterolateral and multiple ligament inju- Shah SN, Karunakar MA. Early wound complications a er
ries in acute knee injuries presenting with a hemarthrosis. operative treatment o high energy tibial plateau ractures
Arthroscopy. 2007;23:13417. through two incisions. Bull NYU Hosp Jt Dis. 2007;65:
Larson RL, ailon M. Anterior cruciate ligament insuf ciency: 1159.
Principles o treatment. J Am Acad Orthop Surg. 1994;2:2635. Sisto DJ, Warren RF. Complete knee dislocation. A ollow-up
Levy BA, Zlowodzki MP, Graves M, Cole PA. Screening or study o operative treatment. Clin Orthop Relat Res. 1985;
extermity arterial injury with the arterial pressure index. 198:94101.
Am J Emerg Med. 2005;23:68995. Swan A, Amer H, and Dieppe P. T e value o synovial uid
Magnussen RA, Carey JL, Spindler KP. Does operative xa- assays in the diagnosis o joint disease: A literature survey.
tion o an osteochondritis dissecans loose body result in Ann Rheum Dis. 2002;61:4938.
healing and long-term maintenance o knee unction? Am joumakaris FP, Forsythe B, Bradley JP. Patello emoral insta-
J Sports Med. 2009;37:7549. bility in athletes: reatment via modi ed ulkerson oste-
Manshady BM, T ompson GR, Weiss JJ. Septic arthritis in a otomy and lateral release. Am J Sports Med. 2010;38:9929.
general hospital 1966-1977. J Rheumatol. 1980;7:52330. Warren RF, Levy IM. Meniscal lesions associated with
Margaretten ME, Kohlwes J, Moore D, Bent S. Does this adult anterior cruciate ligament injury. Clin Orthop Relat Res.
patient have septic arthritis? JAMA 2007;297:147888. 1983;327.
Mathews CJ, Kingsley G, Field M, Jones A, Weston VC, Wascher DC. High-velocity knee dislocation with vascular
Phillips M, et al. Management o septic arthritis: A system- injury. reatment principles. Clin Sports Med. 2000;19:457
atic review. Postgrad Med J. 2008;84:26570. 77.
Mathews CJ, Weston VC, Jones A, Field M, Coakley G. Bacte- Wascher DC, Dvirnak PC, DeCoster A. Knee dislocation:
rial septic arthritis in adults. Lancet. 2010;375:84655. Initial assessment and implications or treatment. J Orthop
McCutchan JD, Gillham NR. Injury to the popliteal artery Trauma. 1997;11:5259.
associated with dislocation o the knee: Palpable distal Welling RE, Kakkasseril J, Cranley JJ. Complete disloca-
pulses do not negate the requirement or arteriography. tions o the knee with popliteal vascular injury. J Trauma.
Injury. 1989;20:30710. 1981;21:4503.
McDonough Jr. EB, Wojtys EM. Multiligamentous injuries o Williams 3rd RJ, Laurencin C , Warren RF, Speciale AC,
the knee and associated vascular injuries. Am J Sports Med. Brause BD, O Brien S. Septic arthritis a er arthroscopic
2009;37:1569. anterior cruciate ligament reconstruction. Diagnosis and
Miller MD, T ompson SR, Hart JA. Review of Orthopedics. management, Am J Sports Med. 1997;25:2617.
Philadelphia, PA: Elsevier Saunders; 2012. Yagupsky P, Peled N, Press J. Use o bactec 9240 blood cul-
Mills WJ, Barei DP, McNair P. T e value o the ankle-brachial ture system or detection o brucella melitensis in synovial
index or diagnosing arterial injury a er knee dislocation: uid. J Clin Microbiol. 2001;39:7389.
A prospective study. J Trauma. 2004;56:12615. Yagupsky P, Press J. Use o the isolator 1.5 microbial tube or
Niall DM, Nutton RW, Keating JF. Palsy o the common pero- culture o synovial uid rom patients with septic arthritis.
neal nerve a er traumatic dislocation o the knee. J Bone J Clin Microbiol. 1997;35:24102.
Joint Surg Br. 2005;87:6647. Ziran BH, Becher SJ. Radiographic predictors o compartment
Nomura E. Inoue M. Cartilage lesions o the patella in recur- syndrome in tibial plateau ractures. J Orthop Trauma.
rent patellar dislocation. Am J Sports Med. 2004;32:498502. 2013;27:6125.
Ur o l o gy
Joseph R. Sterbis
This page intentionally left blank
102
Nephrolithiasis

Peter L. Steinberg

A 35-year-old man presents to the emergency depart- 4. Which actors are predictive o ureteral stone
ment with one hour o severe right-sided ank pain. He passage?
is nauseated and vomited when the pain began. He has A. Stone size
no medical history and is not on any medications. He B. Stone location
has voided and his urinalysis shows 55 RBC per HPF C. Presence or absence o hydronephrosis
and no WBCs. D. A and B
E. A, B, and C
1. What is the single best test to diagnose a kidney or
ureteral stone? 5. Which medical condition(s) would you want
A. KUB to rule out in a recurrent calcium oxalate stone
B. Renal ultrasound ormer?
C. IVP (intravenous pyelogram) A. Hyperparathyroidism
D. Non-contrast computerized tomography (C ) B. Gout
scan o the abdomen and pelvis C. Cushings Syndrome
E. C scan o the abdomen and pelvis with and D. A and B
without IV contrast E. A, B, and C

2. Which o these actors would mandate immediate


decompression o the urinary tract with either a ANSWERS
ureteral stent or a percutaneous nephrostomy tube? 1. D. A non-contrast C (NCC ) o the abdomen in
A. Acute kidney injury pelvis is the single most accurate imaging test or a
B. Sepsis presumed rom the obstructed kidney renal or ureteral stone. T e sensitivity and speci c-
C. Uncontrollable pain ity o NCC are both in the high 90% range. Other
D. A, B, and C imaging modalities, such as renal ultrasound and
E. A and B kidney, ureter, and bladder (KUB) X-ray, can be used
to monitor a known stone, or are help ul i radiation
3. What is the most ef ective analgesia or kidney must be limited (pregnancy), but have a lower sensi-
stone colic in the emergency department? tivity and speci city than a NCC .
A. NSAIDs Renal ultrasound is most help ul i the patient has
B. ylenol hydronephrosis; however, 20% to 25% o patients
C. NSAIDs and IV opiates with a ureteral stone will not have hydronephrosis.
D. IV opiates Likewise, 20% to 30% o ureteral calculi will not
E. ylenol and IV opiates appear on a KUB. As a result, a NCC is the optimal
396 G EN ERAL S U RG ERY EXAM IN ATIO N AN D BO ARD REVI EW

test to assess unilateral ank pain in the emergency hydronephrosis has no bearing on whether or not a
department. A low dose protocol can be per ormed stone will pass.
in patients with a relatively low BMI, in an e ort to
reduce radiation exposure. 5. A. Hyperparathyoridism can cause urinary stones;
however, only 1% to 2% o kidney stone ormers have
2. D. Uncontrollable pain, sepsis rom the renal unit hyperparathyroidism. Surgical management o the
that is obstructed by a stone, and acute kidney injury parathyroid disease is of en curative o uture stone
are the main reasons to per orm immediate decom- ormation. An elevated blood calcium level, or even a
pression o a renal unit. T e choice o a stent or a high normal blood calcium level, as well as recurrent
percutaneous nephrostomy tube is based on local episodes o kidney stones should alert you to this as a
experience, available personnel, the patients condi- diagnostic possibility. A serum parathyroid hormone
tion, and the chance o success ul placement o either (P H) level, as well as serum calcium and vitamin D
type o device. level should be obtained on these patients.

3. C. T e combination o NSAIDs and IV opiates has


been shown to be superior to either agent alone in BIBLIOGRAPHY
the management o renal colic in the emergency EAU/AUA Nephrolithiasis Guideline Panel. 2007 guideline
department. or the management o ureteral calculi. J Urol. December
ylenol has not been widely studied in the man- 2007;178(6):241834.
agement o renal colic, and the pain is usually sub- Fulgham PF, Assimos DG, Pearle MS, Preminger GM. Clini-
stantial, thus this would be a less desirable analgesic. cal e ectiveness protocols or imaging in the management
o ureteral calculous disease: AUA technology assessment.
In addition, many patients with renal colic are vomit-
J Urol. April 2013;189(4):120313.
ing, and require parenteral analgesia and rapid anal- Pearle MS, Gold arb DS, Assimos DG, Curhan G, Denu-
gesia at that. Ciocca CJ, et al. Medical management o kidney stones:
AUA guideline. J Urol. 2014;192(2):31624.
4. D. T e combination o stone location and stone size Sa dar B, Degutis LC, Landry K, Vedere SR, Moscovitz HC,
allows estimation o ureteral stone passage rates. In et al. Intravenous morphine plus ketorolac is superior to
general, the smaller a ureteral stone is the higher either drug alone or treatment o acute renal colic. Ann
Emerg Med. August 2006;48(2):17381, 181.e1
the chance o passage. Likewise, the more distal a
Steinberg PL, Nangia AK, Curtis K. A standardized pain man-
ureteral stone is the higher the chance o passage. agement protocol improves timeliness o analgesia among
Smaller distal stones are more likely to pass than emergency department patients with renal colic. Qual
larger proximal stones. T e presence or absence o Manag Health Care. January-March 2011;20(1):306.
103
Bladder Cancer

Joseph R. Sterbis

A 64-year-old Caucasian male coal miner with a prior C. Hyperkalemic, hypochloremic, hyponatremic
20 pack per year smoking history, diabetes mellitus, metabolic acidosis
hypertension, and hyperlipidemia presents or evalua- D. Hypokalemic, hypernatremic metabolic alkalosis
tion o urinary requency. A urinalysis demonstrates 2 E. None o the above
WBC/high powered eld, 10 RBC/high powered eld,
and is negative or all other components. No red blood 4. T e patient returns to clinic 18 months af er
cell casts or dysmorphic red blood cells are present on surgery with recurrent gross asymptomatic
urine microscopy. hematuria. Which o the ollowing sites is the
likely location or cancer recurrence?
1. Microscopic hematuria is de ned as which o the A. Ileal conduit
ollowing? B. Kidney
A. Positive urine dipstick test C. Psoas muscle
B. 1 cell/high powered eld D. Lef renal pelvis
C. 3 cells/high powered eld E. T yroid
D. 5 cells/high powered eld

2. Which o the ollowing represents the complete ANSWERS


work-up o microscopic hematuria?
1. C. Asymptomatic microscopic hematuria (AMH)
A. Cystoscopy, urine cytology is de ned as three or greater red blood cells per
B. Renal ultrasound and bladder barbotage high-powered eld on a clean catch midstream uri-
C. Computerized tomography (C ) intravenous nary specimen in the absence o any other benign
pyelogram cause. A positive dipstick does not de ne AMH and
D. Cystoscopy, C intravenous pyelogram microscopic examination o the urinary specimen is
E. Urine cytology, urine culture mandatory or diagnosis. T e likelihood o a urologic
malignancy in the presence o microscopic hematu-
3. Cystoscopy revealed a large bladder tumor. A
ria is approximately 10%.
transurethral biopsy o his tumor reveals muscle
invasive, high-grade bladder cancer. T e patient then 2. D. T e complete work-up or hematuria includes
undergoes a radical cystectomy and ileal conduit. cystoscopy and C intravenous pyelogram. With this
Which o the ollowing electrolyte abnormalities is evaluation strategy, the cause or hematuria is identi-
expected af er urinary diversion with ileum? ed in 80% o cases. Patients with persistent hematu-
A. Hypokalemic, hyperchloremic metabolic acidosis ria af er a negative initial evaluation warrant repeat
B. Hypokalemic, hypochloremic metabolic alkalosis evaluation at 48 to 72 months. I the patient cannot
398 G EN ERAL S U RG ERY EXAM IN ATI O N AN D BO ARD REVIEW

obtain a C intravenous pyelogram, then alternative Douglas MD, McDougal WS. Use o Intestinal Segments
acceptable imaging studies include intravenous pye- in Urinary Diversion. In: Wein AJ, Kavoussi LR, Novick
logram, retrograde pyelography, or magnetic reso- AC, Partin AW, Peters CA, eds. Campbell-Walsh Urology.
10th ed. Philadelphia: Saunders; 2007;241149.
nance urography. Fontaine E, Barthelemy Y, Houlgatte A, et al. wenty-year
experience with jejunal conduits. Urology. 1997;50:20713.
3. A. Hypokalemic, hyperchloremic metabolic aci-
Golimbu M, Morales P: Jejunal conduits: technique and com-
dosis is the electrolyte abnormality present when plications. J Urol. 1975;113:78795.
ileum as a urinary conduit. Hypochloremic acido- Gross eld GD, Litwin MS, Wol JS Jr, et al.: Evaluation o
sis occurs when ammonium chloride is absorbed asymptomatic microscopic hematuria in adults: T e Amer-
rom the urine into the blood stream. Hypokalemic, ican urological association best practice policyPart II.
hyperchloremic metabolic alkalosis occurs when the Patient evaluation, cytology, voided markers, imaging, cys-
toscopy, nephrology evaluation and ollow-up. Urology.
stomach is used or urinary diversion and hydro- 2001;57:604.
chloric acid and potassium are lost in gastric secre- Gross eld GD, Litwin MS, Wol JS Jr, et al. Evaluation o asymp-
tions. Hyperkalemic, hypochloremic, hyponatremic tomatic microscopic hematuria in adults: T e American
metabolic acidosis are electrolyte derangements seen urological association best practice policyPart I. De ni-
when jejunum is used or urinary diversion. T e jeju- tion, prevalence and etiology. Urology. 2001;57:599.
Gross eld GD, Wol JS, Litwin MS, Hricak H, Shuler CL,
num secretes sodium and chloride, while potassium
Agerter DC, Carroll PR. Asymptomatic Microscopic
and hydrogen are absorbed. Hematuria in Adults: Summary o the AUA best practice
policy recommendations. American Family Physician. 2001;
4. D. T e patient has developed an upper urinary tract 63(6). http://www.aa p.org/a p/2001/0315/p1145.html
tumor. According to SEER data, the relative risk Accessed May 7, 2011.
or upper urinary tract tumors or white men and Kurzrock EA, Baskin LS, Kogan BA. Gatrocystoplasty: Long-
women was listed as 64.2% and 75.4% at or be ore term ollow up. J Urol. 1998;160:21826.
2 years; 44.3% and 40.5% at 2 to 5 years; 50.8% and Rabbani F, Perrotti M, Russo P, Herr HW. Upper-tract tumors
af er an initial diagnosis o bladder cancer: argument or
42.1% at 5 to 10 years; and 43.2% and 22.2% at more
long-term surveillance. J Clin Oncol. 2001;19:94100.
than 10 years, respectively. Upper tract surveillance Wright JL, Hotaling J, Porter MP. Predictors o upper tract
af er a bladder tumor is necessary and is more likely urothelial cell carcinoma af er primary bladder cancer: A
to occur with high-grade bladder cancer. It can be population based analysis. J Urol. 2009;181:10359.
per ormed with C urography. Yun EJ, Meng MV, Carroll PR. Evaluation o the patient with
hematuria. Med Clin North Am. March 2004;88(2):32943.
BIBLIOGRAPHY
Castro JE, Ram MD. Electrolyte imbalance ollowing ileal uri-
nary diversion. Br J Urol. 1970;42:2932.
104
Renal umor

Joseph R. Sterbis

A 39-yr-old Caucasian male with a history o poorly B. Pulmonary metastases are not amenable to sur-
controlled hypertension, asthma, and hyperlipidemia gical resection.
obtains a C o his abdomen and pelvis to evaluate or C. Metastatectomy has been proven in randomized
vague right-sided abdominal pain and is ound to have control trials to improve disease- ree survival.
a right renal mass, measuring 8 6 7 cm in size. A D. Metastatectomy is contraindicated i there is
complete work-up reveals renal cell carcinoma with a more than 1 pulmonary lesion.
single lung metastasis. His glomerular ltration rate E. Metastatectomy will not improve his length o
(GFR) was calculated to be 30 mL/min. survival.
1. Decreased GFR (<60 mL/min) has been associated 4. T e patients amily history is signif cant or von
with which o the ollowing? Hippel-Lindau disease (VHL). He is tested or
A. Increased risk o death VHL and is ound to have VHL type 2B. He also
B. Decreased cardiovascular health reports having intermittent headaches, occasional
C. Increased risk o hospitalization palpitations, diaphoresis, which he attributed to
D. A and B only his poorly controlled hypertension. What is the
E. All o the above likely source o his prior symptoms?
2. Which o the ollowing is true o cytoreductive A. Retinal hemangioblastoma
nephrectomy? B. Cerebellar hemangioma
C. Pancreatic cysts
A. In rare cases it can lead to urther metastasis o
D. Pheochromocytoma
the tumor.
E. Epididymal cystadenoma
B. It can improve -cell unction against the pri-
mary tumor. 5. O the ollowing, which is the most reliable method
C. It can negatively impact the response to systemic to distinguish renal cell carcinoma rom an upper
cytokine therapy. tract urothelial tumor?
D. It has no impact on survival.
A. Retrograde urography
E. It cannot be per ormed in the setting o meta-
B. C urography
static disease.
C. Non-contrast renal C
3. Which o the ollowing is true o pulmonary D. Urine cytology
metastatectomy in this setting? E. Intravenous pyelography
A. Resection o limited metastatic disease has been
reported to be associated with long disease- ree
intervals and overall survival.
400 G EN ERAL S U RG ERY EXAM IN ATIO N AN D BO ARD REVI EW

ANSWERS lesions rom each other can be di cult, and correct


diagnosis is needed to determine appropriate surgery
1. E. An independent, graded association has been and medical treatment.
documented between a reduced estimated GFR C urography easier to per orm than intravenous
(starting at a GFR < 60 mL/min) and the risk o pyelography and has a higher degree o accuracy in
death, cardiovascular events, and hospitalization. All detecting renal parenchymal lesions. T e sensitivity
o the above are correct. or detecting upper tract malignant disease has been
reported to approach 100% with C urography. C
2. B. Cytoreductive nephrectomy preceding systemic
urography also has a speci city o 60% and a nega-
therapy is per ormed in the setting o advanced
tive predictive value o 100%. Retrograde urography
metastatic disease and has been shown to lead
has an accuracy o 75% in diagnosis o an upper tract
to spontaneous regression o the tumor. It is also
malignant neoplasm. Ureteroscopy is reserved or
believed that -cell unction is inhibited by large
situations when the diagnosis is unclear af er conven-
primary tumors. Overall survival has been shown to
tional radiographic studies and or patients in whom
improve when surgery is combined with cytokine
the treatment plan may be changed on the basis o
therapy.
the ureteroscopic ndings, or example those who
3. A. Resection o limited metastatic disease has been may be amenable to endoscopic resection. Accuracy
reported to be associated with long disease- ree estimates o the sensitivity o cytology range rom
intervals and overall survival. Isolated pulmonary about 20% or grade 1 tumors to 45% and 75% or
metastases are lesions that are most commonly ame- grade 2 and grade 3 tumors. Non-contrast C does
nable to resection with curative intent. T e overall not allow or distinction between parenchymal and
survival o patients undergoing complete resection o urothelial lesions.
limited metastatic disease was reported with median
5-year survival rates o 35% to 50%. No prospective BIBLIOGRAPHY
randomized trains have been per ormed to demon- Bloom HJ. Proceedings: Hormone-induced and spontane-
strate the avorable outcome attributed to resection ous regression o metastatic renal cancer. Cancer. 1973;
o solitary metastatic disease. T e current studies 32:106671.
may re ect patient selection bias, inherent di er- Caoili EM, Cohan RH, Korobkin M, et al. Urinary tract abnor-
malities: Initial experience with multi-detector row C
ences in tumor biology, and natural history or other
urography. Radiology. 2002;222:353.
con ounding actors. T e presence o additional pul- Flanigan RC, Salmon SE, Blumenstein BA, et al. Nephrectomy
monary lesions is not a de nitive contraindication to ollowed by inter eron al a-2b compared with inter eron
metastatectomy. al a-2b alone or metastatic renal-cell cancer. N Engl J Med.
2001;345:16559.
4. D. Pheochromocytoma occurs in 10% to 17% o Friedel G, Hurtgen M, Penzenstadler M, et al. Resection o
patients with VHL. VHL type 2 di ers rom type 1 pulmonary metastases rom renal cell carcinoma. Antican-
cer Res. 1999;19:15936.
in that a ected amily members are at high risk to Go AS, Chertow GM, Fan D, McCulloch CE, Hsu CY.
develop pheochromocytomas. VHL type 2 is ur- Chronic kidney disease and the risks o death, cardiovas-
ther divided into types 2A, 2B, and 2C. Individuals cular events, and hospitalization. N Engl J Med. Septem-
in amilies with VHL type 2A develop pheochromo- ber 23, 2004;351(13):1296305. Erratum in: N Engl J Med.
cytomas but have a low risk or renal cell carcinoma 2008;18(4):4.
Horton WA, Wong V, Eldridge R. Von Hippel-Lindau disease:
(RCC). T ose with VHL type 2B develop pheochro-
Clinical and pathological mani estations in nine amilies
mocytomas and have a high risk or RCC. with 50 a ected members. Arch Intern Med. 1976;136:769.
Levine E, Collins DL, Horton WA, et al. C screening o the
5. B. Upper tract urothelial carcinoma (U -UC) is a abdomen in von Hippel-Lindau disease. AJR Am J Roent-
relatively uncommon orm o cancer arising rom genol. 1982;139:505.
the urothelial lining o the renal pelvis and calyces. Marcus SG, Choyke PL, Reiter R, et al. Regression o meta-
U -UC o the renal pelvis is an aggressive tumor, static renal cell carcinoma af er cytoreductive nephrec-
tomy. J Urol. 1993;150:4636.
which may invade the renal parenchyma and mimic Middleton AW, Jr. Indications or and results o nephrectomy
primary RCC. Advanced RCC can also invade the or metastatic renal cell carcinoma. Urol Clin North Am.
pelvicalyceal system. Distinguishing these primary 1980;7:7117.
C H AP TER 1 0 4 REN AL TU M O R 4 01

Middleton RG. Surgery or metastatic renal cell carcinoma. Snow RM, Schellhammer PF. Spontaneous regression
J Urol. 1967;97:9737. o metastatic renal cell carcinoma. Urology. 1982;20:
Milestone B, Freidman AC, Seidmon EJ, et al. Staging o ure- 17781.
teral transitional cell carcinoma by C and MRI. Urology. Walther MM, Yang JC, Pass HI, et al. Cytoreductive surgery
1990;36:346. be ore high dose interleukin-2 based therapy in patients with
Murthy SC, Kim K, Rice W, et al. Can we predict long-term metastatic renal cell carcinoma. J Urol. 1997;158:16758.
survival af er pulmonary metastasectomy or renal cell Wein AJ, Kavoussi LR, Novick AC, Partin AW, Peters CA.
carcinoma? Ann T orac Surg. 2005;79:9961003. Chapter 49, Malignant Renal umors. In: Steven C. C.,
Russo P, Synder M, Vickers A, et al. Cytoreductive nephrec- Brian R. L., eds. Campbell-Walsh Urology. 2012 ed. Phila-
tomy and nephrectomy/complete metastasectomy or met- delphia, PA: Elsevier Saunders; 2007;141374.
astatic renal cancer. Sci World J. 2007;7:76878. Zbar B, Kishida , Chen F, et al. Germline mutations in the
Sella A, Swanson DA, Ro JY, et al. Surgery ollowing response von Hippel-Lindau disease (VHL) gene in amilies rom
to inter eron-alpha-based therapy or residual renal cell North America, Europe and Japan. Hum Mutat. 1996;8(4):
carcinoma. J Urol. 1993;149:1921; discussion 212. 34857.
105
esticular Lump

Joseph R. Sterbis

A 22-year-old Caucasion male presents to general sur- 4. Following the orchiectomy and staging imaging,
gery clinic or evaluation and treatment o a le inguinal which demonstrated a retroperitoneal mass,
hernia. During the physical examination, the hernia is the patient was treated with three cycles o BEP
identi ed, but a 3 cm le testicular mass is also noted. It is (Bleomycin, Etoposide, and Cisplatin). T is history
painless and rm. T e patient has not noted it previously. should prompt what concerns or subsequent
operative procedures?
1. T e initial radiographic assessment should consist
o what test? A. Bleomycin-induced pulmonary brosis should
prompt the anesthesia provider to limit FiO2 and
A. Scrotal magnetic resonance imaging (MRI)
be judicious in crystalloid and colloid adminis-
B. Pelvic and Scrotal computerized tomography
tration.
(C )
B. Etoposide-induced pulmonary brosis should
C. esticular ultrasound
prompt the anesthesia provider to limit FiO2 and
D. Any o the above
be judicious in crystalloid and colloid adminis-
2. Imaging has con rmed a testicular mass tration.
concerning or malignancy. Pending urologic C. Cisplatin-induced pulmonary brosis should
evaluation, a serum -hCG, Alpha-Fetoprotein prompt the anesthesia provider to limit FiO2 and
(AFP), and lactate dehydrogenase (LDH) tests be judicious in crystalloid and colloid adminis-
are ordered, all o which are signi cantly elevated tration.
above normal limits. Radical orchiectomy will be D. A and B
per ormed via which incision and will demonstrate E. B and C
which most likely pathology?
5. Following his course o chemotherapy, the patient
A. Scrotal incision; Seminoma
has a residual retroperitoneal mass, prompting
B. Scrotal incision; Non-seminoma
his urologist to per orm a retroperitoneal lymph
C. Inguinal incision; Seminoma
node dissection. T e patient should be counseled
D. Inguinal incision; Non-seminoma
regarding what potentially permanent genitou-
3. What is the primary landing zone or retro- rinary side ef ect as a result o this particular
peritoneal metastases in a le -sided testicular procedure?
germ cell tumor? A. Urinary retention
A. Paracaval lymph nodes B. Detrusor overactivity
B. Interaortocaval lymph nodes C. Reduced penile sensation
C. Para-aortic lymph nodes D. Erectile dys unction
D. Inguinal nodes E. Retrograde ejaculation
C H AP TER 1 0 5 TEs Ti C u l AR l u m P 403

ANSWERS 4. A. Bleomycin exposure can generate pulmonary


interstitial brosis. Patients treated with bleomy-
1. C. T e imaging study o choice in testicular cancer cin have experienced a potentially atal respiratory
is a scrotal ultrasound. Classically, testicular cancer distress syndrome ollowing general anesthesia.
will present as a painless testis mass, con rmed with Historically, it has been recommended to reduce
a corresponding hypoechoic lesion on ultrasound. FiO 2 at the time o surgery, however, a retrospec-
T e patient should have an accompanying Posterior- tive study demonstrated that the most signi cant
Anterior (PA) and lateral chest radiograph. Once actor predicting postoperative morbidity was over-
a malignant diagnosis is con rmed, a C o the all perioperative uid balance. In this study, uid
abdomen and pelvis with IV and oral contrast will balance was de ned as the combination o colloid
complete the radiographic staging by assessing or and crystalloid received in surgery, subtracting the
retroperitoneal metastases. urine, blood, and nasogastric uids lost in the same
time rame.
2. D. esticular cancer is managed with a radical
In addition to uid resuscitation and oxygenation
orchiectomy, which consists o a high ligation o the
concerns, one should consider obtaining preopera-
spermatic cord at the level o the internal inguinal
tive pulmonary unction tests prior to major surgical
ring per ormed via an inguinal incision. A long per-
procedures. T e same study cited above also dem-
manent suture is placed on the proximal end o the
onstrated that low preoperative orced vital capac-
spermatic cord be ore it is tucked into the internal
ity also correlated with postoperative oxygenation
ring so that it may be later identi ed during a ret-
problems. Any preoperative assessment o a testicu-
roperitoneal lymph node dissection. T e inguinal
lar cancer patient should determine his exposure
incision allows or complete excision o the cord,
to bleomycin, with appropriate precautions taken i
along with potential microscopic metastases, as well
necessary.
as avoiding contamination o the scrotal lymphatics,
reducing rates o local recurrence. One meta-analysis 5. E. Retrograde ejaculation is a well-known side
demonstrated a local recurrence rate o 2.9% or ef ect rom retroperitoneal lymph node dissection
those with scrotal violation vs. 0.4% with inguinal (RPLND). T e post-ganglionic sympathetic nerves
incisions. Care should also be taken to avoid viola- responsible or ejaculation ( 12-L3) are disrupted
tion o the tunica vaginalis in order to reduce the risk during a RPLND. T is side ef ect may be avoided by
o local tumor recurrence. per orming a template RPLND in low stage disease
T e presence o an elevated AFP indicates that or by per orming a bilateral nerve-sparing RPLND.
the patient will be treated as though he has a non- In the ormer, the predictable metastatic pattern o
seminomatous germ cell tumor as pure seminoma testicular germ cell tumors allow either the right or
will never produce AFP. Seminoma will express hCG le sympathetic nerves to be spared, thus preserving
in approximately 15% o patients. Choriocarcinoma ejaculation. Alternatively, a bilateral nodal dissection
and embryonal carcinoma, non-seminomatous sub- can be per ormed, with care taken to preserve the
types, can also produce hCG. Serum LDH is nonspe- individual nerves along with the hypogastric plexus
ci c, but it can correlate with disease volume. anterior to the aorta, just below the origin o the
in erior mesenteric artery. With appropriate nerve
3. C. Le -sided tumors predominantly metastasize sparing, ejaculation can be preserved in nearly all
to the para-aortic lymph nodes, while right-sided patients.
tumors will generally metastasize to the para-caval An additional complication o RPLND is that o
and interaortocaval lymph nodes. In either case, chylous ascities, due to lymphatic leakage. T is can
metastasis can occur to the opposite side, but such be prevented by meticulous attention to lympho-
crossing metastases are more common with right- stasis.
sided tumors. Generally, testis cancer metastasizes
in an organized, progressive manner, beginning with BIBLIOGRAPHY
the retroperitoneal lymph nodes. One exception is Capelouto CC, et al. A review o scrotal violation in testicular
choriocarcinoma, which can spread in a hematog- cancer: Is adjuvant local therapy necessary? J Urol. March
enous manner. 1995;153(3 Pt 2):9815.
404 G EN ERAl s u RG ERY EXAm i N ATi O N AN D BO ARD REVi EW

Donat SM, et al. Bleomycin associated pulmonary toxicity: Is Jules-Elysee K, et al. Bleomycin-induced pulmonary toxicity.
perioperative oxygen restriction necessary. J Urol. October Clin Chest Med. 1990;11:1.
1998;160:134752. Klein EA. Open technique or nerve-sparing retroperito-
Eggener SE, et al. Incidence o disease outside modi ed ret- neal lymphadenectomy. Urology. January 2000;55(1):
roperitoneal lymph node dissection templates in clinical 1325.
stage I or IIA nonseminomatous germ cell testicular can- NCCN Guidelines, esticular Cancer, 2014.
cer. J Urol. March 2007;177(3):93742.
106
raumatic Ureteral Injury

Ra aella DeRosa and Joseph R. Sterbis

A 30-year-old male is brought to the emergency room B. Spatulating opposing ends o the ureteral seg-
ollowing a gunshot wound to the le lower quadrant. ments can be done selectively.
He undergoes an exploratory laparotomy at which time C. Complete the anastomosis with ne, permanent
a colonic injury is identi ed, requiring resection o the suture is required.
injured segment and a diverting colostomy. During ur- D. Placement o a double-J ureteral stent across the
ther assessment, a le ureteral transection is noted in anastomosis is recommended.
the mid-ureter. E. In the setting o a gunshot wound, a ureteroure-
terostomy cannot be done.
1. T e patient is acidotic, hypothermic, and
hemodynamically abnormal. He has no other 4. Which o the ollowing is true regarding
apparent injuries. T e treatment o choice in this ureteroneocystostomy?
setting is which o the ollowing? A. T e reimplant should be placed on the base o
A. ransureteroureterostomy the bladder.
B. Ureteroureterostomy B. A re uxing reimplant has a higher risk o stric-
C. Boari ap ture.
D. Ligation o the ureteral stumps with permanent C. T is is the treatment o choice or distal ureteral
suture injuries.
E. Psoas hitch with ureteral reimplant D. Stay sutures should not be placed in the ureter to
avoid tissue handling.
2. His preoperative computerized tomography (C ) E. I a psoas hitch is required, the ipsilateral bladder
scan demonstrated a 5 mm lef lower pole renal pedicle may need to be ligated to achieve su -
calculus. Which o the ollowing treatment options cient mobilization o the bladder.
is contraindicated?
A. ransureteroureterostomy 5. Which o the ollowing is true about ureteral injury
B. Ureteroureterostomy due to external trauma rom gunshot wounds?
C. Boari ap A. Ureteral injuries associated with external trauma
D. Ligation o the ureteral stumps with permanent will always present with hematuria.
suture B. T e diagnosis is usually made by a C cystogram.
E. Psoas hitch with ureteral reimplant C. High velocity gunshot wounds (> 350 m/second)
can create a surrounding energy wave 30 to
3. Which o the ollowing considerations is true 40 times the missile diameter.
when per orming an ureteroureterostomy? D. Injury rom missiles and bullets can be located
A. A tension- ree anastomosis is not required. only along the path o tissue penetration.
406 G EN ERAL S U RG ERY EXAM I N ATIO N AN D BO ARD REVI EW

E. Blast injuries to the urinary drainage system will sue handling and preserve the blood supply to the
usually present as an increase in output rom sur- ureter. Re-implated ureters are usually placed at the
gically placed drains immediately (< 24 hours) dome o the bladder.
a er the surgery.
5. C. T e missile or bullet can tumble during penetra-
tion and cause damage to the surrounding tissues at
ANSWERS a signi cant distance rom its path. Hematuria is not
1. D. I recognized at the time o surgery, surgical divi- always present a er ureteral injury. riphasic C can
sion o the ureter and/or partial ureteral excision be used to evaluate tissue trauma and or ureteral
should be managed based on the location and length injury. C cystogram will usually identi y a bladder
o injury. Options or repair include ipsilateral ure- injury. High velocity bullets or missiles can create an
teroureterostomy, ureteral reimplantation with or energy wave that penetrates and injures distant tis-
without a psoas hitch, trans ureteroureterostomy. In sues. Damage to the ureters can present 3 to 5 days
occasions when the patient is unstable, the ureter can a er injury as increased drainage rom the surgically
be ligated with sutures, a nephrostomy tube is placed, placed drains. T is uid can be sent to pathology or
and reconstruction can occur within the next 48 to a creatinine level.
72 hours. I the patient is hemodynamically unstable,
then damage control is recommended and delayed BIBLIOGRAPHY
ureteral repair can be per ormed. Al-Ali M, Haddad L. T e late treatment o 63 overlooked
or complicated ureteral missile injuries: the promise o
2. A. ransureteroureterotstomy places the contralat- nephrostomy and role o autotransplantation. J Urol. 1996;
156:191821.
eral renal unit at risk and is contraindicated in
Carver B, Bozeman C, Venable D. Ureteral injury due to pen-
patients with a prior history o urinary stone disease. etrating trauma. South Med J. 2004;97:4624.
Both upper tracts are at risk o there is any problem Elliot S, McAninch JW. Ureteral injuries: External and iatro-
with one side. T e other reconstructive surgeries genic. Urol Clin North Am. 2006;33:5566.
can be implemented in the setting o known stone Fugita OE, Kavoussi L. Laparoscopic ureteral reimplantation
disease. or ureteral lesion secondary to transvaginal ultrasonogra-
phy or oocyte retrieval. Urology. 2001;58:281.
Ghali A, El Malik E, Ibrahim A, et al. Ureteric injuries: Diag-
3. C. T e use o permanent sutures within the urinary
nosis, management, and outcome. J Trauma. 1999;46:
tract is a nidus or stone ormation. T e proximal and 1508.
distal ends o the ureters should be spatulated and Iwaszko MR, Krambeck AE, Chow GK, Gettman M . rans-
sewn together in a tension- ree ashion with absorb- ureteroureterostomy revisited: Long-term surgical out-
able mono lament suture. A stent should be placed comes. J Urol. March 2010;183(3):10559.
across the watertight anastomosis. Contusions or Kristjnsson A, Mnsson W. Re uxing or nonre uxing ure-
teric anastomosis. BJU Int. November 1999;84(8):905
damaged areas o the ureter need to be debrided 10.
until the edges bleed prior to per orming ureteral Kunkle D, Kansas B, Pathak A, et al. Delayed diagnosis o trau-
anastomoses. matic ureteral injuries. J Urol. 2006;176:25037.
Medina D, Lavery R, Ross S, Livingston D. Ureteral trauma:
4. D. Short de ects involving the distal ureter should Preoperative studies neither predict injury nor prevent
be repaired with ureteroneocystostomy. Non-re ux- missed injuries. J Am Coll Surg. 1998;186:6414.
ing ureteral implants have a higher risk o stricture Minervini A, Boni G, Salinitri G, et al. Evaluation o renal unc-
tion and upper urinary tract morphology in the ileal ortho-
than re uxing ureteral implants. Re uxing anasto- topic neobladder with no antire ux mechanism. J Urol.
moses show no increase in complications related to 2005;173(1):1447.
urine re ux. T e principles o repair include spatula- Modi P, Goel R, Dodiya S: Laparoscopic ureteroneocys-
tion, lack o tension, stenting, postoperative drainage, tostomy or distal ureteral injuries. Urology. 2005;66:
and a watertight anastomosis with ne non-reactive 751.
Noble IG, Lee K , Mundy AR. ransuretero-ureterostomy: A
absorbable suture. T e contralateral bladder pedicle
review o 253 cases. Br J Urol. January 1997;79(1):203.
can be ligated when per orming a psoas hitch using Palmer JK, Benson GS, Corriere JN, Jr. Diagnosis and initial
in order to provide enough mobilization to per orm management o urological injuries associated with 200
a ureteroneocystostomy. Stay suture can improve tis- consecutive pelvic ractures. J Urol. 1983;130:7124.
C H AP TER 1 0 6 TRAU MATIC U RETERAL IN j U RY 407

Perez-Bray eld M, Keane , Krishnan A, et al. Gunshot Wessells H, Deirmenjian J, McAninch J. Preservation o renal
wounds to the ureter: A 40-year experience at Grady unction a er reconstruction or trauma: Quantitative
Memorial Hospital. J Urol. 2001;166:11921. assessment with radionuclide scintigraphy. J Urol. 1997;
Reddy PK, Evans RM. Laparoscopic ureteroneocystostomy. 157:15836.
J Urol. 1994;152:2057. Wein AJ, Kavoussi LR, Novick AC, Partin AW, Peters CA.
Schimp MO, Wagner JR. Robot-assisted laparoscopic boari Chapter 42, Upper Urinary ract rauma. In: Richard AS,
ap ureteral reimplantation. J Endourol. 2008;22:2691. Leon RD, eds. Campbell-Walsh Urology. 2012 ed. Philadel-
Velmahos G, Degiannis E. T e management o urinary tract phia, PA: Elsevier Saunders; 2007;116992.
injuries a er gunshot wounds o the anterior and posterior Wiesner C, T urof JW. echniques or uretero-intestinal
abdomen. Injury. 1997;28:5358. reimplantation. Curr Opin Urol. 2004;14(6):3515.
107
Prostate Cancer

Joseph R. Sterbis

A 58-yr-old male is diagnosed in urology clinic with A. Omental ap coverage ollowing primary repair
clinical 2a Gleason 3 + 4 = 7 prostate cancer. His B. Resection o the injured rectum, ollowed by
serum PSA is 4.2. He has a history o hypertension and primary anastomosis
no prior surgical history. He has no prior history o C. Resection o the injured rectum, primary anasto-
radiation or chemotherapy administration. mosis and diverting ileostomy
D. Diverting colostomy a er resection even in a
1. Which treatment options should NO be recom-
patient without prior radiation therapy
mended to the patient?
E. Double layer, air-tight closure o the rectal injury
A. Radical prostatectomy with bilateral pelvic
lymph node dissection 4. Four months later, the patient is scheduled or a
B. External beam radiation therapy laparoscopic cholecystectomy. T e operating room
C. Brachytherapy seed implants nurse has dif culty placing a oley catheter. What
D. Active surveillance is the most likely source or this dif culty?
A. Anastomotic contracture at the bladder neck
2. T e patient elects or a radical prostatectomy with
B. Prostate cancer recurrence at the bladder neck
bilateral pelvic lymph node dissection. During the
C. Urethral stricture
pelvic lymph node dissection, the le obturator
D. Rectourethral stula
nerve is accidentally divided with metzenbaum
scissors. It is recognized immediately. How do you 5. T e patient experiences a disease recurrence and is
address this complication? initiated on androgen deprivation therapy (AD ).
A. Because the division was done sharply, leave Which o the ollowing is true regarding this
the two ends ree ollowing completion o the treatment?
lymphadenectomy. A. T e patient is at reduced risk or cardiac
B. Reapproximate the two ends with ne, perma- morbidity.
nent suture. B. T e patient is at increased risk or loss o lean
C. Per orm a nerve gra , approximating the two muscle.
ends. C. T e patient is at reduced risk or osteoporosis.
D. All o the above D. T e patient is at increased risk or loss o body
E. A or C at.
3. During the apical prostatic dissection, a small 1
ANSWERS
cm ull thickness rectal injury is incurred. Which
o the ollowing is the best course o action at this 1. D. Active surveillance is appropriate or men
time? with low risk prostate cancer, de ned as having
C H AP TER 1 0 7 P Ro s TATE C An C ER 409

organ-con ned pathology, < 0.2 mL, a Gleason sum have demonstrated a loss o muscle mass and increase
less than or equal to 6, and no Gleason grade 4 or 5 in percentage o body at. AD can adversely a ect
in the biopsy specimen. body habitus, glucose metabolism, lipid pro les, and
increase cardiovascular morbidity and mortality.
2. B. I the obturator nerve is accidentally divided, it
should be reanastamosed with ne, non-absorbable BIBLIOGRAPHY
sutures. T e patient can make a ull unctional Borland RN, Walsh PC. T e management o rectal injury dur-
recovery. ing radical retropubic prostatectomy. J Urol. 1992;147:905
907.
3. D. Rectal injury is a serious intraoperative compli- Daniell HW, Dunn SR, Ferguson DW, et al. Progressive osteo-
cation. I a rectal injury occurs, the prostatectomy porosis during androgen deprivation therapy or prostate
should be completed. Small injuries that are less than cancer. J Urol. 2000;163:1816.
50% o the circum erence o the rectum can be treated Epstein JI, Walsh PC, Carmichael M, et al. Pathologic and clin-
ical ndings to predict tumor extent o nonpalpable (stage
without resection. T e rectum can be closed primar- 1c) prostate cancer. JAMA. 1994;271:36874.
ily, and a piece o omentum should be placed between Krupski L, Smith MR, Lee WC, et al. Natural history o bone
the rectal closure and the vesicourethral anastomosis complications in men with prostate carcinoma initiating
to reduce the incidence o rectourethral stula. T e androgen deprivation therapy. Cancer. 2004;101:5419.
rectal de ect should be closed in two layers and the Nelson CJ, Lee JS, Gamboa MC, Roth AJ. Cognitive e ects
o hormone therapy in men with prostate cancer. Cancer.
wound copiously irrigated with antibiotic solution. I
2008;113:1097106.
the patient has had radiation therapy prior to surgery, Saigal CS, Gore JL, Krupski L, et al. Androgen deprivation
a diverting colostomy should be per ormed. therapy increases cardiovascular morbidity in men with
prostate cancer. Cancer. 2007;110:14931500.
4. A. wo to ve percent o patients develop a blad- Spaliviero M, Steinberg AP, et al. Laparoscopic injury and
der neck contracture approximately 6 to 12 weeks repair o obturator nerve during radical prostatectomy.
a er an open prostatectomy. T e initial management Urology. November 2004;64(5):1030.
ubaro A, Carter S, et al. A prospective study o the sa ety
includes dilation with urethral sounds or a direct
and ef cacy o suprapubic transvesical prostatectomy in
vision incision o the bladder neck. patients with benign prostatic hyperplasia. J Urol. 2001:
166(1):1726.
5. B. General complications o androgen ablation Van Londen G, Levy ME, Perera S, et al. Body composition
include osteoporosis, hot ashes, and decline in changes during androgen deprivation therapy or prostate
cognitive unction, increased cardiovascular mor- cancer: A 2-year prospective study. Crit Rev Oncol/Hematol.
bidity and mortality, and changes in body habitus. 2008;68:1727.
Varkarakis I, Kyriakakis Z, et al. Long-term results o open
Bone mineral density can decrease, and the longer
transvesical prostatectomy rom a contemporary series o
the patient remains on AD , the greater the risk o patients. Urology. 2004;64(2):306310.
racture. AD has been linked to cognitive decline Vasilev SE. Obturator nerve injury: a review o management
in men with prostate cancer. Men treated with AD options. Gynecol Oncol. 1994 May;53(2):1525.
This page intentionally left blank
Index

A American T yroid Association (A A), 216, 218 Attenuated FAP (AFAP), 173
Abbe ap, or lip de ects, 41, 4243 Anal stula, treatment o , 186, 187 Auriculotemporal nerve, 223
Abdominal aortic aneurysm (AAA), 331332 Anastrazole, 193 Autologous tissue reconstruction, 202, 205.
abdominal ultrasound or, 331, 332 Ancrod, 306 See also Breast reconstruction
endovascular repair (EVAR) vs. open repair, Androgen deprivation therapy (AD ), 408, disadvantage o , 205
331, 332 409 Axillary lymph node dissection, 200
natural history o , 332 Aneurysmal rupture, 274 Azathioprine, 322, 323
repair o , indications or, 331, 332 Ankle-brachial index, 254
risk actors with ormation o , 331 Anorectal manometry, 237 B
screening or, 331332 Anterior cruciate ligament (ACL) tear Babinski sign, 269
Abdominal pain, a er laparoscopic gastric complication a er surgical reconstruction, Back pain, 371373
bypass, 103105 388, 391 disc herniation and, 371, 372
Abdominal wall reconstruction, 6163 gra s or ACL reconstruction, 390391 evaluation o , 371, 372
Abdominal wound, 253, 254 Lachman test or, 388, 390 lumbar herniated discs, 371, 372373
FAS examination in, 253, 254 meniscal tears with, 388, 390 microdiscectomy and complications, 372,
Accessory spleens, 150151, 152 treatment or, 388, 390 373
Achalasia, 7981 Anterior drawer test, 390 Bariatric procedures, 109, 110. See also Bariatric
manometry or diagnosis o , 79, 80 Anteroposterior compression (APC III) surgery
treatment or, 21, 23, 79, 8081 racture, 262, 263 duodenal switch, 109, 110
Acute atty liver o pregnancy (AFLP), 133, 134 Anthracycline, 200 malabsorptive e ects, 110
and HELLP syndrome, 134 Antidiuretic hormone (ADH), 280 nutrition complications, 115116
Acute mesenteric ischemia (AMI), 156, 158 Anus-rectal cancer, 183185 restrictive e ects, 109
Acute respiratory distress syndrome (ARDS), Aortic trocar per oration, 1920 Bariatric surgery
292, 294 APACHE score, 294 candidacy or revision, 117, 118
Acute subdural hematoma (aSDH), 266, 271 Apley compression test, 390 ideal candidates or, 117118
Acute tubular necrosis (A N), 322 Appendicitis, 167169 revisional surgery, 117, 118119
Adenoid cystic carcinomas, 222 clinical mani estations, 167, 168169 Barretts esophagus without dysplasia, 95, 96
Adenomatous polyposis syndromes, 172 con rmatory imaging in, 167, 169 Basal cell carcinoma, 42, 43
Adrenal cortical carcinoma, 210 laparoscopic appendectomy or, 168, 169 treatment or, 41, 42
Adrenal gland tumors, 209211 overview, 168 Base de cit, 293
Adrenal mass, 210 and pregnancy test, 167, 168 Becks riad, 254
laparoscopic adrenalectomy or, 211 surgical management or, 167, 169 Beckwith-Wiedemann syndrome, 239, 240
percutaneous biopsy o , 210 Argon beam coagulation, and venous Benign prostatic hyperplasia (BPH), 315
pheochromocytoma evaluation or, 210 embolism, 1315 Bethesda criteria, 185
treatment o , 209, 210, 211 cautions o manu acturer, 1415 Bevacizumab, 356
Adrenocortical carcinoma, 211 management in, 13, 15 Bilateral neck exploration, 212
Adrenocorticotropic hormone (AC H), 57, 209 risk actors or, 13, 14 Bi-layer mesh repair, 72
Advanced cardiac li e support (ACLS), 246 Arterial pressure index, 254 Bile microscopy, 128
Advanced directives, 37, 38 Arteriovenous (AV) stula, 334336 Biliary colic, 127129
oral, 38 distal revascularization and interval ligation and gallbladder cancer, 127, 128
Advanced trauma li e support (A LS), 246 (DRIL) procedure, 335336 medical therapy or, 127, 128
Aganglionic bowel, 237 in ected AV stula gra and management, open cholecystectomy in, 128
Air-contrast barium enema (ACBE), 173 335, 336 repeat right upper quadrant ultrasound in,
Airway re, 11, 12 placement in contralateral arm, 334, 335 127, 128
Airway pressure release ventilation (APRV), temporary dialysis access, 334, 335 ultrasound in, 127, 128
292 Ascites Biliary disease, in pregnancy, 133136
Aldosterone, 9, 210 cirrhosis and, 141 Biologic targeted therapies, or ovarian cancer,
Alpha-adrenergic blockade, 210. See also pericentesis in, 140, 141 356
Pheochromocytoma Aspirin, or vertebral artery injuries, 257 Bipolar energy devices, 30
side e ects o , 210 Asymptomatic microscopic hematuria (AMH), BIRADS 4 lesion, 191, 192
American Society o Colon and Rectal 397 Bladder cancer, 397398
Surgeons (ASCRS), 180, 181 Atracurium, 7 Bladder rupture, 312

411
412 In d e x

Blast injuries, 246, 247248 Carotid endarterectomy (CEA), 330 Chronic venous insu ciency (CVI), 341, 343,
primary, 246, 247248 emergent, 330 344
quaternary, 248 Carotid occlusion, 329330 medical management, 343, 345
secondary, 248 atropine during surgery, 329, 330 surgical management, 343, 345
tertiary, 248 carotid endarterectomy in, 329, 330 Chvosteks sign, 212
Bleomycin, 229 crescendo transient ischemic attacks ( IA) Cirrhosis
Bleomycin-induced pulmonary brosis, 402, and, 330 abdominal wall hernia ormation, actors
403 C angiography (C A) in, 329, 330 or, 64, 65
Blood products and trans usion, 304306 emergent carotid endarterectomy, 330 elective repair o hernias in, 64, 65
Blunt trauma, 263 neurological events in recovery room and perioperative mortality risk, 64, 65
Bochdalek hernias, 78 re-operation, 329, 330 preoperative anemia and postoperative
Body sur ace area (BSA), 298 string sign, and urgent carotid complications, 64, 65
Brain cell swelling, 273 endarterectomy, 329, 330 Cistatracurium, 7
Brain injury, treatment o , 260 Carotid sinus, 330 Clean intermittent catheterization (CIC), 314
Branchial cle cysts, 227, 228 Catheter cerebral angiography, 269 Clevidipine, 9
Branchial cysts and stulas, 229 Catheter-related bloodstream in ections Clindamycin, 309, 310
BRCA1 gene, 191, 192 (CR-BSI), 303 Clostridium dif cile in ection, 292, 293
BRCA2 mutations, 192 Cauda equina syndrome (CES), 372 Colectomies, robotic-assisted, 29
Breast cancer, 192 Ce azolin, 284 Colon cancer, 171175, 183185
chemotherapy or, 193 Cellulitis, 223 colonoscopy in, 171, 173, 183, 184
conserving surgery, 193 Central venous pressure (CVP), 291 C scan in, 171, 173
core biopsy o , 192 Cerebral blood ow, 273 Enhanced Recovery A er Surgery (ERAS)
eatures o , 192 Cerebral contusions, 273 protocols, 172, 174175
and hereditary syndromes, 192 Cerebral per usion pressure (CPP), 260 hereditary syndromes o gastrointestinal
radiation therapy or, 193 Cerebral salt wasting (CSW), 280 tract and, 172173
risk actors or, 192 Cerebral vasospasm, 269 laparoscopic colonic resection or, 171172,
stages, 205 Cerebrospinal uid (CSF), 265 174
ultrasound or, 192 dural injury with persistent, 265 Colon neoplasm, 180
Breast conservation therapy (BC ), 195, 196 post-traumatic, 266, 271 Colonoscopy, 180
Breast mass, 191194 Cerebro vascular resistance (CVR), 273 in cancer diagnosis, 184
Breast reconstruction, 202205 Cervical spine clearance, 256257 Colorectal metastatic cancer, neoadjuvant
autologous/synthetic, 204 Charcot de ormity, 308 chemotherapy or, 138, 139
autologous tissue reconstruction, 202, 205 CHARGE syndrome, 239, 240 Comedonecrosis, 196
sentinel lymph node biopsy, 202 Chemical injury, o eyes, 282, 283 Compressive therapy, or lymphedema, 340, 341
and tissue expanders, 205 uorescein exam, 286 Computed tomographic (C ) scan
Burns and smoke inhalation, 297299 Chest trauma abdomen and pelvis with IV contrast, 312
chemical injury and, 298 and pulmonary complications, 290 colorectal cancer, 184
rule o nine, 298 and tracheobronchial injury, 289, 290 head, 265, 267
Burr-hole drainage, 270, 275 urgent bronchoscopy in, 289, 290 kni e wounds, 249, 251
Cholangitis, 130132 multiple hemorrhagic brain contusions, 267
C causes o , 130131 right rontoparietal chronic subdural
Cadaver allogra , 299 gallstones and, 130 hematoma, 269
Calcaneus ractures, 377379 MRCP or, 130, 131 sigmoid diverticulitis, 179, 180
C scan in, 377, 378379 pathophysiology o , 130, 131 transmediastinal gunshot wound, 289
delayed open reduction and internal xation in pregnancy, 136 Congenital muscular torticollis, 230
in, 378, 379 septic, 130, 131132 Conns syndrome, 10
displaced, intra-articular racture, treatment Cholecystectomy, 128, 220 Contrast esophagography, 251
o , 378, 379 laparoscopic, 21, 22 Contrast esophagram, or dysphagia, 79, 80
and lumbar spine injuries, 377, 378 Cholecystitis, in pregnancy, 133136 Coopers ligament repair, 71, 72
mechanism o injury or, 378 and cholecystectomy, 136 CORAL trial, 10
operative treatment, outcomes o , 378, 379 ERCP with sphincterotomy in, 134, Core needle biopsy, 222
plain x-rays o oot in, 378 136 Corticosteroids, in subdural hematoma
splint placement and elevation, 378, 379 gangrenous cholecystitis, 135 treatment, 275
Cantholysis, steps or, 285 imaging or, 133, 134135 Count ngers (CF), 286
Capacitive coupling, monopolar instruments, management o , 135136 Cowden syndrome (CS), 173, 192
30 Cholecystokinin stimulated cholescintigraphy, CPAP, in obstructive sleep apnea, 5, 7
Carbon monoxide poisoning, 298 128 Cranioplasty, 270
Carcinoembryonic antigen (CEA), 183 Cholelithiasis. See Biliary colic Craniostomy, twist drill, 270, 275
Carcinoid crisis, 161 Cholestasis, 302 Craniotomy, 270, 275
Carcinoid tumors, 160162 Chronic subdural hematoma (CSDH), 270 Crepitus, a er mediastinal dissection, 29
Chromogranin A levels in, 161 age and, 270, 275 Cricoid cartilage, 249, 251
requent location o , 161 bilateral e usion and, 270, 275 Crohns disease, 176178
metastases, 161, 162 development o , 270 approach or surgical management, 176,
surgery or metastatic disease, 161, 162 recurrence rate o , 270 177
urine 5-HIAA levels in, 161162 treatment o , 270, 275 classi cation o , 177
In d e x 413

colorectal cancer in, risk o , 177178 4 Ds, 220 Enteral nutrition, 301, 302
in iximab in, 176, 177 Ductal adenocarcinoma, 192 Epidural hematoma (EDH), 266
non-operative management, 176, 177 Ductal carcinoma in situ (DCIS), 191, 192, causes o , 271
operative intervention or, 176, 177 195198, 205 cranial, 266
and perirectal abscesses, 187 imaging in, 195 complications and outcome o , 267,
C angiography (C A), 249, 288 patient, mortality risk in, 196 272273
and 3-dimensional reconstruction imaging, risk actors or, 196 diagnosis o , 267, 272
274 vs. LCIS, 198 management o , 267, 272
o kni e wounds, 249 Duodenal per oration, incidence o , 234 delayed, 273
C colonography, 184 Duodenal switch procedure, 109, 110 occurrence o , 271
Cushings syndrome, 10, 209, 210 anatomy, 110111 Epirubicin, 200
causes o , 210 bene ts o , 109, 111112 Epstein-Barr virus (EBV), 321
signs and symptoms o , 210 candidacy or, 110 Erysipelas, 309
Cushings triad, 273 complications o , 109110, 112 Escharotomy, 297, 299
Cutaneous lichen amyloidosis, 216 and gastric bypass, 110111 Esophageal per oration
Cyanide poisoning, 298 malabsorptive e ects, 109, 110 gastrogra n esophagography in, 95, 96
nitroprusside and, 8, 9 perioperative complications, and emergent le posteriolateral thoracotomy in, 95, 9697
Cyclophilin protein, 322 surgical exploration, 110, 112 non-operative management in, 95, 96
cyclosporin binds, 322, 323 Duodenal ulcers, 217 operative management in, 96
Cyclophosphamide, 200 Durable power o attorney, 38 Esophagogastroduodenoscopy (EGD), 79, 80,
Cystic hygromas. See Lymphatic mal ormation 128, 250
Cytomegalovirus (CMV), 306 E Estrogen receptor-positive (ER+) tumors, 193
Cytoreduction, 355 Ectopic cordis, 239 European Organisation or Research and
Cytoreductive nephrectomy, 399, 400 Ectopic pregnancy, 358361 reatment o Cancer (EOR C), 196
Cytotoxic brain edema, 274 active, 360 Evaluation o Genomic Applications in Practice
DEME ER trial, 361 and Prevention (EGAPP), 185
D hCG levels in, 358, 359360 External ori ces, o stula track, 187. See also
Damage control resuscitation (DCR), 246, 247 implantation sites, 360 Perirectal abscess
Damage control surgery (DCS), 245, 247 medical treatment with methotrexate, 358, Extra-corporeal membrane oxygenation
Decubitus ulcers, 4446 360 (ECMO), 292
Deep in erior epigastric per orator (DIEP) ap, recurrent, rate o , 359, 361 Extracorporeal shockwave lithotripsy (ESWL),
204, 205 risk actors or, 359 128
Deep tendon re exes (D R), 266 salpingectomy or, 358, 360361 Extraordinary care, and ordinary care, 38
Deep venous thrombosis, 218 surgical management, 360361 Extremity sarcomas, 50, 51
Dermato brosarcoma protuberans (DFSP), 52 trans-vaginal ultrasonography in, 358, 359 histologic subtypes, 50, 51
Dermoid cysts, 228 Ectopic thyroid tissue, 228 incisional biopsy or, 51
Dexamethasone, 260 Elbow injuries, in children, 384385 local recurrence a er treatment or, 52
suppression test, 210 Electrosurgery radiation therapy, 50, 52
Diabetes insipidus (DI), 280 bipolar instrumentation, 30 site o metastasis, 51
nephrogenic, 280 coagulation mode, 30 surgical management o , 50, 5152
neurogenic, 280 cut mode, 30 Eye exam
treatment o , 280281 monopolar instruments, 28, 30 primary survey, 286
Di use esophageal spasm (DES), 81 Elevated intracranial pressure, management o , secondary survey, 286
Di use large B-cell lymphoma (DLBCL), 259261
gastric, 89, 9192 End o li e care, 3738 F
Di use post traumatic intracerebral advance directives in, 37, 38 Facial hemihypoplasia, 230
hemorrhage, 273 decision making in, 3738 Familial adenomatous polyposis (FAP), 173,
Di usion tensor imaging (D I), 271 li e-sustaining medical treatment, 37, 38 185
Discectomy, 372, 373 Endometriosis, 366368 Familial medullary thyroid carcinoma (FM C),
Disc herniation, 371, 372 with bowel involvement, 366367, 368 218
Diverticulitis, 179181 deep in ltrative lesions (DIE), 367 Fecal immunochemical testing (FI ), 173
colon resection in, 179 de nition o , 367 Feculent peritonitis, 181. See also Diverticulitis
complicated endometriomas, 367 Femoral hernia, 68, 70, 7172
treatment o , 179, 180181 laparoscopic evaluation o , 366, 367 McVay repair or, 71, 72
elective resection in, 180 medical therapy, 367 preperitoneal repair technique or, 71, 72
sigmoid presentations, 366, 367 recurrence o , etiology or, 71, 72
diagnosis o , 180 site o intestinal, 366, 367 Femoral-saphenous AV stula, 335
treatment o , 179, 180 super cial lesions, 367 Fenoldapam, 9
DNA mismatch repair genes, 185 surgical treatment or, 366, 368 Fibrin glue, 186, 187
Doctrine o parens patriae, 36 Endorectal ultrasound, 184 Filariasis, 341
Dopamine, 293 Endoscopic ne needle aspiration biopsy, 219 Fine-needle aspiration (FNA), 192, 215, 216,
Doppler ultrasound, in vessel patency EndoWrist technology, 26 221, 229
evaluation, 321 Enhanced Recovery A er Surgery (ERAS) Fires, operating room, 1112
Double e ect, principle o , 38 protocols, or colon surgery, 172, First rib resection, complications o , 348, 349
Doxorubicin, 200 174175 Fistula-in-ano, 187
4 14 In d e x

Fistula, occurrence o , 187 proton pump inhibitor in, 94, 9596 Hepatic steatosis, 302
Fistulotomy, conventional, 187 and sliding hiatal hernia, 9495, 96 Hepatitis C virus (HCV), 324, 325
Fitzhugh-Curtis syndrome, 363 Gastrogra n esophagography, 95, 96 Hepatobiliary (HIDA) scan, 128
Fluid-attenuated inversion recovery (FLAIR), Gastrointestinal stromal tumors (GIS ), 52, Hepatocellular carcinoma (HCC), 324
269 8688 Hereditary nonpolyposis colon cancer
Fluid restriction, 279 diagnosis o , 86, 87 (HNPCC), 185, 192, 354
5- uorouracil, 200 KI positive, 86, 87 Hiatal hernia, 77
Foam sclerotherapy, 345346 metastatic, 88 endoscopy or, 7778
Focused assessment with sonography or preoperative therapy with imatinib in, 86, etiology o , 77, 78
trauma (FAS ), 253, 254, 312 8788 phrenoesophageal ligament weakening and,
Foley trauma, 314315 prognostic actors in, 88 77, 78
Freys syndrome, 223 recurrence ree survival in, 87, 88 slipped Nissen, 77, 78
Fulminant hepatic ailure, 325 solitary, 87, 88 types o , 77, 78
Furosemide, 273 Gastrointestinal tract, hereditary syndromes o , Hickman catheter, 301, 303
172173 High microsatellite instability (MSI-H), 185
G Gastroschisis, 239240 Hirschprungs associated enterocolitis (HEC),
Gail Model, 192 management o newborn with, 239, 240 238
Gallbladder cancer (GBC), 123125 and omphalocele, 240 risk actors or, 238
anatomical resection vs. wedge resection, Genitourinary trauma, 312313 Hirschsprungs disease, 216, 236238
124, 125 Gentamicin, 284 constipation and, 237
MDC ndings, 123, 124125 Glasgow coma scale (GCS), 259, 262, 265 diagnosis o , 237
peritoneal involvement with, 125 components o , 259260 multiple intraoperative biopsies or, 238
prognosis o , 125 trauma bay with, 279 surgery or, 237, 238
radical cholecystectomy or, 123, 125 Glomerular ltration rate (GFR), decreased, Hodgkins disease, 230
risk actor or, 123, 124 399, 400 Hollow-point bullets, 253
Ganglion cells, 237 Glomerulonephritis, 10 Hormonal therapy, or breast cancer, 193
Gastric bypass and bowel obstruction, 103105 Glucagonomas, 220 Horner syndrome, 230
Gastric cancer, 8285 Glucose urine strips, 271 Horseshoe abscesses
CLASSIC trial, 84 Goodsalls rule, 186, 187 intersphincteric plane, 187
di use, 84 Gra ing o deep burn, 299 ischiorectal plane, 187
Intergroup rial 0116, 84 Gravity cystogram, 313 management o , 186
intestinal type, 84 Greater saphenous vein (GSV) stripping supralevator plane, 187
Irishs node and, 82, 85 procedure, 345 Horseshoe stula, 187
neoadjuvant therapy or, 82, 84 Groin pain, a er hernia surgery, 73, 74 Houns eld units, 271
preoperative locoregional staging or, EUS intentional ilioinguinal nerve division and, Hurthle cell neoplasms, 216
or, 82, 83 74, 7576 Hydralazine, and re exive tachycardia, 8, 9
Stage III, treatment or, 82, 8485 management o , 73, 74 Hydrocephalus, 269
Gastric lymphoid tumors, 8992 nerve injuries in hernia repair and, 73, 7475 Hydrocephalus ex vacuo, 271
Ann Arbor Staging System, 90 primary nerve injury and, 73, 74 Hydroxocobalamin, 297, 298
endoscopic ultrasound and, 90 secondary nerve injury and, 74 Hyperaldosteronism, 10
high-grade, treatment strategy or, 89, 9192 triple neurectomy or, 73, 75 Hyperbaric oxygen (HBO), 298
Lugano Staging System, 90 Guaiac ecal occult blood test (gFOB ), 184 treatment, 309, 310
staging o , 89, 90 Gunshot wounds, and ureteral injury, 405, 406 Hypercalcemia, 213
stomach preserving treatment strategies Gustatory sweating, 223 Hyperglycemia
or, 90 and cerebral edema, 273
surgical intervention or, 89, 90 H severe, 302
Gastric MAL lymphomas Hand motion (HM), 286 signs o , 302
H. pylori eradication therapy or, 89, 91 Head and neck malignancies, 230 Hypermethylation analysis, 185
radiation therapy or, 89, 91 Head and neck pediatric tumors, 227230 Hypernatremia, 279281
Gastric ulcers, 83 Head, Eye, Ear, Nose and T roat Exam de ned, 280
biopsy o , 82, 83 (HEEN ), 269 Hyperparathyroidism, 212214, 218
giant, 83 HELLP (Hemolysis, Elevated Liver enzymes, primary, 213
Gastrinoma, 219 Low Platelets) syndrome, 134 retroesophageal space, 212, 213
localization o , 219 Hemodialysis Hypokalemia, 210
Gastrinoma triangle, 219 complications o , 337339 Hypomagnesemia, 306
Gastroduodenal ulcer, per orated dyspnea/chest pain and, 337, 338 Hyponatremia, 269, 275, 279281
chest x-ray or diagnosis o , 156, 157 hypotension and, 337, 338 diagnosis o , 280
H. pylori in ection and, 157158 and muscle cramps in legs, 337338 neurological signs and symptoms o , 279
nonoperative management, 156, 157 Hemodynamic instability, 262, 263 and transtentorial herniation, 279
stimulant drug abuse and, 156, 157 Hemorrhagic brain contusions, 271 Hypotension, intradialytic, 337, 338
surgical management, 156, 157 Hemostatic resuscitation, 247
Gastroesophageal re ux disease, 9497 Hemothorax, retained, 251 I
esophagogastroduodenoscopy in, 94, 9596 Heparin-induced thrombocytopenia (HI ), Idiopathic thrombocytopenic purpura (I P),
uid collection post-operative, 95, 96 304, 306 150152
Nissen undoplication in, 95, 96 Hepatic unction tests, 184 accessory spleens in, 150, 152
In d e x 415

pathophysiology o , 151 Intussusception Laparoscopic gastric bypass, bowel obstruction


postoperative complications, 151, 152 in adult, 160, 161 a er, 103105
splenectomy or, 150, 151 causes or, 161 and abdominal pain, 103, 104
steroid-re ractory, 150, 151 in children, 161 C scan in, 103, 104
thrombopoietin ( PO) receptor agonists in, surgery or, 161 diagnostic laparoscopy in, 103104, 105
role o , 151, 152 Ipsilateral cancer, 192 emergent exploration in, 103, 104105
Iliac vein, injury to, during trocar placement, Ipsilateral pupillary dilation, 272 and internal hernia, 103, 104, 105
19, 20 Irishs node, 82, 85 intusseption at jejunojejunostomy in, 103, 104
Ilioinguinal nerve, injury to, 73, 75 Iron de ciency anemia, a er Roux-en-Y gastric Laparoscopic surgery
Imatinib mesylate, 52 bypass (RYGB), 115116 CO2 insuf ation in, 28, 2930
Imiquimod, or super cial basal cell carcinoma, Ischemic steal syndrome, 334, 335 or colon cancer resection, 174
42 energy devices in, sa e use o , 28, 30
Immunohistochemical ecal occult blood test J robotic-assisted (See Robotic-assisted
(iFOB ), 184 JD Robinson ormula, 5, 6 laparoscopic surgery)
In antile hypertrophic pyloric stenosis (IHPS), Jehovahs Witnesses (JW) robotic-assisted vs. traditional, 29
232 blood products acceptance and Lateral canthotomy, 282, 285
acid-base dys unction, 232 consequences, 35, 36 Lateral compression (LC III) racture, 262
diagnosis o , 232, 233 blood trans usions and, 3536 Latissimus dorsi myocutaneous ap, 204, 205
distal per oration o , 234 dis ellowshipping, 35, 36 Latissimus muscle, 200
pathogenesis o , 232 Junctional injuries, 246 Le Fort II/III injuries, 257
In erior vena cava (IVC), 211 Juvenile polyposis syndrome (JPS), 173 Levophed, 293
In ammatory breast cancer (IBC), 193, Levophed in usion, 291
199200 K Lichtenstein tension ree hernia repair with
endocrine therapy or, 200 Kernohans notch, 267, 271 mesh, 72
neoadjuvant chemotherapy or, 199, 200 Kidney stone Li e-sustaining medical treatment, 37, 38
pathophysiology o , 199 analgesia or renal colic, 395, 396 Li-Fraumeni syndrome, 192
radiation therapy or, 199, 200 immediate decompression o renal unit, Ligaments o Cooper, 194
surgery in, 199, 200 395, 396 Limb salvage surgery, or extremity sarcomas,
tumor emboli in, 199, 200 test or, 395 51
In iximab, or intra-abdominal stulas, 176, Kinetic energy equation, 253 Lip carcinoma, 41, 4243
177 Knee dislocations Liver cirrhosis, 140142
Inguinal hernia and emergent closed reduction with and ascites, 140, 141
asymptomatic, 67, 68 sedation, 386, 388 gastrointestinal bleeding in, antibiotic
complications a er open repair o , 67, 68 neurovascular injuries with, 387 prophylaxis and, 141, 142
direct, 68 popliteal artery, injury to, 387, 389 non-hepatic surgery in, 140, 141142
elective surgery and watch ul waiting, 67, 68 radiographs in diagnosis o , 386, 388 with spontaneous bacterial peritonitis, 141,
indirect, 68 vascular injury with, 386387, 388389 142
laparoscopic repair, 67, 69 Knee injuries, 386391 umbilical hernia and, 140, 141
nerve injury rom repair o , 67, 69 Kni e wounds, 249251 Liver unction tests (LF ), 302
open repair o , 69 chest X-ray and FAS scan o , 250 Liver, herniated, 239, 240
pantaloon-type hernia, 68 C and C A o , 249, 251 Liver metastasis, 137139
Inguinal neuralgia, 7376 hypotension in, 249, 250 colorectal metastases, liver resection o , 137,
Intercostobrachial nerve, injury, 192, 194 le anterolateral thoracotomy, 249, 250 138139
Intersphincteric abscesses, 187, 188 patients, ABCs o , 250 concomitant colon and liver resections in,
Intestinal atresia, 240 patients, management o , 249 137, 138
Intestinal malrotation, 236, 237 tension pneumothorax, 250 and neoadjuvant chemotherapy, 138, 139
Intestinal obstruction, distal, 237 KRAS mutation, 184 surveillance in, 137, 139
Intracerebral hemorrhage, 268 Krukenberg tumors, 85 Living wills, 38
Intracranial hematomas, 265275 Lobular carcinoma, 193
acute, 272 L Lobular carcinoma in situ (LCIS), 191
Intracranial hemorrhage, 267, 268 Labetolol, 9 vs. DCIS, 198
causes o , 268 Laboratory risk indicator or necrotizing Low anterior resection (LAR), 184
and temporal hematomas, 268, 273 asciitis (LRINEC) score, 308 Lower motor neuron (LMN), 265
Intracranial pressure (ICP), 260, 279 Lactated ringers (LR), 297 LRINEC scoring system, 309
dextrose and, 268, 273 Lamivudine, 325 Lumbar puncture, 272, 274
monitor, 260 Laparoscopic adjustable gastric band (LAGB) Lumpectomy, 191, 192, 193, 197
traumatic brain injury, 273 placement, 120, 121 Lung cancer
Intra-ductal papillary mucinous neoplasm complication a er, 120, 121 and hyponatremia, 5657
(IPMN), 147 gastric prolapse a er, 120, 121 non-small cell lung cancer (NSCLC), 55, 56
Intraocular pressure (IOP), 286 gastropexy or gastric prolapse prevention, paraneoplastic syndromes with, 5657
Intraperitoneal (IP) chemotherapy, or ovarian 121 PE scan in, 55, 56
cancer, 356 outcomes rom, 120, 121 relapse o NSCLC, 55, 56
Intra venous immuno globulin (IVIG), 321 treatment or gastric prolapse, right upper lobectomy, 55, 56
Intravenous pyelogram (IVP), 313 121122 small cell lung cancer (SCLC), 5657
Intraventricular catheter, 273 Laparoscopic cholecystectomy, 21, 22 surgical resection, contraindication to, 55, 56
416 In d e x

Lymphangiomas. See Lymphatic mal ormation Mean arterial pressure (MAP), 260, 273, 297 Mucosa-associated lymphoid tissue (MAL )
Lymphangiosarcoma, 341, 342 Mechanical colonic obstruction, 154 lymphoma, 51, 54
Lymphangitis, 342 Mechanical small bowel obstruction, 154 Helicobacter pylori in ection and, 51, 54, 91
Lymphatic disease, 340342. See also Median arcuate ligament syndrome (MALS), Multicenter Selective Lymphadenectomy rial
Lymphedema 164 II (MSL II), 48
Lymphatic mal ormation, 227, 229 Medullary carcinoma, 211 Multi-organ dys unction syndrome (MODS),
Lymphedema, 197, 340, 341 Medullary thyroid cancer (M C), 218 292
diagnosis o , 340, 341 Melanoma, 4749 acute kidney injury and, 292
secondary, 340, 341 acral lentiginous, 48 Multiple endocrine neoplasia (MEN) 2A
treatment or, 340, 341342 desmoplastic, 48 syndrome, 216, 218
Lymphedema praecox, 341 in-transit metastasis, 47, 49 chromosome, gene and pathology o , 219
Lymphedema tarda, 341 lentigo maligna, 48 Multiple endocrine neoplasia (MEN) 2B
Lymphoma, 53 management o , 41, 42 syndrome, 215, 216, 218
Helicobacter pylori in ection, and MAL metastatic sites or, 49 chromosome, gene and pathology o , 219
lymphoma, 51, 54 nodular, 48 Multiple endocrine neoplasia (MEN)
physical examination in, 50, 53 prognostic indicators in, 47, 48 syndrome, 211, 217, 218
risk actors or, 5354 sentinel lymph node biopsy or, 47, 48 chromosome, gene and pathology o , 219
role o surgeon in, 53 super cial spreading, 47, 48 Multiple gunshot wounds, 252254
staging or, 53 wide local excision or, 47, 48 arterial injury, 254
Lymphoscintigraphy, 340, 341 Merkel cell carcinoma (MCC), 41, 42 primary survey o , 253
Lynch syndrome (LS), 172, 185, 354 Merkel cell polyomavirus, 41, 42 resuscitative thoracotomy, 252, 253
Mesenteric ischemia, 163166 stabilization o patients airway, 252, 253
M acute, 164 urgent thoracotomy, 253
Ma enide acetate, or burn wound, 297, 299 chronic, 164 Mycophenolate mo etil, 322
Magnetic resonance angiography (MRA), diagnostic delays or, 163, 166 Myxo brosarcoma, 52
269 pathophysiologies or, 163, 164
Magnetic resonance cholangiopancreatography risk actors with, 163, 164165 N
(MRCP), 147 second-look laparotomy or, 163, 165166 National Comprehensive Cancer Network
or choledocholithiasis, 130, 131 surgical embolectomy or, 163, 165 (NCCN), 169
Magnetic resonance imaging (MRI), 180 venous, 164 National Surgical Adjuvant Breast and Bowel
and nonnecrotizing in ections, 309 Mesenteric venous thrombosis (MV ), 164 Project (NSABP), 193, 196
o pelvis, 183, 184 Mesh in ections, 63 Natural ori ce translumenal endoscopic
o sigmoid diverticulitis, 180 Staphylococcus aureus and, 63 surgery (NO ES), 22, 23
Magnetic resonance venography (MRV), 269 Metallic trocars, 30 Necrolytic migratory erythema, 220
Malignant hypertension, 810 Methemoglobin ormation, 274 Necrotizing asciitis, 308310
de nition o , 9 Methotrexate, 360 occurrence o , 309
goal o care in, 8, 9 Metoprolol, 9 score, 309t
papillary edema in, 8, 9 MIBI scan, 213 surgical debridement and, 308, 310
renal artery stenosis and, 8, 910 Microsatellites (MSI), 185 Necrotizing so tissue in ection, o perianal
sign o , 8, 9 Microscopic hematuria, 397 area, 186, 187. See also Perirectal abscess
Mallampati score, 6 and upper urinary tract tumors, 397, 398 Negative pressure wound therapy (NPW ),
Mammography, 192 urinary diversion with ileum, and electrolyte 45, 298
limitations o , 197 abnormalities, 397, 398 contraindications to, 44, 45
Marcy repair, or hernia, 72 work-up or, 397398 devices or, 45
Marginal ulcers (MU), 100102 Minimally invasive radioguided indications or, 45
gastric bypass surgery and, 100, 101 parathyroidectomy (MIRP), 213 macrode ormation o wound, 45
per oration o , 101, 102 Minor consent authority, 35, 36 microde ormation o wound, 45
risk actors or, 100, 101, 105 Mismatch repair protein analysis, 184 Neoadjuvant chemoradiation therapy, 184
smoking and, 100, 101 Model or End-Stage Liver Disease (MELD), Nephrolithiasis, 395396
symptoms o , 101 324 Neuroblastoma, 230
treatment o , 101102 Modi ed radical mastectomy (MRM), 199, Neurogenic diabetes insipidus, 280
Mastectomy, 196 200 Nissen undoplication, 95, 96
modi ed radical, 193 Mohs microsurgery, or basal cell carcinoma, complications o , 29, 95, 96
nipple-sparing, 202, 205 41, 42 Nitroprusside, 9
radical, 193 Molecular adenomatous polyposis (MAP), 173 side e ects o , 8, 9
simple, 193 Monoamine oxidase inhibitors (MAOI), 209 Non-depolarizing neuromuscular blockers, 67
skin-sparing, 202 Monopolar instruments, use o , 28, 30 Non-Hodgkins lymphoma, 230
Mayo End-Stage Liver Disease (MELD) score, Monro-Kellie hypothesis, 260 Non-occlusive mesenteric ischemia (NOMI),
65 Morel-Lavallee lesion, 262, 263 164
May-T urner syndrome (M S), 343, 344 Motor de cit, 267 Nutcracker esophagus, 80, 81
imaging modalities or, 344 Motor vehicle collision
management o , 344 cardiac contusion, 290 O
pulmonary embolism and, 343, 344 patient, 288, 290 Obesity hypoventilation syndrome (OHS), 6
Mcmurray test, 390 patient, VA S or, 288 criterion or, 5, 6
McVay repair, or groin hernias, 71, 72 Mucoepidermoid carcinoma, 221, 222 Obstructive sleep apnea (OSA), 57
In d e x 417

Octreotide, 220 gallbladder sludge and, 301, 302 Plasmapheresis, 151


or carcinoid crisis, 161 zinc de ciency, 301, 302 Platinum based therapy, or ovarian cancer,
Oculocardiac re ex, 286 Parkland ormula, 298 356
Omega-3 atty acid, 302 Parotidectomy Pleomorphic adenoma, 221, 222
Omeprazole, in gastroesophageal re ux disease, super cial, 221 Plug and patch repair, 72
95 total, 221 Pneumocephalus, 271
Omphalocele, 239240 Parotid gland, 222 compartments o , 271
and gastroschisis, 240 Pathologic complete response (pCR), 200 de ned, 271
and herniated liver, 240 Patient controlled analgesia (PCA), 289 tension, 271
management o newborn with, 239, 240 Pectoralis major muscle, 200 Pneumococci, 271
Operating room res, 1112 Pediatric trauma, 383385 PO antibiotics, in sigmoid diverticulitis
Ophthalmology trauma, 282286 and adults trauma patients, di erence treatment, 179
anti-nausea medications or, 284 between, 383, 384 Polymerase chain reaction (PCR), 291
primary care o , 282, 283 cervical spine evaluation and clearance, 383, Positive end expiratory pressure (PEEP), 259,
tetanus shot or, 284 384 292, 297
Orbital oor racture, 283 elbow injuries, 383, 384 Positron emission tomography (PE ), 183, 216
Orthotopic liver transplantation (OL ), 325 pediatric diaphyseal emur racture, 383, 385 in lung cancer, 55, 56
Osmotic demyelinating syndrome, 275 Pelvic racture, 262264 Post-mastectomy radiation therapy (PMR ),
Osmotic medical therapy, 273 hemodynamic instability o , 262 200
Ovarian cancer, 192 hemorrhage, 263 Post-transplant lymphoproli erative disorder
Ovarian masses, 353356 mortality o displaced, 262 (P LD), 322
adjuvant chemotherapy or, 353, 356 open, 262, 263264 Pregnancy
BRCA1 mutations and, 353, 354 diagnosis o , 264 appendicitis in, 134136
management strategy or, 353, 354355 management o , 262 cholecystitis in, 133136
surgical staging o , 353, 355356 patient, management o , 263 gall bladder disease in, 134
trans-vaginal ultrasound ( VS) or, 353, 354 Pelvic hemorrhage, 245, 246 imaging in, 134135
Pelvic in ammatory disease (PID), 362365 RUQ abdominal pain in, 134
P Penetrating trauma, 252254 Preperitoneal pelvic packing (PPP), 263
Paget-Schroetter syndrome, 348, 349 Pentalogy o Cantrell, 239, 240 Preperitoneal repair, 72
Pagets disease, 202 Peptic ulcer disease per oration, 9899 Presacral drainage, or rectal injuries, 187.
Pancreas mass esophagogastroduodenoscopy in, 98, 99 See also Perirectal abscess
borderline resectable disease, 147, 148 H. Pylori stool antigen testing in, 98, 99 Pressure sores
C scan with arterial and venous phases in, laparoscopic Graham patch with H. Pylori caring or, 45
146, 147 testing in, 98, 99 so tissue reconstruction, 44, 4546
in head o pancreas, 146148 nasogastric tube insertion with NPO status staging o , 45
neo-adjuvant therapy and resection, 146, and IV uid initiation in, 9899 therapeutic treatment o , closure strategy in,
147148 Percutaneous drainage, 180 44, 46
pancreaticoduodenectomy or, 146, 147 Percutaneous transhepatic cholangiography unstageable, 44, 45
Pancreatic endocrine tumors, 217220 (P C), 131132 Primary care provider (PCP), 227, 232
Pancreatic stula, a er pancreatic surgery, 147, Percutaneous twist-drill bedside drainage, 270, Primary myelo brosis (PMF), 151
148 275 Prince v Massachusetts, 36
Pancreatic neuroendocrine tumors (PNE ), Periorbital ecchymosis, 286 Propylthiouracil (P U), 10
217, 218 Perirectal abscess, 186188 Prostate cancer, 408409
clinical symptoms o , 218 Perirectal suppuration, cause o , 187 and active surveillance, 408409
ormal resection o , 217 Peritoneal dialysis, 337, 338 bladder neck contracture a er open
Pancreaticobiliary maljunction (PBM), 124 Peritonitis prostatectomy, 408, 409
Pancreaticoduodenectomy, 147, 148 management o , 337, 338339 obturator nerve, injury to, 408, 409
Pancreatic pseudocysts, 144, 145 peritoneal dialysis and, 337, 338 radical prostatectomy with bilateral pelvic
Pancreatitis Permissive hypotension, 247 lymph node dissection, 408, 409
acute, 144, 145 Per-oral endoscopic myotomy (POEM), or and rectal injury, 408, 409
causes o , 144, 145 achalasia, 21, 23 Proton pump inhibitor (PPI), 219, 294
diagnosis o , 144 Petersens hernia, 104 or gastroesophageal re ux disease, 94,
morality rate, 144, 145 Peutz-Jegher syndrome (PJS), 172 9596
pseudocysts and, 145 Pheochromocytoma, 10, 209, 210, 215, 399, 400 or marginal ulcers, 101102
trypsin in, 144, 145 alpha-adrenergic blockade, 210 Pseudocysts, pancreatic, 144, 145
Pancuronium, 7 beta blockers, 210 Pseudohyponatremia, 280
Papillary carcinoma, 228 normetanephrine levels, 210 Psuedoachalasia, 81
Parathyroid adenoma, 213 plasma- ree metanephrine marker, 210 Ptosis, unilateral, 274
surgical excision o , 213 surgery or, 210 Pulmonary metastatectomy, 399, 400
Parathyroid arteries, 214 Phlegmon, 181 Punch biopsy, 199
Parathyroid gland hyperplasia, 213 Picinabil, 229 Pyloric stenosis, 232234
Parathyroid hormone level (P H), 212 Pitressin, 280 Pyloric vein o Mayo, 234
Parenteral nutrition (PN), 301, 302 Pitressin,280 Pyloromyotomy, 232
catheter line in ection and, 301, 303 Pivot shi test, 390 adequate, 233
ormulations, 301, 302 Plagiocephaly, 230 intraoperative photograph o , 234
418 In d e x

Q Salpingectomy, 360361 non-operative management, 154155


17q chromosome, 191, 192 Sarcomas, 50, 5153 small bowel ileus and, 153, 154
Scalp wound, 265 surgical management, 154, 155
R antibiotics or, 265, 270 Small cell lung cancer (SCLC), 5657
Ramstedt pyloromyotomy, 232, 233 management o , 265 Smoking, and marginal ulcers, 101
Rectal adenocarcinoma, 184 marked cosmetic de ormity, 270 Spinal cord injury, 256257
Recto-sigmoid junction, 184 Scapula ractures, and pulmonary injury, 381 characteristics o , 257
Rectum cancer, 183185 Scapulothoracic dissociation, 380381 classi cation o , 257
chemoradiation o , 183, 184185 chest X-ray in, 380, 381 in-line cervical alignment, 257
Re exive tachycardia, 8, 9 diagnosis o , 380, 381 NEXUS criteria, 256, 257
Relative a erent pupillary de ect (RAPD), 283 Sciatica, 371373 Spinal shock, 256
Renal artery stenosis, and malignant Scintigraphy, somatostatin-receptor, 219 treatment protocols or, 257
hypertension, 8, 910 Sclerotherapy, in macrocystic lesions, 229 Splenectomy, 152
Renal artery thrombosis, 322323 Secretin stimulation test, 99 Spontaneous bacterial peritonitis (SBP), 141, 142
Renal cell carcinoma, 399, 400 Seidels sign, 283 Squamous cell carcinoma (SCC), o lip, 41,
Renal imaging, in blunt trauma patients, 313 Selective estrogen receptor modulator (SERM), 4243
Renal tumor, 399400 197 Stereotactic core needle biopsy, 191, 192
Renal ultrasound, 395 Sentinel lymph node biopsy (SLNB), 193, 195, Sternocleidomastoid muscle (SCM), 228
Renal vein thrombosis, 322 196197 Stimulant drug abuse, 156158
Restless leg syndrome, 302 or melanoma, 41, 42, 47, 48 Stool DNA testing, 173
Retrobulbar hemorrhage, 282, 284 or Merkel cell carcinoma, 42 Subarachnoid hemorrhage (SAH), 274
Retrograde urethrography, 312, 313 Sepsis and multi-organ ailure, 291295 causes o , 274
Retroperitoneal lymph node dissection Sepsis therapy, 291, 292 complications o , 274
(RPLND), complications o , 402, 403 Sequential Organ Failure Assessment (SOFA) nuchal rigidity, 274
Revisional bariatric surgery, 117119 score, 294 Subdural hygroma (SDHG), 271
Rhabdomyosarcoma, 230 Seromas, a er ventral hernia repair, 63 Submandibular gland, 222
Rigid proctoscopy, 183, 184 Serrated polyposis syndrome (SRS), 173 Super cial in erior epigastric artery (SIEA) ap,
Rituximab, 151, 323 Serum-ascites albumin gradient (SAAG), 141 204, 205
Roadside bomb, 245248 Serum CEA, 184 Superior mesenteric artery (SMA) occlusion,
bleeding deaths, 246 Sestamibi scan, 212, 213 163, 164
hemorrhage and therapy, 245, 246 Setons, 187 Superior mesenteric vein (SMV), 325
injury, initial care o , 245, 246 cutting, 187 Suppurative thyroiditis, 229. See also Head and
injury pattern o , 245 non-cutting, 187 neck pediatric tumors
treatment Shoulder dislocations, 380, 381382 Supracondylar humerus ractures, 384385
DCR in, 246, 247 anterior, 381 Supralevator abscesses, 186, 188
DCS in, 245, 247 neurovascular injury with, 380, 382 Surgical nutrition, 301303
Robotic-assisted laparoscopic prostatectomy posterior, 381 Surgical site in ections, a er ventral hernia
(RALP), 25, 26 radiographs in, 381382 repair, 61, 63
Robotic-assisted laparoscopic surgery, Shoulder trauma, 380382 Surrogates, 38
2527 Sigmoidoscopy, in cancer diagnosis, 184 Surveillance, Epidemiology, and End Results
advantages o , 25, 26, 2829, 30 Silver nitrate, 299 (SEER), 193
complications o , management o , 29 Silver sul adiazine, 299 Syndrome o inappropriate antidiuretic
disadvantages o , 26 Single-incision laparoscopic (SIL) hormone (SIADH), 56, 275, 279, 280
EndoWrist technology, 26 cholecystectomy, 21, 2223 Systemic venous oxygen saturation (SvO2), 305
robotic-assisted vs. laparoscopic surgery or Single incision laparoscopic surgery, 174
colorectal cancer, 25, 26 Sister Mary Joseph node, 85 T
and traditional laparoscopic surgery, 29 Sistrunk procedure, 228 acrolimus, 323
training program or, 26, 27 Skin cancer, 4143 ailored neurectomy, 75
Rocuronium, 5, 67 and post-transplant immunosuppression, amoxi en ( MX), 192, 193, 195
Rombergs test, 269 323 adverse e ects o , 197
Romiplostim, 152 Skull racture, 271 in breast cancer treatment, 196, 197
Roux-en-Y gastric bypass (RYGB), 77, 78 compound depressed, 270 axanes, 200
abdominal pain a er, 103, 105 open, 270 emporary access, cu ed, tunneled dialysis
iron de ciency anemia a er, 115116 Sleep apnea, 57 catheter or, 334, 335
and marginal ulcers, 101 (See also Marginal Sleeve gastrectomy leaks, 106108 ension pneumocephalus, 266, 266 , 270
ulcers (MU)) chronic stulae and leaks, management o , with Mount Fuji sign, 266 , 270
Rule o nine, 298 107, 108 eratomas, 230
endoluminal therapies or, 108 eriparatide shots, 213
S median time or leak healing, 108 esticular cancer, 402403
SAGES FUSE course, 30 staple line leak rates, reducing o , 106, 107 bleomycin-induced pulmonary brosis, 402,
Salivary gland tumors, 221223 surgical treatment o , 106, 107108 403
FNA o , 222 sustained tachycardia and, 106, 107 metastasis to para-aortic lymph nodes, 402,
incisional biopsy o , 222 Small bowel obstruction, 153155 403
malignant, 223 oley catheter and acute abdominal series, radical orchiectomy or, 402, 403
signs and symptoms, 222 153, 154 scrotal ultrasound or, 402, 403
In d e x 419

T iamine de ciency, a er bariatric surgery, 115 primary non- unction o , 325 V


T oracic gunshot wounds, 289 replantation, 326 Vacuum-assisted closure (VAC) techniques, or
T oracic injuries, 252, 254 ransureteroureterotstomy, 406 synthetic mesh in ections, 62, 63
T oracic outlet syndrome ( OS), 347349 ransverse rectus abdominis myocutaneous Varicose veins, 344, 345346
anatomic structure in thoracic outlet, 347, ( RAM) ap, 204, 205 Vascular access, 334336
348 rapdoor racture, 285 Vascular injuries, during trocar placement,
arterial, 348 rastuzumab, 193 1920
diagnosis/workup o , 347, 348349 rauma and critical care, 245248 diagnosis o , 19, 20
neurogenic, 348 rauma and subarachnoid hemorrhage, 268, and hemodynamic instability, 20
venous, 348 274 Vasopressor therapy, 293
T oracic trauma, 288290 rauma score, 305 Vecuronium, 7
T oracotomy, emergency, 250 raumatic brain injury, intracranial pressure Venous insu ciency, chronic, 341, 343345
T romboembolism, 218 monitoring in, 259 Venous stasis disease, 343346
T rombophlebitis, 342 rendelenberg test, 343, 344 Venous stasis ulcers, 344345
T rombopoietin ( PO) receptor agonists, 151, riangle o pain, 74 Ventral hernia
152 riple neurectomy, 73, 75 biologic mesh repair, 63
T rombotic thrombocytopenic purpura ( P), roisiers sign, 85 complications a er repair o , 61, 63
151 ube thoracostomy, 250, 252, 254 grading system or, 61, 62
T yroglossal duct cyst, 227, 228 ubo-ovarian abscess ( OA), 363365 inlay mesh repairs, 62
T yroid disease, 215216 antibiotic therapy or, 364 interpositinal repair, and recurrence, 61, 62
T yroid lobectomy, 214, 216 diagnosis o , 363 mesh in ections, 63
le , 215 surgical management, 364365 onlay mesh repair, 62
T yroid nodules, 216 teatment or, 363364 primary, 62
biopsy o , 216 umescent solution, 345 retrorectus repair, 62
risk actors, 216 umor gene sequencing, 185 secondary, 61, 62
T yroid storm, 10 umor, node, metastasis ( NM) model, 193 synthetic mesh in ections, vacuum-assisted
ibial plateau ractures, 387, 389 closure techniques or, 62, 63
knee injury with, 387, 389 U underlay type repair, 62
leg compartment syndrome with, 387, Ultrasound (US) Vertebral artery injuries, 257
390 abdominal aortic aneurysm, 331, 332 Vertical sheer (VS) racture, 262
management o , 389 biliary colic, 127, 128 Video-assisted thoracoscopic surgery (VA S),
idal volume ( V), 297 breast cancer, 192 250, 251, 288, 314
-lymphocytes, 322 IHPS diagnosis, 233, 233 or motor vehicle collision patient, 288
orticollis ovarian masses, 353, 354 Virchows node, 85
botulinum toxin in, 230 sigmoid diverticulitis, 180 Vitamin de ciency, a er bariatric surgery, 115
treatment or, 228, 230 testicular cancer, 402, 403 Vitamin K antagonists, 306
otal thyroidectomy, 209, 212, 215, 216 Umbilical hernia, cirrhosis and, 6465, 140, von Hippel-Lindau disease (VHL), 399, 400
ourniquets, 246 141 Von-Willebrand actor, 211
ranexamic acid ( XA), 246, 305 Uncal herniation, 271, 273
ransabdominal drainage, or cryptoglandular Un ractionated heparin (UFH), 304, 306 W
abscess, 187. See also Perirectal abscess Upper gastrointestinal contrast study (UGI), 232 Warthins tumor, 222
ranscranial doppler ( CD), 274 Upper tract urothelial carcinoma (U -UC), and Wernicke encephalopathy (WE), 115
ranscranial magnetic stimulation ( MS), 271 renal cell carcinoma, 399, 400 management o , 115
rans usion associated circulatory overload Ureteral injury, during colectomy, 405406 thiamine de ciency and, 115
( ACO), 305 Ureteral stone White-eyed blow out racture, 285
rans usion related acute lung injury ( RALI), hyperparathyroidism and, 395, 396 Withdrawing and withholding treatment, 37,
304, 305 passage o , estimation o , 395, 396 38
rans-jugular intra-hepatic shunt ( IPS) test or, 395 Wound healing, in chronic wounds, 45
procedures, 142 Ureteroneocystostomy, 405, 406 Wounds, di cult, management o . See Negative
rans mediastinal gunshot wounds, 289 Ureteroureterostomy, 405, 406 pressure wound therapy (NPW )
ransplant surgery Urethral injury, 315 Wuchereria banchro i, 340, 341
o kidney, 321323 Urethral stricture, in women, 314
hyperacute rejection, 322 Ursodiol, 22 Z
o liver, 324326 or gallstones, 128 Zollinger-Ellison syndrome, 219

Вам также может понравиться